Sunteți pe pagina 1din 631

SPEED's Electric Machines

with problems and solutions

© TJE Miller, 20022014


ii
1. Sizing, gearing, cooling, materials and design

1.1 Motion control systems . . . . . . . . . . . . . . . . . . . . . . . . . . . . . . . . . . . . . . . . . . . . . . . . . . . . 1.1


1.2 Why adjustable speed? . . . . . . . . . . . . . . . . . . . . . . . . . . . . . . . . . . . . . . . . . . . . . . . . . . . . 1.1
1.3 Large versus small drives . . . . . . . . . . . . . . . . . . . . . . . . . . . . . . . . . . . . . . . . . . . . . . . . . 1.3
1.4 Structure of drive systems . . . . . . . . . . . . . . . . . . . . . . . . . . . . . . . . . . . . . . . . . . . . . . . . . 1.3
1.5 Drive system requirements . . . . . . . . . . . . . . . . . . . . . . . . . . . . . . . . . . . . . . . . . . . . . . . . 1.4
1.6 New technology . . . . . . . . . . . . . . . . . . . . . . . . . . . . . . . . . . . . . . . . . . . . . . . . . . . . . . . . . . 1.4
1.7 Which motor ? . . . . . . . . . . . . . . . . . . . . . . . . . . . . . . . . . . . . . . . . . . . . . . . . . . . . . . . . . . . . 1.7
1.8 Sizing . . . . . . . . . . . . . . . . . . . . . . . . . . . . . . . . . . . . . . . . . . . . . . . . . . . . . . . . . . . . . . . . . . 1.13
1.9 Gearing, figures of merit, and inertia . . . . . . . . . . . . . . . . . . . . . . . . . . . . . . . . . . . . . . 1.17
1.10 Saliency . . . . . . . . . . . . . . . . . . . . . . . . . . . . . . . . . . . . . . . . . . . . . . . . . . . . . . . . . . . . . . . . 1.24
1.11 Cooling . . . . . . . . . . . . . . . . . . . . . . . . . . . . . . . . . . . . . . . . . . . . . . . . . . . . . . . . . . . . . . . . . 1.25
1.12 Intermittent operation . . . . . . . . . . . . . . . . . . . . . . . . . . . . . . . . . . . . . . . . . . . . . . . . . . . 1.36
1.13 Permanent magnet materials and circuits . . . . . . . . . . . . . . . . . . . . . . . . . . . . . . . . . . 1.39
1.14 Properties of electrical steels . . . . . . . . . . . . . . . . . . . . . . . . . . . . . . . . . . . . . . . . . . . . . . 1.48
1.15 Effective BH curve for lamination stack . . . . . . . . . . . . . . . . . . . . . . . . . . . . . . . . . . . . 1.53
1.16 Series and parallel inductances . . . . . . . . . . . . . . . . . . . . . . . . . . . . . . . . . . . . . . . . . . . 1.55
1.17 Machine and drive design . . . . . . . . . . . . . . . . . . . . . . . . . . . . . . . . . . . . . . . . . . . . . . . . 1.67
1.18 Computer-aided design . . . . . . . . . . . . . . . . . . . . . . . . . . . . . . . . . . . . . . . . . . . . . . . . . . . 1.68
1.19 The number of phases in an AC system . . . . . . . . . . . . . . . . . . . . . . . . . . . . . . . . . . . . . 1.70
1.20 Half-Turns . . . . . . . . . . . . . . . . . . . . . . . . . . . . . . . . . . . . . . . . . . . . . . . . . . . . . . . . . . . . . . 1.74
1.21 Sign conventions . . . . . . . . . . . . . . . . . . . . . . . . . . . . . . . . . . . . . . . . . . . . . . . . . . . . . . . . 1.76
SEM 1 — ii
1. SIZING, GEARING, COOLING, MATERIALS and DESIGN

1.1 MOTION CONTROL SYSTEMS


Technology is so saturated with developments in microelectronics that it is easy to forget the vital
interface between electrical and mechanical engineering. This interface is found wherever mechanical
motion is controlled by electronics, and pervades a vast range of products. A little consideration reveals
a large and important area of technology, in which motor drives are fundamental. In Japan the term
'mechatronics' is applied to this technology, usually with the connotation of small drives. In the west
the term 'motion control system' is often used for small controlled drives such as position or velocity
servos. In the larger industrial range the term 'drive' usually suffices.
Many engineers have the impression that the technology of motors and drives is mature, even static.
But there is more development activity in drives today than at any time in the past, and it is not
confined to the control electronics. Two important reasons for the development activity and the
increasing technical variety are:
(1) Increasing use of computers and electronics for motion control. Automation demands drives
with a wide variety of physical and control characteristics.
(2) New technology in power semiconductors, sensors, integrated circuits, and microcontrollers,
facilitating the development of nonclassical motors such as brushless DC motors, steppers and
switched reluctance motors in a wide variety of designs.

1.2 WHY ADJUSTABLE SPEED?


Three common reasons for preferring an adjustable-speed drive over a fixed-speed motor are :
(a) energy saving;
(b) velocity or position control; and
(c) amelioration of transients.

(a) Energy saving. In developed economies about one-third of all primary energy is converted into
electricity, and about two-thirds of that is re-converted in electric motors and drives, mostly
integral-kW induction motors running essentially at fixed speed. If a constant-speed motor is used to
drive a flow process (such as a fan or pump), the only ways to control the flow rate are by throttling or
by recirculation, Fig. 1. Since the motor runs at full speed regardless of the flow requirement, there can
be excessive energy losses in the recirculation valve. Similar considerations apply to the control of
airflow by adjustable baffles in air-moving plant.

Fig. 1.1 Flow process controlled by recirculation can produce energy losses in the flow control valve.
Page 1.2 SPEED’s Electric Machines

In such applications it is often possible to reduce average energy costs by 50 per cent or more by using
adjustable-speed drives, which eliminate the throttling or recirculation loss; see Fig. 1.2. The additional
losses in the adjustable-speed drive are generally much less than the throttling and recirculation losses,
since the drive efficiency is usually of the order of 90 per cent or more. The adjustable-speed drive may
be more expensive, but its capital cost can be offset against energy savings and the reduction of
maintenance requirements on mechanical components.
Recirculation in a flow-control process is analogous to the control of a DC motor by means of an
adjustable series resistance. The technique is inherently wasteful, and although it is cheap to
implement, it is increasingly hard to justify in the face of concerns about energy efficiency and
pollution, even at low power levels.

Fig. 1.2 Flow control efficiency is improved by adjustable-speed drive

(b) Velocity or position control. Obvious examples of speed control are the electric train, portable
hand tools, and domestic washing-machine drives. In buildings, elevators or lifts are interesting
examples in which not only position and velocity are controlled, but also acceleration and its derivative
(jerk). Countless processes in manufacturing industry require position and velocity control of varying
degrees of precision. Particularly with the trend towards automation, the technical and commercial
growth in drives below about 20 kW is very vigorous. Many system-level products incorporate an
adjustable-speed drive as a component. A robot, for example, may contain between 3 and 6 independent
drives, one for each axis of movement. Other familiar examples are found in office machinery:
positioning mechanisms for paper, printheads, magnetic tape, and read/write heads in disk drives.

(c) Amelioration of transients. The electrical and mechanical stresses caused by direct-on-line motor
starts can be eliminated by adjustable-speed drives with controlled acceleration. A full adjustable-speed
drive is used in this situation only with very large motors or where the start-stop cycles are so frequent
that the motor is effectively operating as a variable speed drive. Most soft-starting applications are less
onerous than this, and usually it is sufficient (with AC motors) to employ series SCR's (or triacs with
smaller motors) which ‘throttle' the starting current to a controlled value, and are bypassed by a
mechanical switch when the motor reaches full speed.1 The soft-starter is less expensive than a full
adjustable-speed drive, which helps to make it economical for short-time duty during starting.

1
Series control of induction motors is inefficient; produces excessive line harmonics; and is not very stable. These factors can
usually be tolerated for a few seconds during starting, but they render the soft-starter unsuitable for continuous speed control.
1. Sizing, gearing, cooling, materials and design Page 1.3

1.3 LARGE VERSUS SMALL DRIVES


There are marked design differences between large and small drives. Large motors are almost always
chosen from one of the classical types: DC commutator (with wound field); AC induction; and
synchronous. The main reasons are the need for high efficiency and efficient utilization of materials;
and the need for smooth, ripple-free torque. In small drives there is greater variety because of the need
for a wider range of control characteristics. Efficiency and materials utilization are still important, but
so are control characteristics such as torque/inertia ratio, dynamic braking, speed range, acoustic
noise and torque ripple.
There are also several breakpoints in the technology of power semiconductors. At the highest power
levels (up to 20MW) SCRs (thyristors) and GTOs (gate turn-off thyristors) are the only devices with
sufficient voltage and current capability, but IGBTs (insulated-gate bipolar transistors) also now have
voltage ratings measured in kV and current ratings of hundreds of amps. Naturally-commutated or
load-commutated converters are preferred, because of the saving in commutation components and for
reliability and efficiency reasons. In the medium power range (up to a few hundred kW) forced
commutation and PWM are normal, and IGBT’s are very widely used. At low powers (below a few kW)
the power MOSFET is attractive because it is easy to switch at high chopping frequencies.

1.4 STRUCTURE OF DRIVE SYSTEMS

Fig. 1.3 Drive system structure

The general structure of a drive system is shown in Fig. 3. It comprises the load, the motor, the
electronic drive, and the control. The range of modern motion-control applications is virtually
unlimited. Any random list illustrates the variety—aerospace actuators; washing machines; computer
disk and tape drives; printer plattens and printheads; inertial guidance systems; adjustable-speed
pumps, blowers, fans and compressors; locomotive and subway traction; automatic machine tools and
machining centers; servo drives and spindle drives; robots; automotive auxiliaries; refrigeration and
air-conditioning; and many others.
Loads have widely differing requirements. The commonest requirement is for speed control, with
varying degrees of precision and accuracy. Position control is of increasing importance, particularly
in automated plants and processes, and in office machinery and computer peripherals. In some cases
it is the steady-state operation that is most important, for example in air-conditioning and pump drives.
Efficiency is important in motors that run continuously, but less so in intermittent-duty motors.
In robots and servomotors, dynamic performance is important because of the need to minimize the time
taken to perform complex movements. In these cases the torque/inertia ratio of the motor is
important, as is the ability to change torque quickly. Automotive applications require low cost and low
noise. Torque ripple is a critical factor in servo motors and applications such as automotive power
steering, where less than 1% ripple is typically required.
Page 1.4 SPEED’s Electric Machines

1. Compliance with national, EC, USA and industry standards


2. Maximum continuous power or torque requirements
3. Forward/reverse operation
4. Motoring/braking operation
5. Dynamic or regenerative braking
6. Overload rating and duration
7. Supply voltage (AC or DC) and frequency
8. Type of control: speed, position, etc.
9. Precision required in controlled speed or position
10. Programmability: speed and/or position profiles, start/stop ramps etc.
11. Interface with plant control and communications
12. Dynamic requirements: torque/inertia ratio, acceleration/deceleration
13. Gearbox or direct drive; gear ratio
14. Reliability and redundancy of components
15. Protection arrangements, both mechanical and electrical
16. Maximum level of acoustic noise; noise spectrum
17. Compliance with EMC regulations
18. Limitation of harmonics in the supply system
19. Maintenance; spare parts; provision for expansion or reconfiguration
20. Environment: indoor/outdoor installation; enclosure; temperature; humidity; pollution; wind
and seismic factors; type of coolant.
TABLE 1.1
DRIVE SYSTEM REQUIREMENTS CHECKLIST

1.5 DRIVE SYSTEM REQUIREMENTS


Table 1.1 provides a checklist of application requirements. The speed/torque capability diagram is
especially important, Fig. 4. Typically at low speed the drive operates under current limit, and since
torque is generally proportional to current (or nearly so), the torque is controlled in this mode of
operation, such that any value can be obtained up to the value corresponding to maximum current. At
high speed, the back-EMF increases to a level at which the drive cannot maintain maximum current; or
even if it can maintain maximum current, it may not have sufficient voltage to maintain the correct
phase angle or waveform of the current. Base speed is the maximum speed at which rated torque can
be developed. Above this speed, drives are often evaluated according to the speed range over which
they can maintain constant power, the torque decreasing as the speed increases. Some drives (such as
triac-controlled AC universal motors) have almost no constant-torque region and their characteristics
are said to be “highly inverse”. On the other hand, brushless PM motors tend to have a predominantly
constant-torque characteristic with limited speed range above base speed.

1.6 NEW TECHNOLOGY


Several new technologies are contributing to the development of motion control systems.
Digital electronics. It would be hard to overstate the importance of microelectronics in motion
control. At the 'heavy' end of the spectrum are the multiple drives found in steel rolling mills, paper
mills, and other heavy process plants, where it is normal to coordinate the motion of all the shafts by
means of a computer or a network of computers, some of which may be quite large. At the light end of
the power range are the drives found in office machinery, small computers, and portable goods such
as cameras and compact-disk players, where custom integrated circuits and gate arrays are common.
Between these extremes there are many microprocessor-controlled systems of all levels of complexity.
1. Sizing, gearing, cooling, materials and design Page 1.5

Fig. 1.4 Speed/torque characteristic

The first functions implemented with microprocessors were low-speed functions such as monitoring
and diagnostics, but digital control has penetrated from outer position loops through the intermediate
velocity loop and is now routinely used even in high-bandwidth current regulators. The development
of field-oriented control for AC induction and synchronous motor drives would not have been practical
without the microprocessor. Field-oriented control is based on reference-frame transformation which
may require rapid computation of trigonometric functions of rotor position. It permits the outer control
loops of AC and DC drives to be the same, both in hardware and software, and improves the dynamic
performance of the AC drive. Its development since the mid 1970's was a key factor enabling the AC
induction motor to compete in precision speed control with the DC motor, which had been preferred
for speed control for at least 50 years before then.
Such is the sophistication and speed of modern microelectronics that the PWM schemes employed to
regulate the voltage and current can be optimized with respect to many attributes, such as efficiency,
acoustic noise, dynamic response, and harmonic content. There is increasing use of field-programmable
gate arrays for the high-speed functions, often combined with microcontrollers. Digital signal
processors (DSP) are also used in advanced drives, and many of these operate with “sensorless” control,
i.e., without shaft position sensing.
In motion control systems the ultimate objective is true instantaneous control of the torque, preferably
with a minimum of reliance on shaft position sensors and detailed knowledge of motor parameters. In
pursuit of this objective the processing power of modern microcomputers and DSPs has been exploited
together with new forms of control based on neural networks or fuzzy logic. At the other end of the
drive, communication with computers and other controllers is another area of continuous
development.
Power semiconductor devices. The IGBT is the most popular device in integral-kW drives, and the
power MOSFET in low-power drives, especially at low voltage. GTO thyristors are still used in large
drives especially above 1 MW.
New magnetic materials. The permanent-magnet industry has continued to improve the
characteristics of the main families of magnet used in electric machines: neodymium-iron boron, which
was pioneered by Sumitomo and General Motors; rare-earth/cobalt; and ferrite. At room temperature
NdFeB has the highest energy product of all the common magnet materials. In its early days the
performance was rather sensitive to temperature, but it is now widely used even in automotive and
industrial servo applications where high operating temperatures up to and exceeding 100EC are
common, even in the presence of strong demagnetizing fields.
CAD and numerical analysis in design. Motor design has been computerized since the early days
of computers, initially with the coding of well established design procedures. The last 20 years has seen
a steady development of commercially available finite-element software capable of analysing fields in
machines in two and three dimensions. Advanced programs can calculate eddy-currents and transients.
Page 1.6 SPEED’s Electric Machines

However, the coupling of finite-element solvers to circuit-analysis and simulation algorithms is still
in the research laboratory. In spite of the technical advances it remains the case that finite-element
analysis requires the application of highly skilled personnel, and although machine designers use it
fairly widely, they either follow well-established procedures or rely on specialists to practise
computations in support of more conventional design calculations.
The finite-element method is still far from being a complete design tool and is too slow for many of the
routine processes. It is fundamentally an analysis tool rather than a design tool: and it suffers from
certain limitations when applied to motor design. It requires detailed input data and the results need
skilled interpretation. It is accurate only in idealized situations where parasitic effects have been
removed. It is too slow to be cost-effective as part of a synthesizing CAD package, and is likely to remain
so for some time. It is most useful in helping to understand a theoretical problem that is too difficult
for conventional analysis, and in this role it has undoubtedly helped to refine many existing motor
designs and improve some new ones.
At the same time the development of special design software for electric machines has produced a
number of products which are much faster and are specificall intended for motor and generator design.
Such programs are widely used in the industry for initial design, for “what-if” analysis, and (with
suitable calibration against test data) for recording the characteristics of entire lines of motor products.
One example is the SPEED software, which is used in places to illustrate this book.2
The primary problems in motor and generator design are not simply electromagnetic, but require an
integrated approach to materials utilization and design-for-manufacture. The philosophy needs to be
a synthesizing one rather than an analytical one. This multidisciplinary problem includes heat
transfer and mechanical design as well as electromagnetics. The situation is more complicated in
adjustable-speed drives where the supply waveforms are 'switchmode' chopped waveforms rather than
pure sinewaves or DC. In these cases time-stepping simulation may be necessary to determine the
expected performance of a given design over a wide range of operating conditions.
Because of the fact that finite-element analysis usually requires foreknowledge of the current
waveform, it could be argued that preoccupation with this class of software tool could hinder the
development of drive systems which escape from the classical DC and sinusoidal waveforms in order
to explore the possibilities of a wider class of drive current waveforms, coupled with new concepts in
motor geometry. The key lies in using the finite-element tool in the correct way to determine
parameters that can be used in time-stepping simulations that will be executed by other software.
It is perhaps surprising that design software for electric machines is rarely capable of synthesis in the
true sense. It is much more common to find “optimization” techniques which rely on the automatic
generation of large numbers of “feasible” designs, and then rank them according to a more or less
complex criterion which includes constraints on particular dimensions and other parameters. The
imaginative element in the design process remains in the human mind, and the computer appears to
be far from taking it over.
For simulation of complete systems, such as an automotive power steering system or an aircraft flight
control surface actuation system, there are several software packages, such as Simulink™, Saber™,
Simplorer™, Easy5™ and many others. Suitably modified and extended, some of these packages permit
the simulation of detailed motor models and their drives and controls. They may be used for the
development of control algorithms that are subsequently programmed in a microprocessor or gate
array.

Other contributing technologies. Plastics and composite materials find many applications in motors.
Fans, slot liners and wedges, end-bells and covers, and winding supports are the commonest, but
moulded slot insulation and encapsulation of rotors are also widely used. In brushless motors designed
for high peripheral speeds, the magnets are often restrained against centrifugal force by banding or
tubes made of Kevlar™, glass-reinforced plastics, or carbon fiber.

2
Others include design programs from Trimerics in Stuttgart, Germany and Yeadon Engineering Services, Wisconsin.
1. Sizing, gearing, cooling, materials and design Page 1.7

Motor drives generally require transducers for control and protection, and there has been progress in
current-sensor and shaft position sensor technology. In particular the linearity and
temperature-independence of Hall-effect current sensors has improved greatly, and it is common to
mount these devices in the same package, or on the same printed-circuit card, as the driver stage of the
power electronics in small drives. For larger drives flux-nulling current sensors can be used with
bandwidths of up to several kHz and isolation at least as good as that of a C.T.
In brushless drives the commutation signals are often derived from Hall sensors, activated either by
the rotor magnet or by a separate magnet ring. Alternatively, optical interrupters may be used with a
shaft-mounted slotted disk. At high speeds the commutation sensor can be used to generate a speed
signal via a frequency-to-voltage conversion. For motion control systems and servo-quality drives
separate velocity and position transducers usually have to be used. For such systems the resolver is
attractive because of its ruggedness, resolution, and its ability to provide accurate absolute position and
velocity signals from one sensor.

1.7 WHICH MOTOR ?


The proliferation of new ideas, materials and components obviously generates many opportunities but
also complicates the question, what is the best drive for a particular job? We can perhaps address this
by attempting to trace the evolution of the different motor types in such a way as to bring out their most
important advantages and disadvantages. It is the motor that determines the basic characteristics of
the drive system, and it alsodetermines the requirements on the power semiconductors, the converter
circuit, and the control.
Evolution of motors. The evolution of brushless motors is shown in Fig. 5. Row 1 contains the three
'classical' motors : DC commutator (with wound field); AC synchronous; and AC induction. The term
'classical' emphasizes the fact that these motors satisfy three important criteria:
(1) they all produce essentially constant instantaneous torque (i.e., very little torque ripple);
(2) they operate from pure DC, or AC sinewave supplies, from which
(3) they can start and run without electronic controllers.
The classical motors of row 1 are readily coupled to electronic controllers to provide adjustable speed;
indeed it is with them that most of the technical and commercial development of power electronic
control has taken place. Together with the PM DC commutator motor in row 2 and the series-wound AC
commutator motor or 'universal' motor, the row 1 motors account for the lion's share of all motor
markets, both fixed-speed and adjustable-speed, even though they represent only a minority of the many
different principles of electromechanical energy conversion on which motor designs may be based. By
contrast, the nonclassical motors are essentially confined to specialist markets and until recently, few
of them except the brushless DC motor have been manufactured in large numbers. Table 1.2 is a
classification of some common types of motor according to these criteria.
The motors in row 2 are derived from those in row 1 by replacing field windings with permanent
magnets. The synchronous motor immediately becomes brushless, but the DC motor must go through
an additional transformation, from row 2 to row 3 with the inversion of the stator and rotor, before the
brushless version is achieved. The induction motor in row 1 is, of course, already brushless in its 'cage'
version, but not in its wound-rotor or slip-ring version. The brushless motors are all those with three
terminals, together with the switched reluctance motor, which cannot be derived from any of the other
motors. Its awkward placement in Fig. 5 reflects the fact that it has various properties in common with
all the brushless motors.
The DC commutator motor. The traditional DC commutator motor with electronically adjustable
voltage has always been prominent in motion control. It is easy to control, stable, and requires
relatively few semiconductor devices in the drive. For many years the wound-field DC motor held its
own against the challenge of AC drives—arguably for at least fifty years from the mid-1930's until the
mid-1980's—but AC field oriented control, manufacturing cost structures, the development of the IGBT,
and huge R&D investments finally forced it into a declining role.
Page 1.8 SPEED’s Electric Machines

DC wound field (brush) AC synchronous AC induction


AC/DC universal (brush) wound-field

DC PM (brush) Brushless PM AC
[Brushless PM Exterior Rotor]

Switched
reluctance

Brushless PM Synchronous
Squarewave or sinewave reluctance

Fig. 1.5 Evolution of brushless motors from classical AC and DC motors


1. Sizing, gearing, cooling, materials and design Page 1.9

The main objections to the commutator motor are brush and commutator wear, and the fact that the
losses arise mostly on the rotor, making cooling more difficult than in AC motors where the losses arise
mostly on the stator. It is not that brushgear is unreliable—on the contrary, it is reliable, well-proven,
and 'forgiving', as is proven by the widespread use of DC motors in railway systems throughout the
world, and in automotive auxiliaries where the life of the brushes is not a serious limitation.
The PM DC commutator motor. In small DC commutator motors, replacing the field winding and
pole structure with permanent magnets usually permits a considerable reduction in stator diameter,
because of the efficient use of radial space by the magnet and the elimination of field losses. Armature
reaction is reduced and commutation is improved, owing to the low permeability of the magnet. The
loss of field control is not as important as it would be in a larger drive, because it can be overcome by
the controller. In small drives the need for field weakening is less common anyway. The PM DC motor
is usually fed from an adjustable voltage supply, either linear or pulse-width modulated.
In automotive applications the PM DC motor is well entrenched because of its low cost and because of
the low-voltage DC supply. Here it is usually operated at fixed speed or with series-resistance control.
For safety-critical and demanding applications such as electric power steering and braking, brushless
motor drives are more suitable. The development of higher-voltage automotive power supply systems
(above 40V) will help to make brushless motors more acceptable by reducing the current levels and
therefore the size and cost of MOSFETs required in the drive.
AC induction motor drives. AC induction or synchronous motors are often preferred because of the
limitations of commutation and rotor speed in DC motors. Slip is essential for torque production in the
induction motor, and it is impossible, even in theory, to achieve zero rotor losses. This is one of the
limitations of the induction motor, since rotor losses are more difficult to remove than stator losses, and
it is one main reason to use permanent-magnet and/or reluctance-type synchronous motors.

Fig. 1.6 Integrated motor/inverter and hand-held controller


(courtesy of Grundfos A/S, Denmark)

The efficiency and power factor of induction motors falls off in small sizes because of the natural laws
of scaling, particularly at part load. As a motor of given geometry is scaled down, if all dimensions are
scaled at the same rate the MMF required to produce a given flux-density decreases in proportion to the
linear dimension. But the cross-section available for conductors decreases with the square of the linear
dimension, as does the area available for heat transfer. This continues down to the size at which the
mechanical airgap reaches a lower limit determined by manufacturing tolerances. Further
scaling-down results in a more-or-less constant MMF requirement while the areas continue to decrease
with the square of the linear dimension. There is thus an “excitation penalty” or “magnetization
penalty” which becomes rapidly more severe as the scale is reduced. It is for this reason that
permanent magnets are so necessary in small motors. By providing flux without copper losses, they
directly alleviate the excitation penalty.
Page 1.10 SPEED’s Electric Machines

The induction motor is “brushless” and can operate with simple controls without a shaft position
transducer. The simplest type of inverter is the six-step inverter. With no shaft position feedback, the
motor remains stable only as long as the load torque does not exceed the breakdown torque, and this
must be maintained at an adequate level by adjusting the voltage in proportion to the frequency as the
speed changes. At low speeds, oscillatory instabilities may appear. To overcome these limitations there
have been several improvements including slip control and, ultimately, full field-oriented or “vector”
control in which the phase of the stator currents is regulated to control the angle between stator MMF
and rotor flux. Field orientation usually requires a shaft position encoder and may include an in-built
control model whose parameters are specific to the motor, and which must be compensated for changes
caused by changing load and temperature. Such controls are complex and generally cannot be justified
in small drives, but excellent results have been achieved in larger sizes (above a few kW).
In the fractional and low integral-horsepower range the complexity of the AC drive is a drawback,
especially when dynamic performance, high efficiency, and a wide speed range are among the design
requirements. These requirements cannot be met adequately with series- or triac-controlled induction
motors, which are therefore restricted to applications where low cost is the only criterion. Together
these factors favour the use of brushless PM motor drives in the low power range.
The brushless DC PM motor. The smaller the motor, the more sense it makes to use permanent
magnets for excitation. There is no single 'breakpoint' below which PM brushless motors outperform
induction motors, but it is usually in the1!10 kW range. Above this size the induction motor improves
rapidly, while the cost of magnets works against the PM motor. Below it, the PM motor has better
efficiency, torque per ampere, and effective power factor. Moreover, the power winding is on the stator
where its heat can be removed more easily, while the rotor losses are extremely small. These factors
combine to keep the torque/inertia ratio high in small motors. The brushless DC motor is also easier
to control, especially in its ‘squarewave' configuration (SEM- 4). Although the inverter is similar to that
required for induction motors, usually with six transistors for a 3-phase system, the control algorithms
are simpler and readily implemented in 'smartpower' or special-purpose ICs.

Fig. 1.7 The Minas brushless PM motor produced by Matsushita with its stator fabricated from segments. Courtesy of
Matsushita Ltd., Japan

The brushless PM AC synchronous motor. In Row 2 of Fig. 5 the brushless synchronous machine
has permanent magnets instead of a field winding. Field control is again sacrificed for the elimination
of brushes, sliprings, and field copper losses. This motor is a classical salient-pole synchronous AC
motor with approximately sine-distributed windings, and it can therefore run from a sinewave supply
without electronic commutation. If a cage winding is included, it can self-start 'across-the-line'.
The magnets can be mounted on the rotor surface (SEM-5) or they can be internal to the rotor. The
interior construction simplifies the assembly and relieves the problem of retaining the magnets against
centrifugal force. It also permits the use of rectangular instead of arc-shaped magnets, and usually there
is an appreciable reluctance torque which leads to a wide speed range at constant power.
1. Sizing, gearing, cooling, materials and design Page 1.11

The PM synchronous motor operates as a synchronous reluctance motor if the magnets are left out or
demagnetized. This provides a measure of fault-tolerance in the event of partial or total
demagnetization through abnormal operating conditions. It may indeed be built as a magnet-free
reluctance motor, with or without a cage winding for starting 'across-the-line'. Although the power
factor and efficiency are not as good as in the PM motor, synchronous reluctance motors can be
designed with wide speed range and substantial short-term overload capacity..
In larger sizes the brushless synchronous machine is sometimes built with a brushless exciter on the
same shaft, feeding a rotating rectifier which passes DC to a field winding on the main rotor. This
motor has full field control. It is capable of a high specific torque and high speeds. As a generator, this
configuration is popular in high-speed aircraft generators (at 24,000 and 12,000 rpm, 400 Hz) and in a
wide variety of small industrial applications.
All the motors on the diagonal of Fig. 5 operate with inverters that share the same power circuit
topology (three 'totem-pole' phaselegs with the motor windings connected in star or delta to the
midpoints). This gives rise to the concept of a family of motor drives providing a choice of motors and
motor characteristics, but with a high degree of commonality in the control and power electronics and
all the associated transducers. The trend towards integrated phaselegs, or indeed complete three-phase
bridges, with in-built control and protection circuitry makes this concept more attractive. This family
of drives covers a wide range of requirements, the main types being the conventional brushless DC
(efficient in small sizes with good dynamics); the interior-magnet synchronous motor (wide speed
range); the synchronous reluctance motor (free from magnets and capable of high speeds or
high-temperature operation); and the induction motor. It should be noted that all these drives are
essentially “smooth-torque” systems with low torque ripple.
Stepper motors represent a major class of motors not included in Fig. 5. Steppers are always brushless
and usually operate without shaft position sensing. Although they have many properties in common
with synchronous and brushless DC motors, they cannot naturally be evolved from the motors in Fig.
5. By definition they are pulsed-torque machines incapable of achieving ripple-free torque by normal
means. Variable-reluctance (VR) and hybrid steppers can achieve an internal torque multiplication
through the use of multiple teeth per stator pole and through the ‘vernier' effect of having different
numbers of rotor and stator poles. Both these effects work by increasing the number of torque impulses
per revolution, and the price paid is an increase in commutation frequency and iron losses. Steppers
therefore have high torque-to-weight and high torque-to-inertia ratios, but are limited in top speed and
power-to-weight ratio. The fine tooth structure requires a small airgap, which adds to the
manufacturing cost. Beyond a certain number of teeth per pole the torque gain is “washed out” by scale
effects that diminish the inductance variation on which the torque depends. Because of the high
magnetic frequency and the effect of MMF drop in the iron, such motors require expensive lamination
steels to get the best out of them.
Switched reluctance motors are derived from the single-stack VR stepper, in which the current
pulses are phased relative to the rotor position to optimize operation in the 'slewing' (continuous
rotation) mode. This usually requires a shaft position transducer similar to that which is required for
the brushless DC motor, and indeed the resulting drive is like a brushless DC drive without magnets.
With this form of control the switched reluctance motor is not a stepper motor because it can produce
continuous torque at any rotor position and any speed. There is still an inherent torque ripple, however,
which can be compensated only by current waveform profiling and accurate phase control of the
current waveform relative to the shaft position. The switched reluctance motor suffers the same
'excitation penalty' as the induction motor and cannot equal the efficiency or power density of the PM
motor in small sizes.
When the classical motors are interfaced to switchmode converters (such as rectifiers, choppers, and
inverters) they continue to respond to the average voltage (in the case of DC motors) or the fundamental
voltage (in the case of AC motors). The harmonics associated with the switching operation of the
converter cause parasitic losses, torque ripple, and other undesirable effects in the motors, so that
de-rating may be necessary. The nonclassical motors are completely dependent on the switchmode
operation of power electronic converters. In steppers it is acceptable for the torque to be pulsed, but
most brushless drives are designed for smooth torque even though the power is switched.
Page 1.12 SPEED’s Electric Machines

Motor Drive or supply Typical application


DC commutator motors
(a) Wound-field Constant DC or variable DC from DC Integral-kW industrial drives; railway
generator, phase-controlled rectifier, traction
or chopper
(b) Permanent-magnet Automotive and aircraft auxiliaries;
small servo and variable-speed drives
DC homopolar motors with slip-rings or Ship-propulsion specials
segmented slip-rings
Brushless PM motors
(a) Squarewave Electronically commutated rectangular Disk drives, automotive auxiliaries,
current waveform from inverter small portable goods, servo motors,
spindles
(b) Sinewave Sinewave current waveform from Servo motors, spindles
inverter
Universal AC/DC series commutator Fixed AC; triac-controlled AC; also Domestic appliances (washing
motors variable or constant DC machines, food processors); power
tools
Induction motors
(a) Cage-type, 3-phase Constant voltage AC or inverter-fed Pumps, fans, compressors, industrial
machinery of all types
(b) Cage-type, 1-phase Constant-voltage AC Low-power pumps, fans, machinery
(c) Wound-rotor, 3-phase Constant voltage AC or inverter-fed High-power drives with difficult starting
conditions; sometimes used with slip-
recovery energy systems
Synchronous motors
(a) Wound rotor with DC winding and Constant voltage AC or inverter-fed Very large drives
slip-rings
(b) Wound-rotor with brushless exciter Constant voltage AC or inverter-fed Most often used as a generator
(motoring)
(c) Permanent magnet Same as Sinewave brushless PM motor
Reluctance motors
(a) Synchronous reluctance (line-start, Inverter-fed AC Multi-machine variable-speed drives
with rotor cage) with several motors fed from one
inverter
(b) Synchronous reluctance (cageless) Inverter-fed AC Servo/spindle motors
(c) Switched reluctance Electronically regulated and Washing machine drives, mining
commutated DC machinery, door openers, automotive
auxiliaries, aircraft actuators, high-
speed compressors
(d) Single-phase reluctance Switched DC or AC Small actuators, clocks
Stepper motors
(a) VR, single-stack Switched DC, usually current-limited Motion-contol applications dependent
on open-loop stepping for position
(b) VR, multiple-stack
control (indexing); office equipment,
(c) Permanent-magnet industrial machinery
(d) Hybrid
Hysteresis motors
(a) Cylindrical rotor Inverter-fed or constant AC voltage Simple synchronous motor with good
starting characteristics
(b) Can-type rotor Constant-voltage AC Motorized valves and actuators
TABLE 1.2
A SELECTION OF MOTORS WITH TYPICAL APPLICATIONS
1. Sizing, gearing, cooling, materials and design Page 1.13

1.8 SIZING
When a new electric machine is to be designed from scratch, the requirements usually include a set of
performance specifications and a set of constraints or limitations such as the maximum physical size,
the maximum temperature rise, and the supply voltage. This section explains how the basic size of a
machine can be determined, starting from the performance specifications and working within the limits
of material properties and temperature rise.
In many cases, new machine designs are evolved from existing ones, by modifying existing laminations
and components to minimize the cost of changes in tooling and components. Even so, the same
principles determine how much power and performance can be achieved from a machine of given size
and temperature rise.
The output equation. The classical output equation applies to (and unifies) all electrical machines
from the tiniest micromotors (a few μW) up to the largest AC motors used in process plants or ship
propulsion (up to 20MW). Intuitively it comes from the fundamental law of electromagnetic force which
is often loosely stated as “force = flux × current”, according to the left-hand rule. For engineering
purposes we need to derive a more precise statement of this law. Except in linear motors, we are more
interested in torque than force. It is convenient to work with flux-density and current-density, because
these parameters have values which do not change greatly from one machine to another. Further, the
flux and current densities are closely related to the power loss density which determines the cooling
requirements and temperature distribution throughout the machine.
Specifically, the output equation relates the torque per rotor volume (TRV) to the electric loading A and
the magnetic loading B. We will define A and B first before deriving a precise form of the output
equation. The definitions are written in a form suitable for AC synchronous and induction machines.
For other types of machine the definitions are similar, but with slight variations of multiplying
constants and interpretation.
The electric loading A is defined as the linear current density around the airgap circumference, that is,
the number of ampere-conductors per metre around the stator surface that faces the airgap.

Total ampere&conductors 2 m Nph I


A ' ' A/m (1.1)
Airgap circumference BD
where I is the RMS phase current, m is the number of phases, Nph is the number of turns in series per
phase, and D is the diameter of the airgap. The airgap is assumed to be small compared to the rotor
diameter, so that no distinction is made between the rotor diameter and the stator diameter. The RMS
2
current is used because it determines the I R heating, which is what limits the electric loading.
The magnetic loading B is defined as the average flux-density over the rotor surface. In AC motors the
flux-density is distributed sinusoidally so that the fundamental flux/pole is
B D L stk
M1 ' B × Wb (1.2)
2p
where p is the number of pole-pairs and Lstk is the stack length, i.e., the axial length of the active part
of the machine. In slotted stators and rotors, the peak
flux density in the teeth Bt(pk) must be limited to about
1.6T, otherwise the magnetizing current and/or the
iron losses may become excessive. The peak flux-
density Bg(pk) in the airgap is therefore Bg(pk) . JBt(pk),
where J is the ratio of tooth width to slot-pitch,
measured at a diameter where the tooth flux-density
is maximum; see Fig. 1.8. Typically J is of the order of
0.5. Thus B = 2Bg(pk)/B = 2JBt(pk)/B, so B is normally
limited to around 2 × 0.5 × 1.6/B . 0.5T.

Fig. 1.8 Definition of tooth pitch and J


Page 1.14 SPEED’s Electric Machines

The generated EMF per phase is given by the standard equation3

2B B2 kw1Nph B D Lstk f
E ' kw1Nph M1 f ' V. (1.3)
2 2 p
where f is the fundamental frequency, kw1 is the fundamental harmonic winding factor, and the product
kw1Nph is the effective number of turns in series per phase. The maximum available electromagnetic
power at the airgap is mEI. We consider this as being converted into mechanical power TT/p, where
T/p = 2B f is the speed in rad/sec. (Note also that T = 2B/60 × rpm). We can obtain the TRV as
2
T/(BD Lstk/4), and substituting from equations. 1.1, 1.2 and 1.3 we get
T B
TRV ' ' kw1 A B Nm/m 3 . (1.4)
Vr 2
This equation reflects the “flux-current product” in the form AB. The
multiplying factor is simply a constant multiplied by the winding
factor kw1, which incidentally casts kw1 in the role of a utilization
factor—the higher the winding factor, the greater is the utilization of
flux and current in producing torque. Since kw1 is usually about
0.85!0.95, TRV . 2AB.
The TRV is also related to the airgap shear stress F, which is the
tangential (torque-producing) force per unit of swept rotor surface Fig. 1.9 Airgap shear stress
area; see Figs. 1.9 and 1.10. For every unit of rotor surface area, the
electromagnetic torque is r F = D F/2, so the total torque is T = BDLstk
2
× D F/2 = 2F × (B/4)D Lstk, from which it follows that
T
TRV ' ' 2 F. (1.5)
Vr

The airgap shear stress F is measured in kN/m2. Typical values are


given in Table 1.3.4 The winding factor kw1 is generally between 0.8
and 0.95, so that TRV . 2 BA and F . BA. For example, if the electric
loading A = 20 A/mm and the magnetic loading B = 0.5 T, F . 0.5 × 20
× 103 = 10 kN/m2. For totally-enclosed motors the lower values of F
and TRV would apply with natural convection, while the higher values
would apply with forced-air cooling supplied by an external or shaft- Fig. 1.10 Airgap shear stress
mounted fan.5
2
Class of machine TRV kNm/m3 F lbf/in

Small totally-enclosed motors (Ferrite magnets) 7 ! 14 0.5 ! 1

Totally-enclosed motors (sintered Rare Earth or NdFeB magnets) 14 ! 42 1!3

Totally-enclosed motors (Bonded NdFeB magnets) 20 1.5

Integral-hp industrial motors 7 ! 30 0.5 ! 2

High-performance servomotors 15 ! 50 2!4

Aerospace machines 30 ! 75 2!5

Large liquid-cooled machines (e.g. turbine-generators) 100 ! 250 7 ! 18


TABLE 1.3
TYPICAL VALUES FOR TRV AND F (CONTINUOUS OPERATION)

3
See SEM-2.
4
In Imperial units, if D and Lstk are in inches, then T is in lbf-in. If F = 1 lbf/in2, TRV = 13.8 kNm/m3.
5 2
In some references the output coefficient K is defined as T/(D L), so K = TRV × B/4.
1. Sizing, gearing, cooling, materials and design Page 1.15

The coefficient Bkw1//2 in eqn. (1.11) is peculiar to AC machines where the ampere-conductor
distribution and the flux-density are sinusoidally distributed in space around the airgap; this can be
written
A(2) ' A(pk) sin 2 and B(2) ' B(pk) sin 2 . (1.6)

2
The product A(2)B(2) is the force per unit of rotor surface in N/m , and therefore the torque can be
2
obtained by integrating rA(2)B(2)d2 = rA(pk)B(pk) sin 2 over the entire rotor surface, where r = D/2, and
2
dividing by the rotor volume BD Lstk/4. This gives TRV = A(pk)B(pk). But the RMS value of A is A(pk)//2,
while the average value of B is BB(pk)/2, giving TRV = BAB//2. The winding factor appears because only
the fundamental space-harmonic component of the current distribution produces torque, in conjunction
with the fundamental component of flux-density, and the effectiveness of the winding in producing the
fundamental component is represented by kw1.
2
In DC machines the integral rA(2)B(2)d2 has no sin 2 term, and the result is TRV = 2AB, where B is the
average value of flux-density over the whole rotor periphery. The RMS value of A is equal to the peak
value, since the current is uniformly distributed around the rotor, and
Z Ia / a
A ' (1.7)
BD

where Z is the number of rotor conductors, a is the number of parallel paths, Ia is the armature current,
and D is the armature (rotor) diameter.
The electric loading A is limited by the slot fill factor, the depth of slot, and the cooling. It is also related
to the current density J in the conductors. Suppose the area of one slot is Aslot. Let d = slot-depth, t =
tooth width, w = slot width, and 8 = slot pitch = BD/Ns, where Ns is the number of slots. Also let J = t/8.
Then t + w = 8 and Aslot = wd = (1 ! t)8d. Now if the slot-fill factor Fslot is defined as the ratio of actual
copper cross-section area to the total area of each slot, we can write
A8 A
J ' ' . (1.8)
Fslot Aslot Fslot d ( 1 ! J )

For example, if the slot depth is d = 15mm, the slot-fill factor is Fslot = 0.4, the tooth-width/slot-pitch
ratio is J = 0.5, and the electric loading is A = 20 A/mm, the current density is
20
J ' ' 6.7 A/mm 2 . (1.9)
0.4 × 15 × ( 1 ! 0.5 )

Typical values of current density for use in AC or brushless machines for different applications are
given in Table 1.4. Note that in machines operated from electronic drives there are usually time-
harmonics in the current which increase the current-density without increasing the torque-producing
value of A, and it may be necessary to allow for this by multiplying J by a form factor kf. In AC
machines this will be the ratio of the true RMS current to the RMS value of its fundamental component.
In DC machines it will be the ratio of the RMS current to the average current.

Condition A/mm2 A/in2

Totally enclosed 1.5—5 1000—3000

Air-over, fan-cooled 5—10 3000—6000

Liquid cooled 10—30 6000—20000


TABLE 1.4
TYPICAL CURRENT DENSITIES (CONTINUOUS OPERATION)
Page 1.16 SPEED’s Electric Machines

These current-density values assume that the windings are varnished for good heat transfer. In air-
cooled machines, the fan is mounted on the rear of the motor outside the frame with a shroud which
focuses the air over the outside of the motor. Liquid cooled motors may have a passageway around the
outside of the stator with a cooling fluid circulating to remove the heat. The highest values are obtained
with hollow conductors with coolant flowing through them (“direct conductor cooling”).
It might seem strange to evaluate the magnetic loading as the average flux-density in the airgap rather
than the peak or RMS value, but the idea behind this is to indicate how well the entire cylinder of steel
is being utilized.6 Its value is limited by the available MMF of the excitation source, and by core losses
which increase rapidly at high flux-density.
It is interesting to see why it is the rotor volume and not its surface area that primarily determines the
torque capability or 'specific output'. As the diameter is increased, both the current and the flux
increase if the electric and magnetic loadings are kept the same. Hence the diameter (or radius) appears
squared in any expression for specific output. On the other hand, if the length is increased, only the flux
increases, not the current. Therefore the length appears linearly in the specific output. Thus the specific
output is proportional to D2Lstk, or rotor volume. In practice as the diameter is increased, the electric
loading can be increased also, because more intense fan-cooling or liquid cooling can be used without
reducing the efficiency. Consequently the specific output (TRV) increases faster than the rotor volume.
Although it is theoretically possible to write one general equation from which the torque of any electric
motor can be calculated, in practice a different torque equation is used for every different type of motor.
Only in certain cases is it possible to discern in this equation an explicit product of flux and current,
or even of quantities directly related to them. For example, in the DC commutator motor the
electromagnetic torque is given by
T ' k N Ia (1.10)

where N is the flux and k is a constant. Here the flux-current product is obvious. In rotating-field AC
machines the classical torque equations do not contain this product explicitly. However, the recent
development of 'field-oriented' or 'vector' controls has necessitated the transformation of the classical
equations into forms in which the flux and current may appear explicity in a scalar or vector product.
In eqn. (1.6) it is tacitly assumed that the flux and current are oriented at such angles as to maximize
the torque, but this is not automatically the case except in field-oriented drives. By contrast, in DC
machines the commutator automatically maintains the optimum relative angle of orientation between
the flux and the ampere-conductor distribution. In the case of doubly-salient motors such as the
switched reluctance motor and stepper motors, the torque cannot be expressed as the explicit product
of a flux and a current. However, the TRV can still be used for initial sizing provided that A and B can
be meaningfully defined (Miller, [1993]).
So far we have restricted attention to the torque per unit rotor volume, a natural consequence of the
fact that the torque appears at the rotor surface. For a very rough estimation of overall size including
the stator, we can use a typical value of 'split ratio' S (i.e., rotor/stator diameter ratio): thus
Rotor volume
Stator volume ' (1.11)
S2
A typical value of split ratio for an AC motor is in the range 0.55!0.65. For switched reluctance motors
rather smaller values are found. For DC commutator motors the value is usually somewhat higher.
The best way to acquire typical practical values of F or TRV is by experience. An engineer who is
familiar with a particular design of motor will have built and tested several, and the test data provides
values of TRV correlated with temperature rise, electric and magnetic loadings etc. The values quoted
in Table 1.3 relate to the continuous rating. Peak ratings may exceed these values by 2!3 times,
depending on the duration and other factors.

6
Switched reluctance machines have very high local flux-densities but a comparatively low magnetic loading, because the high
flux-density is limited to a small fraction of the stator periphery.
1. Sizing, gearing, cooling, materials and design Page 1.17

The TRV determines the volume of the rotor but not its shape. To estimate the rotor diameter and length
separately, a length/diameter ratio should be specified. A value around 1 is common; however, it is also
common to design motors of different ratings using the same laminations but with different stack
lengths. The length/diameter ratio may then vary over a range of 3:1 or more. Very large
length/diameter ratios are undesirable because of inadequate lateral stiffness, but may be used where
a high torque/inertia ratio is desired, or in special cases where the motor has to fit into a narrow space.
The foregoing discussion concerns the electromagnetic torque, that is, the raw torque produced by the
electromechanical energy conversion process at the airgap. The actual torque available at the shaft
coupling is less than this in motors, or greater in generators, by the amount of the mechanical losses
which include friction, windage, and certain electromagnetic losses appearing on the rotor. Allowance
should be made for these losses, which typically amount to less than 5% of the electromagnetic torque,
and in larger machines or high-efficiency machines, less than 1%.

1.9 GEARING, FIGURES OF MERIT, AND INERTIA


Compared to the torque density in mechanical and hydraulic devices, the torque density (TRV) in
electric motors is miserably low in comparison with what engineers would really like to achieve.7 It
always has been low, and it always will be low until someone discovers or invents a material that can
carry ten times as much flux as steel for the same magnetizing force; or a material that has a fraction
of the resistivity of copper. Such inventions would not by themselves be enough to increase the flea-
power of the electric motor by an order of magnitude, unless they were manufacturable in reasonable
quantity at reasonable cost—a test which has been repeatedly failed by laboratory prototypes and
“wonder motors” for many decades.
For this reason motors are often used with gearboxes to drive the load. A gearbox is the obvious way
to step up the torque. If the gear ratio is n, and Tm is the motor torque, the torque applied to the load is
nTm. The motor speed Tm is increased over the load speed TL by the same ratio. Thus

TL ' n Tm and Tm ' n TL . (1.12)

In most cases the increased motor speed falls in a standard speed range for 'high-speed' motors, which
may be typically anywhere from a few hundred rev/min to 30,000 rev/min or more.
If the gearbox efficiency is 100%, the output power of the motor is equal to the power applied to the load.
The choice of gear ratio depends on how the drive operates. If the speed is constant it is usually a simple
matter of matching the load torque TL to the rated continuous motor torque Tmc :
TL
n ' (1.13)
Tmc

If, however, the load has a 'dynamic' requirement which specifies a profile of speed or position as a
function of time, the choice of the gear ratio and the motor parameters is more complicated.
Simple acceleration of pure inertia load. Referring to Fig. 1.11, if the motor torque is its peak rated
torque Tmp, the acceleration of the load is given by
Tmp
" '
JL (1.14)
n Jm
2
n

where the term in brackets is the inertia of the motor combined with the load inertia, referred to the
motor shaft.

7
This assertion is not universally true. In large intensively-cooled machines such as power-station generators, the electric
machine clearly outperforms the steam turbine — it is typical for the generator to be dwarfed by the turbines.
Page 1.18 SPEED’s Electric Machines

Fig. 1.11 Gear ratio

If n is large the gearing makes the load inertia insignificant, but it reduces the load speed and
acceleration relative to those of the motor. If n is small the referred load inertia is large, and this limits
the acceleration. Between the extremes of large and small n, there is a value that gives maximum
acceleration for fixed values of Tmp and the separate inertias. This 'optimum' value can be determined
by equating the differential coefficient d"/dn to zero, giving

JL
n ' (1.15)
Jm

which is a well-known result. This value of n makes the referred load inertia equal to the motor inertia.
The maximum acceleration of the load is therefore

1 Tmp 1
"max ' (1.16)
2 Jm n

The corresponding acceleration of the motor is n times this value. In this analysis, the inertias of the
pinions (gearwheels) have been ignored. For a very precise evaluation, in the case of a single-stage
gearbox, the pinion inertias can be combined with (added to) the respective motor and load inertias.
Acceleration of inertia with fixed load torque. A slightly more complicated example is where the
load has a fixed torque TL in addition to its inertia.

Tmp & TL / n
" ' . (1.17)
n Jm JL / n 2

Again there is one value of gear ratio n that produces maximum acceleration, and by the same
differentiation process it is found to be

TL JL Tmp2
n ' 1 1 . . (1.18)
Tmp Jm T 2
L

If the inertias are unchanged from the previous case, the gear ratio is increased. The expression for the
optimum ratio can be substituted back in the formula for acceleration to find the maximum load
acceleration. The result is the same as eqn. (1.16); the difference is that with a larger ratio n the load
acceleration will be smaller. It is interesting to note that the maximum acceleration of the motor is
unchanged, and is equal to one-half the torque/inertia ratio of the motor.
1. Sizing, gearing, cooling, materials and design Page 1.19

Peak/continuous torque ratio of motor. In the constant-speed case, the choice of n maximizes the
utilization of the continuous torque rating of the motor, Tmc. In the acceleration case, the choice of n
maximizes the utilization of the motor's peak acceleration capability as expressed by its peak
torque/inertia ratio Tmp /Jm. Consider a load that requires both short periods of acceleration and long
periods at constant speed. Then there is a question, can the two values of n be the same? If so, the
utilization of both aspects of motor capability will be maximized at the same time.
This problem can be solved analytically in a few special cases, and one solution is given here as an
example of the kind of analysis that is needed to get a highly optimized system design. Assume that the
load torque is constant at all times, but that short bursts of acceleration (or deceleration) are required
from time to time. The peak rated torque of the motor will be used for acceleration, and the continuous
rated torque for constant speed. If we equate the two separate values of n from the appropriate formulas
given above, and if we write
Tmp ' kTmc (1.19)
where k is the ratio of peak motor torque to continuous motor torque, then the following relationship
can be derived:
n 2Jm k2
' . (1.20)
JL (k & 1)2 & 1
The left-hand expression is the ratio of the referred motor inertia to the load inertia, and we can refer
to it as the 'referred inertia ratio' or just 'inertia ratio'. For a range of values of the inertia ratio, the
equation can be solved to find the values of k that simultaneously optimize n for both the constant-speed
and acceleration periods. The most interesting result of this is that a large range of inertia ratio is
encompassed by only a small range of values of k : as the inertia ratio changes from infinity down to 2,
k changes only from 2 to 4. But values of k in this range are extremely common: so common, in fact, as
to appear to be a natural characteristic of electric motors. This implies that for most inertia ratios
where the referred motor inertia is more than twice the load inertia, the gear ratio can be chosen to
make good utilization of both the continuous torque and the peak acceleration of the motor, provided
k  2. If k < 2, the gear ratio must be chosen for constant speed or for acceleration, and cannot be optimal
for both. The property of electric motors to provide short bursts of peak torque for acceleration is one
of the most important aspects of their use in motion control systems.
General speed and position profiles. The cases considered are all idealised by rather restrictive
assumptions that may be too simple in a complex motion-control system. For detailed work it is
desirable to simulate the performance of the whole system using system-simulation software.

Figures of Merit — Power Rate, Motor Constant, Mechanical Time Constant, Speed Rate
The torque/inertia ratio is a simple figure of merit. It gives an idea of the acceleration capability of the
motor, but it is important to make clear which of many possible torque values is used to define it, and
the load inertia must not be forgotten. As with all “figures of merit”, these considerations show that
the T/J ratio is not only vulnerable to specmanship, but also conveys an oversimplified view of motion-
control applications, which are often quite complex. Nevertheless, figures of merit are common in
motor catalogue data, so they merit some definition and discussion. While figures of merit often look
good (or bad), they are often less than intuitive, and it is never quite clear how to use them for practical
calculations in an application. The literature is uniformly obscure on this important point, and
engineers can be forgiven for the idea that figures of merit are nothing more than semi-technical
parameters used to promote sales. When they are used to compare motors from different manufacturers,
they should be taken along with many other factors: there is simply no way to reduce the performance
profile of a motor to a single parameter.
Power rate (PR) is a composite figure-of-merit that combines torque T and the torque/inertia ratio T/J:
2
thus PR ' T × T/J ' T /J. There is no obvious logic in this combination, nor even a strong intuitive
interpretation, so we have to dig a little deeper to understand its origins. Arnold [8,9] introduces PR
through the idea of the rate of change of mechanical power, dPm/dt: thus if Tm is the angular velocity,
Page 1.20 SPEED’s Electric Machines

d Pm d dTm T T2
' (TTm) ' T ' T × ' . (1.21)
dt dt dt J J
This equation presupposes that the load is a pure inertia driven by a constant torque T. If we consider
the kinetic energy U of the rotating mass, its rate of change is

dU d 1 J dTm J T
' JTm2 ' × 2 Tm ' × 2 Tm × ' T Tm ' P m . (1.22)
dt dt 2 2 dt 2 J
2 2
Therefore PR ' d U/dt , the second derivative of kinetic energy. It expresses the speed at which the
motor can build up the rate of increase of kinetic energy.
Power rate has another aspect derived by considering the
minimum time required to move an inertial load over a
certain angle. The “move” in Fig. 12 is accomplished in
minimum time by accelerating at the maximum possible
rate from 0 t1 along OA, reaching maximum permissible
speed at A and continuing at that speed along AB, then finally
decelerating at the maximum possible rate along BC. Setting
aside any consideration of the maximum velocity, consider
what is needed to minimize the acceleration time t1 along OA.
This time is given by
Fig. 1.12 Power rate
1
> ' " t12 (1.23)
2 From eqns. (15) and (16) we can write
T T T
" ' ' ' .
JL JL 2 JL Jm (1.24)
n Jm × Jm Jm
2
n Jm
Combining eqns. (23) and (24), we get

2 2 > JL 2 2 > JL 2 2 > JL


t1 ' ' ' (1.25)
4 4
T / Jm T 2 / Jm PR

2
which says that t1 is minimized if T /Jm is maximized. In words, the move time t1 is inversely
proportional to the fourth root of the power rate PR.
4
Eqn. (25) says that to reduce t1 by a factor of 2, the PR must increase by a factor of 2 or 16 times. This
can be achieved by increasing T by a factor of 4, or by decreasing Jm by a factor of 16. The effect of
reducing Jm is weaker because the system is “carrying” a fixed load inertia JL, and although the gear
ratio is selected to make the referred load inertia equal to the motor inertia, this does not change the
amount of kinetic energy that must be imparted to the load.
Power rate is susceptible to specmanship because it depends on which value of torque is used to
calculate it — the rated continuous torque, the stall torque, the torque at the demagnetization point, or
the short-circuit torque — any of these could be used (in increasing order of magnitude and decreasing
order of credibility). Therefore when Power Rate is quoted, it should be stated what value of torque is
being used. Moreover, as we have just seen, the value of Power Rate inflates much faster than the
improvement in move-time t1.
2
Motor Constant Km is the ratio of torque to the square-root of the I R loss. If torque T is expressed as
kTI, where kT is the torque constant, then

kT
Km ' . (1.26)
R
1. Sizing, gearing, cooling, materials and design Page 1.21

The underlying objective is to express the torque capability of the motor in relation to its loss
2 2
dissipation, but the obvious ratio of the “torque per I R loss” would result in kTI/I R ' kT/IR, which is
8
inversely proportional to current and therefore not a motor “constant”.
Km is primarily an indicator of the stall performance. Unlike the power rate, Km is independent of
inertia, so it gives no indication of acceleration capability, although it has been argued that in practice
a high Km is often correlated with a high torque/inertia ratio, [10,11].
Speed rate is defined as

kT2 1
SR ' ' , (1.27)
JR Jm
2
and it can be seen to be the reciprocal of the traditional mechanical time-constant Jm ' JR/kT . The
reason why speed rate is preferable to Jm as a figure-of-merit is that a higher value indicates a better
motor, in common with all the other figures-of-merit.
2
If we multiply numerator and denominator of eqn. (27) byI R, we get

T 2/J PR
SR ' ' (1.28)
2
I R I 2R
2
which says that the speed rate is the ratio of the power rate to the I R loss. The speed rate thus
incorporates all the other figures-of-merit in a single parameter — the electromagnetic torque-
producing capability kT, the inertia J, and the resistive dissipation via the resistance R. At the same
time it remains a motor “constant” not dependent on the level of current or torque.
The original mechanical time-constant Jm derives from the classical analysis of variable-speed DC
motors in which the speed was primarily controlled by the armature voltage, [12]. It is the time-
constant of the exponential response of motor speed following a step-change of supply voltage. Speed
rate can therefore be interpreted as the initial gradient of this response, in “per unit speed per second”.
However, its value as a figure-of-merit is quite general; it applies to all kinds of motor and is not
contingent on the method of control.

Units of inertia
Start with the equation of rotary acceleration (Newton's second law):

T ' J" (1.29)


2
where T = torque, J = inertia, " = acceleration [rad/s ]. This can be rearranged to give a “torque-
based” definition of inertia as

T
J ' . (1.30)
"

The torque-based definition is evidently suitable for determining inertia from measured torque and
acceleration, and is “performance-oriented” in the sense that it tells how much torque is required to
achieve a certain acceleration. The units implied by eqn. (30) are units of torque divided by units of
2
acceleration, for example, [lb-ft]/[rad/s ].
There is also a “mass-based” definition

J ' M k2 (1.31)

where M is the mass and k is the so-called radius of gyration. The mass-based definition is evidently
suitable for design calculations, so it can be said to be “design-oriented”. The units implied by eqn. (31)
2 2
are units of mass multiplied by units of [length] , for example [lb]×[ft ].

8
By this argument, T/J and T2/J are also not motor “constants”.
Page 1.22 SPEED’s Electric Machines

Inertia units defined in terms of eqn. (30) are not necessarily consistent with eqn. (31). Already this
2 2
is true for the quoted examples of [lb-ft]/[rad/s ] and [lb]×[ft ], because the [lb] in the torque-based unit
is force, whereas the [lb] in the mass-based unit is mass. These are not the same, being related by g, the
acceleration due to gravity. Only the so-called fundamental units are simultaneously valid in both
equations.
2
In the S.I. system, the fundamental units are [kg-m ] for inertia J, [kg] for mass M, and [m] for radius
2
k. In the English system, the fundamental units are [slug-ft ] for inertia J; [slug] for mass M; [lb] for
force (sometimes written [lbf]); and [ft] for radius k. The slug is approximately 32@18 lb (mass); it can
2
be identified as the mass that will accelerate at [1 ft/sec ] when a force of 1 [lbf] (pound force) is applied
to it. The term “slug” is rarely used. But if fundamental units are not being used, a conversion factor
will appear in eqn. (30) or eqn. (31), or both, depending on the units chosen. (See eqn. (42), below).
2
If T is in [lb-in] (really [lbf-in]), the units of J in eqn. (30) are [lb-in]/[rad/s ]. This is usually
2
abbreviated as [lb-in-s ]. Now
1 [lb&in] ' 0@11298 [N&m] .
Therefore in “eqn. (30) units”,

1 [lb&in&s 2] ' 0@11298 [N&m&s 2] or 0@11298 [N&m] / [rad/s 2] .


2
The [N], the [m] and the [s] are all fundamental units in the S.I. system: thus 1 [N]/[m/s ] = 1 [kg] for
2 2 2 2
linear motion, and 1 [N-m]/[rad/s ] = 1 [N]/[m/s ] × 1 [m ] = 1 [kg-m ]. These units can be used in eqn.
(31) because they are fundamental units. Hence the conversion

1 [lb&in&s 2] ' 0@11298 [kg&m 2]


where the left-hand side is in “eqn. (30) units” and the right-hand side is in in “eqn. (31) units”.
3 2
Now we can make a further conversion of the right-hand side, using 1 [kg] = 10 [g], 1 [m] = 10 cm :

1 [lb&in&s 2] ' 0@11298 [kg&m 2] × 103 [g/kg] × (102 [cm/m])2 ' 1@1298 × 106 [g&cm 2] .
where the left-hand side is in “torque-based” units and the right-hand side is in “mass-based” units.
2
Consider a further conversion of the right-hand side into [lb-in ], all in “mass-based” units:

1 [lb&in&s 2] ' 0@11298 [kg&m 2] ' 0@11298 × 2@2046 [lb/kg] × (39@37 [in/m])2 ' 386@1 [lb&in 2]
2 2
Clearly we have to be careful with units: not only is 1 [lb-in-s ] equal to 386@1 [lb-in ], but they are not
really compatible because the first is consistent with eqn. (30) (with T in [lbf-in]), while the second is
2 2
not. It is more precise to write [lbf-in-s ] and [lbm-in ], where 1 [lbm] is 1 pound mass to distinguish it
from 1 [lbf] (force); but few do.
2
Now consider an inertia expressed as 1 [lb-in ] in terms of the “mass-based” definition eqn. (31). In
fundamental units compatible with eqn. (31) this inertia is

1 1 1
1 [lbm&in 2] ' × ' [slug&ft 2] .
32@18 122 4633@9
Now 1 [slug] = 14@597 [kg] and 1 [ft] = 0@3048 [m]. We can use these to convert directly to S.I. units:

1
1 [lbm&in 2] ' × 14@597 [kg/slug] × (0@3048 [m/ft])2 ' 0@0002926 [kg&m 2] .
4633@9
2
Both sides of this equation are in “mass-based” units. It can be converted into [g-cm ] as before :

1 [lbm&in 2] ' 0@0002926 × 103 × (102)2 ' 2,926 [g&cm 2] .


Provided that we convert consistently according to eqn. (30) or eqn. (31), unit conversions can be done
directly without going through the intermediate steps. The confusion arises when we convert from
torque-based units consistent with eqn. (30) to mass-based units consistent with eqn. (31), or vice versa.
1. Sizing, gearing, cooling, materials and design Page 1.23
2
Finally consider what happens if inertia is defined as Wk — the “weight-based” definition

J ' Wk 2, (1.40)
where W is the weight, not the mass. If in the S.I. system the weight is expressed in [kg], then it is
numerically equal to the mass M, and J calculated from eqn. (40) is immediately consistent with both
eqns. (30) and (31) and can also be used in eqn. (29) to calculate acceleration. Strictly speaking, weight
is a force and should be expressed in [N], but if we did that we would need to introduce g in eqn. (29).
2 2
In the English system, W is normally expressed in [lb] (really [lbf]). So if Wk is calculated in [lbf-ft ]
it is not consistent with either eqn. (30) or eqn. (31) and cannot be used in eqn. (29) to calculate
2
acceleration. When the “f” is omitted and the unit is written [lb-ft ], as is common, it has the
appearance of a mass-based definition, but the acceleration equation must now be written

T T [lb&ft] T
" ' ' or [rad/s 2] ' or " ' × 32@18 . (1.41)
2 2 2
J Wk /g [lb&ft ] / 32@18 [ft / s ] J
If W is replaced by the mass M in [slug] , using

W [lbf] W
M [slug] ' ' , (1.42)
g [ft/s ] 2 32@18

then all of eqns. (29), (30) and (31) can be used without further modification provided that T is in [lbf-ft].
But the slug is generally eschewed by even diehard users of the English system, who must therefore use
eqn. (41) with the conversion factor g.
This inconvenience is quite possibly why engineers using the English system sometimes define inertia
in terms of eqn. (30) instead of eqn. (31). However, both equations are needed : eqn. (30) for consistency
with eqn. (29) in dynamic calculations, and eqn. (31) for design calculations. Therefore in the English
system there is no way of escaping the conversion factor g completely. The S.I. system escapes it by
expressing weight [force] in [kg] instead of [N], and torque in [N-m]. It appears that the [kg] force unit
is less unpalatable than the [slug] mass unit — with fortunate consequences.
In giving examples of the inertias of different solid shapes, Machinery's Handbook [1948 edn.] gives
examples in terms of dimensions, naturally using the “mass-based” definition of eqn. (31). The
equations are given without units, implying that fundamental units are assumed (in either system) —
laudable and worthy of note . But practical formulas for calculating acceleration are quoted in English
units only — with inevitable conversion factors that are recognizable as g.

Example
3
Consider a solid cylinder of diameter 2", axial length 1@5", and mass density 0@28 [lb/in ]. The mass is

B 2 B 1@3195
M ' D LD ' × 22 × 1@5 × 0@28 ' 1@3195 [lb] or ' 0@0410 [slug]
4 4 32@18
According to eqn. (31) the inertia in fundamental “eqn. (31) units” is calculated with k ' D/(2/2) [ft]:
2
2@0 / 2
J ' M k 2 ' 0@0410 × ' 0@00014327 [slug&ft 2]
12 2
2 2 2
Since 1 [slug-ft ] = 1 [lb-ft]/[rad/s ] or 1 [lb-ft-s ] in fundamental units, we can write this as

J ' 0@00014327 [lb&ft&s 2]


2
and this can be directly converted in terms of “eqn. (30) units” to [lb-in-s ] : thus

J ' 0@00014327 × 12 ' 0@0017084 [lb&in&s 2] .


2 2
To convert to [g-cm ], however, we must go back to “eqn. (31) units” and convert 0@00014327 [slug-ft ] :
thus

J ' 0@00014327 × (32@18 × 0@4536 × 103) × (12 × 2@54) 2 ' 1930@7 [g&cm 2]
Page 1.24 SPEED’s Electric Machines
2
This can be directly converted to [lb-in ]:

1 1
J ' 1930@7 × × ' 0@6597 [lb&in 2] .
453@6 2@542
2 2
Note that we get this same result (much more simply) by multiplying the mass 1@3195 [lb] by k = ½ [in ].
2 2 2
If we use the S.I. system we have M = 1@3195 × 0@4536 ' 0@5985 [kg], and k = ½ × 0@0254 = 0.0003226 [m ].
Then

J ' 0@5985 × 0@0003226 ' 0@00019307 [kg&m 2] .


2
This is in “eqn. (31) units” and can be directly converted to [g-cm ] : thus

J ' 0@00019307 × 103 × (102)2 ' 1930@7 [g&cm 2] ,


which checks the earlier value.

1.10 SALIENCY
Saliency is an important property of electrical machines that determines the ability to produce smooth
torque when operating with “ordinary” current and voltage waveforms, especially pure DC or pure AC.
It is also fundamentally related to the production of reluctance torque.
The basic concept of “saliency” is the presence or absence of projections or saliencies from the surface
of the stator or rotor. The field poles of a universal motor provide a common example, as represented
in Fig. 1.13D. Another well-known example is the field poles of a synchronous machine, Fig. 1.13A.
Slotting is normally neglected when saliency is being considered. It is true that the teeth in a slotted
surface are “saliencies”, and they are even associated with a form of reluctance torque known as
cogging torque. However, this is normally regarded as a parasitic, second-order effect. Unlike the
normal reluctance torque found in salient-pole machines, the cogging torque is zero when averaged over
one revolution, and it serves no useful purpose.
Fig. 1.13 shows the four possible combinations of salient-pole or nonsalient-pole rotors and stators.
Thus Fig. 1.13A shows a nonsalient-pole stator with a salient-pole rotor (wound-field synchronous
machine or synchronous reluctance machine); B shows a machine with no saliency (induction
machine); C shows a machine with “double saliency”(switched reluctance machine); and D shows a
salient-pole stator with a nonsalient-pole rotor (wound-field DC commutator motor or universal motor).
In each case there is a single coil on the stator and a single coil on the rotor to represent the windings.
An electrical test for saliency is the variation in self-inductance of one of these coils when the opposite
member is rotated. For example, if the rotor of A is rotated, the inductance of the stator coil will vary,
and so the rotor is deemed to have saliency. If the rotor of Dis rotated, the inductance of the stator coil
remains constant. Therefore the rotor in Fig. 1.13D has no saliency. On the other hand, if the stator is
rotated, the self-inductance of the rotor coil varies, so the stator is deemed to have saliency.
The generalized machine theory shows that if a machine has at least one member with no saliency, then
there is a form of that machine that can produce constant, ripple-free torque when excited with pure
AC or pure DC. The practical importance of this is profound. For example, machine A can produce
smooth torque when the stator has a polyphase distributed winding fed with balanced polyphase AC
current, when the rotor is either unexcited or fed with pure DC. Again, machine D can produce smooth
torque when the stator is fed with pure DC, and the rotor is fed with pure DC through brushes and a
commutator that has a sufficiently large number of segments.
Machine B (the induction motor) can produce smooth torque when the stator has a polyphase
distributed winding fed with balanced polyphase AC current, while the rotor has a similar winding or
a cage winding with a similar AC current distribution induced by slippage. Machine C (the switched
reluctance machine) cannot produce smooth torque with either a pure DC or pure sinusoidal AC
current waveform. To achieve smooth torque, it requires a specially profiled current waveform.
1. Sizing, gearing, cooling, materials and design Page 1.25

Fig. 1.13 Possible combinations of salient-pole and nonsalient-pole stators and rotors

In most of the common machine types the torque is produced as a result of the variation of mutual
inductance between stator and rotor windings. The variation of self-inductance associated with
saliency can be used to produce reluctance torque, and in reluctance motors this is the only available
component of torque.

1.11 COOLING
The need for cooling
There are two major aspects to the thermal problem in electrical machines:
1. heat removal; and
2. temperature distribution within the motor.
The main reasons for limiting the temperature rise of the windings and frame of a motor are:
1. to preserve the life of the insulation and bearings;
2. to prevent excessive heating of the surroundings; and
3. to prevent injury caused by touching hot surfaces.

In permanent-magnet motors the temperature of the magnets needs to be kept under control, in order
to avoid demagnetization. (See p. 44).
Insulation life. The life of electrical insulation is inversely related to the temperature. A sustained
10EC increase in temperature reduces the insulation life by approximately 50%. Similar considerations
apply to bearings. Bearings may be filled with high-temperature grease for hot-running applications,
but in aerospace machines the bearings are usually lubricated by separately-cooled oil or oil mist.
Page 1.26 SPEED’s Electric Machines

The extent to which excessive temperatures can be tolerated depends on the duration and the actual
temperatures reached. An interesting example of a motor designed for exceptionally high temperatures
is the FUMEX motor. This motor, manufactured by Brook Hansen, is designed to extract fumes via the
ventilation systems of public buildings in the event of fire. It can operate in an ambient temperature
of 300EC for a limited period of 30 minutes.
Heating of the surroundings is obviously undesirable especially if the motor is heating the
equipment it is driving. For this reason it is important to minimize rotor losses conducted along the
shaft. PM motors have cooler rotors than DC or induction motors. In some applications such as
hermetic compressors used in air-conditioning, refrigeration, etc., the motor losses are removed by the
working fluid, reducing the thermodynamic efficiency of the system.
To prevent injury or harm from touching, exposed surfaces must be kept below 50EC. In certain
applications (e.g., under car bonnets), this requirement is impossible to meet because the "ambient"
temperature under the bonnet may reach 100EC. In industrial applications the ambient temperature is
generally less than 50EC, and NEMA ratings for electrical insulation assume an ambient temperature
of 40EC. In aerospace applications motors and generators may be directly cooled by oil or fuel and
coolant temperatures can be as high as 100EC.
The increase in winding temperature increases the resistivity of the windings: a 50EC rise by 20%, and
a 135EC rise by 53%, increasing the I 2R losses by the same amount if the current remains the same. The
resistance increase is used in test procedures to determine the actual temperature rise of the winding,
but this is obviously an average temperature; hot-spot temperatures can be 10!20E higher. At any
temperature T EC the resistivity of copper can be calculated as
D ' D20 [ 1 " ( T ! 20 ) ] ohm&m (1.51)
where " = 0.00393 /EC , is the temperature coefficient of resistivity and D20 is the resistivity at 20EC, that
!8
is, 1.728 ×10 ohm-m.

Cooling and efficiency


The definition of efficiency is
Output power
Efficiency 0 ' × 100% . (1.52)
Input power

Also
Watts loss ' Output power Input power . (1.53)
Hence

1
Watts loss ' Output power × 1 . (1.54)
0

Table 1.5 shows the result of eqn. (1.54) for different levels
Efficiency Watts loss
of efficiency. When the efficiency is 100%, obviously the
Output power
watts loss is zero. When the efficiency falls to 90%, the
watts loss is equal to 11% of the output power, and when it 100 0
falls to 50%, the watts loss is equal to the output power.
90 0@11
Between 90% efficiency and 50% efficiency, the watts loss
75 0@33
increases by a factor of 9. Even between 90% and 75%, the
loss increases by a factor of 3. So although we might 50 1@0
describe a small motor with 75% efficiency as “fairly
efficient”, its losses are three times as high as those of a TABLE 1.5
motor with 90% efficiency and the same output power. EFFICIENCY AND WATTS LOSS
1. Sizing, gearing, cooling, materials and design Page 1.27

Responsibility for temperature rise


The designer of an electrical machine can more or less guarantee the efficiency of that machine when
it is operated under specified conditions, but he clearly cannot be held responsible for the temperature
rise if the machine is used in abnormal conditions. It is the user who determines the actual power
output, the ambient temperature, and in many cases the cooling. Since failures are often attributable
to overheating, it is wise to be clear about where these responsibilities lie.
In the factory, testing is an essential part of the product development process. Prototype testing
normally takes place on a dynamometer, which measures speed, torque, power, and electrical quantities
such as voltage, current, and power factor. Dynamometer testing is commonly used to verify design
calculations, and temperature rise measurements are almost always included, not just at the frame
surface, but throughout the machine. Thermocouples and resistance thermometers are used for this,
and the flow rates of coolants are often measured as well — usually airflow. Temperature rise should
ideally be measured in the final application under worst-case loaded conditions. With very large
machines it is sometimes necessary to conduct “synthetic” load tests, such as back-to-back testing of
two similar machines, or operation with a zero-power-factor load, in order to achieve full-load loss
without having to load the machine to its normal power output.
Life testing may follow prototype testing, to identify defects in the design or the manufacturing process
which were not anticipated at the design stage. Life testing is often “accelerated” by overloading the
machine in order to shorten the time-to-failure so that the results may be obtained in a reasonable time.

Heat Removal

Fig. 1.14 “Degrees C per Watt” values obtained by measurement on a small motor.
(Data kindly supplied, with permission, by Erland Persson)

A very simple measure of the cooling capability of a motor is the “degrees C per watt”. This is the ratio
of the temperature rise to the total watts loss. The temperature rise can refer to the winding or to the
frame surface, as in Fig. 1.14. It has the units of thermal resistance. As a single parameter
characterizing the thermal resistance, it is useful in the analysis of intermittent operation beginning
on p. 36.
The values in Fig. 1.14 are specific to one size of motor. As the motor size increases, the “degrees C per
watt” tends to decrease — indeed it must decrease: in large machines forced cooling, even forced
internal cooling, is required to limit the temperature rise. The values in Fig. 1.14 also include all means
of heat removal (mainly convection with a small amount of radiation).
Page 1.28 SPEED’s Electric Machines

Detailed analysis of cooling


In most industrial and commercial motors, heat is removed by a combination of
1. conduction to the frame mountings;
2. air convection, which may be natural or forced; and
3. radiation.

In highly-rated machines direct cooling methods are used:


1. oil mist, especially in aerospace machines;
2. immersion in refrigerant, in "hermetic" motors used in refrigerator compressors;
3. direct conductor cooling, with hydrogen, oil, or water forced through hollow conductors,
especially in turbine-generators.

Conduction. The conduction equation for a block of thickness t and area A is


dT )T
Q ' k A . k A W (1.55)
dx t

where )T is the temperature difference through the thickness t. The coefficient k is the thermal
conductivity, with units (W/m2) per (EC/m), i.e. W/EC-m. The thermal conductivity is a material
property, and usually it is a function of temperature. Most metals have high thermal conductivities,
especially those which are also good electrical conductors. On the other hand, electrical insulating
materials and most fluids have low thermal conductivities.

As an example, consider the flow of heat along a conductor whose cross-section area is A = 64 mm2 and
length 50 mm, when the RMS current-density is 7 A/mm2. The electrical resistivity of copper is 1.7 × 10!8
3
ohm-m, so heat is produced at the rate of J 2D = (7 × 106)2 × 1.7 × 10!8 = 833,000 W/m3 or 83.3 W/cm . In one
2
conductor the I R loss is therefore 833,000 × 64 × 10 × 50 × 10 = 2.7 W. To take the most pessimistic
!6 !3

estimate, assume that all of this heat is generated at the mid-point of the coil-side, half-way along the
motor. The thermal conductivity of copper is 387 (W/m2) per (EC/m). So the temperature gradient along
the coil-side is given by eqn. (1.22) as

dT Q 2.7
' ' ' 108 EC/m. (1.56)
dx kA 387 × 64 × 10!6

Since the heat can flow in both directions, the temperature-gradient is only half this value, and the
temperature rise between the ends of the stack and the centre is therefore 110/2 × 50 × 10!3/2 = 1.4 EC,
which is negligible. A more thorough analysis would have to consider the full diffusion equation along
the length of the coil-side, but this quick calculation reveals that such sophistication is not needed in
the example considered.
Thermal resistance and contact resistance. Eqn. (1.24) can be used to define thermal resistance as
the ratio of temperature difference )T to heat flow rate Q : the symbol used for thermal resistance is
R, with units EC/W. Thus
)T t
R ' ' EC/W. (1.57)
Q kA

The thermal resistance is a "lumped parameter" that can be used to model the conduction through a
region or interface where the individual values of k, A, and t may be difficult to determine. The contact
resistance between two surfaces is usually treated in this way, as, for example, between the frame and
the stator core. The temperature drop across a thermal resistance is given by eqn. (1.24) as )T = QR. For
example, if the contact resistance between the motor flange and the mounting plate is 1EC/W, then with
40W flowing though it the temperature difference across the interface would be 40EC.
1. Sizing, gearing, cooling, materials and design Page 1.29

Radiation. Radiation is described by the Stefan-Boltzmann equation


Q
' e F ( T14 ! T24) W/m 2 (1.58)
A
where F is the Stefan-Boltzmann constant, 5.67 × 10!8 W/m2/K4 for a black body, T1 is the absolute
surface temperature of the radiating body in degrees Kelvin, and T2 is the absolute temperature of the
surroundings.9 A black body is a perfect radiator, that is, one which reflects no radiated heat but
absorbs all the heat radiated towards it. Real surfaces are imperfect radiators, and their radiative
effectiveness relative to that of a black body is called the emissivity e. A black matt surface can achieve
an emissivity as high as 0.98, but 0@8 0.9 is typical for painted motor surfaces. For example, a surface
with an emissivity of 0.9 that is 50EC above the surroundings at 50EC, has a net heat transfer rate of

0.9 × 5.67 × 10!8 × ( (50 50 273)4 & (50 273)4 ) (1.59)


2 2
which is 432 W/m or 0.28 W/in . This is quite a useful component of the heat-removal capability.
Convection. Heat removal by convection is governed by Newton's Law:
Q
' h )T W/m 2 (1.60)
A

where )T is the temperature difference between the cooling medium and the surface being cooled, and
h is the heat-transfer coefficient. The units of h are W/m2/EC. The value of h depends on the viscosity,
thermal conductivity, specific heat, and other properties of the coolant, and also on its velocity. In natural
convection the flow of coolant is not assisted by fans, blowers, pumps etc. In forced convection the flow is
assisted by one of these external means.
The heat transfer coefficient for natural convection around a horizontally-mounted unfinned cylindrical
motor can be roughly estimated as
1/4
)T
h . 7.5 W/m 2/EC (1.61)
D

where D is in mm. For example, for an unfinned cylinder of diameter D = 100 mm and a temperature rise
of 50EC, the natural-convection heat-transfer coefficient is calculated as 6.3 W/m2/EC. For a )T of 50EC,
the heat transfer rate is then given by eqn. (1.28) as 6.3 × 50 = 315 W/m2. As a first approximation this
value can be applied to the whole surface including the ends, but if the motor is flange-mounted then only
one end is available for convective cooling.
Forced convection (with a shaft-mounted fan or an external blower) increases the heat-transfer coefficient
by as much as 5!6 times, depending on the air velocity. The increase in heat-transfer coefficient is
approximately proportional to the square-root of the air velocity. An approximate formula for the forced-
convection heat-transfer coefficient is

V (1.62)
h . 125 W/m 2/EC
L

where V is the actual air velocity [m/s] and L is the frame length [mm] (assumed parallel to the direction
of airflow). For a motor of length 100 mm, if the air velocity is 4 m/s, this formula predicts h = 25
W/m2/EC. This is 4 times higher than for natural convection.
Some rules of thumb for "calibration". In a water-immersed wire 1 m long, 1 mm diameter, a power
loss of 22 W (0.022 W per mm length) is sufficient to boil the water at the wire surface. The wire surface
temperature is 114EC and the heat transfer coefficient (see below) is 5000 W/m2/EC. The heat flow at the
wire surface is 0.07 W/mm2 and the current-density in the wire is approximately 35 A/mm2. In normal
motors, the rate at which heat can be abstracted is far less than this, and current-densities over 30 A/mm2
2
are achievable only for short bursts. 35 A/mm is sufficient to fuse a copper wire in free air.

9
The absolute temperature in degrees Kelvin (K) is the temperature in EC plus 273.
Page 1.30 SPEED’s Electric Machines

The maximum rate of heat removal by natural convection and radiation (with 40EC rise) is only about 800
W/m2. With forced air convection the rate increases to about 3000 W/m2, and with direct liquid cooling
about 6000 W/m2. A motor that generates more heat than can be removed at these rates must absorb the
heat in its thermal mass, which permits the output power to be increased for a short time. These rates
limit the heat generated per unit volume to about 0.012 W/cm3 for natural convection, 0.3 W/cm3 for
metallic conduction or forced-air convection, and 0.6 W/cm3 for direct liquid cooling.
The permissible current-density cannot be directly related to the temperature rise of the winding by a
simple general equation, because the heat transfer rate depends on the shape of the conductors. As an
example, 1 cm3 of copper can be made into a stubby cylinder of 1 cm diameter and 1.27 cm length, or a long
wire of 1 mm diameter and 1.27 m length. The short cylinder has a cylindrical surface area of 4 cm2 while
the long wire has a surface area of 40 cm2. The loss density in the conductor is J 2D where J is the current
density and D is the resistivity. With ten times the surface area the long wire can dissipate ten times the
heat, assuming the same heat transfer coefficient in both cases. This suggests that the permissible
current-density in the long wire can be /10 times that in the short stubby cylinder.
If rated torque is required at very low speed, a shaft-mounted fan may not provide enough coolant flow
to keep the motor cool. DC motors often have separate AC-driven fans, because they have to work for
prolonged periods at low speed with high torque. Since most of the heat in a DC motor is generated on the
rotor, good internal airflow is essential. In DC motors the external fan is usually mounted to one side of
the motor, where it is easily accessible, and does not increase the overall length. With vector-controlled
induction motors a common practice is to mount the fan in line with the motor at the non-drive end, and
arrange it to blow air over the outside of the finned frame. The fan may increase the overall length by up
to 60%. Brushless motors have less severe problems because most of the heat at low speed is generated
in the stator windings, and very little on the rotor.

Internal temperature distribution


The steady-state temperature distribution within the motor is essentially a diffusion problem. The most
important aspect of the problem is finding the hottest temperature in the motor, given a certain
distribution of losses and a known rate of heat removal. It is difficult to solve precisely, because of three-
dimensional effects and because some thermal resistances (such as the resistance between slot conductors
and slot liner) are hard to calculate.
The differential equation for three-dimensional conduction of heat is the so-called diffusion equation:
1 Mq 1 MT
L 2T ' (1.63)
k Mt " Mt

where
M2T M2T M2T
L 2T ' (1.64)
Mx 2 My 2 Mz 2
and
k
" ' m 2/s (1.65)
Dc

is the diffusivity in SI units. In SI units, k is the thermal conductivity in W/mEC; c is the specific heat in
kJ/kgEC, and D is the density in kg/m3. In a structure as complex as an electric motor the heat conduction
equation is a complex boundary-value problem that is best solved by computer-based numerical methods
such as the finite-element method.
In electric motors internal convection and radiation may be as important as conduction, and when the
differential equation is extended to include them, matters become very complicated, even for steady-state
calculations. During transients the temperature distribution can be very different from the steady-state
distribution, and different methods of analysis may be needed for the two cases.
1. Sizing, gearing, cooling, materials and design Page 1.31

Fig. 1.15 Thermal equivalent circuit. S = stator (tooth centre), T = tooth (at airgap), Y =
stator yoke, E = end-winding, C = conductors (at central plane), G = airgap, H =
shaft, A = ambient. BloCool = heat abstracted by through airflow (W), R =
radiation, U = conduction, V = convection. Double letters refer to thermal
resistances, e.g., CT = thermal resistance from the conductors to the stator teeth.

Thermal equivalent circuit. For most purposes it is sufficient to use a thermal equivalent circuit of the
interior of the motor, Fig. 1.12. This is analogous to an electric circuit, in that heat is generated by
"current sources" and temperature is analogous to voltage. The rate of generation of heat in a source is
measured in Watts. The heat flow rate, which is also measured in Watts, is analogous to current.
Resistance is measured in EC/W. The copper losses, core losses, and windage & friction losses are
represented by individual current sources, and the thermal resistances of the laminations, insulation,
frame, etc. are represented as resistances. In the simplest possible model, all the losses are represented
together as one total source, i.e. the individual sources are taken as being in parallel. The thermal
equivalent circuit is really a lumped-parameter model of all the heat-flow processes within the motor as
well as the heat removal processes discussed earlier.
The thermal equivalent circuit should ideally take into account the anisotropy effects: for example, the
effective thermal conductivity through a lamination stack is lower in the axial than the radial direction.
A more complex thermal equivalent circuit may include provision for direct cooling of the winding
conductors, or for direct cooling of the rotor shaft. If it also includes the thermal masses or capacities of
the winding, the rotor and stator laminations, the frame, the shaft, and other massive components, then
it can be solved for transient as well as steady-state heat transfer. The heat removal routes by conduction,
radiation, and convection are represented by thermal resistances. For convection the appropriate
resistance Rv is given by
1
Rv ' EC/W (1.66)
hA

where A is the appropriate surface area for convective heat-transfer and the subscript "v" stands for
convection. If h is a function of the temperature-difference, the equivalent circuit becomes non-linear and
requires an iterative solution. For radiation the equivalent thermal resistance Rv is the ratio of the
temperature difference T1 ! T2 to the radiation heat exchange rate Q in eqn. (1.25). Clearly this is non-
linear. However, the non-linearity is often neglected and a fixed value of Rv is calculated assuming that
the case temperature is known.
Page 1.32 SPEED’s Electric Machines

Fig. 1.16 Axial-flow heat-transfer model. Filled elements are at uniform temperature, while those
with no fill have axial variation of temperature, as indicated by the arrows.

Axial temperature-variation along certain elements. A lumped-parameter model capable of


including the axial temperature variation in all machine components would need tens or hundreds of
nodes. The simplified model in Fig. 1.16 permits axial variation of temperature only in certain
components, namely the frame, the stator conductors, and the rotor conductors. These elements are
“isolated” from one another by the stator and rotor cores and the airgap, which have no axial variation
of temperature. Each can be modelled by means of one-dimensional thermal conduction. Internally-
generated heat is included in the analysis, and heat may be transferred between the outer surface of each
element and the constant-temperature reservoir surrounding it. Fig. 1.17 shows the model.

Element Internal heat generation External heat transfer


Frame None Convection and radiation to ambient
2
Stator conductors J D [W/kg] Conduction to slot wall
2
Rotor bars J D [W/kg] Conduction to slot wall

The axial temperature distribution is determined by the heat conduction equation [13],

d 2T hP q
(T T0) ' 0 (1.67)
dx 2 kA k
2
where T [EC] is the temperature at a position x [m] measured from the left-hand end, h [W/m EC] is the
2
heat transfer coefficient at the surface, P [m] is the perimeter of the conductor, A [m ] is the cross-section
3
area of the conductor, k [W/mEC] is the thermal conductivity of the conductor, and q [W/m ] is the rate
of internal heat generation in the conductor. The solution is

mx q
T ( x ) ' C1 e C2e m x T0 (1.68)
km 2
2
where m ' hP/kA and

21e 2 m L 22e m L 22e m L 21


C1 ' ; C2 ' (1.69)
e 2mL 1 e 2mL 1
with
q q
21 ' T1 T0 ; 22 ' T2 T0 . (1.70)
2
km km 2
T1 and T2 are the temperatures at x ' 0 and x ' L respectively. By differentiating eqn. (1.68), we can find
the location xTmax of the maximum temperature:

1 C1
xTmax ' ln , (1.71)
2m C2
and if we substitute this into eqn. (17) we obtain the maximum temperature as
1. Sizing, gearing, cooling, materials and design Page 1.33

Fig. 1.17 Heat conduction model. The heat flows Q1, Q2 and Q0 are shown positive when
they are outwards from the conductor.

q
Tmax ' T0 2 C1 C2 . (1.72)
km 2
Similarly the average temperature of the conductor is given by
q 1
Tavg ' T0 C2 (e m L 1) C1 (e mL
1) . (1.73)
km 2 mL

To develop the equivalent-resistance network for linking into the lumped-parameter model, we first
calculate the heat flows. The total heat generated internally is

Qi ' q A L (1.74)
3 2
where q [W/m ] is the rate of heat generation, equal to J D/* in conductors, where J is the current density
2 3
[A/m ], D is the resistivity [S m] and * is the density [kg/m ]. The heat transfer at the left-hand end is

dT
d x /0 x ' 0
Q1 ' kA ' k A m ( C1 C2 ) . (1.75)

Note that if dT/dx is positive at the left-hand end, heat is flowing out of the conductor from right to left,
and Q1 is then positive. At the right-hand end,

dT
d x /0 x ' L
mL
Q2 ' kA ' k A m ( C1e C2e m L ) , (1.76)

If dT/dx is negative at the right-hand end, heat is flowing out from left to right, and Q2 is then positive.
In the steady-state, the heat transferred to the ambient over the surface is given by
L
Q0 ' h p (T T0 ) dx ' Q1 Q2 Qi , (1.77)
0

where Q i ' qAL is the total internal heat generation. With Q1, Q2, Q0 known, the resistances can be
computed as
Tm T1 Tm T2 Tm T0
R1 ' ; R2 ' R0 ' . (1.78)
Q1 Q2 Q0

In the case of the frame, the effective heat transfer coefficient h can be determined from convection and
2
radiation data. A typical value might be 20 W/m EC.
In the case of the stator conductors and rotor bars, the surface heat transfer is by conduction. For a
simple slot-liner of thickness tL and thermal conductivity k L , h can be taken as k L /t L . For example, with
3 2
a slot-liner of thickness t L ' 0@2 mm having k L ' 0@25 W/mEC, we get h ' 0@25/(0@2 × 10 ) ' 1,250 W/m EC.
Page 1.34 SPEED’s Electric Machines

Again, if there is an air layer between the slot-liner and the slot wall, having thickness tA ' 0@03 mm and
2
thermal conductivity kA ' 0@009246 W/mEC, we get h ' 308 W/m EC. With both the slot-liner and the air
layer in series, the effective heat transfer coefficient is
1 1 1
h ' ' ' ' 247@2 W/m 2 EC .
tA tL 0@03 × 10 3
0@2 × 10 3 1 1 (1.79)
kA kL 0@009246 0@25 308 1250

The air layer reduces the effective heat transfer coefficient by a factor of about 5.
On the rotor there may be no slot-liner, and the effective air layer around the rotor bars may be very
2
small, so a high value of h (possibly in the region of 500 ! 1000 W/m EC) may be encountered. This will
tend to reduce the temperature gradient along the rotor bars, by holding the rotor bar temperature close
to the rotor core temperature.
If h ' 0 there is no lateral heat transfer from the surface of the conductor. In this case the heat transfer
equation degenerates to

d 2T q
' 0. (1.80)
dx 2 k
The solution is
q 2
T ' x C1 x C2 , (1.81)
2k
where
T2 T1 qL
C1 ' ; C2 ' T1 . (1.82)
L 2k

The maximum temperature occurs at


k
xTmax ' C1 (1.83)
q

and the maximum temperature is given by substituting this in eqn. (1.81). The average temperature is

q L3 L
Tavg ' C1 C2 . (1.84)
2k 3 2
The heat transfer out at the left-hand end is given by

dT
d x /0 x ' 0
Q1 ' kA ' k A C1 , (1.85)
and at the right-hand end by

dT q
d x /0 x ' L
Q2 ' kA ' kA( L C1 ) , (1.86)
k
Of course Q0 ' 0, but eqn. (1.78) can still be used for R1 and R2. R0 is infinite.
Another special case arises when there is no internal heat generation. Then the temperature distribution
is still given by eqn. (1.81), but with q ' 0. This can be written
T ' (1 u ) T1 u T2 , (1.87)
where u ' x/L. The temperature variation is linear, and the maximum temperature is at the left-hand end
if T1 > T2 or at the right-hand end if T1 < T2. The average temperature is then simply (T1 T2)/2. The heat
transfers out at the left- and right-hand ends are given by

dT T2 T1 dT
d x /0 x ' 0 d x /0 x ' L
Q1 ' kA ' kA ' kA ' Q2 . (1.88)
L
1. Sizing, gearing, cooling, materials and design Page 1.35

Some useful data is provided in the following tables.

Motor type Class B Class F Class H


1.15 Service Factor 90 115 140
1.00 Service Factor 85 110 135
TEFC 80 105 125
TENV 85 110 135
TABLE 1.6
TEMPERATURE RISE BY RESISTANCE AND INSULATION
(NEMA Standard MG-1), EC. Assumes 40EC ambient temperature.

Material Emissivity
Polished aluminium 0.04
Polished copper 0.025
Mild steel 0.2-0.3
Grey iron 0.3
Stainless steel 0.5-0.6
Black lacquer 0.9-0.95
Aluminium paint 0.5
TABLE 1.7
SELECTED EMISSIVITIES

Material D (20EC) k Sp. Heat Density


ohm-m × 10 !8
(W/m K) kJ/kg/EC kg/m3
Copper 1.72 360 0.38 8950
Aluminium 2.8 220 0.90 2700
0.1% Carbon steel 14 52 0.45 7850
Silicon steel 30!50 20!30 0.49 7700
Cast iron 66 45 0.5 7900
Cobalt-iron 40 30 0.42 8000
4
Ceramic magnet 10 4.5 0.8 4900
Re-Co magnet 50 10 0.37 8300
NdFeB magnet 160 9 0.42 7400
Kapton® 303 V/μm* 0.12 1.1 1420
Teflon 260V/μm* 0.20 1.2 2150
Pressboard/Nomex 10kV/0.22mm* 0.13 — 1000
Epoxy resin 30kV/mm* 0.5 1.7 1400
Water (20EC) 0.0153 4.18 997.4
Freon 0.0019 0.966 1330
Ethylene Glycol 0.0063 2.38 1117
Engine oil 0.0037 1.88 888
TABLE 1.8 SELECTED MATERIAL PROPERTIES
*Dielectric strength
Page 1.36 SPEED’s Electric Machines

1.12 INTERMITTENT OPERATION

Fig. 1.18 Intermittent operation

Intermittent operation is normal for brushless PM motors, because most of the applications that use them
are motion-control applications with programmed moves, accelerations, decelerations, stops, starts, and
so on. Consequently the temperatures of the windings and magnets are constantly varying. A simple
example is shown in Fig. 1.13, where the motor executes a simple on-off sequence: on for tON and off for tOFF,
after which the on/off cycle repeats indefinitely. The cycle time tcy is
tcy ' tON tOFF . (1.89)
The duty-cycle d is defined as
tON tON
d ' ' . (1.90)
tcy tON tOFF

The most efficient use of the thermal capability of the motor will be made if the maximum winding
temperature Tmax just reaches the rated value Tr at the end of each on-time. Because the power dissipation
is interrupted with cool-down intervals tOFF, the power Pd that can be dissipated during the on-times may
exceed the steady-state continuous dissipation rating of the motor Pr, and therefore the motor may be
permitted to exceed its steady-state output power rating during the on-times. The simplified thermal
equivalent circuit model in Fig. 1.14 makes it possible to calculate the permissible overload factor as a
function of the on-time tON and duty-cycle d for a given motor.

Fig. 1.19 Simple thermal equivalent circuit for transient calculations

The thermal equivalent circuit is a parallel combination of thermal resistance R and thermal capacitance
C. R represents the steady-state thermal resistance between the winding and the surroundings in EC/W.
C represents the thermal capacity of the entire motor in J/EC. The thermal time-constant J is given [in
seconds] by eqn. (1.36):
J ' RC (1.91)
1. Sizing, gearing, cooling, materials and design Page 1.37

The analysis proceeds by equating the temperature rise during the on-time with the temperature fall
during the off-time. To do this we need the equations for the temperature rise and the temperature fall.
Temperature rise during ON-time. During the on-time tON, the power dissipation in the motor is Pd and
the temperature rises according to the equation

T ! T0 ' R Pd ( 1 ! e ! t /J ) ( Tc ! T0 ) e ! t /J . (1.92)
The temperature rise is expressed relative to the ambient temperature T0. The second term in eqn. (1.37)
is due to the initial condition in which the temperature rise is (Tc ! T0) at t = 0. At t = tON,

!tON /J !tON /J
Tmax ! T0 ' R Pd ( 1 ! e ) ( Tc ! T0 ) e . (1.93)

By definition, the steady-state rated temperature-rise (Tr ! T0) is given by


Tr ! T0 ' R Pr , (1.94)
where Pr is the rated steady-state power dissipation in the motor, i.e., the continuous power dissipation
that produces rated temperature rise. We can use this to "calibrate" Pd in eqns. (1.37) and (1.38), by
defining the dissipation overload factor k2, where
Pd
k2 ' . (1.95)
Pr
The reason for using k2 instead of k is that in most types of brushless servomotor the losses are dominated
by I2R losses while the load torque is proportional to the current I. If the load is increased by a factor k,
it means that the current and torque are increased by the factor k while the losses increase by k2. Thus
k is the overload factor for torque and current.
Substituting equations 1.39 and 1.40 in eqn. (1.38) and rearranging, and assuming that
Tmax ' Tr , (1.96)
we obtain the following equation relating the temperature rise to the overload factor and the on-time:
! tON /J ! tON /J
( Tr ! T0 ) [ 1 ! k 2 ( 1 ! e ) ] ' ( Tc ! T0 ) e (1.97)

Temperature fall during OFF-time. When the motor is switched off, the power dissipation falls to zero
and the winding temperature falls according to the equation

T ! T0 ' ( Tr ! T0 ) e ! t /J (1.98)

where t is measured from the end of the on-time, i.e. the beginning of the off-time. At tOFF,
!tOFF /J
Tc ! T0 ' ( Tr ! T0 ) e . (1.99)
!t /J
Steady-state : equating the temperature rise and fall. First, multiply eqn. (1.44) by e ON :

!tON /J !( tON tOFF ) /J


( Tc ! T0) e ' ( Tr ! T0) e . (1.100)

The left-hand side of eqn. (1.45) is identical to the right-hand side of eqn. (1.42), so the right-hand side of
eqn. (1.45) can be equated to the left-hand side of eqn. (1.42). With suitable rearrangement, the result can
be expressed in different ways, all of which are useful for different purposes.
Maximum overload factor. First, we get a solution for the dissipation overload factor k2 in terms of the
on-time and the duty-cycle: writing tON/d instead of tON + tOFF, i.e., instead of tcy, the expression is

!tON /J d
1 ! e
k2 ' (1.101)
!tON /J
1 ! e
Page 1.38 SPEED’s Electric Machines

For example, if the duty-cycle is 25% (d = 0.25) and tON = 0.2 × J, the dissipation overload factor is

1 ! e !0.2/0.25
k2 ' ' 3.04 , (1.102)
1 ! e !0.2

which means that the dissipation can be increased to 304% of its rated steady-state value for a period of
tON = 0.2J in every cycle of length tcy = tON/d = (0.2/0.25)J = 0.8J. If J = 40 min, the dissipation can be raised
to 304% for 8 minutes followed by a cool-down period of 24 minutes. Increasing the dissipation to 304%
corresponds to an increase in current and torque to %k = %3.04 = 1.74 times their rated values.
If tcy << J, then eqn. (1.46) simplifies so that
1
k2 ' . (1.103)
d

This means that when the on/off cycles are very short compared with the thermal time-constant of the
motor, the mean dissipation will be equal to Pr when the peak dissipation Pd = k2Pr is equal to Pr/d. This
simple result is intuitive.
Maximum overload for a single pulse. Eqn. (1.46) can also be used to calculate the maximum
dissipation overload factor for a single pulse, for which d = 0. In this case
1
k2 ' . (1.104)
!tON /J
1 ! e
For example, if tON = 8 min and J = 40 min, then the maximum dissipation overload factor k2 is 5.5 or 550%,
and the maximum overload factor k is 2.35 or 235%.
Required cool-down period for a given overload factor and on-time. The second result that arises
from equating the temperature rise and temperature fall is an expression for the necessary cool-down
time tOFF as a function of the dissipation factor k2 and the on-time tON. The expression is

tON /J
tOFF ' ! J ln [ k 2 ! ( k 2 ! 1 ) e ]. (1.105)

Together with eqn. (1.35), this can be used to determine the maximum duty-cycle d that can be used with
a given dissipation overload factor k2 and a given on-time tON, for a motor of thermal time-constant J. For
example, if the dissipation is 200% of rated, and if tON = 8 min, J = 40 min,

tOFF ' ! 40 × ln [ 2 ! (2 ! 1) × e 8/40 ] ' 10.0 min . (1.106)


The minimum cycle time is therefore 18 min and the maximum duty-cycle (with 8 minutes' on-time) is
8/18 = 0.44 or 44%.
Maximum on-time for a given overload factor and cool-down time. A third result obtained by
equating the temperature rise and fall is an expression for the maximum on-time tON as a function of the
dissipation overload factor k2 and the off-time tOFF. The expression is

!tOFF /J
k2 ! e (1.107)
tON ' J ln .
k2 ! 1

Maximum duration of single pulse. This expression can be used to calculate the maximum duration
of a single pulse having a given dissipation overload factor k2. For a single pulse, tOFF is infinite and

k2
tON ' J ln . (1.108)
k2 ! 1

For example, if k2 = 5.5 and J = 40 min, then tON = 8 min.


1. Sizing, gearing, cooling, materials and design Page 1.39

Graphical transient heating curves. Fig. 1.15


shows the relationship expressed by eqn. (1.53)
graphically in terms of the duty-cycle d, the on-
time tON as a fraction of the time-constant J, and
the overload factor k.
This graph can be used in a number of ways. For
example, to determine the maximum permissible
duration of a single pulse with a given overload
factor k, the duty-cycle d should be set to zero.
Thus with k = 1.5 the maximum pulse duration is
0.58J. With a time-constant of 40 min this is 23.2
min.
The graph shows the maximum duty-cycle that can
be used with a given overload factor. For example,
at 200% load the maximum duty-cycle is 0.25 or
25%, but in this limiting case the on-time must be
vanishingly small. With an on-time of 0.1J at 200%
load, the maximum duty-cycle is approximately
0.2, which means that the cool-down period in each
cycle must be (1!d)J = 0.9J. If J is 40 min, this
means a maximum operating time at 200% load of
4 min, followed by a cool-down period of 36 min
before the cycle can be repeated. Operations that
need a short on-time with a high duty-cycle must
use a lower overload factor.

Fig. 1.20 Intermittent heating curves

1.13 PERMANENT MAGNET MATERIALS AND CIRCUITS


The permanent-magnet industry has continually improved the properties of PM materials in the past
20!30 years, mainly by painstaking development of the metallurgy of existing materials. Samples of the
main families of PM materials used in electric machines are shown in Table 1.8.

Property Units Alnico 5-7 Ceramic Sm2Co17 NdFeB


Remanence Br T 1.35 0.41 1.06 1.2
Coercivity Hc kA/m 60 325 850 1000
3
Energy product (BH)max kJ/m 60 30 210 250
Relative recoil permeability μrec 1.9 1.1 1.03 1.1
Specific gravity 7.3 4.8 8.2 7.4
4
Resistivity μS-cm 47 >10 86 150
!6
Thermal expansion coefficient 10 /EC 11.3 13 9 3.4
Temperature coefficient of Br %/EC !0.02 !0.2 !0.025 !0.1
Saturation H kA/m 280 1120 > 3200 > 2400
TABLE 1.9
TYPICAL MAGNET PROPERTIES
Page 1.40 SPEED’s Electric Machines

Fig. 1.21 B-H loop of a hard PM material with electrical steel shown for comparison

The 'strength' of a magnet is sometimes measured by its 'energy product' (see below). At room temperature
NdFeB has the highest energy product of all commercially available magnets. The high remanence and
coercivity permit marked reductions in motor size, compared with motors using Ferrite (ceramic)
magnets. However, ceramic magnets are considerably cheaper than Rare Earth or NdFeB.
Both ceramic and NdFeB magnets are sensitive to temperature and special care must be taken if the
working temperature is above 100EC. For very high temperature applications Alnico or Rare Earth/Cobalt
magnets must be used, for example Sm2Co17 which is useable up to 200 EC or even 250 EC.
NdFeB is produced either by a mill-and-sinter process (Neomax) or by a melt-spin casting process similar
to that used for amorphous alloys (Magnequench). NdFeB magnets are often made in rings which may
be sintered or polymer bonded, but they can be formed in a wide variety of other shapes. They are not
100% dense and coatings or electroplating may be necessary to prevent corrosion.
B-H loop and demagnetization characteristics. The starting-point for understanding magnet
characteristics is the B-H loop or 'hysteresis loop', Fig. 1.16. The x-axis is the magnetizing force or
'magnetic field intensity' Hm in the material. The y-axis is the magnetic flux-density Bm in the material.
An unmagnetized sample has Bm = 0 and Hm = 0 and therefore starts out at the origin. If it is subjected to
a magnetic field, as for example in a magnetizing fixture, Bm and Hm in the magnet will follow the initial
magnetization curve as the external ampere-turns are increased. If the external ampere-turns are
switched off, the magnet relaxes along the curve shown by the arrows. Its operating point (Hm, Bm) will
depend on the shape of the magnet and the permeance of the surrounding 'magnetic circuit'. If the magnet
is surrounded by a highly permeable magnetic circuit, that is, if it is 'keepered', then its poles are
effectively shorted together so that Hm = 0 and the flux-density is then equal to the remanence Br. This is
the maximum flux-density that can be retained by the magnet at a specified temperature after being
magnetized to saturation.
External ampere-turns applied in the opposite direction (i.e., Hm < 0) cause the magnet's operating point
to follow the curve through the second and third quadrants until the magnet is saturated in the opposite
direction. Again, if the current is switched off the operating point returns towards the point (0,!Br), but
because of the demagnetizing effect of the external magnet circuit, Bm falls to a (negative) value smaller
than Br. It is now magnetized in the opposite direction and the maximum flux-density it can retain when
'keepered' is !Br.
1. Sizing, gearing, cooling, materials and design Page 1.41

To bring the flux-density to zero from the original positive remanence point (0,Br), the external
ampere-turns must provide within the magnet a negative magnetizing force !Hc, called the coercivity.
Likewise, to return the flux-density to zero from the negative remanence point (0,!Br), the field +Hc must
be applied. The entire loop is usually symmetrical and can be measured using instruments designed
specially for magnet testing.
If negative external ampere-turns are applied, starting from the positive remanence point (0,Br), and
switched off at R, the operating point of the magnet 'recoils' and will operate along the lower curve of a
'minor loop'. For practical purposes the minor loop of high-coercivity magnets is very narrow and can
be taken as a straight line, the recoil line, whose slope is equal to the recoil permeability, μrec. This is
usually quoted as a relative permeability, so that the actual slope is μrecμ0 H/m. Operation along the
recoil line is stable provided that the operating point does not go outside the original hysteresis loop.
A 'hard' PM material is one whose recoil lines are straight throughout all or most of the second quadrant,
which is where the magnet normally operates in service. In very hard magnets that are fully magnetized,
the recoil line is coincident with the second-quadrant section of the hysteresis loop. This is characteristic
of ceramic, Rare Earth/Cobalt, and NdFeB magnets, which usually have μrec between 1@0 and 1@1. 'Soft' PM
materials have a 'knee' in the second quadrant, such as Alnico. While Alnico magnets have very high
remanence and excellent mechanical and thermal properties, they have low coercivity and are therefore
limited in the demagnetizing field they can withstand.
Compared with 'electrical steel' used in laminations, even the 'soft' PM materials are very 'hard' : in other
words, the hysteresis loop of a typical nonoriented electrical steel is very narrow and has a low coercivity
and a high permeability; see Fig. 1.16. The high permeability is desirable in order to minimize the
magnetizing MMF (which is supplied by the magnets in PM motors, or by the magnetizing current in
induction motors). The narrow loop is desirable because the loop area represents an energy loss or
“hysteresis” loss which is dissipated every time the loop is traversed, and in AC motors (including
brushless PM motors) the loop is traversed at the fundamental frequency.
The most important part of the B-H loop is the second
quadrant, Fig. 1.17. This is called the demagnetization curve.
In the absence of externally applied ampere-turns, the
magnet operates at the intersection of the demagnetization
curve and the 'load line', whose slope is the product of μ0 and
the 'permeance coefficient' (PC) of the external circuit: i.e.,
at (Hm,Bm), with Hm < 0.
Since Bm and Hm in the magnet both vary according to the
external circuit permeance, it is natural to ask what it is
about the magnet that is 'permanent'. The relationship
between Bm and Hm can be written

Bm ' μ0 Hm J. (1.109)
Fig. 1.22 2 n d - q u a d r a n t d e m a g n e t i z a t i o n
The first term is the flux-density that would exist if the characteristic showing intrinsic curve
magnet were removed and the magnetizing force remained
at the value Hm. Therefore the second term can be regarded
as the contribution of the magnet to the flux-density within its own volume; accordingly, J is called the
magnetization and it is measured in tesla.10 If the demagnetization curve is straight, and if its relative
slope μrec = 1, then J is constant. This is shown in Fig. 1.17 for negative values of Hm up to the coercivity
!Hc. In most hard magnets μrec is slightly greater than 1 and there is a slight decrease of J as the negative
magnetizing force increases, but this is reversible down to the 'knee' of the B-H loop (which may be in
either the second or the third quadrant, depending on the material and its grade). Evidently the magnet
can recover or recoil back to its original flux-density as long as the magnetization is constant. The
coercive force required to demagnetize the magnet permanently is called the intrinsic coercivity and this
is shown as Hci.

10
Sometimes eqn. (1.54) is written Bm = μ0(Hm + M) and then the magnetization is measured in kA/m instead of T.
Page 1.42 SPEED’s Electric Machines

For engineering purposes we normally represent the recoil line by the equation
Bm ' μ0 μrec Hm Br (1.110)

which can be related to eqn. (1.54) by expanding it as follows:


Bm ' μ0 Hm μ0 ( 1 ! μrec ) Hm Br (1.111)
which indicates that
J ' μ0 ( 1 ! μrec ) Hm Br ' μ0 P Hm Br (1.112)

where P is the susceptibility, 1 ! μrec.


Another parameter often calculated is the magnet energy product, BmHm. This is not the actual stored
magnet energy but simply indicates how hard the magnet is working against the demagnetizing influence
of the external circuit. Contours of constant energy product are rectangular hyperbolas BmHm = constant,
often drawn on data sheets. The maximum energy product (BH)max occurs where the demagnetization
characteristic is tangent to the hyperbola of its (BH)max value. If the relative recoil permability is unity,
this occurs for a permeance coefficient of unity, with Bm = Br/2, provided that there are no externally
applied ampere-turns from windings or other magnets.

Fig. 1.23 Closed and gapped magnetic circuits

Calculation of Magnet operating point. Fig. 1.18 shows a simple magnetic circuit in which the magnet
is 'keepered' by a material or core of relative permeability μr. The core and magnet together form a closed
magnetic circuit. Applying Ampere's law, and assuming uniform magnetizing force in both the magnet
and the core,
H ml m HFelFe ' 0 . (1.113)

where Hm is the magnetic field in the magnet, HFe is the magnetic field in the iron core (assumed to be
uniform around the core length lFe, and lm is the length of the magnet in the direction of magnetization.
1. Sizing, gearing, cooling, materials and design Page 1.43

This is effectively the line integral of H around the magnetic circuit, and it is zero because there are no
externally applied ampere-turns. Hence
lFe
Hm ' ! HFe (1.114)
lm
which establishes that the magnet works in the second quadrant of the B-H loop. Now consider the gapped
magnetic circuit in Fig. 1.18, in which there is an airgap in series with the magnet and the two sections
of iron core. Now
H ml m HFelFe H gl g ' 0 . (1.115)

where Hg is the magnetic field in the airgap and lg is the airgap length. The permeability of the electrical
steel used in motors is usually several thousand times higher than μ0, so that the term HFe lFe can be
neglected as a first approximation, even though lFe may be much bigger than lg. Then
lg
Hm . ! Hg (1.116)
lm

Now by Gauss' law, the flux-densities in the magnet and the airgap are related by
B m A m ' B gA g (1.117)

so that if we take the ratio of Bm/μ0Hm, recognizing that in the airgap Bg = μ0Hg, we get
Bm Ag lm
' ! μ0 ' ! μ0 × PC (1.118)
Hm Am lg

where PC is the permeance coefficient. The ratio of magnet pole area to airgap area is sometimes called
the flux-concentration factor or flux-focussing factor:
Am
CM ' . (1.119)
Ag
In order to minimize the risk of demagnetization we need to operate the magnet fairly close to Br, i.e., with
a high permeance coefficient. On the other hand, the airgap flux-density Bg is increased if we use a high
value of the flux-concentration factor Am/Ag. But this reduces the permeance coefficient and eqn. (1.63)
shows that this reduces the ratio Bm/Hm, which increases the risk of demagnetization because it moves
the operating point further down the recoil line away from Br towards the knee of the B-H curve.
To achieve a high permeance coefficient with a high flux-concentration factor we must increase the ratio
lm/lg to compensate for the demagnetizing effect of the airgap: in other words, use a magnet that is long
in the direction of magnetization and also long relative to the airgap length. It does not mean long in
relation to the lateral dimensions of the magnet, and indeed most modern magnets except Alnico have
such high coercivity that the length in the direction of magnetization is the smallest dimension and is
intuitively referred to as the 'thickness'!
The energy product is given by
Bg Hg A g lg 2 Wg
Bm Hm ' ' (1.120)
Am lm Vm
where Wg is the magnetic energy stored in the airgap volume and Vm is the volume of the magnet. This
shows that the minimum magnet volume required to magnetize a given working volume of airspace is
inversely proportional to the working energy product BmHm . Therefore, in these cases it pays to design
the magnet length and pole area in such proportions relative to the length and area of the airspace, as to
cause the magnet to work at (BH)max, which is a property of the particular material at a given
Page 1.44 SPEED’s Electric Machines

temperature. In motors this principle cannot be applied so simply, because the armature current produces
demagnetizing ampere-turns that may be very great under fault conditions. To eliminate the risk of
demagnetization, motors are designed so that on open-circuit or no-load, the magnet operates at a high
permeance coefficient with an adequate margin of coercive force to resist the maximum demagnetizing
ampere-turns expected under load or fault conditions.
The lower diagrams in Fig. 1.18 illustrate the relative intensities of Bm and Hm under different working
conditions, in all cases with no externally applied ampere-turns. Note that B is continuous throughout
the magnetic circuit (because it obeys Gauss' law), but H is not. The discontinuities of H are associated
with the appearance of magnetic poles at the interfaces between different sections of the magnetic circuit,
notably at the 'poles' of the magnet and the working airspace. The polarization of surfaces gives rise to a
magnetic potential difference across the airspace which is useful for calculating flux distribution in
motors. In Fig. 1.18 this potential difference is
u ' Hg lg A&t (1.121)
If the magnetic potential drop in the steel is neglected, the corresponding magnetic potential difference
across the magnet is
!u . Hm lm A&t . (1.122)
C.g.s. units are still used in the magnet industry, but motors are designed in metric (SI) units in Europe
and Japan, and sometimes in English units in the U.S.A. Some conversion factors are as follows:

1 inch 25@4 mm
1 kg 0@4536 lb
8
1 Wb 10 lines (maxwells)
2 2
1T 10 kG 64@516 klines/in 1 Wb/m
1 kA/m 4B Oe 25@4 A/in
3
1 kJ/m B/25 MGOe
TABLE 1.10
CONVERSION FACTORS

Temperature effects; reversible and irreversible losses


High-temperature effects. Exposure to high temperatures for long periods can produce metallurgical
changes which may impair the ability of the material to be magnetized and may even render it
nonmagnetic. There is also a temperature, called the Curie temperature, at which all magnetization is
reduced to zero. After a magnet has been raised above the Curie temperature it can be remagnetized to
its prior condition provided that no metallurgical changes have taken place. The temperature at which
significant metallurgical changes begin is lower than the Curie temperature in the case of the Rare
Earth/Cobalt magnets, NdFeB, and Alnico; but in ceramic ferrite magnets it is the other way round.
Therefore ceramic magnets can be safely demagnetized by heating them just above the Curie point for a
short time. This is useful if it is required to demagnetize them for handling or finishing purposes. Table
1.10 shows these temperatures for some of the important magnets used in motors.

Metallurgical change EC Curie temperature EC Temp. coefft. of Br %/EC


Alnico 5 550 890 !0@02
Ceramic 1080 450 0@19
Sm2Co17 350 800 0@02
NdFeB 200 310 0@11
TABLE 1.11
METALLURGICAL CHANGE AND CURIE TEMPERATURE AND REVERSIBLE TEMPERATURE COEFFICIENTS OF BR
1. Sizing, gearing, cooling, materials and design Page 1.45

Reversible losses. The B-H loop changes shape with temperature. Over a limited range the changes are
reversible and approximately linear, so that temperature-coefficients for the remanence and coercivity
can be used. Table 1.10 gives some typical data. Ceramic magnets have a positive coefficient of Hc,
whereas the high-energy magnets lose coercivity as temperature increases. In ceramic magnets the knee
in the demagnetization curve moves down towards the third quadrant, and the permeance coefficient at
the knee decreases. Thus ceramic magnets become better able to resist demagnetization as the
temperature increases up to about 120EC. The greatest risk of demagnetization is at low temperatures
when the remanent flux-density is high and the coercivity is low; in a motor, this results in the highest
short-circuit current when the magnet is least able to resist the demagnetizing ampere-turns. In
high-energy magnets the knee moves the other way, often starting in the third quadrant at room
temperature and making its way well into the second quadrant at 150 EC. Grades with a high resistance
to temperature are more expensive, yet these are often the ones that should be used in motors,
particularly if high temperatures are possible (as they usually are under fault conditions).
All the magnets lose remanence as temperature increases. For a working temperature of 50EC above an
ambient of 20EC, for instance, a ceramic magnet will have lost about 10%. This is spontaneously
recovered as the temperature falls back to ambient.
Irreversible losses recoverable by remagnetization. (a) Domain relaxation. Immediately after
magnetization there is a very slow relaxation, starting with the least stable domains returning to a state
of lower potential energy. The relaxation rate depends on the operating point and is worse below (BH)max,
i.e. at low permeance coefficients. In modern high-coercivity magnets at normal temperatures this process
is usually negligible, particularly if the magnets have been stabilized (by temperature cycling and/or AC
flux reduction) immediately after magnetization. Elevated temperatures during subsequent operation
may, however, cause an increased relaxation rate. This can be prevented by temperature-cycling in the
final assembly over a temperature range slightly wider than the worst-case operating range. Subsequent
relaxation is reduced to negligible levels by this means. Table 1.11 shows the stability of different magnet
materials at 24 EC.

Material % loss after 10 years (typ.)

Ceramic < 0.01

Rare earth/Cobalt 0.2

Alnico 0.5
TABLE 1.12
LONG-TERM STABILITY OF MAGNET MATERIALS

(b) Operating point effect. Temperature alters the B-H loop. If this causes the operating point to 'fall off' the
lower end of a recoil line, there will be an irreversible flux loss. This is illustrated in Fig. 1.19. Initial
operation is at point a on the load line Oa, which is assumed to remain fixed. The remanence
corresponding to point a is at point A. When the temperature is raised from T1 to T2 the operating point
moves from a to b, and the corresponding remanence moves from A to B. Note that because the knee
of the curve has risen above point b, the effective remanence at B' is less than that at B, which is what
it would have been if the magnet had been working at a high permeance coefficient.
If the temperature is now reduced to T1 the operating point can recover only to a', which lies on the recoil
line through A'. The recovery from b to a' is reversible, but there has been an irreversible loss of flux-
density )Bm in the magnet, relative to point a. The remanence at T1 has fallen from A to A'. The loss can
be recovered only by remagnetization at the lower temperature.
Manufacturers' data for irreversible loss should be interpreted carefully to distinguish between the
long-term stability and the effects just described. Irreversible loss is usually quoted at a fixed permeance
coefficient. If the magnet is used at a lower permeance coefficient, the irreversible loss over the same
temperature range will be higher.
Page 1.46 SPEED’s Electric Machines

Fig. 1.24 Reversible and irreversbile loss caused by operating at a high temperature with a low permeance coefficient.

Mechanical properties, handling, and magnetization


Magnets are often brittle and prone to chipping, but proper handling procedures are straightforward
enough as long as the rules are followed. Modern high-energy magnets are usually shipped in the
magnetized condition, and care must be taken in handling to avoid injury that may be caused by trapping
fingers. A further hazard is that when two or more magnets are brought close together they may flip and
jump, with consequent risk to eyes. Table 1.12 summarizes some of the important safety precautions.
The best way to 'tame' magnetized magnets is to keeper them. Fixtures for inserting magnets can be
designed so that the magnets slide along between steel guides which are magnetically short-circuited
together. There still remains the problem of entering the magnets between the guides, but usually there
is enough space to provide for this to be done gently. Obviously it is important to keep magnets clear of
watches and electronic equipment that is sensitive to magnetic fields. Floppy disks, magnetic tapes, credit
cards and key cards are particularly vulnerable, and high-energy magnets can distort the image on
computer terminals and monitors.
Magnets are usually held in place by bonding or compression clips. In motors with magnets on the rotor,
adhesive bonding is adequate for low peripheral speeds and moderate temperatures, but for high speeds
a kevlar banding or stainless steel retaining shell can be used. In motors it is not advisable to make the
magnet an integral part of the structure. Mechanically, the magnet should be regarded as a 'passenger' for
which space and fixturing must be provided. The important requirements are that the magnet should not
move and that it should be protected from excessive temperatures.
1. Sizing, gearing, cooling, materials and design Page 1.47

Permanent magnets require strict adherence to safety procedures at all stages of handling and assembly.

Always wear safety glasses when handling magnets. This is particularly important when assembling
magnets into a motor. When a large pole magnet is being assembled from smaller magnets, the magnets
have a tendency to flip and jump unexpectedly and may fly a considerable distance.

Work behind a plexiglass screen when experimenting or assembling magnet assemblies. Watch out
for trapped fingers, especially with large magnets or high-energy magnets.

Avoid chipping by impact with hard materials, tools or other magnets.

Never dry-grind rare-earth magnets – the powder is combustible. In case of fire, use LP argon or nitrogen
dry chemical extinguishers – never use water or halogens.

Use suitable warning labels, especially on large machines. PM motors generate voltage when the shaft
is rotated, even when disconnected from all power supplies. This may be obvious to an engineer, but is a
potential safety hazard for electricians and maintenance personnel.

Never leave magnetized members open or unprotected. When assembling a rotor to a stator, with either
one magnetized, the rotor must be firmly guided and the stator firmly located.
TABLE 1.13
MAGNET SAFETY

A wide range of shapes is available, but in motors the most common are arcs and rectangles. Close
tolerances of +/!0.1mm can be held in the magnetized direction even for standard magnets. But if the
design permits a relaxation of the required tolerance, particularly in the dimensions perpendicular to the
magnetic axis, this should be exploited because it reduces the cost of the finished magnets.
Thermal expansion of magnets is usually different in the directions parallel and perpendicular to the
magnetic axis. The coefficients in Table 1.9 are along the direction of magnetization. Most magnets have
a high compressive strength but should never be used in tension or bending.
Magnetization of high-energy magnets requires such a high magnetizing force that special fixtures and
power supplies are essential, and this is one reason why high-energy magnets are usually magnetized
before shipping. The magnetizing force Hm must be raised at least to the saturation level shown in Table
1.8, and this normally requires ampere-turns beyond the steady-state thermal capability of copper coils.
Therefore pulse techniques are used, or in some cases superconducting coils. Ceramic and Alnico
magnets can sometimes be magnetized in situ in the final assembly, but this is impractical with
high-energy magnets.

Application of permanent magnets in motors


Permanent magnets provide a motor with life-long excitation. The only cost is the initial cost, which is
buried in the cost of the motor. It ranges from a few pennies for small ferrite motors, to several pounds
for rare-earth motors. Even so, the cost of magnets is typically only a small fraction of the total cost of the
motor. Broadly speaking, the primary determinants of magnet cost are the torque per unit volume of the
motor; the operating temperature range; and the severity of the operational duty.
Power density. We have seen that for maximum power density the product of the electric and magnetic
loadings must be as high as possible. The electric loading is limited not only by thermal factors, but also
by the demagnetizing effect on the magnet. A high electric loading necessitates a long magnet length in
the direction of magnetization, to prevent demagnetization. It also requires a high coercivity, and this
may lead to the more expensive grades of material (such as Sm2Co17, for example), especially if high
temperatures will be encountered. The magnetic loading, or airgap flux, is directly proportional to the
remanence, and is nearly proportional to the pole face area of the magnet. A high power density therefore
requires the largest possible magnet volume (length times pole area).
With ceramic magnets the limit on the magnet volume is often the geometrical limit on the volume of the
rotor itself, and the highest power densities cannot be obtained with these magnets. With rare-earth or
other high-energy magnets, the cost of the magnet may be the limiting factor.
Page 1.48 SPEED’s Electric Machines

The airgap flux-density of AC motors is limited by saturation of the stator teeth. Excessive saturation
absorbs too much excitation MMF (requiring a disproportionate increase in magnet volume); or causes
excessive heating due to core losses. For this reason there is an upper limit to the useable energy of a
permanent magnet. With a straight demagnetization characteristic throughout the second quadrant and
a recoil permeability of unity, the maximum energy-product (BH)max is given by

Br2
(BH)max ' J/m 3 . (1.123)
4 μ0

Assuming that the stator teeth saturate at 1.6T and that the tooth width is half the tooth pitch, the
maximum airgap flux-density cannot be much above 0.8T and is usually lower than this. Therefore there
will be little to gain from a magnet with a remanent flux-density above about 1 or 1.2 T, implying that the
3
highest useable energy product is about 300 kJ/m . At 100 EC, such characteristics are just within the
range of available high-energy magnets. Evidently it is just as important to develop magnet materials with
'moderate' properties and low cost, rather than to develop 'super magnets'
Operating temperature range. Because of the degradation in the remanence and coercivity with
temperature, the choice of material and the magnet volume must usually be determined with reference
to the highest operating temperature. Fortunately brushless PM motors have very low rotor losses. The
stator is easily cooled because of the fine slot structure and the proximity of the outside air. Consequently
the magnet can run fairly cool (often below 100 EC) and it is further protected by its own thermal mass and
that of the rest of the motor. The short-time thermal overload capability of the electronic controller would
normally be less than that of the motor, providing a further margin of protection against magnet
overtemperature.
Severity of operational duty. Magnets can be demagnetized by fault currents such as short-circuit
currents produced by inverter faults. In brushless motors with electronic control the problem is generally
limited by the protective measures taken in the inverter and the control. With an over-running load, or
where two motors are coupled to a single load, shorted turns or windings can be troublesome because of
drag torque and potential overheating of the stator. But by the same token, the dynamic braking is usually
excellent with a short-circuit applied to the motor terminals, and motors may well be designed to take
advantage of this. As is often the case, characteristics that are desirable for one application are
undesirable for another. The design must accommodate all the factors that stress the magnet:
electromagnetic, thermal, and mechanical.

1.14 PROPERTIES OF ELECTRICAL STEELS


Fig. 1.20 shows the DC B-H curve in the first quadrant for
two steels. The lower curve is a typical electrical motor
steel having 1.5% Silicon to increase the resistivity to limit
eddy-current losses. The saturation flux-density of such
steels (i.e. the flux-density at which the incremental
permeability becomes equal to μ0) is typically about 2.1T.
The upper curve is for a cobalt-iron alloy with a saturation
flux-density of about 2.3T. This material is much more
expensive than normal electrical steel, and is only used in
special applications such as highly rated aircraft
generators, where light weight and high power density are
at a premium.
The maximum permeability of electrical steels is of the
order of 5,000 μ0, and usually occurs between 1 and 1.5 T. In
Fig. 1.20, the total permeability of the electrical steel at 2.0T
is about 2.0/3,000 which is approximately 530 μ0.

Fig. 1.25 DC B-H curve for electrical steels


1. Sizing, gearing, cooling, materials and design Page 1.49

Losses. Under AC conditions, a power loss arises in


electrical steel as shown in Fig. 1.21, which indicates
increasing loss as the frequency and flux-density
increase. The loss is attributed to
(a) hysteresis;
(2) eddy-currents; and
(3) “anomalous loss”.

The hysteresis component is associated with the


changing magnitude and direction of the
magnetization of the domains, while the eddy-current
loss is generated by induced currents.
Eddy-currents can be inhibited by laminating the
steel, so that the eddy-currents become resistance
limited and the loss is then inversely proportional to
the resistivity. If the eddy-currents are resistance-
2
limited the loss is also proportional to 1/t , where t is Fig. 1.26 Variation of losses in electrical steel,
the lamination thickness. versus frequency and flux-density

At higher frequencies the resistance limited condition is lost, and the losses increase rapidly with
frequency. For this reason, very thin laminations, as thin as 0.1 mm, may be used at very high
frequencies (such as 400 Hz in aircraft generators or 3000 Hz in certain specialty machines). The
“anomalous loss” is associated with domain wall movement and is not often accounted for in empirical
expressions of the iron loss.
Characterization of core loss. Loss data from steel suppliers is usually obtained from measurements
in which a sinusoidal flux waveform is applied to a sample of laminations in the form of a stack of rings
or an “Epstein square” made up from strips interleaved at the corners. The loss may be characterized by
the so-called Steinmetz equation with separate terms for hysteresis and eddy-current loss:

P ' Ch f Bpkn Ce f 2 Bpk2 . (1.124)


The units of P are usually W/kg or W/lb. Bpk is the peak flux-density in T, and f is the frequency in Hz.
Ch is the hysteresis loss coefficient and Ce is the eddy-current loss coefficient. The exponent n is often
assumed to be 1.6!1.8, but it varies to a certain extent with Bpk. To a first approximation we can write n
= a + bBpk. With this modification,
a b Bpk
P ' Ch f Bpk Ce f 2 Bpk2 . (1.125)

The flux-density in motor laminations may be far from sinusoidal, and one approximate way to deal with
this is to modify the Steinmetz equation in the following way, recognizing that the eddy-current loss
component is expected to vary as the square of the EMF driving the eddy-currents, and that this EMF varies
in proportion to dB/dt. Thus
2
a b B pk dB
P ' Ch f Bpk Ce1 . (1.126)
dt

The hysteresis loss component is unchanged, but the eddy-current component is taken proportional to
the mean squared value of dB/dt over one cycle of the fundamental frequency. Eqn. (1.71) can be applied
in the respective sections of the magnetic circuit, after calculating the relevant flux-density waveforms.
The eddy-current loss coefficient Ce1 in the modified form can be derived from the sinewave coefficient
2
Ce if we assume that eqn. (1.71) holds with B = Bpk sin (2B f t). Then dB/dt = 2B f Bpk cos (2B f t) and (dB/dt)
2 2 2 2 2 2 2 2
= 4B f Bpk cos (2B ft), the mean value of which is [dB/dt] = 2B f Bpk . For sinewave flux-density,
equations 1.70 and 1.71 give the same result if
Ce
Ce1 ' . (1.127)
2 B2
Page 1.50 SPEED’s Electric Machines

Extracting the core loss coefficients from test data. Two procedures are used for extracting the
coefficients Ch, Ce1, a and b from sinewave loss data. The more elaborate of these requires a complete set
of curves of core loss vs. frequency at different flux-densities. When this data is not available, a simpler
procedure is used, based on five parameters.
Simple procedure—It is often the case that only a single value of P is available, for example, 8 W/kg at 50
Hz, measured with Bpk = 1.5 T. There is not enough data to determine the four loss coefficients uniquely,
so we use an estimate for n in eqn. (1.69); for example, n = 1.7. It is further necessary to estimate the split
between hysteresis and eddy-current loss. If h is the fraction of the total loss attributable to hysteresis,
then it can be shown that

P (1 h) hP
Ce ' and Ch ' . (1.128)
2
f Bpk2 Bpk n f
2
Then a ' n; b ' 0, and Ce1 ' Ce/2B .
Procedure used with complete set of core-loss data—The core loss data is usually in the form of graphs of
P vs. f at different flux-densities, or P vs. Bpk at different frequencies. The procedure is to try to separate
the hysteresis and eddy-current components of P. First we divide eqn. (1.70) by f :
P a b Bpk
' Ch Bpk Ce f Bpk2 . (1.129)
f

We then plot graphs of P/f vs. f for three values of Bpk, e.g. 1, 1.5 and 2T with f from 50 to the highest
frequency. The graphs should be straight lines and can be represented by
P
' D Ef. (1.130)
f

The intercept D on the vertical (P/f) axis must be equal to

a b B pk
D ' Ch Bpk . (1.131)

The intercepts D1, D2 and D3 for the three values of Bpk are substituted into the logarithm of eqn. (1.76),
giving three simultaneous linear algebraic equations for Ch, a and b of the form

log D1 ' log Ch (a b Bpk 1 ) log Bpk 1 . (1.132)

These are solved for log Ch, a and b; Ch is then obtained from log Ch. Next, three values of Ce are obtained
from the gradients of the three graphs of P/f vs. f , eqn. (1.74). The average or the highest value can be
2
taken for Ce. Finally Ce1 = Ce/2B . The loss curves may be re-plotted from the formula as a check. Any
extrapolation to higher Bpk or f should be checked carefully.
Note that Ce is approximately inversely proportional to t 2 , where t is the lamination thickness. This can
be used to modify Ce (or Ce1) for different thicknesses if test data is not available.
1. Sizing, gearing, cooling, materials and design Page 1.51

AC Magnetizing Volt-Amperes.
Magnetization curves are often supplied by steel B, H
manufacturers in the form of “AC” curves at 50 or 60 Hz.
Usually these curves plot the “VA/kg” or “VA/lb”
versus peak flux-densityB. The VA is the product of RMS
volts and RMS amps from the Epstein test. The RMS amps
represents the RMS value of the magnetic field strength
H.
h
NI
However, magnetic calculations require the
relationship between peak B and peak H, not RMS H.
In the Epstein test, the B waveform is maintained
sinusoidal, but the H waveform cannot be maintained
a
sinusoidal because of the magnetic nonlinearity. It a
becomes decidedly non-sinusoidal as a result of
saturation above about 1@7 T. Consequently it is Fig. 1.27 Magnetizing a block of laminated steel
incorrect to deduce peak H as /2 × RMS H.
The Epstein test standards (IEC, ANSI, BS etc) all recognize this problem, and they refer to the “form
factor” (peak/RMS ratio of the current waveform) as an indicator of the effect of saturation. They also
make provision for the measurement of peak H by means of a peak-reading ammeter incorporated in the
the Epstein test. Taken together, the peak B and peak H measurements describe the locus of the tips of
the hysteresis loops as the AC excitation is increased. When peak H is measured directly in the Epstein
test in this way, the required B/H curve is produced directly.
Unfortunately, however, the peak H data is often missing from manufacturers’ published data, and only
the VA/kg or VA/lb is available. In such cases it is tempting to convert VA/kg or VA/lb into H units by
means of the formulas derived below; but it must be emphasized that this is a poor substitute for the
measured peak H data.11
The magnetization data from the Epstein test usually stops at 1@7 or 1@8 T, but much higher flux-densities
may be encountered in motors (up to 3T), necessitating an extrapolation to higher flux-densities if
calculations are to be reliable (especially finite-element calculations). DC magnetization data obtained
from a magnetometer test usually goes up to a much higher flux-density than the VA/kg Epstein data,
typically to 2@1T or higher; above this level, it is relatively safe to extrapolate with a slope of 1/μ0. It is
generally assumed that the DC curve is a reasonable representation of the “peak B versus peak H” curve
that would be obtained under AC conditions. At high levels of B, the hysteresis loop merges into a single
curve that is essentially the same as the DC curve.
Derivation of conversion formulas. Fig. 1.27 shows a block of laminated steel, of cross-section a a and
height h. A solenoid coil wound tightly around the block carries a current I amperes in each of N turns.
The magnetizing force or "magnetic field strength" is in the vertical direction and is equal to

NI
H ' A/m (1.133)
h
The magnetizing force H sets up a flux-density (also called "induction") B, also in the vertical direction.
B is related to H by the equation
B ' μH (1.134)
2
where the coefficient μ is called the permeability. The units of B are [T] (tesla) — or [Wb/m ] (webers per
square metre), which is an older way of expressing the amount of flux passing through a given area. In
2 2
this case the area is a and the flux is a B [Wb]. The units of μ are [H/m] (henries/metre).

11
Steel datasheets often quote conversion factors from "VA/kg" or VA/lb to H — often without saying whether they mean peak
H or RMS H, and almost always with no details as to how the conversion factors are derived. the old-fashioned VA/kg and Watts/kg
were useful figures-of-merit in the days when motors were designed by slide-rule and the MMF required in the steel was simply
related to the VA/kg, the turns, and the weight of steel. Nowadays with numerically intensive software (especially FEA), the
traditional steel data is not really right for the job. Some companies measure the steel data themselves.
Page 1.52 SPEED’s Electric Machines

The permeability μ varies with H according to the shape of the magnetization curve (BH curve), which
is normally of the form shown in Fig. 1.25.
With AC current, I is the RMS current and it is related to the peak current by I ' Ipk//2, provided that the
current waveform is sinusoidal. The voltage induced at the terminals of the coil by the time-varying AC
flux is given by Faraday's law, and in AC terms it is
2Bf
V ' a 2 Bpk N [V rms] (1.135)
2
where f is the frequency in Hz. Eqn. (1.135) assumes that the B-waveform is sinusoidal, so that the voltage
waveform will also be sinusoidal.
The volt-amperes at the coil terminals is the product VI, also known as the apparent power. Using eqns.
(1.133) and (1.135), we get

2Bf Hpk h
VI ' a 2 Bpk N ' B f Bpk Hpk a 2h. (1.136)
2 N 2
2
But a h is the volume of the block of steel. Therefore the volt-amperes per cubic metre are

VI / m 3 ' B f Bpk Hpk (1.137)


3
If the steel has a mass density of D kg/m , the VA per kg is given by
B f Bpk Hpk
VI / kg ' . (1.138)
D
3
In this equation, Bpk is in [T], Hpk is in [A/m], f is in [Hz] and D is in [kg/m ]. As an example, suppose we
3
have Bpk = 1@7 T and the volt-amperes per kg is 154@1 at 60 Hz, with D = 7560 kg/m . Then from eqn. (1.138)
we get Hpk = 3,679 A/m (46@2 Oe).
Eqn. (1.138) can be used with other units. Suppose we want an equation that gives Hpk in oersteds [Oe]
3
when Bpk is in kilogauss [kG], the density D is in [g/cm ], and the volt-amperes is given in VA/lb. Given
3
that 1 Oe = 10 /4B A/m, 1 kG = 0@1 T, and 1 lb = 0@4536 kg, we get

D [g/cm 3] / 1000 [VA/lb]


10 6 0@4536 4B D [g/cm 3] [VA/lb]
Hpk ' ' 88@18 [Oe] . (1.139)
Bpk[kG] 1000 f Bpk[kG]
B f
10
3 2
Alternatively if we want Hpk in [At/in] when D is in [lb/in ] and Bpk is in [kl/in ], we have

D [lb/in 3] 0@4536 [VA/lb]


0@0254 3 0@4536 D [lb/in 3] [VA/lb]
Hpk ' 0@0254 ' 31823 [At/in] (1.140)
2
Bpk[kl/in ] f Bpk[kl/in 2]
B f
64@5
2
For example, suppose the volt-amperes per lb is 69@9 VA/lb and the peak induction is 110 kl/in , with steel
3
of density 0@276 lb/in . Then eqn. (1.140) gives
0@276 69@9
Hpk ' 31823 ' 93@0 [At / in] . (1.141)
60 110
1. Sizing, gearing, cooling, materials and design Page 1.53

1.15 EFFECTIVE BH CURVE FOR LAMINATION STACK


Fig. 1.28 shows a stack of laminations. The thickness of one
lamination is t, and the thickness of interlaminar insulation is c.
The stacking factor is defined as

t
F ' . (1.142)
t c

The standard Epstein test is designed to produce a curve of


polarization J vs. magnetic field strength H, but magnetization
curves are commonly supplied in terms of B vs. H, and it is slightly
unusual to find commercial data specifically in terms of J vs. H. Fig. 1.28 Thicknesses t and c

The relation between J and B is given by


B ' J μ0 H . (1.143)
Up to about 1@8 T, the difference between B and J is relatively small, but at higher flux-densities the
differences are significant. This is important because analytical design programs and finite-element
programs calculate up to much higher levels, well above 2 T.

2@5

[T] B
J
2@0

1@5

1@0

0@5

0
0 50 100 150 200
H [kA/m]
BJ.wpg

Fig. 1.29 Example of B and J plotted for an electrical steel

In laminated cores, a further complication arises as a result of the stacking factor, F. In Fig. 1.28, t is the
uncoated lamination thickness and c is the double-thickness of the insulation (that is, effectively, the
interlaminar space), and F is defined as

t
F ' . (1.144)
t c

In Fig. 1.28, J exists only in the steel, but H is the same in the steel and in the interlaminar space. Both
J and H are in the vertical direction in Fig. 1.28.
In one “lamination pitch” of width t + c, the total flux per unit of length measured normal to the plane of
the diagram is Jt + μ 0 H(t + c), so the effective or apparent flux-density is

Jt μ0 H ( t c)
BN ' ' FJ μ0 H . (1.145)
t c
Page 1.54 SPEED’s Electric Machines

2@5

[T] B

2@0

1@5

1@0

0@5

0
0 50 100 150 200
H [kA/m]
BJ.wpg

Fig. 1.30 B, BNN and J vs. H for the steel of Fig. 1.29; F = 0@96

It may be that the raw “Epstein” data is given as true B vs. H. In this case the apparent flux-density is

Jt μ0 H ( t c) Bt μ0 H c
BNN ' ' ' FB ( 1 £ F ) μ0 H . (1.146)
t c t c
An example of eqn. (1.146) is given in Fig. 1.30 for the steel of Fig. 1.29 with a stacking factor of F = 0@96.
Since 1 ! F is only 0@04, the apparent flux-density at any point on the curve is nearly FB which is only 4%
less than B. This may not seem significant, but the difference in H is very much greater along the upper
part of the curve. In the example in Fig. 1.30, the difference in H between the B and BNN curves is of the
order of 30 ! 40% at 1@8 T.
Of course, if the Epstein data represents polarization J vs. H (according to the standards), then curve BN
vs. H should be derived using eqn. (1.145). The J vs. H curve should flatten at high field intensities, but
this is rarely apparent in published data which is generally limited to 1@8 T, leaving the designer with the
difficult problem of obtaining satisfactory data up to the required level for design, and particularly for
finite-element analysis. At the very least, it helps if the steel supplier quotes the saturation magnetization
Jsat, which is usually slightly above 2 T.
A third method of accounting for the stacking factor is simply to shorten the stack by the factor F without
making any allowance for the flux in the interlaminar space. The result of this is that B is replaced by
the apparent flux-density BNNN:

Bt
BNNN ' ' F B. (1.147)
t c

Since (1 ! F) is very small (of the order of 0@03), this result is practically the same as eqn. (146).
1. Sizing, gearing, cooling, materials and design Page 1.55

1.16 SERIES AND PARALLEL INDUCTANCES

Fig. 1.31 Series inductance

In the analysis of electrical machines it is often necessary to consider inductances in series or in parallel,
or in more complex circuit configurations. In Fig. 1.31(a), two inductances L1 and L2 are in series with
a common current i ' i1 ' i2. The arrows show the directions of positive current and flux-linkage. For
each coil, positive current flows into the dotted end and produces a positive component of flux-linkage in
that coil: by definition, this component (the self flux-linkage) is always positive and in fact it defines the
direction of positive flux-linkage in that coil. The flux-linkages R1 and R2 also include mutual flux-
linkages and are given by
R1 ' L1 i1 M i2 ;
(1.148)
R2 ' L2 i2 M i1 .

so the total flux-linkage of the series combination is


R ' R1 R2 ' ( L1 L2 2M)i. (1.149)
The series inductance is the ratio of flux-linkage to current R/i : thus
Lseries ' L1 L2 2 M. (1.150)
M can be positive or negative in this equation, according to the relative direction of the flux produced by
coil 1 in coil 2, and vice versa. This depends not only on the physical arrangement of the coils, but also on
the connection. In Fig. 1.31(a) the connection is such that i1 ' i2 ' i. But in Fig. 1.31(b) the connections to
coil 2 have been reversed, so that i2 ' i. The argument leading to eqn. (1.150) now gives
Lseries ' L1 L2 2 M. (1.151)
This might represent two phases of a wye-connected motor winding connected in series, the positive
directions of i1 and i2 having already been defined with respect to the start and finish of each winding
before the connection is made. (See Fig. 1.34). In a balanced winding L1 ' L2 ' L3 ' L, and M12 ' M23 ' M31
' M. The resulting line-line inductance (with the third phase open-circuited) is LLL ' 2(L M). M is
usually negative because the winding axes are at 120Eelec relative to one another, so that if Lph is the
phase self-inductance and Mph is the absolute value of the mutual inductance between phases, we can
write LLL ' 2(Lph Mph). Care is necessary to incorporate the mutual flux-linkage with the correct sign.
In Fig. 1.32(a), the same two inductances L1 and L2 are in parallel with a common flux-linkage R and
respective currents i1 and i2, with i ' i1 i2. It must be the case that
R1 ' R2 ' R ' L1 i1 M i2 ' L2i2 M i1 , (1.152)
which amounts to a constraint on i1 and i2. Rearranging, we have
( L1 M ) i1 ' ( L2 M ) i2 . (1.153)
This can be substituted in the preceding equation for R, and also into the constraint i ' i1 i2, so that the
effective inductance R/i becomes
Page 1.56 SPEED’s Electric Machines

Fig. 1.32 Parallel inductances

L1 L2 M2
L ' . (1.154)
L1 L2 2M
If the connections to coil 2 are reversed, the effect is the same as if the sign of the mutual inductance were
reversed, giving
L1 L2 M 2
L ' . (1.155)
L1 L2 2 M
Note that if L1 ' L2 ' L, eqn. (154) simplifies to (L + M)/2, and eqn. (155) simplifies to (L M)/2, suggesting
the equivalent circuits in Fig. 1.33(c) and (d) with the mutual inductance absorbed into two separate self-
inductances. Likewise for the series connection, eqn. (150) simplifies to 2(L + M) and eqn. (151) to 2(L
M), suggesting the equivalent circuits in Fig. 1.33(a) and (b).

Fig. 1.33 Equivalent circuits when L1 ' L2 ' L

A bifilar winding can be regarded as a pair of identical parallel inductors, so tightly coupled that M is very
nearly equal to L. If the connection is “aiding” as in Fig. 1.32(a) or Fig. 1.33(c), the inductance becomes
(L M)/2 ' L, which is essentially the same as if two “strands in hand” were wound together in one coil.
On the other hand, if the connection between the two strands is “opposing”, as in Fig. 1.32(b) or Fig.
1.33(d), the inductance becomes (L M)/2 ' 0. This principle is used to make a “coil” of zero or very small
inductance: for example, in resistance-start split-phase motors some of the turns may be “bifilar wound”
1. Sizing, gearing, cooling, materials and design Page 1.57

in this way to increase the resistance of the auxiliary winding without changing its inductance, thus
providing an inexpensive way to change the resistance/reactance ratio to provide starting torque at
speeds near zero.
Another interesting property of the equivalent circuits in Fig. 1.33 is the change in inductance when two
inductors in series are reconnected in parallel. For example, if the series circuit of Fig. 1.33(a) is
reconnected as the parallel circuit of Fig. 1.33(c), the inductance changes from 2(L+M) to (L+M)/2, which
is the same change of 4:1 that would be obtained with plain resistors. The necessary conditions are that
the two self inductances be equal, and likewise the two mutual inductances; furthermore, the value and
sign of the mutual inductance must not be changed by the reconnection. In an electrical machine or
transformer, this will tend to be the case because the physical position of the coils is fixed and not altered
by reconnection. Moreover, the necessary equality of the respective self and mutual inductances will
usually be assured by symmetry; if this were not so, the parallel connection would be liable to circulating
current within the loop formed by the parallel inductors. In these circumstances the inductance can be
expressed as
Lseries
L ' (1.156)
a2
where Lseries is the inductance with all the turns in series and a is the number of parallel paths. If all the
turns are in series, a ' 1, but if there are two parallel circuits each with half the number of turns, a ' 2.
Provided that the conditions pertaining to Fig. 1.33 are satisfied, eqn. (156) can be generalized to any
number of parallel circuits.
Eqn. (156) is often the basis of the inductance calculation in electric machine analysis. If there are n coils
per phase, the inductance is much simpler to calculate if they are all in series than if they are in a parallel
paths each having n/a coils in series.

Inductance of wye and delta connections

Fig. 1.34 Wye connection; line-line inductance

Fig. 1.34 shows a wye-connected winding with equal self-inductances L in each phase, and equal mutual
inductances M between each pair of phases; that is to say, a balanced winding. As we have seen on p.55,
the line-line inductance is 2(L M), and if M < 0 we can write Mph ' |M| and Lph ' L to give
LLL ' 2 ( Lph Mph ) . (1.157)
Page 1.58 SPEED’s Electric Machines

Fig. 1.35 Delta connection : line-line inductance

Fig. 1.35 shows the line-line connection with a delta-connected balanced winding. This is slightly more
complicated than the wye connection because it forms a parallel circuit in which the two branches are
dissimilar. The equations will be written out, because it shows the method of analysis for the more
general case of an unbalanced winding, even though the result for a balanced winding is simple. First, the
individual phase flux-linkages are given by
R1 ' L i1 M (i2 i3)
R2 ' L i2 M (i3 i1) (1.158)
R3 ' L i3 M (i1 i2) .
The connection contrains these flux-linkages such that
R ' R1 ' (R2 R3) , (1.159)
while the currents are constrained by the relation
i2 ' i3
(1.160)
i ' i1 i3 ' i1 i2 .

If R1, R2, R3 and i1, i2 and i3 are eliminated from these equations, the result is
2
R ' (L M)i, (1.161)
3

which gives the required inductance as 2(L M)/3. This is 1/3 the value of the line-line inductance for
the wye connection, showing another instance where the mutual inductance is “eliminated” by
incorporating it in a modified self-inductance (in this case L M).

Inductance of more complex connections


The coils in an electrical machine are often grouped in series/parallel connections. Even within one
phase there may be several parallel paths, each having many coils in series. In most cases the disposition
of the coils is symmetrical so that there is no tendency for current to circulate within closed loops formed
by parallel paths that are connected together. In such cases eqn. (156) affords a valuable simplification
in the calculation of phase inductances. But in the general case a more powerful method of determining
the terminal inductance of a series/parallel combination is required.
Such a method is based on inductance matrices that relate vectors of current and flux-linkage together.
Fig. 1.36 shows a circuit containing six coils in three branches, each branch having 2 coils in series. The
“primitive” inductance matrix [L0] is an n×n array of the n unconnected self-inductances and 2n(n 1)
mutual inductances, where n ' 6 in this case. This array is shown in eqn. (162). In a system that is
magnetically linear, the mutual inductances are reciprocal, leaving n(n 1) ' 6 × 5 ' 30 independent mutual
inductances.
1. Sizing, gearing, cooling, materials and design Page 1.59

Fig. 1.36 Inductances connected in series/parallel. (a) Actual circuit with coils connected in parallel; (b) artificial circuit for
analysis; (c) final equivalent circuit .

1 2 3 4 5 6

1 L11 L12 L13 L14 L15 L16


2 L21 L22 L23 L24 L25 L26

L0 ' 3 L31 L32 L33 L34 L35 L36 (162)


4 L41 L42 L43 L44 L45 L46
5 L51 L52 L53 L54 L55 L56
6 L61 L62 L63 L64 L65 L66

The primitive inductance matrix [L0] includes the self and mutual inductances of all the coils, so it
represents all components of flux-linkage. It also defines the relationship between each flux-linkage
component and the current in each coil. If [R] is a column vector of the six coil flux-linkages and [i] is a
column vector of the six coil currents, we can express this as a matrix equation
[ R ] ' [ L0 ] @ [ i ] . (1.163)
Note that [R] and [i] are both matrices with 6 rows and 1 column, while [L0] has 6 rows and 6 columns. In
short-hand notation [R] and [i] are "6 × 1" while [L0] is "6 × 6". This short-hand notation is useful for
confirming that the matrices are "conformable" for multiplication: thus ["6 × 1"] ' ["6 × 6"] @[ "6 × 1"].
The primitive inductance matrix [L0] contains no information about the connection of the coils. A
convenient starting point is to identify series groups such as coils 1 & 2 or coils 3 & 4 in Fig. 1.36. If these
groups are numbered in a suitable order, the primitive inductance matrix [L0] can be partitioned to collect
all the coils in each series group together. The partitions already shown in eqn. (162) reflect the fact that
the circuit is connected in three branches, each containing 2 coils.
The series group connections make it possible to collapse the [L0] matrix into a 3 × 3 matrix by simply
adding the elements in each sub-matrix; for example, the self-inductance of the u branch becomes L11
L22 L12 L21, while the mutual inductance between the u branch and the v branch becomes Luv ' L13
L14 L23 L24. Similarly the mutual inductance Lvu is L31 L41 L32 L42. The resulting primitive matrix
for the 3-branch circuit is then
u v w

u Luu Luv Luw

Luvw ' v Lvu Lvv Lvw (165)


w Lwu Lwv Lww

and
Page 1.60 SPEED’s Electric Machines

[ Ruvw ] ' [ Luvw ] @ [ iuvw ] . (1.166)


Note that [R] and [i] also collapse from "6 × 1" to "3 × 1" because i1 ' i2 ' iu, i3 ' i4 ' iv, and i5 ' i6 ' iw, while
Ru ' R1 R2, Rv ' R3 R4, and Rw ' R5 R6. Evidently the collapsing of the [L0] matrix can be done
mathematically by adding together the appropriate elements without necessarily partitioning the matrix
first. At this stage the partitioning is nothing more than a visual convenience.
The [Luvw] matrix is still a “primitive” matrix because it does not embody the information that the three
branches u, v, w are connected in parallel; nor does eqn. (166). Unfortunately the effect of parallel
connections on the inductance matrix is not as simple as the effect of series connections. Parallel
connection imposes a constraint on the flux-linkages (effectively, on the voltages) of the branches that are
connected in parallel: for example in Fig. 1.36(b) , Ru ' Rv ' Rw. We therefore expect to collapse the 3 × 3
matrix into a single element or 1 × 1 matrix representing the effective terminal inductance when all three
branches are connected together in parallel, Fig. 1.36(c).
The method used here is to connect external “sources” that define a unique set of terminal flux-linkages.
The circuit of Fig. 1.36(b) achieves this in such a way that the parallel connection can be represented by
setting Rc ' 0 and then Rb ' 0 in successive steps, resulting in Fig. 1.36(c). The process is actually nothing
more than writing a suitable set of circuit equations that make it easy to impose the constraints Ru ' Rv
(i.e., Rc ' 0) and Rv ' Rw (i.e., Rb ' 0). But it will be expressed rather formally in order to develop an
algorithm that can be programmed and used for much more complicated cases.
The setting-up of the desired circuit equations is defined as a transformation of the primitive system of
eqn. (166) by means of a connection matrix [C]. The connection matrix transforms the currents iu, iv, iw
into currents ia, ib, ic. With the connection of Fig. 1.36(b), we have ic ' iu, ib ' ic iv ' iu iv, and ia ' ib
iw ' iu iv iw, which can be written
u v w

ia a 1 1 1 iu

ib ' b 1 1 0 @ iv (168)

ic c 1 0 0 iw
or
[ iabc ] ' [ C ] @ [ iuvw ] . (1.169)
Next we seek a transformation of the flux-linkages [Ruvw] that maintains the relationship
[ Rabc ] t @ [ iabc ] ' [ Ruvw ] t @ [ iuvw ] . (1.170)
If the R’s were voltages this would be known as the condition for “power invariance”. Let
[ Rabc ] ' [ D ] @ [ Ruvw ] . (1.171)
Substituting eqns. (169) and (171) in eqn. (170),
[ Ruvw ] t @ [ D ] t [ C ][ iuvw ] ' [ Ruvw ] t @ [ iuvw ] , (1.172)
which requires that
1
[ D ] ' [ C ]t ; (1.173)
i.e.,
u v w

Ra a 0 0 1 Ru

Rb ' b 0 1 1 @ Rv (174)

Rc c 1 1 0 Rw

This is the desired “connection matrix transformation” of the primitive flux-linkage vector [Ruvw] into
the form [Rabc] that corresponds to the connection in Fig. 1.36(b).
1. Sizing, gearing, cooling, materials and design Page 1.61

It now follows from eqns. (166), (169), (171) and (173) that
1 1 1
[ Rabc ] ' [ C ] t [ Ruvw ] ' [ C ] t [ Luvw ] @ [ iuvw ] ' [ C ] t [ Luvw ] @ [ C ] 1 [ iabc ] . (1.175)

This defines the [Labc] matrix that relates [Rabc] to [iabc] : thus
[ Rabc ] ' [ Labc ] @ [ iabc ] , (1.176)
where
1 1
[ Labc ] ' [ C ] t @ [ Luvw ] @ [ C ] ' [ D ] t @ [ Luvw ] @ [ D ] . (1.177)

This is the desired “connection matrix transformation” of the primitive inductance matrix [Luvw] into
the form [Labc] that corresponds to the connection in Fig. 1.36(b). It can be verified by multiplying these
matrices that
a b c

a Lww Lwv Lww Lwu Lwv

Labc ' b Lvw Lww Lvv Lvw Lww Lwv Lvu Lvv Lwv Lwu . (178)
c Luw Lvw Luv Luw Lvw Lvv Luu Luv Lvv Lvu

Provided that the mutual inductances are reciprocal, [Labc] is symmetric.


The final step is to eliminate the Rb and Rc equations from eqn. (176), by setting Rb ' 0 and Rc ' 0. This is
easily accomplished in two stages by a well-known process of reduction to the so-called “short-circuit
matrix”. Considering Rc first, let [Labc] be written more concisely and partitioned such that
a b c

Ra a Laa Lab Lac ia

Rb ' b Lba Lbb Lbc @ ib (179)

Rc c Lca Lcb Lcc ic

which can be written as a compound matrix equation


1 2

R1 1 P Q i1
' @ (180)

R2 2 S R i2
in which

R1 ' Ra i1 ' ia P ' Laa Lab Q ' Lac

Rb ib Lab Lbb Lbc

R2 ' Rc i2 ' ic S ' Lca Lcb R ' Lcc

Then if Rc ' R2 ' 0 it follows that

[ i2 ] ' [ i c ] ' [ R ] 1 @ [ S ] @ [ i1 ] , (1.181)

and if this is substituted in the equation for [R1] we get


[ R1 ] ' ( [ P ] [ Q ][ R ] 1 [ S ] ) [ i1 ] ' [ LN ] [ i1 ] . (1.182)
Page 1.62 SPEED’s Electric Machines

[LN] is a 2 × 2 matrix, i.e., its order is reduced by 1 as a result of the elimination of Rc. It is the “short-
circuit matrix” with respect to a short-circuit in place of Rc. It is now only a matter of repetition to reduce
[LN] to a 1 × 1 matrix [LNN] by eliminating Rb.
An especially simple case arises when Luu ' Lvv ' Lww ' L and Luv ' Lvu ' Lvw ' Lwv ' Lwu ' Luw ' M: then
L 2M
[ LNN ] ' . (1.183)
3
It is interesting to note that with Rc ' 0 the circuit in Fig. 1.36(b) resembles the common condition of a
three-phase motor fed from a power electronic inverter, in which two of the impressed line-line voltages
are defined and the third is zero as a result of the forward-biassing of a freewheel diode. If [LN] is replaced
by an operational impedance matrix, it can be used in the circuit analysis of such a system.
The analysis above can be repeated for a circuit with only two parallel branches (n ' 2). The matrix [Luvw]
is simplified by the omission of the w branch, so that ib ' iu and ia ' ib iv ' iu iv. Hence

[C] ' 1 1 and [D] ' 0 1


(184)
1 0 1 1
and

[Lab] ' [D] @ [Luv] @ [D]t ' Lvv Lvu Lvv


(185 )
Luv Lvv Luu Luv + Lvv Lvu

Using the same partitioning as before, the reduction in eqn. (182) can be applied again to give a single
equivalent inductance, and if we write L1 ' Lvv, L2 ' Luu, and M ' Luv ' Lvu, we get the familiar result for
two parallel circuits,
L1 @ L2 M2
L ' . (1.186)
L1 L2 2M
With simple examples the matrix method may seem a sledgehammer to crack a nut, but “simple” really
means no more than 2 or 3 coils. With larger numbers of coils, manual calculation is practically
impossible unless the coils are arranged in certain regular patterns that permit the use of mathematical
aids such as the summation of harmonic series. For example, the inductance of an integral-slot lap
winding can be computed using harmonic “distribution” factors without resorting to matrix methods.
But matrix methods are ultimately more general and can predict the effect of errors in the winding
connections, even in machines with large numbers of coils and multiple phases.
The connection matrix transformation is an example of a mapping that re-casts a problem in a form
suitable for the analysis of a particular problem. Other transformations continue the process into more
specialized analyses: for example, the symmetrical components transformation for the analysis of
unbalance in three-phase systems, or the dq transformation for the analysis of synchronous machines.
All these transformations have a law of transformation for the vectors of currents and flux-linkages, and
for the inductance matrix. The transformation matrix may itself be a function of time (or rotor position),
in which case the transformation of the voltages involves differential operators derived from Faraday’s
Law. Many remarkable books have been written on this subject, not only in electrical machines but also
in power systems analysis.
In power systems analysis it is common to construct the inductance matrix of the network in a form
suitable for analysis of faults, by a direct algorithm in which elements are added one by one according to
simple rules. The resulting matrix is often called the “bus impedance matrix” Zbus. Resistance is often
neglected in fault studies, so that Zbus is little more than jT times the inductance matrix when the network
is “cast” or drawn in a way that makes it easy to apply short-circuits between nodes, and to calculated the
resulting currents. This approach can also be used in electrical machines, as an alternative to the
connection-matrix approach. In electrical machines the primitive inductance matrix is a real
representation of the actual coils and it may be worth constructing it and displaying it just for the sake
of visualizing the patterns and linkages between coils.
1. Sizing, gearing, cooling, materials and design Page 1.63

Fig. 1.37 Numerical example

Numerical example — Fig. 1.37 shows a an example of six coils connected in three parallel paths, each
containing 2 coils in series. The only mutual inductances are between coils 1 & 3, 3 & 5, and 5 & 1. So the
primitive inductance matrix [L0] can be written down by inspection as follows:
1 2 3 4 5 6
1 1@5 1@0 1@0
2 0@5
L0 ' 3 1@0 1@5 1@0 (187)
4 0@5
5 1@0 1@0 1@5
6 0@5

Then u v w
u 2@0 1@0 1@0
Luvw ' v 1@0 2@0 1@0 (188)
w 1@0 1@0 2@0

For currents the connection matrix is the same as [C] in eqn. (169) because the circuit is the same as Fig.
1.36(a). Likewise the connection matrix [D] for flux-linkages is the same as in eqn. (173). Therefore using
eqn. (177) we can calculate [Labc] by matrix multiplication:

2@0 1@0 1@0 0 0 1 1@0 0@0 1@0


1@0 2@0 1@0 @ 0 1 1 = 1@0 1@0 1@0
1@0 1@0 2@0 1 1 0 2@0 1@0 0@0

0 0 1 1@0 0@0 1@0 2@0 1@0 0@0


0 1 1 @ 1@0 1@0 1@0 = 1@0 2@0 1@0
1 1 0 2@0 1@0 0@0 0@0 1@0 2@0

i.e.,
a b c
a 2@0 1@0 0@0
Labc ' b 1@0 2@0 1@0 (189)
c 0@0 1@0 2@0

By the elimination of Rc according to eqn. (182), we get

2@0 1@0 0@0 0@5 0@0 1@0 2@0 1@0


[LN] ' ' (190)
1@0 2@0 1@0 1@0 1@5

and finally by the elimination of Rb we get


1@0 4
[ LNN ] ' 2@0 × 1@0 ' . (1.191)
1@5 3
Page 1.64 SPEED’s Electric Machines

Fig. 1.38 A more complex example suitable for computation by means of the connection matrix method.

This result can be checked by paralleling any two branches, then paralleling the result with the third
branch. For the first two branches we have L1 ' L2 ' 2@0 and M ' 1@0, so the parallel combination has a
self inductance given by eqn. (186):

2@0 × 2@0 1@02 3


' . (1.192)
2@0 2@0 2 × 1@0 2
Its mutual inductance with the third branch remains 1@0, so the resulting inductance is again given by
eqn. (186) as
3
× 2@0 1@02
2 4
' , (1.193)
3 3
2@0 2 × 1@0
2
which checks the previous result. It will be found that eqn. (183) gives the same result.
A much more complex example is shown in Fig. 1.38. The winding in Fig. 1.38(a) has three phases, each
with four pole-groups, each containing three coils. Thus the total number of coils in Fig. 1.38(a) is 3 × 4
× 3 ' 36. The stator has a second three-phase winding (not shown in Fig. 1.38(a)), which may be co-phasal
with the first winding, or in anti-phase with it to give 6 phases. The total number of coils is therefore 72.
Fig. 1.38(b) shows the problem of calculating the loop inductance or line-line inductance when two of the
other phase terminals are shorted and the other two are open-circuited.
The order of the primitive inductance matrix is 72 × 72. This matrix has 5,184 non-zero elements, of which
72 are diagonal (self-inductance) elements, all equal, and (5,184 72)/2 ' 2,556 are reciprocal off-diagonal
(mutual-inductance) elements. It can immediately be partitioned into six separate phases, each having
an order of 12 × 12, but each phase sub-matrix can be partitioned into four further sub-matrices, one for
each pole-group. Within each pole-group sub-matrix there are 3 distinct mutual inductances. Within each
phase sub-matrix, a single coil has 11 distinct mutual inductances. By symmetry, three of these (e.g., the
ones to the coils of the 2nd pole-group) will be the negative of three others (i.e., those to the coils of the 3rd
pole-group), reducing the number of distinct absolute mutual inductances to 8. Together with the unique
self-inductance, there are only 9 distinct inductances to be calculated, when full account is taken of
symmetry. Each 12 × 12 phase sub-matrix can be very quickly calculated and collapsed to a 1 × 1 sub-
matrix or element of the resulting 6 × 6 matrix. For the normal operation of the machine, this is the final
result. The solution of the problem of Fig. 1.38 is then relatively straightforward using the matrix
methods described earlier for parallel connections. The overall simplification is represented graphically
in Fig. 1.39
1. Sizing, gearing, cooling, materials and design Page 1.65

1 2 3 4 5 6

Fig. 1.39 Simplification of the problem in Fig. 1.38. The dots show the nine distinct inductances from which all others can be
obtained by repetition (with appropriate sign changes where necessary).

Once the process has been programmed, it becomes easy to determine the result of asymmetries, winding
errors, and disturbances in the inductance values caused by end-connectors and other variations.

Synchronous inductance
Synchronous inductance is a fictional inductance that accounts for the inductive voltage-drop in each
phase of a synchronous machine with all three phases conducting balanced sinusoidal currents.12 In
phase 1, for example, it includes the mutual inductive voltage drops due to the currents in phase 2 and
phase 3, in addition to the self-inductance voltage drop due to current in phase 1. It thus "eliminates" the
mutual inductances from AC circuit calculations under balanced conditions, affording a useful
simplification. These calculations often employ the phasor diagram.
In certain types of synchronous machine, including the interior permanent-magnet motor and the wound-
field salient-pole synchronous machine, the phase inductances vary markedly with rotor position. Such
machines are normally analysed by the "dq-axis" method which uses two constant synchronous
inductances Ld and Lq instead of six time-varying self- and mutual inductances. (§2.6).

12
Synchronous inductance is not applicable to DC motors or universal motors, neither is it applicable to permanent-magnet
brushless motors that are supplied with nonsinusoidal currents.
Page 1.66 SPEED’s Electric Machines

Fig. 1.40 Measurement and interpretation of synchronous inductance

A simple way to visualize the effect of synchronous inductance or reactance is to run a synchronous
machine with all three phases shorted together. The EMF generated in the three phases drives a current
in each phase which is limited only by the resistance and inductances. The synchronous reactance can
be taken as the ratio of the open-circuit EMF to the short-circuit current measured with a three-phase
short circuit at rated speed.
For the loop containing E1 and E2 we have the "mesh voltage equation"
E1 E2 jTL (IA IC) jTL (IB IA)
jTM (IB IA) jTM (IA IC) (1.194)
jTM (IC IB) jTM (IC IB) ' 0.

But IA IC ' I1, IB IA ' I2, and IC IB ' I3; and with E12 ' E1 E2 we get

E12 jT(L M ) I1 jT(L M ) I2 ' 0 . (1.195)


jB/6 j2B/3
From Fig. 1 and Fig. 2, E12 ' /3 E1 e and I2 ' I1 e , so

3 E1 e j B / 6 jT(L M ) I1 [ 1 e j2B/3
] ' 0. (1.196)

The apparent impedance per phase is the ratio E1/I1, which is


E1 jT(L M)[1 e j2B/3
]
' ' jT(L M). (1.197)
I1 3e jB/6

If the phase resistance is negligible, this shows that synchronous inductance is equal to (L M). The 'j'
also establishes that I1 lags E1 by 90E, as shown in Fig. 3.
In classical synchronous machine theory, the self-inductance of a nonsalient-pole machine includes an
airgap component Lg0 (associated with airgap flux) and a leakage component LF (associated with slot-
leakage flux and end-winding flux). The mutual inductance includes an airgap component which for
sinewound machines with balanced winding is Lg0/2, and a leakage component MF. Consequently
3
L M ' Lg0 ( LF MF ) . (1.198)
2

This explains why the synchronous inductance is often thought to be 3/2 times the phase self-
inductance—though not exactly 3/2 times.
Strictly speaking, this note applies only to nonsalient-pole (i.e., surface-magnet) 3-phase machines, and
only to sinewound machines operating with sinusoidal current and voltage and negligible saturation.
1. Sizing, gearing, cooling, materials and design Page 1.67

1.17 MACHINE AND DRIVE DESIGN


The equation TRV . 2AB reflects the fact that torque is produced by the interaction between flux and
current. This simple equation is the cornerstone of electrical machine design. It leads to the more
advanced work of
machine design which is concerned with producing the torque with a minimum of material and
power loss; and
drive design which is concerned with the control of torque and speed, subject to constraints on
the electric and magnetic loadings.
In relation to the machine design, it is always important to minimize power losses and temperature rise
2
caused by I R heating of the conductors, core losses caused by hysteresis and eddy-currents in the
magnetic steel, and other losses. But there are many other aspects, such as the need to minimize torque
pulsations and acoustic noise, and to use materials economically. There is a huge variety of different
types of electrical machine, arising partly from constraints imposed by the available power supply. For
example, in automobiles the DC commutator motor is universally used because of the low-voltage DC
power supply. But in industry, AC induction motors are used primarily because of the availability of
polyphase AC power (which has a natural rotation between the phases), and because the induction motor
has no brushes and therefore requires very little maintenance. In traction applications (railways, transit
vehicles etc.), traditionally the DC motor was used because although AC supplies were available, the
control equipment was less expensive for DC drives. Since the 1980's, modern power electronics has
become so cost-effective that AC drives have steadily taken over from DC drives even in the most
demanding traction applications.
At the same time the variety in types and designs of electrical machines has greatly increased in many
other fields of application because of the advances made in power electronics and microelectronic control.
This is clearly evident in such products as tape drives, computers, office machinery, and so on; but there
are many others less well known—for example, the use of very high-speed brushless permanent-magnet
motors in machine tools. These motors can run at several tens of kW and several tens of thousands of
revolutions per minute.
In relation to the drive design, one of the fundamental aspects of electrical machines is the orientation of
the flux and the ampere-conductor distribution in relation to one another. The flux and MMF13 must be
orthogonal in space, i.e. the axes of their spatial distributions must be displaced by B/2p radians, if the
electromagnetic torque is to be maximized for a given flux and current. If the displacement angle is zero,
there is no torque and the power factor is zero. In DC commutator machines the orientation between the
flux and the armature MMF is maintained at B/2p radians by the action of the commutator, and therefore
if the machine is controlled by a chopper or phase-controlled rectifier the controller is not concerned with
orientation and need do no more than regulate the current. By contrast, in AC induction motors and
synchronous machines, the orientation is not guaranteed to be B/2p radians, even though the machines
might be designed to achieve this approximately under normal operation. For this reason, modern AC
drives employ field-oriented control, (also called vector control), to orient the MMF and the flux
orthogonally. This is quite complex and typically requires the use of microcontrollers or DSP’s (digital
signal processors). The most modern embodiments of field-oriented control are sophisticated enough to
include estimators for important parameters such as the flux, the direction of the flux, the rotor
temperature, and the electromagnetic torque itself.

13
The ampere-conductor distribution is often loosely termed the MMF (magneto-motive force), or MMF distribution.
Page 1.68 SPEED’s Electric Machines

1.18 COMPUTER-AIDED DESIGN


When new designs are evolved from old ones, computer-aided design is valuable for
1. calculating and evaluating a large number of options, often characterized by small changes in a
large number of parameters; and
2. performing detailed electromagnetic and mechanical analysis to permit the design to be
"stretched" to its limit. With accurate computer software, we can reduce the need for prototypes,
which are expensive and time-consuming.
Modern computer methods are rapidly reaching the stage where a new prototype can be designed with
such confidence that it will be "right first time", without the need for reiteration of design and test that
would otherwise be necessary. Computer-aided design goes hand-in-hand with the modern design
engineering environment. Custom designs are often required within a very short space of time, while cost
pressures force the designer ever closer to the limits of materials and design capabilities.
Moreover, customers are becoming more sophisticated in their requirements, and may specify (or ask
to see) particular parameters that traditionally were part of the "black art" of the motor builder. Often
these parameters are required for system simulation purposes long before the motor is actually
manufactured. Regulatory pressures on matters such as energy efficiency, acoustic noise, and EMC also
tighten the constraints on the motor designer.
No matter how effective the computer software available, it is always important to check the overall
parameters of a motor design using common sense and fundamental engineering principles. For this
reason it will always be necessary to be able to perform a single set of design calculations on the computer
and check the results against manual calculations. The next stage is to repeat design calculations,
modifying the dimensions and parameters until the performance objectives are attained. These processes
are illustrated graphically in Fig. 1.22. The SPEED software is designed to be used in this way.
The synthesis of a design by an optimization process is a much more complex undertaking beyond the
scope of this book. However, the development of scripting languages which can run programs such as
SPEED motor design programs automatically opens up new opportunities for user-defined design
automation procedures.

Fig. 1.41 Design loop

The wisdom and unwisdom of X-factors


There is often a temptation to “calibrate” calculations by means of arbitrary X-factors (“fudge factors”)
which may be derived from comparisons between calculation and measurement; or by comparing a
simple calculation against a more accurate one. In the latter case, the finite-element method is usually
assumed to be the “more accurate” method, often to such an extent that it acquires the false appearance
of absolute accuracy, inculcating in the unsuspecting engineer a childlike faith in its virtue. Man being
foolish, this faith is often hardened by the fact that finite-element calculations cost time and money. How
can the result be inaccurate if I paid a lot of money for the tool, and waited ages to get a result?
1. Sizing, gearing, cooling, materials and design Page 1.69

An X-factor can be defined as a “calibration factor” if there is a good technical basis for its value. But if
there is no such technical basis, it is nothing more than a lousy fudge factor.
A certain veteran user of SPEED software has his own special version with even more X-factors than the
standard version. Such people are more than mere engineers : they are artists, or gods. Do they know
something about motors that is hidden from the rest of us, or are they just more realistic about the
limitations of engineering calculations?
A problem with X-factors is that they generally adjust only one parameter at a time, even though the
whole model contains many parameters and relationships. If an X-factor is used to pull one parameter
into line (for example, to make it agree with a measured value), the chances are that it will make that
parameter inconsistent with the rest of the model, unless the whole model is recalculated in such a way
as to restore the self-consistency. This is generally impossible except in special cases.
X-factors create other problems, too. If the calculation method is improved or changed in any way, the
X-factors generally become invalid and may require to be determined all over again.
A computer program for calculating electrical machines must necessarily offer alternative methods for
calculating parameters and performance, for the simple reason that no one method is absolutely accurate
or has all the desired properties of speed, interpretability, and so on. Such a program requires continuous
development to exploit improvements in theory and algorithms, and to meet the demands of users who
invent variants which don’t match the original models.
Once such a program becomes embedded in a company’s toolset, the protection of calculated results
becomes vitally important. It is clearly unsatisfactory if new versions of the program deliver different
results on old motors, even if the calculation methods have been improved.
It is particularly unsatisfactory when such changes arise from the elimination of errors in the program,
especially when X-factors may have been used to obtain “satisfactory” agreement with test data from a
faulty version of the program. This last situation looks even uglier when expressed symbolically:
X-factor × Faulty program ' Correct result
X-factor × Corrected program ' Faulty result
Unchecked, the X-factor infects the new version with the disease of the old one! In certain cases the
labour and inconvenience involved in determining new X-factors to match an improved version of the
program is so great that the user may prefer to keep using the old version. Matters are made worse by
having to learn new parameters and methods, and maybe “unlearn” some old ones. The user finds
himself unable to upgrade his program, and suffers an injustice that arises from imperfections in the
previous version, which usually must be laid at the door of the program author.
One way to alleviate this injustice and to clear the way for the user to upgrade his program, is for new
versions of the program to provide switches that reset the calculations to the old methods, so that old
results can be reproduced using old data. (“Old” here does not mean “obsolete”: on the contrary we are
talking about results that must be preserved in a reproducible form, regardless of their age. What is
obsolete is the program version, not the data).
Ideally it would be handy for the new program version to calculate new X-factors automatically when
reading old data files, but in general this is impossible. If the program is written in such a way as to make
“old” calculations recoverable as far back as several previous versions, the source code fills up with
“legacy” material which is extremely difficult to maintain. When this happens, it can be against the
user’s interest for the program authors to try to keep maintaining it. Better to clear the decks, rejecting
intermediate versions and “early attempts”, and to strip the program down to a lean fighting machine
containing only the most up-to-date theory and algorithms with no excess baggage.
The “lean fighting machine” version will probably be welcome to most users, because it will be easier to
drive, more efficient, and inevitably more user-friendly. The preservation of old calculations is still
possible if the user keeps copies of earlier versions of the program, and carefully associates his datafiles
with a particular version. This much the user can do for himself, as many do. The program author can
help by maintaining archives of previous versions (both source and executable) so that old versions can
be re-released if necessary, and technical support provided.
Page 1.70 SPEED’s Electric Machines

1.19 THE NUMBER OF PHASES IN AN AC SYSTEM

(a) I (d) I (f) I

E E E

I I

(b) I E E

E I I

I E E

E I

(c ) (e) (g)
E E E
I I I

E E E
I I I

/2 I E E
I I

I=0 E
I

I=0

(h ) (i ) (j )
E E
/2 E
I /2 I
E E
/3 E
E
I
E
E

2E
Fig. 1.42 Phase relationships in 2-phase, 3-phase and 6-phase systems

The number of phases is a fundamental attribute of AC motors and generators, including those which
operate with electronic drives. In this section we consider the number of phases from a general electrical
engineering point of view, with an initial bias towards power distribution rather than any individual
machine. Later, in SEM-2, we will consider the number of phases in more detail in relation to the design
of brushless PM machines.
Fig. 1.43 shows a variety of circuits having different numbers of phases m. The simplest is (a), in which
m ' 1. Assuming a power-factor of 1, the power is equal to EI, where E is the RMS phase voltage and I is
the RMS phase current. Two conductors are required. If we define one unit of conductor area as that which
is required to carry the current I, then the total conductor area for circuit (a) is 2 units. These parameters
are summarized in Table 1.14.
1. Sizing, gearing, cooling, materials and design Page 1.71

Power Total conductor area No. of Max line-line


conductors voltage

(EI ' 1) (I ' 1) (E ' 1)

(a) 1 phase 1 2 2 1

(b) 2 phases 2 4 4 1

(c) 2-phase quadrature 2 2 + /2 ' 3@41 3 /2

(d) 3 phases 3 6 6 1

(e) 3-phase star 3 3 3 or 4 /3

(f) 4 phases 4 8 8 1

(g) 4-phase star 4 4 4 or 5 2


TABLE 1.14
COMPARISON OF 2-PHASE, 3-PHASE AND 4-PHASE CONNECTIONS
Circuit (b) has two phases, m ' 2, with no connection between them. Likewise circuit (d) has m ' 3 and
circuit (f) has m ' 4. All these circuits are simply m-multiples of circuit (a), so the total power, the total
conductor area, and the total number of conductors are all m times the value in circuit (a).
In circuit (c) a common return conductor is shared between the two phases of circuit (b). If the phases are
balanced and in phase quadrature as in Fig. 1.43(h), the current in the common conductor is /2I, requiring
41@4% more conductor area. The total conductor area is therefore (2 2 /2) ' 3@41 units, a reduction of
14@6% relative to Fig. 1.43(b), for the same power. Although the number of conductors is reduced from 4
to 3, the three conductors must differ in size if they are to have the same current-densities.
Circuit (e) is obtained by sharing a common return conductor between the three phases of circuit (d). If
the phases are balanced and separated in phase by 120E from one another, the phasor diagram of currents
forms a star similar to the phasor diagram of EMFs in Fig. 1.43(i), and the current in the return line is zero.
On this basis the return line can be left out, leaving a 3-wire connection. The total conductor area is only
3 units. Compared with circuit (d) the same power is conveyed with only half the total conductor area and
half the number of conductors.
Compared with the combined 2-phase circuit (c), the 3-phase 3-wire connection conveys 50% more power
with only 87@9% of the conductor.
The star connection can be extended to 4 or more phases, as shown in Figs. 1.43(f) and (g) for m ' 4.
However, no further reduction is possible in the number of conductors: for m > 2, the minimum number
of conductors is m.
Another consideration is the maximum voltage between any two lines, because this can present design
challenges and add to the cost of insulation. The combined circuits (c), (e) and (g) all have a maximum
line-line voltage greater than E, as can be seen in Table 1.14 and from the phasor diagrams in Fig. 1.43
(i) and (j). The 3-phase circuit has a maximum of /3E, while the 4-phase circuit has 2E.
The 3-phase 3-wire connection is notably more effective than the 2-phase circuit in terms of the total
conductor area per unit of power conveyed, and this puts the 2-phase quadrature system out of contention
in virtually all cases. There is also no advantage in having m ' 4, because although the conductor area
per unit of power conveyed is equal to that of the 3-phase 3-wire connection, the number of conductors is
33@3% higher and the maximum line-line voltage is 15@5% higher.
The 3-phase star connection shares its advantage with the 3-phase delta connection. Both of them require
only three conductors between the source and the load. The only difference between them is the
relationship between the phase voltages and currents, and the line-line voltages and currents:

STAR : EL ' 3 E; IL ' I


(1.199)
DELTA : EL ' E ; IL ' 3I
Page 1.72 SPEED’s Electric Machines

No. of phases in an AC electrical machine

Fig. 1.43 Circuits and phasor diagrams for considering the number of phases

In each of Figs. 1.42(a), (b) and (c) we see a succession of 18 phasors representing the EMFs in 18 coils
connected in series in a 2-pole 36-slot machine. This winding is shown in Fig. 1.44. The 18 coils constitute
half a 1-phase lap winding with a phase spread of 180E. It appears from Fig. 1.44 that each slot contains
only one coil-side. When the remaining 18 coils are added, there will be two coil-sides in each slot, and
this is described by saying that the winding has 2 layers. For now we will consider only one layer.

Fig. 1.44 18 coils of a 2-pole lap winding in 36 slots

Since all the coils are connected in series, their EMFs add. The result is the vector sum which is a
diameter of the circle, labelled E in Fig. 1.42. Note that this is considerably less than the arithmetic sum
of the phasor amplitudes of the individual coil EMFs, which is represented very nearly by half the
circumference of the circle. The ratio between the actual phase EMF E and the arithmetic sum is
approximately 2/B ' 0@637, and this is called the spread factor or distribution factor of the winding. It can
be said that only 63@7% of the total voltage-generating capacity of the winding is being utilized. Note that
the EMF in each coil is very nearly equal to BE/36.
Fig. 1.42(a) shows the effect of segregating the winding into two equal phases of 9 coils each. Each phase
EMF is now E//2. The utilization(spread) factor becomes (E//2)/(9 × BE/36) ' 0@900. We can say that the
total usable voltage now being taken from the machine is 2 × E//2 ' /2E, counting both phases; this total
is labelled in Fig. 1.42(a). It exceeds the total for the single-phase winding by /2, a huge increase.
By a similar argument, if the winding is divided into 3 phases as in Fig. 1.42(b), the winding utilization
increases to (E/2)/(6 × BE/36) ' 3/B ' 0@955. The three EMF phasors in Fig. 1.42(b) are equivalent to those
in Fig. 1.43(i), after the points of connection are re-arranged.
If we divide the 1-phase winding into 6 phases, as in Fig. 1.42(c), each phase EMF becomes 2 × E/2 sin 15E
' 0@2588E, and the winding utilization increases to (0@2588E)/(3 × BE/36) ' 0@9886, a further increase of 3@5%
over the 3-phase winding.
1. Sizing, gearing, cooling, materials and design Page 1.73

As in the case of the connections in Fig. 1.43, the 3-phase configuration confers most of the available
benefits of a multi-phase system, and in most cases the additional 3@5% obtainable from 6 phases would
not be justified in view of the increase in the number of terminals, leads, and conductors.
We can now return to consider the second half of the winding in Fig. 1.44. Physically it is constructed by
continuing the series of lap-wound coils from No. 19 to No. 36. The second half is identical to the first half
except that the polarity of the EMFs is reversed, as the concatenation of EMFs completes the circle. The two
halves of the winding provide two paths which can be connected either in series or in parallel.
The phasor diagrams in Figs. 1.43 and 1.42 are drawn on the assumption that the flux-distribution in the
airgap is sinusoidal. If it is not, they refer only to the fundamental of that distribution.
In later chapters we will see that most AC motors (including brushless permanent-magnet motors) have
1 or 3 phases, while 2-phase motors are only occasionally manufactured for special purposes. The single-
phase motor probably owes its existence largely to the fact that the electricity supply to domestic and
small industrial premises is usually single-phase, adjacent houses being connected to different phases to
spread the load evenly over all three phases connected to a particular substation.
In Chapter 2 the subject of "multi-phase" windings is discussed in detail, and there is an account of the
analysis of multiplex windings using dq-axis theory.
Page 1.74 SPEED’s Electric Machines

1.20 HALF-TURNS

Fig. 1.45 A circuit comprising coils with different numbers of turns

The question of "half turns" sometimes arises in electric machine design. The concept of a "half-turn"
challenges both Faraday's and Ampère's laws, which are respectively associated with the concepts of flux-
linkage and MMF :

R ' A @ dl and F ' H @ dl (1.200)

In both cases the integral is round an entire loop, which must be closed. In the process of counting turns,
it is impossible to count a fractional number of turns : the result is always an integer. A fractional turn
is not a complete circuit.
However, suppose we have a winding on two poles, as in Fig. 45. On the left-hand pole, there are 2 coils,
one with 5 complete turns and the other with 7. The right-hand pole has only one coil, with 5 turns. All
three coils link the same amount of flux M, so the total flux-linkage is R ' (5 7 5) M ' 17M Wb-turns or
volt-seconds. For the purpose of calculating the EMF dR/dt, it is common to work with the number of
turns per pole, and in this case with 2 poles in series the flux-linkage per pole is 17M/2 ' 8@5M Vs. So we
have the appearance of a fractional number of turns per pole, even though the number of turns in every
coil in the winding is clearly an integer.
Likewise it is common to work with the MMF per pole, and if the current I flows through all the coils in
series, the MMF per pole is 8@5I ampere-turns, giving the impression of a fractional number of turns.
Whenever we see a fractional number of turns, it must be expected to mean an average taken over more
than one coil; moreover, for the purpose of calculating EMF, all the relevant coils must link the same
amount of flux M, while for the purpose of calculating MMF, all the relevant coils must have an equivalent
disposition in the magnetic circuit, meaning that they must all be similarly positioned relative to the
nearest magnetic pole.

The position of the neutral


In polyphase machines another misconception can arise in relation to half-turns, depending on the
position of the neutral connection between the phases. Fig. 46 shows a simple 3-phase winding with 1 coil
per phase. The neutral point is connected at the front of the machine, that is, at the same end as the phase
terminals. This being so, each coil clearly has an integer number of turns wrapped completely around its
respective pole. A circuit from T1 to T2 is necessarily closed, since T1 and T2 are practically at the same
position, and the flux-linkage of this circuit is N1M1 N2M2.
1. Sizing, gearing, cooling, materials and design Page 1.75

Fig. 1.46 Three coils with "front neutral"

In Fig. 47, however, the neutral point is connected at the "back", remote from the terminal connections.
This makes the coils appear to be one-half turn short : coil 1 appears to have only N1 ½ turns, coil 2 N2
½, etc. The half-turns are misconceived, because they belong to incomplete circuits. In passing through
a complete circuit from terminal T1 to T2 pole 1 is encircled N1 times, not N1 ½ times; the final half-turn
being completed through coil 2. Likewise pole 2 is encircled N2 ½ times by the turns of coil 2, plus ½
turn completed through coil 1. If we neglect the stray flux in the region between pole 1 and pole 2, the total
flux-linkage in circuit T1 T2 remains equal to N1M1 N2M2, as in Fig. 46.
The stray flux between poles will tend to offset the potential of the neutral point, creating a slight
imbalance between phases. The degree of imbalance will be small — of the same order as that which is
caused by differences in the physical shapes of the coils and their positions in the slots. It will be
insignificant unless the number of turns per coil is very small, and even then it should be possible to
reduce it by connecting the neutral as a complete ring to minimize any asymmetry.

Fig. 1.47 Three coils with "back neutral"


Page 1.76 SPEED’s Electric Machines

1.21 SIGN CONVENTIONS


Figs. 1.48 and 1.49 show a single conductor fixed on the stator, and a magnet fixed on the rotor.
Sign conventions follow the mathematical ones: in the radial direction, outwards is positive; in the
circumferential direction, counter-clockwise (CCW) is positive; and in the axial direction, out of the paper
is positive. A dot indicates "out of the paper", while a cross indicates "into the paper".
LeFt-hand rule (Force on current-carrying conductor)
In Fig. 1.48, by the left-hand rule: First finger ' Field (produced by the magnet alone), seCond finger '
Current, and thuMb ' Motion, the force on the conductor is downwards to the right; the reaction force
on the magnet is upwards to the left, producing positive torque T in the CCW direction as shown.

Fig. 1.48 i > 0, T > 0; positive torque. Fig. 1.49 T > 0, e < 0

Right-hand rule (EMF geneRated in conductor moving in magnetic field)


In Fig. 1.49, by the right-hand rule, First finger ' Field (produced by the magnet alone), thuMb ' Motion
(of conductor relative to magnet), seCond finger ' induced Current (which gives the direction of the EMF).
Note that e < 0 in Fig. 1.48, i.e., it is negative. The right-hand rule treats e as a "generated EMF", which has
the opposite sign to that of a "back-EMF" or "inductive voltage drop".
The product ei is the instantaneous electrical power pelec[out]. In Figs. 1.48 and 1.49, pelec[out] is negative,
and this is interpreted as negative output power because e is regarded as a generated EMF and i is in the
direction of positive e. The instantaneous electrical input power is pelec[in] ' pelec[out], and this is positive.
Therefore the conditions shown in Figs. 1.48 and 1.49 correspond to motoring action.
The product TT is the instantaneous mechanical output power pmech[out], because T and T are in the same
direction. If there were no losses, pelec[in] ' pmech[out] and T ' ei/T. Note that if the EMF is proportional to
speed, e ' kE T, then T ' kE i, where kE is the EMF constant or torque constant kT ' kE.

Fig. 1.50 Armature reaction field of a single coil Fig. 1.51 Torque-producing armature current with magnet
1. Sizing, gearing, cooling, materials and design Page 1.77

Fig. 1.50 shows the direction of the magnetic field produced by a single armature coil with a plain airgap.
In Fig.1.51 two magnets are introduced into the airgap with polarities as shown. Using the result of Fig.
1.48, the torque is positive. We can deduce the direction of the torque from the polarity of the armature
reaction flux, which tends to make the stator surface a south pole, as indicated by the lower-case s. Since
like poles sS repel, and opposite poles sN attract, the rotor is pushed in the CCW direction.
The arrows show that the armature reaction field tends to strengthen the magnet flux at one edge of each
magnet, and to weaken it at the other edge. If the rotor is rotating in the forward (CCW) direction, the flux
is strengthened at the leading edges and weakened at the trailing edges (motoring operation).
With perfect symmetry the net flux through the magnet would be unchanged. But perfect symmetry is
not achieved in general, for the following reasons:
(1) The stator slots may not be symmetrically arranged about the magnet centre-line. With squarewave
drive, the current pattern remains fixed for typically 60E intervals of rotation. In sinewave motor
drives, the fundamental component rotates in synchronism with the rotor but space-harmonics in
the winding distribution may distort it.
(2) The stator teeth are liable to saturate more on the side with higher flux-density, that is, where the
magnet flux and armature reaction flux are in the same direction.
(3) If the demagnetization characteristic of the magnet is not linear, the magnet may suffer irreversible
loss of magnetization on the side which has the lowest flux-density.
Fig. 1.52 shows the armature reaction with the
current advanced 90E in phase relative to the
generated EMF. The conductors are in the same
position, with the same currents as in Figs. 1.50 and
1.51, and the phase advance is represented by a 90E
rotation of the rotor in the negative (CW) direction.
The armature-reaction field is demagnetizing across
the entire width of each magnet. We say that the
armature reaction is “in the negative d-axis”. In a
sinewave motor drive in the steady-state, this
relationship is fixed. The MMF axis is aligned with
the magnet centre-line (the d-axis), and the armature
MMF polarity ns opposes the magnet flux polarity NS.

In Fig. 1.52 the flux-linkage produced by the magnet


in the coil is at a negative maximum, since the
magnet flux is radially inwards and it is
symmetrically aligned with the MMF axis of the coil. Fig. 1.52 Armature reaction with the current advanced 90E
in phase relative to the generated EMF.
As the rotor moves forwards (CCW) the flux-linkage
R changes in the positive direction unti l it reaches
zero at the position shown in Fig. 1.51, which is 90 electrical degrees later than Fig. 1.52. The generated
EMF e ' dR/dt is zero in Fig. 1.52 and negative in Fig. 1.51. Also, Fig. 1.51 is consistent with Fig. 1.49 in
relation to the sign of the generated EMF.
If instead of the generated EMF e we consider the back-EMF e, and substitute e instead of e, then e is at a
positive maximum in Fig. 1.51. The back-EMF e ' dR/dt is convenient in motor theory, because it is has
the same sign as the applied voltage and has the same sign as an inductive voltage drop such as L di/dt.
The demagnetizing armature reaction in Fig. 1.52 is associated with a phase advance of the current
relative to the back-EMF. This is exploited in the technique known as flux-weakening, which permits
brushless PM motors to operate at high speeds even when the back-EMF exceeds the supply voltage.
Page 1.78 SPEED’s Electric Machines

REFERENCES

[1] Robinson RC, Rowe I and Donelan LE : The calculation of can losses in canned motors, Trans. AIEE,
June 1957, pp. 312-315.
[2] Russell RL and Norsworthy KH : Eddy-currents and wall losses in screened-rotor induction motors,
Proceedings IEE, April 1958, 105A, pp. 163-175
[3] Heller B and Hamata V: Harmonic field effects in induction machines, Elsevier Scientific Puvlishing
Company, Amsterdam 1977. (See pp. 55-56)
[4] Alger PL: Induction machines, Gordon and Breach Science Publishers, New York, 2nd edn., 1970. (See
p. 183).
[5] Hendershot JR and Miller TJE: Design of brushless permanent-magnet motors, Magna Physics
Publications/Oxford Science Publications, 1994, ISBN 1-881855-03-1/0-19-859389-9. (See pp. 4-30 - 4.32).
[6] Takahashi I et al:A super high speed PM motor drive system by a quasi-current source inverter, IEEE
Industry Applications Society Annual Meeting, Toronto, 1993, Conf. Rec. pp. 657-662.
[7] Bolton H: Transverse edge effect in sheet-rotor induction motors, Proceedings IEE, 116, No. 5, May 1969,
pp. 725-731.
[8] Arnold F and Floresta JG, Power Rate — A Most Important Figure-of-Merit for the Incremental Motion
Designer, Incremental Motion Control Systems Society, Proceedings of the 13th Annual Symposium,
Champaign, Illinois; Prof. B.C. Kuo, Editor, May 1984, pp. 11 18.
[9] Arnold F, Understanding Motion Control Figures-of-Merit, Incremental Motion Control Systems
Society, Proceedings of the 17th Annual Symposium, Champaign, Illinois; Prof. B.C. Kuo, Editor, June 1988,
pp. 1 12.
[10] Taft CK, Brushless DC Motor Figure of Merit Relationships, Incremental Motion Control Systems
Society, Proceedings of the 23rd Annual Symposium, Champaign, Illinois; Prof. B.C. Kuo, Editor, June
1994, pp. 179 199.
[11] Hanselman D, Figure of Merit; Motor Constant Indicates Brushless Motor Performance, PCIM,
December 1998, pp. 32 39.
[12] Fitzgerald AE and Kingsley C Jr., Electric Machinery, McGraw-Hill, 2nd Edition, 1961.
[13] Holman JP : Heat Transfer, McGraw-Hill Book Company, 1989.
1. Sizing, gearing, cooling, materials and design Page 1.79

Index

Acceleration 2, 4, 17-23 Epstein test 51


Adjustable speed 1, 7 Evolution of motors 7
Adjustment 68 Fan 1, 14-16, 29, 30
Airgap shear stress 14 Figures of merit 19
Apparent power 52 Finite-element method 68
Base speed 4 Finite-element 5, 6, 30, 53, 54, 68
Brushless DC 1, 7, 10, 11, 78 Flow process 1
Brushless PM 4, 10, 12, 36, 41, 48, 70, 77 Gearing 1, 1, 17, 18
B-H loop 40 Half turns 74
CAD 5, 6 Heat removal 25, 27, 29-31
Calibration 68 Hysteresis 12, 41, 49-51, 67
Cogging torque 24 Hysteresis loss 49
Commutator 3, 7, 9, 12, 16, 24, 67 Inductance
Computer-aided design 68 and parallel paths 57, 58
Conduction 28, 30-33 in series or parallel 55
Connection matrix transformation 60, 62 line-line 57
Convection 14, 27-33 of wye and delta connections 57
Cooling 1, 1, 9, 13-16, 25-31 synchronous 65
Copper losses 9, 10, 31 Inductance matrix 59
Core loss 49, 50 Induction motor 5, 7, 9-11, 24, 67
Current density 13, 15, 30, 33 Inertia 3, 4, 10, 11, 17-23
DC commutator motor 7, 9, 16, 24, 67 Insulation life 25
Demagnetization 11, 20, 25, 40-45, 47, 48, 77 Intermittent operation 27, 36
Design 1, 1, 3, 5, 6, 10, 16, 17, 19, 21, 23, 27, 43, 47, Irreversible losses 44, 45
48, 53, 54, 67, 68, 70, 71, 74, 78
Left-hand rule 76
Digital electronics 4
Line-line inductance 57
Drive system requirements 4
Magnet operating point 42
Duty-cycle 36-39
Magnet safety 47
Eddy-current loss 49
Magnetic loading 13, 14, 16, 47
Efficiency 2, 3, 5, 9-11, 16, 17, 26, 27, 68
Magnetic materials 5
Electric loading 13-16, 47
Magnetization 40-47, 49, 51-54, 77
Electrical steels 48
Mapping 62
EMF constant 76
Matrix method 59
Energy product 5, 39, 40, 42, 43, 48
Mechanical time constant 19
Energy saving 1
Microelectronics 1, 4, 5
Page 1.80 SPEED’s Electric Machines

Minas 10 Slotting
Motion control systems 1, 4, 5, 7, 19, 78 and cogging torque 24
Motor constant 19 Speed rate 19
New technology 1, 4 Stacking factor 53
Nonsalient-pole 24 Steinmetz 49
Numerical analysis 5 Steinmetz equation 49
Output equation 13 Stepper motors 11, 12, 16
Overload 4, 11, 36-39, 48 Structure of drive systems 3
Parallel paths 57 Switched reluctance 1, 7, 11, 12, 16, 24
Permanent magnet 12, 39, 48 Synchronous inductance 65
Permeance coefficient 42-45 Synchronous motors 9, 12
Polarization 53 Synchronous reluctance 11, 12, 24
Position control 1-3, 12 T/J ratio 19
Power 1-7, 9-14, 17, 19-21, 26, 27, 29, 30, 36, 37, Temperature rise 13, 16, 25-30, 35, 37, 38, 67
47-49, 52, 62, 67, 70, 71, 76, 78
Thermal equivalent circuit 31, 36
Power density 11, 47, 48
Torque 3-5, 7, 9-11, 13-25, 27, 30, 37, 38, 47, 48, 57,
Power factor 9-11, 27, 67 67, 76, 77
Power rate 19 Torque constant
Power semiconductors 1, 3, 7 and EMF constant 76
Radiation 27-33 Torque/inertia ratio 19
Radius of gyration 21 Transformation 62
Reluctance motors 1, 11, 12, 16, 25 connection matrix 60
Reversible losses 45 Transients 1, 2, 5, 30
Right-hand rule 76 TRV 13-17, 67
Saliency 24 VA/kg 51
Salient-pole 24 VA/lb 51
Sign conventions 76 X-factors
Sizing 1, 1, 13, 16 discussion of 68
Slip 7, 9, 10, 12
2. Brushless Permanent-Magnet Machines
2.1 What is a brushless machine? . . . . . . . . . . . . . . . . . . . . . . . . . . . . . . . . . . . . . . . . . . . . . . 2.1
2.2 Basic operation of the brushless DC motor . . . . . . . . . . . . . . . . . . . . . . . . . . . . . . . . . . . 2.3
2.3 Ideal torque/speed characteristic with square-wave drive . . . . . . . . . . . . . . . . . . . . 2.10
2.4 Magnetic circuit analysis . . . . . . . . . . . . . . . . . . . . . . . . . . . . . . . . . . . . . . . . . . . . . . . . . 2.11
2.5 Slotless motors . . . . . . . . . . . . . . . . . . . . . . . . . . . . . . . . . . . . . . . . . . . . . . . . . . . . . . . . . . 2.24
2.6 Current control with squarewave drive . . . . . . . . . . . . . . . . . . . . . . . . . . . . . . . . . . . . 2.26
2.7 Back-emf sensing with squarewave drive . . . . . . . . . . . . . . . . . . . . . . . . . . . . . . . . . . . 2.34
2.8 Unipolar drive circuits, and motors with one or two phases . . . . . . . . . . . . . . . . . . . 2.35
2.9 Windings . . . . . . . . . . . . . . . . . . . . . . . . . . . . . . . . . . . . . . . . . . . . . . . . . . . . . . . . . . . . . . . 2.39
2.10 Winding inductance . . . . . . . . . . . . . . . . . . . . . . . . . . . . . . . . . . . . . . . . . . . . . . . . . . . . . 2.41
2.11 Inductances of salient-pole motors . . . . . . . . . . . . . . . . . . . . . . . . . . . . . . . . . . . . . . . . . 2.56
2.12 Basics of sinewave operation . . . . . . . . . . . . . . . . . . . . . . . . . . . . . . . . . . . . . . . . . . . . . . 2.74
2.13 Synchronous operation of salient-pole PM motors . . . . . . . . . . . . . . . . . . . . . . . . . . . 2.78
2.14 Normalized form of the torque/speed characteristic . . . . . . . . . . . . . . . . . . . . . . . . . 2.87
2.15 History of brushless PM motor drives . . . . . . . . . . . . . . . . . . . . . . . . . . . . . . . . . . . . . . 2.91
2.16 Current control in the sinewave drive . . . . . . . . . . . . . . . . . . . . . . . . . . . . . . . . . . . . . . 2.93
2.17 PM generators . . . . . . . . . . . . . . . . . . . . . . . . . . . . . . . . . . . . . . . . . . . . . . . . . . . . . . . . . . 2.95
2.18 Line-start PM motors . . . . . . . . . . . . . . . . . . . . . . . . . . . . . . . . . . . . . . . . . . . . . . . . . . . . 2.104
2.19 Torque . . . . . . . . . . . . . . . . . . . . . . . . . . . . . . . . . . . . . . . . . . . . . . . . . . . . . . . . . . . . . . . . . 2.126
2.20 Multi-phase machines . . . . . . . . . . . . . . . . . . . . . . . . . . . . . . . . . . . . . . . . . . . . . . . . . . . . 2.135
2.21 Comparison of squarewave and sinewave configurations . . . . . . . . . . . . . . . . . . . . . 2.149
2.22 Permanent magnets versus electromagnetic excitation . . . . . . . . . . . . . . . . . . . . . . . 2.151
2.23 Cogging torque calculations using finite-element analysis . . . . . . . . . . . . . . . . . . . . 2.153
2.24 Performance simulation — the basis of Dynamic Design . . . . . . . . . . . . . . . . . . . . . 2.158
2.25 Some useful relationships in dq axes . . . . . . . . . . . . . . . . . . . . . . . . . . . . . . . . . . . . . . . 2.166
2.26 Motors with fractional slots/pole . . . . . . . . . . . . . . . . . . . . . . . . . . . . . . . . . . . . . . . . . . 2.167
2.27 Causes of rotor losses in PM brushless machines . . . . . . . . . . . . . . . . . . . . . . . . . . . . 2.172
2.28 Structure of Rotor Loss Calculations in PC-BDC . . . . . . . . . . . . . . . . . . . . . . . . . . . . . 2.176
2.29 Solution of the Complex Diffusion Equation . . . . . . . . . . . . . . . . . . . . . . . . . . . . . . . . 2.180
2.30 Evaluation of the Exciting Harmonic Current Sheets . . . . . . . . . . . . . . . . . . . . . . . . 2.196
2.31 Segmented Magnets and Finite-length Effects . . . . . . . . . . . . . . . . . . . . . . . . . . . . . . . 2.209
2.32 Slot ripple . . . . . . . . . . . . . . . . . . . . . . . . . . . . . . . . . . . . . . . . . . . . . . . . . . . . . . . . . . . . . . 2.220
2.33 “Flux-dip-sweeping” Analysis of Losses in Thin Sleeve . . . . . . . . . . . . . . . . . . . . . . 2.224
2.34 Harmonic Losses in the Interior Permanent Magnet Machine (IPM) . . . . . . . . . . . 2.227
2.35 Transients — Symmetrical Three-Phase Short-Circuit . . . . . . . . . . . . . . . . . . . . . . . 2.229
2.36 Transient Magnetic Field in the Magnet : the Fourier Transform Method . . . . . . . 2.239
2.37 Finite-element calculation of losses . . . . . . . . . . . . . . . . . . . . . . . . . . . . . . . . . . . . . . . . 2.240
SEM 2 — ii
2. BRUSHLESS PERMANENT-MAGNET MACHINES

2.1 WHAT IS A BRUSHLESS MACHINE?


"Brushless" electrical machines are those in which all components associated with sliding contacts are
eliminated. These components include brushes, commutators, slip-rings, etc. used to supply current
to the rotor. Although brushed motors are still widely used, their disadvantages have encouraged the
development of brushless machines: for example,
(1) in computer disk drives because of the undesirability of brush debris;
(2) in blowers and fans because of the need for low noise, high efficiency, and speed control;
(3) in precision servomotors for factory automation, where downtime must be minimized; and
(4) in vehicle traction, because of the cost of brushgear maintenance and the need for high
efficiency.
The development of brushless machines would not have been possible without certain enabling
technologies : in particular, power transistors, microelectronic controls and sensors, and permanent-
magnet materials. These components are now so highly developed that they have opened up many new
opportunities for electric motor applications, especially where variable speed is required.
There are so many varieties of brushless electrical machine that it is difficult to classify them, but we
can recognize four main categories:
(a) Brushless DC Machines are derived directly from the classical DC machine by replacing the
commutator and brushes with an electronic power supply. The motor is often designed to have
a trapezoidal back-EMF waveform and the current waveforms are rectangular, with alternating
polarity. The current polarity is switched in synchronism with the rotor position, by means of
power semiconductors which are also used to regulate the current. Fig. 2.1.1 shows a typical
example. Permanent magnets (mounted on the rotor surface) are generally used for excitation.
(b) AC brushless machines are similar to brushless DC machines, but the back-EMF waveform is
designed to be sinusoidal and the current waveform is also controlled to be sinusoidal. They are
used in servosystems where smooth torque control is required. Resolvers are often used for
shaft position feedback. In some cases the permanent magnets are mounted inside the rotor,
as shown in Fig. 2.1.3. This tends to introduce saliency and a reluctance torque component.1
The saliency can also be helpful when the motor is to operate at constant power over a wide
speed range—as, for example, in drives for electric vehicles. It is even possible to dispense with
the magnets altogether, as in the synchronous reluctance motor, Fig. 2.1.4.
(c) Self-synchronous AC brushless machines in which the DC field winding on the rotor is fed
from a rotating rectifier, which in turn is fed from a rotating AC exciter mounted on the same
shaft. This category includes some large machines (several MW) and high-speed AC machines
such as aircraft generators. Fig. 2.1.5 shows an example (not including the rectifier or exciter).
(d) Specialty brushless machines not derived from classical DC or AC machines. This category
covers a huge range of different designs, including a wide variety of blower motors (Fig. 2.1.2),
timing motors, and others. Often the manufacturing volumes of these motors are in millions.
In this introductory theory section we will concentrate on the classical three-phase brushless DC
machine, Fig. 2.1.1. The design principles of machines in category (c) are more in common with those
of classical synchronous machines. For machines in category (b), the design principles are in common
with both categories (a) and (c).
Other important types of "brushless" electric machine include the induction motor, the switched
reluctance motor, and the stepper motor.

1
“Salient” means “sticking out” and refers to the projecting poles on wound-rotor synchronous machines. The term “salient-pole”
is now used more generally to refer to machines with different permeances in the direct and quadrature axes of the rotor, even
when there are no physical salient poles. The difference in d-axis and q-axis permeance leads to a difference in the synchronous
inductances Ld and Lq, and gives rise to reluctance torque.
Page 2.2 SPEED’s Electric Machines

1. Brushless permanent-magnet motor 2. Single-phase brushless permanent-magnet motor

This nonsalient-pole motor can be either This exterior-rotor motor has a tapered airgap to
"brushless DC" or "brushless AC". ensure self-starting from any rotor position

3. Interior-permanent-magnet brushless AC motor 4. Synchronous reluctance motor

5. Salient-pole wound-field synchronous AC motor


Fig. 2.1 Types of brushless motor.
Brushless permanent-magnet machines Page 2.3

2.2 BASIC OPERATION OF THE BRUSHLESS DC MOTOR


Fig. 2.2 shows a 2-pole motor with a magnet rotating
counter-clockwise at the instant when the flux-linkage
with coil 1 is at a negative maximum. The magnet is
shown with an arc $M = 180Eelec., and the coil-pitch is also
180Eelec, so that the flux-linkage R1 of each coil varies with
a triangular waveform as the rotor rotates: see Fig. 2.4.
By Faraday’s Law the EMF induced in coil 1 is
dR1 dR1
e1 ' ' Tm (2.1)
dt d2
where Tm is the angular velocity in mechanical rad/s and
2 is the rotor position in mechanical radians. As long as
the flux-linkage is varying linearly with rotor position, the
induced EMF is constant. When the flux-linkage reaches a
Fig. 2.2 Generation of back-EMF
maximum, it starts to decrease at the same absolute rate,
and the EMF changes polarity. The result is a squarewave
generated EMF, e1.
A second coil is displaced by a certain angle ( from the first coil. It has the same number of turns as the
first coil, Tc. Its EMF waveform is identical to that of the first coil, but retarded in phase by ( electrical
radians. If the two coils are connected in series, their EMFs add, giving an EMF waveform with twice the
peak value of the individual coil EMF. If each phase is made up of just two coils connected in series, this
is the phase EMF. Its waveform is stepped because of the phase displacement between coils 1 and 2. In
practice, because of fringing of the flux at the edges of the magnet, the edges of the EMF waveform are
not sharply stepped but appear smoothed, as shown by the dotted line in Fig. 2.4 . This is the so-called
trapezoidal back-EMF waveform that is characteristic of brushless DC motors.
A 3-phase brushless DC motor is generally designed so that the flat top of the phase back-EMF waveform
is just over 120E wide. Then each phase is supplied with a current waveform consisting of blocks of
constant current 120E wide. During each 120E period, the electromagnetic power conversion is e1i1 =
TeTm, where Te is the electromagnetic torque. If the EMF and current waveforms are sufficiently flat
during this period, and the speed is essentially constant, Te is also constant.
The EMF can be calculated from the airgap flux distribution. If Bg is the average flux-density over one
pole-pitch, the airgap flux Mg is given by

B /p BD Lstk
Mg ' m B(2) r d2 Lstk ' Bg × (2.2)
0 2p

where r is the stator bore radius and D (= 2r) is its diameter; Lstk is the stack length, and p the number
of pole-pairs. The peak flux-linkage of coil 1 is R1 max = Tc Mg, and if the linear variation of R1 with 2 is
p2
R1(2) ' R1 max (2.3)
B/2

then by eqns. (2.1) and (2.3) the peak coil EMF is


2p
e 1 ' Tm × × Tc Mg (2.4)
B

For a machine with Tph turns in series per phase, the peak EMF/phase can be written
2p
e1 (pk) ' Tm × × Tph Mg . (2.5)
B
Page 2.4 SPEED’s Electric Machines

Fig. 2.3 shows the commonly used bridge circuit for a 3-phase brushless permanent-magnet motor. For
“squarewave” operation (Fig. 2.4), it has two phases conducting at any time. If the motor is wye-
connected they carry the same current I in series, and the line-line EMF during each 60E interval is
eLL (pk) ' 2 e1 (pk) ' kE Tm (2.6)
where kE is the back-EMF constant in Vs/rad:
4 p Tph Mg
kE ' . (2.7)
B
The electromagnetic power is eLL(pk)I and the electromagnetic torque is
eLL (pk) I
Te ' ' kT I (2.8)
Tm
where k T = kE is the torque constant in Nm/A. When driven this way with "two phases on", the motor
behaves very much like a permanent-magnet DC commutator motor. The torque is produced in blocks
60E wide, and there are 6 such blocks every electrical cycle. See Fig. 2.7.

A closer look at the torque production


Eqn. (2.8) is an example of an ideal electromechanical device that converts instantaneous electric power
into instantaneous mechanical power, without loss or storage. On p. 74 the same idea is used to develop
a torque equation for sinewave brushless motors. In general, for a nonsalient-pole brushless PM
machine with m phases, the electromagnetic power is a result of the interaction between all the phase
currents i1, i2,...im and the corresponding EMF’S e1, e2,...em generated by the rotation of the magnets. Thus

T e T m ' e 1 i1 e 2 i2 e 3 i3 ... em im (2.9)

In the “squarewave” operation described above, with “two phases on” at any time, we have a 60E
interval in Fig. 2.4 when i3 ' 0, while i1 ' I ' i2, so that

T e Tm ' e 1 I e2 I ' (e1 e2) I ' eLL I , (2.10)

where eLL is the line-line EMF between phases 1 and 2. At the end of the 60E interval, the current I is
commutated so that in the next interval i1 ' I ' i3; and in the next interval after that, i2 ' I ' i3, etc.
Te is, of course, the instantaneous torque and not the average torque. The average electromagnetic
torque (averaged over one or more revolutions) is,
1 2B
Tavg ' Te ( 2 ) d2 (2.11)
2B 0

In order to keep the torque constant, it is not sufficient to commutate the current: the eLL I product must
be constant throughout each conduction interval. In general this requires a flat-topped line-line EMF
and constant current. The flat EMF and flat current persist through only a limited angle of rotation,
both waveforms being basically AC waveforms with alternating positive and negative half-cycles. The
flat segments can be truly flat only if these waveforms contain infinite numbers of harmonics, and it
can be said that all the harmonics contribute to the torque. (See p. 131). In this respect the “squarewave”
motor differs from the “sinewave” motor, which relies on a single harmonic of EMF and current, the
fundamental. (See also p. 149).
The angle of rotation over which each of the product terms eLL I can be kept constant is limited by the
magnet arc and the winding distribution. Although the current can be held constant for nearly 180E of
rotation, it is not possible to maintain a flat-topped phase EMF waveform wider than about 175E. For this
reason it is impossible to achieve constant torque in a 2-phase brushless motor, and it is better to settle
for three phases and 60E flat tops in eLL, which can be adjusted to minimize any ripple arising at the
commutation points. The width of the flat top requires careful choice of the slot/pole combination, the
magnet arc, and the winding distribution.
Brushless permanent-magnet machines Page 2.5

Even with a flat-topped trapezoidal EMF waveform, the current waveform may depart from the ideal 120E
squarewave because of chopping (PWM) and commutation in the drive. Moreover, at high speeds the
ideal 120E squarewave current waveform cannot be achieved because of the combination of series
inductance and the growth in the EMF relative to the available supply voltage. All of these departures
from the ideal case result in torque ripple. Similar considerations apply to sinewave motors.
Two main sources of torque ripple can be identified in brushless permanent-magnet motors:
(1) electromagnetic torque ripple, arising from imperfections in the e and i waveforms;
(2) cogging torque, which is caused by interaction between the magnets and the stator slotting.
Many applications are extremely sensitive to torque ripple, such as automotive power steering and
machine tool feed drives. Power electronics has made it possible to use motors which would not
inherently deliver smooth torque if they were operated from classical DC or AC sources, and at the
same time it is clear that commutation and PWM can exacerbate the torque ripple.

The squarewave 3-phase drive


The electronic power supply is called the drive. In low-power drives, MOSFETS are popular because they
are easy to control, and they can be switched at high frequency. This makes it possible to regulate the
current by chopping with low acoustic noise. MOSFETS are ideal for low-voltage drives because of their
low on-state voltage drop. At higher powers and voltages, IGBTS are used.

Fig. 2.3 Drive circuit for wye-connected brushless DC motor

Current and EMF waveforms are shown in Figs. 2.4 and 2.7 for the wye-connected motor, and in Fig. 2.8
for the delta-connected motor, whose drive circuit is shown in Fig. 2.5. The line current waveforms are
identical for the two connections, as is the commutation table for the transistors, Table 2.1.
Fig. 2.6 shows the interaction of the "phasebelts" of ampere-conductors with the magnet arcs in wye- and
delta-connected motors. In the wye-connected motor, the magnet arc is 180E. With two phases
conducting, the positive and negative phasebelts produce belts of ampere-conductors 120E wide.
Therefore the motor can rotate 60E with no change in the overlap between each magnet and the belt of
ampere-conductors nearest to it. This ensures constant torque over a 60E angle. At the end of each 60E
period, the current is commutated from one phase into the next. In Fig. 2.6, phase 2 will be the next to
turn off, and phase 3 will be the next to turn on, if the rotor is rotating CCW.
In the delta-connected motor, three phases are conducting at any time, giving 180E belts of ampere-
conductors. To ensure that the overlap is constant for 60E, the magnet arc is reduced to 120E.
Page 2.6 SPEED’s Electric Machines

1 2
Flux-linkage

Rotor position
0 90 180 270 360

Coil EMF

e1 e2

Phase EMF e1 + e2

Current 120°

i1 i2 i3

!i 2 !i 3 !i 1 !i 2

Transistors conducting

5 1 1 3 3 5 5 1

6 6 2 2 4 4 6 6

60°

Fig. 2.4 Flux-linkage, EMF and current waveforms for the motor in Fig. 2.2.
Brushless permanent-magnet machines Page 2.7

Fig. 2.5 Drive circuit for delta-connected brushless DC motor.

Fig. 2.6 Interaction of arc magnets and ampere-conductor phasebelts in wye- and delta-connected
brushless DC motors.
Page 2.8 SPEED’s Electric Machines

0 30 60 90 120 150 180 210 240 270 300 330 360

i1

i2

i3

e1

e2

e3

T1 T1

T2 T2

T3 T3

Te

Q1 Q3 Q5
Q6 Q2 Q4

Fig. 2.7 Ideal waveforms of line currents iA = i1, iB = i2, iC = i3; phase EMFs e1, e2, e3; phase torques T1, T2, T3, and
total electromagnetic torque Te in wye-connected brushless DC motor drive.
Brushless permanent-magnet machines Page 2.9

0 30 60 90 120 150 180 210 240 270 300 330 360

iA

iB

iC

i1

e1

Te

Q1 Q3 Q5
Q6 Q2 Q4

Fig. 2.8 Ideal waveforms of line currents iA, iB, iC; phase current i1; and phase EMF e1 in delta-connected
brushless DC motor and drive. For more detail see Hendershot & Miller[1994].

Line Phaseleg A Phaseleg B Phaseleg C


Rotor position [Eelec] A B C Q1 Q4 Q3 Q6 Q5 Q2
330!30 0 !1 +1 0 0 0 1 1 0
30!90 +1 !1 0 1 0 0 1 0 0
90!150 +1 0 !1 1 0 0 0 0 1
150!210 0 +1 !1 0 0 1 0 0 1
210!270 !1 +1 0 0 1 1 0 0 0
270!330 !1 0 +1 0 1 0 0 1 0
TABLE 2.1
COMMUTATION TABLE FOR BRUSHLESS DC MOTOR DRIVE: 120E SQUAREWAVE LINE CURRENTS
Page 2.10 SPEED’s Electric Machines

2.3 IDEAL TORQUE/SPEED CHARACTERISTIC WITH SQUARE-WAVE DRIVE

Fig. 2.9 Torque/speed diagram

The torque/speed characteristic is similar to that of the permanent-magnet DC commutator motor, Fig.
2.9. For two-phase-on operation, if E is the line-line EMF and R is the resistance of two phases in series,
then during any 60E period we can write
Vs ' E RI (2.12)
where I is the DC current. Using eqns. (2.6) and (2.8), and neglecting losses, the torque/speed
characteristic can be derived in the form

T T I
' 1 ' 1 (2.13)
T0 T0 I0
where the no-load speed is
Vs
T0 ' rad/s (2.14)
kE
and the stall torque is
T0 ' kT I0 (2.15)
and the stall current or locked-rotor current is
Vs
I0 ' . (2.16)
R
The stall current given by this expression may be large enough to demagnetize the magnets, but usually
this current is far beyond the capability of the power transistors in the drive. Therefore, the current
is limited by chopping to a safe value that is normally well below the "demag." current.
The torque/speed characteristic is plotted in Fig. 2.9. As in the case of the DC commutator motor, the
speed is controlled by the voltage and Fig. 2.9 shows the effect of reduced voltage. The voltage is varied
by chopping the power transistors with a certain duty-cycle d; or, alternatively, by regulating with a
constant current in such a way that the effective duty-cycle is automatically maintained at the correct
value corresponding to the actual speed of the motor.
Fig. 2.9 also shows the effect of imposing a current limit, which limits the torque. For a short time it
may be permissible to operate at currents higher than the normal limit; accordingly, Fig. 2.9 is divided
into continuous and intermittent operating regions by the current-limit line.
Brushless permanent-magnet machines Page 2.11

2.4 MAGNETIC CIRCUIT ANALYSIS


The most basic magnetic calculation in brushless permanent-magnet motors is to determine the flux
produced by the magnets. An important result of this calculation is the “operating point” of the
magnets and the general saturation level of the iron. It is also important to determine the distribution
of the flux around the airgap, because this in turn determines the waveform of the generated EMF. The
EMF waveform is affected also by the winding distribution, but in this section we will concentrate only
on the calculation of magnet flux, that is, the “open-circuit” condition.
Magnetic circuit calculations in brushless permanent-magnet machines fall into three main classes:
(1) magnetic equivalent-circuit methods of varying levels of complexity;
(2) purely analytical solutions of the Laplace/Poisson equations; and
(3) finite-element methods which always require the use of a computer.

All three of these classes are important. The first two are fast and closely related to the classical theory
of machines, including the circuit theory. Finite-element methods are slow, but detailed and accurate,
and essential in cases where saturation is important, or where the geometry is complex.

Simple magnetic circuit analysis


The magnet flux can usually be calculated approximately by means of a simple magnetic circuit, such
as the one in Fig. 2.10. Under open-circuit conditions it is usually sufficient to consider only one pole,
and to make use of symmetry. Thus in Fig. 2.10 the quadrature or interpolar axes qq can be assigned
zero magnetic potential. The method is to reduce the equivalent circuit as far as possible by means of
series/parallel connections and conversions between Thévenin and Norton equivalents, and then to
“work back up the chain” to extract the required branch fluxes. When the magnetic circuit is saturated
the nonlinear reluctances can be calculated recursively, updating them by means of the BH curve
together with the appropriate geometric dimensions.
The method is fast and robust, and is a good starting point. Its main weakness is the use of lumped
parameters for components in which the field may be far from uniform. Armature reaction is not easily
incorporated, and although a separate inductance calculation can be used to model the circuit effects
of armature reaction, this approach will overlook any modification of the flux distribution by the stator
current. The magnetic equivalent circuit also does not recognize the spatial distribution of airgap flux.
It is shown later that this can be imposed independently by a semi-empirical shape function; but when
this is done, it is important to be sure that the integral of this function and its peak values are consistent
with the values predicted by the magnetic equivalent circuit.

Fig. 2.10 Simple magnetic circuit for one pole


Page 2.12 SPEED’s Electric Machines

From Fig. 2.10, the flux through the magnet is, in general,
PL prl
Mm ' Mg ML ' Mg (Mr & Mg) ' Mg (Mr & Mg) . (2.17)
Pm0 PL 1 prl
where Pm0 is the internal magnet permeance μ0μrecAm/Lm and prl is the per-unit leakage permeance
PL
prl ' . (2.18)
Pm0
In the simple case of a single external reluctance Rg, the airgap flux is
1/Rg 1/Rg 1 1
Mg ' Mr ' Mr ' Mr ' Mr , (2.19)
1/Rg Pm0 PL 1/Rg Pm 1 P mR g 1 (1 prl) Pm0Rg
where
Pm ' Pm0 PL ' Pm0(1 prl) . (2.20)

If Mg from eqn. (2.19) is substituted in eqn. (2.17), we obtain also an expression for Mm in terms of Mr:
1 P LR g 1 prlPm0Rg
Mm ' Mr ' Mr . (2.21)
1 P mR g 1 (1 prl)Pm0Rg
The leakage factor is defined in general as the ratio of airgap flux to magnet flux:
Mg Mg
fLKG ' ' < 1, (2.22)
Mm Mg ML
and if Mg and Mm are substituted from eqns. (2.19) and (2.21) we get
1 1
fLKG ' ' . (2.23)
1 PL Rg 1 prlPm0Rg

for the particular case of a single external reluctance Rg. This can be rearranged to give
1/fLKG & 1
prl ' . (2.24)
Pm0Rg
Eqn. (2.24) can be substituted in eqn. (2.21) to give
1
Mm ' Mr , (2.25)
1 fLKGPm0Rg

and in eqn. (2.19) to give


fLKG
Mg ' Mr . (2.26)
1 fLKGPm0Rg
We now have several interchangeable equations for Mg and Mm. For surface-magnet motors it is usually
more convenient to use eqns. (2.25) and (2.26) which characterize the leakage in terms of f LKG, which
typically has a value in the range 0@85 to 0@95. But for interior-magnet motors it is more convenient to
use eqns. (2.19) and (2.21), which characterize the leakage in terms of prl, because prl can often be
estimated from the geometry. In interior-magnet motors the leakage path represented by PL in Fig. 10
almost always includes a saturable element in the form of a magnetic bridge, as in Fig. 2.12, and one
way to deal with this is to assume that the bridge is permanently saturated with a flux-density of, say,
2@1T, and to subtract the flux in the bridge (or bridges) from Mr, with which they are in parallel.
Finally, given that Mg = BgAg and Mr = BrAm, eqn. (2.26) gives another convenient formula for Bg:
fLKG Am
Bg ' × Br . (2.27)
1 fLKGPm0Rg Ag
Brushless permanent-magnet machines Page 2.13

The permeance coefficient is defined as the slope or


tangent of the load line, that is, tan " in Fig. 2.11. Since
Bm ' Br μ0μrecHm , (2.28)
it follows that
Bm μrecBm / Br
PC ' ' . (2.29)
μ0|Hm| 1 & Bm / Br
This can be arranged to give another formula for Bm:
PC
Bm ' Br . (2.30)
μrec PC

Example
2
Consider a magnet with pole-face area Am ' 625 mm and
length Lm = 8 mm. The relative recoil permeability is 1@05
and the remanent flux-density is 1@1 T. Then Fig. 2.11 Permeance coefficient.

Mr ' 1@1 × 625 × 10&6 ' 687@5 μWb


and the internal magnet permeance is
μ0μrecAm 4B × 10&7 × 1@05 × 625 × 10&6
Pm0 ' ' ' 1@03084 × 10&7 Wb/A .
Lm 8 × 10 &3

2
Suppose the airgap area is Ag = 700 mm with effective gap length gN = 0@7 mm; the airgap reluctance is

0@7 × 10&3
Rg ' ' 0@7958 × 106 A/Wb ,
4B × 10 &7
× 700 × 10 &6

giving
Pm0Rg ' 0@08203 .
Suppose f LKG ' 0@95, a typical value for a surface-magnet motor. From eqn. (2.26),
0@95
Mg ' Mr ' 0@88132 Mr .
1 0@95 × 0@08023

From eqn. (2.24), prl = 0@6416. From eqn. (2.22) the flux in the magnet is
Mg 0@88132
Mm ' ' Mr ' 0@92771 Mr .
fLKG 0@95
The airgap flux-density is
Mg 0@88132 × 687@5 × 10&6
Bg ' ' ' 0@86558 T ,
Ag 700 × 10&6
which is consistent with eqn. (2.27) with Am/Ag = 625/700 ' 0@89286. The flux-density in the magnet is
Mm 0@92771 × 687@5 × 10&6
Bm ' ' ' 1@02048 T .
Am 625 × 10&6
In the example, Bm/Br ' 1@02048/1@1 ' 0@92771, so
1@05 × 0@92771
PC ' ' 13@475 .
1 & 0@92771

This value is rather high, mainly because of the large ratio of magnet length to airgap length, Lm/gN.
With a thinner magnet the permeance coefficient would be lower, and values as low as 5 are not
impractical; but allowance must be made for temperature effects and the demagnetizing influence of
the stator current, as well as the MMF expended in the steel components, which has been ignored here.
Page 2.14 SPEED’s Electric Machines

Fig. 2.12 Nonlinear magnetic circuit

Nonlinear calculation : The magnetic circuit in Fig. 2.12 is drawn for half of one Ampère's Law
contour, representing the MMF drops associated with one airgap. The magnet is represented by a
Thévenin equivalent circuit in which Fma is the "open-circuit" MMF and Rm0 = 1/Pm0. The flux densities
in the yoke and teeth sections are calculated from their permeance areas, and the associated MMF drops
F are obtained using the nonlinear BH curve of the steel. Bridge leakage in interior-magnet motors is
modelled by the flux source Mb. The total magnetising force in the magnet is calculated as

Hm ' ( Fg FSY FRY FST ) / Lm (2.31)

The circuit is solved iteratively. The basic result of the nonlinear magnetic circuit calculation is the
airgap flux Mg and from this the average flux density over the magnet pole arc can be calculated.

Airgap flux distribution used with simple magnetic equivalent circuit


It has already been pointed out that the magnetic equivalent-circuit method does not recognize the
spatial distribution of flux. To maintain the simplicity and speed of the method, it is possible to impose
a distribution function of arbitrary shape, which can subsequently be modified or corrected by
comparison with test or finite-element data. Such a distribution function is shown in Fig. 2.13 in which
b is the normalized value of the flux-density
1 & ( 2 & 2a ) / a B
b ' 1 & e , 2a < 2 < ;
2 2
1 ( 2 & 2a ) / a (2.32)
b ' e , 0 < 2 < 2a .
2

and a is an empirical coefficient given by


1
a ' g[g Lm / μrec ] . (2.33)
2

Fig. 2.13 Airgap flux distribution


Brushless permanent-magnet machines Page 2.15

Similar functions are used on the right half of the distribution, symmetrical about 2b. This fringing
function can be modified for skew, and although it is approximate it is extremely fast in computation.
Once the flux and its distribution are known, it is a straightforward matter to calculate the fundamental
space-harmonic component B1(2) and from this the fundamental magnet flux/pole MM1 and the peak
(oc)
fundamental open-circuit airgap flux density, B1 .

Demagnetizing effect at locked-rotor: The magnetic circuit model can be used to estimate the
demagnetizing field in the magnet under various conditions, for example, if the rotor is locked or in any
other condition where high current is liable to flow in the windings. The armature ampere-turns Fa per
half-pole are calculated according to the winding type and are assumed to be concentrated at a single
point in the airgap. An example of this calculation is given in Chapter 5 in connection with DC
commutator motors, which have a similar magnetic circuit. See also Hendershot & Miller [1994].

Analytical solution of Laplace/Poisson equation


Another class of analytical methods for calculating the magnet flux distribution is based on the direct
solution of Maxwell’s equations, which reduce to the Laplace equation in the air region and the Poisson
equation in the magnet. The original basis for this class of methods is the book by Bernard Hague
[1929],2 which provides a comprehensive solution for the magnetic field between two concentric smooth
iron cylinders, for an arbitrary distribution of current-carrying conductors in the airgap or on the
surfaces of the cylinders. This work was applied by Boules [1984], who replaced the magnet by an
equivalent distribution of ampere-conductors and used Hague’s solution to compute the field. The
“equivalent distribution of ampere-conductors” can be determined only in special cases: generally
where there is no irregular iron shape in the rotor, the stator has a smooth bore, and the magnet has
a simple geometric shape and a certain direction of magnetization. Boules developed solutions for
certain basic shapes of magnet including surface magnets with radial and parallel magnetization.

Equivalent ampere-conductor distributions: The magnet is replaced by a current sheet K = M × n


[A/m], where M is the magnetization vector inside the magnet and n is the unit vector normal to the
magnet surface. Since M and n are both always in the x,y plane, transverse to the axis of rotation, K
is always in the z direction along the axis of rotation, i.e. K = (0,0,K). M is the actual magnetization of
the magnet, which includes an induced component due to the demagnetizing field of the external
magnetic circuit. Unfortunately this is not known a priori. However, if the recoil permeability is near
1, the susceptibility P m of the magnet is nearly zero, and the induced magnetization is small. Boules
points out that on open-circuit the magnets are normally worked between B r/2 and B r, and he uses the
average magnetization over this range, i.e., M = k mM 0 = k m B r /μ 0 where k m = (1 + 0.75 P m)/(1 + P m).
Note that M is equivalent to the “apparent coercivity” H ca, i.e. the coercivity that the magnet would
have if its recoil line was straight throughout the second quadrant with relative permeability μr . The
value of the susceptibility P m and the constant k m can be seen in Table 2.2 for typical values of μ r. For
most magnets μ r does not exceed 1.1, so the maximum error from this approximation is less than 2.5%.

μr Pm km
1 0 1
1.05 0.05 0.988
1.1 0.1 0.977
1.2 0.2 0.958
TABLE 2.2

2
Hague’s work was done at the University of Glasgow in the 1920's. It was adapted for permanent-magnet motors by Boules [1984,
1985] by means of the equivalent current-sheet . Subsequently, new original solutions of the Maxwell equations were published
by Zhu et al [1993] and Rasmussen et al [1999]. These later solutions relied on a harmonic series representation of the
magnetization vector and therefore considerably extended the scope of the analysis.
Page 2.16 SPEED’s Electric Machines

Boules derived the ampere-conductor distributions for arc magnets whose edges xy lie along radii. For
radially-magnetized magnets, K = 0 on the curved surfaces; and on the edges, K = M. For parallel-
magnetized magnets, on the curved surfaces K = M sin 2 and on the edges K = M cos $M/2, where $M is
the magnet pole arc expressed in mechanical degrees. The equations for magnets with parallel edges
are as follows: along the outer curved surface, K = M sin 2; on the inner curved surface K = !M sin 2;
and on the edges K = M. For the “full ring” (solid 2-pole) magnet, on the outer curved surface K = M sin
2; and on the flat chamfers K = M. On the inner circular curved surface K = M sin 2.
The magnetic field is given by Hague’s solution for the field of coils distributed in the airgap between
two concentric cylinders. Fig. 2.15 shows a basic 4-pole distribution of single-turn coils having a radius
c and span 2 >, equivalent to the magnet arrangement in Fig. 2.14. The field produced by the coilset of
Fig. 2.15 at the point (r, 2) is given by

μ0 i 4

j
an c 2n b 2n rn an
Br ' 2 p @ @ @ kF n sin n > cos n 2 (2.34)
Br n cn a 2n ! b 2n an rn

where i is the coil current, p is the number of pole-pairs, and the sum is taken over all odd electrical
harmonics, i.e., n = (2j ! 1), j = 1,2,3.... The factor k Fn is the n’th harmonic skew factor, which is equal
to sin (n F/2) /(n F/2), where F is the skew in electrical radians. When the contributions of all the
filamentary coils are summed, a similar factor arises if the magnetization tapers off from a peak value
to zero over an angle F electrical radians, proving that tapered magnetization and skew are equivalent
in terms of their effect on the airgap field.

Fig. 2.14 Airgap model of surface magnet Fig. 2.15 Filamentary coil model

The methods developed by Rasmussen [1999] and by Zhu et al [1993] go beyond the Hague-Boules method
just described, by using a direct scalar potential solution that relies on a harmonic series
representation of the magnetization vector. The airgap field is given by expressions of the form

q (MnHn N nK n)
Br ' j [r q!1 a 2qr !q!1]c !q 1Hn cos q2 (2.35)
2
n μr (q ! 1)

where q = np, p is the number of pole-pairs, Mn and Nn are the n’th harmonic components of the radial
and tangential components of magnetization, and Hn and Kn are functions of q, μr and the various radii
given in Rasmussen [op cit.]. The sum is taken over all odd electrical harmonics, i.e. n = (2j ! 1), j =
1,2,3... and similar expressions are given for B2 and for the field in the magnet itself. The magnetization
is assumed to be invariant with r, i.e., it does not vary through the thickness of the magnet.
Figs. 2.16!19 show examples of different permanent-magnet brushless motors for which the above
methods are appropriate.
Brushless permanent-magnet machines Page 2.17

Fig. 2.16 Example with typical proportions. Good results Fig. 2.17 With very thin magnet and a short airgap, good
can be obtained even with the simple magnetic results can be obtained with all methods.
circuit analysis model.

Fig. 2.18 With a small rotor diameter to accommodate a Fig. 2.19 “FullRing” magnet type, a solid 2-pole magnet.
thick non-magnetic retaining ring, the simple In this case the Hague/Boules method is the
magnetic circuit model is inadequate and the best analytical method although Rasmussen’s
Hague/Boules or Rasmussen methods give much method is also appropriate. The simple
better results. magnetic circuit model is inadequate in this
case.
Page 2.18 SPEED’s Electric Machines

Clearance gap and equivalent magnet

Fig. 2.20 Clearance gap and equivalent magnet

Fig. 2.20 shows an actual magnet of length m in the direction of magnetization (vertical), with a
clearance gap n. On the right is shown an equivalent magnet whose length is
h ' m n. (2.36)
In the actual magnet,
Bm ' Br μrecμ0 Hm . (2.37)
In the clearance gap,
Bn ' μ0Hn . (2.38)
Assuming that the flux lines are all vertical,
Bm ' Bn ' B, say. (2.39)
To maintain the same magnetic potential difference between the upper and lower faces of the equivalent
magnet, by Ampere's Law we have
Hn n Hm m ' H (m n) ' H h . (2.40)
i.e.,

B B Br
n m ' Hh. (2.41)
μ0 μrecμ0
Rearranging and collecting terms in B,
1 h/m
B ' Br μrec μ0 H. (2.42)
1 μrec n/m 1 μrec n/m

This can be written


B ' Br < μrec < μ0 H , (2.43)
where Br< is the remanence of the equivalent magnet and μrec< is its recoil permeability, given by
Br h/m
Br < ' and μrec < ' μrec . (2.44)
1 μrec n/m 1 μrec n/m
In terms of a single parameter, let
< ' n/m ; (2.45)
then
Br 1 <
Br < ' and μrec < ' μrec . (2.46)
1 μrec < 1 μrec <
For example, if μrec ' 1@1, Br ' 0@4 T, m ' 5 mm, and n ' 0@5 mm, < ' 0@1, Br< ' 0@901 Br ' 0@360 T, and μrec<
' 0@991 × μrec ' 1@090. The actual and equivalent demagnetization curves are shown in Fig. 2.21.
Brushless permanent-magnet machines Page 2.19

Fig. 2.21 Example

Note that once we have a solution for H, the magnetizing force in the actual magnet can be recovered
by substituting eqn. (2.38) in eqn. (2.40) and rearranging:

1 B B
Hm ' Hh n ' (1 <)H <. (2.47)
m μ0 μ0

In the example, suppose B ' 0@2 T. Then eqn. (2.43) gives


0@2 0@360
μ0H ' ' 0@147 T, (2.48)
1@090

which implies that H ' 117 kA/m in the equivalent magnet. But eqn. (2.47) gives
μ0 Hm ' 1@1 × ( 0@147) 0@2 × 0@1 ' 0@182 T , (2.49)
which shows that Hm ' 145 kA/m in the actual magnet.
Fig. 2.22 shows the airgap flux-density distribution Bgap for a typical interior-magnet motor computed
by finite-elements (PC-FEA) in two cases: one with the actual clearance gap and magnet, and one with
the equivalent magnet. With < ' 0@167 the error in the peak Bgap is about 1%. For < ' 0@25 the error in
the peak Bgap is about 2%; for < ' 0@1 it is about 0@7%; and for < ' 0@05 it is about 0@4%. In each case the
equivalent magnet predicts a lower Bgap than the full finite-element analysis with the clearance gap.
With actual magnet and clearance gap

With equivalent magnet

Fig. 2.22 Bgap distribution test with < ' 0@167


Page 2.20 SPEED’s Electric Machines

Magnet divided by thin bracing bridges

Fig. 2.23 Magnet divided by thin bridges

Fig. 2.23 shows an interior magnet divided by thin bridges which provide bracing or support against
mechanical forces during assembly and when the motor is running. The smaller detailed drawing
shows the leakage flux in the bridges, which depletes the useful magnet flux. Generally the bridges will
be dimensioned so that they saturate, and it is the saturation that limits the loss of useful magnet flux.
For the purposes of magnetic circuit analysis each unit can be represented by an equivalent magnet of
width w, having an effective remanence BrN and recoil permeability μrecN. Let m be the width of one
magnet block, and b the width of one bridge. Then one "unit" of the array of magnets and bridges can
be taken to comprise one magnet and one bridge. The width of one unit is w ' b m, and we can define
a "bracing index" $ that characterizes the amount of bracing:
b b
$ ' ' (2.50)
w b m

With five magnets and four bridges, Fig. 2.23 suggests a slight refinement of the form $ = 4b/(4b 5m),
or (4b 2e)/(4b 2e 5m) if the edge bridges are included in the equivalent magnet.
If BmN is the average flux-density produced over the width w, Bm the flux-density in the actual magnet,
and Bs the saturation flux-density in the bridges (typically 2.1T), then
B mN w ' B m m Bs b . (2.51)
But
Bm ' Br μrec μ0 Hm , (2.52)

and if we substitute eqns. (2.50) and (2.51) in eqn. (2.52), after some rearrangement we get

BmN ' BrN μrecN μ0 Hm (2.53)


where
BrN ' ( 1 $ ) Br $ Bs and μrecN ' ( 1 $ ) μrec (2.54)

are respectively the remanence and recoil permeability of the equivalent magnet of width w. The
equivalent magnet produces the same useful magnet flux as the combination of one actual magnet and
one bridge, provided that the bridge remains saturated. Moreover, if it does remain saturated, the
linearity of eqn. (2.53) in Hm can be taken to imply that the equivalent magnet and its properties should
be used in the calculation of the synchronous inductance Ld.
The equivalent magnet provides a simplified representation of fine details such as the bracing bridges
and the clearance gap, which otherwise would need finite-element analysis with fine meshing.
Brushless permanent-magnet machines Page 2.21

Finite-element analysis
Because the Hague-Boules and Rasmussen methods are analytical, they are not well adapted to deal
with saturation effects, but the method is nevertheless still useful, either for machines where saturation
is not important, or where simplified allowances can be made for it. The simple magnetic circuit model
can make crude allowances for saturation, but for thorough analysis of the magnetic field the finite-
element method is by far the most powerful. It is particularly effective in computing the details of local
geometric features and the effects of arbitrary distributions of ampere-conductors and magnetization
patterns. These details continually increase in importance, partly because of competitive pressure to
improve performance and cost-effectiveness, but also because of the need to reduce torque ripple and
acoustic noise.

Fig. 2.24 Finite-element mesh for brushless PM motor Fig. 2.25 Magnetic field for Fig. 2.24 (open circuit)

Figs. 2.24 and 2.25 show typical examples of finite-element computations for a simple brushless
permanent-magnet motor on open-circuit, and Fig. 2.26 shows the comparison of the airgap flux-density
distribution obtained by the finite-element and magnetic-circuit methods. The finite-element solution
includes the effect of the slot-openings, which is absent from the analytical solution.

Fig. 2.26 Comparison of finite-element and magnetic circuit model


Page 2.22 SPEED’s Electric Machines

Flux spreading in the rotor yoke


d-axis
a
b q-axis

Magnet
y
b

Lm G
A
Direction of
H
c E
F
rotation

D
J
I S1
t
S2
L
B
S2
K
Steel shell
d-axis
Rotor yoke
region C

q-axis
ShellFlux.wpg

Fig. 2.27 Flux spreading in the rotor yoke

Fig. 2.27 shows the "spreading" of flux in the rotor yoke, relieving the flux-density in the interpolar
region. The model is a "developed" model wherein the magnets and the steel shell of the rotor yoke are
rolled out flat, so that cartesian geometry can be used in the analysis.
Consider one quarter of a rectangular magnet, magnetized in the direction shown. Flux leaving the
lower face must pass through the ficititious surface EFGHIJ inside the steel shell (yoke) on its way
towards the q-axis. That part of the flux passing through FGHI is labelled S1 ; it is what is normally
considered in 2-dimensional analysis. The remaining part S2 emerges through surface EFIJ, curves
round to pass through surface FLKI, and finally joins S1 to pass through the section ABCD at the q-axis.
It is assumed that the steel does not saturate underneath the magnet or in the region between EFIJ and
FLKI. It is further assumed that the flux-density is uniform as it crosses the section GFLKIH, and
remains unchanged as it passes to the q-axis at ABCD. This leaves a simple rectangular solid region
with a 1-dimensional flux pattern, which is saturable and which is to be incorporated in the lumped-
parameter magnetic equivalent circuit. This region is denoted the "rotor yoke" region and it represents
one half-pole and one half-length (in the axial direction).
The length of the rotor yoke region is simply y, where 2y is the distance between the edges of adjacent
magnets in the interpolar region. The cross-section area (through which the flux passes) is ABCD, and
this is denoted as Ary ' (b c)t, where t is the yoke thickness. Neglecting the MMF drop in the "curved"
region, the permeance of the rotor yoke region is given (per half-pole per half-length) by

(b c) t bt ) c
Pry ' μFe ' μFe where t) ' t 1 . (2.55)
y y b
where μFe is the nonlinear permeability of the steel. The equivalent rectangle btNhas the same area as
ABCD and the same nonlinear permeance as the actual rotor region, but its length in the axial direction
is b, which is the same as that of the magnet. In the lumped-parameter magnetic equivalent circuit this
is merely a convenience, since it permits the length b to be omitted from the formulation (or, what is
equivalent, it permits the analysis to proceed on the 2-dimensional basis of "unit axial length").
The equivalent rectangle btN also suggests a simple way of allowing for the flux-spreading in a 2-
dimensional FE analyis, where the main interest is in the slotted region, but the saturation of the rotor
yoke can create an erroneous solution when the formulation is based on "unit axial length".
Brushless permanent-magnet machines Page 2.23

Fig. 2.28 Tapered rotor yoke, approximately representing the effect of "flux-spreading" in the axial direction

The equivalent rotor yoke thickness is tapered from its true thickness t at the d-axis to the effective
thickness tN at the q-axis. This is done by defining two circular arcs of radius R, with centres offset from
the origin by x on either side.
2 2 2
We have x ' R r, where r is the outer radius of the rotor yoke or shell. Also R ' x h where h ' r
(tN t) ' r tc/b. Hence

r2 h2 tc tc
R ' ' r 1 ' r 1 J((1 J (/2 ) , (2.56)
2r b 2rb
where J ' t/r and ( ' c/b. The proportions in Fig. 2.28 are J ' 1/11 ' 0@0909 and ( ' 0@773 (representing
quite a large overhang). This gives R ' r(1 0@773/11 × (1 0@773/22)) ' 1@0727r and tN t ' J(r ' 0@773/11r
' 0@0703r ; or tN/t ' 1 ( ' 1@773.
An example of a 2D finite-element analysis is shown in Fig. 2.29, using the equivalent thickening of the
rotor yoke. The uniformly distributed radial flux-lines through the magnet suggest a separation of the
magnetic field into an inner part and an outer part, linked only by the magnetic potential difference
across the magnet and the airgap. Because the tapered equivalent yoke avoids the erroneous excessive
saturation of the yoke that would otherwise appear around the q-axes in a 2D calculation, the finite-
element method retains it usefulness for the calculation of the flux profile, cogging torque, etc.

ShellFluxPlot.wpg

Fig. 2.29 Example of shell flux calculated with 2D finite-elements


Page 2.24 SPEED’s Electric Machines

2.5 SLOTLESS MOTORS


The availability of very high energy rare-earth and Neodymium-Iron-Boron magnets has re-awakened
interest in the slotless motor, in which the stator teeth are removed and the resulting space is partially
filled with additional copper. At least one such motor is manufactured commercially. The slotless
construction permits an increase in rotor diameter within the same frame size, or alternatively an
increase in electric loading without a corresponding increase in current density. The magnetic
flux-density at the stator winding is inevitably lessened, but the effect is not so drastic as might be
expected.
For a motor with an iron stator yoke and an iron rotor body the magnetic field and its harmonic
components can be calculated by Hague’s method described in §2.4, or the methods described by Hughes
and Miller [1977]. Considering the fundamental radial component of B, the value is greatest at the rotor
surface (radius r) and falls off with increasing radius to its smallest value just inside the stator yoke
(radius R). The ratio between the values of the fundamental radial component at these two radii is given
by:
2(r/R)p 1
b ' (2.57)
[1 (r/R)2p]

Consider a rotor body of 40 mm diameter with a high-energy magnet of remanent flux-density 1.2 T and
thickness 5 mm. If the radial thickness of the stator winding is 5 mm (including the airgap), then for a
4-pole magnet b ' 0.78. The magnet flux-density will be about half the remanent flux-density with these
proportions, so that the radial flux-density in the stator winding varies from about 0.6 T near the bore
to 0.47 T just inside the stator yoke, giving a mean value of fundamental flux-density of about 0.53 T. The
electric loading may be increased relative to that of a slotted stator, because of the additional space
available for copper; but the increase may not be much because the close thermal contact with the teeth
is lost, and the cooling of the slotless winding by conduction to the stator steel may not be as effective.
Taking these factors into account, the power density should be roughly the same as that of the
conventional motor, and possibly a little higher, since the stator tooth iron losses are eliminated. This
machine may well accept less expensive grades of lamination steel because of the absence of slotting
and the relatively low flux-density in the stator yoke. The reactance is also lessened by the elimination
of slot leakage effects, and the risk of demagnetization is decreased.
In this type of motor the maximum useable magnet energy is obviously higher than in a conventional
slotted motor, because the there are no teeth to limit the flux by saturating; indeed the concept would
not be viable at all without magnets of high remanence and coercivity.
Once the stator teeth are removed, the conductors are no longer constrained to lie parallel to the axis.
They may be skewed by a small amount to reduce torque ripple (which is already reduced by the
elimination of cogging effects against the stator teeth). A further possibility is a completely helical
winding such as that proposed for superconducting AC generators [Ross, 1971], or as used in very small
PM commutator motors. Because the helical winding has no end-turns its utilization of copper is higher
than the severe skew might suggest, and it might permit the design of a very compact motor.

Optimum ratio of radii in slotless motor


Fig. 2.30 shows a magnet of radius r with a stator core of bore
radius R, with a winding space lying between r and R. The
torque is proportional to the force exerted on each filament of
2 2
winding, integrated over the entire winding volume, B(R r )
per unit axial length. This force is proportional to the product
BJ, where J is the current-density in the filament and B is the
radial component of flux-density produced at the winding
filament by the magnet, [13]:
μ0 A D ( p 1)
B ' [1 (r / R)2 p] cos p 2 . (2.58)
2 r
Fig. 2.30 Radii in slotless motor
Brushless permanent-magnet machines Page 2.25

A is the current-sheet density at the surface of the magnet, representing its magnetization or
polarization.3 Consequently the torque per unit length can be written
2B R
T ' J B( D ) dD d2 (2.59)
0 r

For nonzero torque J must be distributed with cos p2, so we can drop the integral with respect to 2, and
treat J as the peak value. If we calculate the torque only at the mean winding radius D = (r + R)/2, we
can drop the integral with respect to D; this assumption is rather approximate, but it becomes more
accurate as r approaches R : in other words, it is less accurate if the rotor is small. The error in this
assumption also increases with the pole-number; but motors proportioned in the “inaccurate” zone (i.e.
higher pole-number and smaller rotor) will generally be impractical anyway.
With these simplifications, if we write x = r/R and substitute D = (r + R)/2 we can write the torque as
B μ0 A 2x
p 1
T ' k B J × B (R 2 r 2) ' R 2J (1 x 2p ) ( 1 x 2) (2.60)
2 1 x
where k is a constant that includes the stack length.
2 2 2 2 2 2
Now the copper loss is proportional to J DCu × B(R r ) = J DCuBR (1 x ). By dividing this by eqn.
(2.60), we can deduce that the ratio of copper loss to torque is proportional to

(1 1 / x) p 1
(2.61)
2 (1 x 2p)
which has no minimum. Expression (2.61) implies that the least copper loss is obtained by using the
maximum possible x: that is with r as close as possible to R; but then the torque approaches zero.
This conclusion is based on the assumption of a fixed magnet remanent flux-density, represented by the
constant surface-current density A. A different conclusion is reached if we assume that the flux-density
at the winding remains constant. What this means is that as r decreases the magnet strength must
(p+1)
increase to overcome the attenuation with radius D represented by the (D/R) term in eqn. (2.58).4
Conversely, as r increases the magnet strength can be decreased : in other words, the rotor can be made
larger, but with a weaker magnet. In this case the torque function simplifies to

T ' kN B J R 2 x ( 1 x 2) (2.62)
This is plotted in Fig. 2.31. It has a maximum of 2/3/3 = 0@385 at r = R//3, implying that these
proportions give the minimum copper loss for a given torque.

0.50

0.40

0.30
y
0.20

0.10

0.00
0.0 0.1 0.2 0.3 0.4 0.5 0.6 0.7 0.8 0.9 1.0
x

2
Fig. 2.31 Optimum ratio of radii : x = r/R, y = x(1 x )

3
It is proportional to the remanent flux-density and can be obtained using Stokes’ theorem
4
Note that this term is proportional to 1/D2 for 2-pole motors, but 1/D3 for 4-pole motors, so its severity increases considerably with
the number of pole-pairs.
Page 2.26 SPEED’s Electric Machines

2.6 CURRENT CONTROL WITH SQUAREWAVE DRIVE


In this section we return to the subject of squarewave control and current regulation. For operation
with squarewave drive, ideally the motor should be designed with “concentrated windings” instead of
distributed windings, and it should have a trapezoidal EMF waveform as described earlier. The dq-axis
theory, including the phasor diagram, does not apply to this mode of operation, because it relies on the
assumption of sinusoidal voltages and currents.5 Instead, all analysis is performed in terms of actual
waveforms of voltage, EMF, current and torque.

Current regulation
We have seen in Fig. 2.7 that the currents are commutated into the appropriate phases in synchronism
with the rotation of the rotor. For 3 phase sinewave drives it is often the practice to have three
transistors conducting at any time, but with squarewave drives normally two transistors are
conducting: one “upper” and one “lower” transistor in the inverter bridge, Fig. 2.32. This is a “2Q”
switch control strategy.6
The line currents are regulated or controlled by chopping one or more power transistors in the drive.
Fig. 2.32(a) shows the normal path of the current in two lines A and B, and in two phases 1 and 2 in
series, during the interval from 30E to 90E in Fig. 2.7. The line currents and the states of the transistors
are identical for wye and delta connected motors, but the way in which the current divides between
the phases is different in the two cases. (A detailed analysis is given in Hendershot and Miller, [1994]).

Fig. 2.32 3-phase drive for wye-connected brushless DC motor.


(a) Conduction in lines A and B with transistors 1 and 6 conducting.
(b) Conduction in lines A and B with transistor 1 switched off ("chopped"). Diode 4 freewheels the line A current.

With both transistors Q1 and Q6 conducting, the voltage across the series combination of phases 1 and
2 is v12 ' Vs. When Q1 switches off, v12 ' 0, and the current freewheels through diode D4, as shown in Fig.
2.32(b). Instead of switching off Q1, we could equally well switch off Q6; then v12 ' 0 and the current
would freewheel through D3. If both transistors Q1 and Q6 are turned off, the current freewheels
through both D4 and D3, but now v12 ' Vs. This is summarized in Table 2.6.

5
The theory of space vectors relies only on the assumption of sinusoidally distributed windings, so that the voltages and currents
need not be sinusoidal. The theory of space vectors could be applied to the squarewave drive, but only if the motor had sine-
distributed windings — in which case it would be a sinewave motor fed with squarewave currents, not a squarewave motor in
the normal sense of the electronically commutated brushless DC motor.
6
The “dwell” or conduction period of the transistors in a 2Q drive can be shortened below 120E. It can also be extended beyond
120E, in which case there are periods with 3 transistors conducting. If the dwell is extended to 180E the 2Q scheme becomes a 3Q
scheme all the time. All these possibilities are included in the PC-BDC program.
Brushless permanent-magnet machines Page 2.27

Q1 D4 Q6 D3 v12
1 0 1 0 Vs
0 1 1 0 0
1 0 0 1 0
0 1 0 1 Vs
TABLE 2.6
TRUTH TABLE FOR SWITCHES IN FIG. 2.32

Fig. 2.33 Switching duty-cycle

If the transistors are switched at sufficiently high frequency, the motor inductance keeps the current
waveform smooth and the motor responds to the average applied voltage V12. Suppose the circuit is
switched between the states in Fig. 2.32(a) and 2.32(b) at high frequency, by chopping Q1 with a
duty cycle d, where d is the ratio ton /ts with which Q1 is switched: see Fig. 2.33. The switching
frequency is fs ' 1/ts. Then the average voltage V12 is given by
Vs × tON 0 × (ts ! tON) tON Vs
V12 ' ' ' d V s. (2.63)
ts ts
The effect of the chopping on the motor current can be determined by applying this voltage to the
equivalent circuit of phases 1 and 2 in series. The simplified circuit is shown in Fig. 2.34.

Fig. 2.34 Simplified circuit for chopping Fig. 2.35 Average di/dt

We have
v1 ' e1 R i1 L p1 M p2
(2.64)
v2 ' e2 R i2 L p2 M p1

where p1 ' di1/dt and p2 ' di2/dt. R is the phase resistance, L is the phase self inductance, and M is the
mutual inductance between phases. Since i3 ' 0 during the 60E interval of interest, we have i2 ' i1 and
therefore p2 ' p1. Then if we write e12 ' e1 e 2,
v12 ' v1 ! v2 ' e12 2 R i1 2 ( L ! M ) p1 . (2.65)
Now if the circuit is switched at high frequency between Fig. 2.32(a) and Fig. 2.32(b), we can replace v12
by V12, and p12 by the averaged value of p1 in the sense depicted in Fig. 2.35: this is denoted P1 ' )i/)t.
The most important practical case is where the chopping is arranged to maintain the current constant
and equal to a set point value Isp, as shown in Fig. 2.36.
Page 2.28 SPEED’s Electric Machines

In this case P1 ' 0 and from eqns. (2.63) and (2.65), the
required duty cycle d is given by
e12 2 R Isp
d ' . (2.66)
Vs
The maximum value of d is 1. When d is 1, the chopping
transistor (Q1) is on all through the 60E interval. Since
e12 (the line line voltage eLL) is proportional to speed,
this condition occurs at a particular speed, called the
base speed Tb. Evidently
Vs ! 2 R Isp Fig. 2.36 Chopping to maintain constant current
Tb ' . (2.67)
kE
The base speed is the maximum speed at which the
set point current Isp can be forced into the motor. If Isp is
equal to the rated current, Tb is therefore also the
maximum speed at which rated torque can be developed.
At higher speeds, even though d ' 1, the rising EMF rapidly
makes it impossible to force current into the motor, and
the torque decreases quickly to zero as the speed rises.
This is shown in Fig. 2.37, which also shows the hyperbola
of constant power Pmax passing through the base speed
point. Sometimes this is called the "corner point".
Commutation Fig. 2.37 Torque/speed curve
At the end of each 60Einterval the current must switch from one pair of lines to another pair. In the
preceding 60E interval the current would have been flowing in lines BC, and at the end of that interval
it is “commutated” into lines AB.
Ideally, the current in line B remains constant while the current in line C switches to line A. In
practice, however, iC does not fall to zero instantaneously, neither does iA rise from zero to Isp
instantaneously. Rather, there is a brief interval when iC is falling and iA is rising, and there is current
in all three lines at once. This interval is called the freewheeling or commutation interval: Fig. 2.38.

Fig. 2.38 Commutation interval between line C and line A

The circuit can be analyzed during the commutation interval by writing the voltage equations for two
meshes: one in which the current is rising or "building up", and the other in which it is falling or
"freewheeling". We will apply this to the commutation from lines BC to lines AB, where transistor Q5
has reached the end of its conduction period and Q1 is switched on. The current in line C freewheels
through D2, and when this current is extinguished the current in lines AB continues ("two phase on"
operation) for the remainder of the 60E interval. The chopping analysis in the previous section applies
to this condition, i.e., after the extinction of iC.
Brushless permanent-magnet machines Page 2.29

The voltage Vbld controlling the build up of current in phases 1 and 2 (lines A and B) is the line line
voltage across these two phases in series, which can be written
Vbld ' v12 ' v1 & v2 ' vAB . (2.68)
Similarly the voltage Vfwh ("fwh" ' "freewheel") controlling the decay of current in phases 3 and 2 (lines
C and B) is the line line voltage across these two phases in series, which can be written
Vfwh ' v23 ' v2 & v3 ' vBC . (2.69)
If Q1 is the controlling (chopping) transistor, the value of Vbld is determined by the state of Q1, which is
labelled sQ1 :
*V & 2V & 2R i (sQ1'1)
s q q 1
Vbld ' * (2.70)
&Vq & Rqi1 & Vd (sQ1'0)
*

Note that each transistor is modelled by a resistance Rq and a voltage drop Vq ; and each diode by a
voltage drop Vd. These equations can be combined:
Vbld ' sQ1 (Vs & 2Vq & 2Rqi1) (1 & sQ1) (&Vq & Rqi1 & Vd ) . (2.71)
The value of Vfwh is not affected by the state of Q1:
Vfwh ' Vd Vq Rq (&i2) . (2.72)
If Q1 is chopped at a sufficiently high frequency then sQ1 can be set equal to the duty cycle d, and Vbld can
be interpreted as the average voltage across the terminals AB, or V12. When we make sQ1 equal to d we
are using the principle of "state space averaging", to replace a binary value sQ1 by a real number d, as
suggested by eqn. (2.63). d is the effective value of sQ1 under the stated conditions, where the chopping
is at high frequency and the inductance is sufficient to keep the current waveform reasonably smooth.
While we are using the state space average value d in algebraic calculations of the steady state
currents, the very same formulas for Vbld and Vfwh can be used in digital simulation, where sQ1 is the
actual state of the chopping transistor, either 1 or 0 (on or off).
At the end of the commutation interval the line current iC ' i3 reaches zero and the line current iA ' i1
arrives close to Isp, the set point value of the current regulator. iA is then maintained constant by
chopping Q1. The current i1 is essentially a DC value. This is possible only if d < 1. Evidently
Vbld & ( e1 & e2 )
i1 ' . (2.73)
2R
If we set i1 ' Isp then d can be calculated as

e1 & e2 2R Isp Vd Vq Rq Isp


d ' . (2.74)
Vs & Vq Vd & Rq Isp

A simplified version of this arises if the voltage drops across the transistors and diodes are negligible
compared with the supply voltage Vs and the line line back EMF of the motor, e1 e2 ' e12. In this case,
writing Vd ' Vq ' Rq ' 0,
e12 2 R Isp
d ' (2.75)
Vs
which is exactly the same as eqn. (2.66). In 12V systems this approximation is often not admissible
because the volt drop across power transistors and diodes can be appreciable in comparison with Vs.
Page 2.30 SPEED’s Electric Machines

Conduction mode diagrams

Fig. 2.39 (case 1) shows the circuit diagram. For purposes of analysis one cycle is divided into six
segments and the interval from 30E to 90E is the “base segment” (see Fig. 2.7). The base segment starts
when Q5 switches off and Q1 switches on. Fig. 2.39 (case 2) shows the "build" loop (solid line) via lines
A and B, and the "freewheel" loop (dotted line) via lines C and B. The state of the chopping transistor(s)
is sQChop ' 1: i.e., both transistors Q1 and Q6 are on.
Fig. 2.40 shows the effect of soft chopping the line A current while the freewheeling current in line C
is still flowing: i.e., during the commutation interval shown in Fig. 2.38. Case (4) is where Q1 is
switched off and and case (5) is where Q6 is switched off.
Fig. 2.42 (case 3) shows the effect of hard chopping, where both Q1 and Q6 are switched off. Case (6)
shows the over running condition, where the generated EMF exceeds the supply voltage and the diodes
are rectifying the generated voltage. Note that this is an unregulated condition.
Reconstruction of waveforms for 3 phase squarewave motors
The PC BDC program reconstructs 360E of waveforms from the base segment which is only 60E wide.
For "120E" controls segment A [30 90E] and segment B [90 150E] are not the same (Fig. 2.39). In these
cases the base segment is calculated twice, with a different switch control in the two segments. When
2
both passes are complete, the integrals of current and current in the lines, transistors, and diodes are
constructed as indicated by the shaded areas in Fig. 2.39.

2
Fig. 2.39 Accumulation of current and current

Accumulation of Ii dt and Ii 2 dt for mean and r.m.s. currents


During any 60E commutation interval three accumulations are made forIi dt and Ii2 dt:
AQn incoming transistor (e.g. Q1)
ADn diode that freewheels the current for the incoming transistor (e.g. D4 for Q1)
AQo outgoing transistor
ADo diode that freewheels the current for the outgoing transistor
In addition, the freewheeling current in the "third line" is accumulated as ADt. This current flows only
during the commutation interval. The currents in lines A and B are accumulated as ALA and ALB
respectively. Note that ALA ' AQn ADn, and ALB ' AQo ADo. The label “C60 Q1" in Fig. 2.39
refers to one of the standard switching strategies in PC BDC, in which each transistor has control of
the chopping for 60E, and during the base segment it is Q1.
Brushless permanent-magnet machines Page 2.31

Fig. 2.40 (1) circuit diagram and (2) forward conduction mode in 3-phase squarewave drive

Fig. 2.41 Soft chopping with Q1 or Q6

Fig. 2.42 Hard chopping and over-running (rectifying)


Page 2.32 SPEED’s Electric Machines

Low-speed regeneration using step-up chopper principle

is
1 3 5 Q1 D1
e AC
Vs

4 6 2 D2

iA i iC
B
a e AB e BC
b
Q1 ON
Q5 just OFF

Q1 1 3 D3 5 1 3 D3 5

Vs Vs

4 6 2 4 D4 6 2

c d
e AB e AB

ZCond123.wpg

Fig. 2.43 Circuit diagram showing commutation and chopping

A low-speed regenerating mode is possible using the principle of the step-up chopper or "up converter".
Current is built up in a short-circuited loop by the back-EMF. This loop is intermittently open-circuited
by switching off the controlling transistor, whereupon the current freewheels back into the supply.
Fig. 2.43 (a) shows the circuit and Fig. 2.43 (d) shows the main generating mode, in which the current
is freewheeling through diodes D3 and D4. This current is built up by the EMF eAB during a prior
interval when lines A and B are shorted through Q1 and D3. By chopping Q1, the current is sustained
at a constant level Isp. Clearly the load current is is discontinuous, implying that the supply circuit is
non-inductive; in fact it is treated as a constant DC voltage source in series with a resistance Rs.
The chopping of Q1/D4 continues more or less through a 120E interval, during which D3 remains on.
At the end of this interval (150E in Fig. 2.44), Q1 switches off and remains off, while Q3 switches on. The
current freewheels through D3 and D4 until it extinguishes. With Q3 on, the EMF eBC is short-circuited
through Q3 and D5, and the current begins to build up again.
PC-BDC analyzes only the base interval 30 90E. For this reason the freewheeling to be considered at the
end of the conduction interval must be the freewheeling segment in D1 and D2 just after Q5 switches
off and Q1 switches on (at 30E). This is shown in Fig. 2.43(b) . Note that Q1 is on in this segment.
The lower transistors Q2, Q4, Q6 never switch on, in the scheme as described. However, it is possible
to have a "complementary" scheme that uses the lower transistors and equalizes the thermal duty.
Note that torque zeroes are inevitable, because the current in Fig. 2.43(b) must extinguish completely
before the current in Fig. 2.43(a) can start to build. Fig. 2.45 shows an example.
Brushless permanent-magnet machines Page 2.33

Fig. 2.44 Line current waveforms

Fig. 2.45 Example of line current waveforms, phase EMF, and torque
Page 2.34 SPEED’s Electric Machines

2.7 BACK-EMF SENSING WITH SQUAREWAVE DRIVE


Motors with 120E squarewave currents are often
operated with “back-EMF sensing” to eliminate the
need for a shaft position sensor. The schematic in
Fig. 2.46 produces a waveform of the detection
voltage e det, whose zero-crossings are used for
commutation. This voltage is obtained by switching
the detection circuit to the idle line every 60E.
The detection EMF e det is multiplexed from the
voltages e AM, e BM, and e CM according to the states of
the power transistors. The waveforms of the
current and EMF repeat every 60E (commutated into
different phases), so it is necessary to analyse only
one 60E period—the base segment described earlier.
At the beginning of this period Q5 has just switched
off, and i C continues to freewheel through D2,
forcing e det ' e CM ' Vs/2. During the freewheeling
period (“PERIOD A”) current flows in all three lines.
Since e det' Vs/2, due to the clamping action of D2,
the detection voltage is useless during this period.
When D2 switches off, PERIOD A ends and the
remainder of the conduction segment, “PERIOD B”, Fig. 2.46 Back-EMF sensing
starts. During PERIOD B, only two lines are
conducting, A and B. There is only one mesh current, that is, the one flowing through phase 1 and
negatively through phase 2. The detection voltage is given by
Vs
edet ' ! v1 e3 (2.76)
2
where dR1
v 1 ' e1 Rph i1 . (2.77)
dt
The flux linkage R1 includes a self and a mutual term, and since i2 ' i1 this is
R1 ' ( L1 ! L12 ) i1 . (2.78)
In a nonsalient pole motor, L1 ' Lph and L12 ' Mph, and both these inductances are constant so that
di1
v 1 ' e1 Rph i1 ( Lph ! Mph ) . (2.79)
dt
In a salient pole motor, if it is a sinewound motor then
L1 ' LF Lg0 Lg2 cos 2 2
1 (2.80)
L12 ' MF ! Lg0 Lg2 cos ( 2 2 ! 2 B/3 )
2
with Lg2 < 0 in most cases. Both i1 and the inductances are functions of time, so that R1 in eqn. (2.77)
must be differentiated by the chain rule. If we write L1B ' L1 L12 for the apparent inductance of phase
1 during PERIOD B, then
dR1 di1 dL1B di1 dL1B
' L1B i1 ' L1B i1 T e (2.81)
dt dt dt dt d2
where
3
L1B ' ( LF ! MF Lg0 ) Lg2 3 cos ( 2 2 B/6 ) and
2
dL1B (2.82)
' ! Lg2 2 3 sin ( 2 2 B/6 ) .
d2
Brushless permanent-magnet machines Page 2.35

By substituting all these equations back into eqn. (2.76), we arrive at an equation for the detection
voltage e det during PERIOD B. However, we still need the differential voltage equation for the whole
circuit so that the current waveform i1 can be calculated. This is simple enough if we combine all the
inductances in the single conducting loop AB into one line line inductance:
LLL ' L1 L2 ! 2 L12
(2.83)
' 2 ( LF ! MF ) 3 Lg0 ! 3 Lg2 sin ( 2 2 ! B/6 )

The derivative of LLL with respect to 2 is


dLLL
' ! 6 Lg2 cos ( 2 2 ! B/6 ) (2.84)
d2
and the required voltage equation is
di1 dLLL
Vbld ' e1 ! e2 2 Rph i1 LLL Tei1 (2.85)
dt d2
which can be rearranged for integration by Euler's method. The "build" voltage V bld is the voltage
applied to loop A B or 1 2, to build up the current in phase 1. If both Q1 and Q6 are on, it is equal to the
supply voltage Vs minus the voltage drops in the power transistors. Eqn. (2.76) is valid only for
chopping strategies in which Q1 remains on during PERIOD B, which requires that any chopping must
be performed by Q6 during the base segment. It is straightforward to program the solution for i1 in
PERIOD B using direct phase variables, because there is only one conducting mesh or loop. However,
PERIOD A with all three phases conducting is slightly more complicated.

The zero crossing angles are subject to variation caused by induced speed voltages associated with any
second harmonic variation in the self and mutual inductances of the phases. Other sources of
variation in the zero crossings include even harmonic distortion of the EMF, causing the waveform
over one half pole to differ from the waveform over the second half pole. This would happen if the
magnets were not centred, or if their magnetization varied over their width, or if the magnetization of
magnets was not uniform under all the poles. These effects can be checked by running the motor
open circuit and recording the line neutral (phase) voltage waveform.

2.8 UNIPOLAR DRIVE CIRCUITS, AND MOTORS WITH ONE OR TWO PHASES
Fig. 2.48 shows the basic waveforms of EMF, current and torque in
motors with one or two phases. In any phase winding the EMF e1 is an
alternating quantity in which the positive and negative half cycles are
similar in shape.7 This being the case, equal impulses of torque in the
positive and negative half cycles of EMF require current pulses of
opposite polarity and similar waveshape, as shown by the symmetrical
current squarewave i1. The torque produced by the interaction of e1 and
i1 is shown as T1. The EMF and the current both require a finite time to
change from positive to negative, and this results in a dip in the torque
waveform every half cycle, i.e., at twice the fundamental frequency.
A bridge circuit such as the one labelled “H-bridge” in Fig. 2.48 is
capable of driving a symmetrical alternating current waveform i1. This Fig. 2.47 Bifilar motor windings
circuit has four transistors, two of which are referenced to the negative
supply rail and the other two to the positive rail. The upper transistors are called “high-side”
transistors and the lower ones “low-side” transistors. The gating and protection circuits for low-side
devices are simpler than for high-side ones, and for this reason the bifilar circuit is sometimes used.
In this case the phase winding is split into two equal parallel paths which are connected within the
motor with opposite polarity, as shown in Fig. 2.47. The half-wave or “unipolar” current i1a flows in
one path for half a cycle; then it is switched off and the current i1b flows in the other path.
7
i.e., there are no even harmonics in the EMF waveform unless they are deliberately introduced by means of geometric or other
imbalances or asymmetry in the magnetic circuit. Generally such imbalances are acceptable only in very small motors.
Page 2.36 SPEED’s Electric Machines

e1

i1 Full
wave
i1 Ph.1
i 1a
Half
i 1b wave

T1
90°
T2
i 1a i 1b

e2
Ph.2
i2
i 2a
i 2b

Fig. 2.48 Single-phase and two-phase motor waveforms

In Fig. 2.47 the paths are shown as separate phases 1 and 2, but this is a matter of convention; the bifilar
motor in Fig. 2.47 is often termed “1-phase bifilar”. Complementary switching is used with the
transistors controlling the two paths or phases: normally each conducts for half a cycle in any full cycle.
When one switches off, the current ideally transfers immediately to the other, but in practice the
mutual coupling between the two paths is imperfect and the resulting leakage inductance retains a
fraction of the inductive energy, which must be dissipated. There is also an overvoltage on the outgoing
transistor, which may cause avalanching if a snubber is not used.
The torque dips in the T1 waveform in Fig. 2.48 are not a problem in applications such as low-powered
fans or blowers, provided that the motor can start satisfactorily. Sometimes a tapered airgap is used
to “park” the rotor at a position away from the location of zero torque when the motor comes to rest.
Alternatively the torque dips can be eliminated by adding a second phase, similar to the one in Fig. 2.48,
but with windings displaced 90Eelec from those of the first phase, and currents 90E out of phase with
those of the first phase. The current, EMF and torque contribution of a second phase are shown in Fig.
2.48. The drive circuit requires a duplication of the first H-bridge (leading to a total of 8 transistors for
2 phases), or a duplication of the bifilar drive circuit (leading to a total of 4 transistors). The bifilar
drive with two orthogonal sets of windings is often termed “2-phase bifilar” but it can equally well be
regarded as a 4-phase motor in which the complementary phases (1&3 and 2&4) are tightly coupled.
The total torque (T1 T2) in the case shown in Fig. 2.48 will have no zeroes, but it may have
considerable ripple because of the particular waveforms of the individual phase torques. This ripple
can be reduced by reducing the conduction angle from 180E to 90E in each half-cycle in each phase,
which is easily accomplished with the H-bridge full-wave circuit. But in the bifilar circuit, with less
than 180E conduction the inductive energy in the winding is not transferred to the complementary
winding. Unipolar circuits usually have no means of returning it to the supply, being designed for low
component cost: therefore this energy must be dissipated, with inevitable loss of efficiency.
Note that if the number of phases is increased to 3, the 3-wire connection can be used (with internal wye
or delta connection of the windings), and in this case the 6-transistor bridge can be used with no
restriction on the current control, in the sense that positive and negative current and voltage can be
applied to the motor terminals by appropriate switching controls. The three-phase motor has better
overlap for covering the natural torque dips, as we can see in Fig. 2.7 where each line need conduct for
only 120E in each half-cycle, leaving 60E for the EMF waveform to change polarity. Because of the control
flexibility of the three-phase drive, and the fact that it requires only 3 leads and 6 transistors, it is
overwhelmingly the most popular choice except where extremely low component cost is required.
Brushless permanent-magnet machines Page 2.37

A more detailed look at unipolar (half wave) drive circuits

Fig. 2.49 Single-ended circuit with no Fig. 2.50 Single-ended circuit with Zener
separate freewheeling path diode and damping resistor

Fig. 2.49 shows the simplest form of unipolar circuit,8 in which the winding W is controlled by a single
transistor Q. Diode D is not a freewheel diode but is the body diode if Q is a field-effect transistor.
There is no separate freewheel diode shown in Fig. 2.49. Positive current i flows from the DC supply
Vs when Q is switched on. When Q switches off, a reverse voltage v < 0 must be developed across the
winding in order to reduce the current to zero. The only source of the necessary reverse voltage is the
avalanche voltage of the transistor or diode, Vz.
It is possible to protect the transistor and diode by connecting a separate voltage-limiting circuit in
parallel with them, with a Zener diode Vz and a damping resistor Rd as shown in Fig. 2.50. To ensure
that v < 0 it is obviously necessary to have Vz Rd × i > Vs Rs × i where Rs is the supply resistance.
The peak forward voltage appearing across the transistor when it turns off is Vz Rd × i.
For a single-ended circuit such as the one
in Fig. 2.49, the EMF, current and torque
waveforms are ideally of the form shown in
Fig. 2.48. However, Fig. 2.51, shows the
possibility of spurious conduction during
the negative half-cycles of EMF, if the sum
of Vs and the peak EMF epk exceeds the
avalanche voltage Vz. This uncontrolled
current produces a pulse of negative
torque. As we shall see shortly, the
prevention of the spurious conduction
requires a zener diode with a certain
Fig. 2.51 Waveforms corresponding to Fig. 2.49
minimum avalanche voltage Vz, which can
be estimated using eqn. (2.90).
The circuit equations for the circuit in Fig. 2.50 are:
ON : v ' Vs Vq ( Rs Rq ) i (2.86)
OFF : v ' Vs Vz ( Rs Rd ) i (2.87)
where Vq is the forward voltage drop in the transistor during conduction and Rq is the transistor
resistance. These equations can be used with the circuit of Fig. 2.49 if Rd ' 0.
Fig. 2.52 shows an alternative single-ended circuit in which the suppression circuit is in parallel with
the winding, so that the supply voltage does not appear in the freewheeling path. The suppression
voltage is developed across the damping resistor Rd together with a small additional forward voltage
drop across the freewheel diode. A disadvantage is that the suppression voltage Rd × i decays with i,
so that the defluxing of the motor winding is slow.

8
Unipolar or “half-wave” circuits are also sometimes called “single-ended”.
Page 2.38 SPEED’s Electric Machines

Fig. 2.52 Unipolar circuit with Fig. 2.53 Unipolar circuit with Fig. 2.54 Current in bifilar circuit with
plain freewheeling diode Zener suppression diode negative spike at commutation

Fig. 2.53 shows a modification with a Zener diode to sustain the suppression voltage at a higher level.
As in the circuits of Figs. 2.49 and 2.50, spurious conduction is possible during the negative half cycles
of EMF, unless Vz is high enough to exceed the peak EMF epk.
When the transistor is on, eqn. (2.86) also describes the operation of the circuits in Figs. 2.52 and 2.53.
When it is off, the operation is described by the equation
OFF : v ' Vd Vz Rd i . (2.88)
The circuits in Fig. 2.52 and 2.53 can be used with any number of phases, with or without bifilar
windings, and they can be used with tapered-gap motors or motors with a “parking” device. However,
there is no point in using them with 2-phase motors which have phases displaced by 90E, or with 4-phase
motors which have phases uniformly displaced by 45E, because they cannot deliver the alternating (full-
wave) currents required in these cases.
It is important to account for mutual coupling between phases during the freewheeling period after each
phase transistor turns off. The circuit equations for the bifilar motor can be taken in pairs, since there
is no coupling between the first pair (phases 1&3) and the second pair (phases 2&4) in a 4-phase motor.
With no loss of generality we can consider the 2-phase motor, labelling the phases 1&2.9 Thus
vt1 ' e1 Rph i1 Lph p1 Mph p2
(2.89)
vt2 ' e2 Rph i2 Mph p1 Lph p2

where p ' di/dt and vt1 and vt2 are the terminal voltages of the phase windings, which depend on the
states of the transistors and diodes. These equations are rearranged in the canonical Euler form for
stepwise integration of p1 and p2 to give new currents i1 and i2, and in a simulation program the states
of the transistors are updated according to the switching algorithm and the values of the currents.
For the Zener diode circuit of Fig. 2.50, which also represents the circuit of Fig. 2.49 when the transistor
is avalanching, eqns. (2.89) can be solved algebraically for the turn-off period of the transistor. If EMF
and resistance are neglected and Mph/Lph ' k, the coupling coefficient, the solution for p1 and p2 is
(1 k ) Vs Vz (1 k ) Vs k Vz
p1 ' ; p2 ' . (2.90)
2 2
L(1 k ) L(1 k )
If k is nearly 1, these equations show that if Vz # 2Vs, p1 ' p2 $ 0, so that commutation requires Vz > 2Vs.
2
For example if Vz ' 3Vs, p1 ' p2 ' Vs/L (1 k ), and since both of these are negative it is evident that
when one transistor switches off, a negative current is induced in the complementary phase. This
current appears as a spike which disappears and is followed by the positive current pulse after a few
degrees of rotation, Fig. 2.54. If chopping is employed, the negative spike current recurs every time a
transistor is switched off, and the inductive energy in the leakage inductance is dissipated. To avoid
chopping, phase advance can be used to control the torque, (also at the expense of efficiency). High
voltage stress on the transistor (>Vz) is inevitable with these unipolar circuits during turn-off.

9
This is the one that is sometimes termed “single-phase bifilar”!
Brushless permanent-magnet machines Page 2.39

2.9 WINDINGS

One coil
T = 160
c

Phase
belt

Main
terminal

I
ph

AC motor stator showing one phase winding (of 3)


T T
c c
Turns/Coil = Tc

Coils/Pole = 2

Parallel paths a = 2
T T
Coils/phase = 8 c c

Turns in series/phase

T × Coils/pole × No. of poles


T = c
ph
Parallel paths T T
c c
160 × 2 × 4
=
2
= 640

T T
One turn = 2 conductors ("go" and "return") c c

I
Each turn (and each conductor) carries the current ph
a
Internal
Total No. of conductors Z = Tph × 2 × Nph × a star point

One phase (of 3)


= 640 × 2 × 3 × 2 = 7,680

Fig. 2.55 Definition of some terms used in motor windings

The subject of windings forms a convenient bridge between the squarewave drive and the sinewave
drive, because the layout of the stator winding has a strong influence on the all-important EMF
waveform. Fig. 2.55 shows some of the terms used in connection with motor windings. Classical
windings for AC motors were of the so-called distributed type, usually with several slots/pole and
several conductors per phasebelt, either lap-wound or concentric, as in Fig. 2.55. This type of winding
was developed for classical synchronous machines and induction machines, with the aim of
minimizing EMF harmonics and MMF harmonics. Squarewave brushless PM motors require a flat-
topped or trapezoidal EMF waveform, which can be obtained with a winding such as that in Fig. 2.2,
which essentially has a small number of conductors per phasebelt spanning almost a whole magnet pole
and several slots. This type of winding used to be termed concentrated, but the term concentrated has
recently come to refer to coils wound around a single tooth, which is a very different type of winding.
Because the number of slots/pole/phase is often quite small, and because of the widespread use of
single-tooth windings with fractional slots/pole for both squarewave and sinewave motors, the practical
differences between these windings are often not apparent.
Page 2.40 SPEED’s Electric Machines

The winding shown in Fig. 2.55 has three phases, of which only one is shown. There are four poles
(corresponding to the number of magnet poles), and for each pole there is a group of 2 coils which are
said to be “concentric” because of the way in which the outer coil embraces the inner one. The outer
coil in each pole-group in Fig. 2.55 has a span of 5 slots, and the inner one a span of 3 slots.10
Taking two coil groups together, and recognizing that adjacent coil-groups alternate in polarity , in Fig.
2.55 there are four coil-sides in adjacent slots all carrying current in the same direction, and these coil-
sides constitute what is known as a phase-belt. In this winding the phase belt is spread across 4 slot-
pitches, and since there are 24 slots covering 4 poles, this is 4/6 or 2/3 of a pole-pitch or 120 electrical
degrees. In a perfectly concentrated winding (in the old sense of “concentric”) the phase-belt would
have a spread of zero: in other words, all the coilsides in that phase-belt would be in the same slot. Thus
the winding in Fig. 2.55 is a distributed winding.
In order to make a perfectly concentrated winding, again in the old sense of “concentric”, the coils in
Fig. 2.55 would all have to have a span of 6 slot-pitches: that is, they would be “full-pitch” coils. Only
4 slots would be occupied by any phase, and half the slots in the machine would be empty. As it is, every
slot holds two coil-sides, one from each of two different phases (this is called a “double-layer” winding).
The distribution of coil-sides is critical in the determination of the EMF waveform, and also has a
significant influence on the winding inductance and the mutual inductance between phases. A full-
pitch concentrated winding produces an EMF time-waveform on open-circuit that is a replica of the
spatial distribution of magnet flux-density around the airgap. It is possible to construct the EMF
waveform of any symmetrical winding by adding together the EMF waveforms of an equivalent
distribution of full-pitch coils. The phase-shifts between the coils can be cleverly combined to eliminate
selected harmonics from the resulting phase EMF waveform. For example, windings in which all the
coils have 2/3 pitch (i.e. 2/3 of a pole-pitch, or a span of 4 in Fig. 2.55) have no 3rd harmonic in the EMF
waveform.11 To eliminate the 5th harmonic, the coil span should be 5/6 of the pole-pitch (5 in Fig. 2.55).
Formulas for the harmonic winding factors of standard windings are given in SEM-3.

10
The terms “coil span” and “coil pitch” are synonymous. They are usually measured in slot-pitches. The term “throw” is also
often used with the same meaning. However, in some factories “throw” is one less than “span” : a coil that is said to be wound
in slots “1 and 6” has a “throw” of 6 and a span of 5. In SPEED software, however, “throw” means the same as “span”.
11
Such windings are suitable for delta connection since there will be no zero sequence EMF to drive current around the delta.
Brushless permanent-magnet machines Page 2.41

2.10 WINDING INDUCTANCE

Fig. 2.57 Distribution of armature reaction MMF and flux

Fig. 2.56 Calculation of inductance

Winding inductance is very important in brushless permanent-magnet motors because it determines


the rate of rise and the rate of fall of current when the power electronic drive circuit is chopping. It
therefore determines the relationship between chopping frequency and current ripple, in much the
same way as in power electronic circuits with passive inductive loads, the main difference being due
to the EMF. At high speed, when the EMF rises above the DC supply voltage, the inductance has a
profound effect on the ability to get current into the machine, and therefore on the ability to produce
torque. Machines that are intended to operate over a very wide speed range often have to be designed
with low inductance to achieve the required torque or power at the highest speed, with the result that
a higher chopping frequency may be needed at low speed to control the current ripple.
The phase self-inductance Lph can be considered to have three components: the airgap component Lg,
the slot-leakage component Lslot, and the end-turn component Lend : thus
Lph ' Lg Lslot Lend . (2.91)
Likewise the mutual inductance between two phases has three components
Mph ' Mg Mslot Mend . (2.92)
In the following sections Lph and Mph will be considered in enough detail to show how they can be
calculated for surface-magnet motors. The slot-leakage inductances Lslot and Mslot are considered in
§3.6. Lend is considered later in this section, but Mend is ignored.

Airgap inductance of surface-magnet motors


Fig. 2.56 shows the magnetic flux established by a full-pitch coil. In a motor with one slot/pole/phase,
this could be a complete phase winding. The total MMF around a complete flux path is equal to N i,
where N is the number of turns in the coil and i is the current. If the steel in the rotor and stator is
assumed to be infinitely permeable, then the MMF is concentrated across two effective airgaps. Each
effective airgap includes the thickness of the magnet, L M, which is assumed to have a relative recoil
permeability μrec close to 1. Therefore the MMF drop across each airgap is Ni/2. If the flux is assumed
to cross the gap in the radial direction, the magnetizing force in each gap is
Ni
H ' (2.93)
2 gN

where gN is given by
LM
gN ' kc g (2.94)
μrec
Page 2.42 SPEED’s Electric Machines

and g is the actual airgap modified for slotting by Carter’s coefficient k c. An approximation for Carter’s
coefficient suitable for surface-magnet motors is
5 s
kc ' (2.95)
5 s s 2/8
where s is the ratio of slot-opening to the total airgap g, and 8 is the ratio of the slot-pitch to the gap g.
The flux-density produced by this magnetizing force at the stator bore diameter D is
μ0 N i
Bga ' . (2.96)
2 gN
The ideal flux distribution around the airgap is plotted in Fig. 2.57. The flux linkage of the coil is
BD
R ' N × Bga × Lstk (2.97)
2p

where p is the number of pole-pairs. The self-inductance is given by Lg = R/i, that is,
2
B μ0 Tph Lstk D
Lg ' (2.98)
4 p 2 gN
where Tph is the number of turns in series per phase (' Np/a, a being the number of parallel paths).
The mutual inductance between phases can be calculated in a similar way, by adding the flux-linkages
of a second coil placed in the field of the first one, Fig. 2.57. The second coil (phase 2) is located in slots
at the angles !90 + 120 = 30E and +90 + 120 = 210E. Its flux-linkage due to current in coil 1 is

D 7B B B B
R21 ' N × Bga × Lstk × (2.99)
2p 6 2 2 6
from which it can be deduced that the airgap component of mutual inductance is
Lg
Mg ' . (2.100)
3
This result is common but not general; it depends on the disposition of the windings.
The method can be used to calculate the self-inductance of a coil of any span, and the mutual inductance
between any pair of coils. From this basis the primitive airgap inductance matrix of any distribution
of coils can be calculated, and then the method of matrix reduction described in Chapter 1 can be used
to determine the inductances at the terminals, if the connection matrix is known. Alternatively, the
total terminal inductances can be determined by summing the flux-linkages in appropriate connections
of coils, given the appropriate distribution of currents among them.
An indication of the method is given in Fig. 2.58. In all cases the outer coil has a "go" coilside at G1 and
a return coilside at R1, and the current in this coil produces a flux-density Bi "inside" the arc G1R1 and
Bo outside it. If the azimuthal angles of the coilsides are G1, R1, etc., then by Gauss' law we have
B i ( R1 G1 ) B o[ 2 B ( R1 G1 ) ] ' 0 . (2.101)
The flux-linkage of a second coil is obtained by integrating the flux-density between the limits G2 and
R2, so for example in (a) the flux-linkage of coil G2R2 due to current in coil G1R1 is
R21 ' Bi ( R2 G2 ) (2.102)
In (b) we get the negative of this value, because R2 and G2 have changed places. In case (c), we get
R21 ' Bi ( R1 G2 ) B o ( R2 R1 ) (2.103)
and case (d) gives
R21 ' [ B i ( R1 R2 ) Bo ( G2 R1 ) ] . (2.104)
which proves to be the negative of case (c) when the values of the angles are substituted correctly.
Brushless permanent-magnet machines Page 2.43

Bo R1 Bi Bo R1 Bi Bo R1 Bi

R2 G2
R2
G2 G2
G1 G1 G1
R2

(a) (c) (e)

Bo R1 Bo R1 Bo R1
Bi Bi Bi
G2 R2
G2
R2 R2
G1 G1 G1
G2

(b) (d) (f)


Lgap.wpg

Fig. 2.58 Calculation of Lg

Hague's method [5] can be used to calculate the airgap components of the phase inductance and the
mutual inductance between phases, if the ampere-conductors in the slots are represented by current
filaments at the centres of the slot openings, and the stator bore is assumed to be smooth. The airgap
length is augmented by Carter’s coefficient and the magnet length, according to eqn. (2.94). It is possible
to program such a calculation for any arbitrary distribution of winding conductors, but the
computation may require a large number of harmonics (e.g., 100) to produce a satisfactory result.
Because Hague's method relies on a Fourier series expansion of the space-harmonics of the winding
distribution, it is possible to isolate the contribution of the fundamental, and then the sum of all higher
harmonic terms is called the "differential leakage inductance" Ldiff.12 The fundamental term is the one
that gives rise to the "magnetizing" or "airgap" component of the synchronous inductance, which is
treated in more detail in the next section.
It will be clear from the last three pages that the airgap inductances Lg and Mg depend on the airgap
length. The same is true of all their harmonic components including the fundamental, which gives rise
to the "magnetizing" component of synchronous inductance, and all the other harmonics which give rise
to the differential leakage inductance Ldiff. In salient-pole machines the effective airgap length is not
uniform, and the dq transformation will be used to surmount this difficulty, effectively by defining
separate effective airgap lengths for the d-axis and the q-axis, using the functions 'd and 'q in eqns.
(2.169) on p. 59.

12
Classical AC machine theory gives the differential leakage reactance via the differential leakage factor

kwn2
j
1
kd '
kw12 n>1 n2
Page 2.44 SPEED’s Electric Machines

Airgap inductance of continuously distributed winding

Fig. 2.59 Airgap inductance of continuously distributed winding

Fig. 59 shows the flux distribution over one pole-pitch, plotted in a normalized form such that x = 1
means an angle of B/2p mechanical or B/2 electrical radians, corresponding to half a pole-pitch.
The phase “spread” is 2F = 2(1 "). Within the angle 2F we will find Tc conductors. Within the angle
F = 1 " we will find C = Tc/2 conductors.
In the range 0 — " the B-distribution is written y = 1, where y is the normalized flux-density B/B0 and

Tc I CI
B0 ' μ0 ' μ0 . (2.105)
2 gN gN
I is the current and gN is the airgap length modified, if necessary, to take into account the thickness of
magnets using gN = g + LM/μrec.
In the range " < x ˜ 1 the B-distribution is written
1 x
y ' . (2.106)
1 "

Thus when x = ", y = 1; and when x = 1, y = 0.


The flux within the angle 0 — x is the integral of y from 0 to x : thus

1 1 x 1 x " (2.107)
N ' " (1 x ) dx ' " x(1 ) "(1 ) .
2 1 " " 1 " 2 2

When x = " = 1 this gives N = 2; (rectangular flux distribution, concentrated full-pitch ampere-conductor
distribution). When " = 0 and x = 1, it gives N = 1; (triangular flux distribution, continuous winding
distribution as in a DC machine armature).
Now consider the flux-linkage of an element of winding dC at x and dC at x. This winding element
links the flux N, and it has a number of turns given by
C
dC ' dx , (2.108)
(1 ")
such that the total number of turns per pole is C.
The flux-linkage of the winding element dC is NdC and so the total flux-linkage per pole is
Brushless permanent-magnet machines Page 2.45

x'1 1
1 x " C
R ' N dC ' 2 " x(1 ) "(1 ) . × dx
x'" "
1 " 2 2 (1 ")
(2.109)
2
' C (1 2").
3
When " = 1, this gives R = 2C as expected.
We can now scale the flux-linkage into ordinary units by multiplying R by B0 times half the area of one
pole-pitch, Ap/2 (which corresponds to x = 1). If all the coils of one phase are in series, the total flux-
linkage per phase is Q = 2pR, but if there are a parallel paths it is Q = 2pR/a while the terminal current
remains I and the coil current is I/a. Also the turns in series per phase is

C × 2p
Tph ' . (2.110)
Hence a

2p 2p 2 C I Ap (2.111)
Q ' R ' × C (1 2 " ) × μ0 × ×
a a 3 gN a 2

from which it follows that

Q μ0 Ap 1 2 "
L ' ' Tph 2 . (2.112)
I 2 p gN 3

In the case of a cylindrical-rotor machine we have

B D Lstk
Ap ' (2.113)
2p
and so if we write

1 2" 2
kw ' ' 1 F (2.114)
3 3
we get
μ0 B D Lstk
Lg ' kw Tph 2 , (2.115)
4 p 2gN

which we have seen before in [1] and [2]. Both these references give expressions giving kw = 0@833 with
q = 2 slots/pole/phase and kw = 0@802 with q = 3 slots/pole/phase for motors with F = 1/3, for which eqn.
(2.114) gives kw = 0@778. In the present analysis q is effectively infinite, because we have treated the
winding as a continuous band of conductors, and so it gives the lowest inductance — an interesting
example of the fact that the more “concentrated” the winding, the higher the inductance.
The mutual inductance between two phase windings can be derived by extension, but it is probably
adequate to follow the examples in [1] and [2] for a 3-phase machine and set Mg = 0@4 Lg.
The fundamental component of the B-distribution in Fig. 59 is given by
B/2 "B/2 B/2
2 4 1 22/B
y1 ' × 4 × y (2) cos 2 d2 ' cos 2 d2 cos 2 d2 (2.116)
2B 0
B 0 "B/2 1 "

where the angles are in electrical radians. The angle 2 = "B/2 corresponds to x = " in Fig. 59, and the
angle B electrical radians corresponds to one pole-pitch. The result is

1 8 "B
y1 ' × cos . (2.117)
1 " B 2 2
Page 2.46 SPEED’s Electric Machines

As " approaches 1 this converges to 4/B. If " = 2/3, y1 = 1@2159. It is interesting to compare this with the
fundamental of a rectangular wave of width " : in that case

4 "B
y1 ' sin , (2.118)
B 2
and if " = 2/3 this gives y1 = 1@1027.
Now " = 1 represents a full-pitch winding with all conductors concentrated at 0, B, 2B,... so if we take the
ratio of y1 for the actual distributed winding to y1[FP] for the concentrated full-pitch winding, and if we
substitute " = 1 F, eqn. (2.117) gives

1 8 FB FB
× 2 sin sin
y1 F B 2 2
kd1 ' ' ' , (2.119)
y1[FP] 4 FB
B 2

which is recognized as the fundamental distribution factor. It now follows from eqn. (2.117) that

4
y1 ' k . (2.120)
B d1

We can now scale y1 to ordinary units of flux-density by multiplying by B0, so that

4 μ0 CI
B1 ' kd1 . (2.121)
B gN
The flux-density B1 cos 2 produces a flux N = 2B1 sin 2 between the angles 2 and +2. Consider a circuit
element at 2 and +2 spanning d2 radians, having dC = C/[(1 ")B/2] d2 turns. The flux-linkage with
this circuit element is N dC, and the total flux linkage per pole is given by the integral

B/2
C 2 Ap 2
R ' 2 B1 sin 2 × × d2 × ' C B1 kd1 Ap. (2.122)
"B/2 1 " B B B

(Note the scaling factor Ap/B replaces Ap/2, since Ap corresponds to the angle B). As before, the total
flux-linkage per phase is Q = 2pR/a and this is

2p 2 4 μ0 CI 4 μ0 Ap
Q ' × C kd1 Ap × k ' (Tph kd1) 2 I . (2.123)
a B B d1 gN a 2
B p gN 2

The fundamental inductance per phase is therefore Q/I or


4 μ0 Ap
L1 ' (Tph kd1) 2 . (2.124)
2
B p gN
In a cylindrical-rotor machine this becomes
2 μ0 D Lstk
L1 ' (Tph kd1) 2 . (2.125)
2
Bp gN
which we have seen before in [1] and [2].

References
[1] Miller TJE [1989] Brushless permanent-magnet and reluctance motor drives. Oxford University Press 207pp ISBN 0-19-859369-4
[2] Hendershot JR and Miller TJE, Design of Brushless Permanent-Magnet Machines, 2010, published by Motor Design Books LLC,
102 Triano Circle, Venice, FL 34292, USA, ISBN 978-0-9840687-0-8, (822pp).
Brushless permanent-magnet machines Page 2.47

End-turn inductance

Fig. 2.60 Calculation of end-turn inductance

End-turn inductance is the inductance associated with the end-turns. Many factors affect the end-turn
inductance including the shape and packing density of the coils, the presence of image currents in the
stator core, as well as the shape and relative positions of the end-turns around the end of the stator core.
Fig. 2.60 shows the geometry assumed for the end-turns of a single coil. An arc length " is calculated
along a circle of radius Rw, which is the radius of a cylinder on which the straight coilsides lie. If F is
the coil span, " is somewhat less than RwF because the coilsides do not lie exactly on the slot centre-
lines, but are liable to be closer to the slot wall. The end-turns curve around a surface that has
additional curvature outside the radius Rw, but this is generally not known precisely at the design
stage, and is neglected. The end-turns from both ends are flattened on to an imaginary circle of radius
Re ' "/2, and then the inductance of the circle is calculated as

8 Re
Lend ' μ0 ReTc2 ln 2 (2.126)
R
where R is the geometric mean distance of the coilside from itself, and Tc is the number of turns per
coil, [17]. If the coilside is assumed square (as in PC-BDC), with area a, then R is given by

R ' 0@447 a . (2.127)

If the coilside is circular, with area a, R ' 0@4394 /a. Formulas for other shapes of coilside are given in
[17]. As an example, suppose " ' 40 mm and that the coilside is circular with diameter 4 mm and Tc '
2 2
100 turns. Then Re ' 20 mm, a ' B/4 × 4 ' 12@57 mm , R ' 0@4394 × /12@57 ' 1@558 mm, and

3 8 × 20
Lend ' μ0 × 20 × 10 × 1002 ln 2 ' 0@66 mH . (2.128)
1@558
The proportions of this coil are those of the "thin" coil in Fig. 2.60, i.e. Re/R ' 12@8. On the right-hand
side of Fig. 2.60 is shown a "fat" coil with the same Re but 5 times the coilside diameter, so that R ' 7@788
mm and Re/R ' 2@568, and now

3 8 × 20
Lend ' μ0 × 20 × 10 × 1002 ln 2 ' 0@257 mH , (2.129)
7@788

which is only 39% of the inductance of the thin coil.


Page 2.48 SPEED’s Electric Machines

Fig. 2.61 Coupled inductances

So far the end-turn inductance has been calculated for only a single coil. The effect of mutual
inductance between different coils of the same phase is much more difficult to calculate precisely. The
following approach is one of estimation based on certain basic principles of coupled inductances in
series.
Fig. 2.61a shows two inductors in series. Each has self-inductance L. The mutual inductance between
them is M ' kL, where k is the coupling coefficient. The inductance of the series combination is
Lt ' L L ± 2M ' 2L(1 ± k). (2.130)
The coupling coefficient can be positive or negative and |k| # 1. Perfect coupling is achieved when |k|
' 1. When k > 0, the flux linkages of the two separate inductors add; when k < 0, they subtract.
It is easy to extend eqn. (2.130) when more than two inductors are connected in series. Fig. 2.61b shows
four inductors L1, L2, L3, L4 with mutual inductances M12 ' M21, M13 ' M31, M14 ' M41, M23 ' M32, M24 '
M42, and M34 ' M43. The series inductance is
Lt ' L1 L2 L3 L4 2 M12 2 M13 2 M14 2 M23 2 M24 2 M34 . (2.131)
If L1 ' L2 ' L3 ' L4 ' L and M12 ' M13 ' M14 ' M23 ' M24 ' M34 ' L then
Lt ' 16 L ' 4 ( L1 L2 L3 L4 ) . (2.132)
In other words, if all the coils are perfectly coupled and all the flux-linkages are adding, the total
inductance is equal to the sum of the self-inductances multiplied by the number of inductors. Moreover,
this total inductance exceeds the inductance that would be obtained if the coils were uncoupled, by a
factor that is equal to the number of inductors, or

Lt ' n EL
1
' n 2L ' n × nL. (2.133)

This is the maximum inductance that can possibly be obtained with the n inductors. It is of passing
interest to construct this sum as the sum of n self-inductances L and (n 1) pairs of mutual inductances
2
M ' L, giving nL (n 1)n/2 × 2L ' n L. If the mutuals are all equal to kL, the sum is
Lt ' n 1 (n 1)k L. (2.134)
Suppose the n inductors are grouped as in Fig. 2.61c into m groups each containing q ' n/m inductors.
The q inductors of each group are perfectly coupled (M ' L), but there is no mutual coupling between
2
groups. Then the total series inductance is nL (q 1)q/2 × m × 2L ' n L/m. In Fig. 2.61c, m ' 2 groups
and n ' 4, so q ' 2 and Lt ' 8L. This is twice the inductance that would be obtained if all the coils were
uncoupled, but only half the inductance that would be obtained if they were all perfectly coupled. If the
mutuals within the group are all equal to kL, the sum is

Lt ' n 1 (q 1)k L. (2.135)

With k ' 0@5, the total inductance is only 6L.


Brushless permanent-magnet machines Page 2.49

In the phase winding of a brushless PM motor, in many cases the coils are identical and they all have
the same end-turn self-inductance L. It is clearly not the case that they are all perfectly coupled. Fig.
2.62 shows an example where there might be q ' 2 or 3 coils per group; but this machine might have m
' 4 or even many more groups, depending on the number of slots and poles. Let us suppose that m ' 8,
with q ' 2. Then n ' 16, and even if both coils of one group were perfectly coupled, the total end-turn
2
inductance would be no more than 16 /8L ' 32L. If all the coils of one group were uncoupled, the total
would be no more than 16L. Inspection of Fig. 2.62 suggests that the mutual coupling between the two
coils of each group is quite weak, with k perhaps of the order of 0@5. Eqn. (2.135) then gives 24L.
PC-BDC computes 3L coil-by-coil, initially without regard to mutual coupling between coils. The end-
turn inductance of each individual coil is calculated independently so that any differences in coil span
or number of turns is included in 3L. The sum 3L must be multiplied by a correction factor km to
account for the mutual coupling. If all the coils are identical this correction factor is obtained from
eqn. (2.135) as
km ' 1 (q 1)k. (2.136)
giving

Lt ' km E L . (2.137)

The example just quoted with q ' 2 and m ' 8 and k ' 0@5 has km ' 1 (2 1) × 0@5 ' 1@5. The maximum
possible value of km is n, when all the coils are perfectly coupled in one group (q ' n and k ' 1). The
minimum value is 1, when all the coils are uncoupled (k ' 0). Low values of q are common in PM
brushless motors (q ' 1 or 2), and k will often be near zero, so km will often be close to 1.
If m ' 8 with q ' 3 (including the third dotted coil in Fig. 2.62), we have n ' 24. The coupling coefficient
between coils 1 & 2 might be 0@5, as before, and this value can be assumed also for coils 2 & 3. But
between coils 1 & 3 it is clearly lower, maybe of the order of 0@25. Taking the weighted average for the
coupling coefficient gives k ' (2 × 0@5 1 × 0@25)/3 ' 0@42, so that eqn. (2.136) gives km ' 1@833. This can
be compared with the case where all coils are perfectly coupled, which has km ' n ' 24; or with the case
where the three coils in each group are perfectly coupled but the groups are uncoupled, giving km ' 1
+ (3 1) × 1 ' 3.
Returning for a moment to eqns. (2.126) and (2.127), if the coilsides in Fig. 2.60 were square (as assumed
in PC-BDC), the end-turn inductance of the thin coil would be 0@657 mH and that of the thick coil would
be 0@253 mH, hardly any different from the values obtained with circular coilsides.

Fig. 2.62 End-turns of a group of 2 or 3 coils

A more detailed way of accumulating the mutual inductances of a group of coils is developed in the
following pages.13

13
The idea for this extended method was suggested by Dr. P. Anpalahan of Cummins Generator Technologies.
Page 2.50 SPEED’s Electric Machines

Fig. 2.63 Concentric end-winding

Fig. 2.63 shows a set of concentric end-windings. The inductance of one of these is calculated from
Maxwell's formula, eqn. (2.126) on p. 47, including both ends. The radius R e in Fig. 2.63 is that of the
central filament; the coil cross-section is not shown, but it can be treated as a square of area a and
geometric mean radius given by eqn. (2.127), or as a circle of area a and GMD 0@4394a, as discussed
earlier on p. 47.
Considering only the end-winding flux-linkages and inductances, let L 1 , L 2 , etc. be the self-inductance
components of the coil inductances, and let M 12 , M 13 , etc. be the mutual-inductance components. The
self- and mutual inductance components can be ordered in the matrix in eqn. (2.138) below. Only the
upper diagonal is shown, as the matrix is always symmetric.

L1 M 12 M 13 M 14 M 15
L2 M 23 M 24 M 25
[LE] ' L3 M 34 M 35 (2.138)
L4 M 45
L5

If there were no magnetic coupling between coils, there would be no mutual inductances and the total
inductance of a coil-group would be the sum of the self-inductances, assuming that all coils in the group
are connected in series. This is obtained by adding all the diagonal elements.14
In general, the total end-winding self-inductance of the coil-group is equal to the sum of all the elements
in the matrix. These concepts can be extended to deal with the mutual inductance between phases, and
with arbitrary connections of coils using connection matrices; see SEM-1.
While the self-inductance components can be estimated using Maxwell's formula, eqn. (2.126) on p. 47,
there is no simple equation for the mutual-inductance components. For engineering purposes we need
a simple way to estimate them, and three different methods are postulated below.
1. Total enclosed linkage method. In Fig. 2.63, if we assume that the flux produced by end-winding
3, for example, is linked approximately entirely by end-windings 1 and 2, we can write M 13 = M 31 = L 3 .
Continuing this argument, the complete set of self- and mutual inductances of this set of end-windings
can be tabulated in the symmetric matrix in eqn. (2.140), in which only the upper triangle is filled.
If all the coils are in series, the sum of all the end-winding inductances is

LE ' L1 3 L2 5 L3 7 L4 9 L5 . (2.139)

14
That is, the “trace” or “spur” of the matrix.
Brushless permanent-magnet machines Page 2.51

L1 L2 L3 L4 L5
L2 L3 L4 L5
[LE] ' L3 L4 L5 (2.140)
L4 L5
L5

If all the coils are in series, the sum of all the end-winding inductances is

LE ' L1 3 L2 5 L3 7 L4 9 L5 . (2.141)

If there are Q coil-groups per phase, and a parallel paths, with n coils per group, the total contribution
to the end-winding inductance per phase can be written
n

j ( 2 j £ 1 ) Lj
Q
LE ' (2.142)
a 2 j'1
This simple formulation requires only the elements in the first row of the matrix.

2. Area ratio method. Considering current in, say, only coil 2 in Fig. 2.63, assume that the flux linked
of coil 4 is equal to A4/A2 times the flux linked by coil 2 itself, where A 2 and A 4 are the areas enclosed
by the respective end-windings. The flux-linkages are therefore in the ratio k 24 = N 4 A 4 /N 2 A 2 , so it
follows that M42 = M24 = k24L2, where N 2 and N 4 are the respective numbers of turns. This principle can
be applied to all the off-diagonal elements of the matrix, as in the example in eqn. (2.143). As explained
earlier, the total end-winding inductance of the coil-group is the sum of all the elements.

L1 k 12 L 2 k 13 L 3 k 14 L 4 k 15 L 5
L2 k 23 L 3 k 24 L 4 k 25 L 5
[LE] ' L3 k 34 L 4 k 35 L 5 (2.143)
L4 k 45 L 5
L5

3. Coil-span ratio method. This is illustrated in Fig. 2.64, where the coil-ends are compressed in the
axial direction and are no longer regarded as ideal semicircles. The area-ratio method can be adapted
for this case by setting k 24 = N 4 F 4 /N 2 F 2 , where F is the coil-span in suitable units, again using coils 2
and 4 as typical of all combinations. Again the total end-winding inductance of the coil-group is the sum
of all the elements.

Fig. 2.64 Concentric end-windings compressed in the axial direction


Page 2.52 SPEED’s Electric Machines

4. Rosa & Grover method. All the previous methods tacitly assume that the flux is uniformly
distributed across the area of the coil whose current is exciting it. Rosa and Grover [17] give a formula
for the mutual inductance between concentric filamentary coplanar circles of radii a and a+c: thus if
" = a/c and ( = 1/" = c/a,

( (2 (3 17 (4 19 (5
M ' μ0 a Ln 8 " 1 £ £ ...
2 16 32 1024 2048 (2.144)
2 3 4 5
( 3( ( 19 ( 379 (
£ 2 £ £ ...
2 16 48 6144 61440

As with the self-inductance formula (2.126) on p. 47, the mutual inductance is divided equally between
the two ends. This method gives considerably smaller mutual inductances than the others.

Fig. 2.65 Diamond end-windings

The area ratio method can be adapted for diamond end-windings, as shown in Fig. 2.65. In this case all
the self-inductances are equal, L. The end-winding mutual inductance between any two coils is written
kL, where k is a coupling factor that can be approximated as the ratio between the hatched overlap area
and the apparent coil area in Fig. 2.65. From the geometric construction

s h(b £ s) 2by
k ' 1 £ @ . (2.145)
b b(h 2y)

L k 12 L k 13 L k 14 L k 15 L
L k 23 L k 24 L k 25 L
[LE] ' L k 34 L k 35 L (2.146)
L k 45 L
L

The inductance matrix is constructed as in eqn. (2.146). In the example with 5 coils/group the total
contribution to the phase inductance is given by

2Q
LE ' L 5 2 ( 4 k12 3 k13 2 k14 k15 ) , (2.147)
a2
and in general with n coils/group
Brushless permanent-magnet machines Page 2.53

j 2(n £ j
2Q
LE ' L (2n £ 1) 1 ) k1 j . (2.148)
a2 j'1

Again the only inductances that need be known are those in the top rows of the matrix in eqn. (2.146).
The method of assembling the total end-winding contribution L E from the sum of all the elements in
the end-winding inductance matrix is quite general, and could be used with a more sophisticated
method of computing not only the self-inductance components L but also the mutual-inductance
components M : for example, a 3D finite-element computation or even a measurement on a suitably
constructed test machine.
All the estimates described here are very rudimentary. No account is taken of the linkage outside the
overlap area. The true three-dimensional shape of the end-windings is not accounted for.

Fig. 2.66 Overlap of concentric coils from different coil-groups

In the case of concentric windings, Fig. 2.66 shows the construction for applying the area-ratio method
to the overlap between two coils belonging to different coil-groups (and generally to different phases).
This construction provides the equivalent of eqn. (2.145) for the diamond-shaped endwindings of a lap
winding, though not in closed form. The overlap area (hatched) is

1
A ' R 2 ( 2 2 £ sin 2 2) r 2 ( 2 N £ sin 2 N) y(u v) (2.149)
4
and the following auxiliary equations are required :

h ' R sin 2 ' r sin N ;


u v ' R r £ b; and
(2.150)
R2 £ r2 b2
R cos 2 ' .
2b

Finally, an allowance can be made for the proximity of the iron surfaces at the end of the lamination
stack by multiplying the final inductances by a factor whose value (from the theory of images) must lie
between 1 and 2, and is probably generally of the order of 1@2 — 1@5.
Page 2.54 SPEED’s Electric Machines

Fig. 2.67 Three-phase short circuit used to determine synchronous inductance.

Synchronous inductance
Synchronous inductance Ls is a fictional inductance that accounts for the inductive voltage-drop in each
phase of a synchronous machine with all three phases conducting balanced sinusoidal currents.15 In
phase 1, for example, it includes the mutual inductive voltage drops due to the currents in phase 2 and
phase 3, in addition to the self-inductance voltage drop due to current in phase 1. Ls thus "eliminates"
the mutual inductances from AC circuit calculations under balanced conditions. In a balanced machine
Ls replaces three self- and three mutual inductances by a single parameter.
A simple way to visualize the effect of Ls is to run a synchronous machine with all three phases shorted
together: i.e., with a balanced three-phase short circuit. The EMF generated in the three phases drives
a current in each phase which is limited only by the resistance and inductances of the windings. At all
but the lowest speeds the resistance can be neglected, and the following analysis shows that the short-
circuit current is limited by the single parameter Ls.
For the loop containing E1 and E2 we have the "mesh voltage equation"
E1 E2 jTL (IA IC) jTL (IB IA)
jTM (IB IA) jTM (IA IC) (2.151)
jTM (IC IB) jTM (IC IB) ' 0.

But IA IC ' I1 , IB IA ' I2, and IC IB ' I3; and with E12 ' E1 E2 we get

E12 jT(L M ) I1 jT(L M ) I2 ' 0 . (2.152)


jB/6 j2B/3
From Fig. 2.67(a) and (b), E12 ' /3 E1 e and I2 ' I1 e , so

3 E1 e j B / 6 jT(L M ) I1 [ 1 e j2B/3
] ' 0. (2.153)

The apparent impedance per phase is the ratio E1/I1, which is


E1 jT(L M)[1 e j2B/3
]
' ' jT(L M ) ' j T Ls . (2.154)
I1 3 e jB/6

If the phase resistance is negligible, this shows that the synchronous inductance Ls is equal to (L M).
The 'j' also establishes that I1 lags E1 by 90E, as shown in Fig. 2.67(c).

15
Synchronous inductance is not applicable to PM brushless motors that are supplied with nonsinusoidal currents.
Brushless permanent-magnet machines Page 2.55

In classical synchronous machine theory, the self-inductance of each phase of a nonsalient-pole machine
comprises an airgap component Lg0 associated with airgap flux, together with a leakage component LF
which is associated with slot-leakage flux and end-winding flux. Likewise the mutual inductance
between phases comprises an airgap component Mg and a leakage component MF. For 3-phase
sinewound nonsalient-pole machines with balanced windings, Mg is equal to Lg0 multiplied by cos 120E,
i.e., Lg0/2. Consequently
3
Ls ' L M ' Lg0 ( LF MF ) . (2.155)
2

This explains why the synchronous inductance is often thought to be 3/2 times the phase self-
inductance—though not exactly 3/2 times, because of the leakage components LF and MF.
Under sinusoidal AC conditions the synchronous inductance Ls gives rise to the synchronous reactance
Xs ' T Ls ,
where T is the electrical radian frequency 2B f.
So far we have considered only nonsalient-pole 3-phase machines, and then only sinewound machines
operating with sinusoidal current and voltage and negligible saturation. The commonest example of
this is the surface-magnet permanent-magnet AC brushless motor operating with sinewave drive. The
steady-state operation of this machine is described in §2.7 in terms of the phasor diagram.
First, however, the analysis of inductance is extended to salient-pole machines such as the interior PM
motor, in §2.6. We shall see that two synchronous reactances are needed for these machines, Ld and Lq.
The effect of saturation is taken into account in §2.23.
Page 2.56 SPEED’s Electric Machines

2.11 INDUCTANCES OF SALIENT-POLE MOTORS

Fig. 2.68 Cross-section of 4-pole salient-pole PM motor Fig. 2.69 Reference axes in electrical degrees
showing dq-axes and one phase winding

In “embedded-magnet” motors, including “interior-magnet” and “inset-magnet” motors, the winding


inductances vary as the rotor rotates. This property is known as saliency and such motors are classified
as salient-pole motors. Most salient-pole motors have a narrow airgap, which increases the likelihood
of saturation and causes the inductance to vary with current as well as rotor position, but saturation
is not considered until §2.23. We shall see that salient-pole machines can be analyzed with two
synchronous inductances Ld and Lq, which is still much simpler than using three time-varying self-
inductances and three time-varying mutual inductances.

Fig. 2.70 Variation of inductance in the motor of Fig. 2.68

Fig. 2.70 shows the variation of the phase self-inductance Laa and the phase-phase mutual inductance
Lbc in the motor of Fig. 2.68.16 The self-inductance Laa has a positive mean term together with a double-
frequency variation with 2, which contains second and higher-order even harmonics. The mutual
inductance Lbc has a negative mean term that is approximately half the mean term of Laa, together with
a double-frequency varation containing second and higher-order harmonics. Because the finite-element
calculation is 2-dimensional, end-turn inductance is not included.
16
The calculation was performed by finite-element analysis, in which the rotor was rotated through 360E elec at 8E intervals,
maintaining sinusoidal phase currents in the three phases. At each position the phase self-inductance and the phase-phase mutual
inductance was computed with frozen permeabilities. With a ferrite magnet and a maximum current-density of about 0@7A/mm2
in the slots, the flux-densities did not exceed 1@5T in the teeth and 0@9T in the rotor poles, so the effect of saturation was quite small.
Brushless permanent-magnet machines Page 2.57

The dq-axis transformation (Park's transformation) is normally used for analyzing salient-pole motors,
because it attempts to make the inductances independent of rotor position by working in a frame of
reference that rotates in synchronism with the rotor, [9]. Under steady-state AC conditions the currents
and voltages in this frame of reference appear constant, as though they were DC quantities.
In the dq-axis transformation it is normally assumed that the motor is sinewound: i.e., the windings are
sine-distributed (or nearly so). It is almost universally assumed, irrespective of the rotor shape, that
there is only a second-harmonic variation, and higher-order harmonics are ignored. Then the phase
self-inductances (Laa, Lbb, Lcc) and mutual inductances (Lab, Lbc, Lca) can be expressed as
Laa ( 2 ) ' ( LF Lg0 ) Lg2 cos 2 2
Lbb ( 2 ) ' ( LF Lg0 ) Lg2 cos 2 (2 2B/3)
Lcc ( 2 ) ' ( LF Lg0 ) Lg2 cos 2 (2 2B/3)
(2.157)
Lab ( 2 ) ' Lba ' ( MF Lg0 / 2 ) Lg2 cos 2 (2 2B/3)
Lbc ( 2 ) ' Lcb ' ( MF Lg0 / 2 ) Lg2 cos 2 (2)
Lca ( 2 ) ' Lac ' ( MF Lg0 / 2 ) Lg2 cos 2 (2 2B/3) .
Fig. 2.71 shows this form of variation for one phase self-inductance Laa and one phase-phase mutual
inductance Lbc, with rotor position 2 defined in Fig. 2.69. The units of inductance are arbitrary in Fig.
2.71 but it is helpful to think of them as “per-unit”. In both Laa and Lbc the "double-frequency" variation
is a pure second-harmonic. Note the negative value of Lg2, which is characteristic of interior-magnet
motors. In contrast, wound-field machines generally have Lg2 > 0.

Fig. 2.71 Variation of Laa and Lbc with rotor position 2.

Self-inductance — The constant term in the phase inductance comprises a leakage component LF and
two "airgap components" Lg0 and Lg2. Lg0 and Lg2 are attributed to the “airgap” component of the
magnetic field produced by the fundamental space-harmonic component of the stator conductor
distribution. With fixed current in one phase, this component of the magnetic field rises and falls with
the double-frequency modulation caused by the saliency of the rotor, [18]. The leakage component
includes slot-leakage, end-turn leakage, and "differential" leakage —i.e., inductance associated with
higher-order space harmonics of the winding distribution and the effects of any skew.
The self-inductance attains extreme values when the d- and q-axes of the rotor are aligned with the
phase axis: thus, for phase a:
Laa[d] ' ( LF Lg0 ) Lg2
(2.158)
Laa[q] ' ( LF Lg0 ) Lg2 .

These inductances can be measured and then eqn. (2.158) can be solved to give
Laa[d] Laa[q] Laa[d] Laa[q]
( LF Lg0 ) ' ; Lg2 ' . (2.159)
2 2
Page 2.58 SPEED’s Electric Machines

Mutual inductance — The constant term in the mutual inductance is dominated by Lg0/2 which is
really Lg0 cos (2B/3), reflecting the 120E spacing between the axes of adjacent phases. MF represents
mutual coupling between phases in the slots and end-turns, together with any contributions from
higher-order space-harmonics of the winding distribution. Pronounced saliency can distort the airgap
field of the stator current of one phase to such an extent that it is possible for Lbc to be zero at certain
rotor positions, and even to reverse sign, as shown in Figs. 2.70 and 2.71.
The phase-phase mutual inductance attains extreme values when the d- and q-axes are aligned mid-way
between the respective phase axes. For example Lbc reaches a maximum value MF Lg0/2 Lg2 when
2 ' 0, 2B, ...; and a minimum value MF Lg0/2 Lg2 when 2 ' B, 3B/2, ...
With wye connection the line-line inductance between lines b and c is
LLL ' Lbb Lcc 2 Lbc , (2.160)
the negative sign of 2Lbc being due to the reverse connection of phase c in series with phase b in the
line-line circuit. When eqns. (2.157) are substituted in eqn. (2.160), LLL simplifies to
LLL ' LLL0 3 Lg2 cos ( 2 2 ) with LLL0 ' 2 ( LF MF ) 3 Lg0 . (2.161)
Like the phase inductance, LLL also has a constant term and a second-harmonic term. It attains an
extreme value at 2 ' 0, when the q-axis is aligned with the effective magnetic axis of the series
connection of phases b and c; and another extreme value when 2 ' ±B/2, when the d-axis is aligned with
this axis. Thus
LLL[d] ' 2 ( LF MF ) 3 [ Lg0 Lg2 ] ' LLL0 3 Lg2 when 2 ' B/2
(2.162)
LLL[q] ' 2 ( LF MF ) 3 [ Lg0 Lg2 ] ' LLL0 3 Lg2 when 2 ' 0 .

With Lg2 < 0, LLL[d] < LLL[q]. If the windings are connected in delta, the line-line inductance between
lines b and c can be shown to be
LLL 0
LLL [)] ' Lg2 cos ( 2 2 ) (2.163)
3
which, as expected, is 1/3 the value obtained with wye connection in eqn. (2.161).
The extreme values of LLL could be measured, and then eqns. (2.162) could be solved for LLL0 and Lg2 :
LLL[d] LLL[q] LLL[d] ! LLL[q]
LLL0 ' ; Lg2 ' (2.164)
2 6

Synchronous inductances — The self- and mutual inductances in eqn. (2.157) form a full 3×3
inductance matrix in stationary axes defined by the phase windings a,b,c. The dq-axis transformation
is used to diagonalize this matrix into diag[Ld, Lq, L0], in which the synchronous inductances Ld, Lq are
independent of rotor position 2: (see [9] and p. 142). Thus
Ld ' LF MF Lmd ; Lq ' LF MF Lmq (2.165)
where
3 3
Lmd ' ( Lg0 Lg2 ) ; Lmq ' ( Lg0 Lg2 ) . (2.166)
2 2

In most brushless PM motors Lg2 < 0, so that Lmd< Lmq and Ld < Lq. In contrast, wound-field salient-pole
machines usually have Ld > Lq.
Because Lmd and Lmq are defined in a frame of reference that is synchronous with the rotating magnetic
field, they can be calculated from the flux-linkage of a sine-distributed winding calculated with the
excitation current oriented along the d or q axis, as described in [2] and below. The result is
Lmd ' 'd Lm0 ;
(2.167)
Lmq ' 'q Lm0 ,

where Lm0 is the airgap component of the synchronous inductance of a nonsalient pole machine having
an airgap length equal to gN ' k c g, where g is the actual airgap and kc is the Carter coefficient. Thus
Brushless permanent-magnet machines Page 2.59

3 μ0 D l
Lm0 ' ( kw1 Tph )2 . (2.168)
2
B p gN
where kw1 is the fundamental winding factor, Tph is the number of turns in series per phase, and T ' 2Bf.
For 2-phase motors, replace 3 by 2 in this equation. The coefficients 'd and 'q are given by
gN gN
'd ' ; 'q ' . (2.169)
gdNN gqNN

Here gdNN is the effective airgap in the d-axis including the effects of the magnet and the saliency; and gqNN
is the same in the q-axis. gN is given by eqn. (2.94). Formulas for gdNN and gqNN are derived for a number
of rotor types in [2], using a magnetic field solution directly and thereby avoiding the need to
precalculate Lg0 and Lg2. The synchronous reactances follow as
Xd ' 2 B f Ld ' Xmd XF
(2.170)
Xq ' 2 B f Lq ' Xmq XF
where
Xmd ' 2 B f Lmd ' 'd Xm0
(2.171)
Xmq ' 2 B f Lmq ' 'q Xm0 .

Once Lmd and Lmq have been calculated from eqn. (2.167), Lg0 and Lg2 can be calculated from eqn. (2.166):

1 1
Lg0 ' [Lmd Lmq ] ; Lg2 ' [Lmd Lmq ] . (2.172)
3 3

Differential leakage inductance


This component of phase inductance is supposed to represent the effect of winding harmonics other
than the fundamental. It thus represents the difference in inductance between the actual phase winding
and an equivalent winding which has only the fundamental space-harmonic of MMF. The differential
leakage appears in both the self and mutual inductances.
A sine-distributed 3-phase winding has an airgap inductance of Lg0 and a mutual inductance of Lg0/2
between phases. Therefore if Lg and Mg are the actual airgap self- and mutual inductances (including
winding harmonics), the respective components of differential leakage inductance can be estimated as
Ldiff ' Lg Lg0 ; Mdiff ' Mg ( Lg0 / 2 ) . (2.173)
Ldiff and Mdiff can be added to LF and MF to account for the harmonic terms above the second-harmonic
in 2 in eqn. (2.157). But since Lg and Mg themselves vary with 2, it is unclear what value to use. One
possibility is to take the average value, but if higher-order inductance harmonics are truly significant
the dq transformation should not be being used.
PC-BDC’s procedure begins by calculating the airgap inductance Lgg with the rotor replaced by a steel
cylinder of infinite permeability and zero conductivity. The calculation of Lgg uses the actual winding
distribution and it assumes that the flux crosses the airgap in the radial direction, as shown in Fig. 2.56
for a simple full-pitched coil. It accumulates the flux-linkage in every coil with 1A flowing in the
winding, using Bgap distributions of the type shown in Fig. 2.57 Then Lg is scaled from Lgg according
to the following equations. For surface-magnet machines,
gN
Lg ' Lgg × (2.174)
gNN

where gN is the actual airgap modified by Carter’s coefficient, and gNN ' gN LM/μrec, where LM is the
magnet length and μrec is its relative recoil permeability. For salient-pole machines,
'd 'q
Lg ' Lgg × . (2.175)
2
Page 2.60 SPEED’s Electric Machines

'd is the airgap ratio gN/gdNN, which is equivalent to the airgap ratio gN/gNN in eqn. (2.174) when the d-axis
of the rotor is aligned with the axis of the phase winding. Similarly 'q is the ratio gN/gqNN when the q-
axis is aligned with the phase axis. Expressions for 'd and 'q are given in the next section. In PC-BDC
the coefficients 'd and 'q can be adjusted directly using factors XCd and XCq.
Note that eqn. (2.175) is taking the average of the d- and q-axis values, implying that Ldiff will be some
kind of average between d- and q-axis values. Furthermore, 'd and 'q are strictly only valid for sine-
distributed windings, so it is not strictly correct to use them for Lg. This means that Ldiff will be, at best,
only an indicative value.
Returning to eqn. (2.173), Lg0 is the average value of the airgap component of phase self-inductance as
the rotor rotates through 360Eelec. It is computed from the fundamental of the MMF distribution by
'd 'q 2
Lg0 ' × Lm0 , (2.176)
2 3
where Lm0 is the classical value of the magnetizing inductance per phase, when the rotor is a cylinder
of infinite permeability and zero conductivity. The 2/3 factor comes from the particular choice of dq
transformation for 3-phase motors, and is not necessary for 2-phase motors. Lm0 is given by eqn. (2.168).
Substituting Lg from eqn. (2.175) and Lg0 from eqn. (2.176) in eqn. (2.173) for Ldiff, we get
'd 'q 2
Ldiff ' Lgg Lm0 . (2.177)
2 3
Note that although the differential leakage inductance is calculated here as an average between the d-
and q-axes, PC-BDC has the facility to adjust the d- and q-axis values independently, and it is found with
salient-pole machines that this is often necessary to match the flux-linkage calculations of the finite-
element method.
With multiplex windings the mutual inductances mF12d, mF12q, mF23d etc are required, as described in
eqn. (2.474) on p. 144, and these contain a differential or harmonic component which we calculate as
Lmd Lmq
m*12d ' 8* cos " μ* cos (" 120E) <* cos (" 120E) . (2.178)
2
The mutual inductances 8*, μ* and <* are "airgap" inductances calculated by the method described on
p. 41, using the effective airgap gN, and respecting the angles between the phases defined in Fig. 159 on
p. 144. They include the effect of all winding harmonics including the fundamental, which needs to be
removed to isolate the pure "differential" component. Because of the uncertainty introduced by the non-
uniform airgap in salient-pole machines, the removal of the fundamental is deferred until the required
inductance is "in" the dq frame of reference, and then the average of Lmd and Lmq is subtracted, leaving
m*12d ' m*12q ' m*23d ' m*23q ' m*31d ' m*31q etc.

Criterion for sinewound motors


To help decide whether a motor is sinewound, a parameter
2
Lm0
3 2 Xm0 (2.179)
ksg ' '
Lgg 3 T Lgg

can be calculated. ksg is unity for sinewound motors. Lm0 is the synchronous inductance and Lgg is the
actual airgap inductance that would be obtained if the winding was perfectly sine-distributed and the
rotor was a smooth iron cylinder with the same airgap as in the actual machine. Both Lm0 and Lgg are
free of end-effects and slot-leakage. For 1- or 2-phase motors the 2/3 factor is omitted. If ksg is close to
1, the winding is nearly sine-distributed, and calculations with dq equations should agree with those
performed with direct phase variables. On the other hand, if ksg is far from 1, the differential leakage
will not be negligible and calculations with dq equations may not agree with those performed with
direct phase variables. This applies equally to nonsalient-pole motors, so ksg is calculated for these too.
Brushless permanent-magnet machines Page 2.61

Calculation of synchronous inductance coefficients


The method for calculating the coefficients 'd and 'q used in
eqns. (2.167) was first described in [2], but it can be generalised
to calculate Ld and Lq for almost any rotor configuration. Fig.
2.72 shows a 2-pole IPM rotor with n ' 3 magnet blocks per pole.
The d-axis is vertically upwards and the magnets in the upper
half of the rotor are shaded to indicate that they have the same
polarity.
Three lines 1,2,3 of stator flux ("armature reaction") are shown
passing across the poles in a direction parallelling the q-axis.
They are excited by a stator MMF distribution having the form
u ( 2 ) ' û s cos 2 (2.180)
where 2 is measured from the horizontal axis. This MMF Fig. 2.72 Salient-pole rotor
distribution is excited by q-axis current iq.

Fig. 2.73 Calculation of flux distribution excited by stator ampere-conductor distribution

Fig. 2.73 shows the method of analysis. In the example in Fig. 2.72 the rotor has six sections which are
identical apart from the polarity of the magnets. In Fig. 2.73 we take one of these sections by itself and
calculate the airgap flux-distribution produced by the stator MMF distribution, eqn. (2.180).
Following the method described in [2], across the face of the pole-cap we have
μ0
Bg ( 2 ) ' û s cos 2 u1 , 21 < 2 < 22 (2.181)
gN
where u1 is the undetermined magnetic potential of the pole-cap, which is assumed to be “floating”. The
flux-density is zero outside this section in the ranges 0 21 and 22 B/2. The flux passing into the pole-cap
is obtained by integrating Bg(2) between 21 and 22 :
22 1
Mg ' Bg ( 2 ) r1 l d 2 ' û s ( sin 22 sin 21) u1 ( 22 21 ) (2.182)
21 " Rg

where r1 is the radius at the airgap, l is the stack length, " ' 22 21, and
gN
Rg ' (2.183)
μ0 r1 l "

is the reluctance of the airgap between 21 and 22. By Gauss’ law, the flux Mg entering the pole-cap is
equal to the flux leaving it, and this comprises the flux Mm through the magnet plus the flux My passing
out through the bridges. (My will often be negligible). Thus
Mg ' My P m ( u1 u0 ) (2.184)
Page 2.62 SPEED’s Electric Machines

where Pm is the effective permeance of the magnet,


μ0μrecAm
Pm ' . (2.185)
LM

Am is the pole area of the magnet, and the permeance Pm may be augmented by a leakage factor to allow
for the additional flux path to either side of the magnet. Substituting these relationships into eqn.
(2.182) with u0 ' 0, we obtain an expression for the undetermined magnetic potential u1 :

sin 22 sin 21
û s Rg My
" (2.186)
u1 ' .
1 PM Rg
The field distribution is now completely determined. It has the form shown in the graph in Fig. 2.73,
following the sinusoid of the ampere-conductor distribution but depressed by the effect of the “floating”
magnetic potential u1. Note that because of the constant bridge flux My, u1 is not proportional to the
current, so the synchronous reactance becomes a function of current even when the rest of the iron is
considered infinitely permeable. However, the bridge flux is usually small and it is neglected here.
The synchronous reactance is associated with the fundamental component of stator flux, which can be
computed from the fundamental component of Bg(2). By Fourier’s theorem this is
4 22
Bg1 ' Bg ( 2 ) cos 2 d 2
B 21
(2.187)
4 μ0 û s " 21 22
' " sin " cos (21 22 ) 2 u1 sin cos
B gN 2 2 2

The fundamental stator flux per pole is then


Bg1 D l
M1 ' ' Bg1 D l, (2.188)
p
given that the analysis is for p ' 1 (2 poles). The voltage drop across the synchronous reactance is
2B
XI ' kw1 Tph M1 f V rms (2.189)
2
and the ampere-conductor distribution is related to the current by

4 kw1 Tph 3
û s ' × × I 2 (2.190)
B 2 2
assuming a 3-phase machine. From these equations the synchronous inductance can be deduced as

m μ0 (kw1 Tph)2 D l
L ' × ', (2.191)
B p 2 gN
where m is the number of phases and p (the number of pole-pairs) has been reinstated, and
Bg1
' ' , (2.192)
μ0û s / gN
which can be calculated directly from eqn. (2.187). Eqns. (2.191) and (2.192) can be compared with eqns.
(2.167) (2.169). They give the contribution of one rotor section (Fig. 2.73) to the synchronous inductance.
They can be used for both the d- and the q-axis, by setting appropriate values for 21 and 22 and adding
the contributions of all relevant sections to get the total unsaturated synchronous inductance.
Symmetric rotors with n > 1 have little or no saliency with respect to the fundamental MMF distribution,
but a strong magnet and any saturating bridges may bias the field to induce a degree of saliency.
Brushless permanent-magnet machines Page 2.63

Synchronous reactance of claw-pole machine


z

uN y u
h
Central
plane u0 y u0 h !g
O 2

wt ut

F
g tan F g

wv s wt

J ClawPole_LdLq.wpg

Fig. 2.74 Coordinates for calculation of Ld and Lq

The developed diagram Fig. 74 shows the coordinates and dimensions for the analysis leading to the
synchronous reactances Ld and Lq. The basic dimensions (measured in length units) are as follows: h
is the axial length of one claw; F is the angle of inclination of one side of the claw, measured in
mechanical radians, and 2wt is the width of the claw at its tip (which may be chosen more or less freely).
Together these define the azimuthal span of the inclined edge,
s ' h tan F . (2.193)
The width of the “valley” is 2wv, where
wv ' J ( wt s). (2.194)

At the axial position z = (h g)/2, the half-width of the centre pole is ut, and if J is the pole-pitch,

ut ' J ( wv g tan F ) , (2.195)


The lateral gap y measured in the azimuthal direction between the inclined edges is

y ' wv wt g tan F . (2.196)


The half-width at z = 0, that is, at the central plane, is given by

1 (h g) (u uN )
u0 ' (J y ) ' wt tan F ' . (2.197)
2 2 2
At any axial position z between (h + g)/2 and +(h g)/2, the half-width u of the centre pole varies
linearly with z from s + wt at (h + g)/2 to wt at +(h g)/2. Between these limits

2z
u ' u0 (u wt ) ' u0 z tan F . (2.198)
h g 0
At the same axial position z the half-width of the neighbouring pole is
2z
uN ' u0 (u wt ) ' u0 z tan F . (2.199)
h g 0
Page 2.64 SPEED’s Electric Machines

B0
z>0
u < uN

2u
B ! uN
0 u B
2 2uN

B0
z=0
u = uN

2u0
B !u0
u0 B
0
2 2u 0

B0
z<0
u > uN

2u
B !uN
u B
0
2 2uN

ClawPole_LdLq_Sine.wpg

Fig. 2.75 d-axis flux distributions produced by stator current

The pole-pitch J corresponds to B electrical radians, so each of the azimuthal dimensions has a
corresponding angle obtained by multiplying it by p/R, where R is the rotor surface radius and p is the
number of pole-pairs. This angle can be expressed per unit of the pole-pitch by dividing it by B. For
example the per-unit electrical angle corresponding to the half-width u0 at the central plane is

u0 p
"0 ' . (2.200)
RB

The pole-arc varies from a maximum (s + wt) at z = (h + g)/2 to a minimum wt at z = (h g)/2, giving
p p
"max ' ( s wt ) and "min ' wt . (2.201)
RB RB

We can now consider the airgap flux distribution produced by a sinudoidal distribution of stator
ampere-conductors aligned with the d-axis, that is, the polar axis Negecting fringing, it will have the
form shown in Fig. 75 at three locations along the axial length. The middle graph is at z = 0, that is, on
the central plane, and it is perfectly symmetrical. The lower graph is at a position z < 0, that is, in the
lower part of Fig. 74 where the width u of the central pole is greater than the width uN of the
neighbouring poles: u > uN. The upper graph is at a position z < 0, that is, in the upper part of Fig. 74
where the width u of the central pole is less than the width uN of the neighbouring poles: uN > u.
Brushless permanent-magnet machines Page 2.65

2B0
B1
z=0

B0

2u0

u0 B
2
0
2B0

B1

2u
B0

2uN

0 u uN B
2
z>0
combined
z<0

ClawPole_LdLq_Sine_Doubled.wpg

Fig. 2.76 Combined d-axis fluxes from both ends of claw pole

The “off-centre” distributions are asymmetrical and contain even-order harmonics, but the distribution
at z on a N pole can be combined with the distribution at +z on a neighbouring S pole to produce the
composite flux distirbution shown in Fig. 76. The even-order harmonics cancel, while the odd-order
harmonics combine, leaving a waveform suitable for Fourier analysis to find the fundamental
component of the d-axis armature-reaction flux as described earlier in SEM-2. It is actually simpler,
and entirely equivalent, to obtain the result directly by averaging the synchronous inductance
coefficient 'd over z from (h + g)/2 to +(h g)/2, assuming that the stator stack length Lstk is equal to
(h + g). At any position z with per-unit pole-arc ",

k1 k" d gN
'd ' k1ad ' , (2.202)
1 Pm Rg gdNN

where Pm, Rg, gN, gNN, k1, k1ad and k"d have all been defined earlier as well as in references [1] [3]. The
k coefficients are

sin " B 4 "B sin " B / 2


k1ad ' " , k1 ' sin , and k" d ' , (2.203)
B B 2 "B/2

and the integration is best done numerically.


Page 2.66 SPEED’s Electric Machines

B0
d
z<0
u < uN

0 B! uN
u B
2u 2
2uN

B0
z=0
u = uN

0 B! u 0
u0
B
2
2u 0 2u 0

B0

z>0
u > uN

0 B! uN
u B
2

2u 2uN

ClawPole_LdLq_Sine_q.wpg

Fig. 2.77 q-axis flux distributions produced by stator current

Finally the d-axis synchronous inductance is computed as

3 μ0 D Lstk
Ld ' 2
(kw1 Tph) 2 LF . (2.204)
p B gdNN

For the q-axis the process is similar. The flux distributions at different axial positions z are shown in
Fig. 77, and the expressions for Lq is the same as for Ld except that gdNN is replaced by gqN ' 'q gN where

sin " B
'q ' k1aq ' " . (2.205)
B

As with 'd, 'q is to be averaged between the limits "max and "min.
Brushless permanent-magnet machines Page 2.67

Fig. 2.78 Coefficients used to calculate 'd and 'q vs. "

Fig. 78 shows the various coefficients used in calculating 'd and 'q versus the per-unit pole-arc ". The
curves labelled with numbers on the right are of 'd, and the numbers are of PmRg. For a salient-pole
wound-rotor machine PmRg is infinite and 'd = k1ad. For an unsaturated wound-rotor synchronous
machine with no magnets, with a typical per-unit pole-arc " = 0@7, the ratio k1ad/k1aq = 0@958/0@442 = 2@17,
and this is the “basic saliency” for the fundamental armature reaction before leakage inductance
components are added. Although higher values of saliency ratio k1ad/k1aq are obtained for this machine
at smaller values of ", such narrow poles are not practical because of the reduction of main flux.
Permanent-magnet machines are often considered to have naturally inverse saliency 'q > 'd, but the
chart shows that this condition is met only if PmRg is relatively low (say, < 1) and " is relatively high
(say, > 0@9). For example, with PmRg = 0@2 and " = 0@9, 'q/'d = 0@802/0@266 = 3@02.
Page 2.68 SPEED’s Electric Machines

Summary—phase inductances
In nonsalient-pole motors, Lph and Mph are constant and are calculated from
Lph ' Lg Lslot Lend ; Mph ' Mg Mslot . (2.206)
The airgap components Lg and Mg can be calculated by summing the self- and mutual flux-linkages of
each coil due to current flowing in every coil, preferably using a method based on Hague’s analysis
discussed earlier, which makes it possible to account for any winding distribution. This procedure
includes the differential leakage reactance in Lg, that is, the reactance associated with all space-
harmonics of the armature-reaction field except the fundamental.
In salient-pole motors Lph and Mph vary with rotor position. For analysis by dq-axis theory the
variation is assumed to follow eqns. (2.157). The leakage inductances LF and MF must include the
differential leakage components Ldiff and Mdiff respectively, and these are estimated using eqn. (2.173).

Summary—synchronous inductances
In general the synchronous reactances are calculated by eqns. (2.170). They are used in the phasor
diagram. In nonsalient-pole motors, ideally Xd ' Xq. However, there may be a slight difference between
Xd and Xq, even in surface-magnet motors, if the relative permeability of the magnet is slightly higher
than 1 and the magnet is not rotationally symmetrical. In salient-pole motors Xd and Xq are unequal,
usually with Xq > Xd, and in embedded-magnet motors they can vary significantly as a function of
current. The component inductances Lg0 and Lg2 can be recovered using eqn. (2.172). The maximum
and minimum values of the phase inductance can be obtained with eqn. (2.158), and the maximum and
minimum values of the line-line inductance with eqn. (2.216).

Inductance of particular connections of 3-phase windings

Fig. 2.79 Particular connections of a 3-phase wye-connected winding. These connections can be used for static measurement of
Ld and Lq as described on p. 2.70.

Fig. 2.79 shows two particular connections of a three-phase wye-connected winding that occur
frequently during the operation of an inverter-fed motor. It is therefore of interest to calculate the
"terminal" inductance Q/I as a function of rotor position 2.
For the left-hand connection (a) the current I flows through phase a and divides equally between phases
b and c, so that ia ' I, ib ' ic ' I/2. The flux-linkage in phase a is

1
Ra ' Laa ( Lab Lca ) I (2.207)
2
Brushless permanent-magnet machines Page 2.69

while 1
Rb ' Lab ( Lbb Lbc ) I . (2.208)
2
Substituting from eqns. (2.157) and rearranging, we get

3 3 1 2B
Q ' Ra Rb ' LF MF Lg0 Lg2 cos 2 2 sin 22 I. (2.209)
2 2 3 3
When 2 ' 0 this reduces to

3 3 3
Q ' LF MF Lg0 Lg2 I ' Ld I , (2.210)
2 2 2
after comparing with eqns. (2.165) and (2.166). By the same process it is shown that when 2 ' ±B/2,

3 3 3
Q ' LF MF Lg0 Lg2 I ' Lq I . (2.211)
2 2 2
The inductance Q/I of the left-hand connection in Fig. 2.79 varies between 3/2 Ld and 3/2 Lq as the rotor
rotates. At a general position 2 it is given by eqn. (2.209). Note that with ia ' I, ib ' ic ' I/2, Rc ' Rb.
The right-hand connection in Fig. 2.79(b) is the line-line connnection with ia ' 0, ib ' I, and ic ' I. Then
Rb ' ( Lbb Lbc ) I and Rc ' ( Lbc Lcc ) I (2.212)
and

3
Q ' Rb Rc ' 2 LF MF Lg0 Lg2 cos 22 I . (2.213)
2
When 2 = 0 this reduces to

3
Q ' 2 LF MF Lg0 Lg2 I ' 2 Lq I . (2.214)
2
after comparing eqns. (2.165) and (2.166). By the same process it is shown that when 2 ' ±B/2,

3
Q ' 2 LF MF Lg0 Lg2 I ' 2 Ld I . (2.215)
2
Therefore the inductance Q/I of the connection in Fig. 2.79(b) varies between 2 Lq and 2 Ld as the rotor
rotates. At a general position 2 it is given by eqn. (2.213). From eqns. (2.162) with (2.161), (2.165) and
(2.166),
LLL[q] ' 2 Lq and LLL[d] ' 2 Ld , (2.216)
in agreement with eqns. (2.214) and (2.215).
Eqns. (2.161), (2.209) and (2.213) show that the terminal inductance presented to the drive varies with
rotor position even when the machine is perfectly sinewound. This underlines the fact that the constant
synchronous inductances Ld and Lq exist only in the synchronously rotating frame of reference fixed
to the rotor. In general there will be slight differences between the values of Ld and Lq obtained with
the two connections in Fig. 2.79, if the inductance harmonics are significant and the windings are not
perfectly sine-distributed.
Eqns. (2.210), (2.211), (2.214) and (2.215) show that it is possible to measure Ld and Lq directly with the
rotor locked in a fixed position, as described below.
Page 2.70 SPEED’s Electric Machines

Fig. 2.80 Jones' bridge circuit for measuring flux-linkage and inductance.

Measurement of flux-linkage and inductance


The relationship between flux-linkage Q and current I in a machine winding — or the inductance L '
Q/I — should be measured by a DC method such as the Jones bridge, which uses an integrating
voltmeter to measure flux-linkage Q directly, [18]. See Fig. 2.80.

Self inductance
The inductance to be measured is L, and R is its internal resistance. It is connected in a bridge circuit
which is supplied with DC current via a reversing switch S. The detector is an integrating voltmeter
or fluxmeter. The bridge is balanced for DC, and then the switch S is opened (or reversed). For the
right-hand branch we have
di
v ' (R R2 ) i L (2.217)
dt

and the voltage across R2 is therefore


R2 di
v 2 ' R2 i ' v L . (2.218)
R R2 dt
Across R3 we have
v 3 ' v R3 / ( R3 R4 ) (2.219)
and the voltage across the detector is
v b ' v2 v3 . (2.220)
If the bridge is balanced we have
R3 R2
' (2.221)
R3 R4 R R2

Substituting these equations into vb, we get


R2 di
vb ' L . (2.222)
R R2 dt
Brushless permanent-magnet machines Page 2.71

The integrating voltmeter therefore reads


4 R2 0 R2
Q ' vb dt ' L di ' LI, (2.223)
0 R R2 I R R2

whence

R Q
L ' 1 . (2.224)
R2 I

The inductance L is the "total" inductance, that is, the ratio of flux-linkage to current Q/I. This quite
distinct from the "incremental" inductance dQ/di, which is what would be measured by a "small-signal"
instrument (such as an RF bridge). Methods that use "small signal" measurements, even in the presence
of DC bias current, measure the incremental inductance. This is of interest for power electronic circuit
design, because it determines the relationship between chopping frequency and current ripple; but the
inductance often varies as a result of saturation and such measurements are of limited use especially
when they are conducted at low current levels. Measurements with high-frequency bridge instruments
are also unreliable, not only because of the low current level but also because the high frequency
renders them susceptible to eddy-current effects.
Jones [18] points out that induced currents in shorted circuits coupled to the inductance L contribute
nothing to the voltage integral in eqn. (2.223), so the Jones bridge method measures the true DC
inductance without having to remove or decouple these coupled circuits. This is extremely useful in
electrical machines where coupled short-circuited loops exist in the laminations, frame, etc. —
anywhere eddy-currents might flow.

Mutual inductance
The bridge is balanced with the desired current I in the "primary" and J in the "secondary". When J
is switched off, the voltage in the primary is induced by mutual coupling instead of self-coupling and
so by analogy with eqn. (2.224), M is given by

R Q
M ' 1 . (2.225)
R2 J

In both the L and M measurements, the switches S and T can be reversed instead of being simply
opened, and in that case Q should be replaced by Q/2 in eqns. (2.224) and (2.225).

Static measurement of Ld and Lq


The Jones bridge method can be used to measure Ld and Lq directly using the connections shown in Fig.
2.79, with the rotor locked in the appropriate position, (2 ' 0 in Fig. 2.69).
Page 2.72 SPEED’s Electric Machines

Getting inductance from finite-element calculations

Fig. 2.81 Calculation of flux-linkage from vector potential

Most finite-element programs claim to be able to calculate stored field energy W, (and coenergy),
suggesting that inductance can be deduced from the formula W = ½LI 2, if the current I is known.
However, if there is any saturation the meaning of L obtained from this formula becomes unclear,
because it relies on the assumption that L does not vary with current. Moreover, as we have seen in
§2.17, the stored energy in permanent magnets introduces another ambiguity.
A more rigorous approach is to use the vector potential A directly, with the equation

N ' I A @ dl (2.226)

in which N is the flux linking the contour along which A is integrated. In 2-D problems, the flux N
linking a coil (per metre of axial length) is given by N ' Ac1 Ac2, where Ac1 and Ac2 are the vector
potential values at the coilside positions, Fig. 2.81. If there is a complete winding with coilsides in
different locations, the method can be extended by summing the fluxes with appropriate polarities
according to the direction of the conductors. If the coils are in series carrying current I, and all have
Nc turns, the inductance is L ' Nc EN /I. This inductance is the total inductance, not the incremental
inductance. The method is simple to implement because it uses point values of vector potential. It is
represented mathematically by eqn. (2.455) on p. 132.
Incremental inductance — For calculations relating to the power electronic circuit, it may be
important to know the incremental inductance dR/dI when the machine is fully fluxed with full
current, because this is the inductance presented to the current regulator. Using the A method
described above, the incremental inductance is given by )R/(I1 ! I2), where )R is the difference in
computed flux-linkage at two slightly different current levels I1, I2. Alternatively, the inductance can
be calculated with the W = ½LI 2 formula after freezing the permeabilities in the solution domain at the
end of a nonlinear solution at the required load point.
Synchronous inductance — The vector-potential method described above and in eqn. (2.455)
calculates the total flux linkage of a definite winding, and it cannot be used to resolve the flux-linkage
uniquely into separate “magnetizing” and “leakage” components. For a winding which is essentially
sinewound, a practical procedure is to use a finite-element calculation with the winding connected as
in Fig. 2.79(a) together with eqn. (2.210) or (2.211); or connected as in Fig. 2.79(b) together with eqn.
(2.214) or (2.215). End-winding inductances (not calculated in 2-D FE) must be added to these values of
Ld and Lq. Since these are generally small, approximate methods of estimation should suffice.
The values of Ld and Lq are subject to variation caused by saturation,and their effective values may also
vary with rotor position if the winding is not perfectly sine-distributed, or if the d- and q-axis currents
vary during one electrical cycle. In such cases the best thing to do is to calculate Ld and Lq from
analytical formulas (2.167), and then adjust them to force the corresponding analytical values of torque
to agree with those calculated by eqn. (2.453) on p. 132, and the analytical value of flux-linkages obtained
from eqns. (2.455) and (2.456). This process is called “loop matching” and is used in PC-BDC to deal with
quite difficult cases where the machine is heavily saturated and the windings are not sine-distributed,
yet the user still wants answers in terms of classical dq parameters.
Brushless permanent-magnet machines Page 2.73

Classification of motors according to their saliency


Table 2.3 shows a suggested classification of different types of permanent-magnet rotor into “salient-
pole” and “nonsalient pole” types.

Rotor type Inductance variation with 2 Salient-pole or nonsalient pole


Surface radial or parallel Constant nonsalient pole
Breadloaf Weakly variable Can be treated as nonsalient pole
Spoke Strongly variable salient pole
Exterior radial or parallel Constant nonsalient pole
IPM Strongly variable salient-pole
Inset magnet Variable salient-pole
TABLE 2.3
VARIATION OF INDUCTANCE WITH COMMON ROTOR TYPES

Classification of inductances
Because salient-pole motors are analyzed using the dq-axis equations, they require the calculation of
the synchronous inductances Ld and Lq and associated component inductances. Nonsalient pole motors,
on the other hand, are analyzed in direct phase variables and they require the actual phase inductances.
To clarify these differences, the inductance parameters can be classified into two groups labelled
“actual” and “synchronous”, as shown in Table 2.4.

Actual (phase) Lph Mph


SP = salient pole Lg (NSP) or Lgg (SP) Lslot Lendt
NSP = nonsalient-pole Mg (NSP) or Mgg (SP) Mslot Ldiff
LLL[d] LLL[q] LLL (NSP) or LLL0 (SP)
Lg0 Lg2 Laa[d]

Synchronous Ld Lq Laa[q]
Xd Xq LF
'd 'q MF
kw1 Xm0 XF
TABLE 2.4
CLASSIFICATION OF INDUCTANCEPARAMETERS

With salient-pole motors the inductances Lph, Mph, Lg and Mg are variable, but Lgg and Mgg are the
approximate values that would be obtained with a solid steel rotor of the same radius and airgap g.
These can be used in the calculation of ksg.
For time-stepping simulations, nonsalient-pole motors need not be sinewound, because their circuit
equations can be expressed in direct phase variables. Salient-pole motors, on the other hand, should be
sinewound if at all possible, to facilitate the solution of their circuit equations either by the phasor
diagram, or by time-stepping simulation in dq axes.
Page 2.74 SPEED’s Electric Machines

2.12 BASICS OF SINEWAVE OPERATION


In SEM-1 we considered the factors affecting the choice of the number of phases in an AC system, and
derived some of the standard relationships between power, voltage, and current. Here we consider the
power and torque associated with the generated EMF and the current in the sinewave motor and drive.

Single-phase machine
Consider a single-phase machine that has a sinusoidal time-waveform of generated voltage or EMF,

e ' 2 E sin T t (2.227)

and suppose that the current waveform is also sinusoidal:

i ' 2 I sin ( T t (). (2.228)

The EMF and current are not in phase unless ( ' 0. A positive value of ( means that the current is
leading the EMF. For the moment we will ignore the resistance and inductance of the windings.
The instantaneous electromagnetic power pe is the product ei, which is
pe ' e i ' E I [ cos ( cos ( 2 T t ()]. (2.229)
The sin-sin product becomes the sum of two
cosine terms, one of which is oscillatory with a
frequency 2T, and the other is constant with a
value cos (, which is effectively the power-
factor angle between E and I. The constant
term P ' EI cos ( is the average power. The
double-frequency oscillatory power has a
constant amplitude EI.
Let us postulate an ideal energy-conversion
process represented by the equation
e i ' T Tm , (2.230)
Fig. 2.82 Current i, EMF e, instantaneous power p , and
where T is the electromagnetic torque and Tm average power P in a single-phase circuit with I ' 1,
is the rotational angular velocity, equal to T/p, E ' 1, and ( ' 35E.
where p is the number of pole-pairs.17
The nearest approach to this in practice would be a single-phase permanent-magnet machine with a
sinusoidal airgap flux-distribution, very low resistance and inductance, and no conducting material
on the rotor, operating at constant speed Tm. The electromagnetic torque T is given by
ei EI
T ' ' [ cos ( cos ( 2 T t ( ) ] . (2.231)
Tm Tm
The average torque is proportional to cos (, but the double-frequency ripple has a constant amplitude.

2-phase machine
Now let us consider a 2-phase machine in which the phase EMFs are balanced:

e1 ' 2 E sin T t ; e2 ' 2 E cos T t . (2.232)

Suppose the currents are slightly unbalanced:

i1 ' 2 I sin ( T t (); i2 ' 2 I (1 g ) cos ( T t (). (2.233)

17
Note that Eqn. (2.230) is familiar from the theory of DC machines, which do not suffer the double-frequency oscillatory torque
because e and i are both constant. However, they are only constant by virtue of the frequency-changing action of the commutator
and brushes. In each coil in the armature of a DC machine, both the voltage and the current are alternating.
Brushless permanent-magnet machines Page 2.75

The currents are in phase quadrature, but i2 exceeds i1 by the factor (1 g). The power is
p e ' e 1 i1 e2 i2 ' E I [ (2 g) cos ( g cos (2 T t () ] (2.234)
When the phases are balanced g ' 0 and the double-frequency oscillatory component vanishes, while the
average component is exactly doubled (—in effect, multiplied by the number of phases). Again invoking
the ideal energy-conversion process of eqn. (2.230), the instantaneous torque at constant speed is then
pe 2EI
T ' ' cos ( , (2.235)
Tm Tm
which is constant. The elimination of the oscillatory torque constitutes an enormous advantage of the
balanced 2-phase motor over the 1-phase motor.

3-phase machine
A similar elimination of the ripple torque and power arises in a balanced 3-phase machine, for which
pe EI
T ' ' 3 cos ( (2.236)
Tm Tm
under balanced conditions. This is easily proved by summing the ei products when

e1 ' 2 E sin T t
e2 ' 2 E sin ( T t 2 B/3) (2.237)
e3 ' 2 E sin ( T t 2 B/3)
and

i1 ' 2 I sin ( T t ()
i2 ' 2 I sin ( T t ( 2 B/3) (2.238)
i3 ' 2 I sin ( T t ( 2 B/3) .

The addition of resistance and inductance in series with the EMFs makes no difference to the foregoing
arguments provided that the EMFs are kept distinct from the terminal voltages, and provided that the
inductance is constant. In the next section the effects of resistance and inductance under AC conditions
will be considered. Furthermore, the variation of inductance and the resulting reluctance torque will
be taken into account by means of the two-axis theory, also known as dq-axis theory.
Page 2.76 SPEED’s Electric Machines

The phasor diagram on open-circuit

Fig. 2.83 Phasor diagram on open-circuit

The simplest type of “drive” is a plain AC voltage source with 2 or 3 phases. In the steady state, a motor
operating from such a supply can be represented by its phasor diagram, provided that it is sinewound
or approximately so. Even before the motor is connected to the supply, the open-circuit phasor diagram
can be drawn as shown in Fig. 2.83. Since the motor is on open-circuit there is no current, and the
phasor of the generated EMF E leads the phasor of the fundamental flux-linkage Q1Md by 90E. The
fundamental flux-linkage Q1Md is the product of the effective number of series turns/phase kw1Tph and
the fundamental component of magnet flux in the airgap M1Md, where kw1 is the fundamental harmonic
winding factor and Tph is the number of series turns per phase.18 Thus if B1Md is the peak value of the
fundamental flux-density produced by the magnet on open-circuit,
kw1 Tph B1Md D Lstk
Q1Md ' kw1 Tph M1Md ' [V&s r.m.s.] (2.239)
2 p

and is centred on the d-axis. The subscript 1 refers to the fundamental space-harmonic component. The
phasor relationship between E and Q1Md is
E ' j Eq1 ' j T Q1Md (2.240)
where the subscript 1 emphasizes the fundamental, q the q-axis, d the d-axis, and M the magnet. The
90E phase lead comes from Faraday’s law, when the phasors are expressed as complex numbers, and it
is normal to consider the magnet flux-linkage phasor to be aligned with the d-axis while the generated
19
EMF is aligned with the q-axis.

Connection to an AC source
The connection of the motor to a balanced AC voltage source is shown in Fig. 2.84. If the motor is
nonsalient-pole it can be represented electrically by its generated EMF E, its phase resistance R and its
synchronous reactance Xs (which is equivalent to Xd in a salient-pole machine). The AC voltage source
is assumed to have a constant voltage V and no internal impedance. The phasor diagram for the circuit
in Fig. 2.84 is shown in Fig. 2.85. It shows that the supply voltage V comprises the series combination
of the volt-drops RI, jXsI, and E, which must be added “vectorially” because they are not in phase with
one another. The volt-drop RI is in phase with the current I and its phasor is parallel to the current
phasor, but the volt-drop jXsI is at right-angles to the current and leads it in phase because Xs is an
inductive impedance.

18
Because of the distribution of conductors the winding links only a fraction kw1 of the fundamental flux. It acts as a filter. To get
a sinusoidal EMF requires low winding factors kwn for the harmonic fluxes (n > 1) and a value of kw1 close to 1.
19
The phase lead results from the use of the “sink” convention in which e = dR/dt rather than !dR/dt. This is appropriate for
motors, but less intuitive for generators. However, permanent magnet motors are vastly more common than generators.
Brushless permanent-magnet machines Page 2.77

Fig. 2.84 Circuit diagram of one phase of nonsalient pole brushless PM motor, with AC voltage supply

Fig. 2.85 Phasor diagram of nonsalient-pole machine: motoring operation

The angles shown in Fig. 2.85 are important. The angle N between V and I is the power-factor angle, and
the power factor is cos N.20 The angle ( between E and I is the “torque angle”, which is important in
drives which control the phase and magnitude of the current relative to the shaft position. The angle
* between V and E is called the “load angle” and may also be called the “torque angle” when the motor
is operating from an AC voltage source (i.e., without current control). In motoring operation, the
terminal voltage V leads the EMF E in phase, but in generating, E leads V.
Operation from an AC voltage source without electronics is useful for understanding the basic concepts
of the phasor diagram. It shows the correspondence between the PM brushless motor and the classical
synchronous machine, and it forms a useful basis for understanding the sinewave drive. In practice,
however, operation without electronics is relatively uncommon, being restricted to the so-called “line-
start” motors; (see p. 104). Unless they are fitted with damper windings on the rotor, brushless PM
motors operated in this way are not self-starting and have a tendency to “hunt” or oscillate about the
synchronous speed.21 This instability may also be observed if the motor is fed from an open-loop
voltage-source inverter. Even this is relatively rare, as most brushless PM motors have dedicated PWM
inverters with “shaft position feedback” that makes them inherently stable.

20
This definition of power factor is valid only if the voltage and current are sinusoidal.
21
Also known as “amortisseur” windings, these are usually in the form of a cast cage similar to the cage of an induction motor
rotor. See, for example, Jordan [1983], Miller [1984].
Page 2.78 SPEED’s Electric Machines

2.13 SYNCHRONOUS OPERATION OF SALIENT-POLE PM MOTORS


dq axes and salient-pole machines
The terms direct axis and quadrature axis refer to the two axes of symmetry of the magnetic field system
as defined by the field winding or excitation winding. In the DC machine the excitation winding is on
the stator and therefore the dq axes are fixed to the stator. In the AC synchronous machine the
excitation winding is on the rotor and therefore the dq axes are fixed to the rotor, Fig. 2.86. The d-axis
is the axis of symmetry centred on one rotor pole. Sometimes it is called the polar axis or field axis.
The q-axis is also an axis of symmetry and it is known as the interpolar axis. Because there are two axes
of symmetry, the rotor is said to have two-axis symmetry. In electrical terms the d- and q-axes are
orthogonal, i.e., separated by 90 “electrical degrees”. Indeed the definition of “electrical degrees” is
such that there are 180 electrical degrees between consecutive d-axes.

Fig. 2.86 Wound-field synchronous machine Fig. 2.87 Surface-magnet motor

Fig. 2.88 Interior-magnet motor (IPM)

In the PM machine the wound poles are replaced by permanent magnets, Figs. 2.87 and 2.88, but the
meaning of the d and q axes is unchanged. The interior-magnet motor in Fig. 2.88 is a salient-pole
machine with different inductive properties along the d- and q-axes, but the surface-magnet motor in
Fig. 2.87 is nonsalient-pole, being rotationally symmetric apart from the magnetization of the magnets
and the possibility of slight differences in permeability along the d- and q-axes. A further distinction
can be made between “strong” and “weak” PM rotors (“PM-assisted reluctance motors”), but this is
deferred to the section on line-start motors; see p. 106.
To understand steady-state operation at synchronous speed, we need to understand the phasor diagram,
Fig. 2.89. This is slightly more complex than the one in Fig. 2.85, in that it is drawn for a salient-pole
machine with Xd different from Xq. The voltage drop Xs I is replaced by two separate voltage drops XqIq
and XdId, aligned along the d and q axes. Fig. 2.89 shows only the RMS values of the phasors: in other
words, the complex phasor notation is not used. As an additional simplification, resistance is neglected.
Brushless permanent-magnet machines Page 2.79

Fig. 2.89 Phasor diagram of salient-pole motor, including flux-linkages

In the steady-state the RMS voltage V in each phase is related to its RMS flux-linkage Q by a simple
equation V ' TQ where T is the frequency in electrical rad/sec. We have seen that the phase angle of
this voltage is 90E ahead of the flux-linkage. For example, in Fig. 2.89, V is represented by an arrow
(called a phasor) which is 90E ahead of the arrow representing Q. Mathematically, the phasor value of
j(* + B/2)
V is the complex number V ' V e , the angle (* B/2) being the phase angle of V relative to the
reference axis (the d-axis). Both V and Q are sinusoidal quantities in time, and both are represented
in the diagram by their RMS values, while the continual advance of their phase angles is represented
by the rotation of all the phasors at the angular velocity T rad/sec. During this rotation the phase
displacement between Q and V remains at 90E.
The phasor diagram is drawn for one phase, and it is tacitly assumed to be similar for the other phases:
in other words, balanced operation is assumed. The phasor diagram in Fig. 2.89 is split into two parts.
On the left are the electrical quantities, i.e., voltages and currents. On the right are the corresponding
magnetic flux-linkages. The separation into two parts makes the diagram clearer. The flux-linkage part
of the diagram is usually omitted, but it can be considered to have a physical reality, in that the flux
actually rotates in space, at an angular velocity of T elec. rad/s. The instantaneous physical orientation
of the flux is also implicit in the related concept of space vectors; but in Fig. 2.89 the phasors simply
represent time-varying sinusoidal quantities.
We can now examine how the entire phasor diagram is built up. In a permanent-magnet machine the
magnet flux links all the windings in turn, and gives rise to the flux-linkage Q1Md in each phase when
there is no current flowing. Corresponding to this flux-linkage is the “open-circuit” voltage E or Eq1,
which leads Q1Md in phase by 90E as we have seen in Fig. 2.85, just as V leads Q by 90E. In the phasor
diagram, the flux Q1Md is along the d-axis, and therefore E is along the q-axis. Note that the dq axes in
the phasor diagram on the left-hand side are really fictional axes defined in terms of the time phasor
diagram. However, since the time phasor diagram (of voltages and currents) and the space vector
diagram (of flux-linkages) both rotate synchronously in their respective coordinate systems, we tend
to blur the distinction and regard the dq axes as being the same for both. In common engineering
parlance, everyone takes this for granted and one would be thought pedantic if one continually
reiterated the distinction between them.
Page 2.80 SPEED’s Electric Machines

When current flows in the stator windings it creates an additional flux. To analyze this we first
“resolve” the phasor current into two components: Id along the d axis and Iq along the q axis. Noting
that ( is measured from the q-axis in the positive (counter-clockwise) direction in Fig. 2.89, we have
Id ' I sin ( ; Iq ' I cos ( . (2.241)
In terms of the complex phasor I, this is written

I ' Id j I q ' I e j (( B/2)


. (2.242)
The flux-linkage produced by Id is LdId, where Ld is the d-axis synchronous inductance. It is in phase
with Id and induces a voltage XdId which is 90E ahead of Id (i.e., parallel to the q axis). Xd ' TLd is the
d-axis synchronous reactance. Likewise the current Iq produces a flux-linkage LqIq and a voltage XqIq
which is parallel to the negative d axis, with Xq ' TLq the q-axis synchronous reactance. The total
voltage at the phase terminals is the sum of the component voltages E, XdId, and XqIq, added together
“vectorially” by means of the polygon formed by the respective phasors placed nose-to-tail. Similarly
the total flux-linkage is the vector sum of the component flux-linkages Q1Md, LdId and LqIq.
The phase flux-linkage can also be expressed as a phasor
Q ' Qd j Qq , (2.243)
where
Qd ' Q1Md Ld Id (2.244)
and
Qq ' Lq Iq . (2.245)
Qd and Qq are the d- and q- axis components of the flux-linkage per phase. Like all the other phasors,
they are scaled to their RMS values. In the d-axis the flux-linkage Qd has two components, Q1Md due to
the magnet and LdId due to the stator current component Id. In the q-axis there is no magnet flux but
only the armature-reaction component LqIq.

Torque
The time-averaged power associated with V and I is

Pe ' m Re {VI(} ' m ( Vd Id Vq Iq ) , (2.246)

where m is the number of phases. The concept of the ideal energy-converter in eqn. (2.230) on p. 74 can
now be extended to deduce the time-averaged electromagnetic torque Te as Pe/Tm or Pe/(T/p), where
p is the number of pole-pairs. Substituting from eqn. (2.243) in eqn. (2.246), we get
Te ' m p ( Qd Iq Qq Id ) . (2.247)
(This equation is derived more rigorously on p. 128ff). Alternatively,

mp
Te ' m p [ Q1Md Iq Id Iq ( Ld Lq ) ] ' [ E Iq Id Iq ( Xd Xq ) ] . (2.248)
T

This reveals two components of torque, a permanent-magnet alignment torque mpQ1MdIq and a
reluctance torque mpIdIq(Ld Lq). If there is no saliency Ld ' Lq, and the reluctance torque disappears.
If the magnet flux is constant the torque is proportional to Iq, and the torque constant k T ' Te/I is
constant as long as the controller maintains I ' Iq, which is sometimes called “quadrature control”.
In the interior-magnet type of PM brushless motor (IPM), usually Lq > Ld, so Id must be negative if the
reluctance torque is to be positive. Negative Id is in the demagnetizing or flux-weakening direction.
In general when there is saliency and Ld and Lq are unequal, the mix of permanent-magnet alignment
torque and reluctance torque can be adjusted by changing the phase angle of the current (() as well as
its magnitude. It should also be pointed out that in many salient-pole PM machines saturation reduces
the difference between Ld and Lq and reduces the reluctance torque. Moreover the very concept of
reluctance torque relies on the principle of superposition, so under saturated conditions the separation
of reluctance torque and alignment torque requires careful definition; see p. 162.
Brushless permanent-magnet machines Page 2.81

Fig. 2.90 Construction of required voltage locus as ( varies

Voltage
The voltage phasor V is obtained from the total flux-linkage Q by multiplying it by jT:

V ' Vd j V q ' V e j (* B/2)


' jTQ ' T Qq j T Qd ' Xq Iq j (E Xd Id) . (2.249)
When resistance is included, the voltage phasor V is given by
V ' R I Xq Iq j Xd Id jEq1
(2.250)
' (R Id Xq Iq) j (R Iq Xd Id Eq1 ) .

Substituting for Id and Iq from eqn. 2.241, we get


V ' ( R sin ( Xq cos ( ) I j [ (R cos ( Xd sin () I Eq1 ] ' Vd j Vq (2.251)

Voltage and current loci : the circle and ellipse diagrams


Eqn. (2.251) describes the locus of the voltage phasor required to force the current Id jIq into the
machine. Fig. 2.90 shows an example in which the phase angle ( varies while the RMS current I
remains constant, so that the tip of the current phasor traces a circular locus. The corresponding
voltage locus given by eqn. (2.251) is elliptical. It shows several interesting properties of the voltage
required from the inverter. For example, when the current has a demagnetizing component Id < 0, the
required voltage is reduced; this is known as flux-weakening. When ( > 90E, Iq < 0 and the torque
reverses, so the machine is generating and in this case V lags behind E; the tip of the voltage phasor
then lies on that part of the ellipse to the right of the q-axis.
j((+90E)
Current-limit circle — If I is equal to the rated or maximum current Im, the circle I ' I e defines
a boundary enclosing all permissible current phasors, giving rise to the circle diagram, Fig. 2.91. Im can
be limited by the inverter or the motor. A current-regulated drive normally has complete control of the
current waveform, and it can place the current phasor anywhere inside the circle, provided it has
sufficient voltage available to overcome the EMF and the impedance of the machine.
Voltage-limit circle — The voltage-limit circle is a current locus defining all possible currents that can
be obtained when the inverter voltage is limited, rather than its current. Let Vm be the maximum
available supply voltage per phase.22 Imagine that the drive is supplying this voltage to each phase of
the motor, such that the phase angle between Vm and E is * (Fig. 2.89). From the phasor diagram,

22
Vm is the fundamental harmonic component of the actual phase voltage.
Page 2.82 SPEED’s Electric Machines

Fig. 2.91 Current-limit circle and voltage-limit circle at the change-over speed; non-salient-pole motor with Xd ' Xq.

Vq E Vm cos * E Vd Vm sin *
Id ' ' ; Iq ' ' . (2.252)
Xd Xd Xq Xq

and Vm2 ' Vd2 Vq2 , (2.253)

so that ( Xq Iq ) 2 (E X d I d ) 2 ' V m2 . (2.254)

This is an ellipse in the plane of (Id, Iq); see Fig. 2.93 on p. 86. If Xd ' Xq, the voltage-limit ellipse becomes
a circle, centre ( E/Xd, 0), as in Fig. 2.91. Note that E/Xd is the short-circuit current, which is normally
a large current, so that point C normally lies outside the current-limit circle.

Variable speed
In eqn. (2.254), Vm is fixed; but the EMF E ' TQ1Md and the reactances Xd ' TLd and Xq ' TLq all increase
in proportion to speed, while the short-circuit current E/Xd remains constant. Consequently as the
speed increases, the voltage-limit circle or ellipse shrinks, while it remains centred on point C in Figs.
2.91 and 2.93. At certain values of ( the drive may not have sufficient voltage to maintain the current
Im, so the current becomes voltage-limited. This condition is associated with “saturation” of the
current-regulator, which starts to lose control of the current waveform.
Base speed or “corner” speed — For nonsalient-pole motors with Xd ' Xq there is a clearly defined
maximum speed at which rated current Im can be driven with ( ' 0, producing rated torque. This speed
is sometimes called the “corner” speed because it is the point on the torque/speed characteristic at
which the torque begins to fall. It is given by point Q in Fig. 2.91, where the voltage-limit circle is just
large enough to intersect the current-limit circle with Iq ' Im and Id ' 0. Therefore

( V m / X d )2 ' ( E / X d )2 I m2 . (2.255)

In this equation Xd and E are both proportional to speed or frequency T, but Im and Vm are fixed. If we
substitute E ' TQ1Md and Xd ' TLd we can re-arrange eqn. (2.255) to give the corner frequency:
Vm
TQ ' rad/sec . (2.256)
Q1Md2 ( Ld Im ) 2

The corresponding speed in rpm is NQ = TQ /p × 30/B.


Brushless permanent-magnet machines Page 2.83

Fig. 2.92 Circle diagram at four different speeds (non-salient-pole motor)

At speeds higher than NQ it is still possible to drive rated current Im into the motor, but not at the
maximum-torque angle (Tmax. We can therefore begin to formulate a control strategy for achieving
maximum torque at any speed, as follows:

Low speed Control current with ( = (Tmax

High speed Work along the intersection between the voltage-


limit ellipse and the current-limit circle
TABLE 2.5

As the frequency increases the voltage-limit ellipse shrinks and the intersection with the rated-current
circle moves along the arc QD. To illustrate this, Fig. 2.92 shows the conditions at four different speeds,
N1 < N2 (' NQ) < N3 < N4.
At low speed (circle 1) the voltage-limit circle completely encloses the current-limit circle, which means
that the current Im can be driven into the motor with any phase angle. In fact the current could be
increased up to the value OL, approximately twice Im, under the control of the current regulator.
As the speed increases the voltage-limit circle shrinks, until at NQ the maximum current that can be
driven along the q-axis is Im, circle 2. At a still higher speed circle 3 shows operation at P with I ' Im,
but the phase angle ( is advanced as shown, and the torque is reduced by the factor cos (.
Eventually a speed is reached at which the current Im can be driven only along the negative d axis,
circle 4. All the current is now used to suppress the flux (flux weakening), and none of it is available
to produce torque. The intersection is at point D and the torque is zero. To achieve this point the
current regulator must operate with a massive phase advance of 90E and with maximum current
reference.
Page 2.84 SPEED’s Electric Machines

It can be shown that the speed at which point D is reached is related to the corner speed by
NQ
' u 1 u2 (2.257)
ND
where u ' TQ Q1Md/Vm ' EQ/Vm, EQ being the value of E at the corner speed. For a solution to exist at
a positive speed, we must have
1
< u < 1 (2.258)
2
For example, if u ' 0@8, ND ' 5 NQ, but if u ' 0@9, ND ' 2@155 NQ. Alternatively suppose that the motor must
maintain maximum torque at speeds up to 3,000 rpm and be capable of just reaching 6,000 rpm. Then
ND/NQ ' 2 and according to eqn. (2.257), u must be no higher than 0@911.
The ratio of the speeds ND and NQ can be expressed in terms of the reactance or inductance of the motor.
Define the per-unit synchronous inductance as

Ld Im Xd Im
x ' ' . (2.259)
Q1Md E
The per-unit synchronous reactance is identical to x, and by equating CD to OP in Fig. 2.92 we get

ND x2 1
' . (2.260)
NQ x 1
Even with x ' 0@3 (a fairly high value for a surface-magnet motor), ND/NQ ' 1@491, which is quite a
narrow speed range above the corner speed. To maintain constant torque up to 3,000 rpm and be able
to reach 6,000 rpm (ND/NQ ' 2), eqn. (2.260) says that x must be at least 2@347, which explains why
additional series inductance extends the speed range above the corner speed. The additional inductance
makes the system more responsive to phase advance, but only at the expense of additional flux-linkage
that is in phase with the current and therefore does not contribute directly to the torque. Note that
eqns. (2.257) and (2.260) are independent and must both be satisfied.
Although eqns. (2.257) and (2.260) are an incomplete account of the variation of torque with speed, and
apply only to non-salient-pole (surface-magnet) motors, they show that phase advance is needed both
“to overcome the rising EMF” and “to compensate for inductance” as the speed increases.

Maximum torque and torque loci


Current-limited maximum torque — Noting that ( is measured from the q-axis in the positive (CCW)
direction in the phasor diagram, if we substitute eqns. (2.241) in equation (2.248) we get

Te ' m p [ Q1Md I cos ( I 2 sin ( cos ( ( Ld Lq ) ] . (2.261)

At any given current level I, we can differentiate this expression with respect to ( to find the value of
( which gives maximum torque. The result is

2
1 1 Q1Md Q1Md (2.262)
(Tmax ' sin 8
4 )Q )Q

where )Q ' (Ld Lq)I. Unfortunately (Tmax is not a fixed, but depends on the current. Moreover, Ld and
Lq both vary as a result of saturation, and this further complicates the problem of finding the optimum
value of (.
If there is no saliency )Q ' 0; then from eqn. (2.261) the phase angle that maximizes the torque is ( '
0: i.e., the current must be oriented in the q-axis ($ ' 90E) in phase with the EMF E. Phase advance ((
> 0) in a surface-magnet motor reduces the torque constant k T, which is defined as Te/I. This can be
seen in eqn. (2.261) (with Ld ' Lq). The reduction in k T is in the ratio cos (.
Brushless permanent-magnet machines Page 2.85

Voltage-limited maximum torque — With constant voltage Vm but variable phase angle * between
Vm and E, the torque can be calculated by substituting eqns. (2.252) in eqn. (2.261) to eliminate Id and
Iq. The result is

mp E Vm V m2 1 1
Te ' sin * sin 2 * . (2.263)
T Xd 2 Xq Xd

We can differentiate eqn. (2.263) to find the phase angle * which maximises the torque. After some
simplification the result is
1
*Tmax ' cos ( . ± .2 8 )/4 (2.264)
where
E / Vm
. ' . (2.265)
1 Xd / Xq
If there is no saliency, the angle which gives maximum torque is *Tmax = 90E. In principle this could
be used as the basis of a torque-maximizing control strategy, but the disadvantage is that the current
is uncontrolled, and its magnitude and phase will vary in a more complex manner as * varies.

Constant-torque loci
A constant-torque locus can be superimposed on the circle diagram, as in Fig. 2.93, by writing the torque
equation as
mp
Te ' IdN Iq ) X (2.266)
T

where E
) X ' Xd Xq and IdN ' Id . (2.267)
)X

This is a rectangular hyperbola whose asymptotes are the negative d-axis and a false q-axis which is
shifted to the right of the true q-axis by E/)X.
With high-energy magnets the constant-torque contours are more nearly horizontal, but with low-
energy magnets they have more curvature. Fig. 2.93 is drawn for a salient-pole motor with Xd < Xq. For
a nonsalient pole motor the torque loci are horizontal straight lines, with constant Iq.
The constant-torque loci in Fig. 2.93 are drawn for three torques T1 < T2 < T3. The middle one goes
through the point Q which we earlier associated with the corner-point for a nonsalient pole motor. With
saliency, however, it appears from the constant-torque loci that the torque can be increased by phase
advance between Q and P, with the current maintained at the rated value Im. Thus for example the
torque at point T is greater than it is at either P or Q, and in fact there is a maximum torque at any
given value of current, which is obtained with a definite phase-advance angle (Tmax.
The additional torque obtained with phase advance is reluctance torque, the second term in eqn. (2.261),
which comes from the saliency. This equation is re-arranged slightly as follows,
Te ' m p [ Q1Md I sin ( ( Ld Lq ) ] I cos ( . (2.268)
As ( increases from zero, the reluctance term I sin ( increases quickly while cos ( changes only slowly.
Although the alignment torque is decreasing, the reluctance torque is increasing at a faster rate, until
( reaches (Tmax. In terms of EMF and reactances,

mp
Te ' [E I sin ( ( Xd Xq ) ] I cos ( . (2.269)
T

where E is the RMS open-circuit EMF also written as Eq1.


Page 2.86 SPEED’s Electric Machines

Fig. 2.93 Constant-torque loci (salient-pole motor)

As we have seen, eqn. (2.262) can be used to determine the phase advance angle (Tmax which maximises
the torque for any given value of current. Having defined the per-unit synchronous reactance x for a
non-salient-pole motor, it is a simple matter to extend this to xd and xq, the per-unit synchronous
reactances in the d- and q-axes respectively, and put )x ' (xd xq).
If Im is the rated current, its per-unit value can be taken as 1, and if we use the per-unit EMF u as defined
earlier, eqn. (2.262) gives

2
1 1 u u (2.270)
(Tmax ' sin 8 .
4 )x )x

For example, suppose u ' 0@9 and xd ' 0@5 and xq ' 1@5. Then )x ' 1@0 and

2
1 1 0@9 0@9 (2.271)
(Tmax ' sin 8 ' 31@13E .
4 1@0 1@0

The phase advance can be used to achieve the same torque at a lower current, or the same torque at a
higher speed as the voltage-limit ellipse shrinks. In Fig. 2.93 the maximum torque is shown at point T.
It should be pointed out that the theory of the optimum torque angle has been developed here without
regard to the effect of resistance or other losses, particularly iron losses. In practice these parasitic
effects distort the idealized circle and ellipse diagrams and the idealized torque loci. In most cases
where the high-speed behaviour is important, this distortion is very significant, because every effort
must be made to maximize the capability as well as the efficiency of the drive system, and even small
distortions make a difference to the performance. Nevertheless, we will continue in the next section
with further “lossless” analysis of the torque/speed characteristic, because of the powerful insights
which can be gained into field-weakening and control strategy.
Brushless permanent-magnet machines Page 2.87

2.14 NORMALIZED FORM OF THE TORQUE/SPEED CHARACTERISTIC


The voltage and torque equations are often normalized to reduce the number of independent variables.
The per-unit equations bring out the essential nature of the machine in the simplest possible terms.
They also form a basis by which different machines can be compared.
With the saliency ratio defined as > ' Xq/Xd (normally > 1), neglecting resistance and losses, we have
Vd ' > Xd Iq ;
(2.272)
Vq ' T Q1md Xd Id ' E Xd Id .

The short-circuit current at any speed is determined by setting Vd ' Vq ' 0. Then Iq ' 0, and the short-
circuit current is entirely in the d-axis,
E
Id [ sc ] ' . (2.273)
Xd

Let this be the base current, I0. Let T0 (elec. rad/sec) be the base speed. Let the base voltage be the open-
circuit EMF at the base speed T0, that is, E0 ' T0 Q1md. Normalizing eqns. (2.272) to these base quantities,
with S ' T/T0 the per-unit speed, and writing per-unit voltages and currents in lower-case, we get
vd ' > S iq
(2.274)
vq ' S ( 1 id )

It is implicit in the choice of base current and base voltage that the per-unit synchronous reactance in
the d-axis is 1@0 at base speed and frequency, and therefore in the q-axis it is > : thus in general
xd ' S p.u. ; xq ' > S p.u. (2.275)
Also the per-unit EMF is e ' 1 at base speed and frequency, and in general it is given by
e ' S. (2.276)
In ordinary units the torque is given as a function of Id and Iq by eqn. (2.248), with p pole-pairs and m
phases. Also Id ' I sin ( and Iq ' I cos (. The base power for the whole machine is P0 ' m E0I0, so the
base torque is T0 ' P0/(T0/p), where (T0/p) is the base mechanical speed. If eqn. (2.248) is normalized
by dividing both sides by appropriate base values, we get the simple "current-limited" torque equation
J ' iq [ 1 (> 1 ) id ] , (2.277)
Substituting id ' i sin ( and iq ' i cos (, where i is the per-unit phase current, we get
J ' i cos ( [ 1 (> 1 ) i sin ( ] . (2.278)
Eqn. (2.277) shows that the reluctance torque is zero if > ' 1 (no saliency). In salient-pole PM brushless
machines normally > > 1, so id must be negative (i.e., demagnetizing) to get positive reluctance torque.
If we substitute eqns. (2.274) in eqn. (2.277), we get the "voltage-limited" torque equation

vd vq 1
J ' 1 1 . (2.279)
S S >
Substituting vd ' v sin * and vq ' v cos *, where v is the per-unit phase voltage, we get

v sin * v cos * > 1


J ' 1 . (2.280)
S S >
The second term in eqn. (2.280) is not the same as the reluctance torque in eqn. (2.277). (This is clear
when deriving eqn. (2.279) or (2.280) from eqn. (2.277)). The "reluctance" term in eqn. (2.279) or (2.280)
is positive only if vq < 0 which means that the flux in the d-axis has been reversed. The peak torque (vs.
*) thus occurs at a condition where the current is extremely high. To reach this point without
demagnetization, the straight part of the BH curve of the magnet must extend well into the 3rd quadrant,
even at full temperature; this point is experienced transiently during the final stages of start-up, with
little screening from the rotor cage.
Page 2.88 SPEED’s Electric Machines

The per-unit system based on E0 and I0 is defined without reference to the drive or its voltage/current
capability, since E0 and I0 are natural properties of the machine, independent of the drive. The base
volt-amperes per phase is the product E0I0. In practice most machines are thermally limited to a power
level far below 1 p.u. in this system, and are normally driven by inverters whose volt-ampere rating is
also well below 1 p.u. This is obvious from the fact that 1 p.u. current is the short-circuit current at rated
speed, and for most permanent-magnet machines this far exceeds the rated current. Such machines
usually have low per-unit reactances, which is another manifestation of this property.
Other analyses of the torque/speed characteristics of these machines sometimes use per-unit systems
in which the inverter current is chosen as the base current, and the inverter voltage as the base voltage:
see, for example, [21]. But here the "machine base" per-unit system is retained because it is not found
elsewhere in the literature and it is the simplest representation from the point of view of the machine.
If im (p.u.) is the maximum current available from the drive, we can express the current-limit circle as

id2 iq2 ' im2 . (2.281)


The voltage-limit ellipse (eqn. (2.254)) is expressed in per-unit and in terms of id and iq as

>2 iq2 (1 i d )2 ' v m N 2 (2.282)


where vmN ' vm/S and vm (p.u.) is the maximum voltage available from the drive. The centre of the
voltage-limit ellipse is at id ' 1, iq ' 0. This is the short-circuit condition, point C in Fig. 2.93. As
already anticipated, this point will usually lie outside the current-limit circle, as it does in Fig. 2.93.
If we write
idN ' 1 id (2.283)

then the voltage-limit eqn. (2.282) becomes

vm 2
>2 iq2 idN 2 ' vmN 2 ' . (2.284)
S2
This makes it easy to locate certain key points on the voltage-limit ellipse (Fig. 2.93). When id ' 1, idN
' 0 and iq ' ±vmN/> ' ±v m/S>, which locates the points Y1 and Y2. They define the maximum q-axis
current that can possibly be supplied within the voltage limitation of the drive at the per-unit speed S.
When iq ' 0, idN ' ±vmN ' ±vm/S, which locates the points X1 and X2. They define the maximum d-axis
current that can possibly be supplied, in both the magnetizing and demagnetizing directions. All four
points migrate towards C as the speed increases. Also note that the ratio of the major axis to the minor
axis is equal to >, the saliency ratio. In Fig. 2.93 this is > ' 1@43.
We can calculate S for any operating condition if we rearrange eqn. (2.284) to give S directly:

vm
S ' . (2.285)
>2 iq2 (1 id ) 2

At low speed the voltage-limit ellipse is large enough to enclose the current-limit circle, and then the
current can have any magnitude up to 1 p.u. oriented at any angle. As the speed increases it first
reaches a value SQ at which a specified current can only just be supplied along the q-axis, as shown by
point Q in Fig. 2.93. The diagram happens to be drawn with Im ' 0@78 p.u. (' OD/OC) and a saliency
ratio of 1@43 (X1X2/Y1Y2). With id ' 0 and iq ' 0@78 and > ' 1@43, eqn. (2.285) gives SQ ' 0@668 vm. If we need
to maintain quadrature control up to the rated speed then SQ ' 1@0 and this requires the drive to have
a maximum voltage of 1/0@668 ' 1@5 p.u., that is, 1@5 times the motor EMF at the rated speed. The higher
the saliency ratio >, the lower the maximum speed at which quadrature-axis control can be maintained.
Brushless permanent-magnet machines Page 2.89

For a nonsalient pole machine (> ' 1) with a drive having a maximum current Im,

vm
SQ ' . (2.286)
1 > 2 Im 2

It is obvious from this that the drive voltage must exceed the EMF at rated speed if quadrature control
is to be maintained up to rated speed, since vm > 1 if SQ ' 1.
Let (Tmax be the phase advance angle that maximizes the torque. By differentiating eqn. (2.278) with
respect to (, we get
2
1 ± 1 8 i 2(> 1)
sin (Tmax ' (2.287)
4i(> 1)
and then idTmax ' i sin (Tmax and iqTmax ' i cos (Tmax. We can now find the maximum speed at which
(Tmax can be maintained. Using eqns. (2.274) and (2.284), the result is
vm
STmax ' . (2.288)
> 2 iq Tmax2 (1 id Tmax) 2

Taking the example from Fig. 2.93 with > ' 1@43 and i ' 0@78, eqn. (2.287) gives (Tmax ' 16@382E and then
eqn. (2.288) gives STmax ' 0@755 vm. To achieve the optimum phase angle at rated speed (STmax ' 1), the
inverter voltage must be vm ' 1/0@755 ' 1@324 p.u., that is, 1@324 times the back EMF at rated speed. In this
condition the component currents are id ' 0@78 sin 16@382E ' 0@220 p.u.; iq ' 0@78 cos 16@382E ' 0@748 p.u.,
j106@382E
and i ' 0@78 e p.u. The component voltages are vd ' S> iq ' 1@43 × 0@748 ' 1@070 p.u. and vq ' S
(1 id) ' 0@780 p.u. The per-unit magnitudes of the reactive voltage drops are Sxdid ' 1@0 × 1@0 × 0@220 '
0@220 p.u. and Sxqiq ' 1@0 × 1@43 × 0@748 ' 1@070 p.u. The phasor diagram is drawn to scale in Fig. 2.94.
From eqn. (2.277) the per-unit torque is 0@748 × [1 (1@43 1) × ( 0@220)] ' 0@819 p.u. The power factor can
be calculated from the angle between the terminal voltage and the current, and is 0@793 lagging.

Fig. 2.94 Phasor diagram for the example with > ' 1@43, i ' 0@78. (a) Operation at rated speed with 1@324 p.u. voltage and maximum
torque obtained with the optimum phase advance ((Tmax ' 16@382E). (b) Operation at 1@4 p.u. speed with the same voltage
and current with maximum torque obtained at the intersection of the voltage-limit ellipse and the current-limit circle.
Page 2.90 SPEED’s Electric Machines

Control strategy for maximum torque over a wide speed range — When the drive has sufficient
voltage margin over the EMF, the rated current can be oriented at the optimum phase angle (Tmax from
zero speed up to STmax. At higher speeds the torque can be maximized by working at the intersection
of the voltage-limit ellipse and the current-limit circle, which is obtained by solving the simultaneous
2 2 2
equations (2.281) and (2.282). If we substitute iq ' im id from eqn. (2.281) into eqn. (2.282),

(1 > 2 ) id2 2 id (1 > 2 i m2 v mN 2 ) ' 0 , (2.289)


which is a quadratic equation that can be solved for id. There can be intersections at one or at most two
values of id, with two values of iq at each one, and the solution is given by

1 K 1 (>2 1) ( 1 >2 i m2 v mN 2 )
id ' . (2.290)
>2 1
A necessary (but not sufficient) condition for intersection is that
1
v mN 2 # 1 > 2 i m2 . (2.291)
2
> 1
If vmN exceeds this critical value, the ellipse encloses the current-limit circle and there is no constraint
on the current or its phase angle.
As an example, suppose the speed in the previous example is increased from 1@0 to 1@4 p.u.: then S ' 1@4.
If the terminal voltage limit remains the same, vmN ' 1@324/1@4 ' 0@946. With im ' 0@78 p.u. the right-hand
2
side of eqn. (2.291) is 3@201, which exceeds (0@946) and therefore we expect at least one intersection
between the voltage-limit ellipse and the current-limit circle. From eqn. (2.290),

1 K (1 (1@432 1) (1 1@432 × 0@782 0@9462)) 1 K 1@552


id ' ' ' 2@443 or 0@529 . (2.292)
1@432 1 1@045
The first solution is not acceptable because it exceeds the stated current (0@78 p.u.), but the second
2 2
solution is viable with iq ' /(0@78 0@529 ) ' 0@574 p.u.
Although the current is the same as it was at 1 p.u. speed, the demagnetizing or flux-weakening
component id has increased from 0@220 p.u. to 0@529 p.u. The torque is again given by eqn. (2.277) as J '
0@574 × [1 (1@43 1) × ( 0@529)] ' 0@443 p.u., which is only 54% of the value at rated speed. However, the
per-unit power is 0@443 × 1@4 ' 0@620 p.u., which is 75@7% of the power at rated speed.
The component voltages are vd ' S> iq ' 1@4 × 1@43 × 0@574 ' 1@149 p.u. and vq ' S (1 id) ' 1@4 × (1
0@529) ' 0@659 p.u. The per-unit magnitudes of the reactive voltage drops are Sxdid ' 1@4 × 1@0 × 0@529 '
0@741 p.u. and Sxqiq ' 1@4 × 1@43 × 0@574 ' 1@149 p.u. Of course the EMF is increased to 1@4 p.u. The phasor
diagram is drawn to scale in Fig. 2.94.
The maximum theoretically attainable speed with a lossless motor is when the current is at its rated
value and entirely in the negative d-axis, giving the maximum possible flux-weakening effect. The
torque just reaches zero with iq ' 0. From eqn. (2.284), with id ' im and ( ' 90E,
vm vm
S ' ' . (2.293)
1 id 1 im
If the drive has sufficient current capacity im to make im ' 1 or greater — in other words, if it can supply
at least the natural short-circuit current of the machine — then the theoretical maximum speed is
infinite. In practice this would usually require a drive with a current capacity much bigger than that
of the motor. Even in the worked example with im ' 0@78 p.u., the drive is very large. With this example,
assuming the same terminal voltage of 1@324 p.u., the maximum theoretical speed is 1@324/(1 0@78) ' 6@02
p.u. In practice, losses would limit the speed to a substantially lower value.
Even though losses and saturation distort the simple analysis provided here, the underlying theory
helps to explain some of the natural limitations in the torque/speed characteristic, as well as the
relationship between the motor's current and voltage ratings and those of the drive.
Brushless permanent-magnet machines Page 2.91

2.15 HISTORY OF BRUSHLESS PM MOTOR DRIVES


The permanent-magnet synchronous machine was certainly known in the early 1950's, [22,23]. Although
most permanent-magnet machines at that time were generators, motors were also manufactured [24],
and the classical two-axis theory was used to analyze them. The motors were “line-start” motors
supplied directly from the AC mains, without electronics. In the early 1970's the discovery of high
energy cobalt-samarium magnets gave new impetus to the development of permanent-magnet AC
motors. Lower-energy ferrite magnets were already used in DC brush-type motors and improvements
in these magnets also encouraged new work in AC line-start machines, notably by Brown Boveri (Isosyn
motor, 1978) and Reliance Electric (1979). Some of these motors were used with inverters, but they were
still “line-start” motors without shaft position sensing: therefore they were not self-synchronous in the
sense that modern servo-motors and brushless DC motors are.
The so-called “brushless DC” motor emerged at this time (mid-1970's), notably from Papst. This system
is equivalent to a DC machine with electronic commutation. Although the machine is physically
similar to the AC brushless permanent-magnet machine, and in many cases identical, the method of
driving it is fundamentally different. The brushless DC or “electronically commutated” motor,
sometimes also known as the “squarewave” or “trapezoidal” motor, does not have a rotating ampere-
conductor distribution. Since it does not have sine-distributed windings, phasor analysis and dq-axis
theory are not applicable to it. Squarewave drive is usually applied with surface-magnet motors which
have no “saliency” (i.e. Ld ' Lq) and no reluctance torque. If squarewave drive is used with a salient-
pole motor such as the IPM, the torque ripple will generally be substantial.
While “brushless DC” and “line-start AC” motors were emerging in the 1970's, many engineers
envisaged the possibility of removing the rotor cage from the “line-start” AC motor and of feeding this
motor with sinewave currents phase-shifted to maximize the torque per ampere. An account of such
an investigation is reported by Lajoie-Mazenc [25], including a salient-pole IPM motor
— self synchronization by means of shaft encoder feedback;
— the addition of a variable phase shift to optimize the torque production;
— the use of a digital encoder signal to index a sinewave reference for the current waveform; and
Although Lajoie-Mazenc used dq-axis theory in deriving equations for the optimum phase-shift angle,
he did not describe what would now be termed a field-oriented dq controller. The architecture of his
controller is reproduced in Fig. 2.95, and it is evident that it is an “I-( controller” in the sense that it
provides for the adjustment or control of the magnitude and phase of the current, as discussed in the
previous section. Since the current magnitude is determined in the rectifier upstream of the inverter,
it is not possible for this controller to exercise direct independent control of Id and Iq.

Fig. 2.95 Lajoie-Mazenc’s I!( controller (1983)


Page 2.92 SPEED’s Electric Machines

Fig. 2.96 dq controller described by Jahns et al, [26].

A field-oriented dq controller in the strict sense is one in which the d- and q-axis components of the
current are controlled independently, one of them being oriented to control the flux and the other one
being oriented to control the torque. In general this gives rise to a control block diagram in which the
separate d- and q-axis components are identifiable, as well as the means of controlling them.
Another example of a true dq controller is shown in Fig. 2.96, [26]. In the simplest case the d-axis
component Id can be held at zero ( fd ' 0) while the torque would be varied by controlling Iq via fq. At
high speed, when the EMF of the machine approaches the supply voltage, the flux can be “weakened”
by applying negative current in the d-axis: Id < 0. Alternatively, especially in salient-pole machines,
more complex variation with Id and Iq with torque and/or speed can be contemplated.
There is only one choice of reference frame for permanent magnet synchronous machines that aligns
the d-axis with the magnet flux, and in nonsalient pole machines the q-axis is the natural axis for the
torque-producing component of current. This reference frame is the only one which separates the flux-
controlling component of current from the torque-controlling component in nonsalient pole machines.
With salient-pole machines there is more justification for adopting a different dq axis frame of
reference: e.g., one in which the d-axis is aligned with the total flux rather than just the magnet flux.
Effect of phase advance on torque/speed characteristic: Fig. 2.97 shows a torque/speed
characteristic for an IPM motor (Fig. 2.88). With no phase advance, the torque starts to fall off at about
1400 rpm, and becomes zero when the speed reaches 2500 rpm. Also shown is the torque/speed
characteristic with a phase advance angle ( which increases linearly from 1200 rpm to 2400 rpm. The
torque does not reach zero. Even at 2400 rpm it is 0@85 Nm. At 2100 rpm it is still more than 1 Nm.
If we take base speed as 1200 rpm, the power at the base speed is 1200 rpm × 1@55 Nm ' 195 W. With no
phase advance, at 1800 rpm the torque is 0@7 Nm and the power is about 132 W and is less than the power
at base speed. With phase advance, the torque at 1800 rpm is 1@3 Nm and the power is 245 W. Phase
advance almost doubles the power at 1800 rpm, which is 50% higher than the base speed.
The principle of phase advance is equally applicable to the squarewave drive. But since squarewave
operation cannot be expressed in terms of phasors, computer simulations are used for analysis instead.

Fig. 2.97 Effect of phase advance on torque/speed characteristic.


Brushless permanent-magnet machines Page 2.93

2.16 CURRENT CONTROL IN THE SINEWAVE DRIVE


The switch control strategy for obtaining sinusoidal current
can be expressed in terms of voltage vectors, as shown in
Fig. 2.98.23 This diagram can be explained with the help of
the connection diagrams in Fig. 2.99, which show the
polarities of the motor line terminals corresponding to the
states of the six transistors in the bridge circuit, Fig. 2.3.
Note that the numbering of the transistors in Fig. 2.3 reflects
the order in which they switch on, as in the squarewave
drive; but in the sinewave drive it is normal to have three
transistors conducting instead of only two, and the
conduction period for each transistor is 180E instead of 120E.
Fig. 2.98 Voltage vectors

Fig. 2.99 Six-step connections

Such switching schemes are called “3 phase on” and “2 phase on” respectively; or, for short, “3Q” and
“2Q”.
Each of the six vectors in Fig. 2.98 corresponds to one of the connections in Fig. 2.99. For example, Q612
means that transistors 6,1 and 2 are on, so that line A is connected to the positive terminal and lines B
and C to the negative terminal. In a simple “six step” controller, this condition may persist for 60E, and
then transistor 6 switches off and transistor 3 switches on, producing the state Q123 in which lines A
and B are connected to the positive terminal and line C to the negative terminal. In the motor, the
orientation of the stator ampere conductor distribution (or “MMF vector”) advances 60E, corresponding
to the transition from the 1,0,0 to the 1,1,0 state. Over a full 360 electrical degrees of rotation the six
states follow one another in this way to produce a coarse approximation to a rotating MMF.
Even though there are only six possible states in a 3Q scheme, the current regulator or PWM controller
can switch between states at a much higher frequency than 6 times per cycle in such a way as to make
the current track a sinusoidal reference signal whose amplitude and phase are determined by the
torque demand and the speed. The coding example on the following page describes a simple, intuitive
current regulator algorithm to achieve this. Only the four highlighted vectors are used during the base
segment.24 In the regulator algorithm, the currents (ik1 and ik2) in two lines are sensed at each step
of the time stepping solution, and the decision is made as to which transistors to switch on or off. The
coding is as follows:

{Current regulation in Sixstep}


begin
iCR1 :' ISP * Sin(ThRe); {Set reference in line 1}

23
The term “switch control strategy” means much the same thing as “pulse-width modulation strategy”. There is a large number
of different PWM strategies and the subject has been extensively researched, especially in relation to the control of field-oriented
AC motor drives. The particular strategy described here is a simple intuitive one.
24
The “base segment” is a 60E interval during which PC-BDC computes the current waveforms recursively, reconstructing the
entire period from this segment by a substitution process once the solution has converged.
Page 2.94 SPEED’s Electric Machines
iCR2 :' ISP * Sin(ThRe vert -15 scalesym 150 vert 15 2*pi/3); {Set
reference in line 2}
If (k*j) mod kTs ' 0 then {Check switching freq. }
If ik1 > iCR1 then
if ik2 > iCR2 then
VoltageVector :' Q456
else
VoltageVector :' Q234
else
if ik2 > iCR2 then
VoltageVector :' Q612
else
VoltageVector :' Q123;
end;

The transistors do not always begin to conduct when they are switched on, because there may be a
freewheeling current in the antiparallel diode. At high speed the diode currents may be flowing all the
time so that although the transistors receive turn on signals they may never conduct. Note that it is
not immediately obvious from the current waveforms whether, say, a positive current in phase 1 is
flowing in Q1 or D4, etc.
Although this description is written for a wye connection, the same control logic applies with a delta
connection. Also note that the 3Q strategy uses complementary switching of the upper and lower power
transistors in each phase leg.
Analytical note: Time stepping simulation in dq axes requires that the d,q terminal voltages vd and vq be determinate. In a
squarewave 2Q drive, normally two transistors are conducting (PERIOD B) and two terminal voltages and one current are
determinate, instead of the three terminal voltages. Therefore vd and vq are not determinate and the d,q equations cannot be used.
During commutation or freewheeling (PERIOD A), however, a diode connects the freewheeling line terminal to the positive or
negative rail and all three terminal voltages are known. Therefore PC BDC switches from dq axes in PERIOD A to direct phase
variables in PERIOD B. With two phases conducting, the inductance is the line line inductance of two phases in series. It varies
sinusoidally with rotor position (assuming that the machine is sinewound), and it and its derivative with respect to position are
derived from Ld and Lq.
In a 3Q drive the terminal voltages and hence vd and vq are determinate at all times, permitting the dq equations to be used
throughout. Note that the actual phase self and mutual inductances Lph and Mph are used when he dynamic simulation is
working in actual phase variables, while the synchronous inductances Ld and Lq. are used when the dynamic simulation is in dq
axes.
Fig. 2.100 summarizes these considerations. The solution is no faster in dq axes than in direct phase variables, because under
transient conditions the reference frame transformations to vd, vq from va,vb,vc and to id,iq from ia,ib,ic (and their inverses) must
be performed every time step. This neutralizes any saving in computation time which might be expected with the dq solution.

Fig. 2.100 Usage of dq equations and direct phase variables


Brushless permanent-magnet machines Page 2.95

2.17 PM GENERATORS

A PM brushless machine can operate as a motor or a generator. When it is motoring, the


electromagnetic torque is in the same direction as the rotation. When it is generating, the
electromagnetic torque opposes the rotation. In either case, there is a generated EMF in each phase
proportional to speed. This EMF is an AC quantity. Its fundamental frequency is given by
f ' n × p Hz (2.294)
where n is the speed in rev/sec, i.e. rpm/60, and p is the number of pole pairs. Alternatively,
rpm Poles rpm × Poles
f ' × ' . (2.295)
60 2 120

The waveform of the EMF is not necessarily sinusoidal. It depends on


— the profile of the airgap flux density waveform produced by the magnet
— the winding distribution
— the connection of the winding (wye, delta, etc.)
— the amount of skew (if any).
Sometimes the frequency is expressed in electrical radians per second, with symbol T; thus
T ' 2B f rad/sec (2.296)
and the period is J ' 2B/T. The period J is the time taken for the rotor to rotate through 1 cycle, i.e.
through two pole pitches, where the “pole pitch” is B/p radians or 360/Poles in degrees. A “cycle” is
also known as 2B “electrical radians” or 360 “electrical degrees”. This is equal to 2B/p mechanical or
actual radians, or 360/p mechanical degrees.
To begin, we will assume that the EMF is sinusoidal, because this is normally the case for AC generators,
and it means that we can use the classical methods of analysis to describe the performance, especially
the phasor diagram. The generated EMF is proportional to the product of the speed and the flux
produced by the magnet, and it obeys the classical equation for AC machines:
2B
Eq1 ' kw1Tph f M1 ' 4@44 kw1Tph f M1 V rms . (2.297)
2
Here Tph is the number of “turns in series per phase”. If each phase has a total of T turns and they are
connected in a parallel paths, then Tph ' T/a. The factor kw1 is the fundamental harmonic winding
factor. A properly designed winding has the property of filtering out harmonics in the EMF waveform,
rendering it more sinusoidal. This filtering is achieved at the expense of a slight loss of EMF compared
to that which would be obtained if all the coils were “fully pitched” and concentrated together. kw1
expresses this reduction. Usually it has a value between 0.8 and 1, so the slight loss of EMF is not a high
price to pay for the elimination of unwanted harmonics which would distort the waveform. Sometimes
the product kw1 Tph is termed the “effective series turns per phase”, Tph1.
The quantity M1Md is the “fundamental flux per pole” produced by the magnet when the machine is
running on open circuit. Only the fundamental harmonic component of airgap flux contributes to the
fundamental component of EMF. M1Md can be obtained from the total open circuit airgap flux by Fourier
[oc]
analysis. It is related to the peak open circuit airgap flux density B1 by the equation

B1[oc] D Lstk
M1Md ' (2.298)
p

where D is the stator bore diameter and Lstk is the stack length.
Page 2.96 SPEED’s Electric Machines

The subscript q in Eq1 tells us that the EMF lies along the q axis in the phasor diagram; the flux M1 lies
along the d axis. In phasor terms, E is written 0 jEq1, or simply jEq or even just jE. Also

E ' j T Q1Md (2.299)

where Q1Md ' Q1Md j0 is the fundamental flux linkage per phase, and Q1Md ' kw1TphM1//2 represents
the product of the magnet flux and the effective series turns/phase. The units of Qd1 are volt seconds
and the /2 converts the peak value into the r.m.s. value, since Q1 represents a flux linkage that is
varying sinusoidally in time. All the quantities in the phasor diagram are r.m.s. values. Loosely
speaking, eqn. (2.299) means that the generated EMF is proportional to speed times flux, or frequency
times flux. Strictly speaking, it states that the EMF is proportional to speed times flux linkage, because
it is also proportional to the number of effective series turns per phase Tph1 ' kw1Tph. The “j” in eqn.
(2.299) means that the EMF phasor E leads the flux linkage phasor Q1 by 90E. The phasor diagram on
open circuit is shown in Fig. 2.239. With no current the terminal voltage V is just equal to E.25
Load current and impedance: The current that flows from a PM generator depends on the load
connected to it. Four important kinds of load can be identified as
being important in understanding the operation of the PM
generator:
— The "infinite bus".
— A fixed impedance load.
— Another synchronous machine.
— A rectifier.26
The infinite bus is represented as a voltage source having a fixed
voltage and frequency. Physically it is approximated by a very
large network such as the U.K. National Grid, which is so large
(60,000 MW) that neither the voltage nor the frequency can be Fig. 2.101 G e ne r a t o r c o nne c t e d to
perceptibly altered by connecting one small additional generator infinite bus
(or load) to it. Fig. 2.101 shows the connection of a generator to
an infinite bus. This diagram is only a schematic diagram
because it does not show the internal impedance of the
generator; (it also omits the circuit breaker!)
A fixed impedance load is represented electrically as a passive
electrical circuit containing resistance R and inductance L in
each phase. This is represented in Fig. 2.102.
Fig. 2.103 shows a generator connected to another synchronous
machine, and Fig. 2.104 shows a generator connected to a
rectifier. The rectifier itself is loaded with an impedance
comprising a resistance R and inductance L. The complex
representation R jTL is not used on the DC side, since it applies Fig. 2.102 G e ne r a t o r c o nne c t e d to
only when the voltages and currents are sinusoidal AC. R!L load

The rectifier is probably the commonest type of load to which


PM generators are applied. Unfortunately, a rectifier is a non-linear load, and even though it is
connected to a sinusoidal AC supply (the PM generator), it draws a non-sinusoidal current i. This
means that we cannot use phasors to calculate the current.

25
Note that a spinning PM generator is electrically "hot" (i.e. "live") whenever it is spinning, and it is dangerous to assume that
it is safe just because it is disconnected or isolated from its load. PM generators should ideally have warning labels on the
terminal box to remind electricians of this fact.
26
Occasionally a PM generator may be connected to a cycloconverter, as in one or two VSCF (variable-voltage/constant-
frequency) aircraft power generating systems; but this is beyond the scope of these notes.
Brushless permanent-magnet machines Page 2.97

Fig. 2.103 Generator connected to another synchronous Fig. 2.104 Generator connected to a rectifier with a DC load
machine having resistance R and inductance L.

The DC current id is also not a pure DC current, because it contains harmonics.


The computation of rectifier loads requires computer simulation. To get round this difficulty we design
the PM generator in two stages. The first stage involves designing as though the generator was going
to be connected to a linear AC load. The generator can be "rated" to operate with this linear load. We
call this the sinewave rating, because it is strictly valid only for loads that draw sinusoidal current.
Then, in the second stage, we make an allowance for the nonlinear effects of the rectifier load, leading
to a "rectifier rating" which can go into the catalogue as an addition to the sinewave rating.
Generator equivalent circuits — internal impedance: The windings of the generator have
resistance R, and they also have self inductance and mutual inductance between phases. The simplest
type of generator is a nonsalient pole generator, in which the phase inductances are unaffected by the
rotor position. In this case, in the steady state with AC sinusoidal current and EMF, the generator has
a simple equivalent circuit for each phase, Fig. 2.105.

Fig. 2.105 Equivalent circuit of one phase of nonsalient-pole generator

The total internal impedance of the generator is Z ' R jXd. The reactance Xd is the synchronous
reactance and it is equal to TLd where Ld is the synchronous inductance. The synchronous inductance
is not simply the self inductance per phase, but includes the effect of the total armature reaction flux
in generating an internal voltage jXdI which is in series with E and the resistance voltage drop RI.
Since the total armature reaction is the sum of contributions from all phases, the internal voltage jXdI
includes the effect of mutual inductance under balanced conditions. It is a sinusoidal voltage
represented as a phasor, and therefore Xd also incorporates the filtering effect of the winding
distribution (and any skew).
When current flows, the terminal voltage V deviates from the open circuit value E, and according to
Ohm’s law applied to Fig. 2.105, the relationship between V and E is
V ' E ! (R j Xd ) I . (2.300)
Page 2.98 SPEED’s Electric Machines

Fig. 2.106 Phasor diagram of nonsalient-pole generator with lagging power factor angle N.

This is represented in the phasor diagram, Fig. 2.106, which is drawn to emphasize the relationships
between the phasors, so the resistance R and reactance Xd have been exaggerated relative to their
normal values in a PM generator. Points to note about the phasor diagram are as follows:
(a) The phasor diagram and all equations derived from it are strictly valid only when the voltage
and current are sinusoidal. Two cases where the voltage and current are not sinusoidal are (a)
when the output is rectified and (b) when the PM machine has a nonsinusoidal EMF waveform.
(b) With lagging power factor the terminal voltage V is generally less than the open circuit voltage
E. The “armature reaction” (i.e., the internal voltage drop across Xd) has a demagnetizing effect
and the machine is said to be “overexcited” (i.e., with E > V). If the load is disconnected, the
terminal voltage will jump up to the higher value E. This condition must be considered in
practice because it might be unsafe if E is too high.
(c) Since E is proportional to speed, V may have to be regulated by an electronic controller (such
as a phase controlled rectifier) on order to maintain constant output voltage as speed varies.
(d) The output power per phase is given by
P ' V I cos N (2.301)
jN
in watts, where N is the power factor angle. If the load is an impedance ZL ' ZLe ' RL jXL,
2 2
then cos N ' RL/ZL ' RL//(RL XL )
(e) If R is much smaller than Xd, eqn. (2.301) can be expressed in terms of E and V in watts/phase:
EV
P ' sin * (2.302)
Xd

where * is the load angle. This equation is commonly used in power systems engineering to
express the fact that there is a maximum power Pmax which can be generated stably. In power
systems the generator is usually considered to be connected to an infinite bus with V ' constant.
Pmax is the power obtained when * ' 90E and sin * ' 1.
(f) If we write ( ' * N, where ( is the angle between the current phasor I and the q axis, the power
per phase can be written as

P ' E I cos ( ! R I 2 (2.303)


2
in which RI represents the resistive power loss per phase. The term EI cos ( represents the
mechanical power per phase, which is written Pm/m, where m is the number of phases (usually
3). Now I cos ( ' Iq, the q axis component of the current, and E ' Eq ' TQ1Md. If Te is the
electromagnetic torque and Tm is the angular velocity, then Tm ' T/p and TmTe ' Pm, so
Te ' m p Q1Md Iq Nm. (2.304)
Brushless permanent-magnet machines Page 2.99

This equation is commonly used in connection with PM motors. It shows that the torque is
maximized if the current phasor is oriented along the q axis such that it is “in quadrature with
the flux” (and in phase with the EMF E). [Note: Iq is in r.m.s. amperes, and Q1Md in r.m.s. V sec].
2
(g) The phasor diagram includes resistive losses (RI ) but it does not include mechanical losses or
iron losses. The mechanical losses can easily be allowed for by a friction torque Tf which must
be added to Te to give the shaft torque T. Thus

T ' Te Tf . (2.305)
The simplest way to deal with the iron losses is to treat them as a mechanical loss and include
the corresponding torque along with Tf. Thus the power loss in the core is WFe ' TmTFe and
WFe
T ' Te Tf . (2.306)
Tm
It is also possible to represent the core loss as an electrical loss, but this modifies the phasor
diagram in rather a complicated way, requiring an iterative solution.

Salient pole machines: A salient pole machine is one in which the rotor has two axes of symmetry.
Generally one of these axes is the d axis which is the axis of magnetization, and the other one is the
q axis or interpolar axis. In a salient pole machine the phase inductances vary with rotor position,
but if the machine has sinusoidally distributed windings the analysis can be simplified by Park’s
transformation such that the phasor diagram can be used for sinusoidal operation in the steady state,
with a relatively simple modification. This modification is to split the armature reaction voltage drop
into separate components aligned respectively with the d and q axes, as shown in Fig. 2.107.
The power per phase is still given by eqn. (2.301), but eqns. (2.302 2.304) acquire additional “saliency”
terms. First we resolve the voltages and currents into their d and q axis components
Vd ' V sin * ; Vq ' V cos *
(2.307)
Id ' I sin ( ; Iq ' I cos (

then from the phasor diagram


Vd ' R Id Xq Iq ;
(2.308)
Vq ' E R Iq Xd Id .
*
The electrical power output per phase is P ' Re{VI } ' VdId VqIq, giving

P ' E Iq ( Xq Xd ) Id Iq R (Id2 Iq2) (2.309)

and if R is negligible then


E Vd 1 1
P ' Vd Vq (2.310)
Xd Xd Xq
in watts per phase. This is often written

EV V2 1 1
P ' sin * sin 2 * . (2.311)
Xd 2 Xd Xq
The electromagnetic torque is

Te ' m p ( Qd1 Iq Qq1 Id ) Nm (2.312)

where Qd1 is the total fundamental d axis flux linkage and Qq1 is the fundamental q axis flux linkage.
Page 2.100 SPEED’s Electric Machines

Fig. 2.107 Phasor diagram of salient-pole generator with a load having a lagging power-factor angle N

Rectifier loads and “rectifier rating”


Rectifiers draw nonsinusoidal current waveforms, and a rigorous analysis would involve complex
time stepping simulation which is a difficult and time consuming process, especially if there is
significant saturation in the machine. To continue to use the simpler theory of the phasor diagram it
is desirable to develop an equivalent linear load with constant impedance. This involves the following
steps:
(1) Determine the r.m.s. value of the fundamental AC line current. For a three-phase diode rectifier
we can equate the DC power to the AC input power:
3VLL IL cos N ' VDC IDC (2.313)
where VDC is the mean DC voltage and IDC is the mean DC current, VLL is the r.m.s. AC line line
voltage at the terminals of the generator, and IL is the AC line current. For a diode rectifier with
a purely inductive load, the input power factor is
3
cos N ' cos " (2.314)
B
where " is the phase delay angle. The AC line current is therefore
V D ID
IL ' A [rms] (2.315)
3 VLL cos N

(2) Determine the form factor Q of the rectifier load current. This is the ratio of the actual r.m.s.
current I to the r.m.s. value of its fundamental component I 1 . For the ideal case of a diode
rectifier with a purely inductive load it is given by

2/3 1
Q ' ' . (2.316)
( 6 / B ) cos " cos N

The generator is designed for a sinewave current of IL and power factor cos N, but if there are additional
harmonics in the current waveform, Q should be estimated and the calculated I 2 R loss in the generator
should be increased by the factor Q 2 to allow for the additional losses caused by them.
The process may need to be repeated several times to determine a complete operating chart covering
a range of currents and power factors, especially if the generator is to operate at different load levels.
Brushless permanent-magnet machines Page 2.101

Fig. 2.108 3-phase generator with diode rectifier, loaded with a pure inductance

Diode rectifier.
The voltage equation (2.317) gives the DC terminal voltage under load, including the effect of the
internal inductance of the generator. This inductance is taken as the synchronous inductance Ld.
When Idc = 0, eqn. (2.317) gives the open-circuit DC voltage.

3 3
Vdc ' e cos " T Ld Idc (2.317)
B LL pk B
where
e LL pk is the peak line-line EMF of the generator
" is the phase-control angle (= 0 in a diode rectifier)
T is the fundamental AC radian frequency 2B f
L d is the synchronous inductance of the generator in [H] (Note Xd = T Ld)
V dc is the mean DC voltage at the rectifier terminals (no filter components)
I dc is the mean DC current

Also

eLL pk ' kE Tm (2.318)

where
k E is the EMF constant in [Vs /rad]
T m is the mechanical angular velocity [rad/s] = RPM × B/30

Circuits with additional elements — that is to say, more complex circuits — generally require computer
simulation for accurate prediction of the behaviour.
Page 2.102 SPEED’s Electric Machines

Control and protection


If the generator is running at a certain speed, the generated EMF is fixed and the power output depends
on the load impedance, which determines the current and the power factor. If the load draws more
power, then the prime mover must respond by providing more torque, otherwise the generator will slow
down. For isolated generators it is normally necessary to provide a governor to make sure that the
prime mover does this. The simplest form of governor is one that maintains the speed constant, i.e. a
closed loop speed controller, because this will ensure that the generator will receive whatever torque
is necessary to supply the load (plus the losses in the generator).
The governor must also protect the generator and prime mover against overspeed. Overspeed is most
likely to occur if the generator loses its load, which can happen quite normally if the load is switched
off, or because of a malfunction that causes the load to disappear or to be disconnected from the
generator. A sudden loss of load causes all the prime mover power to be applied in accelerating the
generator/prime mover, and unless the prime mover is shut down quickly, a dangerous overspeed can
result very quickly. For this reason the governor must be designed with a sufficiently fast response to
any speed error between the actual speed and the set point speed.
In wound field synchronous generators connected to a local load, isolated from the grid system, the
voltage can be varied by changing the field current. The power factor, however, is still determined by
the load impedance. If a wound field generator is connected to an infinite bus, the voltage cannot be
changed by varying the field current. Instead, the power factor changes. Increasing the field current
tends to make the power factor more leading (i.e. "overexcited"), whereas decreasing it makes the power
factor more lagging ("underexcited").27 When a generator is connected to an infinite bus, the power is
controlled by the prime mover torque (and by that alone).
PM generators have no means of excitation control, i.e., no field winding. Therefore the voltage at the
generator terminals cannot be varied without changing the load impedance or the speed. Changing the
speed also changes the frequency. If a PM generator is connected to an infinite bus, the frequency and
voltage are fixed by the infinite bus, and the only means of control is the prime mover torque, which
determines the power. The current and power factor both vary in a manner that depends on the
internal impedance and open circuit EMF of the generator.
If the generator is connected to an infinite bus, the output power must be limited such that the
generator remains synchronized with the frequency and phase of the infinite bus. With isolated
generators the issue of maintaining synchronism does not arise.
Another hazard with all generators is the over voltage which occurs when the load is suddenly
disconnected. If the load current was at its maximum value just before disconnection, there may be
considerable energy stored in the internal reactance of the generator. When the disconnection takes
place, the circuit breaker must dissipate this energy. When the current has fallen to zero, however,
the voltage drop across the internal impedance disappears, and the generated EMF now appears at the
terminals. Generally this will exceed the rated terminal voltage; by how much depends on the internal
(synchronous) reactance. The internal reactance of PM generators tends to be fairly low, but on the
other hand PM generators are also used at extremely high speeds, so the potential overvoltage must be
allowed for in the insulation system. There is no possibility of switching it off, since the magnets are
permanently excited. If there is a filter capacitor on the DC side, overvoltage protection is essential.
This can be provided by a fast acting “crowbar” circuit which detects the overvoltage and connects a
dump resistance across the DC terminals.
The overvoltage problem can also arise with a PM motor if it is driven into an overspeed condition by
the mechanical system to which it is connected. In this case there is also a possibility of destroying the
freewheel diodes which rectify the generated current and feed it to the DC link capacitor. This is one
reason for caution in the use of “embedded magnet” motors, which have higher per unit inductance
than surface magnet motors, so that if the load is lost at high speed the generator terminal voltage
tends to rise more than that of a surface magnet machine.
With all generators the load current must be monitored, and if the current exceeds the generator rating,
27
By convention, an overexcited generator is said to be “generating VArs” (reactive power), while an underexcited generator
is “absorbing VArs”. For example, if the load is inductive, the VArs generated by the generator are absorbed by the load.
Brushless permanent-magnet machines Page 2.103

appropriate protective measures must be activated. If the current is only a few percent above the rated
value, it may be sufficient to do no more than display a warning light. More sophisticated protection
would use inverse time overcurrent relays and automatically disconnect the generator by opening a
circuit breaker. Generator protection may also include differential current relays (to detect internal
generator faults); over temperature relays; and negative sequence relays to protect against excessive
unbalance between phase currents. In addition, the overall system will generally require a ground
fault protection scheme. One of the things to bear in mind with electrical generators is that there may
be huge amounts of energy stored in the magnetic field and even more in the rotating mass, and the
protection system must be designed, in general terms, to protect against its uncontrolled release.
Page 2.104 SPEED’s Electric Machines

2.18 LINE-START PM MOTORS


History

Fig. 2.109 2-pole PM Line-start motor [Adkins, 1962]

By the early 1950s, PM generators had already been in service for several years, but examples of PM
line-start motor from that era are hard to find.28 One of the first appears to have been the 4-pole
Permasyn motor described in 1955 by F.W. Merrill [23] . Fig. 2.109 shows a 2-pole PM line-start motor
rotor similar in construction to the Permasyn motor, described and analyzed by Cahill and Adkins [29]
in 1962. Both of these motors used Alnico magnets and standard induction-motor stators. By the early
1960s, Volkrodt (Siemens) was extolling the virtues of ferrite magnets in this type of motor, [30].

Fig. 2.110 4-pole Synchronous induction motor


(Synduction Motor®, Allis-Chalmers / Honsinger [31]); not to scale

The development of PM line-start motors seems to have lain almost dormant during the 1960s and into
the 1970s, when line-start synchronous reluctance motors were developed intensively by Siemens in
Germany, by Lawrenson, Fong and others in the UK, and by Honsinger in the USA: an example
attibutable to Honsinger is shown in Fig. 2.110. These motors were specially advocated for their
synchronous operation, and were often used with variable-frequency inverters. In many cases several
motors were operated in precise synchronism from a common inverter. The rotor cage makes it
possible to start the motor “across the line”, and provides damping to prevent speed oscillations; such
oscillations were problematic and much analysis was published on them in the 1960s and 1970s.
In retrospect it might seem strange that the synchronous reluctance motor was apparently preferred
over the PM motor at that time. But before the mid-1970s the only available magnet materials suitable
for PM motors were Alnico, which has a high remanent flux-density but a low coercivity; or ferrite,
which has a much higher coercivity but a much lower remanent flux-density. Thus the Alnico motors
would be prone to demagnetization, while the ferrite motors would have low power density.

28
Small timing motors and clock motors are not included in this discussion, which focusses on power levels above 100W or so.
Brushless permanent-magnet machines Page 2.105

Fig. 2.111 Isosyn® motor concept described by Laronze [1978] (not to scale)

In about 1977 interest in the PM line-start motor suddenly re-awakened. By that time high-energy
cobalt-samarium magnets had become commercially available (though expensive at $100/lb at 1980
prices). Fig. 2.111 shows a 2-pole European concept on which the Isosyn® motor was produced by CEM,
a division of Brown Boveri (now ABB), in a range of sizes from 0@37kW to 18@5kW. Fig. 2.111 shows the
4-pole, Reliance motor described by C.R. Steen [36], and Fig. 2.114 a 2-pole motor similar to the one
reported by Miyashita of Hitachi in 1980, [35].

Fig. 2.112 Line-start motor of the type described by Steen / Reliance [1979]

During this period of experimentation with the new high-energy magnets, a wide range of rotor
configurations was investigated. Typical of this period are the rotors shown in Fig. 2.113 described by
Binns, [33, 37, 45]. Rotor (a) is quite similar to the Synduction® motor of Fig. 2.110, with magnets
inserted in the low-inductance axis, [31]. In rotor (b), circumferentially-magnetized magnets add
narrow “pulses” to the airgap flux just outside the edges of the main soft-iron pole-pieces. Rotors (b)
and (c) both embody the idea that the magnet flux should be in parallel with the “reluctance” flux
produced by the stator current. In rotor (c), the d and q axes are not fixed by the geometry, but depend
on the load.

Fig. 2.113 Line-start motors described by Binns [1978]


Page 2.106 SPEED’s Electric Machines

Fig. 2.114 Line-start motor of the type described by Miyashita Fig. 2.115 2-pole line-start motor, [38]
/ Hitachi [1980]

Finally Fig. 2.115 shows a 2-pole rotor which has been studied intensively for single-phase applications,
particularly refrigerator compressors, since about 2000, [38].

“Strong” PM machines, “Weak PM machines”, and “PM-assisted reluctance machines”

Fig. 2.116 “Strong” and “weak” PM rotors

In the search for the “optimum” line-start rotor configuration, two main classes of synchronous motor
emerged, and this classification has continued throughout the more recent development of inverter-fed
motors which have no rotor cage. These two classes are represented by the two motors in Fig. 2.116. At
one time they were called “strong” and “weak” PM motors. The one on the left is a 2-pole motor in
which the magnets of each pole are divided into 3 separate blocks. This motor has a very high airgap
flux and almost no saliency. The motor on the right has very high saliency, mainly due to the shape and
the large number of flux-guides and barriers, [39, 44]. With the d- and q-axes defined as shown, Ld <<
Lq. Permanent magnets are inserted between the flux-guides with the intention of suppressing the d-axis
flux still further, which is tantamount to increase in the effective saliency ratio Lq/Ld; (see Fig. 2.117).

Definition of d- and q-axes


In most of the literature on synchronous reluctance motors, the d-axis is defined to be the high-
inductance axis, and the q-axis the low-inductance axis. In the PM machine it makes more sense to
define the d-axis as the axis of magnetization of the magnets. In most PM motor configurations the
magnet axis is the low-inductance axis, with Ld < Lq. This convention is used in all the figures in this
chapter, even in Fig. 2.110, where the d-axis is defined as the axis where the magnets would be if they
were added (as in Fig. 2.113).
Brushless permanent-magnet machines Page 2.107

Magnets in both the d- and q-axes (a slight theoretical diversion)

Fig. 2.117 Phasor diagram of hybrid synchronous motor with magnets in both the d- and q-axes

Fig. 2.117 shows the theoretical effect of having magnets in both the d- and the q- axes. The d-axis
magnet generates Eq1 and the q-axis magnet generates Ed1. The phasor diagram is drawn in such a way
as to show the addition of fluxes (induced voltages) in both axes. Thus eqn. (2.243) on p. 80 becomes
Q ' Qd j Qq , where Qd ' Q1Md Ld Id ; Qq ' Q1Mq Lq Iq . (2.319)
For current-forced operation, the torque is derived from eqn. (2.247), and eqn. (2.248) on p. 80 becomes

Te ' m p [ Q1Md Iq Q1Mq Id Id Iq ( Ld Lq ) ]

mp
' [ Eq1 Iq Ed1 Id Id Iq ( Xd Xq ) ] (2.320)
T
Strong Weak Reluctance
PM alignment PM assist

The d- and q-axis magnets are likely to generate unequal EMFs Eq1 and Ed1, so one or other of them will
appear to contribute a strong alignment component of torque, while the other will merely augment the
effective saliency by reducing still further the flux on the low-inductance axis, thus enhancing the
reluctance torque. This “2-magnet” theory does not appear to have been published or associated with
a particular rotor design, but it suggests an investigation. Moreover, these equations could be adapted
to deal with cross-coupling effects that arise under saturated conditions.
For voltage-fed operation, which is more relevant to the line-start motor, eqn. (2.252) on p. 82 becomes
Vq Eq1 Vm cos * Eq1 Vd Ed1 Vm sin * Ed1
Id ' ' ; Iq ' ' . (2.321)
Xd Xd Xq Xq
and if we susbtitute eqn. (2.321) in eqn. (2.263) on p.85 we get the modified torque with two magnets:

mp Eq1 Vm Ed1 Vm V m2 1 1
Te ' sin * cos * sin 2 * .
T Xd Xq 2 Xq Xd
(2.322)
Strong PM Weak Reluctance
Alignment PM&assist
(diversion over!)
Page 2.108 SPEED’s Electric Machines

Analysis : polyphase motors — steady-state


Under balanced conditions, the steady-state performance of the polyphase line-start motor is described
by the equations in §2.13, including the torque equation (2.263) on p. 85.

Analysis : polyphase motors — asynchronous and starting [42, 43]

Torque

B
A

0 0  Tmax 180°  Load-angle

Fig. 2.118 Synchronization of PM line-start motor

The starting process can be considered in two parts : asynchronous run-up and synchronization. The
rotor cage provides the asynchronous run-up torque, and as the speed approaches synchronous speed
the slip decreases towards zero. During the latter phases of run-up the pole-slipping can be
characterized by the dotted curve in Fig. 2.118, which represents the operating point in terms of the
transient torque and the load-angle *. In a successful synchronization, the operating point is
“captured” by the steady-state T * curve which is plotted from eqn. (2.263) on p. 85.
In Fig. 2.118, synchronous speed is first reached at B, but operation to the right of the pull-out point M
is unstable, and any small displacement that reduces * will initiate an acceleration that causes the
machine to stabilize at A. A small acceleration that increases * will initiate a deceleration leading to
another pole-slip.
The final synchronization process requires
a certain impulse related to the available
area under the T * curve, and since this is
finite, there is a limit to the inertia that can
be synchronized at a given load torque: in
other words, there is a critical inertia
which is defined as the largest that can be
synchronized at a particular load . Fig.
2.119 shows this capability in the form of a
chart of load torque vs. total inertia,
normalized to the motor inertia. Obviously
a higher inertia can generally be
synchronized if the load torque is lower.
The “synchronizing capability” is not a
limitation in induction motors, which are
not required to “pull-in” to synchronism.
Fig. 2.119 Synchronizing capability
Brushless permanent-magnet machines Page 2.109

Fig. 2.120 Asynchronous torque/speed curve of PM line-start motor

A typical average asynchronous torque/speed curve is shown in Fig. 2.120. Average means that the
torque is notionally averaged over each electrical cycle to remove the transient torque components
which can be seen in Fig. 2.121. Also shown in Fig. 2.120 is the synchronous torque, represented by a
vertical line at synchronous speed. Different points on this line correspond to different load-angles. The
operating point must transfer from the asynchronous curve to the synchronous characteristic, and this
process is highly dynamic, as can be seen in the following three Figures.
There is also a magnet braking torque produced by the fact that the magnet generates current in the
stator circuit resistance, and the associated power losses produce the braking torque. This torque has
a speed dependence similar to the slip-dependence of the asynchronous torque, and it tends to peak at
a fairly low speed.

Fig. 2.121 Transient torque-speed trajectory — PM line-start motor, 5 kW, load inertia ' 10 × motor inertia
Page 2.110 SPEED’s Electric Machines

Fig. 2.122 Speed vs. time — PM line-start motor, 5 kW, load inertia ' 10 × motor inertia

It is characteristic of the PM line-start motor that the starting current remains high throughout the run-
up. The transient torque contains a strong oscillatory component due to the pole-slipping, and a very
severe case of this can be seen in Fig. 2.121, which shows many torque reversals at low speed. These
torque reversals impose a severe duty on the shaft couplings and on the rotor itself, and the load.

Fig. 2.123 Speed vs. time — PM line-start motor, 5 kW, load inertia ' 10 × motor inertia

The effects of scale are important in the starting behaviour. The motor in Figs. 2.121, 2.122, and 2.123
has a stator diameter of 200 mm and a stack length of 100 mm, so its rated power is of the order of 5kW.
If the linear dimensions are doubled, the asynchronous torque becomes relatively smaller at low speed,
while the magnet braking torque also tends to peak at low speed. Consequently this motor has difficulty
starting. If the dimensions are halved, the rotor resistance becomes proportionally larger, and so the
torque/speed curve becomes less steep at small slip. This makes it more difficult for the small motor
to jump into synchronism. These ideas give some vague corroboration to the range of sizes introduced
for the Isosyn® motor mentioned on p. 105, [34].
Brushless permanent-magnet machines Page 2.111

Analysis : single-phase motors — steady-state, no rotor cage — simple preliminaries

Fig. 2.124 Equivalent circuit for 1-phase operation

The simplest representation of a single-phase synchronous motor is shown in Fig. 2.124, where the
motor is represented by its EMF E and impedance R jX. This representation is sufficient if the motor
has no saliency and no rotor cage. The current is given by
V E
I ' . (2.323)
R jX

As long as V and E are both sinusoidal, and R and X are constant, the current will be sinusoidal. The
torque is given by eqn. (2.231), and the power and torque both have a double-frequency pulsation, as
described on p. 74.
A single-phase motor with no rotor cage is impractical: it is not self-starting, and even if some means
were provided to start and synchronize it, it would tend to oscillate about synchronous speed, and it
might even be unstable. A rotor cage is necessary on both counts, and it is advantageous to design with
a more-or-less uniform ring of cage bars, with “buried” magnets inside, as in Fig. 2.115. This usually
gives rise to a degree of saliency (with Ld < Lq). The starting and running performance are strongly
influenced by the rotor geometry and the saliency. With saliency and/or a rotor cage, eqn. (2.323)
becomes seriously inadequate.
The key to the analysis of the single-phase motor is based on the idea of forward and backward rotating
fields, and there are two ways to approach it. A stator MMF distribution that is sinusoidally distributed
around the airgap, but pulsating in proportion to the current /I cos Tt, can be written
1 1
F ' Fmax cos p 2 cos T t ' Fmax cos (p 2 T t) Fmax cos (p 2 T t) (2.324)
2 2

This states that a stationary pulsating MMF can be resolved into equal but oppositely-rotating
components of half amplitude, the so-called forward and backward rotating components. One way of
extending the analysis is to extend the equivalent circuit of Fig. 2.124 with separate impedances and
induced voltages for the two components. This method is attributed to W. Morrill, and it is described
for the induction motor in SEM-3. The other way is to use the more formal procedure of symmetrical
components, and this is the method that will be used here.
The symmetrical component method obtains the forward and backward components in phasor form,
by a mathematical transformation. They are called positive and negative sequence components.
However, since there are two components, we have to start from a 2-phase circuit. This is not
inconvenient, because the method is to be used for capacitor motors which really do have two phases,
even though they may be unbalanced. The pure single-phase motor will therefore be analyzed as a 2-
phase motor with zero current in one of the phases. Although this may seem strange, it leads to some
useful physical interpretation of the operation of the pure single-phase motor. (See also p. 121).
Note that if there is no rotor cage, the circuit can be solved using eqn. (2.534) on p. 158, and the torque
can then be obtained using eqn. (2.420) on p. 126, even if there is saliency. However, eqn. (2.534) is a
differential equation and its solution is not so convenient as one that uses phasors. A solution in
phasors requires only algebraic calculations, and even though these use complex numbers they are
more convenient, and easier to interpret, than any operations using the calculus.
Page 2.112 SPEED’s Electric Machines

Analysis : single-phase PM motor at synchronous speed — symmetrical components

Fig. 2.125 Circuits for analysis of single-phase machines

Fig. 2.125 shows the electrical circuits to be used for machines connected to a single-phase AC source
Vs. The most general case is Fig. 2.125(a), which has
a split-phase winding;
a salient-pole rotor
a conducting cage on the rotor, and
an auxiliary capacitor.

The term “split-phase” expresses the fact that the single-phase current from the supply is divided
between two phases, the main and the auxiliary. The motor in Fig. 2.125(a) is really a two-phase motor,
and is is generally unbalanced, which means that the main and auxiliary windings have different
numbers of turns, different wire gauges, different distributions of coils, and different external circuits.
The axis of the auxiliary winding is oriented so that with forward-rotating flux, the voltage induced in
the auxiliary winding leads the voltage induced the main winding. The displacement between the
auxiliary winding axis and the main winding axis is usually 90E, but other angles are sometimes used.
For self-starting “across the line” from standstill by means of a contactor S, the machine must have a
rotor cage and a split-phase winding with a capacitor, as in Fig. 2.125(a). Without the cage, the machine
cannot produce any average torque unless it is running exactly at synchronous speed.29
If the auxiliary winding is absent or open-circuited, as in Fig. 2.125(b) and (c), the machine can still
operate as a motor provided that some external means is provided to start it and bring it to synchronous
speed. It can also operate as a generator with driven by a prime mover. When connecting to the AC
line, a synchronizing procedure will be needed to prevent uncontrolled transient currents when the
switch S is closed. No such requirement exists for a single-phase generator supplying a passive load.
29
The capacitor can sometimes be omitted if the two phases have sufficiently different resistance/reactance ratios. The term
“split-phase” is sometimes reserved (especially in the U.S.) to refer to such motors without capacitors. Further, the capacitor may
be switched between two values, one for starting and one for running; or there may be a start capacitor and no run capacitor. All
these variants are used with single-phase induction motors, but here the focus is on the analysis of PM motors and so we restrict
attention to a small number of cases, sufficiently general so that other variants can be treated as special cases.
Brushless permanent-magnet machines Page 2.113

Transforming the actual windings into an equivalent balanced winding


The main winding has Nm turns and the auxiliary winding has Na turns. The winding axes are assumed
to be orthogonal (see p. 122), but in general the currents Im, Ia and the voltages Vm, Va are unbalanced,
so that there exist both forward and backward-rotating fields. The rotor rotates in the positive (CCW)
direction so that EMFs and currents in the auxiliary winding lead those in the main winding.
The symmetrical components are obtained by a series of reference-frame transformations or mappings.
First the actual machine [a,m] is replaced by an imaginary balanced one [",$] in which both windings
have Na turns.30 With $ = Nm/Na, the currents are related by

I" 1 0 Ia

I$
' 0 $
@
Im
, i.e. [I",$ ] ' [ C ] [Ia,m ] (2.325)

with inverse

Ia 1 0 I"
' @ , i.e. [Ia,m ] ' [ C ]!1 [I",$ ] . (2.326)
Im 0 1/$ I$
!1
The real transformation matrix [C] is symmetric since [Ct] = [C], but not orthogonal since [Ct] [C] .
!1
To maintain power invariance we must transform the voltages using [Ct] , giving
V" 1 0 Va
' @ , i.e. [V",$ ] ' [ Ct ]!1 [Va,m ] (2.327)
V$ 0 1/$ Vm
with inverse

Va 1 0 V"

Vm
' 0 $
@
V$
, i.e. [Va,m ] ' [ Ct ] [V",$ ] . (2.328)

Symmetrical components — The symmetrical components V1, V2 and I1, I2 are now introduced by the
following mapping from the balanced [",$] machine:
V1 1 j V"
1
' ; i.e., [V]12 ' [S] [V] "$ (2.329)
V2 2 1 j V$

with inverse

V" 1 1 V1
1
' ; i.e. [V",$ ] ' [ S ]!1 [V1,2 ] . (2.330)
V$ 2 j j V2
* !1
Since the complex transformation matrix [S] is unitary, i.e. [St ] = [S] , power invariance will be
maintained if the same transformations are applied to the currents in both directions: i.e.,
I1 1 j I"
1
' ; i.e., [I]12 ' [S] [I] "$ . (2.331)
I2 2 1 j I$

with inverse

I" 1 1 I1
1
' ; i.e. [I",$ ] ' [ S ]!1 [I1,2 ] . (2.332)
I$ 2 j j I2

30
For the purposes of analysis the turns Na and Nm are understood to be the effective turns, i.e. including the winding factors.
All turns are assumed to be in series in each winding.
Page 2.114 SPEED’s Electric Machines

Circuit constraint — pure single-phase motor


Let the auxiliary phase be open-circuited, while the main phase $ is connected to the supply voltage Vs:
I" ' 0 and V$ ' V s . (2.333)

If we substitute eqn. (2.333) into eqn. (2.331) we get


I$
I1 ' j ' I2 . (2.334)
2
Now suppose that at a particular operating point, we can write31
V1 ' Z1 I1 ;
(2.335)
V2 ' Z2 I2 .

Note that Z1 is an apparent impedance since the term Z1I1 incorporates E1. If we substitute eqns.(2.335)
into eqns. (2.330), we get for V" and V$ :

1 I$
V" ' ( Z1I1 Z2I2 ) ' j ( Z1 Z2 ) ; (2.336)
2 2

1 I$
V$ ' ( j Z1I1 j Z2I2 ) ' ( Z1 Z2 ) . (2.337)
2 2

If we now substitute eqns. (2.336) and (2.337) in eqn. (2.329), we get expressions for V1 and V2 :

1 jV s Z1 Z2 Z1
V1 ' ( V" j V$ ) ' 1 ' j 2Vs ; (2.338)
2 2 Z1 Z2 Z1 Z2

1 Z2
V2 ' ( V" j V$ ) ' j 2Vs . (2.339)
2 Z1 Z2

The circuit constraints on I" and V$ expressed by eqn. (2.333) have thus been translated into equivalent
constraints on V1 and V2. When Z1 and Z2 are known, eqn. (2.338) gives V1 directly. Then I1, I2 and I$
follow from eqns. (2.334).
Now Z1 is not fixed, but depends on the EMF and the operating point. If Z2 is assumed known, the
following iterative method can be used for a pre-set value of *, the the load angle in the positive-
sequence diagram (i.e., the angle by which V1 leads E1)
Initialize V1 (for example, set V1 ' Vs)
Repeat
Calculate the positive-sequence phasor diagram using R, Xd and Xq : this gives I1
Calculate Z1 from eqn. (2.335)
Update V1 from eqn. (2.338)
until V1 converges to a steady value
Calculate I$ from eqn. (2.334); then finally I2 from eqn. (2.334) and V2 from eqn. (2.335).

The solution of the positive-sequence system requires the positive-sequence EMF E1, which is obtained
as follows. Suppose that the rotating magnet generates an EMF E" in phase " and E$ in phase $. If the
two windings have equal effective numbers of turns we can write
E$ ' E ; E" ' j E . (2.340)
Eqn. (2.329) can now be used with E" and E$ to determine the positive- and negative-sequence EMFs

E1 ' j 2 E ; E2 ' 0 . (2.341)

31
When the impedance matrix [Z]12 is obtained formally from the transformation as [S][Z]"$[S] 1, off-diagonal impedances appear.
They are omitted from eqns.(2.335) because they cancel in eqns. (2.336) and (2.337).
Brushless permanent-magnet machines Page 2.115

Backward flux suppression


Observe from eqns. (2.338) and (2.339) that
Z2
V2 ' V1 × (2.342)
Z1
which is consistent with eqns. (2.334) and (2.335). The negative-sequence voltage V2 is associated with
a backward-revolving flux, and in order to minimize it we need Z2 << Z1. The cage does this, exactly
as in the single-phase induction motor. As described so elegantly by Veinott, the negative-sequence
current in the cage suppresses the backward flux, so the negative-sequence flux and voltage become
small, while the negative-sequence current is equal and opposite in phase to the positive-sequence
current (eqn. (2.334)). To achieve a low value of Z2 the cage should have the smallest possible resistance.
By combining eqns. (2.338) and (2.342) it can also be shown that

V2 ' V1 j 2 Vs. (2.343)

Circuit constraint — capacitor motor


The circuit is constrained by the equation
Vs ' Vm ' Va j Xc I a . (2.344)
In terms of the [",$] voltages and currents, by substituting from eqns. (2.326) and (2.328) in (2.344),
V s ' $ V$ ' V" j X c I" . (2.345)

Substituting from eqns. (2.330) and (2.332) in (2.345), we get

2 $ V$ ' a1 V1 a2 V2 (2.346)

where
jXc jXc
a1 ' 1 ; a2 ' 1 (2.347)
Z1 Z2
and again,
V1 ' Z1 I1 ; V2 ' Z2 I2 . (2.348)

Combining eqn. (2.346) with eqn. (2.330), we get the circuit constraints in terms of V1 and V2, just as we
did earlier in eqns. (2.338) and (2.339) for the pure single-phase motor. The result is

2 $ j a2
V1 ' Vs (2.349)
$ a1 a2
and Vs
V2 ' V1 j 2 . (2.350)
$
As would be expected, these equations reduce to eqns. (2.338) and (2.339) if Xc is infinite.
Once I1 and I2 are known (from the solution described below), the actual winding currents follow:
1 j
Im ' I$ ' [ I1 I2 ] (2.351)
and $ 2$
1
I a ' I" ' [ I1 I2 ] . (2.352)
2
Page 2.116 SPEED’s Electric Machines

Solution of the positive-sequence system


The positive-sequence system must be solved to obtain Z1. If V1 is assumed known, the phasor diagram
can be used to find I1, and since the positive-sequence system involves a forward-rotating ampere-
conductor distribution synchronized with the rotor, it is convenient to solve the phasor diagram in dq
axes as shown in Fig. 2.89 on p. 79. As for the pure single-phase motor, it is assumed that * and V1 are
known, and the phasor diagram is solved for I1. The torque is given by eqn. (2.248) with m ' 1, because
there is only one positive-sequence “phase” and it accounts for all the positive-sequence power.
The scaling of the positive-sequence EMF Eq1 needs care. When the magnet rotates on open-circuit it
generates the EMF Em ' jTNmM in the main winding and Ea ' TNaM in the auxiliary winding, where
M is the fundamental magnet flux/pole. Thus Ea ' +j Em/$. From eqn. (2.330) we get E" ' Ea ' +j Em/$
and E$ ' Em/$. Then from eqn. (2.329)

1 Em
E1 ' [ E" j E$ ] ' j 2 . (2.353)
2 $

Thus if Eq1 is normally calculated for the main winding (Nm turns), it must be scaled by /2/$ to get the
positive-sequence value. Note that the dq axes in the phasor diagram can be interpreted as a further
reference-frame transformation from [1,2] components into [d,q] components.

Solution of the negative-sequence system


The negative-sequence system must be solved to obtain Z2. For this purpose the motor equations are
written in dq axes fixed to the rotor, and solved for a slip s = 2. (See [41]). The negative-sequence
solution for variable s will also be useful for calculating the asynchronous torque/speed characteristic.
For the stator we use the transformed voltage equations (2.434) and (2.435) on p. 129, where Rd ' Rq ' R,
the stator resistance/phase. Evidently all the impedances in this system, including Z2 itself, are
referred to a winding with a particular number of turns. It is usual to calculate impedances referred
to the main winding with Nm turns. If we take Rm for instance, the resistance of the main winding, and
if we assume that the auxiliary winding has the same total copper cross-section, we can write Ra '
2
Rm/$ . The resistance matrix of the [a,m] machine is then

Ra 0 R m / $2 0
[Ra,m ] ' ' . (2.354)
0 Rm 0 Rm

Using the transformations (2.325) and (2.327), if we first write


[ Va,m ] ' [Ra,m] [Ia,m] (2.355)
we get

[V",$ ] ' [Ct]!1 [Va,m] ' [Ct]!1 [Ra,m] [Ia,m] ' [Ct]!1 [Ra,m] [C]!1 [I",$] (2.356)

from which
[R",$ ] ' [Ct] 1 [Ra,m] [C] 1 . (2.357)

Multiplying out gives the balanced resistance matrix

R m / $2 0
[R",$] ' . (2.358)
0 R m / $2

!1
The same process is applied to obtain [R1,2] from [R",$] using the transformations [S] and [S] in eqns.
(2.329) and (2.332): thus
1
[R1,2 ] ' [S] [R",$] [S] [R",$] . (2.359)
Brushless permanent-magnet machines Page 2.117

From this it appears that in the solution of the negative-sequence system all resistances and
inductances should be referred to the auxiliary winding: for example if they were initially calculated
2
for the main winding turns Nm, they should be scaled by 1/$ .
The voltage and current must also be scaled correctly. The negative sequence voltage is not known a
priori, but suppose the value is V2. Considering the negative-sequence system in isolation, we can set
V1 ' 0. Then from eqn. (2.330),
1 j
V" ' V2 and V$ ' V2 . (2.360)
2 2
Then from eqn. (2.328),
1 j$
V a ' V" ' V2 and V m ' $ V$ ' V2 . (2.361)
2 2
From eqn. (2.361) it appears that if the voltages to be used in the negative-sequence system are to be
referred to the auxiliary winding, they must be scaled from V2 by 1//2.
It is now possible to proceed with the solution of eqns. (2.434) and (2.435) (p. 129) under asynchronous
conditions with a slip of s ' 2. Relative to a balanced 2-phase [",$] machine with Na turns in both
windings, the dq voltages can be written
vd cos 2 sin 2 v"
' @ . (2.362)
vq sin 2 cos 2 v$

Now let the [",$] machine be supplied with positive-sequence voltages


v" ' Vpk cos T t ,
(2.363)
v$ ' Vpk sin T t ,

in which v" leads v$. The negative-sequence behaviour will be imposed by rotating the rotor backwards
relative to the rotating ampere-conductor distribution established by these voltages. If we have
2 ' (1 s) T t (2.364)

and substitute eqns. (2.363) and (2.364) in eqn. (2.362), we get


vd ' Vpk cos s T t and vq ' Vpk sin s T t , (2.365)

which shows that with positive slip, vd leads vq. The rotor is slipping backwards relative to the stator
ampere-conductor distribution, which is rotating at synchronous speed according to eqn. (31). For the
purpose of analysis with eqns. (2.434) and (2.435) (p. 129) we can represent vd and vq as phasor values at
the slip frequency sT:
V d ' Vpk and Vq ' j Vpk . (2.366)

Eqns. (21) are now expressed in phasor terms:


V d ' Rd I d jsT Qd (1 s)TQq,
(2.367)
V q ' Rq I q jsT Qq (1 s)TQd,

in which
Q d ' Ld ( j s T ) I d ,
(2.368)
Q q ' Lq ( j s T ) I q .

The synchronous inductances Ld(jsT) and Lq(jsT) are functions of frequency because of the coupled
rotor circuits in each axis. We can derive expressions for Ld(jsT) and Lq(jsT) as follows. First the stator
and rotor flux-linkages are written in the d and q axes as
Page 2.118 SPEED’s Electric Machines

Q d ' Ld I d Md I dr ,
(2.369)
Q dr ' Md I d Ldr I dr .

where Ldr is the rotor self-inductance in the d-axis referred to the same circuit as Lds for the stator, and
similarly for Lqr and Lqs. The mutual inductance between these circuits is written as

Md ' kd Ld Ldr (2.370)

where kd is the d-axis coupling coefficient; kd < 1. Similar equations can be written for Mq.
The rotor circuits are short-circuited, so that in the d-axis for example
0 ' Rdr I dr j s TQ dr
(2.371)
' Rdr I dr j s T ( Md I d Ldr I dr )

This equation can be used to eliminate Idr from eqn. (2.369). First
j s T Md
I dr ' Id, (2.372)
Rdr j s T Ldr
then after some manipulation

j s T kd2 Td
Lds (j s T) ' Lds 1 , (2.373)
1 j s T Td
with
Ldr
Td ' , (2.374)
Rdr

the so-called “open-circuit” rotor time-constant. Equations similar to (2.373) and (2.374) can be written
for the q-axis. At any particular slip frequency Ld(jsT) and L(jsT) are calculated and when the rotor
current is eliminated using eqn. (2.372), equations (2.367), (2.368) become
(Rd j s T Ld) I d (1 s ) T Lq I q Vd ' 0,
(2.375)
(1 s ) T Ld I d (Rq j s T Lq) I q Vq ' 0.

with the voltages Vd and Vq given by eqn. (2.366). These can be solved for Id and Iq, whence Qd and Qq
follow from eqns. (2.368), and
Vd jVq
Z2 ' . (2.376)
Id jIq

Finally the average negative-sequence torque is computed as


( ( (2.377)
Tns ' p Re [ Q d I q Qq Id ].

Again, as in the positive-sequence case, the number of “phases” does not appear in eqn. (2.377) because
there is only one “phase” in the negative-sequence system.

Combined solution of the positive and negative sequence systems


So far we have treated the positive- and negative-sequence systems separately. In actual operation the
link between them is provided by eqn. (2.349), which “ties” the positive-sequence voltage V1 to the
supply voltage Vm, and eqn. (2.350), which does the same for the negative-sequence voltage V2. Only a
single control parameter is needed to set the “level of operation”, and the one chosen is the positive-
sequence load angle * (see Fig. 2). The algorithm is expressed on p. 114.
Brushless permanent-magnet machines Page 2.119

“Natural” symmetrical components


Unfortunately the symmetrical component transformations on p. 113 give rise to a set of phasors for the
positive and negative-sequence components that are hard to interpret. This problem can be remedied
by a further transformation into a second set of symmetrical components U1 and U2 for voltages, and
Y1 and Y2 for currents, as follows:

U1 j 1 V"
1
' ; i.e., [U]12 ' [N] [V] "$ (2.378)
U2 2 j 1 V$
with inverse

V" j j U1
1
' ; i.e., [V] "$ ' [N] [U]12 (2.379)
V$ 1 1 U2
and similarly

Y1 j 1 I"
1
' ; i.e., [Y]12 ' [N] [I] "$ (2.380)
Y2 2 j 1 I$
with inverse

I" j j Y1
1
' ; i.e., [I] "$ ' [N] [Y]12 (2.381)
I$ 1 1 Y2

By susbtituting eqn. (2.330) in eqn (2.378) it is easy to show that

U1 1 0 V1
j
' ; i.e., [U]12 ' [N] [S] 1 [V] 12 (2.382)
U2 2 0 1 V2
or
j j
U1 ' V1 and U2 ' V2 (2.383)
2 2
Eqn. (2.379) is written
Main V$ ' U1 U2
(2.384)
Aux V" ' j U1 j U2

Especially the first of these equations makes it easier to interpret the phasor diagram, since the voltage
across the main winding $ appears as the vector sum of the positive- and negative-sequence components
U1 and U2. In contrast, it is hard work to interpret eqns. (2.330) in terms of the components V1 and V2.
If the EMFs are given by eqn. (2.340) on p. 114, the new transformation gives
1
E1 ' j ( j E$) E$ ' E$ ' j E q1 ; E" ' 0 . (2.385)
2

Also, since I$ ' Y1 Y2, we can write the following equations for interesting special cases:
I$
(a) If I" ' 0 then Y1 ' Y2 ' ;
2 (2.386)
(b) If Y2 ' 0 then I$ ' Y1 and I" ' j Y1 .

Case (a) is the pure single-phase motor, while case (b) is a perfectly balanced motor. These equations
are easy to check in the phasor diagram. Finally, it may be desirable to rotate the entire phasor
diagram so that E1 appears on the q-axis, rather than having V$ on the real axis as it was in [38].
Page 2.120 SPEED’s Electric Machines

Interpretation of the phasor diagram

Fig. 2.126 Phasor diagram of split-phase PM capacitor motor

Fig. 2.126 shows the phasor diagram of a split-phase PM capacitor motor, including the “natural”
symmetrical components and the "$ components. The orientation of the whole diagram is such that
the positive-sequence EMF E1 is on the q-axis together with the EMF Eq1 in the main phase (1).
The main and auxiliary phase voltages Vm and Va are held almost at right-angles by the capacitor
voltage Vc, indicating relatively good balance. This is also evident from the small value of V2 (about
6 V), relative to the positive-sequence voltage V1 (308 V). So the backward-rotating flux is much smaller
than the forward-rotating flux.
The "$ voltages are also shown. We can see that V$ ' V1 V2, in accordance with eqn. (2.384).32 Note
that V" ' Va, but V$ ' Vm/$ ' Vm × 1@4 in accordance with eqn. (2.327) on p. 113.
The angle between the main and auxiliary currents is notably less than 90E, indicating that the currents
are not so well balanced as the voltages. This is also evident from the fact that I2 is quite noticeable
compared to I1. We can see that I$ ' I1 I2, in accordance with eqn. (2.381). Similarly, I" ' Ia, but I$ '
$Im ' Im/1@4, in accordance with the current eqn. (2.325) on p. 113. Also note that the auxiliary current
leads the capacitor voltage Vc by exactly 90E.
Fig. 2.126 includes the entire positive-sequence phasor diagram E1, RI1, jXdId1, jXqIq1, V1 together with
I1 and its components Id1 and Iq1. All these phasors are scaled to the number of effective turns on the
auxiliary winding, by virtue of the choice of transformations (2.325) and (2.327) on p. 113. They are
therefore scaled by 1/$ (for voltages) or by $ (for currents) relative to the effective number of turns on
the main winding. Thus, for example, E1 ' Eq1/$ ' 1@4Eq1, where Eq1 is the fundamental EMF generated
by the magnet flux in the main winding. In the induction motor, the components jXdId1 and jXqIq1
would be replaced by a single phasor representing the voltage drop across the stator leakage reactance,
but otherwise the diagram for a capacitor induction motor is similar.
Finally the supply current Is is the sum of Im and Ia. In Fig. 2.126 it lags behind the supply voltage Vm
by about 8E, giving an overall power factor close to 1. The main and auxiliary phases are working at
a lower power factor, with angles of about 33E between Vm and Im, and about 53@7E between Va and Ia.
* *
Both windings are absorbing reactive power. The sum of the reactive powers Im(VmIm ) Im(VaIa ) is
* *
equal to the sum of the reactive powers Im(VmIs ) Im(VcIa ) obtained from the supply and the capacitor
respectively. Thus the capacitor is acting in a real sense, as a power-factor correction capacitor.

32
The symbols V and I are restored in Fig. 2.126. Symbols U and Y were used to differentiate the natural symmetrical components
from the original ones, but there is no need to retain them once the point has been made.
Brushless permanent-magnet machines Page 2.121

Torques
The positive-sequence system gives rise to a forward torque Tf that has a magnet-alignment component
and a reluctance component :33
p
Tf ' E I (Xd Xq) Id1 Iq1 . (2.387)
T 1 q1
where Id1 and Iq1 are the d- and q-axis components of the positive-sequence current I1.
The negative-sequence system gives rise to a backward torque Tb. This is exactly analogous to the
asynchronous torque of an induction motor when the slip is s ' 2, in which half the rotor loss (W2R/2)
is supplied across the airgap and half is supplied from the shaft. Since the speed of the rotor relative
2
to the backward-rotating field is 2T/p, with T ' 2B f, the slip is 2 and the I R loss in the rotor cage is

2T
W2R ' Tb . (2.388)
p

The total electromagnetic torque applied to the rotor is


Te ' Tf Tb . (2.389)
This “electromagnetic” torque is available at the shaft after subtracting friction and windage losses.
It is associated with the electromechanical power Pe which is given by

T T T W2R
Pe ' T ' (T Tb) ' T . (2.390)
p e p f p f 2
Pe is by definition the “synchronous watts” of induction-motor theory, as it gives the actual
electromagnetic torque Te when divided by T/p. The power crossing the airgap is

T W2R T T T (2.391)
Pg ' Pe W2R ' T ' (T Tb) ' T T .
p f 2 p f p f p b
2
Eqns. (390) and (391) show that half the I R loss in the rotor cage is provided “electrically” across the
gap as a component of Pg, while the other half appears as a deduction from the electromechanical power
Pe. As in the induction motor, Pg exceeds Pe by the amount of W2R.
The term (Tf Tb) represents the combined action of forward and backward fields in transmitting
power across the airgap; but it does not represent a physical torque because Tf and Tb act in opposite
directions and are associated with fields that are moving in opposite directions. The actual reaction
torque on the stator is Te ' Tf Tb, the same as the electromagnetic torque on the rotor, but acting in
the opposite direction.

Symmetrical components of the simple single-phase circuit on p. 111


The current is given by eqn. (2.323), and the positive and negative sequence currents by eqn. (2.334). The
positive-sequence EMF is given by

E1 ' j 2 E ' j 2 j Eq1 ' 2 Eq1 . (2.392)

The positive-sequence voltage is given by

1 1
V1 ' (V" j V$) ' [ Eq1 j ( j Eq1 (R j X) I$ )] ' E1 (R j X) I1 . (2.393)
2 2

The load-angle * for the positive-sequence system is equal to arg(V1) arg(E1). Similarly the negative-
sequence voltage can be shown to be V2 ' (R jX)I2 ' j//2 (R jX)I$ with E2 ' 0.

33
Note again that the number of phases does not appear in eqn. (2.387), because there is only one “positive-sequence” component
producing torque. The RMS values of the positive-sequence EMF E1 and the d- and q-axis component currents Id1 and Iq1 already
contain a /2 factor arising from the transformation from phase variables to symmetrical components in eqn. (2.331). When they
appear in eqn. (2.387) they appear in pairs, so the /2 factors combine to give a factor of 2, which is, in effect, the number of phases.
Page 2.122 SPEED’s Electric Machines

Fig. 2.127 Non-orthogonal windings

Non-orthogonal windings
Non-orthogonal windings may be used to give some design flexibility in the value of capacitor required
for balanced operation. For non-orthogonal windings the reference transformations (2.325) and (2.326)
must be modified to include the angle ., Fig. 2.127. The transformation for current is developed on the
basis of equal MMF on two orthogonal axes. Again referring all windings to the auxiliary turns Na,

Na ia Nm im cos . ' Na i" i" 1 $ cos . ia


or ' @ or [i",$] ' [ N ] [ia,m] (2.394)
Nm im sin . ' Na i$ i$ 0 $ sin . im

with inverse

ia 1 ctg . i"
' 1 @ or [ia,m] ' [ N ] 1 [i",$] . (2.395)
im 0 csc . i$
$

Evidently [N] is not orthogonal. The voltage transformation must therefore be

v" 1 0 va
1
[v",$] ' [ Nt ] [va,m] or ' 1 @ . (2.396)
v$ ctg . csc . vm
with inverse $

va 1 0 v"
[va,m] ' [ Nt ] [v",$] or ' @ . (2.397)
vm $ cos . $ sin . v$

These transformations must replace eqns. (2.325-2.328). The subsequent theory follows the same lines
as already described. Note that in [40] the non-orthogonal transformation equivalent to eqn. (2.394) is
incorrectly applied to both the voltages and the currents, rendering the subsequent equations incorrect
unless . ' 90E.

Analysis : single-phase motors — asynchronous and starting


The asynchronous operation can be calculated exactly as for the negative-sequence components on the
preceding pages, with the average asynchronous torque given by eqn. (2.377) on p. 118. The slip s is
varied to cover the speed range of interest. The positive-sequence components are ignored, on the basis
that there is no average positive-sequence torque at non-synchronous speeds.
The starting performance can be analyzed as for the polyphase motor discussed earlier, with suitable
circuit constraints on the terminal voltages : see [40].
Brushless permanent-magnet machines Page 2.123

Transient performance
The calculation of transient performance requires the simultaneous solution of the differential voltage
equations for each circuit in the machine. For the three phases of a 3-phase machine these equations
are eqns. (2.433) on p. 128. Especially for salient-pole machines, it is common to transform these
equations into a reference-frame that is fixed to the rotor. The dq transformation (also known as Park's
transformation) is widely used for this, and it is very well documented (see, for example, [9]).
Although there are many variants of the dq transformation, the differences are confined to the details
of scaling and phase sequence, and related questions of power invariance and orthogonality, [20]. All
that is required for a particular analysis is to have a consistent set of transformation equations and
their inverses. The 3-phase dq transformation used in SPEED is given on p. 128, and it results in the
standard dq stator voltage equations (2.434) and (2.435).
When the machine has a cage winding on the rotor, additional circuit equations must be written. In
principle it is possible to write a separate voltage equation for each pair of bars, or even for each bar,
in the cage. However, the simplest approach is to treat the cage with only two circuits — one for the
d-axis and one for the q-axis. Line-start motors are usually salient-pole motors, so the properties of the
d- and q- circuits are different. In every other aspect they are similar to the rotor circuit parameters
used in the theory of induction motors, where the use of multiple circuits for the cage is extremely rare.
In dq axes the rotor circuit equations are simply
vD ' 0 ' RD iD p RD ;
(2.398)
vQ ' 0 ' RQ iQ p RQ .

Note that vD ' vQ ' 0, because the cage is short-circuited by its end-rings. We now need relationships
linking all the flux-linkages with all the currents. In the d-axis the flux-linkages are Rd for the stator
and RD for the rotor, and likewise Rq and RQ in the q-axis. Then we have
d&axis Rd ' Ld id LaD iD stator
RD ' LaD id LD iD rotor
(2.399)
q&axis Rq ' Lq iq LaQ iQ stator
RQ ' LaQ iq LQ iQ rotor.
The procedure is to integrate eqns. ( 2.398 ) together with eqns. (2.434) and (2.435) on a step-by-step basis
using Euler's method or an equivalent such as the Runge-Kutta method.34 At the end of each step, the
currents are obtained from eqns. (2.399), which can be inverted algebraically as they are two
independent pairs of simultaneous equations. The torque is calculated from eqn. (2.439) for a three-
phase machine; for a 2-phase, 1-phase, or split-phase machine, the same equation is used but without
the 3/2 coefficient.
If the speed is free to vary, two mechanical differential equations must be added :
T
p Tm ' (2.400)
and J

p 2 ' Tm , (2.401)
where 2 is the rotor position, Tm is the angular velocity, J is the total inertia of the motor plus load, and
T is the nett torque, equal to the electromagnetic torque minus all the mechanical loss torque
components (friction, windage, etc.)
The standard dq stator voltage equations (2.434) and (2.435) together with the equations on this page are
quite general in the sense that they may be used with any number of phases. Moreover, there is no
constraint on the terminal voltage waveforms, so the dq equations are not restricted to AC sinewave
operation.

34
With flux-linkages as state variables, it is usually possible to use Euler's method.
Page 2.124 SPEED’s Electric Machines

dq transformations for 2-phase and split-phase machines; capacitor motors


In the solution of transients for two-phase and single-phase machines, including split-phase capacitor
motors, we need to be able to relate the dq-axis quantities to the phase quantities and so we need the
transformations between dq-axes and the "$ axes which we have already used for the steady-state
analysis of the capacitor motor. The "$ axes are identical to those of the symmetrical 2-phase motor,
in which phase 1 corresponds to the " phase and phase 2 to the $ phase. The phase sequence is 1-2 or "-$.
In the split-phase capacitor motor, the " phase is the auxiliary and the $ phase is the main; again, the
phase sequence is "-$. The transformations are
id ' i" cos 2 i$ sin 2 ;
(2.402)
iq ' i" sin 2 i$ cos 2 .

with inverse i" ' id cos 2 iq sin 2 ;


(2.403)
i$ ' id sin 2 iq cos 2 .

The same transformations are used for flux-linkages and voltages. The transformation actually
represents a projection of MMFs which can be expressed by Fig. 2.128 with " written instead of a, $
instead of m, and . ' 90E. Similarly, Fig. 2.69 on p. 56 represents the 3-phase projection on to dq axes.
For completeness it is perhaps worth adding the transformations between 3-phase abc and 2-phase
stationary "$ components. These are already implicit in the 2-phase and 3-phase dq transformations
and can easily be obtained by substitution:
1
i" ' i a ; i$ ' (ib ic ) . (2.404)
with inverse 3

1 3 1 3
i a ' i" ; ib ' i" i$ ; ic ' i" i$ . (2.405)
2 2 2 2
In general the transformation of three variables into only two variables is not complete, but 3-phase
electrical machines are usually assumed to have a 3-wire connection which enforces the condition
ia ib ic ' 0 (2.406)

so that only two currents are independent. Evidently the same constraint must apply to the voltages
and flux-linkages.
Terminal voltage constraints for the capacitor motor
The phase voltages vm and va in the capacitor motor are constrained by Fig. 2.125, so that
vm ' vs ' va vc and is ' im ia . (2.407)
where vc is the capacitor voltage and vs is the supply voltage. When this equation is transformed into
d,q axes, with $ the turns ratio (main/aux), we get

sin .
vd ' vs (1 cos .) cos 2 sin 2 vC cos 2 ;
$
(2.408)
sin .
vq ' vs (1 cos .) sin 2 cos 2 vC sin 2 .
$

which is convenient for inclusion in the step-by-step integration. Similarly


csc .
im ' id sin 2 iq cos 2 ;
$
(2.409)
ctg . ctg .
ia ' id cos 2 sin 2 iq sin 2 cos 2 .
$ $

which is convenient for recovering the phase currents at each step.


Brushless permanent-magnet machines Page 2.125

Synchronous operation

Fig. 2.128 Reference axes defined in the 1985 paper

For synchronous operation the rotor position 2 advances in synchronism with Tt, but its phase must
be carefully coordinated with the phasor diagram and the calculated steady-state operation.
Fig. 2.69 on p. 56 shows the reference axes for the 3-phase motor and Fig. 2.128 for the split-phase motor.
The rotor rotates in the anticlockwise (positive) direction, so that voltages and currents in the auxiliary
(a) normally lead the corresponding voltages and currents in the main (m).
The position of a rotor d-axis is defined relative to the axis of the auxiliary winding by
2 ' Tt 20 elec rad . (2.410)
The position of the same south d-axis relative to the main winding is
2m ' 2 . ' Tt 20 .. (2.411)
When 2m ' 0, the d-axis is aligned with the main winding, and the magnet flux-linkage in the main
winding is at its maximum value Q1Md. At other rotor positions the magnet flux-linkage in the main
winding is
Rm ' Q1Md cos 2m ' Q1Md cos (T t 20 .). (2.412)

The EMF in the main winding is eq1 ' dRm/dt


d Rm
eq1 ' ' T Q1Md sin (T t 20 .) ' T 2 Eq1 sin (T t 20 .) . (2.413)
dt
The terminal voltage of the main winding has a peak value vpk ' /2Vs, and its phase is defined by the
load-angle *m, which can be seen in the phasor diagram as the angle between the phasors Vm and Eq1.
Keeping the same reference frame as above, it follows that
vm ' vpk sin (T t 20 . *m ) . (2.414)

For symmetrical 3-phase and 2-phase motors, the distinction between 2 and 2m disappears, because eq1
is defined for phase 1, which is the same as phase a in the above. Instead of (eqn. 2.414) we have
v1 ' vpk sin (T t 20 *m ) . (2.415)
For starting calculations, or other calculations at speeds other than synchronous speed, there is no a
priori connection between 2 and Tt. In this case we can use
v1 ' vpk sin T t . (2.416)
and the same equation for vm in the split-phase motor, while the rotor position is the result of the
integration

2 ' Tm dt 20 . (2.417)

In all cases, 20 is the rotor position at t ' 0.


Page 2.126 SPEED’s Electric Machines

2.19 TORQUE
We have already used several of the key formulas for the electromagnetic torque. For squarewave
drives with “two phases on”, we used eqns. (2.8) and (2.10) on p. 4, based on the concept of the ideal
energy-converter expressed in eqn. (2.230) on p. 74. We then extended the concept of the ideal energy-
converter to derive eqn. (2.231) on p. 74 for the single-phase sinewave motor, followed by eqns. (2.235)
and (2.236) on p. 75. for 2-phase and 3-phase sinewave motors. We developed these formulas extensively
for average torque in a sinewave motor under current-controlled conditions [eqn. (2.247) on p. 80] and
voltage-limited conditions [eqn. (2.263) on p. 85], and subsequently developed the normalized torque
equations [(2.277 280) on p. 87ff]. We have also identified the negative-sequence torque in the split-phase
line-start motor, using induction-motor theory. In all these formulas we were careful to distinguish
between instantaneous torque and average torque, which are related by eqn. (2.11) on p. 4.
All the above-mentioned torque formulas are specific to particular cases, and although they include the
most common practical cases, there remain many other forms of torque calculation which are useful
in computation, or which provide further physical insight. We are now ready to embark on a deeper
study of torque production. In following sections this study will be carried through into the wider
subject of dynamic simulation and the coupling with finite-element analysis.

Torque from coenergy


Textbooks often quote the most general form of the torque equation in the form
MWc ([i],2)
Te ' , (2.418)
M2
where Wc is the coenergy, 2 is the rotor position, and [i] is the vector of all phase currents. In a single-
phase reluctance machine or actuator with no magnets, Wc is defined by

Wc ' R(i,2) di , (2.419)

but in permanent-magnet machines the definition is more complex, as discussed later. Except in very
simple cases, eqn. (2.418) does not give a practical method for torque calculation. This is due to the
difficulty of computing Wc, and the even greater difficulty of determining its partial derivative.
Furthermore, eqn. (2.418) is for instantaneous torque, so it must be repeated or integrated to get the
average electromagnetic torque. Unlike the torque formulas discussed earlier, it provides very little
physical insight. Quite possibly its only immediate practical application is as an embedded torque-
calculation algorithm in the finite-element method, when calculating cogging torque.

Torque in terms of circuit parameters


In magnetically linear machines it is convenient to derive the torque equation from power-balance
considerations in terms of equivalent-circuit parameters together with EMF and current. In a single-
phase machine this gives the familiar formula for instantaneous electromagnetic torque:
1 2 dL
Te ' e i i . (2.420)
2 d2
The first term is the alignment torque which we have already seen in connection with the ideal energy-
converter of eqn. (2.230), which operates without loss or storage. The second term is the reluctance
torque which arises when the circuit has inductance that varies with rotor position 2. In nonsalient-
pole machines the inductance does not vary with rotor position and the reluctance torque is zero. Note
that while eqn. (2.420) is usually derived by considering the electromechanical power balance, it can
also be derived from eqn. (2.418).
Brushless permanent-magnet machines Page 2.127

Eqn. (2.420) can be generalized for more phases. For example, with three phases a,b,c

1 1 2 dLa dLb dLc dLab dLbc dLca


Te ' ea ia eb ib ec ic ia ib2 i c2 ia ib ib ic ic ia . (2.421)
Tm 2 d2 d2 d2 d2 d2 d2
In a 3-phase machine the phase self-inductances Laa, Lbb, Lcc and the phase-to-phase mutual inductances
Lab, Lbc, Lca can be written
Laa ( 2 ) ' L0 L2 cos 2 2
Lbb ( 2 ) ' L0 L2 cos ( 2 2 2B/3)
Lcc ( 2 ) ' L0 L2 cos ( 2 2 2B/3)
(2.422)
Lab ( 2 ) ' Lba ' M0 M2 cos ( 2 2 2B/3)
Lbc ( 2 ) ' Lcb ' M0 M2 cos ( 2 2)
Lca ( 2 ) ' Lac ' M0 M2 cos ( 2 2 2B/3) .
This equation is the same as eqn. (2.157) on p. 57, with some of the terms grouped together for simplicity.
Note that the phases have been labelled a,b,c instead of 1,2,3, and the variation of inductance with rotor
position 2 is restricted to a single term in 2, representing a double-frequency harmonic variation of
inductance. The reluctance torque is calculated from the dL/d2 terms in eqn. (2.421), and for sinewave
operation it can be evaluated by substituting eqns. (2.238) (p. 75) and (2.422) into eqn. (2.421). For this
purpose we can write
2 ' Tt ( (2.423)
without loss of generality, where ( is constant at synchronous speed. The phase of the current relative
to the rotor position is defined by (. Then eqn. (2.421) gives

L2
Trel ' 3 I 2 M2 sin 2 ( . (2.424)
2

If L2 ' M2, as would normally be the case, this simplifies to


3
Trel ' 3 I 2 ×
L2 sin 2 ( . (2.425)
2
This is consistent with the reluctance torque obtained using dq-axis theory in eqn. (2.261), where L2 '
(Ld Lq)/3. With balanced operation there is no torque ripple: the reluctance torque is constant. The
same is true in a balanced 2-phase motor.
If ib ' ic ' 0, we have single-phase operation, which is highly unbalanced, and eqn. (2.421) gives

1 1
Trel ' L2 I 2 sin ( sin 2 2 sin ( 4 2 2() (2.426)
2 2
which shows that the single-phase reluctance motor has both a 2nd and a 4th harmonic torque ripple at
synchronous speed. The average is only 1/9 of that of the 3-phase machine. A reduction of 1/3 is
attributable to two absent self-inductance terms, but even with all three currents flowing the self-
inductance terms contribute only 1/3 of the total, so the overall reduction is by a factor of 9.
In the application of eqn. (2.421), eqn. (2.425) is an example of the derivation of a constant steady-state
torque component, while eqn. (2.426) is an example of the computation of a torque component that
varies with rotor position. Both cases require a good deal of algebraic manipulation, which renders the
method slow and susceptible to errors. Moreover, it is limited to magnetically linear machines.
Although it provides more physical insight than the direct use of eqn. (2.418), two alternative methods
to be developed below provide even more physical insight, together with the ability to deal with
saturation and the ability to calculate average torque directly.
The two alternative methods are (a) the dq transformation and (b) the energy-conversion loop method,
also known as the i-R diagram method, or “the method of the flux-linkage/current diagram”.
Page 2.128 SPEED’s Electric Machines

Torque in terms of dq-axis quantities


The dq transformation is presented next in sequence because historically it was developed to overcome
the limitations of eqn. (2.421) particularly when used with 3-phase salient-pole machines. We have
already used it for steady-state sinewave operation in its time-averaged form in eqn. (2.247) and related
equations on p. 80, including voltage-limited conditions [eqn. (2.263) on p. 85], and subsequently also
in the normalized torque equations [(2.277 280) on p. 87ff].
What follows is a formal exposition of the dq transformation, leading to the instantaneous form of the
torque equation in dq axes.
The dq-axis transformation is
2
Rd ' [ Ra cos 2 Rb cos (2 2 B/3) Rc cos (2 2 B/3)] (2.427)
3

2
Rq ' [ Ra sin 2 Rb sin (2 2 B/3) Rc sin (2 2 B/3)] (2.428)
3

and this is the same for voltages, flux-linkages, and currents. 2 is the angle between the d-axis and the
axis of phase a in electrical radians, Fig. 2.69. This transformation, generally known as Park's
transformation, appears in the literature in various forms with different coefficients [9, 19, 20]; but its
general aim is always the same: namely to project the actual phase quantities from the stationary
reference frame a,b,c to the rotating frame dq which is fixed to the rotor. Its inverse is given by
Ra ' Rd cos 2 Rq sin 2
Rb ' Rd cos ( 2 2 B /3 ) Rq sin ( 2 2 B /3 ) (2.429)
Rc ' Rd cos ( 2 2 B /3 ) Rq sin ( 2 2 B /3 ) .

1
The dq transformation can be written as a matrix operator T, which together with its inverse T is

C C C C S 1

2 S S S 3 C S 1
T ' ; T 1
' TN ' (2.430)
3 1 1 1 2
2 2 2 C S 1

where C ' cos 2, C ' cos (2 2B/3), C ' cos (2 2B/3), and S ' sin 2, S ' sin (2 2B/3), S ' sin (2 2B/3).
Note that the zero-sequence component is included, because in general 3 independent abc variables
must be transformed into 3 dq0 variables. With a 3-wire connection only two of the three abc variables
are independent and the zero-sequence components are generally ignored; in any case the zero-sequence
component plays no part in the torque equation. If vabc is the column vector of instantaneous voltages
[va, vb, vc], [ia, ib, ic] the column vector of instantaneous phase currents, etc., the transformations can
be written in the concise form
v dq0 ' T v abc (2.431)

and their inverses similarly. The instantaneous power is


3 3
pabc ' vabcN iabc ' {T 1 vdq0}N{T 1 idq0} ' vdq0N T T 1idq0 ' vdq0N idq0 ' pdq0 . (2.432)
2 2
1
Note that T ' (3/2)TN, i.e., the transformation is nearly orthogonal, but not quite, because of the 3/2
factor. Consequently the power is “not invariant under the transformation” — fancy language to
remind us to put the 3/2 factor when we calculate power (and torque) in dq coordinates.
If we write p for the operator d/dt, the voltage equations of the individual phases are:
va ' R ia p Ra ; vb ' R ib p Rb ; vc ' R ic p Rc . (2.433)
Brushless permanent-magnet machines Page 2.129

If we write eqns. (2.427) and (2.428) for voltages, and substitute eqns. (2.433) into them, with p as a live
operator (d/dt), and p2 ' T, we get
vd ' R id p Rd T Rq (2.434)
and
vq ' R iq p Rq T Rd . (2.435)

Eqns. (2.434) and (2.435) appear to be valid regardless of the winding distribution, since no assumption
about the winding distribution is needed to derive them.
The derivation of vd is as follows:
2
vd ' [ (R ia p Ra) cos 2 (R ib p Rb) cos (2 2 B/3) (R ic p Rc) cos (2 2 B/3)]
3
2
' R [ia cos 2 ib cos (2 2B/3) ic cos (2 2B/3)]
3
2
pRa cos 2 Ra sin 2 @ p2
3
(2.436)
2
pRb cos (2 2B/3) Rb sin (2 2 B/3) @ p2
3
2
pRc cos (2 2B/3) Rc sin (2 2 B/3) @ p2
3

' R id p Rd T Rq .

and similarly for vq.

An expression for the instantaneous electromagnetic torque can be deduced from the power balance
in the steady-state at constant speed. Under these conditions pRd ' pRq ' 0, and if resistance is ignored
[as it was in the ideal energy-converter in eqn. (2.230)], we get
vd ' T Rq ; vq ' T Rd . (2.437)
If these are substituted in eqn. (2.432) we get
3 3
pe ' pabc ' (vd id vq iq) ' T ( Rq id Rd iq), (2.438)
2 2

and it follows that


pe 3
Te ' ' p ( Rd iq Rq id) , (2.439)
T/p 2
where p is, of course, the number of pole-pairs and not the derivative operator d/dt. Under steady-state
sinusoidal conditions, suppose that

Rd ' 2 Qd cos T t and iq ' 2 Iq cos T t , (2.440)

where Qd and Iq are RMS values. The time-average value of the product Rd iq is equal to Qd Iq, showing
that the time-average torque can be obtained from eqn. (2.439) in terms of RMS quantities as
Te ' m p ( Qd Iq Qq Id) , (2.441)
exactly the same as eqn. (2.247) on p. 80. Note that the number of phases m has been substituted in place
of 3 in this equation; this permits the expression to be used for 2-phase motors when the flux-linkages
and currents are per-phase quantities.
In the following section, it is shown how eqn. (2.441) can be evaluated using flux-linkages computed by
the finite-element method.
Page 2.130 SPEED’s Electric Machines

Computation of torque from non-rotating finite-element solutions (Sinewave drive)


With sinusoidal phase currents, the average electromagnetic torque can be computed from eqn. (2.247)
on p. 80 [or eqn. (2.441), which is the same]. The flux-linkages must be those which generate the airgap
voltage, that is, Q1d and Q1q calculated from the fundamental components of the airgap flux resolved
along the d- and q-axes respectively. The result does not include cogging or any harmonic torques.
Eqn. (2.441) does not require the d-axis flux-linkage to be resolved into the separate components
contributed by the magnet and the d-axis stator current as expressed in eqn. (2.243). Neither does it
even require a knowledge of Xd or Xq: these are necessary only when the current has to be computed
from the voltage equation such as eqn. (2.249) on p. 81. If the current is known in magnitude and
direction, as is normally the case with a current-regulated drive, then Xd and Xq are not required.
The fundamental flux-linkages Q1d and Q1q are obtained from the finite-element procedure as follows.
First, the d-axis is aligned with the axis of phase a. Then the field solution is obtained for a range of
values of I and ( [see eqn. (2.242) on p. 80]. For each solution the airgap flux-density distribution is
Fourier-analyzed to give B1d and B1q, where B1d is the peak value of the d-axis component and B1q is the
peak value of the q-axis component. Then Qd1 and Qq1 are obtained using the form of eqn. (2.239), i.e.
kw1 Tph B1d D Lstk kw1 Tph B1q D Lstk
Q1d ' ; Q1q ' (2.442)
2 p 2 p

The leakage components IdLF and IqLF should be added to Q1d and Q1q respectively. If the leakage
reactance is saturable, the flux-linkage/current diagram technique should be used instead (see below).
The method has the advantage that multiple solutions covering a range of both I and ( can be obtained
quickly without rotating the rotor and therefore without re-meshing, to produce a set of torque curves
such as those illustrated in Fig. 2.129.

Fig. 2.129 Torque vs. torque angle (, for various currents

The values of currents in the individual slots are obtained from the conductor location vector (CLV), that
is, an array C which has m columns, one for each phase, and n rows, where n is the number of slots. The
th th th
value of the (j,k) element is the number of conductors belonging to the j phase in the k slot. C[j,k]
th
is signed according to the direction of the conductors. Then for the k slot the total current is
Ik ' C [a,k] ia C [b,k] ib C [c,k] ic . (2.443)
Instantaneous phase currents ia, ib and ic are obtained from the inverse dq transformation, eqn. (2.429),
using peak values Id ' /2 I sin ( and Iq ' /2 I cos (. If the d-axis is aligned with phase a, 2 ' 0 and eqn.
(2.429) simplifies to
ia ' Id
ib ' Id cos (!2 B / 3) ! Iq sin (!2 B / 3) ' (!Id 3 Iq) / 2 (2.444)
ic ' Id cos (2 B / 3) ! Iq sin (2 B / 3) ' (!Id ! 3 Iq) / 2 .
Brushless permanent-magnet machines Page 2.131

Torque computed from the flux-linkage/current loop


During one electrical cycle of operation the
electromechanical energy conversion per phase is
equal to the area W of a closed loop traced by the point
whose coordinates are (i,R), the phase current and
phase flux-linkage. With m phases and p electrical
cycles per revolution, the average torque is therefore
mp
Te ' × W. (2.445)
2B

Fig. 2.130 shows an example for a sinewave machine.


Suppose this machine has a sinusoidal airgap voltage
and a sinusoidal current. In one phase,
Fig. 2.130 i-psi loop
v ' 2 V cos T t ;
(2.446)
i ' 2 I cos (T t N) .
By Faraday’s law the airgap voltage v is the time derivative of the flux-linkage R in the winding, so

2V
R ' sin T t ' 2 Q sin T t , (2.447)
T
where Q is the RMS flux-linkage, V/T. Now the work done in one cycle by the vi product is given by
T 2B 2B
1 1
W ' v i dt ' v i d2 ' 2 Q I cos 2 cos (2 N) d2
0 T 0 T 0 (2.448)
' 2 B Q I cos N .
This is the area of the loop formed by plotting R vs. i, Fig. 2.130. The mean electromagnetic torque is
given by W/(2B/p) where p is the number of pole-pairs. If there are m phases balanced, then

mp
Te ' m p Q I cos N ' V I cos N . (2.449)
T

The induced voltage v includes not only the rotational EMF produced by the rotation of the magnet, but
also the EMF induced by variation in phase inductance. Consequently Te includes the reluctance torque.
Now suppose the induced voltage and the current contain harmonics. It is enough to consider just one
th
representative harmonic — the n — by adding it to the fundamental: thus

v ' 2 V1 cos T t 2 Vn cos n T t ;


(2.450)
i ' 2 I1 cos ( T t N1) 2 In cos ( n T t Nn ) .

The integral in eqn. (2.448) results in


W ' 2 B ( Q1 I1 cos N1 n Qn In cos Nn ) , (2.451)
where Qn ' Vn/nT. Because of the othogonality of the harmonic functions, there are no cross-products
involving Vn and I1 or V1 and In. This means that if the current is purely sinusoidal, harmonics in the
voltage contribute no mean torque; conversely, if the voltage is purely sinusoidal, harmonics in the
th
current contribute no mean torque. However, if the n harmonic exists in both the voltage and the
current, it will, in general, contribute to the mean torque.
Eqn. (2.451) can be written for the electromagnetic torque,

mp mp
Te ' W ' V1 I1 cos N1 Vn In cos Nn . (2.452)
2B T

In this equation V1, I1, Vn and In are all RMS quantities.


Page 2.132 SPEED’s Electric Machines

Eqn. (2.445) is not sensitive to the waveshape of the


current or the flux-linkage, so the i-R loop can be used
to calculate the torque for square-wave drives, as in
Fig. 2.131, in which the current is regulated to follow a
120E squarewave by chopping.
In a 3-phase machine the average electromagnetic
torque over one electrical cycle is given by
1 Poles
Tem [avg] ' × Wa Wb Wc (2.453)
2B 2

where Wa is the "i-psi loop" area


1
Wa ' (Ra d ia ia d Ra) (2.454)
2 Fig. 2.131 i-psi loop

evaluated over one electrical cycle, and similarly for Wb and Wc. In normal balanced operation we can
assume that Wa ' Wb ' Wc, so that only one i-psi loop need be evaluated (and multiplied by 3). Eqns.
(2.453) and (2.454) are, in effect, the integral form of eqn. (2.418).
The i-R loop method requires the waveforms of current and flux-linkage in each phase. When the
current waveform is known, it lends itself well to the finite-element method. The calculation proceeds
in stepwise manner, creating a series of field solutions, one for each rotor position covering a complete
electrical cycle. At each rotor position 2 the phase flux-linkages in phase k are recovered using to a
relationship of the form N

j Tqk × [ Agqk (2)


k

Rk(2) ' Arqk (2) ] (2.455)


q ' 1

where Nk is the number of coils in phase k, Tqk is the number of turns in the q'th coil of phase k, and
Agqk(2) and Arqk(2) are the average vector potentials over the coil-sides of the go and return conductors
of the q'th coil of phase k. At each step, the instantaneous phase currents i1, i2 and i3 are assumed
known, and the ampere-conductors in each slot are obtained from the conductor distribution according
to a relationship of the form
m
AC [j] ' j C [ j,k] × i [k] (2.456)
k ' 1

where AC[j] is the signed number of ampere conductors in slot j, C[j,k] is the signed number of
conductors in slot j belonging to phase k, i[k] is the instantaneous current in phase k, and m is the
number of phases.
Figs. 2.132-143 show examples of i-R loops for different current and flux-linkage waveforms. The pure
sinewaves in Fig. 2.132 give a perfectly elliptical loop, Fig. 2.133. Fig. 2.134 has 30% 3rd harmonic in the
voltage, but no harmonics in the current. Consequently the loop area in Fig. 2.135 is the same as in Fig.
2.133. Fig. 2.136 has the converse : no harmonics in the voltage but 30% of 3rd harmonic in the current.
Again the loop area, Fig. 2.137, is unchanged from Fig. 2.133. Fig. 2.138 has 30% 3rd harmonic in both the
voltage and the current, and accordingly the loop area, Fig. 2.139, has changed.
Fig. 2.140 shows the interesting theoretical case of a pure squarewave current and triangular flux-
linkage; (cf. Fig. 2.4). The i-R loop, Fig. 2.141 is a perfect rectangle. With 10% inductance and a phase
advance as shown in Fig. 2.142, the loop is distorted in an interesting way, Fig. 2.143. The inductance
adds a component of flux-linkage proportional to the current. The effect of phase-shift is to move the
right-hand parallelogram vertically in the opposite direction to the left-hand parallelogram.
Brushless permanent-magnet machines Page 2.133

Fig. 2.132 Waveforms of e, R and i with N1 ' 30E


Fig. 2.133 i-R loop corresponding to Fig. 2.132.
and no harmonics
W ' cos N1 ' cos 30E ' 0@866 or /3/2.

Fig. 2.134 Waveforms of e, R and i with N1 ' 30E and 30% of 5th
Fig. 2.135 i-R loop corresponding to Fig. 2.134.
harmonic EMF
W is unchanged from Fig. 2.133.

Fig. 2.136 Waveforms of e, R and i with N1 ' 30E and 30% of 5th Fig. 2.137 i-R loop corresponding to Fig. 2.136.
harmonic current W is unchanged from Fig. 2.133.
Page 2.134 SPEED’s Electric Machines

Fig. 2.138 Waveforms of e, R and i with 30% 5th harmonic


in EMF and current Fig. 2.139 i-R loop corresponding to Fig. 2.138. With both
harmonics present, the loop area changes from
2
Fig. 2.133 and is now W ' /3/2 + (0@3) ' 0@956.

Fig. 2.140 Waveforms of i and R : 180E squarewave current


and triangular flux-linkage Fig. 2.141 i-R loop corresponding to Fig. 2.140

Fig. 2.142 Waveforms of i and R : 120E squarewave current;


15E phase advance; triangular R; 10% inductance Fig. 2.143 i-R loop corresponding to Fig. 2.142.
Brushless permanent-magnet machines Page 2.135

2.20 MULTI-PHASE MACHINES


Polyphase
A "multi-phase" machine is obviously one with more than one phase. But we already have the term
"polyphase" to describe machines with more than one phase. So why do we need a new term, "multi-
phase"? — and do we really need this term at all? Let's first understand the term "polyphase".
The traditional polyphase machine has three phases and is balanced, meaning that the phase voltages
and currents are equal in magnitude and separated in phase by 23 ' 120E, Fig. 2.144.

Fig. 2.144 Balanced polyphase machine; 3 phases Fig. 2.145 Balanced polyphase machine; 2 phases

Two-phase machines are also known. For a two-phase machine, balanced operation usually implies a
phase displacement of 22 ' 90E, Fig. 2.145.
It would be convenient if the phase displacement 2 in polyphase machines with m phases was always

360E
2 ' . (2.457)
m
This works with m ' 3, but with m ' 2 we would get 2 ' 180E. A 2-phase machine with a phase
displacement of 180E cannot produce a rotating ampere-conductor distribution; it is in fact a single-
phase machine, in effect, with two anti-parallel windings. If one of these phases is reverse-connected,
the windings will be in parallel rather than anti-parallel; but it is still impossible to produce a rotating
ampere-conductor distribution.
Eqn. (2.457) works with m ' 4, in the sense that the resulting 4-phase machine in Fig. 2.147 can produce
a rotating ampere-conductor distribution. Even so, phases a1 and c1 are anti-parallel, and so are phases
b1 and d1, so this machine can be reduced to the 2-phase machine of Fig. 2.145.

Fig. 2.146 Balanced polyphase machine; 6 phases Fig. 2.147 Balanced polyphase machine; 4 phases

With m ' 6 and 2 ' 60E, again we have a machine that can produce a rotating ampere-conductor
distribution, but again pairs of phases (a1 & d1; b1 & e1; and c1 & f1) are anti-parallel, so this machine can
be reduced to the 3-phase machine of Fig. 2.144.
Page 2.136 SPEED’s Electric Machines

Fig. 2.148 Balanced polyphase machine; 6 phases Fig. 2.149 Balanced polyphase machine; 4 phases

We could define a balanced 6-phase machine as shown in Fig. 2.148, with

180E
2 ' . (2.458)
m

In that case 2 ' 180E/6 ' 30E which is the same as 360E/12, showing that the machine in Fig. 2.148 is the
"irreducible" derivative of a 12-phase machine formed using eqn. (2.457). Similarly we could define a
balanced 4-phase machine as shown in Fig. 2.149, which follows eqn. (2.458): 2 ' 180/4 ' 45E. Again this
is the irreducible derivative of an 8-phase machine formed using eqn. (2.457).
Clearly the machines in Figs. 2.148 and 2.149 occupy only half the stator periphery, so we would expect
another "set" of phases to occupy the other half. As we have already observed, every phase in the second
set would be in phase or in anti-phase with one of the phases of the first set. Therefore the number of
phases in the supply remains equal to 6 (in Fig. 2.148) or 4 (in Fig. 2.149).
More importantly, the same principle applies to the machine of Fig. 2.146. It appears to have 6 phases,
evenly distributed around the stator periphery; but because they are in anti-parallel opposite pairs, only
three phases are needed in the supply.

Fig. 2.150 Balanced polyphase machine; 5 phases

Next consider the 5-phase machine in Fig. 2.150. The phases are evenly distributed around the stator
periphery, but no two phases are opposite. Therefore this is irreducibly a 5-phase machine requiring
five supply phases. It obeys eqn. (2.457) for the number of phases in both the machine and the supply.
From this we can deduce that if m is odd, eqn. (2.457) can be used in general for the number of "machine
phases" and the number of "supply phases". On the other hand, if m is even, eqn. (2.457) can still be used
for the number of machine phases, but the number of supply phases could be reduced to m/2 — in effect,
by using eqn. (2.458) for the number of supply phases.
Brushless permanent-magnet machines Page 2.137

Fig. 2.152 Duplex 2-phase machine; 4 phases

Fig. 2.151 Duplex 3-phase machine; 6 phases

Multiplex windings
Any of the polyphase sets in Figs. 2.144 2.150 can be duplicated or triplicated or — to coin a phrase,
"multiplicated" to form a duplex, triplex or multiplex arrangement. For example, Fig. 2.151 shows a
duplex 3-phase machine and Fig. 2.152 shows a duplex 2-phase machine. In each case there are two sets
of polyphase windings, and there is a physical displacement " between the sets. In the multiplex
winding, " is to some degree arbitrary, because it is possible in principle to operate, say, two three-
phase windings in the same slots completely independently. However, in practice " must be chosen so
that the the fluxes in common sections of the magnetic circuit make sensible utilization of the available
cross-section; and likewise, the slot-fill factors and the need to equalize the loss distribution impose
further constraints on ".
The term multiplex has almost the same meaning here as it does in DC machines, although multiplex
windings are rare in DC machines, and the "phases" are not connected separately but form a continuous
winding with multiple tappings to the commutator bars.
The term plex or "multiplicity" is used, with symbol x. Thus a duplex 3-phase system has x ' 2 and six
phases; a triplex 3-phase system has x ' 3 and nine phases. A simplex 3-phase winding has x ' 1.

Reasons for using multiplex windings


• With very large machines, it may be more practical or more economical to use, say, x inverters
of 1/xth the rating, rather than one huge inverter of 100% rating.
• With x inverters instead of 1, there is a chance of being able to operate even when one of the
inverters is malfunctioning or switched off.
• It becomes possible to operate as many as x independent channels, by connecting the sets of
windings to different buses. This can be done with pure AC systems, as in Fig. 2.153, or with a
DC/inverter system, as in Fig. 2.154.
• For a given number of slots/pole, the winding factor of a duplex winding is higher than that of
a simplex winding, because the "phase spread" is half the value.

Fault tolerant machines


The "multi-phase", "multi-channel" concept brings with it the idea of independent channels and fault-
tolerance, whereby the machine can continue to operate even when one or more phases has suffered a
fault. This idea is not limited to the narrow definition of "multiplex" windings. For example, Fig. 2.155
shows a simplex 4-phase machine that is designed to have independent fault-tolerant channels without
employing the multiplex concept. This machine is specially designed to have nearly zero magnetic
coupling between the four phases, and very low fault current (resulting from high reactance). In
general, the x sets of windings in a multiplex machine are magnetically coupled, and this must be taken
into account in their analysis.
Page 2.138 SPEED’s Electric Machines

Fig. 2.153 Duplex wye-delta system; AC-fed; 6 phases

Fig. 2.154 Duplex wye-delta system; DC-fed; 6 phases

Fig. 2.155 "Fault-tolerant" 4-phase machine with 8 wound teeth and 8 unwound teeth
(Alan Jack, B. Mecrow & Co; University of Newcastle; published in IEEE IAS and UKMagSoc)
Brushless permanent-magnet machines Page 2.139

Analysis of multi-phase windings

.2. 156 page 139

Fig. 2.156 Axes of windings in multi-phase machine

Fig. 2.156 shows the winding axes of a 6-phase machine which has two 3-phase windings a1b1c1 and a2b2c2
— in other words, a duplex three-phase winding. Each three-phase winding is wye-connected and is
balanced in itself. In general all six phases are mutually coupled. If the rotor rotates counterclockwise,
then the voltages and currents in a2b2c2 lead those in a1b1c1 by the phase angle ", which is the angular
displacement between the two 3-phase windings. The rotor d-axis is positioned at the angle 21 with
respect to the axis of phase a1, and at the angle 22 with respect to the axis of phase a2, and
" ' 22 21 . (2.459)
The dq transformation applied to a1b1c1 gives
vd1 ' R1 id1 p Rd1 T Rq1 ;
(2.460)
vq1 ' R1 iq1 p Rq1 T Rd1 .

Normally with only 3 phases, there is just one d-coil and one q-coil and the flux-linkages are given by
Rd1 ' QMd1 Ld1 id1 ;
(2.461)
Rq1 ' Lq1 iq1 ,

where QMd1 is the flux-linkage produced by the magnet. We are going to transform the second winding
a2b2c2 into a second pair of coils d2q2 on the same direct and quadrature axes as the d1q1 coils, and these
will be coupled to the d1q1 coils, so we can write
Rd1 ' QMd1 Ld1 id1 Md1d2 id2 ;
(2.462)
Rq1 ' Lq1 iq1 Mq1q2 iq2 .

The analytical model can now be completed by adding the equations for the a2b2c2 winding
corresponding to eqns. (2.460) and (2.462):
vd2 ' R2 id2 p Rd2 T Rq2 ;
(2.463)
vq2 ' R2 iq2 p Rq2 T Rd2 ,

and
Rd2 ' QMd2 Ld2 id2 Md2d1 id1 ;
(2.464)
Rq2 ' Lq2 iq2 Mq2q1 iq1 .
Page 2.140 SPEED’s Electric Machines

Fig. 2.157 Connection with a1b1c1 and a2b2c2 both aligned with the d-axis. This is not a practical
operating connection, but is merely used in the technical development.

Fig. 2.158 Connection with a1b1c1 and a2b2c2 in quadrature. This is not a practical
operating connection, but is merely used in the technical development.

The mutual inductance between the d1 coil and the d2 coil is important, and we must consider whether
it depends on the displacement angle " between the two 3-phase windings. Later we will also consider
the possibility of mutual coupling between the d and q-axes; such cross-coupling is absent, for the
moment, from eqns. (2.462) and (2.464).
If " ' 0, and if all phases have the same effective number of turns, then phases a1 and a2 will be aligned
with the d-axis at the same time. A particular example of this is the connection shown in Fig. 2.157. In
this case we expect Md1d2 to be equal to Lmd, the magnetizing component of Ld, plus another term mF12d
arising from mutual coupling in the slots and in the end-windings: thus
Md1d2 ' Lmd mF 12d ( " ) . (2.465)

The mutual inductance term mF12d may include a component due to differential or harmonic leakage,
while Lmd is related to the conventional leakage inductances LF and MF by eqn. (2.165) on p. 58, and is
associated with the fundamental ampere-conductor distribution as noted in Fig. 2.157. Any
relationship between LF, MF and mF12d is not immediately obvious, but mF12d can be calculated formally
by applying the dq-axis transformation to the entire inductance matrix as shown below.
Brushless permanent-magnet machines Page 2.141

If " ' B/2, phase a2 will be aligned with a q-axis of the rotor when phase a1 is aligned with a d-axis. Such
a case is the connection in Fig. 2.158. If the d-axis inductance Ld and its components Lmd, LF and MF are
independent of the instantaneous orientation of the d-axis relative to the phase winding axes, eqn.
(2.465) remains valid, but the value of mF12d will be different from what it was in Fig. 2.157.
As a simple verification of the main principle — that Md1d2 is independent of the rotor position or the
orientation of the windings — consider the flux-linkage produced in phase a1 by the current I in Fig.
2.157: in per-unit terms it is

1 1 1 1 3
R1 ' 1 × × ' (2.466)
2 2 2 2 2
In Fig. 2.158 the same flux-linkage is given by

3 3 3 3 3
R1 ' × × ' , (2.467)
2 2 2 2 2
since cos (30E) ' cos (150E) and the currents in phases b2 and c2 are I/3/2 and I/3/2 respectively. At
the instant shown, there is no flux in the q-axis. This example serves to show that in terms of normal
phasor quantities, when " ' B/2 the currents in phases a2b2c2 are 90E out of phase with the currents in
phases a1b1c1. In general, the currents (and voltages) must be phase-shifted by ".

By similar reasoning for the q-axis coils,


Mq1q2 ' Lmq mF12q ( " ) . (2.468)

Balance
It seems possible to draw two very important inferences from eqns. (2.465) and (2.468). In a duplex wye-
connected 6-phase machine there are only 2 coils on the d-axis and 2 coils on the q-axis. If there is no
cross-coupling between the d- and q-axes, and if the phases are fed with equal voltages phase-shifted by
" in the second set, the currents in the two sets will be equal in magnitude and phase-shifted by " : in
other words, the two sets will be balanced, both within themselves and with respect to each other.
However, consider the extension to a 9-phase machine which will have 3 coils on the d-axis and 3 on the
q-axis. We will have
Md1d2 ' Lmd mF 12d ( " ) ;
Md2d3 ' Lmd mF 23d ( " ) ; (2.469)
Md3d1 ' Lmd mF 31d ( " ) .

If mF12d, mF23d and mF31d are not all equal, it will be impossible to achieve balance between the three sets
of windings. For if they are all supplied with the same currents (correctly phase-shifted by 0, " and 2"
respectively), the voltages will be unequal because of the differences in the mutual inductances. This
imbalance will show itself in the actual phase voltages (if the currents are forced); or in the currents
(if the voltages are forced).
To represent this inference as a general rule, if x is the plex of the windings, we can say that
• a duplex winding (x ' 2; 4 or 6 phases) can always be balanced regardless of " ;
• a triplex 9-phase winding, or a winding of higher multiplicity (x  3), cannot be
balanced unless all the d-axis mutual inductances are equal to one another; and
all the q-axis mutual inductances are separately equal to one another.
It can also be observed that the accuracy of the performance calculation will depend on the accuracy
of the values of the mutual inductances Md1d2, Mq1q2 etc.
To pursue this further, it is necessary to resort to a matrix analysis of the inductances.
Page 2.142 SPEED’s Electric Machines

Matrix analysis of the inductances


The conventional dq-axis transformation for plain 3-phase machines is given on p. 128. In terms of flux-
linkages [R] and currents [i], it can be expressed as follows :
[Rabc] ' [Labc] [iabc] ; (2.470)
[idq0] ' [T] [iabc] ; [iabc] ' [T] 1 [idq0] . (2.471)
1
ˆ [Rdq0] ' [T] [Rabc] ' [T] [Labc] [T] [idq0] ' [Ldq0] [idq0] . (2.472)

The matrix [Labc] is defined by eqns. (2.422) [or equivalently, by eqns. (2.157) on p. 57]. The result is

Ld

[Ldq0] ' Lq

L0

where Ld and Lq are given by eqns. (2.165) and (2.166) on p. 58. The elements of [T] are trigonometric
functions of 2 in Fig. 2.69, and we can denote this by writing [T] as [T(2)].
To consider the duplex 6-phase machine we start with the abc inductance matrix:
a1 b1 c1 a2 b2 c2

a1 La1 Ma1b1 Ma1c1 Ma1a2 Ma1b2 Ma1c2

b1 Mb1a1 Lb1 Mb1c1 Mb1a2 Mb1b2 Mb1c2

c1 Mc1a1 Mc1b1 Lc1 Mc1a2 Mc1b2 Mc1c2

a2 Ma1a2 Mb1a2 Mc1a2 La2 Ma2b2 Ma2c2

b2 Ma1b2 Mb1b2 Mc1b2 Mb2a2 Lb2 Mb2c2

c2 Ma1c2 Mb1c2 Mc1c2 Mc2a2 Mc2b2 Lc2

It is useful to partition this matrix into

a1 b1 c 1 a2 b2 c 2

a1 b1 c 1 [A1] [B]

a2 b2 c 2 [BN] [A2]

If [B] ' 0, there is no mutual coupling between the a1b1c1 windings and the a2b2c2 windings. In that case
the dq0 transformation [T1] ' [T(21)] can be applied to [A1], and [T2] ' [T(22)] to [A2], and the result is two
completely independent, uncoupled dq0 systems

d1q101 d2q202
1
d1q101 [T1][A1][T1]
1
d2q202 [T2][A2][T2]

In practice the d1q101 system is mutually coupled to the d2q202 system, and we have already postulated
the form of the mutual inductances in eqns. (2.465), (2.468) and (2.469). The situation so far can be
summarized by writing the duplex dq0 inductance matrix as follows. (Extension to the triplex winding
is obvious).
Brushless permanent-magnet machines Page 2.143

d1 q1 01 d2 q2 02

d1 Ld1 Lmd mF12d

q1 Lq1 Lmq mF12q

01 L01

d2 Lmd mF12d Ld2

q2 Lmq mF12q Lq2

02 L02

The terms in Lmd and Lmq are easy but the terms in mF12d and mF12q require more analysis. First, the
formal means of generating the duplex dq0 matrix is to apply the transformation

d1q101 d2q202

d1q101 [T1]

d2q202 [T2]

to the complete 6-way matrix a1b1c1a2b2c2. In terms of the partitions already considered, this gives

d1q101 d2q202
1 1
d1q101 [T1][A1][T1] [T1][B][T2]
1 1
d2q202 [T2][BN][T1] [T2][A2][T2]

We already have the upper-left and lower-right partitions, and we also have the Lmd and Lmq terms in
the upper-right and lower-left partitions. Therefore it should be possible to consider only the remaining
mutual inductance terms independently, by transforming the matrix
a1 b1 c1 a2 b2 c2

a1 8 μ <

b1 < 8 μ

c1 μ < 8

a2 8 < μ

b2 μ 8 <

c2 < μ 8

where 8, μ and < are mutual inductances between a1 a2, a1 b2 and a1 c2 respectively, excluding the
airgap fluxes which give rise to Lmd and Lmq. Therefore 8, μ and < are confined to slot-leakage flux, end-
winding flux, and harmonic leakage, and we can consider them to be independent of rotor position; (see
p. 60). Thus for the upper-right partition,
1
[T1][B][T2] '

cos 21 cos (21 120E) cos (21 120E) 8 μ < cos 22 sin 22 1
2
sin 21 sin (21 120E) sin (21 120E) @ < 8 μ @ cos (22 120E) sin (22 120E) 1
3
1/2 1/2 1/2 μ < 8 cos (22 120E) sin (22 120E) 1
Page 2.144 SPEED’s Electric Machines
th
Without writing out the entire solution, it is instructive to work out and examine just the (1,1)
element, which represents mF12d :

[T1][B][T2] 1 (1,1) ' mF12d ' 8 cos (22 21) μ cos (22 21 120E) < cos (22 21 120E) (2.473)
Since 22 21 ' ", we can write this as
mF12d ' 8 cos " μ cos (" 120E) < cos (" 120E) . (2.474)
By the same process we find that

[T1][B][T2] 1 (2,2) ' mF12q ' 8 cos " μ cos (" 120E) < cos (" 120E) ' mF12d . (2.475)
This shows the feature we might have hoped to avoid: that although mF12d ' mF12q, these mutual
inductances appear to depend on ", and further analysis is necessary to find out the constraints that
apply to the choice of " for duplex, triplex, and higher multiplex windings.
For the duplex winding we have already observed that it does not matter if mF12d and mF12q depend on
", because there is only one mutual inductance on the d-axis and one on the q-axis.
For triplex windings we can expect mF12d and mF23d both to be given by eqn. (2.474) directly, because the
displacement angle between sets 1 and 2 is the same as the displacement angle between sets 2 and 3, both
being equal to ". But the displacement angle between sets 1 and 3 is 2", so we expect mF31d to be given
by eqn. (2.474) with 2" substituted in place of ". Now we know that it is necessary to have
mF12d ' mF23d ' mF31d (2.476)

to operate a triplex winding balanced. So what we really need, at this stage, is to understand whether,
and under what conditions, eqn. (2.474) gives the same value for 2" as it does for ".

Fig. 2.159 Angles for contemplating 8,μ and <

Now consider the mutual inductances 8, μ, < in terms of the winding harmonics. From Fig. 2.159 we can
see that 8 is a function of the angle ", that is, 8 ' 8("). Similarly μ is a function of the angle (" 120E),
and if we assume the same functional dependence, we can write μ ' μ(" 120E). Finally < ' <(" 120E).
For each harmonic component of the inductance, let
8 ' 7n cos n " ; μ ' 7n cos n ( " 120E ) ; < ' 7n cos n ( " 120E ) . (2.477)
This is, in effect, one term of a Fourier series expansion of the inductances 8, μ and <. If we substitute
th
this in eqn. (2.474) we get the total n harmonic inductance
mn(") ' 7n [ cos n " cos " cos n ( " 120E ) cos ( " 120E ) cos n ( " 120E ) cos ( " 120E ) ] (2.478)
It can be shown that mn(2") ' mn(") for " ' 20E, 40E, 60E, 80E, 100E, 120E etc., but not for " ' 15E or 30E.
This implies that " ' 20E, 40E or even 80E is acceptable for triplex (9-phase) windings, but not 15E or 30E.
On the other hand 15E and 30E are acceptable for duplex (6-phase) windings. In general it appears that
" ' 180k/mxE gives the required result expressed by eqn. (2.476), at least for duplex and triplex cases.
Brushless permanent-magnet machines Page 2.145

We can also contemplate the conditions that would achieve 8 ' μ ' < ' 0. If this could be achieved, then
any value of the displacement angle " would be acceptable with any value of plex.
If all the mutual coupling between phases of different sets were via the slot-leakage flux, this condition
could be satisfied by ensuring that the windings of a1b1c1 have no shared slots with a2b2c2 or a3b3c3. The
differential leakage is more troublesome, since it is a function of several space-harmonics of MMF having
different pole-pitches. For example, if " ' 30E there should be no third-harmonic linkage between a1 and
a2, since 3 × 30 ' 90E, rendering a1 and a2 orthogonal and therefore uncoupled for this harmonic. But
the same cannot be said of the non-triplen harmonics 5th, 7th, etc. One might hope that the differential
leakage is small; but the very nature of multiplex windings is to reduce the number of slots per pole per
phase, potentially increasing the winding factors of some of the harmonics we would like to eliminate.
The inference is that it is safest to select " from the preferred values indicated on the previous page,
when designing triplex or higher-multiplex windings.
th
Finally let us test the symmetry of the combined dq0 matrix by evaluating the [1,1] term of
1
[T2][BN][T1] , the lower-left partition. The evaluation is very similar to that of the upper-right partition
1
[T1][B][T2] , with 21 and 22 interchanged. The result is
mF21d ' 8 cos " μ cos ( " 120E) < cos ( " 120E) . (2.479)
Thus
mF21d ' mF12d (2.480)

and the mutual (off-diagonal) inductances in the dq0 matrix are indeed reciprocal.

Simplified calculations
In order to proceed with manageable calculations, the simplest expedient is to drop mF12d from eqn.
(2.465) and mF12q from its q-axis counterpart, leaving
Md1d2 ' Lmd (2.481)
and Mq1q2 ' Lmq . (2.482)
Alternatively, mF12d and mF12q can be retained in the equations but treated as a user-defined
"perturbation" parameter. This permits an experimental numerical approach to determine how large
these parasitic mutual inductances must be to cause significant errors in the calculation (and/or to
cause significant imbalance between winding sets in cases of multiplicity 3 or higher). This approach
is followed in the remainder of this section.

Torque
The electromagnetic torque is the result of interaction between the currents and the rotational voltages
in eqns. (2.460) and (2.463). For the first set of windings,

T1 ' m p ( Rd1iq1 Rq1 id1 ) , (2.483)


where m is the number of phases (3) and p is the number of pole-pairs. Substitute for the flux-linkages
from eqns. (2.461); the result is

T1 ' m p [ QMd1 iq1 ( Ld1 Lq1 ) id1 iq1 Md1d2 id2iq1 Mq1q2id1iq2 ] . (2.484)

For the second set of windings the process is the same:


T2 ' m p ( Rd2iq2 Rq2 id2 ) , (2.485)
so that
T2 ' m p [ QMd2 iq2 ( Ld2 Lq2 ) id2 iq2 Md2d1 id1iq2 Mq2q1id2iq1 ] . (2.486)
Page 2.146 SPEED’s Electric Machines

In each of eqns. (2.484) and (2.486), the first term is the familiar permanent-magnet alignment torque,
while the second term is the familiar reluctance torque. The third and fourth terms arise from
interaction between the two sets of windings: d-axis flux produced by one set interacts with the
orthogonal q-axis current component of the other set. Consequently four additional terms appear for
a duplex winding; eight for a triplex winding, and 4(x 1) for an x-plex winding.
The torque equations (2.484) and (2.486) can be used for transient or steady-state torque. In the steady-
state they can be written in terms of phasors (RMS AC quantities):

mp
T1 ' Eq1 Iq1 (Xd1 Xq1) Id1Iq1 Xd1d2 Id2 Iq1 Xq1q2Id1Iq2 ;
T
(2.487)
mp
T2 ' Eq2 Iq2 (Xd2 Xq2) Id2Iq2 Xd2d1 Id1 Iq2 Xq2q1Id2Iq1 .
T
where T ' 2Bf and X ' TL. By regrouping the terms of this equation, we can separate the alignment
torques and the reluctance torques as

mp
Te i ' Eq1 Iq1 Eq2 Iq2 ;
T
mp (2.488)
Trel A ' (Xd1 Xq1) Id1Iq1 (Xd2 Xq2) Id2Iq2 ;
T
mp
Trel B ' ( Xd2d1 Xq1q2 ) Id1 Iq2 ( Xd1d2 Xq2q1) Id2Iq1 ;
T

where Tei is the total alignment torque, Trel A is the "self" reluctance torque, and Trel B is a "mutual"
reluctance torque. With a triplex winding the result is

mp
Tei ' Eq1 Iq1 Eq2 Iq2 Eq3 Iq3 ;
T
mp
Trel A ' (Xd1 Xq1) Id1Iq1 (Xd2 Xq2) Id2Iq2 (Xd3 Xq3) Id3Iq3 ; (2.489)
T
mp
Trel B ' ( Xd1d2 Xq1q2 ) ( Id1 Iq2 Id2 Iq1 ) ( Xd2d3 Xq2q3) ( Id2Iq3 Id3 Iq2 ) ( Xd3d1 Xq3q1) ( Id3Iq1 Id1 Iq3 ) .
T

Steady-state operation : phasor diagram


Under AC steady-state conditions the RMS values of the d- and q-axis flux-linkages Rd and Rq in eqn.
(2.461) can be combined into a phasor
Q1 ' Qd1 j Qq1 , (2.490)

and likewise the current can be expressed as a phasor


I1 ' Id1 j Iq1 . (2.491)
The voltage phasor is then given by
V1 ' Vd1 j Vq1 ' R1 I1 j T Q1 , (2.492)
in which
Vd1 ' R1Id1 Xq1Iq1 Xq1q2Iq2 ;
(2.493)
Vq1 ' Eq1 R1Iq1 Xd1Id1 Xd1d2Id2 .

This can be expressed graphically in the phasor diagram, Fig. 2.160. The terms R1Id1, Xq1Iq1, Eq1, R1Iq1,
and Xd1Id1 are familiar from the simplex winding; but the cross-coupling terms Xq1q2Iq2 and Xd1d2Id2 are
peculiar to the duplex winding. Without the cross-coupling terms the terminal voltage would be U1, but
with them the terminal voltage is V1. A similar phasor diagram is obtained for the second set of
windings, for which the corresponding voltage equations are eqns. (2.494).
Brushless permanent-magnet machines Page 2.147

q
!X q1 I q1

jX d1 I d1 R I1

!X q1q2 I q2 E q1
U1
jX d1d2 I d2
V1

V q1 I1
I q1

 

d
V d1 I d1

PhDiag_Duplex.wpg

Fig. 2.160 Phasor diagram for the first set of a duplex winding

Vd2 ' R2Id2 Xq2Iq2 Xq2q1Iq1 ;


(2.494)
Vq2 ' Eq2 R2Iq2 Xd2Id2 Xd2d1Id1 .

The cross-coupling terms appear very clearly in the phasor diagram as additional voltage-drops which
tend to limit the current. If " ' 0 we have, as already observed, a machine with tightly coupled
inductances between the two sets, and if these sets are fed from a common voltage source the current
in each set will be approximately half the current that would flow in one set if the other were open-
circuited. This is an important practical point because it implies that in a duplex winding, if one set is
open-circuited the current in the other set could increase by a factor approaching 200%, if it were not
regulated. Likewise if one set is short-circuited, the impedance of the second set will be reduced and its
current could also increase to a high value if it is not regulated.
The behaviour of the duplex sets is analogous to that of parallel coils which can be represented as two
parallel uncoupled inductances, each of value L + M (see SEM-1). When " ' 0, M becomes close to L and
the overall inductance is approximately (L M)/2 ' L. The total current is that which is limited by L,
and half flows in each set. But if one set is open-circuited, the same total current will tend to flow in one
set. The implication is that regulation of the current is essential.
We are now in a position to solve the system with any given voltages or currents applied to the terminals
of the two sets of windings. In the steady state this is a question of solving eqns. (2.493) and (2.494) when
either the voltages V1 and V2 are given, or the currents I1 and I2 are given.
Solution method — transient
When the machine is inverter-fed, the applied voltages are transient PWM waveforms determined by
a current regulator. Where we normally solve only eqn. (2.460) for Rd and Rq in a 3-phase machine, we
must now integrate eqns. (2.460) and (2.463) to produce updated values of Rd1, Rq1, Rd2 and Rq2 at each
time-step, also inverting eqns. (2.462) and (2.464) for id1, iq1, id2 and iq2 and updating the torque.
Extension to triplex and multiplex windings
The extension to triplex and multiplex windings is a straightforward matter of adding additional
voltage equations of the form of eqn. (2.460), and additional mutual terms in the corresponding flux-
linkage equations of the form of eqn. (2.462). Everything else follows as described above.
Page 2.148 SPEED’s Electric Machines

Use of i-psi GoFER


The GoFER works with instantaneous phase currents which represent steady-state operation over one
electrical cycle with sinusoidal current waveforms.35 the GoFER must use the total MMF of both sets of
windings for this calculation. Treating a1b1c1 as the reference winding, we have

ia1 ' 2 I1 sin ( T t ()


ib1 ' 2 I1 sin ( T t ( 120E ) (2.495)
ic1 ' 2 I1 sin ( T t ( 120E )
where I1 is the RMS current in windings a1b1c1. Similarly

ia2 ' 2 I2 sin ( T t ( ")


ib2 ' 2 I2 sin ( T t ( " 120E ) (2.496)
ic2 ' 2 I2 sin ( T t ( " 120E )
where I2 is the RMS current in windings a2b2c2, and the currents in a2b2c2 are leading the corresponding
currents in a1b1c1 by ". This can be extended to the triplex winding, x ' 3. The average electromagnetic
torque can be computed from the sum of the areas of the mx i-R loops as described on p. 131.
It is self-evident that Lmd and Md1d2 will share the same d-axis saturation factor, while Lmq and Mq1q2 will
share the same q-axis saturation factor. These saturation factors can be determined in the usual way
from the flux-linkages Ra1, Rb1 and Rc1 of the a1b1c1 winding set, or their dq equivalents Rd1, Rq1.

Equivalent simplex system


Consider the non-power-invariant transformation
idp ' ( id1 id2 )/2 ; Rdp ' ( Rd1 Rd2 )/2 ;
idn ' ( id1 id2 )/2 ; Rdn ' ( Rd1 Rd2 )/2 ;
(2.497)
iqp ' ( iq1 iq2 )/2 ; Rqp ' ( Rq1 Rq2 )/2 ;
iqn ' ( iq1 iq2 )/2 ; Rqn ' ( Rq1 Rq2 )/2 .
with inverse
id1 ' ( idp idn ) ; Rd1 ' ( Rdp Rdn ) ;
id2 ' ( idp idn ) ; Rd2 ' ( Rdp Rdn ) ;
(2.498)
iq1 ' ( iqp iqn ) ; Rq1 ' ( Rqp Rqn ) ;
iq2 ' ( iqp iqn ) ; Rq2 ' ( Rqp Rqn ) .

If we substitute eqns. (2.498) into the torque equations (2.483) and (2.485), in general we get a torque
equation that is much more complicated than the original torque equations; but under balanced
conditions where id1 ' id2, iq1 ' iq2, Rd1 ' Rd2, Rq1 ' Rq2, we have idn ' iqn ' Rdn ' Rqn ' 0, and the torque
equation simplifies to
T ' 2 m p (Rdp idp Rqpidp) . (2.499)

It can be inferred that with plex ' 3, the factor 2 in eqns. (2.497) and (2.499) can be replaced by 3. What
this implies is that under balanced conditions the torque is doubled for a duplex winding, or trebled for
a triplex winding, other things being equal. However, it does not mean that the torque is 2 or 3 times
the torque of one set acting alone, because of the interaction between sets that is evident, for example,
in the mutual voltages in the phasor diagram, Fig. 2.160.

35
See the PC-BDC GoFER manual; theory section
Brushless permanent-magnet machines Page 2.149

2.21 COMPARISON OF SQUAREWAVE AND SINEWAVE CONFIGURATIONS

Comparison in terms of the energy-conversion loop


The i-R loop in Fig. 2.141 is a perfect rectangle, and its area W is 4/B times the area of a circular i-R loop
with the same peak flux-linkage and the same peak current. Such a circular loop can be obtained if the
current and EMF are in phase, so that N1 ' 0. At first sight this implies that the squarewave drive can
produce more torque than the sinewave drive. However, the RMS current in Figs. 2.140 and 2.141 is equal
2
to the peak current, so the I R loss in the winding is doubled, when compared to the sinewave drive in
Figs. 2.132 and 2.133 with the same peak current. If the squarewave drive were operated with the same
RMS current, its peak current would need to be reduced to 0@5 and the area W would also be reduced by
50%. The 4/B advantage of the squarewave drive now becomes a B/2 advantage to the sinewave drive.
Moreover, in a 3-phase drive the full squarewave current waveform is not possible with the common
3-wire connection and 6-transistor bridge. It is more usual to operate with “2-phases on”, so that the
current waveform in each phase is of the form of Fig. 2.142. For the same peak current as the sinewave
drive, this reduces the area W by a factor 2/3, but the RMS current is reduced by the factor /(2/3).
Therefore for the same rms current the area W is reduced only by the factor (2/3)//(2/3) ' /(2/3). This
reduces the advantage of the sinewave drive to (B/2) × /(2/3) ' B//6 ' 1@283, which interestingly is very
nearly the reverse of the original supposed advantage of the squarewave drive.
So far in this argument we have tacitly assumed the same peak flux-linkage in both the sinewave and
the squarewave cases. To achieve the triangular flux-linkage waveform in Fig. 2.140, the phase winding
must be fully-pitched and concentrated into two slots 180E elec apart. Moreover the magnet must span
exactly 180E with an abrupt transition from north-pole to south-pole flux — in other words, the airgap
flux distribution must be a squarewave. Such a magnet would have not only the maximum possible
2
flux but also the maximum possible fundamental component of flux, which is 8/B times the total flux.
2
The peak flux-linkage of a practical phase winding in a sinewave machine will therefore be 8/B kw1
times the peal flux-linkage of the ideal concentrated squarewave winding, where kw1 is the fundamental
2
harmonic winding factor. Assuming a value of 0@9 for kw1, this factor is 8/B × 0@9 ' 0@73, and this will
reduce the advantage of the sinewave drive to 1@283 × 0@73 ' 0@935, which is a 6@5% advantage to the ideal
squarewave drive. But because of fringing and the inevitable spread that will be required even with
the squarewave winding, this 6@5% will most likely be lost, leaving very little “natural advantage” to
either configuration in terms of their torque production in a given size.

Comparison of torque per ampere and kVA requirements


With a torque angle of 90E degrees the torque per RMS ampere of phase current in the three-phase
sinewave motor is
3 B r1 Lstk B1Md kw1 Tph
T/I ' 2 Nm/A (2.500)
2 2
where r1 is the stator bore radius and B1Md is the peak fundamental flux density produced by the
magnet in the airgap. The torque per peak ampere is /2 times smaller. In the squarewave motor
(assuming 180E magnet arcs, star connection and 120E squarewave phase currents), the torque per peak
ampere of phase current is
T/i ' 4 r1 BMg Lstk Tph Nm/A (2.501)
where BMg is the peak flux density produced by the magnet in the airgap.
The r.m.s. phase current I is derived from the 120E squarewave as:

I ' i 2/3 (2.502)


where i is the peak or flat-top value of the phase current. Therefore the torque per r.m.s. ampere of
phase current is

T/I ' 4 (3 / 2) r1 BMg Lstk Tph Nm/A (2.503)

With these expressions the ratio of torque per ampere in the two motors can be compared.
Page 2.150 SPEED’s Electric Machines

With equal RMS phase currents, the torque of the squarewave motor exceeds that of the sinewave motor
by the factor 4 3/2
' 1@47 (2.504)
3/2 × 2 × B/2
With equal peak currents, the factor is 1@27. The squarewave motor appears to have a significantly
higher torque per ampere. However, the comparison assumes equal peak magnet flux-densities in the
airgap, which is likely to require significantly more magnet and a thicker stator yoke in the
squarewave motor. To rectify this imbalance it is perhaps better to compare the motors on the basis of
the same flux per pole. For the same peak flux-density the flux per pole of the square-wave motor
exceeds that of the sinewave motor by the factor B/2. Now with equal r.m.s. phase currents the torque
ratio is only 0@94, and with equal peak currents it is 0@81. This analysis neglects many important effects,
such as armature reaction and losses, but it indicates that with equal amounts of copper, iron, and
magnet, the torque per ampere is not greatly different between the two machines.
The comparison is now carried to the volt-ampere requirements of the drive. A simple estimate of the
drive “rating” can be made in terms of the total kVA rating of its main switches, per kW of power fed
to the motor. The relevant parameters are defined as follows. With respect to the r.m.s. current in each
switch, if q is the phase number,
rms kVA/kW ' 2q × Is × Vs (2.505)
where Is is the r.m.s. current in each switch and Vs is the peak voltage across each switch. For the
drives normally used with brushless DC motors the peak device voltage is nominally equal to the DC
supply voltage, because each switch must block this voltage while the other one in the same phaseleg
is conducting. Obviously there must be a margin for voltage spikes caused by stray inductance and
reverse-recovery of diodes. While these effects are not small, they are parasitic and not fundamental
to the operation of the motor. Therefore the DC supply voltage will be used in this comparison.
With respect to the peak current in each switch,
peak kVA/kW ' 2q × is pk × Vs (2.506)
where is pk is the peak current in each switch, and the voltage conditions are unchanged.
In the sinewave motor the line currents are assumed to be sinewaves and each switch conducts a half
sinewave for 180E and is then off for 180E. The RMS switch current is therefore 1//2 times the RMS line
current, which will be assumed to be the same as the phase current (i.e. the motor is star-connected).
The peak device current is equal to the peak phase current. The peak line-line voltage of the motor is
approximately equal to V. Thus

v̂LL ' V ' 3 v̂ph ' 6Vph (2.507)


We can now summarize:

Sinewave Squarewave
RMS switch VA 6VI//2 6Vi/2/3
Peak switch VA 6Vipk 6Vi
Drive power output 3VphI ' 3VI//6 Vi
Ratio RMS switch VA/W 3@5 4@9
Ratio peak switch VA/W 6@9 6@0

Thus the squarewave motor has a slightly better utilization of the peak current capability of the drive
switches. Although the sinewave motor appears from this analysis to have a better utilization of their
RMS current capability, this advantage may be offset by the greater duty cycle in the sinewave drive,
where 3 switches are conducting at any instant, not 2. In the analysis it has been assumed that the ideal
waveforms are achieved by PWM at a sufficiently high frequency, and of course this incurs switching
losses which tend to lessen the significance of the lower RMS switch currents required by the sinewave
motor. More detailed comparison requires the use of computer simulation and practical testing.
Brushless permanent-magnet machines Page 2.151

2.22 PERMANENT MAGNETS VERSUS ELECTROMAGNETIC EXCITATION


The laws of electromagnetic scaling impose an “excitation penalty” on small motors. As the size is
decreased, the cross-section area available for conductors decreases with the square of the linear
dimension, but the need for MMF decreases only with the linear dimension, being primarily determined
by the airgap length g. As the motor size is further decreased, g reaches a minimum manufacturable
value (typically of the order of 0.15 0.3 mm). Past this point the MMF requirement decreases only slowly
while the copper area continues to decrease with the squared linear dimension. The per-unit copper
losses increase even faster, and the efficiency decreases rapidly. The loss-free excitation provided by
permanent magnets therefore increases in relative value as the motor size is decreased.
In larger motors magnets improve efficiency by eliminating the losses associated with electromagnetic
field windings. But in larger motors the relative excitation penalty is small. At the same time the
required volume of magnets with adequate properties increases with motor size to the point where PM
excitation is just too expensive. It is therefore rare to find PM motors of more than a few kilowatts.
Sometimes operational or safety considerations work against the PM motor. For example, in vehicle
traction, a PM motor with a short-circuit winding fault would have a large braking torque, possibly
causing sudden deceleration and a risk of overheating which could demagnetize the magnets or even
cause a fire. There is no simple way to protect against this type of fault.
There is no hard-and-fast power level below which PM excitation becomes advantageous, but it is
possible to examine the excitation penalty in ways which indicate roughly where the breakpoint lies,
and why. For a given level of excitation the choice can be made between magnets or copper windings
operating at a current density J (in the copper). In the following analysis, several gross assumptions
are made, and the equations derived are only indicative.
Using a permanent magnet, the fundamental flux per pole is B1 DLstk/p, where B1 is the peak
fundamental flux-density produced by the magnet in the airgap. If rotor leakage is neglected this flux
can be taken to be approximately equal to the flux through the magnet in a surface-magnet motor, that
is, Bm Am. If the magnet is operated at a fraction ( of its remanent flux-density, Bm ' (Br and hence
B1DLstk
Am ' (2.508)
p(Br
where D is the rotor diameter. From Ampere’s law we have
H ml m ' H g g ' 0 (2.509)
from which
gBg μrec
lm ' (2.510)
(1 & () Br
where Bg is the average airgap flux-density. The required volume of magnet is then
Vm ' 2pAmlm (2.511)
If B1 ' k1 Bg, and if we assume that l ' D, then
2
2 B1 D 2 g μrec
Vm ' (2.512)
2
k1 Br ((1 & ()
Now we can calculate the amount of copper needed to magnetize the airgap to the same level. Assuming
a sinusoidal distribution of conductors,
Bg g
F g ' g Hg ' g ' B1 cos p2 (2.513)
μ0 μ0
But with N conductors in series per phase,
2 N iN
Fg ' i sin p2 d2 ' cos p2 (2.514)
0 2 2p
Page 2.152 SPEED’s Electric Machines

Hence g B1 iN
' (2.515)
μ0 2p
and the total ampere-conductors required are
i N × 2p ' 2piN (2.516)
If Ac is the cross-section of the copper winding in the whole motor cross-section,
AcJ ' 2piN (2.517)
where J is the current density. Hence
4p 2gB1
Ac ' (2.518)
μ0J
2
The copper cross-section required is proportional to p , unlike the magnet volume which is independent
of the number of poles. The volume of copper can be estimated by assuming a mean length of conductor
equal to twice the stack length (to allow for end-turns); thus
8p 2gDB1
Vc ' . (2.519)
μ0J
The relative volumes of magnet and copper required can be compared by taking the ratio
Vm μ0 μrec J B1 D
' . (2.520)
Vc 2
4 k1 Br p 2 ( (1 & ()
Consider two four-pole motors with B1 ' 0@7T and k1 ' 1@1. The PM motor has rare-earth magnets with
2
Br ' 0@8T and μrec ' 1@05. The electrically-excited motor has J'4 A/mm , giving
Vm D
' (2.521)
Vc 488
where D is measured in mm. This means that for motors less than about 500 mm in rotor diameter the
magnet volume is less than the volume of copper needed for excitation by a separate field winding.
Unfortunately the cost per unit volume of high-energy magnets at this level is of the order of 25 times
that of copper. For the magnet cost to be less than the cost of the copper in a separate field winding, the
rotor diameter must therefore be less than 500/25, i.e. only 20 mm, giving a stator diameter of about 40
mm. The technical potential of high-energy magnets is thus offset by their high cost in all but the
smallest motors. In very small motors a smaller value should be used for the current density J; with J
2
' 2@5 A/mm the stator diameter for equal cost would be increased from 40 mm to 64 mm (2@5 in). In
general, high-energy magnets can only be justified where there is a special premium on efficiency or
compactness. Of course this argument is simplistic, ignoring factors such as process and manufacturing
costs and many others, but it provides a basic physical understanding of the application potential of
magnets, and the effects of scale. Motors magnetized with ceramic magnets must settle for a lower
2
airgap flux-density. Using values of J ' 4 A/mm ; B1 ' 0@3 T; μrec ' 1; Br ' 0@35 T, the result is
Vm D
' (2.522)
Vc 230
For motors of less than 230 mm rotor diameter the magnet volume indicated is less than the volume of
copper in a separate field winding. Ceramic magnets are much less expensive than high-energy
magnets, the cost per unit volume being of the order of 0@6 times that of copper, so that the magnet cost
will be less than the cost of field copper in motors of rotor diameter less than 230 × 0@6, i.e. 138 mm. In
practice PM motors as large as this are relatively uncommon. With ferrite, the flux-density is too low;
with rare-earth magnets the cost is too high.
If running costs are taken into account, the comparison between PM and electrically excited motors
changes significantly. With the present cost of raw materials and present kWh tariffs, the kWh cost of
electrical excitation surpasses the cost of the copper in just a few months, assuming the motor runs at
full excitation 24 h/day. The PM motor should eventually pay for itself in this way.
Brushless permanent-magnet machines Page 2.153

2.23 COGGING TORQUE CALCULATIONS USING FINITE-ELEMENT ANALYSIS36

Fig. 2.161 Representation of magnet by equivalent air-cored coil

A magnet (or any element thereof) can be represented by an equivalent distribution of linear surface
current density K ' M × n, together with a volume distribution of current density J ' curl M, where M
is the magnetization and n is the normal to the magnet surface. For isotropic magnets J ' 0 and K ' Hca,
the apparent coercivity Br/μrecμ0. In the following, Hca is written Hc. The total coercive MMF of the
magnet is Fc ' Hc Lm, where Lm is the magnet length in the direction of magnetization.
At first sight it appears that the stored magnetic field energy can be obtained by calculating the integral

m
1
Wf ' BH dV (2.523)
2

in all regions of the motor, and adding the resulting integrals together. In the air regions, B ' μ0H. In
the iron regions, B and H are related by the B/H characteristic of the iron. But within the volume
occupied by the magnet,
B ' Br μrec μ0 H (2.524)

where Br is the remanence and μrec is the relative recoil


permeability. It is not obvious that we can substitute
this equation in eqn. (2.523) and get the correct value for
the energy in the volume occupied by the magnet. An
energy analysis is required, and this must use the
principle of virtual work, because a change in stored
energy can be brought about only by an exchange of
mechanical work when the motor is on open circuit.
Consider a small displacement )2 such that the
operating point of the magnet moves from X to Y, as a
result of the change in the permeance of the magnetic
circuit as the magnet moves past the slot openings. The
magnet flux changes by )Mm, and if the change takes
place in a time )t, an induced voltage e ' )Mm/)t
appears at the terminals of the equivalent coil in Fig.
2.161, if we assume that the coil has one turn. The power
at these terminals is just p ' e i and the energy exchange
during the time interval )t is e i)t ' i )Mm. But i ' Fc,
the coercive MMF of the magnet. Therefore the energy
exchange with the fictitious excitation coil is Fc )Mm. Fig. 2.162 Result of a small perturbation in magnet
operating point from X to Y.
Because the coil and current source are fictitious, there
is no external electrical exchange of energy via i and e.
Nevertheless, the fictitious coil and current source must be capable of “sourcing” or “sinking” energy.
The flux change )Mm causes a change )Wmag in the stored field energy in the space occupied by the
magnet, and a further change )Wgap in the remainder of the magnetic circuit, (i.e., the airgap and the
iron). Let Wmag ' ½FmagMmag and Wgap ' ½FgapMgap, where Fmag and Fgap are the respective MMF drops
across these sections of the magnetic circuit, and Mmag ' Mgap ' Mm, the total flux.

36
These notes describe a method for calculating cogging torque which has been superseded in SPEED by various finite-element
methods [27,28]. They involve a somewhat academic discussion of the definition of energy and coenergy in magnetic circuits
containing magnets, which has been the subject of debate. There may be some small residual value in including them here.
Page 2.154 SPEED’s Electric Machines

Then ) Wmag '


1
( Fmag @ ) Mmag )Fmag @ Mmag )
2
1 (2.525)
) Wgap ' ( Fgap @ ) Mgap )Fgap @ Mgap )
2
The sum of these is

1 1
) Wmag ) Wgap ' (Fmag Fgap) ) Mm ( ) Fmag ) Fgap ) Mm (2.526)
2 2

But )Fmag )Fgap ' )Fc ' 0, since Fc is by definition constant and Fmag Fgap ' Fc. Therefore

1
) Wmag ) Wgap ' Fc ) Mm (2.527)
2

We can now complete the energy audit using the law of energy conservation. If the movement XY is
from a position of higher permeance to one of lower permeance, then mechanical work is supplied to the
motor via the shaft. The stored magnetic energies increase while the flux Mm decreases. Therefore

1
) Wm ' ) Wmag ) Wgap ! Fc ) Mm ' ! Fc ) Mm (2.528)
2

In order to move the rotor and cause the flux change )Mm, mechanical work ) Wm must have been
exerted via a torque at the shaft. This torque is the cogging torque Tcog, and if we regard torque exerted
by the motor as positive, then Tcog )2 ' )Wm and in the limit as )2 tends towards zero,

1
d Mm
Tcog ' Fc (2.529)
2 d2
The shaded triangle )Wm in Fig. 2.162 is equal to
½Fc )Mm, and therefore represents the work of cogging
torque during the rotation from X to Y. Fig. 2.163 shows
that the area )Wm is also equal to the triangle OXY.37
The change of energy )Wmag in the volume Vm occupied
by the magnet is suggested by classical theory as

mmmV m
1
) Wmag ' H dB d Vm (2.530)
2
m

This is represented by the shaded area in Fig. 2.164.


The difference between )Wm and )Wmag is the energy
exchanged with the air iron regions of the motor,
denoted as )Wgap. It is important to recognize that )Wm
' ½Fc )Mm includes all the mechanical work, even
though the expression ½Fc )Mm contains only magnet
parameters. Indeed B and H do not need to be known in
the airgap and iron regions in order to calculate )Wm
Fig. 2.163 Work of cogging torque
(and from it the cogging torque). All the necessary
information about the mechanical energy exchange is
included in the expression ½Fc )Mm. This expression also applies to magnetically linear
variable reluctance devices that are supplied from a constant current source. The factor ½ reflects
a partition of energy between mechanical energy conversion and field storage. Evidently permanent
magnets have additional storage capability beyond the integral in eqn. (2.530). It is associated with the
internal current source in Fig. 2.161 and it is, of course, what distinguishes them as “permanent”.

37
See Deodhar RP, Staton DA, Miller TJE and Jahns TM : Prediction of cogging torque using the flux-mmf diagram technique, IEEE
Transactions on Industry Applications, Vol. IAS-32, No. 3, May/June 1996, pp. 569!576.
Brushless permanent-magnet machines Page 2.155

Fig. 2.164 Change in Wmag Fig. 2.165 Wmag and Wgap

The equivalent coil model helps remove doubt about the absolute value of the stored energy in the
magnet. Consider a change in the external circuit permeance that takes the operating point up to the
point R, where B ' Br, the remanence, and H ' 0 in the magnet. The mechanical energy exchange along
XR is shown as Wmag in Fig. 2.165. For the purposes of calculating cogging torque, with the motor
windings open circuited, the magnet energy at R can be taken to be zero, because any displacement
from R requires mechanical work to be provided. Of course, in a motor with a finite airgap, the magnet
never operates at R, but approaches closest to it when no external torque is applied.
It follows from Fig. 2.163 that the total energy exchange along XR is OXR ' Wm, and therefore the energy
exchange with the air and iron regions is Wm Wmag , which is shown as Wgap in Fig. 2.165. Fig. 2.165
also shows the separate components of the coercive MMF Fc : that is, Fgap across the air iron part of the
magnetic circuit and Fmag across the magnet itself. When the magnet is working near the remanence
point R, the total stored energy is small and the magnet energy is smaller than the airgap component.
If the rotor is displaced so that the magnet operating point moves close to the coercive point C, the total
stored energy is large: an external torque must have been applied to the shaft to make this change, and
the energy stored in the magnet is larger than that stored in the gap. The gap energy is low because the
flux density is reduced to a low level, as a result of the reduction in the circuit permeance. The
potential energy given to the magnet will be recovered if the shaft is allowed to turn back to its original
position. During continuous rotation the cogging torque is generated as the operating point cycles
between points such as X and Y. Point X is a point of maximum permeance and point Y is a point of
minimum permeance. When the rotation is from X to Y, the stored energy is increasing and the cogging
torque is a retarding torque in the direction of rotation. At Y the rotor continues in the same physical
direction but the operating point moves towards X, which corresponds to a new maximum permeance
position. The angle of rotation between two X’s depends on the number of slots and poles; in an
integral slot motor, it is equal to the slot pitch.
Fig. 2.163 also shows that the area ) Wm used to evaluate the cogging torque can be taken as an
increment in the total stored field energy, or as an equal and opposite increment )Wm in the coenergy.
The total coenergy is represented by triangle OCX and its components WNgap and WNmag are shown in Fig.
2.165. Since there is no external electrical energy exchange, it does not matter which one is used, and
the cogging torque is the total derivative dW m/d2. The PM motor on open circuit differs from the
electrically excited actuator or motor, where it is necessary to use partial derivatives MWN(i, 2)/M2 or
MWf(M , 2 )/M2 and keep to the rule of “constant MMF” or “constant flux” respectively. In the PM motor
on open circuit there is no distinction between these partial derivatives, except as to their sign.
Page 2.156 SPEED’s Electric Machines

Even when the magnetic circuit is linear, the coenergy and energy are in general unequal, i.e. OXR is
not generally equal to OCX. This is a further difference between the PM motor on open circuit and the
normal electrically excited device.
The sign of the cogging torque is not particularly important, because it is purely oscillatory when the
rotor rotates continuously, and since it cannot sustain any continuous energy conversion, its average
value must be zero. Suffice it to say that the cogging torque will be such that mechanical work must
be supplied to the motor when the magnetic stored energy is increasing, and mechanical work will be
supplied by the rotor when the magnetic stored energy is decreasing.
Finite element procedure
In a finite element analysis we must rotate the rotor in small steps (e.g. 1E) and at each position
calculate the change in the total stored energy. By evaluating the triangle OCX for each magnet
element, we are automatically including the coenergy stored in the “airgap” (i.e., in the rest of the
magnet circuit), and therefore it is not necessary to evaluate Wgap separately by integrating BH/2 in the
air and iron regions. The same is true if we use triangle OXR to evaluate the total stored field energy
due to the current magnet element. The required process is as follows: for each magnet element, per
metre of stack length,

Coenergy Triangle OCX ½Hc C B × element area


Stored field energy Triangle OXR ½ Br C H × element area
TABLE 2.7

Fig. 2.166 Coenergy and energy calculations

The coenergy and energy calculations are shown in Fig. 2.166. Since Br and Hc are constants, the areas
require the integration of a linear function over each element. The integral on the right of Fig. 2.166 is
2 2
the coenergy evaluation that would be done in air regions using ½B H ' ½B /μ0 ' ½μ0H , and it is
quadratic. According to the theory, this is not needed for calculating the cogging torque.
We can now use one of the magnetic energy evaluations in Figs. 2.166. Once the element coenergy
integral of Fig. 2.166 or Table 2.7 has been summed over the entire magnet region, it represents the
coenergy of the entire machine in the same way that OCX represents this coenergy for a lumped magnet
in Fig. 2.163. When the rotor moves from position X to position Y, the change in this coenergy sum will
be of the form OCX OCY, and this will give the average torque through the interval )2 ' XY. On the
other hand, if we use the alternative magnetic energy integral in Fig. 2.166, the total energy change from
X to Y will be of the form OYR OXR. Both methods should produce the same result.
Because of “discretization noise”, the total energy sums should be extracted from the FE analysis and
fitted with a cubic spline; then the differentiation to get the cogging torque will be smoother. This
method helps avoid the discretization noise which troubles the Maxwell stress method.
Brushless permanent-magnet machines Page 2.157

Deodhar’s method: A simple method for evaluating the


triangles OXY treats the entire magnet as a single element.
The magnet flux Mm is evaluated the from the difference of
vector potential at the magnet edges (multiplied by the
stack length), and used together with the magnet MMF
Fig. 2.167 Flux calculation
obtained from the relation F ' Lm/μrecμ0 × (Mm /Am Br) to
plot the triangle OXY directly, moving from position to
position and calculating the torque at each step. The vector
potentials can be evaluated at points shown in Fig. 2.167 on
the edges of the magnets, using the expression Mm ' (A1
A2) × Lstk, where Lstk is the stack length or magnet length in
the axial direction. If the centreline of the magnet is a line
of symmetry, generally A ' 0 there and A2 ' A1, so that Mm
' 2A1 Lstk. An alternative is shown in Fig. 2.168, where Mm
is approximately equal to 2(AQ AZ) × Lstk. In many cases
AZ will be zero.
This method could be made more efficient by recognizing
that OXY ' ½Fc )Mm where )Mm ' MY MX. Alternatively,
OXY ' ½Mr (FY FX) where FY ' HY × Lm and FX ' HX × Lm,
and HX and HY are the values of H in the magnet at the
positions X and Y respectively, evaluated by H ' (B
Br)/μrec μ0. Together with the integrals in Table 2.7, Fig. 2.168 Alternative method for Mm
evidently there are several simple processes for
determining the cogging torque once the FE solution is
available.

Fig. 2.169 Variation of airgap flux-density

Approximate method: The rotor is stepped round in intervals of 1Eelec. At each position the airgap
flux density distribution is calculated including the effect of slotting. In the interests of rapid
calculation the slot modulation is very crude. It is estimated using an effective airgap that varies as
shown in Fig. 2.169 as the point P sweeps across the slot opening. Mm is evaluated at each rotor position
by means of a ratio function Lm/(Lm gN), where gN is the effective airgap at each position of the point
P. Then the total airgap flux is evaluated by integrating the airgap flux over the pole pitch, and the
rotor is stepped to the next position.
Fractional slot motors can be accommodated by evaluating the cogging torque for the N and S pole
magnets independently, and then adding them together. Both methods allow for skew by summing the
effects taken at several axial positions.
Page 2.158 SPEED’s Electric Machines

2.24 PERFORMANCE SIMULATION — THE BASIS OF DYNAMIC DESIGN


Simulation means the computation of time-waveforms of current, torque, etc. It is almost always based
on the terminal voltage equations. The simplest case is that of a single-phase machine which has only
one terminal voltage equation,
v ' Ri pR, (2.531)

where v is the applied voltage at the terminals, R is the resistance, i is the current, R is the flux-linkage,
and p is the operator d/dt so that pR is the rate of change of flux-linkage. By Faraday's law, the term pR
is the total voltage induced by time-variation of the flux-linkage, including any EMF as well as the
familiar L di/dt. Thus it includes both “transformer” and “rotation” voltages.
Eqn. (2.531) is an ordinary differential equation in three variables v, i and R. Usually v is known (at
each instant during the simulation) from the states of the power electronic switches in the drive. This
leaves two unknowns, i and R. For PM machines it is common to write
R ' QM ( 2 ) Li (2.532)
where QM(2) is the flux-linkage produced by the magnet as a function of rotor position 2, and L is the
inductance.
As the rotor rotates, the flux-linkage QM varies, often sinusoidally, even though the magnet flux is
approximately constant. In that case
MQM MQM d 2 di dL MQM di dL di dL
pR ' L i ' T L iT ' e L iT , (2.533)
Mt M2 d t dt dt M2 dt d2 dt d2
the first term being zero if the variation of QM is solely due to rotation, and the fourth term being zero
if the inductance is constant. Here T ' p2 or d2/dt, the angular velocity, and e is the EMF, which is a
known function of 2. Substituting eqn. (2.533) in eqn. (2.531), we get
di dL
v ' Ri e L iT , (2.534)
dt d2

Eqn. (2.532) relies on the principle of superposition, so it and eqns. (2.533) and (2.534) are valid only when
the magnetic circuit is linear. This cannot be overemphasized, and it is important in the following
sections to distinguish between solutions of eqn. (2.534) and solutions of eqn. (2.531). Both forms will
be used, because some machines (such as the surface-magnet machine) are sufficiently linear to permit
the decomposition in eqn. (2.532), while others (such as the IPM) are so saturated that the EMF (which
is strictly definable only under open-circuit conditions) becomes unobservable under loaded conditions,
and it becomes necessary to work with the total flux-linkage R.
The solution of the ordinary differential eqn. (2.531) is usually a straightforward matter of numerical
integration. For example, by Euler's method we can write the integration as a time-stepping process

R ' (v R i ) dt . ( v R i ) )t [R] , (2.535)

where )t is the time step and [R] means the value of R at the previous time-step. After each time-step
it is necessary to determine the current from the new value of R at the new rotor position, which is
simple to calculate if the speed is assumed constant.38 If the machine is linear, eqn. (2.532) can be used
to extract the new current i, and then the integration process can proceed to the next step. If it is
nonlinear, the new current must be extracted by inverting or solving a relationship of the form
R ' R ( i,2 ) , (2.536)

which expresses the so-called magnetization curves.39 Eqn. (2.532) is an example of linear magnetization
curves, depicted in Fig. 2.170. The curves move up and down cyclically as the magnet rotates.

38
If the speed is not constant, the rotor position must be determined by integrating the two mechanical equations of motion.
39
Note that in this process it is the flux-linkage that is the state variable, not the current.
Brushless permanent-magnet machines Page 2.159

Eqn. (2.533) is often integrated with current as the state


variable rather than flux-linkage. But current varies much
more rapidly than flux-linkage, placing more demands on the
numerical integration process and ultimately resulting in a
slower solution. With current as the state variable, it is
necessary to resort to “incremental inductance” and “frozen
permeability” models to extract the separate terms in eqn.
(2.533), and physical interpretation becomes difficult. This is
ironic in the sense that inductance is intended to simplify
physical interpretation, not to complicate it. It results in
simplification only when the machine is magnetically linear.
Eqns. (2.531-2.536) are valid for all single-phase machines
including reluctance machines (in which QM ' 0), and also for
machines with saliency on the rotor or the stator or both. Fig. 2.170 Linear magnetization curves

We have already seen examples of computer simulation equations for magnetically linear machines in
§2.6, applied to small surface-magnet motors in which saturation was negligible. For motors having
more than one phase, a voltage equation of the form of eqn. (2.534) is written for each phase, including
mutual inductance terms. The electromagnetic torque is calculated using eqn. (2.421).

Simulation in dq axes
The dq transformation was developed to deal with salient-pole machines. It transforms the voltage
equations (such as eqn. 2.534) into a frame of reference fixed to the rotor, and this removes the basic
mathematical nonlinearity associated with the dL/d2 terms. There are several other advantages for
working in dq-axes. First, the large number of variable inductances is reduced to essentially two
constant inductances — the synchronous inductances Ld and Lq. In the steady-state, all voltages,
currents and flux-linkages appear to be constant. This in turn makes it much easier to deal with the
effects of saturation, since the main effect is to alter the values of Ld and Lq (and possibly also the EMF
E). As a result, physical interpretation of the operation of the saturated machine is much clearer.
The dq model is developed from Fig. 2.69 which defines the reference axes of the three phases a,b,c and
of the d and q axes. The transformation is given on p. 128, and it results in two differential voltage
equations (2.434) and (2.435), which replace the three voltage equations of the form of eqn. (2.534).
Step-by-step integration of the differential equations (2.434) and (2.435) is the same as for eqn. (2.531).
For example, by Euler's method eqn. (2.434) becomes

Rd ' ( vd R id p 2 @ Rq ) dt . ( vd R id p 2 @ Rq ) )t [Rd] , (2.537)

where [Rd] means the value of Rd at the previous time-step. Eqn. (2.435) is treated similarly.

Inversion of the flux-linkage to recover the current — magnetization curves


At the end of each integration-step, the currents id and iq must be updated from the new flux-linkages.
This is tantamount to inverting the inductance matrix. In general the relationships between flux-
linkage and current are called magnetization curves. In the classical synchronous machine with sine-
distributed windings and no saturation, they are linear:
Rd ' QMd1 Ld id ; and Rq ' Lqiq , (2.538)

where Ld and Lq are the synchronous inductances and QMd1 is a constant flux-linkage produced by the
fundamental component of flux excited by the magnet or the field-winding, depending on the type of
machine. When Ld and Lq are constant, these equations have the form shown in Fig. 2.171.
It is a trivial matter to invert eqns. (2.538) to get the new currents from the new flux linkages at each
time-step. In the classical case the d and q axes are uncoupled and the magnetization curves are
independent of the rotor position 2.
Page 2.160 SPEED’s Electric Machines

Fig. 2.171 Linear magnetization curves in dq axes Fig. 2.172 Nonlinear uncoupled magnetization curves in dq axes

Saturation, cross-coupling, and residual variation of parameters with rotor position


Especially in IPM motors Ld and Lq are not constant, but vary greatly as a result of saturation. In the
simplest case Rd is assumed to be a function only of id, and Rq only of iq, with no “cross-saturation”.
Then the magnetization curves have the form
Rd ' Rd ( id ) and Rq ' Rq ( iq ) , (2.539)
in which there is no cross-coupling and no variation with rotor position. These curves are shown in
Fig. 2.172. They are easily inverted to get id from Rd and iq from Rq.
Unfortunately there often is cross-coupling between the d and q axes. The q-axis flux path is normally
highly permeable and may saturate easily at quite low values of iq, but in doing so it reduces the
permeability of the d-axis flux-path and interferes with the values of both QMd1 and Ld. This can be seen
in Fig. 2.172 as a distortion of the Rq(iq) curve that depends on the value of id. Conversely the Rd(id)
curve is distorted by iq. Assuming that these effects are independent of the rotor position, the
magnetization curves can be represented by equations of the general form
Rd ' Rd ( id , iq) and Rq ' Rq ( id , iq) . (2.540)
Because of the absence of rotor position 2 from these equations, they are still classified as static
magnetization curves in spite of the cross-saturation effect. But there is a still more complex case
wherein the nonlinear cross-coupled magnetization curves vary with the rotor position 2 in spite of the
fact that they are in dq axes. The magnetization curves then have the general form
Rd ' Rd ( id , iq ,2) and Rq ' Rq ( id , iq ,2) . (2.541)
In other words, Rd is a function not only of id but also of iq (cross-coupling) and 2, and likewise Rq is a
function of id and 2 as well as iq. The additional dependence on 2 makes matters very much more
complicated and is liable to arise if the windings are not sinewound; in other words, if the harmonic
winding factors are not all zero. Unfortunately this is often the case, to a greater or lesser degree, with
all practical windings. The curves (2.541) require a vast number of finite-element calculations to define
them, and an appropriate interpolating function that could be used to invert them.
The aim of classical dq theory is to get rid of 2 from the modelling equations. Physically it arises from
the winding harmonics and from the variation of the overall magnetic permeance as the rotor rotates.
The obvious source of such permeance variation is saliency on the stator, which includes residual
grain-orientation in the steel, the presence of notches or holes in the lamination, and the effects of
slotting. Even with sine-distributed windings, if the magnetic circuit is saturable there remains the
possibility of a functional influence of 2, induced by the variation of current as the rotor rotates,
especially if the current waveform is not sinusoidal and even more so if the phases are unbalanced. But
all of these effects are normally ignored in classical analysis because of the tacit assumption that
machines are designed to be "sinusoidal" so that the influence of 2 in eqns. (2.541) is weak.
Brushless permanent-magnet machines Page 2.161

Even without 2 the cross-coupled magnetization curves in eqn. (2.540) are complicated enough, but for
accurate simulation these curves will often need to be modelled in detail. They can be calculated by the
finite-element method, and with a suitable interpolating function their inverses
id ' id ( Rd , Rq ) and iq ' iq ( Rd , Rq ) (2.542)
can be used in the step-by-step integration of eqns. (2.434) and (2.435). Of course the finite-element
calculations must be set up with the rotor in a particular orientation relative to the slotting and the
winding axes. If the orientation turns out to affect the resulting curves, it means that we were wrong
to drop 2 from eqns. (2.541) in the first place. The process works rigorously with "static" magnetization
curves only if they really are "static" (i.e., independent of 2). The extent to which "static" curves can
still be used when there is a variation with 2 is a matter of judgement, but it is an important question
because the time, cost and complexity of the solution are important practical matters.
In the general case we must keep the influence of 2 and use eqns. (2.541), not the static version (2.540)
or the linearised version (2.538). Given that Rd and Rq both depend on three variables (id, iq and 2), it is
questionable whether the dq transformation has simplified the problem. Why not integrate the voltage
equations directly, without transformation? This would require magnetization curves in the form
Ra ' Ra ( ia , ib , ic , 2 ) ; Rb ' Rb ( ia , ib , ic , 2 ) ; Rc ' Rc ( ia , ib , ic , 2 ) , (2.543)
in which we have three flux-linkages, each dependent on four variables (ia, ib and 2). An immediate
simplification arises if we assume that
ia ib ic ' 0 (2.544)
which is characteristic of a 3-wire connection (wye or delta), and likewise for the flux-linkages
Ra Rb Rc ' 0 . (2.545)
The constraints (2.544) and (2.545) can be used to reduce the number of independent currents and flux-
linkages from three to two, so that we will have two flux-linkages dependent on three other variables
(two currents plus 2) — exactly the same level of complexity as in eqn. (2.541). We should note (perhaps
belatedly) that eqns. (2.544) and (2.545) were tacitly assumed throughout the earlier discussion of the
dq equations, by ignoring the “zero-sequence” component. For mathematical completeness, it is
included in the transformation (p. 128), even though we have no plans (and no need) to use it.

Other reference frames : Clarke components, space vectors


The formulation of a 2-current/2-flux-linkage model is well known as the “3-phase to 2-phase
transformation”, and one form of it is known as Clarke's transformation, or “",$ components”, which
we have seen in eqn. (2.404) on p. 124. Written for flux-linkages, the transformation equations are

2 Rb Rc 1
R" ' Ra ; R$ ' Rb Rc (2.546)
3 2 3
with inverses

1 1
Ra ' R" ; Rb ' 3 R$ R" ; Rc ' 3 R$ R" (2.547)
2 2
when eqns. (2.544) and (2.545) are satisfied. The same transformation is used for currents and voltages.
In ",$ components the magnetization curves have the form
R" ' R" ( i" , i$ ,2) and R$ ' R$ ( i" , i$ ,2) . (2.548)
For synchronous machines the functional influence of 2 is strong, because the ",$ components are
essentially fixed in the stator frame of reference. By contrast the d,q components are essentially fixed
in the rotor frame of reference, and this is what weakens or eliminates the functional influence of 2.
Therefore in ",$ components there is no possibility of simplifying the magnetization curves as we did
in going from eqns. (2.541) to eqns. (2.540) or (2.539).
Page 2.162 SPEED’s Electric Machines

The essential practical difference between the d,q system and the ",$ system is that the flux-linkages
Rd, Rq tend to vary slowly — indeed in the steady state in an ideal machine they are constant. But R"
and R$ are continually varying, even in the steady state. For computational efficiency and stability in
integrating the differential voltage equations, all these factors make Rd and Rq the better choice.
It can be observed in passing that for induction motors there is ideally no distinction between the d axis
and the q axis of the rotor — i.e., no saliency — and this simplifies the structure of the magnetization
curves to some extent. For digital simulation purposes it remains preferable to work in dq axes for the
reasons stated above. If the dq axes are fixed to the rotor, then Rd and Rq in the steady state will vary
at slip frequency (which is normally low); but if the dq axes are fixed to the “rotor flux”, then they are
constant as in the synchronous machine.
It is also common to analyse the operation of induction and synchronous motors controlled by
frequency inverters using ",$ components in complex form such as i" ji$, which is known as a space
vector. This formulation is popular in the theory of field-oriented control. If the windings are sinewound,
the polar form of the complex number i" ji$ represents the magnitude and phase of the ampere-
conductor distribution at any instant. When the phase currents are sinusoidal functions of time, i" and
jT t
i$ can be represented by complex phasors which are coefficients of the operator e , which interpreted
as rotating the instantaneous ampere-conductor distribution in the machine at the synchronous speed
T electrical radians/second. The voltage and the flux can also be represented by space vectors, giving
rise to an elegant means of expressing the circuit theory of the inverter-fed motor and its dynamic
operation. For non-sinewound motors this theory is not strictly valid.

Determining saturated Ld and Lq using the finite-element method


The objective is to determine suitable “saturated” values of Ld, Lq and Q1Md. For any set of values of id
and iq, the flux-linkages Rd and Rq can be computed by finite-element analysis, and then Ld and Lq can
be deduced from eqns. (2.538) as
Rd QMd1
Ld ' ; (a)
id
(2.549)
Rq
Lq ' . (b)
iq

A unique value of Lq can be deduced from eqn. (2.549b), for any value of iq . But in order to deduce Ld
we also need a value for the “open-circuit” value QMd1. The obvious choice is to set QMd1 equal to the
value of Rd computed with id ' 0 and iq ' 0. However, because of cross-saturation, there is a possibility
for the numerator (Rd QMd1) to become nonzero when iq is restored to the normal load-point value,
even when id ' 0. Eqn. (2.549a) would then produce an indefinite result for Ld if it was used at (or near)
such a load-point where id ' 0 and iq was nonzero, which is quite a normal condition.
This problem is avoided if QMd1 is calculated with id ' 0 and the load-point value of iq. In this case QMd1
is computed with the full effect of cross-saturation in the q-axis, which tends to decrease its value.
Thereafter the additional term Ldid can be interpreted as the flux-linkage of armature reaction in the
d-axis, in the presence of the cross-magnetizing current iq.
This interpretation is “heuristic”, and not mathematically rigorous, because it tacitly relies on the
notion of superposition. However, the process is systematic and reproducible, and makes sense in
engineering terms because it somehow separates the effect of Ld and id from the effect of iq. It also avoids
the need to account for cross-saturation by introducing an artificial mutual inductance Ldq with an
additional term in eqns. (2.549a and b). This term would itself be saturable. There is in fact an infinite
number of possible values of QMd and Ld that will satisfy eqn. (2.549a). The reason is that the
superposition implied by the addition in this equation is not valid under saturated conditions.
Brushless permanent-magnet machines Page 2.163

Properties of the elliptical energy-conversion loop (i-psi diagram)

Flux-linkage ψa N
(phase a)

M ψpk
ψM
Z P
Q ψQ
High current loop

Current in Phase a ia

d
a b c
Low current loop

Fig. 2.173 i-R diagram or energy-conversion loop


Two elliptical i-R loops are shown in Fig. 2.173, for two operating conditions, one at high current and
one at low current. The general form of the phasor diagram is also shown, together with the space
phasor diagram of flux-linkages, in which QMd is the notional “open-circuit” flux-linkage due to the
magnet. LdId and LqIq are the flux-linkages of armature reaction in the d- and q-axes respectively. The
rotating fluxes associated with these flux-linkages generate the respective induced voltages in the
phasor diagram: thus QMd generates Eq1, LdId generates jT LdId , LqIq generates jT LqIq , and the
resultant Q generates the “airgap voltage” V. Resistance is omitted.
Under ideal conditions the i-R loop is elliptical, which implies that both the current and the flux-linkage
waveforms are sinusoidal in all three phases, and this in turn implies that Ld, Lq ,QMd, id and iq are all
constant throughout the cycle.
For the high-current loop, the current reaches its maximum positive value at M, and passes through
a negative-going zero at Q. The flux-linkage Ra reaches its maximum value at N. Since the loop is
traversed counterclockwise, M precedes N. This is consistent with the fact that I leads Q.
The phasor diagram tells us that the angle of rotation between M and N is $, which is the phase angle
in Eelec between the current phasor I and the flux-linkage phasor Q. If we take the “open-circuit” flux-
linkage QMd as reference, we can use the phasor diagram to write the following equations for the
instantaneous currents, with 2 = Tt :
ia ' ipk cos ( B / 2 ( 2) ' ipk sin ( ( 2);
ib ' ipk cos ( B / 2 ( 2 2B/3) ' ipk sin ( ( 2 2B/3); (2.550)
ic ' ipk cos ( B / 2 ( 2 2B/3) ' ipk sin ( ( 2 2B/3).
Note that ipk is the abscissa at point M. If these equations are substituted in the forward dq
transformation (see eqns. (2.427) and (2.428) on p. 128), we get
id ' ipk sin ( ; iq ' ipk cos ( . (2.551)
in which id and iq are constant. The corresponding equations for the instantaneous flux-linkages are
Ra ' Rpk cos ( B / 2 ( $ 2) ' Rpk sin ( ( $ 2);
Rb ' Rpk cos ( B / 2 ( $ 2 2B/3) ' Rpk sin ( ( $ 2 2B/3); (2.552)
Rc ' Rpk cos ( B / 2 ( $ 2 2B/3) ' Rpk sin ( ( $ 2 2B/3).
Page 2.164 SPEED’s Electric Machines

Note that Rpk is the ordinate at point N. Again from the dq transformation, we get
Rd ' Rpk sin ( ( $ ) ' Rpk cos " ;
(2.553)
Rq ' Rpk cos ( ( $ ) ' Rpk sin " .
Like id and iq, these are constant values; they do not appear in Fig. 2.173.
When ia ' 0, we have ib ' (/3/2)ipk and ic ' (/3/2)ipk, and this represents a negative-going zero of ia
at the instant Q defined by ( 2 ' 0, or Tt ' (. To determine the value of Ra at this point (Q), we can
use eqn. (2.552), which gives
Ra ' Rpk sin $ ' RQ . (2.554)
Thus when ia ' 0, the flux-linkage Ra in phase a at point Q is not simply the open-circuit value, but
depends on the inductances Ld and Lq and the phase angle $. In fact the true open-circuit flux-linkage
is not observable in Fig. 2.173 unless the i-R loop is drawn for zero current. When this is done, the loop
degenerates into a vertical straight line and the maximum value of Ra is equal to the peak open-
circuit flux-linkage /2QMd. The low-current loop in Fig. 2.173 shows this point very nearly at Z. There
is no simple relationship between Z and Q. But note that the R-values at Q and N are in the ratio sin $.
When ( ' 0, the current is in the q-axis and RQ is then equal to the peak value of “magnet flux-linkage”
in phase a, since $ ' B/2 " and Q cos " ' Q sin $ ' QMd. If the magnetic circuit is linear, this is equal
to the open-circuit value of the magnet flux-linkage in phase a, because although ib and ic are not zero
at point Q, they produce a transverse flux which does not link phase a. In that case points Q and Z are
coincident. But if Iq is sufficient to cause appreciable saturation of the magnetic circuit, then sin $ and
RQ can both be affected. In that case point Q deviates from point Z.
Another point of interest on the ellipse diagram is M, where ia ' ipk. At this point, sin (( 2) ' 1, so
2 ' ( B/2 and if this is substituted in eqn. (2.552) we get
Ra ' Rpk cos $ ' RM . (2.555)
Note the ratio
RQ
' tan $ . (2.556)
RM
But $ can be written as B/2 N, where N is the “internal power-factor angle”, that is, the phase angle
between the “internal” or “airgap” voltage V and the current I. Then
RM
tan N ' , (2.557)
RQ
from which the internal power factor cos N can easily be determined. When cos $ ' 0, $ ' B/2 and the
current I is in quadrature with the flux-linkage Q. In this case M lies on the horizontal axis and the
ellipse is not tilted; the internal power-factor is then 1. Although this condition represents the
maximum possible internal power-factor, it does not necessarily indicate the maximum torque per
ampere, unless Ld ' Lq. The greater the tilt angle, the lower the internal power-factor.
Torque — The average electromagnetic torque can be calculated from eqn. (2.247) or (2.248), using d,q
values for flux-linkage and current. The d,q values can be determined from instantaneous values using
the following expressions, which follow from eqns. (2.551) and (2.553):
1 1
Qd ' Rpk cos " ; Qq ' Rpk sin " ;
2 2
(2.558)
1 1
Id ' ipk sin ( ; Iq ' ipk cos ( .
2 2
Substituting in eqn. (2.247),
1
Te ' m p Rpk ipk ( cos " cos ( sin " sin ( )
2
. (2.559)
1
' m p Rpk ipk cos( " ( )
2
Brushless permanent-magnet machines Page 2.165

But from Fig. 2.173, B


" ( ' $. (2.560)
2
Therefore Rpk ipk B Rpk ipk
Te ' m p cos ( $) ' mp sin $ . (2.562)
2 2 2
If Rpk sin $ is now substituted from eqn. (2.554), we get
mp
Te ' ipk RQ . (2.563)
2

Eqn. (2.563) shows that the average electromagnetic torque can be computed from a single finite-element
computation in which RQ is computed. If the current and flux-linkage waveforms are both sinusoidal,
the result will be independent of the rotor position. If they are not sinusoidal, the i-R loop is not
perfectly elliptical, and it is unsafe to rely on a single point.
Torque per ampere — Eqn. (2.563) also shows that RQ is a direct measure of the torque per ampere.
If ( ' 0 and the current is in the q-axis, Id ' 0 and LdId ' 0, and the phasor diagram then shows that if the
magnetic circuit is linear, point Q is independent of the current and the torque per ampere is also
constant. But if Iq is sufficient to saturate the magnetic circuit, point Q moves downwards and the
torque per ampere decreases. This is often termed “kT roll-off”, kT being the torque per ampere. If (
is non-zero, the effect of saturation is more complicated because although eqn. (2.563) remains valid,
RQ varies in a more complex manner as a function of both the current and the phase angle (.

Fig. 2.174 i-R loop for squarewave drive Fig. 2.175 i-R loop for six-step operation

Squarewave and six-step drives — Fig. 2.174 shows the ideal form of the i-R loop for a squarewave
drive in a motor which has a trapezoidal EMF waveform, as in Fig. 2.7. When the current waveform is
phase-advanced relative to the flux-linkage waveform, the right-hand paralellogram ABCD moves
vertically downward, while the left-hand parallelogram moves upward. The loop area W is twice the
area of parallelogram ABCD, and if AD ' 2Rpk, then from eqn. (2.445) we get
2mp
Te ' Rpk ipk ×
, (2.564)
B
where m is normally equal to 2 for a three-phase drive, since only 2 phases are on at any time. p is the
number of pole-pairs, and the torque constant given by this equation is identical to kE in eqn. (2.7).
The same process using eqn. (2.445) can also be used to derive eqn. (2.563) for the sinewave motor with
sinewave drive, since the area W of the ellipse in Fig. 2.173 is simply BRpkipk.
When the shape of the i-R loop is regular, as in Figs. 2.173 and 2.174, the loop area W in eqn. (2.445) can
be evaluated very simply, and it may be admissible to use only a few finite-element computations. The
accuracy will depend on the degree to which the shape of the loop is affected by saturation, winding
harmonics, or harmonics in the waveforms of the current and/or flux-linkage.
Fig. 2.175 shows the i-R loop of a three-phase motor with six-step drive. Because of the irregular shape
of the loop, it is clear that many finite-element computations may be needed to get an accurate value
for the area W and hence the electromagnetic torque.
Page 2.166 SPEED’s Electric Machines

2.25 SOME USEFUL RELATIONSHIPS IN DQ AXES

dq voltages in terms of line-line voltages


In a 3-phase drive with a 3-wire connection, it is the line-line voltages that are known, not the line-
neutral voltages. For the wye connection we have
2
vd ' vab cos 2 vbc cos (2 B/3)
3
(2.565)
2
vq ' vab sin 2 vbc sin (2 B/3)
3
in which only vab and vbc are required, because vab vbc vca ' 0. The inverse is

vab ' va vb ' 3 [ vd sin (2 B/3) vq cos (2 B/3) ]


vbc ' vb vc ' 3 [ vd sin 2 vq cos 2 ] (2.566)
vca ' vc va ' 3 [ vd sin (2 B/3) vq cos (2 B/3) ] .

Eqns. (2.434) and (2.435) are solved for id and iq, with vd and vq as driving functions, and then the inverse
transformation eqn. (2.429) is used to recover the phase currents.
There is no distinction between phase currents and line currents in the wye connection. But for the
delta connection the line currents must be reconstructed from
i A ' i1 i3 ; i B ' i2 i1 ; i C ' i3 i2 , (2.567)
in accordance with Fig. 2.5. For the delta connection the phase voltages are identical to the line-line
voltages, so vd and vq can be obtained directly using eqns. (2.434) and (2.435) : according to Fig. 2.5,
vAB ' v1 ; vBC ' v2 ; vCA ' v3 . (2.568)
When the delta is closed, v1 v2 v3 ' 0, and if this is used to eliminate v3 (i.e., vCA), the result is

2 B
vd ' 3 vAB sin ( 2 ) vBC sin 2 ;
3 3
(2.569)
2 B
vq ' 3 vAB cos ( 2 ) vBC cos 2 .
3 3

This form is more convenient to use in simulation when vAB and vBC are the known driving functions.

Particular positions
Finally, let's have a look at some interesting examples (particular positions), which may be useful in
measurements with the rotor stationary. For example, if id ' I and iq ' 0, then when the rotor d-axis is
aligned with the axis of phase 1, 2 ' 0 and
ia ' I ; ib ' ic ' I/2. (2.570)

This condition is shown in Fig. (2.79). Again, if id ' 0 and iq ' I, then with 2 ' 0

3
ia ' 0 ; ib ' ic ' I . (2.571)
2
In general when 2 ' 0,
ia ' id
1 3
ib ' id cos ( 2 B / 3 ) iq sin ( 2 B / 3) ' id iq
2 2 (2.572)
1 3
ic ' id cos ( 2 B / 3 ) iq sin ( 2 B / 3) ' id iq .
2 2
Brushless permanent-magnet machines Page 2.167

2.26 MOTORS WITH FRACTIONAL SLOTS/POLE

Pitch factor 
(a)
The basic idea of “pitch factor” is to measure the
effectiveness of a coil or winding in producing or
linking flux, relative to that of a “full-pitch” coil.
S N S
The fundamental pitch factor of a coil is the ratio
Fund. MMF produced by coil
k p1 ' (2.573)
Fund. MMF produced by full&pitch coil
Flux
This definition can be shown to be identical to or Coil MMF

Fund. flux linked by coil


k p1 ' (2.574) 
Fund. flux linked by a full&pitch coil  Pole-pitch
 Coil-pitch

(b)
Fig. 2.176(a) shows a full-pitch coil and an array of
full-pitch magnets which are included to define the
pole-pitch: each magnet spans exactly 180 electrical S N S
degrees. The coil has a span J ' 180E and produces
the MMF distribution shown by the solid line. The
fundamental of this MMF wave is shown by the
dashed line; by the well-known Fourier analysis of
Flux
a square wave, its amplitude is 4/B times the or Coil MMF
amplitude of the rectangular wave.
Fundamental_Pitch_Factor.wpg

Fig. 2.176(b) shows a short-pitched coil having a span


" < J, together with its fundamental component. Fig. 2.176 Pitch factor

The fundamental pitch factor is


"/2
1 @ cos 2 d2
&"/2 "
kp1 ' ' sin . (2.575)
J/2 2
1 @ cos 2 d2
&J/2

For example, a coil with a span of 150E has a fundamental pitch factor kp1 ' sin 75E ' 0@966.
The pitch factor can be extended to include harmonics. For the n'th harmonic we have
"/2
1 @ cos n2 d2
&"/2 n"
kpn ' ' sin . (2.576)
J/2 2
1 @ cos n2 d2
&J/2

Note that kpn = 0 if n"/2 ' 180E. For example, if " = 150E and n = 5, kp5 = 0. A coil with a span or pitch
of 150E produces no 5th harmonic MMF and therefore it produces no 5th harmonic flux. It also links no
5th harmonic flux produced by the magnet. Similarly a coil with a span of 120E produces no 3rd
rd
harmonic MMF and links no 3 harmonic flux.
For a winding made up of many coils, the winding factor for any harmonic can be computed as the
weighted sum of the pitch-factors of the individual coils, taking into account the direction of the MMF
axis of each coil: thus with coils of N1, N2, ...turns with spans "1, "2, ... oriented at angles N1, N2, ...
relative to a reference axis,
N1 sin n"1 cos nN1 N2 sin n"2 cos nN2 ...
kpn ' (2.577)
N1 N2 ...
The denominator of this expression represents the pitch-factor of the winding that would result if all
the coils had a span of 180E and were concentrated together with the same MMF axis. Fig. 2.177 shows
the addition of harmonic MMFs (or EMFs) in a winding of this type, having three phases.
Page 2.168 SPEED’s Electric Machines

Terminal phase voltage

Voltage of one coil

Fig. 2.177 Addition of coil MMFs or EMFs, illustrating the composition of the winding factor for any harmonic

Squarewave and sinewave motors. With squarewave motors it seems logical to design the winding
to maximize the winding factors for all harmonics — in other words, to make the winding behave as
nearly as possible to a full-pitch winding with coilsides concentrated exactly at intervals of 180E elec.
With sinewave motors, on the other hand, it seems logical to design the winding to maximize the
fundamental winding factor and, as far as possible, to minimize the winding factors for all the
harmonics. In both cases, practicalities interfere with the idealized objectives.
In the case of sinewave motors the mathematically rigorous way to eliminate all the harmonic winding
factors except the fundamental is to design a winding with a perfectly sinusoidal distribution of
conductors per radian, but this is clearly impossible because it would require an infinite number of
infinitesimally small conductors. A satisfactory practical arrangement is often to design for relatively
small 5th and 7th harmonic winding factors, while leaving the third-harmonic to be suppressed by the
three-wire star connection. Although this is far from the perfect winding, the back EMF will be nearly
sinusoidal if the rotor is designed with low flux harmonics, such as, for example, with a profiled rotor
surface or a Halbach-type magnet. Higher-order harmonics are often ignored on the supposition that
their magnitudes decrease with increasing harmonic order.
The harmonic MMFs contribute to the so-called differential leakage inductance, as we have seen in an
earlier section. In some cases it may be desirable to design for a higher inductance, in which case the
combination of a winding with high MMF harmonics with a sinusoidally-magnetized rotor will produce
a sinusoidal EMF with a relatively high inductance.
In recent years there has been much interest in motors with a small number of slots/pole having single-
layer windings and even irregular slotting to maximize the winding factor at the same time as reducing
cost by minimizing the number of coils. An example is shown in Fig. 2.183 on p. 171.

Examples of winding harmonics


Figs. 178, 179, and 180 show examples of the harmonic content of three windings which are typical of
the modern interest in fractional values of slots/pole close to 1. In each case the winding factors are
displayed in the chart at lower right, while the plain harmonic analysis (Fourier analysis) is shown in
the chart at upper right. The difference between these charts is that the winding factors are given for
one phase, normalized to the full-pitch winding, while the raw harmonics are given for a fixed current
flowing into one phase and out of the other two.
Brushless permanent-magnet machines Page 2.169

Working
harmonic
(4th)

Actual MMF harmonics

Winding factors
(MMF harmonics normalized to full-pitch winding)

Fig. 2.178 12-slot 8-pole motor with double-layer winding

In the 12/8 motor with 1@5 slots/pole, the lowest harmonic is also the working harmonic; but in the 12/10
motors with 1@2 slots/pole, the working harmonic or first “electrical” harmonic is the 5th “mechanical”
harmonic, while the 1st and 3rd mechanical harmonics are non-zero. From an electrical point of view
these appear as subharmonics and they tend to increase the differential leakage inductance. The single-
layer 12/10 motor has a very high fundamental winding factor, 0@9659, 11@5% higher than the 12/8.

Subharmonics

Actual MMF harmonics

0@9659

Winding factors
(MMF harmonics normalized to full-pitch winding)

Fig. 2.179 12-slot 10-pole motor with single-layer winding


Page 2.170 SPEED’s Electric Machines

Actual MMF harmonics

0@933

Winding factors
(MMF harmonics normalized to full-pitch winding)

Fig. 2.180 12-slot 10-pole motor with double-layer winding

Fig. 181 shows the concept of irregular slotting used to maximize the winding factor. It can be seen that
for the tooth at the centre of the figure, almost all the magnet flux passes through this tooth.
Consequently a coil wound around this tooth links 100% of the magnet flux, so that by definition its
winding factor is unity. Only the wide teeth are wound, giving a single-layer winding. The narrow
teeth serve merely to help in providing a flux-return path.
At first sight it is not obvious how the tooth-arcs and other proportions in Fig. 181 are determined, but
in fact there is a systematic procedure. Consider Fig. 182, in which the wide tooth-arc is mF and the
“auxiliary” tooth-arc is aF, F being the slot-pitch. The effective “collecting” arc of the wide tooth is
approximately equal to (1 + m a)F/2, and the idea is to make this equal to the pole-pitch.

Fig. 2.181 Flux-plot in motor with irregular slotting to increase the winding factor
Brushless permanent-magnet machines Page 2.171

Fig. 2.182 Irregular slotting

For example, in an 18-slot 22-pole machine, F = 360/18 = 20E while the pole-pitch is J = 360/22 = 16@364E.
Therefore we need (1 + m a) × 20/2 = 16@364, which implies that m a = 0@636. Only the difference
between m and a is controlled by this requirement. Another constraint is required to define them
separately, and this comes from the slot-opening. Suppose the slot-opening is yF; then in general we
have (m + a + 2y) = 1. In the example, if y = 0@1, we get m = 0@7182 and a = 0@0818. The resulting tooth-arcs
are 14@36E and 1@636E, making a ratio of 8@778.
The subject of irregular slotting was expounded in more general terms by Cros and Viarouge, [47]. Fig.
183 is redrawn from this interesting paper together with an outline of the steps involved in reducing
the number of coils and determining the basic relationships between the widths of wound and unwound
teeth. Cros and Viarouge extended the concept to three different tooth-arcs. They and others studied
the general patterns of slotting and showed that as the number of slots and poles increases, the
possibility for high winding factors also increases. Moreover the separation of the windings gives rise
to the possibility of “fault-tolerant” windings. However, there may be significant disadvantages
associated with machines of this type, including torque ripple and losses in the magnets, as discussed
elsewhere in the literature, [48,49].

Fig. 2.183 Systematic development of irregular slotting, by Cros and Viarouge [2002]
Page 2.172 SPEED’s Electric Machines

2.27 CAUSES OF ROTOR LOSSES IN PM BRUSHLESS MACHINES

In an ideal synchronous machine the field rotates in synchronism with the rotor, and the flux-density
is time-invariant throughout the rotor cross-section. There is no tendency for eddy-currents to flow
anywhere in the rotor. There are no losses in the magnets, the rotor body, or the shaft. Such ideal
conditions would exist at constant speed in a machine with smooth cylindrical surfaces (no slotting);
with sine-distributed windings; and with balanced polyphase sinusoidal currents.
Eddy-currents are induced in practice by imperfections or departures from the ideal synchronous
machine. The main ones are listed here with code letters that are explained in the following text.
h1 Space-harmonics in the stator ampere-conductor distribution and time-harmonics in the stator
current waveform produce asynchronous field components that rotate forwards or backwards
relative to the rotor. In general the space- and time-harmonics combine to produce a potentially
large number of asynchronous components. (See p. 201ff.)
For example in motors with squarewave drive, the stator ampere-conductor distribution
remains fixed in space for successive intervals, typically of 60E duration. As the rotor moves
relative to the fixed “armature reaction” field, EMFs and eddy-currents may be induced in it.
Again, commutation of the current from one phase to another is, in effect, a voltage step at the
stator terminals, inducing a “transformer” EMF in closed paths in the rotor. The resulting eddy-
currents are transient, but excited repetitively, resulting in a steady-state average loss.
h2 PWM-harmonic frequency components of the stator current produce asynchronous fields.
h3 Even when the phase currents are sinusoidal, any imbalance can produce a negative-sequence
component of the armature-reaction field, which rotates backwards and induces EMFs in the
rotor at twice the fundamental frequency. This is akin to the h1 and h2 losses.
m1 The overall permeance of the magnetic circuit may be modulated at the slot-passing frequency.
The “permeance harmonics” producing cogging torque, “slot ripple” in the EMF waveform, and
eddy-currents in the rotor.
m2 Stator slot-openings modulate the airgap flux-distribution by creating “dips” which rotate
backwards at synchronous speed relative to the rotor. They produce pools of motion-induced
eddy-currents that remain stationary in space, roughly opposite the slot openings.
All of the h-type losses can be analyzed by solving the electromagnetic field excited by time-harmonics
in the current waveform acting together with the space-harmonics in the winding distribution. If the
induced currents are assumed not to affect the stator current waveform, the field and circuit analyses
can be separated, simplifying matters greatly. This solution is called “current-forced”.
For surface-magnet machines this solution for h-type losses can be developed by solving the complex
diffusion equation in a multi-layer cylindrical structure, including the shaft and any retaining sleeve;
(Miller and Lawrenson, [66]). A complete simulation of the machine and drive is required to obtain the
time-harmonics of the current waveforms, and a harmonic analysis of the winding is required to obtain
the space harmonics of the ampere-conductor distribution. The method is also extended by means of
an equivalent harmonic current sheet to give a unified treatment of m1- and m2-type losses (Lawrenson
et al, [53]). Approximate post hoc modifications are applied to the basic 2-dimensional result, to deal
with finite length and/or the segmentation of magnets in the circumferential and axial directions.
Several original methods developed for PC-BDC are described.
For interior-magnet machines the solution of the complex diffusion equation is too difficult and so a
completely different approach is used, based on the calculated frequency-response of the complex
synchronous inductance in the d-axis. An important by-product of this analysis is the calculation of
the subtransient reactance and time-constant, which are applied to the calculation of a sudden short-
circuit fault. The frequency-response method is also developed for surface-magnet machines from the
complex diffusion equation. Several unique properties of faulted PM machines emerge from this work.
The old classical method of “flux-dip-sweeping” is included with several modifications and
improvements to calculate m2-type losses, and extended to estimate the m1-type losses.
Brushless permanent-magnet machines Page 2.173

Loss mechanisms in the magnets themselves


Eddy-current losses — Losses in the magnets are normally assumed to be caused by conventional
eddy-currents which are due to the variation of flux-density in the magnets. The process is exactly the
same as the generation of eddy-currents in general, and the basic theory is presented in several books
(for example, Lammeraner and Štafl [93]; Stoll [91]).
The engineering analysis of eddy-currents is mathematically delicate even in idealized 2-dimensional
models, and very few 3-dimensional effects can be fully analyzed by classical methods. 3-d effects
include the effects of “finite-length” and of shapes other than simple rectangular blocks or continuous
cylinders. Of course finite-element methods can be used in such cases, but they are slow and expensive.
Dimensional analysis has been used to help understand the behaviour of eddy-currents, and the
commonest example of this is to calculate the ratio between a key dimension h of the eddy-current
conductor and the so-called skin-depth,

2 (2.578)
* ' [m] .
TμF
6
For example, consider a permanent magnet with a conductivity of 0@6 × 10 S/m40 and relative recoil
permeability μrec = 1@05, operating in a field that alternates with a frequency 1 kHz, (which is typical of
PWM ripple in the current waveform, or the slot-passing frequency at normal speeds). Then

2
* ' ' 0@020 [m] (2.579)
2 B × 1000 × 0@6 × 106 × 1@05 × 4 B × 10 7

or 20 mm, or approximately 0@8". It is tempting to compare the skin depth with the length of the magnet
in the direction of magnetization, LM; but in many cases the magnet width and axial length are more
appropriate “key dimensions”; (see p. 238). If the ratio h/* is < 1, it is apt to say that the eddy-currents
are resistance-limited. This term is explained in the next section, but the essential properties are that
1. the eddy-currents have negligible effect on the field, and
2. the eddy-current losses decrease if the conductivity of the magnets decreases.
If, on the other hand, the ratio h/* is greater than 1, we are inclined to expect that the eddy-currents in
the magnets will be starting to be inductance limited. The essential properties are that
3. the eddy-currents produce a reaction field which tends to oppose the exciting field; and
4. the eddy-current losses increase if the conductivity of the magnets decreases;
The best way to limit eddy-current loss in magnets is to use magnets with zero or very low conductivity
(such as polymer-bonded or ferrite magnets), but this may not be possible if other properties take
precedence. This being so, it becomes important to understand the factors that cause the eddy-currents
so that other aspects of the design can be adjusted to minimize them.
Inductance-limited eddy-currents have the effect of screening: in other words, they tend to oppose the
penetration of flux, attenuating the field on the side of the conducting component that is remote from
the high-frequency source of excitation.
The “rule-of-thumb” test using h/* is an oversimplification. It was shown by Stoll and Hammond [55]
that the behaviour of eddy-currents depends on other dimensions and dimensionless ratios; for
example, the pole-pitch and the permeability of the eddy-current conductor. (See p. 195). In some
situations it is not sufficient to compare a single key dimension with *, but both widths of the magnet
measured in a plane normal to the direction of the flux (Lammeraner and Štafl, [93]).
It can be said that the determination of eddy-current loss requires a detailed calculation by the most
powerful methods available, and it should not be left to rule-of-thumb approximations.
40
This conductivity can also be expressed as approximately 1% of the conductivity of OFHC copper. NdFeB magnets have typical
conductivities in this range. Sm2Co17 magnets have higher conductivities typically around 2% OFHC.
Page 2.174 SPEED’s Electric Machines

Resistance-limited and inductance-limited eddy-currents

Fig. 2.184 Eddy-current loss vs. frequency

Resistance-limited eddy-currents are low-frequency eddy-currents that are limited by the resistivity
of the material. Although they may cause significant losses, they have little effect on the magnetic field
that is driving them. As indicated in Fig. 2.184, they are driven by a dB/dt that is unaffected by the
eddy-currents themselves, and the induced electric field E produces a current-density J that is
inversely proportional to the resistivity of the conducting material. At a specific frequency T, dB/dt
can be expressed as TB, where B is the peak value of a sinusoidal variation at that frequency.
Accordingly the losses produced by these eddy-currents can be characterized by a function of the form
2
(TB) /R, where R expresses the resistance of the eddy-current path.
In short, resistance-limited eddy-current losses are proportional to the square of the exciting frequency,
and they can be reduced by increasing the resistivity.
As the frequency increases, the eddy-currents increase in magnitude and their phase tends to shift such
that at very high frequency they form a flux-screen that prevents the original B-field from penetrating
into the conducting material. Only a “sufficient” current is required to perform this screening effect.
At the same time the resistance of the eddy-current path tends to increase with frequency because the
eddy-currents are concentrated into a skin depth of the form already stated in eqn. (2.184). If we take
the resistance of the eddy-current path to be inversely proportional to *, it appears that inductance-
2
limited eddy-current losses will have the form I R, where I is independent of frequency and R is
proportional to /T, the square-root of frequency.
This implies that reducing the resistivity will decrease the losses, but this is true only if the eddy-
currents are inductance-limited. Lower resistivity will tend to make them even more inductance-
limited and will tend to shift the behaviour towards that of a perfectly conducting AC flux screen.
It can be said that the preferred means of limiting eddy-current losses is to use a high resistivity to keep
the eddy-currents resistance-limited, except that in components used as flux-screens the resistivity
should be minimized.
Further discussion of the physical behaviour of eddy-currents — in relation to geometry and frequency
— can be found in Stoll [91] and Davies [95].
Voltage-forced and current-forced solutions — With the exception of the fault calculations, all the
loss analysis is based on a current-forced solution, in which it is assumed that the induced currents in
the rotor have no effect on the exciting stator current waveform. This assumption makes it possible
to calculate the exciting current waveform independently, a great simplification. It is justified as long
as the induced-current circuits are only weakly coupled to the stator circuits, and/or as long as the
induced currents are essentially resistance-limited.
Brushless permanent-magnet machines Page 2.175

Fig. 2.185 Magnet hysteresis loss

Hysteresis loss in magnets — Hysteresis losses are well known in electrical steels, but rarely
mentioned in connection with permanent magnets. Yet Fukuma et al [79] published data in which they
recorded hysteresis losses “twice as great” as the eddy-current loss in a permanent-magnet.
Fukuma measured hysteresis losses of approximately 1@3 W/kg at 50 Hz with a field variation of 0@1 T
3
in a fully magnetized sample of NEOMAX-44H.41 The density of his magnet was 7500 kg/m , so this
corresponds to a minor loop area (Fig. 184) of
D 7500
a ' Wh × ' 1@3 × ' 195 J/m 3 . (2.580)
f 50

With a remanence of 1@26 T and relative recoil permeability of 1@05, the area A under the
demagnetization curve is
2
1 Br 1 1@262
A ' ' ' 601@6 × 103 J/m 3 . (2.581)
2 μrec μ0 2 1@05 μ0

The ratio x = a/A gives an idea of the order of this effect:


a 195
' ' 0@00032 . (2.582)
A 601@6 × 103
In other words, a hysteresis loop of only 0@03% of the area under the demagnetization curve is enough
to produce hysteresis losses of 1@3 W/kg at 50 Hz. In general we can write

Br2 f
W ' x W/kg . (2.583)
2 μrec μ0 D

Fukuma surmised that the hysteresis loss was proportional to the square of the variation in flux-
density. In his measurement of 1@3 W/kg at 50 Hz, the flux-density variation was 0@1T throughout the
magnet. In his test motor, such a large variation occurs only near the surface of the magnets and not
through their entire volume. Nevertheless, the figures are such as to indicate the need for checking the
possibility of hysteresis loss.

41
Fukuma gives the conductivity as 6@9 × 105 S/m, that is, 1@2% of the conductivity of OFHC copper.
Page 2.176 SPEED’s Electric Machines

2.28 STRUCTURE OF ROTOR LOSS CALCULATIONS IN PC-BDC


Surface-magnet machines — The analysis begins with a purely 2-dimensional system along well
established lines.42 This includes exterior- and interior-rotor motors in which the magnet is a
continuous cylinder. Interior-rotor machines may have a conductive retaining sleeve on the rotor.
The eddy-currents are excited by space- and time-harmonics in the stator ampere-conductor
distribution; or by negative-sequence currents flowing under unbalanced conditions. The excitation
is represented by a current-sheet having an arbitrary number of pole-pairs and an arbitrary frequency
of rotation. Losses due to “permeance harmonics” caused by slotting are incorporated in the general
analysis using the method of Lawrenson et al [53], in which the modulation of the airgap field is
represented by an equivalent harmonic current-sheet whose fundamental wavelength is equal to the
slot-pitch; this current-sheet rotates backwards relative to the rotor at twice synchronous speed.
Solutions are given in both cylindrical and cartesian coordinate systems, for comparison and checking.
In cases where curvature is minimal, the two systems should produce comparable results. The
solutions are also checked against independently-derived Laplacian solutions for zero frequency,
(Hughes and Miller [13]). Finite-element calculations are also used for this purpose, using PC-FEA.
The main machine geometries included in this study are shown in Figs. 187 ? on pages 178 ?.
The laminated steel components are assumed to have infinite permeability, as is normal in this type
of analysis. This makes it possible to represent such regions (particularly the stator core) as a surface
at which the tangential field strength is defined by a current-sheet representing the stator ampere-
conductors in their entirety. Were this not so, the solution would require more regions and the
algebraic complexity would increase with little improvement in the results.
The conventional 2-dimensional solution is presented in full detail for two models : (i) an exterior-rotor
motor with 2 regions; and (ii) an interior-rotor motor with 4 regions. However, more complex models
are programmed into PC-BDC, including a 3-region exterior-rotor model with a conducting rotor
shell, and a 6-region interior-rotor machine with an airgap winding of finite radial thickness.
These are based on the same principles as the simpler models, but the algebra is more complicated.
Methods are then introduced to deal with the effects of finite axial length and segmentation of the
magnets in both the circumferential and axial directions.
The screening effect of a conducting rotor sleeve is a natural by-product of the general analysis.
The old method of flux-dip-sweeping published by Robinson et al [70] and by Russell and Norsworthy
[69] is presented as an alternative to Lawrenson's method [53]. Originally developed to calculate losses
in a cylindrical can affixed to the stator bore or the rotor surface, this method is constrained by the
assumption that the eddy-currents are resistance–limited. It is also limited in application to very thin
cans, and is therefore of little use for calculating magnet losses. Nevertheless, the method is here
improved as far as possible by introducing an improved method of calculating the flux dips (Zhu et al,
63]). The method is worth preserving for the calculation of losses in rotor and stator cans.
Interior-magnet machines — The solution of the complex diffusion equation is too difficult for these
machines, which are generally salient-pole machines with complex geometric lamination shapes. For
these machines the eddy-current losses in the magnets are analyzed through the frequency-dependent
synchronous inductance in the d-axis.
Subtransient reactance, time-constant, and fault analysis — Calculation of subtransient reactance
and time-constant follow naturally from the loss analysis, particularly from the frequency-response
model. These calculations are developed for both surface-magnet and interior-magnet machines, and
applied to the calculation of a sudden short-circuit fault. This reveals the unique behaviour of the
faulted PM machine, with important implications for estimating the risk of demagnetization. The
frequency-response model can be can be developed in d- and q-axes and transformed into a circuit model
for system analysis including the effects of induced currents in the rotor. Furthermore, transient
effects can be studied using the Fourier Transform; see Miller [67,68], Miller and Lawrenson, [66]).

42
Related recent examples include (Zhu, [63]), and Deng [73-75]; but much earlier examples were published (for example, Miller
[68], Miller and Lawrenson [66]). Similar analyses in cartesian coordinates include Stoll [55], Shah and Lee [64].
Brushless permanent-magnet machines Page 2.177

NO YES
Salient-pole? No rotor can
SPM IPM

Calc. subtransient
parameters (SPM) Calc. subtransient
[Frequency scan] parameters (IPM)

CDE Ldjw
Short-circuit
Td0' ' , Td' ', L d' ', Ld (jT) calculation Td0' ' , Td' ', L d' ', Ld (jT)

OPTIONAL
HH Magnet flux
Flux-dip sweeping
pulsation
1 Rotor Can
2 Magnet
WRCanFDS FDS
WMag_FDS
WMag_PHx
WRCanPHx

Calc SlotMod Loss SPM Flux harmonics


Calc SlotMod Loss IPM
[equivalent current sheet] Mm(n)
WMag_PHx Ldjw
WMag_SOR CDE
WRCanSOR
WShftSOR

Calc IWHx Loss SPM CalcACHx Calc IWHx Loss IPM


Current time-harmonics
WMag_IWH CDE I ph(n) WMag_IWH Ldjw
WRCanIWH
WShftIWH
PC-BDC circuit &
control simulation
Static/Dynamic design RotorLossFlowChart.wpg

Fig. 2.186 Flowchart of rotor loss calculations

Fig. 186 shows the flowchart of rotor loss calculations in PC-BDC.43 The main methods are indicated :
CDE means the complex diffusion equation; Ldjw means the frequency-dependent synchronous
inductance, and FDS means the flux-dip-sweeping method.
A limitation is that the methods are applied independently, which means that any interaction of
harmonics from difference sources is not taken into account. However, the methods can be switched
on or off, and the number of space- and time-harmonics can be selected or even evaluated in isolation
from all the others. It is therefore possible to assess the relative importance of the different loss-
producing phenomena.

43
Stator can losses are also performed independently by the flux-dip-sweeping method but are not shown on the chart.
Page 2.178 SPEED’s Electric Machines

rs
r2
F
r1

ÎÏ
Ð
Ñ rH

SLEEVE SHAFT K

ROTOR MAGNET Q

GAP SLOT

STATOR

MagLoss_01.wpg

Fig. 2.187 4-region model of 9-slot 6-pole machine showing main dimensions and regions for
analysis. The magnet sits directly on the shaft, and both the magnet and the shaft may
be conductive. The solution domain lies between rH and rS.

WINDING rS
rW
GAP

r2
F
r1
0 Î
Ï Ð
Ñ rB J
Ò
SLEEVE SHAFT
K
rH HUB
C
Q
ROTOR MAGNET

STATOR

MagLoss_02.wpg

Fig. 2.188 6-region model with airgap winding and additional nonconducting rotor body region
4. The solution domain lies between 0 and rS. The winding has a finite radial depth.
The magnet, sleeve, and shaft may all be conductive. Current-sheets K and Q are
provided for checking and additional experimental analysis.
Brushless permanent-magnet machines Page 2.179

μ = 4 in rB
2-region model
r1

rS
SLOT

Ð Ï
Î STATOR
K

GAP
rH
in 3-region
MAGNET
model only

MagLoss_03.wpg
ROTOR

Fig. 2.189 2- or 3-region model of 9-slot 6-pole exterior-rotor motor showing the main dimensions and regions
for analysis. The solution domain lies between rS and rB in the 2-region model; or between rS and
rH in the 3-region model when the rotor shell (3) is conducting.

IR4 IR4C IR6 ER2 ER3

STATOR STATOR STATOR SHELL r


H SHELL Ð
r y r 0
r r
S S S B B
Î WINDING Ï Ï
AIRGAP AIRGAP r
W AIRGAP Î MAGNET MAGNET
r y r
2 2 2
SLEEVE Ï SLEEVE Ï SLEEVE Ï
r y r r r
1 1 1 1 1
Ð Ð Ð Î Î
MAGNET MAGNET MAGNET AIRGAP AIRGAP
r y r r r
H H B Ñ S S
Ñ Ñ
ROTOR BODY STATOR STATOR
r
H
ROTOR ROTOR Ò
BODY/SHAFT BODY/SHAFT SHAFT

0 0 0 0 0
R_PV.wpg

Fig. 2.190 Cross-section of 4-pole interior PM motor (IPM), showing the main dimensions.
Page 2.180 SPEED’s Electric Machines

2.29 SOLUTION OF THE COMPLEX DIFFUSION EQUATION


Starting from the vector potential equation
curl A ' B (2.584)
and taking the curl of both sides, we get
curl curl A ' curl B . (2.585)
Now if J is current-density we have
curl H ' J (2.586)

and in linear materials B = μH, so curl B = μ curl H = μJ. Hence

curl curl A ' μ J . (2.587)

The current can be forced from an external circuit, or it can be induced in closed loops as eddy-current.
In the first case no change is needed in the curl curl A equation, but for eddy-currents we write
MA
J ' FE ' F (2.588)
Mt
Hence
MA
curl curl A ' μF . (2.589)
Mt

In cartesian coordinates we have

curl curl A ' grad div A grad div A ' L 2A (2.590)

provided that div A = 0, which is normally assumed in such analysis. Hence


MA
L 2A ' μ F . (2.591)
Mt

In cylindrical coordinates curl curl A is more complicated, but if it is assumed that A has only a z-
component A, the equation simplifies to the so-called two-dimensional diffusion equation:

M 2A 1 MA 1 M 2A MA
' μF . (2.592)
Mr 2 r Mr 2
r M2 2 Mt
For regions carrying forced current-density J, the right-hand side is replaced by μJ.
A well known solution (by separation of variables) takes the form:

A (r,2,t) ' A (r) cos p 2 e j T t (2.593)


for stationary pulsating fields; or

A (r,2,t) ' A (r) e j (T t p 2)


(2.594)

for rotating fields. In both cases A is complex and is written in boldface to indicate that it is essentially
a phasor quantity, giving44

d 2A 1 dA p2
jTμF A ' 0. (2.595)
dr 2 r dr r2

44
In some technical papers and books the coefficient of A in eqn. (2.595) appears as (p2/r2 jTμF), owing to the use of e jTt instead
of e+jTt. The forward progression of time is preferred, even though the backward solution is mathematically valid (but rarely
mentioned; see, for example G.W. Whitrow, The Natural Philosophy of Time, Nelson, 1961).
Brushless permanent-magnet machines Page 2.181

This is the modified Bessel equation, with solution [McLachlan 85]

1
A ' c1 Ip ( j1/2 r/d ) c2 Kp ( j1/2 r/d ) with d ' . (2.596)
TμF

Ip and Kp are modified Bessel functions of the first and second kind respectively, while c1 and c2 are
complex constants. The solution can also be written in terms of the Kelvin functions with real
arguments defined by the identities on p. 184. Note that the characteristic length d is equal to 1//2
times the conventional “skin depth”.
In non-conducting regions F = 0 and eqn. (2.595) degenerates to the Laplace equation

d 2A 1 dA p2
A ' 0. (2.597)
dr 2 r dr r2
with solution (Binns, [90])

A ' c3 r p c4 r p
. (2.598)

In regions carrying forced current the equation is Poisson's equation

d 2A 1 dA p2
A ' μJ. (2.599)
dr 2 r dr r2
The solution is extended with an additional term in μJ : if p = 2,

μJ 2
A ' c3 r p c4 r p
r ln r ; (2.600)
4
otherwise
μJ
A ' c3 r p c4 r p
2
r 2. (2.601)
p 4
While the basic solution is expressed in terms of vector potential A, we also need expressions for the
other field vectors B, H, E and J, which are needed to calculate practical quantities. Also, in a multiple-
region problem, the boundary conditions between regions are expressed in terms of B and H. Thus
1 MA MA
B ' curl A or Br ' ; B2 ' . (2.602)
r M2 Mr

where the overline indicates a true vector and boldface continues to indicate a complex phasor
component. Finally

MA
E ' ; or E ' jTA. (2.603)
Mt
In conducting regions the eddy-current density is
J ' FE ' j T FA. (2.604)
2 *
The power loss in conducting regions can be determined by integrating J /F or EJ over the region
volume. For a continuous conducting cylinder with radii r1 and r2, this can be shown to be
r2
2B 2
W ' T F AA(r dr W/mz (2.605)
2 r1
*
Alternatively the radial component of Poynting's vector E × H can be used to give the difference
between the average power losses over the inner and outer surfaces of the cylinder,

1 ( r2
W ' Re j T AH2 W/m 2 .
2 r1

The sign of W indicates whether the power flow is radially inwards or outwards.
Page 2.182 SPEED’s Electric Machines

Exterior-rotor machine; 2-region model


The analysis is performed first for the exterior-rotor machine with a non-conducting rotor shell, as
shown in Fig. 189 on p. 179. Of all the cases involving eddy-currents in magnets, this is the simplest,
because it has only two regions. (In PC-BDC this model is extended to 3 regions with a conducting rotor
shell).
In region 1 (the airgap),

A ' c1r p c2 r p ;
p
B r ' j [ c1r p c2 r p ] ;
r (2.607)
p
H2 ' [ c r p 1 c2 r p 1 ] .
μ0 1

The magnet may be conducting or non-conducting. If it is conducting, then in region 3 we have

A ' c3 I p ( z ) c4 K p ( z ) ;
p
B r ' j c3 I p ( z ) c4 K p ( z ) ;
r (2.608)
j 1/2
H2 ' c I N(z) c4 KpN ( z ) .
μM dM 3 p
where

r 1 *M
z ' j1 / 2 and dM ' ' . (2.609)
dM T μM FM 2

*M is recognized as the skin depth in the magnet material at the radian frequency T. μM is the magnet
permeability in H/m and FM is the magnet conductivity in S/m. The functions Ip and Kp are modified
Bessel functions of the first and second kinds respectively. When they are differentiated with respect
1/2
to r, as is necessary to determine H2 = (1/μ) MA/Mr, the additional factor j /dM emerges.
When the magnet is non-conducting we have the simpler Laplacian solution in region 2 :

A ' c3r p c4 r p ;
p
B r ' j [ c3r p c4 r p ] ;
r (2.610)
p
H2 ' [ c r p 1 c4 r p 1 ] .
μ0 3
The complex coefficients c1, c2, c3 and c4 must be found by solving four simultaneous algebraic
equations obtained from the boundary conditions. These will be written for the case of a conducting
magnet, but the results are formally applicable when the magnet is non-conducting, as will be seen.
At r = rs we have a boundary with an infinitely permeable material, on which the current sheet K gives
rise to a discontinuity in the tangential magnetic field H2. However, H2 = 0 inside the stator, since its
permeability is deemed to be infinite. Therefore the boundary condition is

p
[c r p 1
c2 r s p 1
] ' K. (2.611)
μ0 1 s

Across the boundary at r = r1, H2 and Br are both continuous. Thus if z1M = r1/dM we have
p j 1/2
[ c1 r1p 1 c2 r1 p 1 ] ' c I N ( z ) c4 KpN ( z1M ) ; and
μ0 μM dM 3 p 1M
jp jp
[c r p c2 r1 p ] ' c I (z ) c4 Kp ( z1M ) .
r1 1 1 r1 3 p 1M
Brushless permanent-magnet machines Page 2.183

Finally at r = rB we have an infinitely permeable boundary at which H2 = 0 :

j 1/2
c I N( z ) c4 KpN( z BM ) ' 0,
μM dM 3 p BM
where zBM = rB/dM. Now let

a1 ' p r sp 1 ; a2 ' p rs p 1
; a3 ' p r1p 1
a4 ' p r1 p 1
;

a7 ' r1p ; a8 ' r1 p ;


and let

j 1 / 2 μ0 j 1 / 2 μ0
a5 ' I N(z ); a6 ' K N(z );
dM μM p 1M dM μM p 1M

a9 ' Ip ( z1M ) ; a10 ' Kp ( z1M ) ;

j 1/2 j 1/2
a11 ' I N(z ); a12 ' K N(z ).
dM p BM dM p BM

When the magnet is non-conducting, we have instead


μ0 μ0
a5 ' p r1p 1
; a6 ' p r1 p 1
; a9 ' r1p a10 ' r1 p ;
μM μM

a11 ' p rBp 1 ; a12 ' p rB p 1


.

The boundary conditions now become


a1 c1 a2 c2 ' μ0 K
a3 c1 a4 c2 a5 c3 a6 c4 ' 0
a7 c1 a8 c2 a9 c3 a10 c4 ' 0
a11 c3 a12 c4 ' 0 .
The coefficients a being known, these complex boundary-condition equations could be solved for the
unknown coefficients c by computer matrix methods such as Gaussian elimination; (see, for example,
Shah and Lee [64]). Here, however, we will solve them by simple algebra. From the last equation,

a11
c4 ' c3
a12
so the second and third equations become

a11
a3c1 a4c2 a5 a6 c3 ' 0 ;
a12

a11
a7c1 a8c2 a9 a10 c3 ' 0 .
a12
Let

a11 a11
b1 ' a5 a6 ; b2 ' a9 a10 .
a12 a12
Then

a7 a8
c3 ' c1 c2
b2 b2
Page 2.184 SPEED’s Electric Machines

with which

a7 a8
a3 b1 c1 a4 b1 c2 ' 0 .
b2 b2
Now let

a7 a8
b3 ' a3 b1 ; b4 ' a4 b1 .
b2 b2
Then
b3c1 b4c2 ' 0 .
This can be written
b3
c2 ' c1 .
b4
Hence the first boundary condition can be written

b3
a1 a2 c1 ' μ0 K ,
b4
from which

μ0 K
c1 ' .
b3
a1 a2
b4

We can now work back through the formulas to calculate c2, c3 and c4, and this completes the solution.

Bessel functions with very small arguments


1/2
The modified Bessel functions Ip and Kp have complex arguments of the form z =j r/d. The magnitude
z = |z| may range from 0 (at zero frequency) to a high number (say, 1000) in the case of inductance-
1/2 jB/4
limited eddy-currents, while the argument arg z is always B/4, since j = e . The order of the Bessel
functions is equal to the number of harmonic pole-pairs, p, and is therefore real and positive. In the
mid-range (say, 1 < z < 100) Ip and Kp can be calculated using the algorithm of Amos [82] or Maver [83].45
With extreme small values of z it may be convenient to use asymptotic expansions such as those given
by Abramowitz and Stegun [84]:

(z/2) p (p 1)! p
Ip (z) 6 ; Kp (z) 6 (z/2)
p! 2
Furthermore, the derivatives IpN and KpN are required: in the normal range these are obtained using
“recurrence”-type formulas given by McLachlan [85]:
2 IpN( z ) ' Ip 1( z ) Ip 1( z ) ; 2 KpN( z ) ' Kp 1( z ) Kp 1( z )

These work with the asymptotic expansions; alternatively the expansions can be differentiated directly.
The problem of large arguments is discussed briefly on p. 191.

45
The solution could also be developed using Kelvin functions, which permit the solutions to be calculated using real
functions with real arguments as follows; with z real,

j p Ip (j1 /2z) ' berp z j beip z ' Jp (j3/ 2z)


p
j Kp (j1 /2z) ' kerp z j keip z
3/2
Jp(z), Yp(z) could also be used as a "fundamental system of solutions" (McLachlan [85]), but in this case z = j z and
1/2
this could be said to be ever so slightly less natural than the j z used with the modified Bessel functions; moreover,
p
while Ip(z) = j Jp(jz), Kp(z) is not so simply related to Yp(z).
Brushless permanent-magnet machines Page 2.185

Checking the solution under zero-frequency conditions

r1
rS
35
34 Y

MAGNET rB
ÏÎ 42
STATOR K
GAP Y

Peak amp-conductors/m
X = 8756

B r at Y = 0@022 T
B 2 at X = 0@004 T

CheckBesselExtRotor.wpg ROTOR
X
Fig. 2.192 Finite-element flux-plot for checking
Fig. 2.191 Geometry for checking the solution; the solution at zero frequency
exterior-rotor motor

The solution for T = 0 can be checked by referring to the complete solution (Hughes and Miller [13]) for
the Laplacian field in a series of concentric cylinders with an exciting current sheet at the radius r0 :
2 Tph kw1
K ' i sin p 2 A/m (2.630)
B r0
Tph is the number of turns in series per phase, kw1 is the fundamental winding factor, and i is the
instantaneous phase current. Only one phase is active. Eqn. (630) is derived on p. 186. In the geometry
of Fig. 191, a 4-pole exterior-rotor motor is represented by two regions with infinitely permeable
boundaries at rS and rB, as in Fig. 189. The exciting current sheet is located on the stator surface; i.e.,
r0 = rS. Then at any radius r in the magnet region 2, we have simplified expressions for Br and B2 :
p 1
μ0 K rS 2 1 (r / rB)2p
Br ' × cos p 2 ;
2 r 1 (rS / rB) 2p
p 1
(2.631)
μ0 K rS 2 1 (r / rB)2p
B2 ' × sin p 2 .
2 r 1 (rS / rB) 2p

If Tph = 36, kw1 = 0@866, i = 15 A, rS = 34 mm, r1 = 35 mm, rB = 42 mm, r = 38@5 mm and p = 2, the result is
K = 8756 A/m ; Br = 0@02265 cos p2 T (peak value at X) ; and B2 = 0@00390 sin p2 T (peak value at Y).
Fig. 192 shows a finite-element calculation of the same field, with spot values of Br and B2 which
corroborate the analytical calculation. Interestingly, the analytical solution is more precise than the
finite-element one.
These solutions are used to verify that the complete solution developed earlier is correct for the
exterior-rotor case. That solution contains a true zero-frequency solution which does not use Bessel
functions and corresponds exactly to the Hughes/Miller analysis, [13]. It is also verified that the Bessel-
function solution converges towards the true zero-frequency solution when the frequency is reduced
towards zero. Of course, this tends to make the skin-depth in the magnet region much larger than any
of the magnet dimensions.
Page 2.186 SPEED’s Electric Machines

B
2p

C sin p 2
conductors/radian
2F
r
0

p = pole-pairs

Total conductors
Total amp-conductors
2C
2S p = 4 k w1 Tph
p i
B p

Fig. 2.193 Sinusoidal current sheet; 4-pole

Sinusoidal current sheet in terms of winding factor and turns per phase
Fig. 193 shows a sinusoidally-distributed sheet of conductors with four poles, represented by
C sin p 2 conductors radian (2.632)
where p = 2. The total number of sinusoidally-distributed turns per pole is obtained by counting the
conductors in one half-pole — in this case, from 0 to 45E or B/2p : thus
B/2p
C
SP ' C sin p 2 ' .
0 p
If C is multiplied by the instantaneous phase current i, we have the ampere-conductor distribution. If
we also divide by r0 we have K sin p2 = (Ci/r0) sin p2 ampere-conductors per metre measured around
the stator bore. Also the total number of ampere-turns per pole is SPi.
The MMF distribution F(2) is obtained as for any other winding by integrating the ampere-conductors
enclosed by a contour such as the one shown dotted in Fig. 193. In this case we will have
F (2) ' FP cos p 2 , (2.634)

Fig. 2.194 MMF distribution of full-pitch winding, with fundamental


Brushless permanent-magnet machines Page 2.187

where

Ci K r0
FP ' ' ' SP i . (2.635)
p p
Next consider a full-pitch winding with Tph turns per phase, all in series. The MMF wave is rectangular
with a peak value Tphi/2p, and the fundamental component is

4 Tphi
F1 ' . (2.636)
B 2p
For a general winding the peak of the fundamental MMF wave is given by

4 kw1Tphi
F1 ' , (2.637)
B 2p
where kw1 is the fundamental harmonic winding factor. Equating F1 = FP in eqns. (635) and (637), we get
2 kw1 Tph i
K ' . (2.638)
B r0
The total effective number of sine-distributed ampere-conductors in one band of angular extent equal
to one pole-pitch is indicated in Fig. 193 and is (4/B) × kw1Tphi/p = 2SPi = 2Kr0/p.

Fig. 2.195 2-pole example

Fig. 195 shows a 2-pole example of an isolated winding in air with an effective sinusoidal distribution
of ampere-conductors having (4/B) × kw1Tphi ampere-conductors per pole. The H-field inside the
winding is uniform and half the MMF is expended inside (and half outside) the winding, so that

4
k T i
B w1 ph 1 K
B ' μ0 H ' μ0 × × ' μ0 ,
2 r0 2 2

a known result, [13].


Page 2.188 SPEED’s Electric Machines

Interior-Rotor Machine; Slotted stator, 4-region model


The next analysis is performed for the interior-rotor machine of Fig. 187 on p. 178. The method is
essentially the same as for the exterior-rotor machine studied earlier. However, it is spelt out in detail
for the sake of rigorous completeness. This model has four regions : the airgap, the rotor sleeve, the
magnet, and the shaft or hub. (In PC-BDC it is extended to 6 regions as shown in Fig. 188 on p. 178).
In region 1 (the airgap),

A ' c1r p c2 r p ;
p
B r ' j [ c1r p c2 r p ] ;
r (2.640)
p
H2 ' [ c r p 1 c2 r p 1 ] .
μ0 1

The rotor sleeve may be conducting or non-conducting. If it is conducting, then in region 2 we have

A ' c3 I p ( z ) c4 K p ( z ) ;
p
B r ' j c3 I p ( z ) c4 K p ( z ) ;
r (2.641)
1/2
j
H2 ' c I N(z) c4 KpN ( z ) .
μS dS 3 p
where
r 1 *S
z ' j1 / 2 and dS ' ' . (2.642)
dS T μS FS 2

*S is recognized as the skin depth in the sleeve at the radian frequency T. μS is the sleeve permeability
in H/m and FS is the conductivity in S/m.
When the sleeve is non-conducting we have the simpler Laplacian solution in region 2 :

A ' c3r p c4 r p ;
p
B r ' j [ c3r p c4 r p ] ;
r (2.643)
p
H2 ' [ c r p 1 c4 r p 1 ] .
μS 3

In effect, the sleeve becomes an extension of the airgap with c1 = c3 and c2 = c4. These solution
equations for the sleeve region are identical to those for the magnet in the next region.
The magnet may be conducting or non-conducting. If it is conducting, then in region 3 we have

A ' c5 I p ( z ) c6 K p ( z ) ;
p
B r ' j c5 I p ( z ) c6 K p ( z ) ;
r (2.644)
j 1/2
H2 ' c I N(z) c6 KpN ( z ) .
μM dM 5 p
where
r 1 *M
z ' j1 / 2 and dM ' ' . (2.645)
dM T μM FM 2

*M is again recognized as the skin depth in the magnet at the radian frequency T. μM is the magnet
permeability in H/m and FM is the conductivity in S/m.
Brushless permanent-magnet machines Page 2.189

When the magnet is non-conducting we have the simpler Laplacian solution in region 3 :

A ' c5r p c6 r p
;
p
B r ' j [ c5r p c6 r p
];
r (2.646)
p
H2 ' [c r p 1
c6 r p 1
].
μM 5

If the sleeve and the magnet are both non-conducting, and if their relative permeabilities are both 1,
then both regions become, in effect, extensions of the airgap with c1 = c3 = c5 and c2 = c4 = c6.
Finally if the shaft is conducting we have

A ' c7 I p ( z ) ;
p
B r ' j c7 I p ( z ) ;
r (2.647)
1/2
j
H2 ' c I N(z).
μH dH 7 p
where
r 1 *H
z ' j1 / 2 and dH ' ' . (2.648)
dH T μH FH 2

*H is recognized as the skin depth in the shaft at the radian frequency T. μH is the shaft permeability
in H/m and FH is the conductivity in S/m. There is no term in Kp (z) because the solution must be finite
at r = 0, and Kp increases without limit as r 6 0.
When the shaft is non-conducting we have the simpler Laplacian solution in region 4 :

A ' c7 r p ;
p
B r ' j c7 r p ;
r (2.649)
p
H2 ' c r p 1.
μH 7
p
There is no term in r because the solution must be finite at r = 0.
Solution — The complex coefficients c1, c2, ... c7 must be found by solving seven simultaneous algebraic
equations obtained from the boundary conditions. These will be written for the case where the magnet,
the sleeve, and the shaft are all conducting, but the results are formally applicable when any of these
regions is non-conducting, as will be seen.
At r = rS we have a boundary with an infinitely permeable material, on which the current sheet K gives
rise to a discontinuity in the tangential magnetic field H2. However, H2 = 0 inside the stator core, since
its permeability is deemed to be infinite. Therefore the boundary condition is

p
[c r p 1
c2 r S p 1
] ' K. (2.650)
μ0 1 S

Across the airgap/sleeve boundary at r = r2, H2 and Br are normally continuous. However, we will
introduce a second current sheet Q at r2 for diagnostic purposes to be discussed later. Thus

p j 1/2
[c r p 1
c2 r2 p 1
]' c I N(z ) c4 KpN ( z2S ) Q; and
μ0 1 2 μS dS 3 p 2S
jp jp
[c r p c2 r1 p ] ' c I (z ) c4 Kp ( z2S ) .
r2 1 2 r2 3 p 2S
Page 2.190 SPEED’s Electric Machines
1/2
where z2S = j r2/dS. A similar boundary condition is found between the sleeve and the magnet, but
without the hypothetical current sheet Q. Thus at r = r1 we have

j 1/2 j 1/2
c I N(z ) c4 KpN ( z1S ) ' c I N(z ) c6 KpN ( z1M ) ; and
μS dS 3 p 1S μM dM 5 p 1M
jp jp
c I (z ) c4 Kp ( z1S ) ' c I (z ) c6 Kp ( z1M ) .
r1 3 p 1S r1 5 p 1M
1/2 1/2
where z1S = j r1/dS and z1M = j r1/dM.
Finally at the surface of the shaft or hub, r = rH, we have
j 1/2 j 1/2
c I N(z ) c6 KpN ( zHM ) ' c I N(z ); and
μM dM 5 p HM μH dH 7 p HH
jp jp
c I (z ) c6 Kp ( zHM ) ' c I (z ).
rH 5 p HM rH 7 p HH
1/2 1/2
where zHM = j rH/dM and zHH = j rH/dH.
The boundary conditions can now be written
a1 c1 a2 c2 ' μ0 K
a3 c1 a4 c2 a5 c3 a6 c4 ' μ0 Q
a7 c1 a8 c2 a9 c3 a10 c4 ' 0
a11 c3 a12 c4 a13 c5 a14 c6 ' 0
a15 c3 a16 c4 a17 c5 a18 c6 ' 0
a19 c5 a20 c6 a21 c7 ' 0
a22 c5 a23 c6 a24 c7 ' 0,

where
a1 ' p rsp 1 ; a2 ' ! p rs p 1
; a3 ' p r2p 1
a4 ' p r2 p 1
;

a7 ' r2p ; a8 ' r2 p ;

and the remaining a-coefficients take on different forms depending on whether the corresponding
regions are conducting or non-conducting. Thus if the sleeve is conducting we have

j 1 / 2 μ0 j 1 / 2 μ0
a5 ' I N(z ); a6 ' K N(z ); a9 ' Ip ( z2S ) ; a10 ' Kp ( z2S )
dS μS p 2S dS μS p 2S

and if it is non-conducting we have

μ0 μ0
a5 ' p r2p 1 ; a6 ' p r2 p 1
; a9 ' r2p ; a10 ' r2 p .
μS μS

Similarly if the magnet is conducting we have


j 1/2 j 1/2
a13 ' I N(z ); a14 ' K N(z ); a17 ' Ip ( z1M ) ; a18 ' Kp ( z1M ) ;
dM p 1M dM p 1M
j 1/2 j 1/2
a19 ' I N(z ); a20 ' K N(z ), a22 ' Ip ( zHM ) ; a23 ' Kp ( zHM ) ,
dM p HM dM p HM

and if it is non-conducting we have

a13 ' p r1p 1 ; a14 ' p r1 p 1


; a17 ' r1p ; a18 ' r1 p ;
a19 ' p rHp 1 ; a20 ' p rH p 1
a22 ' rHp ; a23 ' rH p .
Brushless permanent-magnet machines Page 2.191

Finally if the shaft is conducting we have

j 1 / 2 μM
a21 ' I N(z ); a24 ' Ip ( zHH ) ,
dH μ p HH
H

and if it is non-conducting

μM
a21 ' p r Hp 1 ; a24 ' r Hp .
μH

The solution for the coefficients c1, c2, ... c7 is obtained by plain algebra as before, and the result is
expressed with the aid of auxiliary or composite coefficients b1, b2, ... b12 as follows :
b1 ' a19 a21 a22 / a24 ; b2 ' a20 a21 a23 / a24 ; b3 ' b1 / b2 ;
b4 ' a13 a14 b3 ; b5 ' a17 a18 b3 ; b6 ' a11 b4 a15 / b5 ; b7 ' a12 b4 a16 / b5 ;
b8 ' a9 b6 a10 / b7 ; b9 ' a5 b6 a6 / b7 ; b10 ' a3 b9 a7 / b8 ; b11 ' a4 b9 a8 / b8 ;
b12 ' a1 b10 a1 / b11 .

Then
μ0 K a2 μ0 Q / b11 b10 μ0 Q a7 a8 b6
c1 ' ; c2 ' c1 ; c3 ' c1 c2 ; c4 ' c3
b12 b11 b11 b8 b8 b7
a15 a16 a22 a23
c5 ' c3 c4 ; c6 ' b3 c5 ; c7 ' c5 c6 .
b5 b5 a24 a24

Bessel functions with large arguments


When the skin-depth is very small in any region, the arguments of the Bessel functions become large,
and numerical problems may arise in calculation. This problem is most likely to occur in the shaft
region because of the combination of high permeability and high conductivity. Ironically the shaft
region is often the least important in terms of losses, and it may be advisable simply to avoid the
problem by setting the shaft conductivity to zero. A more rigorous approach is to set c7 = 0, which
“abandons” the detailed loss data for the shaft while preserving the correct solution outside,
2p
represented with c2 = c1rH . Under these conditions the shaft is behaving as a perfectly impermeable
cylinder, as though its permeability were zero at the particular frequency. (The behaviour of cylinders
in rotating AC fields is discussed by Perry [96] in terms of magnetic Reynolds number. See also Hughes
and Miller [13]; and p. 195).

Checking the solution under zero-frequency conditions


Just as in the case of the 2-region exterior-rotor model, the solution for T = 0 can be checked by means
of Hughes and Miller [13]. The equations for Br and B2 in the magnet region are

μ0 K r p 1
2 1 (rH / r)2p
Br ' × cos p 2
2 rS 1 (rH / rS) 2p
(2.664)
μ0 K r p 1
2 1 (rH / r)2p
B2 ' × sin p 2
2 rS 1 (rH / rS) 2p

and if K = 8756 A/m, p =2, r = 22@5, rS = 26, rH = 20, the result is Br = 0@02379 cos p2 and B2 = 0@005503 sin
p2 T, both of which agree with the solution described above.
Page 2.192 SPEED’s Electric Machines

The Cartesian Solution

Fig. 2.196 Analysis in rectangular or cartesian coordinates

Fig. 196 shows the coordinate system for analysis of the eddy-current diffusion in rectangular
coordinates. The field obeys the diffusion equation (591) in conductive regions (sleeve, magnet, and
shaft):

M2A M2A MA
' μF ; (2.665)
Mx 2 My 2 Mt
otherwise it obeys Laplace's equation

M2A M2A
' 0. (2.666)
Mx 2 My 2
in the airgap and any non-conductive regions. There is no variation in the z-direction. Using the
principle of separation of the variables, as before, we have solution of the form

A (x,y,t) ' A (y) e j (T t q x)


(2.667)

and when this is substituted into eqn. (2.665) we have

d 2A( y )
(q 2 j T μ F) A( y ) ' 0 . (2.668)
dy 2
2
The coefficient (q + jTμF) has a real part
2B
q ' (2.669)
8

where 8 is the harmonic wavelength, and an imaginary part that is recognized as

j * 2 (2.670)
jTμF ' where d ' and * ' .
d2 2 TμF

* is the conventional skin-depth in the material whose conductivity is F S/m and permeability μ H/m.
If we write
q2 j T μ F ' "2 (2.671)

and if we continue to use boldface for complex (phasor) quantities eqn. (2.668) becomes
d 2A( y )
"2 A( y ) ' 0 . (2.672)
dy 2
Brushless permanent-magnet machines Page 2.193

This is a standard first-order differential equation with the following solutions in all the regions of Fig.
196. First, in the airgap,

A ' c 1 eq y c2 e qy
;

1 MA q
Hx ' ' c 1 eq y c2 e qy
;
μ0 My μ0 (2.673)

MA
By ' ' j q c 1 eq y c2 e qy
.
Mx
The solution includes expressions for Hx and By because the boundary conditions are expressed in
terms of these field quantities, and these will be applied shortly to find the unknown coefficients c.
Note: to make the solution consistent with the cylindrical solution, it may be necessary to negate both
Hx and By, since x = r2 in Fig. 2.196 as a result of the choice of coordinate axes.
In the sleeve,

A ' c 3 es y c4 e sy
;

s
Hx ' c 3 es y c4 e sy
; (2.674)
μS

By ' j q c 3 es y c4 e sy

where

1
s ' q2 j T μS FS ' q2 j / dS2 and dS ' . (2.675)
T μS FS
In the magnet,

A ' c 5 em y c6 e my
;

m
Hx ' c 5 em y c6 e my
; (2.676)
μM

By ' j q c 5 em y c6 e my

where

1
m ' q2 j T μM FM ' q2 j / dM2 and dM ' . (2.677)
T μM FM

Finally in the rotor body/shaft,

A ' c 7 eh y c8 e hy
;

h
Hx ' c 7 eh y c8 e hy
; (2.678)
μH

By ' j q c 7 eh y c8 e hy

where

1
h ' q2 j T μH FH ' q2 j / d H2 and dH ' . (2.679)
T μH FH
Page 2.194 SPEED’s Electric Machines

The boundary conditions are applied next at each interface between regions, and at y = yS and y = 0.
Across each interface )Hx = K, where K is a current-sheet in A/m; this means that the tangential
component of H is discontinuous by the amount of ampere-conductors per metre in the current-sheet.
Also )By = 0, which means that the normal component of B is continuous across each interface.
Thus at y = yS we have
Hx ( y ' y S 0 ) Hx ( y ' y S 0 ) ' K . (2.680)
If the stator core is assumed to be infinitely permeable, Hx (y = yS + 0) = 0, so
q yS q yS
q c1e c2e ' μ0 K . (2.681)

At y = y2 a second current sheet Q is introduced for diagnostic reasons and for checking, so
q y2 q y2 s y2 s y2
j q c1e c2e ' j q c3e c4e (for )B x) ; and
q y2 q y2 μ0 s y2 s y2 (2.682)
q c1e c2e ' s c3e c4e μ0 Q (for )H y) .
μS
At y = y1,
s y1 s y1 m y1 m y1
j q c3e c4e ' j q c5e c6e (for )B x) ; and
s y1 s y1 μS m y1 m y1 (2.683)
s c3e c4e ' m c5e c6e (for )H y) .
μM
At y = yH,
m yH m yH h yH h yH
j q c5e c6e ' j q c7e c8e (for )B x) ; and
m yH m yH μM h yH h yH (2.684)
m c5e c6e ' h c7e c8e (for )H y) .
μH
To enforce a finite solution at y = 4, c8 = 0. This leaves seven simultaneous linear complex equations
to be solved for the unknown coefficients c1, c2, ...c7. Let
q yS q yS
a1 ' q e ; a2 ' qe ;
q y2 q y2 s y2 sy
a3 ' e ; a4 ' e ; a5 ' e ; a6 ' e 2 ;
q y2 q y2 μ0 s y μ0 s y2
a7 ' q e ; a8 ' qe ; a9 ' s e 2 ; a10 ' s e ;
μS μS
s y1 s y1 m y1 my
a11 ' e ; a12 ' e ; a13 ' e ; a14 ' e 1 ; (2.685)
s y1 s y1 μS m y1 μS m y1
a15 ' s e ; a16 ' se ; a17 ' m e ; a18 ' m e ;
μM μM
m yH m yH h yH h yH
a19 ' e ; a20 ' e ; a21 ' e ; a22 ' e ;
m yH m yH μM h y H μM h yH
a23 ' m e ; a24 ' me ; a25 ' h e ; a26 ' h e .
μH μH

Then the boundary-condition equations become

a1 c 1 a2 c 2 ' μ0 K ;
a3 c 1 a4 c 2 a5 c 3 a6 c 4 ' 0;
a7 c 1 a8 c 2 a9 c 3 a10 c4 ' μ0 Q ;
a11 c3 a12 c4 a13 c5 a14 c6 ' 0; (2.686)
a15 c3 a16 c4 a17 c5 a18 c6 ' 0;
a19 c5 a20 c6 a21 c7 ' 0;
a23 c5 a24 c6 a25 c7 ' 0.
Brushless permanent-magnet machines Page 2.195

These can be solved by the same algebraic elimination process as before, using auxiliary variables

a23 a24 b1 b1
b1 ' a19 a21 ; b2 ' a20 a21 ; b3 ' a17 a18 ; b4 ' a13 a14 ;
a25 a25 b2 b2
b3 b3 b5 b5 (2.687)
b5 ' a15 a11 ; b6 ' a16 a12 ; b7 ' a9 a10 ; b8 ' a5 a6 ;
b4 b4 b6 b6
b7 b7 b9
b9 ' a7 a3 ; b10 ' a8 a4 ; b11 ' a1 a2 .
b8 b8 b10

The solution is then

1 a2 b9 μ0 Q a3 a4
c1 ' μ0 K μ0 Q ; c2 ' c1 ; c3 ' c1 c2
b11 b10 b10 b10 b8 b8 (2.688)
b5 a11 a12 b1 a23 a24
c4 ' c3 ; c5 ' c3 c4 ; c6 ' c5 ; c7 ' c5 c6 .
b6 b4 b4 b2 a25 a25

The eddy-current density in conductive regions can be determined from eqn. (2.604) on p. 181. The
losses in the conductive regions can be determined by the same methods described on p. 181.

Stoll and Hammond's solution


A particularly interesting example of the cartesian solution was given by Stoll and Hammond [55], using
fewer regions than in the present analysis (and only one conductive layer).

Fig. 2.197 Analysis in rectangular or cartesian coordinates

They presented the following equation (2.689) (Stoll's eqn. (17)) for the field in the airgap region. μ is the
2 2
relative permeability of the slab, g is the airgap length, u = 2/q * , and * is the skin-depth in the slab.

μ0 K q (y y S) μ 1 ju 2 qg q (y y2)
A ' e e e (2.689)
2q
μ 1 ju 2

The first term is attributed to the current-sheet and the second term to the effect of the slab. This
equation expresses the relative effects of the permeability of the slab and the eddy-currents. When *
is small the coefficient of the second term tends towards 1, and the eddy-currents are inductance-
limited. But if the slab is highly permeable (large μ), the inductance-limited condition does not arise
until the frequency is much higher. Stoll showed that a high permeability tends to extend the frequency
range of “resistance-limited” behaviour, in which an increase in resistivity decreases the losses. The
original paper is worth further study. (See also Lawrenson et al, [53]).
Page 2.196 SPEED’s Electric Machines

2.30 EVALUATION OF THE EXCITING HARMONIC CURRENT SHEETS


The current waveform in phase k is assumed to be periodic although it is otherwise not restricted. It
is therefore written in terms of the electrical angle Te t, where Te is the fundamental electrical radian
frequency 2B f :
i k ' i k ( Te t ) . (2.690)

This current can be analyzed into time-harmonic components : thus


ik ( Te t) ' Ik 1 cos ( Tet "k1 ) Ik 3 cos ( 3 Tet "k3 ) ... Ik n cos ( n Tet "kn ) ... (2.691)
The harmonic coefficients are computed using
2 B / Te
2
Ik n ' ik ( Te t ) cos ( n Te t "kn ) d( Te t) ; n ' 1,3,5,. . . (2.692)
2B 0

Ampere-conductor distribution
Each phase has a fixed conductor distribution in the slots, which can be denoted Ck. Ck is called the slot
conductor distribution for phase k. It is natural to think of Ck as an array of values denoted Ck q or Ck[q],
each of which means the number of conductors in slot q belonging to phase k. It is a signed quantity,
depending on the direction of the conductors in the slots.46 When Ck q is plotted as a function of N, the
azimuthal angle around the stator bore, it is a series of impulse functions, as shown in Fig. 198.
Alternatively it can be represented as a series of bars, each bar having the angular width of the slot
opening, as shown in Fig. 199. The functions drawn in Figs. 198 and 199 are denoted Ck(N), and the units
of Ck(N) are conductors per radian.

Fig. 2.198 Slot-conductor distribution (impulse functions)

Fig. 2.199 Slot-conductor distribution (bars)


If we multiply Ck q by the phase current i k, we have the slot ampere-conductor distribution Ak q which
is simply a scaled replica of Ck q. For reasons which appear later, it is better to express the function
Ak(N) in ampere-conductors per metre around the periphery of the stator bore at radius R. Thus in
mathematical function notation,
Ak(N) ' ikCk(N) / R Amp&conductors radian (2.693)
The graph of Ak(N) is similar to Fig. 198 or Fig. 199.

46
Ck q corresponds to one phase of the conductor location vector (CLV) in PC-BDC.
Brushless permanent-magnet machines Page 2.197

The so-called MMF distribution Fk (N) is the integral of the ampere-conductor distribution Ak (N):

Fk(N) ' Ak(N) R dN ' i Ck(N) dN . (2.694)

This is evaluated with a constant of integration such that the mean value of Fk(N) over one period is
zero. Examples corresponding to Figs. 198 and 199 are shown in Figs. 200 and 201. The units of Fk are
ampere-conductors or just amperes. The MMF distribution is central to the classical theory of electric
machines, particularly the theory of windings and winding factors. It can be used to predict the flux
distribution across a short airgap, when the iron is assumed to be infinitely permeable.

Fig. 2.200 MMF distribution (step functions)

Fig. 2.201 MMF distribution (ramp functions)

The MMF distribution is calculated in PC-BDC's winding editor together with its Fourier components
or space-harmonic components. With i k = 1@0 A flowing in the winding this is written in boldface Fk(N);
N is measured in mechanical radians and since windings may have more than one pair of poles, the
harmonic series includes both odd and even harmonics : thus

F k(N) ' Fk 1 sin ( N $k1 ) Fk 2 sin ( 2 N $k2 ) ... Fk m sin ( m N $km ) ... (2.695)

The corresponding space-harmonics of the ampere-conductor distribution are obtained by


differentiation, that is, from the inverse of eqn. (2.694): thus with i k = 1@0 A

A k(N) ' [Fk 1 cos ( N $k1 ) 2 Fk 3 cos ( 2 N $k2 ) ... m Fk m cos ( m N $km ) . . .] / R (2.696)

From this we can isolate the mth space-harmonic of the ampere-conductor distribution

1
Akm(N) ' m Fk m cos ( m N $km ) (2.697)
R

Now multiply this by the n'th time-harmonic in the current waveform Ikn cos (n Te t + "kn), to give

1
Akmn(N,t) ' m Ik n Fk m cos (m N $k m) cos (n Tet "k n) . (2.698)
R

This ampere-conductor distribution is sinusoidally distributed in space around the stator bore, with
m pole-pairs, and it pulsates in time with frequency n Te rad/s. It can be resolved into forward- and
backward-rotating components
Page 2.198 SPEED’s Electric Machines

1
Ak f m n(N,t) ' m Ik n Fk m cos ( m N n Tet $k m "k n ) forward (2.699)
2R

and

1
Ak bm n(N,t) ' m Ik n Fk m cos ( m N n Te t $k m "k n ) backward (2.700)
2R

The forward component rotates in the positive direction at the angular velocity (n/m) Te rad/s, while
the backward component has the angular velocity (n/m) Te rad/s and rotates in the negative direction.
The velocity of the harmonics in either direction is proportional to the order n of the time harmonic
in the current, and inversely proportional to the order m of the space-harmonic in the winding
distribution.
Now consider these components in a frame of reference rotating synchronously with the rotor. Let 2
be the azimuthal coordinate in the rotating frame, as distinct from N which is the azimuthal coordinate
in the stationary frame. They are related by
2 ' N T0 t , (2.701)

where T0 is the angular velocity of the rotor in mechanical rad/s. The rotor rotates in the forward
direction at the angular velocity T0 rad/s. If the number of rotor pole-pairs is P,
Te
T0 ' . (2.702)
P
To express the forward- and backward-rotating components of the stator ampere-conductor distribution
in the rotating frame, substitute for N using eqns. (2.701) and (2.702) in eqns. (2.699) and (2.700):

1
Ak f m n(N,t) ' m Ik n Fk m cos [ m 2 (m n P) T0 t $k m "k n ] forward (2.703)
2R

and

1
Ak bm n(N,t) ' m Ik n Fk m cos [ m 2 (m n P) T0 t $k m "k n ] backward (2.704)
2R

The angular velocity of the ampere-conductor distribution relative to the rotor is

T0
S ' (m K n P) rad/s . (2.705)
m
For example, suppose we have a 4-pole rotor rotating at 1,000 rpm. Then T0 = 2B/60 × 1000 = 104@7 rad/s
and P = 2. Consider the 5th “electrical” space-harmonic of the winding distribution, with fundamental
current flowing. In a 4-pole machine the 5th “electrical” harmonic is the 10th “mechanical” harmonic,
so m = 10 and n = 1. The relative velocity of the forward component is (10 1 × 2)/10 × 104@72 = 4/5 ×
104@72 = 83@76 rad/s, and the relative velocity of the backward component is (10 + 1 × 2)/10 × 104@72 =
12/10 × 104@72 = 125@66 rad/s. In the stationary frame the forward component rotates at 104@72 83@76
= 104@72 × (1 4/5) = 104@72/5 = 20@94 rad/s, and the backward component at 104@72 125@66 ' 104@72 × (1
1@2) = 104@72/5 = 20@94 rad/s.
As a second example, consider the same space-harmonic but with 5th time-harmonic current flowing in
the windings. Then m = 5 × 2 = 10 as before, while n = 5 instead of 1. Eqn. (2.705) shows that the forward
component rotates at a velocity of (10 5 × 2) T0/10 = 0 relative to the rotor, while the backward
component rotates at (10 + 5 × 2) T0/10 = 2 T0 = 209@44 rad/s relative to the rotor. In the stationary
frame the forward component rotates at 104@72 rad/s, synchronously with the rotor; while the backward
component rotates at 104@72 209@44 = 104@72 × (1 2) = 104@72 rad/s.
Brushless permanent-magnet machines Page 2.199

Extracting the required amplitude, pole-number and frequency of the harmonic waves
For the analysis of harmonic losses we need all the space/time harmonic components of the stator
ampere-conductor distribution expressed in the form

A e j (m 2 T t)
(2.706)
in a frame of reference rotating synchronously with the rotor. Such current sheets have already been
used [for example in eqn. (2.611) on p. 182 and eqn. (2.650) on p. 189, and elsewhere], where they were
used to introduce the excitation via a boundary condition at the stator bore.47
The parameters A and T are yet to be determined from the previous analysis, but m is the number of
harmonic pole-pairs, which is solely a function of the stator winding distribution. Thus m is the same
in either the rotating or the stationary frame of reference: it is invariant.
The radian frequency T must be chosen to give the correct angular velocity relative to the rotor. In
terms of eqn. (2.706) this velocity is T/m rad/s, and it must be equal to S from eqn. (2.705): thus
T ' ( m ± n P ) T0 . (2.707)
If we now define the “electrical” harmonic order as
m
mN ' , (2.708)
P

and substitute this together with eqn. (2.702) in eqn. (2.707), we have

T ' ( mN ± n ) Te rad/s . (2.709)


Finally, the amplitudes of the forward and backward components are found directly from the
amplitudes in eqns. (2.703) and (2.704).
As an example, consider a case where mN = 19 and n = 1. Then eqn. (2.709) gives T = ( 19 ± 1) Te which
is 20 Te or 18 Te. The 19th space-harmonic produces a forward component rotating at 1/19 of the
synchronous speed in the forward direction. In a frame of reference rotating forwards at synchronous
speed, this component appears to be rotating backwards at the relative speed (1/19 1) = 18/19 times
the synchronous speed. The number of pole-pairs is 19, so the frequency is 18/19 × 19 = 18 times the
fundamental frequency Te.
The 19th space-harmonic also produces a backward component rotating at 1/19 of the synchronous speed
in the backward direction. Again, in the rotor frame of reference the backward component appears to
be rotating backwards at the relative speed ( 1/19 1) = 20/19 times the synchronous speed. The
number of pole-pairs is 19, so the frequency is 20/19 × 19 = 20 times the fundamental frequency Te.
In both cases the negative sign simply means that the field is rotating backwards relative to the rotor.
When a negative value of T ' 18 Te is substituted in eqn. (2.706) along with mN = 19 = 19P, it becomes
j (19 P 2 18 Te t) j (19 P 2 18 P T0 t)
Ae ' Ae . (2.710)

This represents a field of 38P poles rotating backwards at 18/19 times the rotor speed relative to the
rotor. For a fixed supply frequency Te, all physical speeds are inversely proportional to P.

Balanced operation of 3-phase machines


In 3-phase machines with balanced windings and balanced 3-phase currents, some 2/3 of the possible
harmonic fields vanish, as will now be shown. The analysis begins in the stationary frame of reference
with eqns. (2.699) and (2.700). Considering first the forward component, I kn and " kn will be the same
for all three phases k = 1,2,3, but Te t will be replaced by Te t 2B/3 in phase 2 and by Te t + 2B/3 in phase
3. Similarly Fkm and $km will be the same for k = 1,2,3, while N will be replaced by N 2B/3P in phase
2 and by N + 2B/3P in phase 3. If we write (mn for $m "n and h ' 2B/3, the total forward component for
the three phases becomes

47
In those cases the symbol K was used instead of A.
Page 2.200 SPEED’s Electric Machines

1
A f m n(N,t) ' m In Fm cos [ m N n Tet (mn ]
2R (2.711)
cos [ m (N h/P) n (Tet h) (mn ] cos [ m (N h/P) n (Tet h) (mn ]

Let . = mN nTet + (mn. Then with mN = m/P this expression can be reduced to

1
A f m n(N,t) ' m In Fm cos . × 1 2 cos (mN n) h (2.712)
2R

Similarly for the backward component, writing > = mN + nTet + "m + $n,

1
A bm n(N,t) ' m In Fm cos > × 1 2 cos (mN n) h (2.713)
2R

The coefficient term [1 + 2 cos (mN n) × 2B/3] in the forward component vanishes unless (mN n) is zero
or a multiple of 3. Similarly the term [1 + 2 cos (mN + n) × 2B/3] in the backward component vanishes
unless (mN + n) is a multiple of 3.
Thus for example if mN = 1 and n = 1 we have the fundamental electrical space-harmonic of the winding
distribution and the fundamental component of phase current, mN n = 0, and

3 3
A f 1 1(N,t) ' I F cos . ' I F cos( P N Tet $1 "1 ) , (2.714)
2R 1 1 2R 1 1
while Ab11 = 0. This represents a 2P-pole field rotating at the angular velocity Te rad/s with a phase
angle $1 "1. The frequency relative to the rotor is given by eqn. (2.709) as ( 1 + 1) Te = 0. Note that eqn.
(2.709) also gives the frequency of the backward field relative to the rotor as ( 1 1) Te = 2 Te, but since
the coefficient Ab11 = 0, there is no backward field.
Next consider mN = 5 and n = 7. The coefficient term [1 + 2 cos (mN n) × 2B/3] in the forward component
is 0, so there is no forward field: Af 57 = 0. The term [1 + 2 cos (mN + n) × 2B/3] in the backward
component is equal to 3, so there is a backward field

3 3 × 5
A b 5 7(N,t) ' m I7 F5 cos > ' I F cos( 5 P N 7 Tet $5 "7 ) . (2.715)
2R 2R 7 5
This represents a field with 5P pole-pairs rotating at the angular velocity (7/5) Te t rad/s. Relative to
the rotor, assuming P = 1, the angular velocity is (1 + 7/5) Te t = 12/5 Te t and with 5 pole-pairs the
frequency induced in the conductive components on the rotor is 5 × ( 12/5 Te) ' 12 Te rad/s. Eqn.
(2.709) gives T = ( 5 × 1 7) Te = 12 Te, which checks the result.
The rotor frequencies given by eqn. (2.709) are displayed in Tables 1 and 2 for the forward and backward
components respectively. In all cases the units are multiples of Te, and the space-harmonics are
electrical (i.e, mN is the parameter in the left-hand column).
Counting only odd harmonics up to the 37th in mN and n, there are 102 forward harmonics in Table 1 and
120 non-zero backward harmonics in Table 2. Along the diagonal in Table 1 are the “synchronous”
harmonics including the fundamental. The total number of field solutions required is 222.
Tables 3 and 4 show the rotor frequencies for an unbalanced machine. Both tables are full, except for
the diagonal of Table 3 which again represents the synchronous harmonics. In this case there are 703
non-zero entries requiring 703 field solutions. The balanced machine thus has only 222/703 or
approximately one-third the number of effective harmonics.
If we delete all triple-m space harmonics of the winding distribution and all triple-n time-harmonics
from the current waveforms, Tables 1 and 2 for a balanced 3-phase machine reduce to Tables 5 and 6,
with 72 and 85 non-zero entries respectively, making a total of 156 — only 22% of the original 703.
To achieve this “triple-n” reduction it is only necessary to connect the windings in wye (star) with a 3-
wire connection, since the triple-n time harmonics only interact with triple-m space harmonics.
Conversely the same effect can be achieved in a winding with all triple-m winding factors zero.
Brushless permanent-magnet machines Page 2.201

n ,
mN 1 3 5 7 9 11 13 15 17 19 21 23 25 27 29 31 33 35 37
1 0 6 12 18 24 30 36
3 0 6 12 18 24 30
5 0 6 12 18 24 30
7 -6 0 6 12 18 24 30
9 -6 0 6 12 18 24
11 -6 0 6 12 18 24
13 -12 -6 0 6 12 18 24
15 -12 -6 0 6 12 18
17 -12 -6 0 6 12 18
19 -18 -12 -6 0 6 12 18
21 -18 -12 -6 0 6 12
23 -18 -12 -6 0 6 12
25 -24 -18 -12 -6 0 6 12
27 -24 -18 -12 -6 0 6
29 -24 -18 -12 -6 0 6
31 -30 -24 -18 -12 -6 0 6
33 -30 -24 -18 -12 -6 0
35 -30 -24 -18 -12 -6 0
37 -36 -30 -24 -18 -12 -6 0

Table 1 Rotor frequencies for forward field component in 3-phase balanced machine
n ,
mN 1 3 5 7 9 11 13 15 17 19 21 23 25 27 29 31 33 35 37
1 -6 -12 -18 -24 -30 -36
3 -6 -12 -18 -24 -30 -36
5 -6 -12 -18 -24 -30 -36 -42
7 -12 -18 -24 -30 -36 -42
9 -12 -18 -24 -30 -36 -42
11 -12 -18 -24 -30 -36 -42 -48
13 -18 -24 -30 -36 -42 -48
15 -18 -24 -30 -36 -42 -48
17 -18 -24 -30 -36 -42 -48 -54
19 -24 -30 -36 -42 -48 -54
21 -24 -30 -36 -42 -48 -54
23 -24 -30 -36 -42 -48 -54 -60
25 -30 -36 -42 -48 -54 -60
27 -30 -36 -42 -48 -54 -60
29 -30 -36 -42 -48 -54 -60 -66
31 -36 -42 -48 -54 -60 -66
33 -36 -42 -48 -54 -60 -66
35 -36 -42 -48 -54 -60 -66 -72
37 -42 -48 -54 -60 -66 -72

Table 2 Rotor frequencies for backward field component in 3-phase balanced machine
Page 2.202 SPEED’s Electric Machines

n ,
mN 1 3 5 7 9 11 13 15 17 19 21 23 25 27 29 31 33 35 37
1 0 2 4 6 8 10 12 14 16 18 20 22 24 26 28 30 32 34 36
3 -2 0 2 4 6 8 10 12 14 16 18 20 22 24 26 28 30 32 34
5 -4 -2 0 2 4 6 8 10 12 14 16 18 20 22 24 26 28 30 32
7 -6 -4 -2 0 2 4 6 8 10 12 14 16 18 20 22 24 26 28 30
9 -8 -6 -4 -2 0 2 4 6 8 10 12 14 16 18 20 22 24 26 28
11 -10 -8 -6 -4 -2 0 2 4 6 8 10 12 14 16 18 20 22 24 26
13 -12 -10 -8 -6 -4 -2 0 2 4 6 8 10 12 14 16 18 20 22 24
15 -14 -12 -10 -8 -6 -4 -2 0 2 4 6 8 10 12 14 16 18 20 22
17 -16 -14 -12 -10 -8 -6 -4 -2 0 2 4 6 8 10 12 14 16 18 20
19 -18 -16 -14 -12 -10 -8 -6 -4 -2 0 2 4 6 8 10 12 14 16 18
21 -20 -18 -16 -14 -12 -10 -8 -6 -4 -2 0 2 4 6 8 10 12 14 16
23 -22 -20 -18 -16 -14 -12 -10 -8 -6 -4 -2 0 2 4 6 8 10 12 14
25 -24 -22 -20 -18 -16 -14 -12 -10 -8 -6 -4 -2 0 2 4 6 8 10 12
27 -26 -24 -22 -20 -18 -16 -14 -12 -10 -8 -6 -4 -2 0 2 4 6 8 10
29 -28 -26 -24 -22 -20 -18 -16 -14 -12 -10 -8 -6 -4 -2 0 2 4 6 8
31 -30 -28 -26 -24 -22 -20 -18 -16 -14 -12 -10 -8 -6 -4 -2 0 2 4 6
33 -32 -30 -28 -26 -24 -22 -20 -18 -16 -14 -12 -10 -8 -6 -4 -2 0 2 4
35 -34 -32 -30 -28 -26 -24 -22 -20 -18 -16 -14 -12 -10 -8 -6 -4 -2 0 2
37 -36 -34 -32 -30 -28 -26 -24 -22 -20 -18 -16 -14 -12 -10 -8 -6 -4 -2 0

Table 3 Rotor frequencies for forward field component in unbalanced machine


n ,
mN 1 3 5 7 9 11 13 15 17 19 21 23 25 27 29 31 33 35 37
1 -2 -4 -6 -8 -10 -12 -14 -16 -18 -20 -22 -24 -26 -28 -30 -32 -34 -36 -38
3 -4 -6 -8 -10 -12 -14 -16 -18 -20 -22 -24 -26 -28 -30 -32 -34 -36 -38 -40
5 -6 -8 -10 -12 -14 -16 -18 -20 -22 -24 -26 -28 -30 -32 -34 -36 -38 -40 -42
7 -8 -10 -12 -14 -16 -18 -20 -22 -24 -26 -28 -30 -32 -34 -36 -38 -40 -42 -44
9 -10 -12 -14 -16 -18 -20 -22 -24 -26 -28 -30 -32 -34 -36 -38 -40 -42 -44 -46
11 -12 -14 -16 -18 -20 -22 -24 -26 -28 -30 -32 -34 -36 -38 -40 -42 -44 -46 -48
13 -14 -16 -18 -20 -22 -24 -26 -28 -30 -32 -34 -36 -38 -40 -42 -44 -46 -48 -50
15 -16 -18 -20 -22 -24 -26 -28 -30 -32 -34 -36 -38 -40 -42 -44 -46 -48 -50 -52
17 -18 -20 -22 -24 -26 -28 -30 -32 -34 -36 -38 -40 -42 -44 -46 -48 -50 -52 -54
19 -20 -22 -24 -26 -28 -30 -32 -34 -36 -38 -40 -42 -44 -46 -48 -50 -52 -54 -56
21 -22 -24 -26 -28 -30 -32 -34 -36 -38 -40 -42 -44 -46 -48 -50 -52 -54 -56 -58
23 -24 -26 -28 -30 -32 -34 -36 -38 -40 -42 -44 -46 -48 -50 -52 -54 -56 -58 -60
25 -26 -28 -30 -32 -34 -36 -38 -40 -42 -44 -46 -48 -50 -52 -54 -56 -58 -60 -62
27 -28 -30 -32 -34 -36 -38 -40 -42 -44 -46 -48 -50 -52 -54 -56 -58 -60 -62 -64
29 -30 -32 -34 -36 -38 -40 -42 -44 -46 -48 -50 -52 -54 -56 -58 -60 -62 -64 -66
31 -32 -34 -36 -38 -40 -42 -44 -46 -48 -50 -52 -54 -56 -58 -60 -62 -64 -66 -68
33 -34 -36 -38 -40 -42 -44 -46 -48 -50 -52 -54 -56 -58 -60 -62 -64 -66 -68 -70
35 -36 -38 -40 -42 -44 -46 -48 -50 -52 -54 -56 -58 -60 -62 -64 -66 -68 -70 -72
37 -38 -40 -42 -44 -46 -48 -50 -52 -54 -56 -58 -60 -62 -64 -66 -68 -70 -72 -74

Table 4 Rotor frequencies for backward field component in unbalanced machine


Brushless permanent-magnet machines Page 2.203

n ,
m' 1 5 7 11 13 17 19 23 25 29 31 35 37
1 0 6 12 18 24 30 36
5 0 6 12 18 24 30
7 -6 0 6 12 18 24 30
11 -6 0 6 12 18 24
13 -12 -6 0 6 12 18 24
17 -12 -6 0 6 12 18
19 -18 -12 -6 0 6 12 18
23 -18 -12 -6 0 6 12
25 -24 -18 -12 -6 0 6 12
29 -24 -18 -12 -6 0 6
31 -30 -24 -18 -12 -6 0 6
35 -30 -24 -18 -12 -6 0
37 -36 -30 -24 -18 -12 -6 0

Table 5 Rotor frequencies for forward field component in 3-phase balanced machine,
with all triple-m space and triple-n time harmonics deleted
n ,
mN 1 5 7 11 13 17 19 23 25 29 31 35 37
1 -6 -12 -18 -24 -30 -36
5 -6 -12 -18 -24 -30 -36 -42
7 -12 -18 -24 -30 -36 -42
11 -12 -18 -24 -30 -36 -42 -48
13 -18 -24 -30 -36 -42 -48
17 -18 -24 -30 -36 -42 -48 -54
19 -24 -30 -36 -42 -48 -54
23 -24 -30 -36 -42 -48 -54 -60
25 -30 -36 -42 -48 -54 -60
29 -30 -36 -42 -48 -54 -60 -66
31 -36 -42 -48 -54 -60 -66
35 -36 -42 -48 -54 -60 -66 -72
37 -42 -48 -54 -60 -66 -72

Table 6 Rotor frequencies for backward field component in 3-phase balanced machine,
with all triple-m space and triple-n time harmonics deleted
Page 2.204 SPEED’s Electric Machines

Unbalanced operation
Unbalanced operation, or “imbalance” for short, comes in many combinations: for example, unbalanced
currents may flow in balanced windings, or conversely, balanced currents may flow in unbalanced
windings. In the general case both the currents and the windings are unbalanced.
We should first make sure of what we mean by the term “balanced”. It is fortunate that in the midst of
a forest of mathematical complexity, the intuitive engineering notion of symmetry is all we need to
define “balance”. Thus balanced windings are symmetrical, and in a three-phase machine this means
that the windings are physically identical, but displaced from one another in the circumferential
direction around the stator by 2B/3P radians or 120/PE, that is, 120 electrical degrees, (P being the
fundamental number of rotor pole-pairs). Similarly, balanced currents have identical waveforms and
are displaced from one another in time phase by 120E at the fundamental frequency, giving rise to a
“star of phasors” displaced from one another by 120E as shown by the phasors I1N, I2N, I3N in Fig. 2.202a.
The phasor diagram is familiar for the fundamental harmonic component of current. When there are
harmonics, there is a phasor diagram for every harmonic. It is hardly practical to draw this diagram
superimposed on the fundamental phasor diagram, because the phasor diagram for a harmonic does
not rotate at the same speed as the fundamental, and it may not even have the same phase sequence.
The geometric construction of the “star of phasors” can also be used to represent the windings in terms
of the harmonic MMF vectors Fkm considered earlier. The Görges diagram is based on this principle.
Every electrical engineer knows that symmetrical components are commonly used for the analysis of
unbalanced operation, especially in power systems. When the imbalance is confined to the currents
and the windings are balanced, this method (see below) is not only manageable but sufficiently general
to deal with all cases of steady-state imbalance. However, in our case we have to deal with imbalance
in the windings as well as in the currents, and we have already developed the mathematics to do this
in the form of eqns. (2.709), (2.703) and (2.704). For this reason it is more useful to consider special
simple cases of imbalance as simplifications or reductions of these equations, leaving the method of
symmetrical components as a means of illustration and interpretation and not the main analytical tool.
Indeed we have already considered two special cases of “no imbalance” and “no triplen harmonics”,
which are summarized in Tables 1,2,5 and 6.
Two “special simple cases of imbalance” will be considered, each of which requires only a single
parameter to define the amount of imbalance. The first has a negative-sequence component but no zero-
sequence component, while the second has a zero-sequence component but no negative-sequence
component. Both also have a positive-sequence component, in general. For more general cases of
imbalance, it is necessary to revert to the general equations cited above.
Negative-sequence only — Fig. 2.202a shows a simple case of imbalance in which the current in phase
1 is increased by the factor (1+u), while the currents in phases 2 and 3 remain unaltered from their
normal balanced values; when u = 0 the currents are balanced. In Fig. 2.202b the phasors I2 and I3 are
copied to form a closed triangle with I1, representing conditions in a machine with a 3-wire connection
and no zero-sequence component:
I1 I2 I3 ' 0 . (2.716)

With the constraint of eqn. (2.716), the increase in I1 drags the I2 and I3 phasors to the right in the closed
triangle in Fig. 2.202b. They retain the same magnitude, but I2 is retarded in phase by 0 while I3 is
advanced by the same angle 0. These phase shifts can be seen more clearly in the star diagram in the
left-hand part of Fig. 2.202b. With unchanged magnitudes I2 = I3 = 1, it appears from the closed triangle
I1I2I3 in Fig. 2.202b that
1 u 1 u
I2 ' I3 ' 1 ' ' . (2.717)
2 cos N 2 cos (B/3 0)

The phase-shift angle 0 can be solved from this equation, giving

B 1 1 u
0 ' cos . (2.718)
3 2
Brushless permanent-magnet machines Page 2.205

I 3' I 3'
I3

Original balanced set


0
u u
I 1'

I 1' I1 N I1

0 I3 I 2'

I 3' I2
I2

I 2' I 2'
(a) (b)

Fig. 2.202 A simple case of imbalance, with negative-sequence but no zero-sequence component

Hence
j (2B/3 0)
I1 ' 1 u; I2 ' 1 e ; I3 ' 1 e j (2B/3 0)
. (2.719)

The fundamental components of the instantaneous currents are given by


i1 ' (1 u) I1 /2 cos T t
i2 ' I1 /2 cos [ T t (2B/3 0) ] (2.720)
i3 ' I1 /2 cos [ T t (2B/3 0) ] ,
th
and from this the n harmonic can be inferred as
i1 ' (1 u) In /2 cos n T t
i2 ' In /2 cos n [ T t (2B/3 0) ] (2.721)
i3 ' In /2 cos n [ T t (2B/3 0) ] .

These currents can be substituted in eqns. (2.699) and (2.700) to give an equation similar to eqn. (2.711),
which can then be reduced to the following equations for the forward and backward components,
similar to eqns. (2.712) and (2.713):
1
Af mn(N,t) ' m In F m cos . × {1 u 2 cos [(mN n) h n 0]} ; (2.722)
2R

1
Abmn(N,t) ' m In F m cos > × {1 u 2 cos [(mN n) h n 0]} . (2.723)
2R

If u = 0 these equations reduce to eqns. (2.712) and (2.713) for balanced operation. We have already seen
that for balanced operation the forward component vanishes unless (mN n) is zero or a multiple of 3;
but for unbalanced operation it is generally non-zero for all values of mN and n; hence the fullness of
Tables 3 and 4. For example if mN = 1 and n = 1, with u = 0 we have already seen that the coefficient of
the forward component is 3I1F1/2R. With u ' 0@1 we have 0 = 3@367E and

1 1
Af 11(N,t) ' I F cos . × {1 0@1 2 cos [ 3@367E]} ' I F cos . × 3@09655 ; (2.724)
2R 1 1 2R 1 1
1 1
Ab 11(N,t) ' I F cos > × {1 0@1 2 cos [(1 1) × 120E 3@367E]} ' I F cos > × 0@20345 . (2.725)
2R 1 1 2R 1 1
Page 2.206 SPEED’s Electric Machines

The forward component Af11, which is synchronous with the rotor, is increased by approximately 3@2%.
The backward component Ab11, which has a frequency of 2Te relative to the rotor, and which was zero
under balanced operation, has magnitude 0@20345/3@09655 = 0@0657 compared to the forward component.
Again consider mN = 5 and n = 7. Previously with balanced operation Af57 = 0, but with u = 0@1 we get

5 I7 F5
Af 57(N,t) ' cos . × {1 0@1 2 cos [(5 7) × 120E 7 × 3@367E]}
2R
(2.726)
5 I7 F5
' cos . × 0@87599 .
2R

The backward component is

5 I7 F5
Ab 57(N,t) ' cos > × {1 0@1 2 cos [(5 7) × 120E 7 × 3@367E]}
2R
(2.727)
5 I7 F5
' cos > × 2@93316 ,
2R

which is reduced in the ratio 2@93316/3 = 0@97772 compared with the balanced case.
The imbalance characterized by Fig. 2.202 is represented by a single imbalance parameter u that refers
to the currents, and the foregoing analysis assumes a balanced winding. In terms of symmetrical
components, the imbalance produces a negative-sequence component but no zero-sequence component.
While it is convenient to work with only a single imbalance parameter and a graphical interpretation,
it might also be important to associate the imbalance parameter u with the amount of negative-sequence
current produced, and with the effect of the imbalance on the positive-sequence component, at least for
the fundamental time-harmonic of the current.
The symmetrical-component transformation can be written

2
Ip 1 h h I1
1 2 (2.728)
In = 1 h h I2
3
I0 1 1 1 I3

j2B/3
where h = e . Substituting from eqns. (2.719), and simplifying, we get

1
Ip ' 1 u 2 cos 0 ;
3
1
In ' 1 u 2 cos (2B/3 0) ; (2.729)
3
1
I0 ' 1 u 2 cos (2B/3 0) .
3

For example with u = 0@1, 0 = 3@367E,


1
Ip ' [1 0@1 2 cos 3@367E ] ' 1@03218 ;
3
1
In ' [1 0@1 2 cos ( 120E 3@367E) ] ' 0@06782 ; (2.730)
3
1
I0 ' [1 0@1 2 cos ( 120E 3@367E) ] ' 0 .
3

Note that In/Ip = 0@06782/1@03218 = 0@06571, which is the same ratio found between Ab11 and Af11 above.
The converse problem is also of interest : that is, to find the value of u when the sequence components
are given. However this requires an iterative solution.
Brushless permanent-magnet machines Page 2.207

I 2'
I3 I3

I2

N I1
I0
N' I 1'

I2
I 3'

Fig. 2.203 "Single parameter" imbalance : zero-sequence but no negative-sequence

Zero-sequence only — A second example of "single-parameter" imbalance is shown in Fig. 2.203. In


this case there is a zero-sequence component I0 = NN', but no negative-sequence component. It is
characteristic of such systems that the vertices of the unbalanced current phasors I1, I2 and I3 form an
equilateral triangle, but the neutral point N is displaced from the centroid N', and NN' = I0. In Fig. 2.203
I2 and I3 are copied and translated to try to form a closed triangle, but it cannot be done : instead there
is formed a quadrilateral whose sides are I1, I2, I3, and 3NN'. Only if NN' = 0 does the triangle close.
The unbalanced system in Fig. 2.203 is equivalent to the sum of the balanced system I1N, I2N, I3N and the
zero-sequence component I0. Evidently

I1 ' I1N I0 ; I2 ' I2N I0 ; I3 ' I3N I0 . (2.731)

The balanced system and its harmonics obey the rules already worked out in relation to eqns. (2.712)
and (2.713). So it remains to add the zero-sequence component in the rotor frame of reference. Starting
with eqn. (2.711), the zero-sequence component uIn cos (. R) is added to each phase, where R is the
phase angle between I0 and I1, so with . = mN nTet + (mn as before, the forward component becomes

1
Af mn(N,t) ' m In F m cos . cos ( . mNh ) cos ( . mNh ) 3 u cos (. R)
2R
(2.732)
1
' m In F m cos . × 1 3 u cos R 2 cos (mN n) h) 3 u sin R sin. .
2R

This simplifies if R = 0, which is not unduly restrictive for loss calculations: thus
1
Af mn(N,t) ' m In F m cos . × 1 2 cos (mN n)h 3u . (2.733)
2R

The frequency relative to the rotor is given by eqn. (2.709) as (mN n)Te electrical rad/s. Similarly for
the backward component, which has a frequency (mN n)Te relative to the rotor,
1
Abmn(N,t) ' m In F m cos . × 1 2 cos (mN n)h 3u . (2.734)
2R

If u = 0 these equations again reduce to eqns. (2.712) and (2.713) for balanced operation. Otherwise both
components are generally nonzero even when (mN ± n) is zero or a multiple of 3; hence again the fullness
of Tables 3 and 4.
Page 2.208 SPEED’s Electric Machines

Fig. 2.204 An alternative single-parameter definition of imbalance, with no zero-sequence component.

The idea of forward and backward zero-sequence components perhaps seems strange, because the co-
phasal zero-sequence phasors seem to have had all sense of rotation knocked out of them, so an example
may be helpful. First notice that Afmn and Abmn are both zero for non-triplen values of the electrical
space-harmonic mN. This helps to explain the common association (and even confusion) of “zero
sequence” with “third harmonic”. So consider first the case mN = 3 and n = 1. According to eqn. (2.709)
the frequency relative to the rotor is 2Te for the forward component and 4Te for the backward
component, just as they appear in Tables 3 and 4 respectively.
Remembering that the zero-sequence components of current in the three phases are in phase with one
another, they produce a stationary armature-reaction field with 3P pole-pairs pulsating or alternating
at the fundamental frequency. This is resolved into a forward component rotating at Te/3 electrical
rad/s or Te/3P mechanical rad/s; and a backward component rotating at Te/3 electrical rad/s or
Te/3P mechanical rad/s. Both components have 3P pole-pairs. Relative to the rotor, the angular
velocity of the forward component is Te/3 Te = 2/3 Te electrical rad/s or Te/3P Te/P ' 2/3 Te/P
mechanical rad/s. Since there are 3P pole-pairs, the electrical frequency generated in the rotor is 3P
× [ 2/3 Te/P] = 2Te rad/s.
Relative to the rotor, the angular velocity of the backward component is Te/3 Te = 4/3 Te electrical
rad/s or Te/3P Te/P ' 4/3 Te/P mechanical rad/s. Since there are 3P pole-pairs, the electrical
frequency generated in the rotor is 3P × [ 4/3 Te/P] = 4Te rad/s.
For a wye-connected machine with no neutral connection there can be no zero-sequence currents under
normal operating conditions, but in the closed delta connection there is nothing to restrict them.
According to our theory and eqns. (2.732, 733, 734) and (2.?) in particular, they can be completely
suppressed in windings whose triplen-harmonic winding factors are all zero, even when the winding
is otherwise unbalanced. If this is not the case, one must try to eliminate triplen-harmonic EMFs from
the phases by profiling the airgap flux-density due to the magnets. The former method is the surer.
An alternative model of imbalance is shown in Fig. 2.204a. The unbalanced phasors I1, I2, I3 form a
closed triangle, so that there is no zero-sequence component and the neutral point is at the centroid in
the star diagram, Fig. 2.204b. The balanced set I1N, I2N, I3N forms an isosceles triangle, with I1N = I1. The
MMF of the unbalanced set I1, I2, I3 is equal to the sum of the MMF of the balanced set I1N, I2N, I3N and the
MMF of the single-phase current IN flowing in phases 2 and 3. From Fig. 2.204,
I1 ' I1N ; I2 ' I2N IN ; I3 ' I3N IN . (2.735)
These equations can be solved to find IN :
j 2N
IN ' (I3 I3N) (I2 I2N) / 2 ' IN e . (2.736)

The forward MMF component due to IN at the fundamental frequency is


m IN F m
Af mn ' cos m(N h/P) (Tet 2N) (mn cos m(N h/P) (Tet 2N) (mn , (2.737)
2R
a messy expression that shows this method of “parameterizing” the imbalance to be unpromising.
Brushless permanent-magnet machines Page 2.209

2.31 SEGMENTED MAGNETS AND FINITE-LENGTH EFFECTS


Magnets are often divided into blocks which are essentially isolated from one another electrically. They
are nearly always divided in the circumferential direction with at least one block per pole. Further
division in the circumferential direction is common, especially in larger machines. Segmentation in
the axial direction is also common, even in small machines. A basic reason for the segmentation is to
keep the individual blocks of magnets small, for obvious manufacturing reasons. But another reason
is to break up the eddy-current paths into smaller loops, which increases the effective resistance
presented to the eddy-currents. Provided that the eddy-currents are resistance-limited, this will
decrease the associated power loss.
The analysis of the effects of segmentation falls outside the scope of 2-dimensional analysis, while full
3-dimensional analysis is generally not feasible except by numerical methods. For rapid calculation
there is no alternative other than to use approximate estimates, and a number of methods are developed
below. Suffice it to say that these methods generally do not obey Maxwell's equations. Unless tested
or calibrated, they are fraught with risk, and their practical usefulness is their only justification.
Circumferential segmentation can in principle be included in a conventional 2-dimensional
formulation, solving the complex Helmholtz equation by separation of variables, but it is necessary to
subdivide the magnet regions into isolated segments, with air in the “interpolar” regions between them.
Such solutions are difficult and rare, the best (and possibly the only) published example being that of
Deng [73-75]. Zhu [63] embarked on a similar analytical path but he represented the magnets by a single
region, a continuous ring, and subsequently integrated the eddy-current loss density only over the
actual magnet material. He did, however, include a continuous conductive sleeve.
The main method developed here is based on the 2-dimensional solution for a continuous ring, which
is then segmented and subjected to “residual current suppression” in each segment. This approach can
be justified only for eddy-currents that are approximately resistance-limited.
Axial segmentation and finite length. Eddy currents which are induced to flow in the axial direction
must close their paths at the ends of the machine. If the ratio of the magnet axial length to the harmonic
pole-pitch is small, the additional impedance of the "closure" sections may not be negligible.
Axial segmentation could, in theory, be analyzed by postulating a harmonic variation of the field in the
z-direction. This very old idea (Wood and Concordia [59], Angst [58], Preston and Reece [57]) was first
formulated to deal with finite-length effects, which are approximated by postulating a series of rotors
stacked end-to-end, possibly with a gap between each pair. If it works for finite-length effect it must also
work for multiply-segmented magnets. The basic notion can be expressed in terms of an extended
solution for the vector potential in the form of an infinite series of axial harmonics m,

A (r,2,t) ' A (r) cos p 2 sin m z e j T t (2.738)

in which m is an axial wavenumber. The idea was used recently by Kirtley et al [60], but a much more
detailed development was given 20 years earlier by Ralph ([102], Lawrenson and Ralph [56]), who
discovered previously unknown terms in the solution of the complex diffusion equation, including one
with linear variation with z. This raises questions about the rigour of simplified approaches such as
the one in eqn. (2.738). Lawrenson commented (modestly but somewhat prophetically) that
“the complexity of the solutions probably leaves the reader sharing the view . . . that contemplation
of anything more algebraically complex is unlikely to be worthwhile and purely numerical
approaches must be taken up” (Binns, Lawrenson and Trowbridge [90]).

They certainly have been “taken up”.


The methods developed here for axial end-effects and axial segmentation are based on postulated eddy-
current paths at the ends of the magnets (or magnet segments). These paths have a predefined shape
which makes it possible to estimate not only the change in the path resistance but also the change in
the path EMF which results from finite length and segmentation. A similar approach used by Russell
and Norsworthy [69] is also reviewed. But first we consider circumferential segmentation.
Page 2.210 SPEED’s Electric Machines

Segment analysis — circumferential segmentation — residual current suppression

Fig. 2.205 Magnet segmentation in the circumferential direction

Fig. 2.205 shows a magnet of width $ in the circumferential direction. It is shown without curvature,
to clarify the analysis. Also shown is a wave of the current-density that would be induced by a
harmonic field component rotating relative to the magnet, if the magnet were a complete unbroken ring.
This wave is labelled J, and it can be calculated using the methods of earlier sections. The diagram is
drawn for one instant in time, and end-effects are neglected so that the model is 2-dimensional and
invariant in the axial (z) direction. The eddy-currents are not necessarily assumed to be resistance-
limited.
The position of the magnet relative to the J-wave can be defined as the phase > of the centre-line of the
magnet relative to the J-wave: thus
21 22
> ' and $ ' 22 21 (2.739)
2
or
$ $
21 ' > ; 22 ' > . (2.740)
2 2
The angles 21, 22, > and $ are all measured here in electrical degrees or radians relative to the harmonic
wavelength 8. Thus if $m is the magnet width in actual mechanical radians, $ = $m × 2B/8.
If the J-wave is represented as J1 sin 2, the integral
22
J1 sin 2 d 2 (2.741)
21

represents the net current that would flow in a section of a complete ring of magnet between the angles
21 and 22. This integral is not in general zero. However, the net current in a magnet block of finite
width $ must be zero, because there is no connection by which it can be routed to a return path in
another magnet carrying current in the opposite direction. In fact the net current must return through
the magnet block itself. The distribution of return current-density across the cross-section of the
magnet block is not known a priori; but it can be supposed to be uniform on the grounds that any non-
uniformity would imply that there were loops in the return current path containing induced EMFs,
when the induced EMFs have in fact already been completely accounted for by the solution of the field
equations leading to the J-wave.48
48
In finite-element programs the imposition of zero net current is achieved by a related process (often without proof), in which
the end-faces of the conducting regions are shorted together.
Brushless permanent-magnet machines Page 2.211

Thus with a uniformly-distributed return current density J0 we have


22
( J0 J1 sin 2 ) d 2 ' 0 , (2.742)
21

from which
cos 22 cos 21 sin ($ / 2)
J0 ' J1 ' J1 sin > . (2.743)
22 21 $/2
2
The loss in the magnet block will be proportional to (J0 + J1 sin 2) integrated over $ : thus if q = J0/J1
22
1 $ 1
7 ' J12 (q sin 2 )2 d2 ' J12 ( q 2 )$ 4 q sin sin > cos 2 > sin $ . (2.744)
21
2 2 2

When $ ' 360kE (for integer k), the magnet width is an integral multiple of the wavelength 8 of the
harmonic field component, and eqn. (2.742) is automatically satisfied, with J0 = 0, regardless of the phase
or position of the J-wave relative to the magnet. The harmonic loss in a magnet whose width is equal
to an integral multiple of the harmonic wavelength is therefore the same as would be calculated in a
segment of a complete ring according to the theory described earlier using Bessel functions. The
integral 7 in eqn. (2.744) then degenerates to

1
70 ' J12 $. (2.745)
2

and so it becomes possible to characterize the effect of circumferential segmentation of the magnets by
the loss reduction factor

7 2 1 $ 1
' (q 2 )$ 4 q sin sin > cos 2 > sin $ . (2.746)
70 $ 2 2 2

One or two examples will help to illustrate the effect. In Fig. 2.205 the magnet width is $ = 90E relative
to the harmonic wavelength, and its phase position is > = 80E. Thus 21 = 80 90/2 = 35E and 22 = 80 + 45/2
= 125E. Eqn. (2.743) gives q = J0/J1 = 0@88664, and then with $ = B/2 radians eqn. (2.746) gives the loss
reduction factor 7/70 = 0@0207. While this seems a substantial reduction, it needs further comment.
First, this value applies only at one instant corresponding to the phase angle > = 80E. As the cycle
progresses and the harmonic wave moves relative to the magnet, 7/70 varies and there are instants
when it is equal to 1. Therefore we can expect the average value of the loss reduction factor will be
markedly greater than 0@0207.
Secondly, a magnet width of $ = 90E means that the width of the magnet segment is one-quarter of the
harmonic wavelength. If we consider, say, the 5th space-harmonic of the stator winding, it has a
th
wavelength at fundamental frequency equal to 1/5 of a pole-pitch. Consequently $ = 90E for this
harmonic implies that the magnet is segmented into 20 blocks per pole — rather a large number. A
magnet with 180E arc not segmented would have $ = 5 × 90 ' 450E, and it can be verified from eqn. (2.746)
that the loss reduction factor does not fall much below 1. A magnet with 80%pole-arc (144E) would have
$ = 360E relative to the 5th winding space-harmonic, and in this case the loss reduction factor would be
1. Of course the magnet “region” is only 80% filled with magnet, so there is a reduction of 0@8 relative
to a full-ring magnet; but there is no reduction due to the “residual current suppression” effect.
The example with $ = 90E and > = 80E is shown graphically in Fig. 206. The upper graph labelled J1 is
the “original” current distribution and the lower graph labelled J is the net current distribution with
“residual suppression”. Also shown are the squares of these two graphs. At this phase position the
reduction in squared current-density is very noticeable.
Fig. 207 shows the same magnet and the same harmonic wave at a different phase position, > = 0. At
this instant the net current in the magnet block is naturally zero; J0 is not necessary and is zero.
Page 2.212 SPEED’s Electric Machines

Fig. 2.206 Example of current distribution with “residual suppression”

Fig. 2.207 An instant when the residual suppression current is zero

The arrows in Figs. 206 and 207 show schematically the direction and magnitude of the current density
in the magnet block, “seen from above”. At the instant > = 0 in Fig. 207 the distribution is momentarily
symmetrical with positive current in one half of the block, and negative in the other half. But at the
instant > = 80E in Fig. 206, it is asymmetrical. When $ is an integral multiple of the harmonic
wavelength, the distribution is similar to the one in Fig. 207, except that as > advances the arrows at one
end leave the block and re-appear at the other end, while J0 remains zero at all times.
The loss reduction factor eqn. (2.746) and the diagrams in Figs. 206 and 207 are specific to one field
harmonic. This means that simplified methods of allowing for segmentation (such as equivalent
resistivity) are hardly likely to be successful. Moreover, eqn. (2.746) cannot be used to introduce the
effect of segmentation into the results of a 2-dimensional finite-element calculation.
Brushless permanent-magnet machines Page 2.213

Average loss reduction factor over a complete harmonic cycle — The loss reduction factor 7/7o
in eqn. (2.746) must be integrated over one harmonic cycle to get the average value. With q expressed
in terms of > from eqn. (2.743) and a = 2/$ × sin ($/2), 7/7o can be expressed as

7 sin $ 2 $
u ' ' 1 $a 2 4 a sin sin $ sin2 > (2.747)
70 $ $ 2

Then the average value of 7/7o is given by

2B 2
1 sin($ / 2)
U ' u (> ) d> ' 1 . (2.748)
2B 0 $/2
..a remarkably simple function.
For the example considered earlier with $ ' 90E or B/2, we get U = 0@18943. When $ ' 2B or a multiple
thereof, U = 1 as expected.

Simplified analysis of double segmentation

L
n

m 

(a) (b)

Fig. 2.208 Eddy-current break-up

Fig. 208 shows one eddy-current pole or loop in a magnet of axial length L. In (b) the magnet is divided
into n × m segments. Assume that the driving E-field is unaffected by the segmentation, so that only
the effect on the resistance of the eddy-current path needs to be considered.
For eddy-currents in the monolithic magnet, the path length is of the order 2(L J), where J is the pole-
pitch, which is taken to be the pole-pitch of the exciting space-harmonic. Let the EMF driving eddy-
currents in this loop be represented by E. In one element of the segmented magnet, the driving EMF will
be of the order of E/mn and the path length will be of the order of 2(Lstk/n J/m). Therefore the total
losses will be changed in the ratio

( E / mn ) 2
mn
Wsegmented 2 (L/n J/m ) L J
' ' . (2.749)
Wmonolithic E 2 mL nJ
2 (L J)

For example if L is approximately equal to the pole-pitch J,


Wsegmented 2
' . (2.750)
Wmonolithic m n
Thus if the magnet is segmented in both directions with m ' n ' 10, the losses should be reduced by a
factor of 2/(10 10) ' 0@1.
This simple example provides an introduction to the more detailed methods for axial segmentation,
which is described next.
Page 2.214 SPEED’s Electric Machines

End effect — segmentation in the axial direction

Fig. 2.209 Approximate calculation of end-effect

Fig. 209 shows half a block of magnet in rectangular coordinates. The direction of rotation is in the y
direction and z is the axial coordinate. The axial length of the block is 2h and its width in the
circumferential direction is 2b. This width can be taken as the wavelength of the exciting harmonic.
Consider a filamentary loop of eddy-current. If the block was infinitely long in the axial direction, the
current density in the filament would be Jz0 = FE = jTFA, where A is the solution of the 2-dimensional
field at the circumferential position y.
The cut edge of the magnet at z = h forces all the current to veer into the circumferential direction and
rejoin its return path. In Fig. 209 the current is assumed to be symmetrical about the centre-line of the
block. It is further assumed that the current in the filament )y turns abruptly into the circumferential
direction and flows in a circumferential filament of width )z such that
J y )z ' J z )y . (2.751)
The filament widths are further assumed to be related by
)y
' tan ( ' m , (2.752)
)z

where ( is an arbitrary angle defining the slope of the “break” line shown dashed in Fig. 209. Hence
Jy ' mJz. (2.753)
Considering just one quadrant of the loop, the driving EMF can be identified as Ed. In the case where
the magnet is infinitely long in the z direction we have Ed = DJz0 h, where D = 1/F is the resistivity.
When the length is finite, the filament acquires a second segment of length y in the y-direction, while
the segment in the z-direction is shortened from h to d, where

b y
d ' h a ' h . (2.754)
m

The shortening of the axial length reduces the EMF proportionally, while the addition of the second
segment increases the total resistance. This is expressed by the equation

Ed ' D Jzd Jyy (2.755)


which can be expanded as
Brushless permanent-magnet machines Page 2.215

y my m 2y
E ' DJz 1 m ' DJz 1 ' DJz 1 ,
d b y m h (b y) (2.756)
h
m
and so
E
Jz ' with Jy ' mJz.
m 2y (2.757)
D 1
m h (b y)

The current-density Jz is decreased by the factor in brackets. However, the losses are proportional to

w ' D (J z2 d ) y J y2 y ) z ) (2.758)
and this can be expanded as

)y
w ' D J z2 d ) y m 2 y ) z ' D J z2 (m h (b y) m 2y (2.759)
m

Substituting for Jz from eqn. (2.757), and simplifying, we get

E2 [ m h (b y) ] 2
w ' )y. (2.760)
D m [ m h (b y) m 2 y ]
If there is no end-effect we have simply

E2
w0 ' h )y. (2.761)
D
Hence it is possible to define a loss reduction factor 8 for the filament at y :

w [m h (b y)] 2
8(y) ' ' . (2.762)
w0 mh [ mh (b y) m 2 y ]

If we normalize h and y to b by writing h = h/b and y = y/b, this can be written

w [m h (1 y)] 2
8(y) ' ' . (2.763)
w0 mh [ mh (1 y) m 2 y ]

If ( = 45E, m = 1 and the expression simplifies further:

w [h (1 y)] 2
8(y) ' ' . (2.764)
w0 h[h 1 2y]
A magnet which is long in the axial direction has h >> 1, so that 8(y) ÷ 1 for all values of y. For such long
magnets the value of ( is not critical: see Fig. 210. However, in short magnets ( will have a minimum
value. For example a “square” magnet has h = 1 or h = b. In this case ( must not be less than 45E, giving
m > 1. A magnet for which h = 0@5 (or h = b/2) restricts ( to values greater than arc tan(2) = 63@4E. These
limits of course have no physical basis, but are constraints imposed by the modelling assumptions.
The loss reduction factor for the whole block is obtained as the average of 8(y) for all the filaments, or
1
1
7 ' 8( y ) dy . (2.765)
1 0

This can be integrated numerically or formally, giving


(A C B 1 / C) ln (1 C) (B 1/2) C 1
7 (h) ' (2.766)
2
C k (k 1)
2 2
where k = mh, A = (k 1) ; B = 2 (k 1) ; and C = (1 + m )/(k 1).
Page 2.216 SPEED’s Electric Machines

Fig. 2.210 “Long” magnet with two values of (, 45E and 75E

Fig. 2.211 End-effect loss reduction factor 7(h) with k = 5


1 — SPEED method; 2 — Russell & Norsworthy

Fig. 211 shows examples of 7(h) calculated by eqn. (2.766) for various values of the magnet length/width
ratio, h. The parameter k is arbitrary and should be adjusted to match test or finite-element data.
The overall factor 7 in eqn. (2.766) is so far simply an end-effect factor for the losses in magnets of
different length/width ratio h. Now suppose we start with a full-length magnet with a certain value of
length/width ratio h1 and an end-effect factor 71. If this magnet is divided into n segments in the axial
direction, the end-effect factor for each segment becomes 7n, calculated with hn = h1/n. Although 7n
operates on only (1/n) of the losses, there are now n segments, so the overall effect is that the end-effect
factor is 7n for the whole array, instead of 71.
As an example, suppose we start with a full-length magnet having h = 3, and divide it into 3 segments.
The end-effect factor with ( = 60E is 0@699 for the undivided magnet, and 0@2935 for the divided magnet.
Thus the division into 3 segments reduces the loss by a factor of 0@2935/0@699 = 0@419.
It should be said that this end-effect analysis is a rough-and-ready estimate, and not “analytical” in the
sense of conforming with Maxwell's equations. (See also p. 209).
Brushless permanent-magnet machines Page 2.217

Russell and Norsworthy's method

Fig. 2.212 Russell and Norsworthy's eddy-current flow-lines

Russell and Norsworthy [69] have already been mentioned on p. 224 in connection with the calculation
of stator can losses in a screened-rotor induction motor. Their analysis included the effect of finite
axial length. It also included the effect of overhangs in the axial direction (that is, where the can is
longer than the axial length of the stack). The overhangs were allowed to have different thicknesses and
conductivities from those of the central section of the can.
Fig. 212 shows the essential concept in Russell and Norsworthy's finite-length analysis. They solved a
form of Laplace's equation in the plane of a rolled-out can in cartesian coordinates, to establish flow-
lines for the eddy-currents in the can. Fig. 212 shows one quadrant of this model. Russell and
Norsworthy measured these flow-lines (using a pair of point-contact probes with a high-impedance
voltmeter, under conditions of single-phase excitation), and showed good agreement with the calculated
lines. The solution included the current-density, from which the loss is calculated. They presented the
result in terms of a coefficient Ks to be used with the power loss calculated by eqn. (2.798) on p. 225.
Russell and Norsworthy's formula for Ks is
pl
tanh
a
Ks ' 1 (2.767)
pl
(1 8)
a
where l is the half-length of the sleeve (i.e., Lstk/2), a is the mean radius of the sleeve, and p is the
number of pole-pairs. For an open-ended can,

pl pl
8 ' tanh tanh " (2.768)
a a

where " is the per-unit overhang, that is, the overhang length at one end divided by l. If the overhang
lengths are different, Ks can be calculated separately for the two ends and then averaged. For a can
with zero-resistance end-rings, tanh ["pl/a] is simply replaced by its inverse, coth ["pl/a].
Fig. 211 shows Ks compared with the loss reduction factor calculated by eqn. (2.766) for various values
of the magnet length/width ratio, h. The equivalence is based on

pl B h B
' ' h.
a 2 b 2

The difference between curves 1 and 2 in Fig. 211 is partly due to the fact that the SPEED analysis
explicitly allows for the reduction of driving EMF in the eddy-current loops. Although Russell and
Norsworthy did not make this allowance, they nevertheless achieved good agreement with test data,
and their work has since been used elsewhere with satisfactory results. The Russell and Norsworthy
result is based on the fundamental harmonic of the main field (2-pole in their test), using a formal
solution of Laplace's equation for the flow lines, whereas the SPEED method is relatively arbitrary.
Page 2.218 SPEED’s Electric Machines

Alternative analysis of segmented magnets


When the eddy-currents in the magnets are resistance-limited, they can be postulated to flow in
predefined physical circuits. For a single magnet block, the simplest form of these circuits is a set of
nested loops as shown in Fig. 210. The resistance of each filamentary loop can be calculated from its
dimensions, while the EMF induced in it can be calculated from the 2-dimensional field solution
developed earlier. In terms of this solution, the magnets will be found in a "non-conducting" region
since, by the definition of "resistance-limited", the eddy-currents in the magnets have negligible effect
on the magnetic field. By the same token, the inductance of the filamentary loops can be ignored.
Although this model of the eddy-current paths in the magnets is crude and arbitrary, it deals directly
with the effects of segmentation in both directions and it will be justified by its simplicity (and
ultimately by comparison with test data). It has already been pointed out that modelling the 3-
dimensional eddy-current paths by infinite series of analytic functions such as eqn. (2.738) has not
delivered working calculation procedures for the designer. It is possible that in future refinements, the
shapes of the eddy-current paths could be made more “analytical”, and that the inductances and mutual
inductances of the filamentary loops could be calculated and introduced into the calculation.
To the extent that the "fixed circuit" eddy-current model neglects the inductive effects of the eddy-
currents, it requires a test to determine whether or not the eddy-currents are indeed resistance-limited
for a particular time- or space-harmonic of the exciting field. This test is constructed in stages. First,
the complete field solution is computed (for each time- and space-harmonic) with a complete ring of
magnet material concentric with the rotor sleeve and the rotor body. The magnet simply becomes a
third conducting region in a series of concentric conducting regions, and the phase of the induced eddy-
currents indicates immediately whether the resistance-limited assumption is valid. This calculation
is the only calculation required when the magnet is physically in the form of a continuous ring, since
it calculates the eddy-currents without any a priori assumption that they are resistance-limited. If they
are, it follows that they will be "even more resistance-limited" if the magnets are segmented in either
direction. No further test is necessary in that case, and the fixed-circuit model can be applied to
calculate the eddy-currents in the segmented magnets. An intermediate case arises if the magnets are
segmented but the eddy-currents in the continuous ring are not resistance-limited. In that case a
further test is required to determine whether they will be resistance-limited in the segments. This test
is essentially a skin-depth test applied to each segment in the 2- and z-directions according to the
"planar" theory of eddy-currents developed in Lammeraner and Štafl [93]; (see also Kirtley [60]). When
this test indicates that the eddy-currents in the magnets are not resistance-limited, the analytical
method is not quite defeated because it is possible to introduce a reactive element from the "planar"
theory into the eddy-current impedance. In practice the likelihood of fully inductance-limited eddy-
currents in segmented magnets is very small, even with the most conductive magnet materials; (see
Irenji, [77,78]).
Brushless permanent-magnet machines Page 2.219

Fig. 2.213 Filamentary “coil” inside a conductive cylindrical region

Consider a filamentary coil embedded in a conductive cylindrical region representing a segmented


magnet, Fig. 213. It lies on a cylindrical surface of radius r and its radial thickness t is much less than
r. Its straight sides are located at the angles 21 and 22, and they subtend an angle $ at the axis, so that
$ ' 22 21 . (2.770)
The straight sides are, in effect, strips of length h, width wb, and thickness t, giving a cross-sectional
area wbt for current flow. The curved sides are strips of length r$, width wa and cross-sectional area wat.
The resistance of the loop is

2h 2r$ 2 h r$ (2.771)
R ' ' .
F wb t F wa t F t wb wa

The flux N linking the loop is given in general by the line-integral of vector potential A around the
circumference. The whole circumference is 2(h + r$), but if A has only a z-component the integral
degenerates to
N ' h ( A2 A1 ) (2.772)
j(Tt p2)
where A1 is the value at 21 and A2 is the value at 22. If we reintroduce the common factor e ,
j (T t p 22) j (T t p 21)
N ' h A (r) e e (2.773)

If we write 20 = (21 + 22)/2, the 2-coordinate of the centres of the curved sides, this reduces to

$ j (T t p 20)
N ' h A( r ) sin p e (2.774)
2

The EMF induced in the loop is dN/dt or

$ j (T t p 20)
e ' j T h A( r ) sin p e (2.775)
2
and the (peak) phasor value is

$
E ' j T h A( r ) sin p (2.776)
2
The power loss is now calculated as

1 E E( T2 h 2 sin2 ( p $ / 2 ) AA(
W ' Re ' . (2.777)
2 R 2R
Page 2.220 SPEED’s Electric Machines

2.32 SLOT RIPPLE


Slot ripple (also known as tooth ripple) refers to the modulation of the airgap flux-density distribution
(Bgap) caused by the stator slot-openings. The modulation causes surface losses in the rotor, due to eddy-
currents which are excited by the rotation of the rotor past the stationary “dips” in the flux wave.
Calculation of the slot modulation and its effects is an old topic in electrical machine theory; Carter's
coefficient is a well- known example of it [51,87,88]. Freeman [52] used a sophisticated procedure
involving elliptic integrals to determine the harmonics in the flux-density. The method of Zhu and
Howe described below is a modern development adapted for brushless PM machines. Of course the
finite-element method is also a powerful and flexible method for calculating the slot modulation.
The simplest method for calculating slot-ripple loss in a thin sleeve is “flux-dip-sweeping ”.49 This
method uses the calculated magnetostatic slot-modulation in the Bgap distribution. It is closely related
to the calculation of losses in stator cans due to the rotation of the main field, and is developed from that
starting point in detail on p. 224ff. Its main limitation is the assumption that the eddy-currents are
resistance-limited. It is also not suited for calculating magnet loss, because the magnets are generally
much thicker than a metallic sleeve and the radial variation of the slot-modulation is in general
significant. PC-BDC can calculate the losses in both rotor and stator cans by flux-dip-sweeping.
The eddy-current reaction (skin effect) may be important in high-speed PM machines. When it is, the
thickness of the conductive components must be fully recognized. Lawrenson et al [53] developed the
analysis of the surface losses to include the reaction effect of the eddy-currents, pointing out the
limitation of previous works which dismissed or ignored it. They used an equivalent AC current-sheet
on the bore of the stator, whose wavelength was equal to the tooth-pitch, and whose magnitude was just
sufficient to establish a flux variation equal to the variation calculated without eddy-currents or
rotation by one of the classical methods based on conformal transformation (for example, Gibbs [89]).
The AC current-sheet was then used to calculate the resulting field distribution in a simple 3-region
cartesian solution of the complex diffusion equation, as on p. 192ff.50 Lawrenson limited his analysis
to the fundamental slot-harmonic, allowing for saturation by means of saturated permeabilities in an
otherwise linear analysis. Oberretl considered this further [54].
Lawrenson's method is implemented in PC-BDC by first determining the slot modulation in the Bgap
distribution by the method of Weber (Heller and Hamata [98]) extended by Zhu and Howe (see below).
The modulated Bgap distribution is Fourier-analyzed to determine its harmonic components.
th
Transformed into rotor coordinates, the typical n harmonic in the Bgap distribution is of the form
Bn cos n ( 2 Tt) (2.778)
which represents a field with n pole-pairs rotating past the rotor at the relative angular velocity T, the
synchronous speed. The fundamental slot-ripple harmonic has n equal to the number of slots, and
generally only the fundamental (and possibly the second harmonic) is significant. For each harmonic
current-sheet Kn sin n2, Kn can be determined from the equations in Hughes and Miller [13], examples
of which are given on pp. 185 and 191. From this point, the calculation of the slot-ripple losses and the
slot-ripple field proceeds in the same way as for stator MMF and time harmonics, as described earlier.
Slot-ripple is related to the concept of permeance harmonics. As the rotor rotates past the stator slots,
the overall permeance is modulated at the slot-passing frequency, causing the main flux to pulsate at
this frequency. However, permeance is a lumped-circuit concept rather than a distributed-field
concept, and so the calculation of permeance variations or flux variations is useful only in the context
of a lumped-parameter magnetic circuit model. This approach is used later for the IPM. For surface-
magnet machines, where the magnets (and any cans) are located in the distributed and time-varying
field, a field-theory approach is more appropriate.
49
The term “flux-dip-sweeping” is meant to describe the movement of the rotor as it sweeps past fixed dips in the flux.
50
When the skin-depth is small enough, it is even possible to treat the magnets as infinitely thick, on the grounds that the skin
effect (and also the natural $(r) attenuation) tends to limit the eddy-current to a layer near the surface. Nothing happens beneath
that layer, so its depth ceases to affect the losses.
In Lawrenson's analysis of tooth-ripple losses, the solid steel rotor pole was in fact assumed to be infinitely thick ab initio. The
analysis of Stoll and Hammond [55] starts with a conductive slab of finite thickness and develops definite criteria for treating the
slab as being infinitely thick. Fortunately in the present context, the assumption of infinitely thick magnets (or rotor hub) is not
necessary, since the complete field solution is available.
Brushless permanent-magnet machines Page 2.221

Determination of the slot-modulation in PM machines


The first step is to determine the modulation in the flux-density distribution as a function of the
machine dimensions. For surface-magnet PM machines the effective airgap includes the magnet length
LM/μrec : thus

LM
gN ' kC g (2.779)
μrec

where kC is Carter's coefficient. gN is generally much larger than the physical airgap g. Furthermore,
PM machines are often designed with a small number of slots/pole, so that the ratio of slot-opening s
to slot-pitch J is often relatively small. It is common to find that the ratios s/g and s/J are at the
extremes of the classical analysis, necessitating a revised field analysis to be sure of a method that is
appropriate for PM machines.51 Reproduced here is the method of Zhu [62], who used the following form
of the variation or modulation in the Bgap wave in common with Heller and Hamata [98]:

B(r,") ' Bm [ 1 $(r) ( 1 cos B" /"s ) ] , 0 < * " * < "s ;
(2.780)
B(r,") ' Bm , "s < * " * < (s / 2 .

where " is the angle measured from the slot centre-line, (s is the slot-pitch angle corresponding to the
linear tooth-pitch J, and Bm is the maximum flux-density that would be obtained without slotting. Also
"s ' 0@8 "o ' 1@6 × "o / 2 (2.781)

where "o is the slot-opening angle s/rS, and rS is the stator bore radius. Thus the angle "s is extended
60% beyond the edges of the slot ("o/2) to account for the observed fact that the slot modulation “dip”
is wider than the slot opening. $(r) is half the maximum relative “dip” in the Bgap wave at the centre-
line of the slot, so if Bd is the actual maximum dip, we have $ = Bd/Bm × ½.
Whereas Heller and Hamata provide a graph of $ vs. s/g without derivation at a single radius (the rotor
surface radius), Zhu re-calculates $(r) as a function of r from first-principles using conformal
transformation, and validates the calculation with finite-elements. (The same has been done with the
equations reproduced below). It is shown that $(r) varies across the airgap and right through the
magnet. An example is shown in Fig. 214. Zhu's equation for $(r) is

1 1
$(r) ' 1 (2.782)
2 1 (1 v 2)/u 2
where v is the solution of

B 1 a2 v2 v uv (2.783)
y ' ln u arc tan
s 2 a 2
v 2
v a 2
v2
with

2 gN
u ' and a2 ' 1 u 2. (2.784)
s

and
y ' r rS gN interior rotor (2.785)
y ' rS r gN exterior rotor (2.786)
For surface-magnet machines Bm is taken as the mean flux-density at the rotor surface, calculated by
PC-BDC. Then B(r,") is determined from eqn. (2.780).

51
Freeman's method [52] could also be used. The approach described here is quicker but more approximate, although it can easily
be checked (or even substituted) by finite-element analysis which was not available in 1962.
Page 2.222 SPEED’s Electric Machines

Fig. 2.214 Flux-density profile calculated by eqn. (780)

Other methods of estimating the slot-modulation

Fig. 2.215 Estimate of slot-ripple amplitude (Robinson et al [70])

Robinson et al [70] began by estimating Bm as the fraction (kC 1) of the local amplitude Bg of the main
field, kC being Carter's coefficient; see Fig. 215. They applied a second factor of 0@5 to account for the
assumption that the main field Bg itself is sinusoidal. They did not attempt to determine individual
harmonics of the field. Instead they quoted an empirical correction factor of 2@6 × bt/bs to account for
the non-sinusoidal shape of the slot-modulation. The overall effect is to replace Bm in eqn. (2.798) by

kC 1
Bm × × 2@6 bt / bs . (2.787)
2

where Bm is the peak value of the main field (assumed sinusoidal). When considering only the local
variation of the field due to slotting, the amplitude of the slot-modulation (Fig. 215) would be

Bm ' Bg × ( kC 1) × 2@6 bt / bs . (2.788)

This last formula is useful for comparing Robinson et al with other approaches (see below).
Brushless permanent-magnet machines Page 2.223

Takahashi [71,72] gives a formula

1 u 2 ! 2u
Bo ' kC Bg (2.789)
2(1 u2 )
2
where u = (s /2g) + /[1 + (s /2g) ]; s is the slot opening; g is the airgap length; kC is Carter’s coefficient
2
1/[1 ! (s/g) /(5 + s/g) × g/J]; and J is the slot-pitch. Bg is described as the “average flux-density over the
airgap”. Values of Bo obtained from eqn. (2.789) can be found in Fig. 216. These values are lower than
the finite-element calculation and do not agree with Figs. 4 and 5 in Takahashi [71,72].
Earlier versions of PC-BDC used the formula (now obsolete)
Bd h ! g
' (2.790)
Bg[oc] h LM / μrec

2 2
where h = /(g + (s/2) ), g is the airgap length, s is the slot-opening, LM is the magnet length, μrec is the
[oc]
recoil permeability of the magnet, and Bg is the open-circuit value of the unmodulated airgap flux-
density shown as Bg in Fig. 218.

Fig. 2.216 Comparison of slot-ripple amplitudes (at mid-gap)

Comparison of slot-modulation "dips"


Fig. 216 shows calculations of the slot-modulation “dip” Bd normalized to the flat-topped airgap flux-
density obtained without slotting. The motor is similar to Takahashi's high-speed motor. It has a stator
bore of 40 mm, an an airgap length g = 2 mm, with a rotor sleeve of 1 mm thickness. The slot-opening
in the cross-section in Fig. 216 is 4 mm, and it was varied from 0@5 mm to 6 mm.
Only Zhu's method comes anywhere near to predicting dips consistent with finite-element analysis. For
this reason Zhu's method [eqn. (2.782)] is preferred as an analytical method for determining the size of
the dips, together with Heller and Hamata's formula (780) for the shape of the dips, (Fig. 218b). This
combination is used in PC-BDC. Moreover, in PC-BDC the MATCHFE function can be used to adjust the
depth Bd and the width "s to match the dips obtained from a Bgap finite-element calculation.
It should be noted that the “early” formulas of Robinson et al [70] and Russell & Norsworthy [69] were
developed for induction motors with narrow airgaps, and they are not suitable for use with surface-
magnet PM brushless motors. Moreover, the old PC-BDC method [eqn. (2.790)] shown in Fig. 216 was
originally intended to apply at the centre of the magnet and is therefore not really suitable for the
sleeve, especially when the magnet is thick.
Page 2.224 SPEED’s Electric Machines

2.33 “FLUX-DIP-SWEEPING” ANALYSIS OF LOSSES IN THIN SLEEVE

Fig. 2.217 Slot-ripple analysis with a thin conductive sleeve

Fig. 217 shows the parameters of a simple analysis of slot-ripple in a thin conductive sleeve. Examples
are Robinson et al [70], Russell & Norsworthy [69], and Takahashi et al [71,72]. Here the method will be
derived from first principles because of some uncertainties in the original papers.
The “flux-dip-sweeping” analysis is based on the assumption of resistance-limited eddy-currents in the
sleeve, so the eddy-current reaction on the field is neglected. Moreover there is no variation of flux-
density through the thickness of the sleeve. Cartesian coordinates are used for simplicity, since the
wavelength 8 of the slot-ripple is assumed to be small compared with the radius.
The central plane of the rolled-out conductive sleeve is shown by a dotted line in Fig. 217. This line
represents a constant radius r = D/2, where D is the diameter. The linear coordinate x is given by
x ' r2, (2.791)
where 2 is angular distance in the circumferential direction. Along this line the flux-density is assumed
to have a sinusoidal variation given by
By ' Bm sin p 2 , (2.792)
where p is the number of pole-pairs. Robinson et al and Russell & Norsworthy both began by deriving
formulas for the loss in a stator sleeve, and in this case Bm is the peak airgap flux-density of the main
flux, assumed sinusoidal, while p is the number of fundamental pole-pairs. Only the y-component is
shown because it is assumed to be the only component that generates eddy-currents in the thin sleeve.
The relative linear velocity between the field and the stator sleeve at diameter D is given by

2BN D
v ' Tr ' × ; (2.793)
60 2
where N is the speed in rev/min and T is the angular velocity in mechanical rad/s. The motion-induced
EMF per metre of axial length (normal to the page) is given by
Ez ' v By ' v Bm sin p 2 , (2.794)
and the current-density (assuming resistance-limited eddy-currents) is

Jz ' F Ez ' F v Bm sin p 2 . (2.795)


where F is the conductivity of the sleeve in S/m. The power loss per unit volume is

D Jz2 ' F v 2 Bm2 sin 2 p 2 , (2.796)


2
where D = 1/F is the resistivity in ohm-m. The sine function has a time-averaged value of 0@5 at every
point in the sleeve regardless of its position. Therefore the total power loss in the sleeve is
Brushless permanent-magnet machines Page 2.225

2 2
1 v Bm (2.797)
P ' D J z2 V ' × 2BrtL W,
2 D
where V = 2BrtL is the volume of the sleeve and L is its axial length. End-effects are ignored, on the
grounds that 8 is generally much shorter than L. This equation can be written in terms of the speed
N in rpm, and the sleeve diameter D = 2r, if we substitute from eqn. (2.793): thus
2 2 3
B3 Bm N D t L (2.798)
P ' W.
2 3600 D

This equation is identical to the formulas presented by Robinson et al [70] and by Russell & Norsworthy
[69], for losses induced in a sleeve fitted to the stator. In both cases Bm was the peak amplitude of the
main airgap field, not the slot-ripple.
It is interesting to note that the loss P is independent of the number of poles. This is because all
elements of EMF are induced by relative motion at the common velocity v, which is independent of the
number of poles or indeed of any aspect of the spatial variation of the field; see eqn. (2.794). Still, the
pattern of eddy-currents induced in the conductive sleeve is sinusoidal and has 2p poles.

Rotor sleeve losses


Robinson et al [70] deduced a formula for rotor sleeve loss from eqn. (2.798), essentially by exchanging
the source of excitation from the main field to the slot-ripple modulation of the main field. Similar
processes can be found in Russell & Norsworthy [69] and in Takahashi et al [71.72]. The dimensions
and resistivity in eqn. (2.798) are substituted by those of the rotor sleeve, while N remains the same, on
the grounds that the relative velocity between the slot-ripple field and the rotor sleeve is that same as
the relative velocity between the main field and the stator. The number of slots does not appear, for the
same reason that the number of main-field poles does not appear in eqn. (2.798), as discussed earlier.
The main difficulty in using eqn. (2.798) is the determination of the effective value of Bm for the slot-
modulation. In general the slot modulation is not sinusoidal and it should be treated harmonic-by-
harmonic if the formula is to be used according to the assumptions on which it was derived.
Takahashi et al [71,72] published a similar analysis which seems to have been derived independently.
They do not refer to Robinson et al [70] or to Russell & Norsworthy [69]. They present an equation

B3
P ' B 2N 2D 3tL W, (2.799)
1800 D o
which gives four times the loss of eqn. (2.798) when Bo is given the same value as Bm. Bo is called the
“amplitude of the slot ripple”, so it appears to have the same meaning as Bm. To investigate this
further, eqn. (2.799) in the original Japanese [Obaraki et al 1991] is derived from an equation

2
P Bm T (2.800)
Weff ' J3 t L
2D B

where P is the number of poles. This equation can be shown to be identical to eqn. (2.798) if

2BN P BD
T ' × and J ' . (2.801)
60 2 P

In other words, T is the electrical radian frequency 2B f and J is the pole-pitch. Next, Takahashi
substitutes P by Ns, the number of slots; J by BD/Ns; T by 2BN/60 × Ns; and Bm by Bo, giving eqn. (2.799).
However, it appears that P should be substituted by 2Ns instead of Ns, and J should be substituted by
BD/2Ns instead of BD/Ns. If this is done, the result is eqn. (2.798). Even when these “corrections” are
applied to Takahashi's eqn. (2.799), uncertainties remain. Takahashi's slot-modulation is about half the
finite-element value (Fig. 216), thus appearing to compensate for the apparent error of 4 in eqn. (2.799);
but it remains unclear how the formulas should be used.
Page 2.226 SPEED’s Electric Machines

Fig. 2.218 Slot modulation of the open-circuit airgap flux distribution

In PC-BDC the sleeve loss equation (798) is rewritten in terms of the RMS value of the slot-ripple field:

Brms2 N 2 D 3 t L
W ' B3 × (2.802)
3600 D

where Brms is determined from the dips in the flux-density wave in the rotor sleeve, caused by the slot
modulation. Brms would be equal to Bm//2 for a sinusoidal slot-ripple wave of peak value Bm. There is
only one application of eqn. (2.802), so it is deemed to include all harmonics.
In the old ("Cog2") method, a in Fig. 218, each dip has the shape of a half-sinewave of peak value Bd. The
width of the dip is $ radians, corresponding to the slot-opening augmented by 2 g to allow for the fact
that the dip is somewhat wider than the slot-opening s. The period of the slot-modulation is equal to the
slot-pitch 8 ' 2B/Ns. Assuming an uninterrupted series of dips, the RMS value of the modulation is

Bd $
Brms ' × . (2.803)
2 8

If the airgap flux distribution is substantially flat, a further scale factor /($M/180) may be applied to
account for the finite magnet arc $M, measured in electrical degrees. If it is sinusoidal, Brms should be
calculated from Bd at the peak, and then multiplied by a further factor 1//2. In general it can be scaled
by the ratio of the RMS value of the unslotted Bgap waveform to the peak value. It can also be calculated
as the RMS value of the difference between the modulated and unmodulated Bgap distributions (Fig. 218).
An alternative shape can be postulated for the dip, as in Heller and Hamata's formula (780). This is
shown as b in Fig. 218. In this case the RMS value of the dip is calculated as

2 2
2 Bd "s
" Bd 3 "s (2.804)
Brms ' 1 cos B d" ' × .
8 4 0
"s 2 2 8

PC-IMD — For induction motors the amplitude of the dip Bd is given by Heller and Hamata [98] and by
Alger [97]. The RMS value is obtained by multiplying Bd by the factor /[(s + 2g)/gs], to allow for the fact
that the dip is wider than the slot-opening s, as explained in Hendershot and Miller [2]. A second factor
1//2 allows for the sinusoidal variation of the main flux, which modulates the value of the dips, and a
further 1//2 to allow for the half-sinewave shape of the dip. Thus

Bd s 2g
Brms ' × (2.805)
2 gs

where gs is the slot-pitch and g is the airgap length.


Brushless permanent-magnet machines Page 2.227

2.34 HARMONIC LOSSES IN THE INTERIOR PERMANENT MAGNET MACHINE (IPM)

Losses caused by time-harmonics in the stator current


The IPM presents a problem in that the magnet region inside the rotor is not a plain cylinder and
therefore it does not conform to the solutions of the diffusion equation or Laplace's equation. However,
an approximate estimate of harmonic losses can be made with the aid of the equivalent-circuit model
and the frequency-dependent synchronous reactance Ld(jT), which is given by eqn. (2.820) on p. 230.
We can assume that Ld(jT) is known, because we have expressions for Td0NN [eqn. (2.860) on p. 238], TdNN
[eqn. (2.822) on p. 231], and k [eqn. (2.852) and preceding equations on p. 237].52 We also know that the
phase angle of Ld(jT) is always negative, so we can write
Ld ( j T) ' LdR j LdX , (2.806)
where the real and imaginary components LdR and LdX are both functions of frequency T.
A d-axis current Id(jT) which alternates at the radian frequency T produces a voltage
V d ( j T) ' [ R d j T Ld ( j T ) ] I d ( j T ) (2.807)
where Rd is the armature resistance. Substituting the real and imaginary components of Ld(jT),
Vd ' ( Rd T LdX ) j TLdR I d . (2.808)
The term TLdX represents the resistance of the conductive elements on the rotor, referred to the d-axis
circuit of the stator. Generally these elements are just the magnets. The losses in the magnets are
therefore given by

Wm(d) ' T LdX Id2 , (2.809)

where Id is the RMS value of the current at the harmonic frequency T. In a balanced 3-phase machine
the simplest cases of such a current arise from the (6k ± 1)th time-harmonics interacting with the
fundamental electrical space-harmonic of the winding distribution. For example the 5th harmonic
produces a rotating ampere-conductor distribution rotating backwards relative to the rotor at six times
the fundamental synchronous speed, and this can be resolved into d- and q-axis components which are
stationary with respect to the rotor, but which pulsate at six times the fundamental frequency. (See
Tables 1-6 on p. 201ff.). The analysis for higher-order space-harmonics is more complex, but these
should be attenuated relative to the fundamental. Losses of this type can in principle also occur in the
q-axis, but here they are ignored on the grounds that the q-axis armature-reaction flux does not pass
through the magnets, and in any case the circuits formed by induced currents in the magnets will be
far less effective in the q-axis.

Losses caused by flux pulsations (slotting)


The flux Mm through the magnet can be seen as the integral of the airgap flux modulated by slotting, as
shown in Fig. 219. Suppose the d-axis is at an angle > relative to a fixed point on the stator (such as the
axis of phase 1). If the pole-arc is $, then the limits of integration are 21 = > $/2 and 22 = > + $/2.
Ignoring leakage, and taking R as the mean radius in the airgap, and Lstk as the axial length, we have
> $/2
Mm (>) ' R Lstk B g (2 ) d 2 . (2.810)
> $/2

The fundamental time-harmonic component of Mm(Tt) can be represented as a phasor Mm(jT) if we write
> = Tt, where T/2B is the slot-passing frequency; the harmonics can be treated likewise. The harmonic
flux per pole Mm(jT) is now assumed to link the fictitious N-turn coil wrapped around each magnet, and
if all the poles are assumed to be in series it will produce a total flux-linkage per phase equal to Qm(jT)
= 2pN Mm(jT). The induced voltage will be jTQm(jT) at the harmonic frequency, and this is “applied”
to a circuit whose inductance LR has already been calculated in eqn. (2.851) on p. 237. The resistance
of this circuit is equal to LR/TdoNN, so the impedance at the harmonic frequency is

52
Only the longest time-constant is included here. Higher harmonics could be included — see eqn. (2.859) on p. 238.
Page 2.228 SPEED’s Electric Machines

Fig. 2.219 Integration of airgap flux distribution modulated by slotting

LR
Z( j T ) ' (1 j T Td0NN) . (2.811)
Td0NN
The current is
j T Q m( j T )
I( j T) ' (2.812)
Z( j T )
and the associated losses are

I 2 LR
Wm [slot mod] ' . (2.813)
Td0NN

Segmentation of the magnets in both the circumferential and axial directions is taken into account in
the calculation of Td0NN, as discussed later.

Equivalent harmonic ampere-conductor distribution for surface-magnet machines


For surface-magnet machines the fundamental component of ripple flux Mm(Tt) can be used to find the
amplitude and harmonic frequency of an ampere-conductor distribution that produces the same
harmonic field. This can then be used as the source of excitation for the classical solution of the
diffusion equation described earlier. In this way the losses caused by slot-modulation can be treated
in the same way as those due to the space- and time-harmonics in the tables on pp. 201ff.
Brushless permanent-magnet machines Page 2.229

2.35 TRANSIENTS — SYMMETRICAL THREE-PHASE SHORT-CIRCUIT

Fig. 2.220 Three-phase symmetrical short-circuit currents.

The symmetrical short-circuit of an alternator is analyzed in many classic texts. Adkins [41] gives the
following formula for the phase current (transcribed for SPEED's notation):

1 1 1 t/T dN 1 1 t/T dNN t/T a cos 8 cos ( 2 T t 8)


ia ' E e e cos ( T t 8 ) Ee
Xd XdN Xd XdNN XdN xm xn (2.814)
SS TRANSIENT SUBTRANSIENT
AC DC 2F
where E = /2 Eq1 is the peak phase EMF and
2 XdNN XqNN 2 XdNN XqNN
xm ' ; xn ' . (2.815)
XdNN XqNN XdNN XqNN
The current is plotted for all three phases in Fig. 220. It contains an AC component cos (Tt + 8), a DC
offset component cos 8 shown for the phase a, and (for salient-pole machines) a double-frequency
component 2F. 8 is the angle between the d-axis and the axis of phase a at time t ' 0. The worst-case DC
offset in phase a occurs when 8 ' 0. It decays with a time-constant Ta, the armature time-constant.
The AC component has a “steady-state” part controlled by the synchronous reactance Xd = TLd; a
“transient” part controlled by (1/XdN 1/Xd); and a “subtransient” part controlled by (1/XdNN 1/XdN). The
permanent-magnet machine has no field winding, so the transient part does not appear. This can be
represented in eqn. (2.814) by setting XdN = Xd, which has the effect of eliminating the transient term and
leaving the correct subtransient term. After the decay of the subtransient the current is limited solely
by Xd. The transient time-constants TdNand Td0N are meaningless in the permanent-magnet machine,
but there is no harm in setting them equal to TdNN and Td0NN, on the grounds that once the subtransient
has decayed the period in which the current is limited by Xd establishes itself immediately.
The subtransient part decays with a time-constant TdNN, called the subtransient time-constant, or
sometimes the “short-circuit” subtransient time-constant. Both XdNN and TdNN depend on the conductive
components on the rotor, including the magnets and any retaining can, and even the shaft. In
conventional wound-field machines the subtransient decays rapidly within a small number of cycles,
but the transient persists much longer because of the large inductance of the field winding.
At time t ' 0, ia ' 0. Consider a nonsalient-pole machine with 8 = 0. After half a cycle, Tt ' B, and if we
assume that the exponential decay terms are still substantially equal to 1, we get
Eq1 Eq1 Eq1
ia . 2 2 ' 2 2 . (2.816)
XdNN XdNN XdNN
Thus the DC offset is at most equal to the AC subtransient term, and doubles the peak current at the
beginning of the transient, as is well known, in accordance with simple AC circuit theory.
Page 2.230 SPEED’s Electric Machines

Now consider the case of a rapidly-decaying subtransient term with a persistent DC term. This will
happen if Td NN << Ta. In the PMG this is quite possible, because Td NN depends on the effective resistance
of the magnets, while Ta depends on the resistance of the armature, which is likely to be small. To show
this effect, put Td NN ' 0 in eqn. (2.814), and Ta ' 4. Then

Eq1 Eq1
ia ' 2 cos ( T t 8) 2 cos 8 (2.817)
Xd XdNN
AC DC

This shows that a persistent DC term can exceed the AC term once the subtransient AC current has
vanished. It is thus possible for the DC term to offset the current to such an extent that the entire AC
waveform is displaced to one side of the t-axis, so that it is (for a time) all positive or all negative with
no reversals. Provided that Ta is sufficiently longer than TdNN, this will happen if XdNN < Xd, which is
normally the case. This “total offset” condition arises because the persistence of the DC term depends
on Ta and not on Td NN, even though its magnitude depends on the subtransient reactance.
In short-circuit oscillograms of conventional AC machines, it is uncommon to see this "total offset"
effect because it is masked by the transient component, whose time-constant is intermediate between
the subtransient time-constant and the armature time-constant.

Subtransient reactance and time-constant

Fig. 2.221 Coupled circuits

During transients, rapid changes of current occur, and currents induced in the rotor are “reflected” in
the phase impedances of the stator. Fig. 2.221 shows the underlying physical principle in terms of two
coupled circuits. The primary (1) represents the stator; more particularly it will be used to represent
the synchronous inductances Ld and Lq. The secondary (2) represents the rotor, and it includes all
components in which induced currents may flow. That includes the magnets, the shaft, and any
retaining sleeve. The induced currents in these components are “distributed” and not in fixed wiring,
so their representation by a single circuit is an oversimplification. An exact equivalent circuit would
need an infinite number of coupled circuits, like a transmission line. Nevertheless, an analysis with
only a “first-order” equivalent circuit is instructive and potentially useful.
The voltage equations for the two circuits in Fig. 221 are
V1 ' jTL1I1 jTM I2 ;
(2.818)
0 ' jTM I1 (R2 jTL2) I2 .

The secondary voltage is zero because the rotor circuit is short-circuited, and the primary resistance
R1 is omitted because it is not needed for the present. If the second equation is used to eliminate I2,

j T M 2 / L1
V1 ' j T L1 1 I1 . (2.819)
R2 j T L2

Thus the primary inductance appears complex, and if we write Ld instead of L1, we get

1 j TT d ( 1 k 2) 1 j TTdNN
L d ( j T) ' L d ' Ld , (2.820)
1 j T Td 1 j T Td0NN
Brushless permanent-magnet machines Page 2.231

where
L2
Td0NN ' (2.821)
R2
and
TdNN ' Td0NN ( 1 k 2). (2.822)

Here Ld has been written instead of L1, and Rd instead of R1. k is the coupling coefficient M//(L1L2).
These substitutions yield a result which is recognizable in the classical theory of synchronous
machines, where Ld is the synchronous inductance, Td0 NN is the open-circuit subtransient time-constant,
and Td NN is the short-circuit subtransient time-constant. The resistance R2 is the referred resistance of
the conductive circuits on the rotor in the d-axis, and the inductance L2 is the referred self-inductance
of these circuits.
If T is very large, we get
TdNN
Ld( j T) . Ld ' Ld ( 1 k 2 ) ' LdNN , (2.823)
Td0NN

where Ld NN is the so-called subtransient reactance. Similar equations apply to the q-axis.
In the PM machine there is no field winding, and therefore the transient reactance and its associated
time-constants Td0N and Td N do not exist. For surface-magnet machines, we can expect the d- and q-axis
reactances and time-constants to be practically the same.
The parameters Td NN, Td0 NN, Ld and Ld NN are all essential to the analysis of sudden short-circuits, which
is treated elsewhere in SPEED's Electric Machines.53 The synchronous inductance Ld is associated with
conditions in which there are no induced currents on the rotor, and we can assume that it is already
known. What follows is a method of estimating the subtransient parameters using a frequency-response
approach based on the earlier solution of the complex diffusion equation.
From eqn. (2.820) we can see that the phase angle of Ld (jT) is

" ' ArcTan ( T TdNN) ArcTan ( T Td0NN) . (2.824)


2
When TTd0 NN = 1, we have a phase angle of 45E in the denominator of eqn. (2.820) and ArcTan (1 k ) in
the numerator, and the overall phase angle can be written

B
"1 ' ArcTan ( 1 k 2) [rad] . (2.825)
4

Fig. 2.222 Variation of phase angle of Ld(jT) with frequency T

53
This analysis does not require explicit values for L2 or R2.
Page 2.232 SPEED’s Electric Machines

Fig. 222 shows an example of the variation of " with frequency T, with k = 0@95 and Td0NN = 5 ms. The
2
short-circuit time-constant is Td NN = (1 0@95 ) × 5 = 0@4875 ms, approximately ten times shorter. There
is always a minimum phase angle. At frequencies well below the frequency of minimum phase, the
induced currents are resistance-limited; at frequencies well above it, they are inductance-limited.
Suppose k is known. Then if we calculate Ld(jT) and find the frequency T1 at which " = "1, it follows that

1
Td0NN ' . (2.826)
T1
2
Then Td NN can be found from eqn. (2.822). In the example, "1 = ArcTan(1 0@95 ) 45E = 39@43E. This
point is marked in Fig. 222. It occurs at exactly 200 rad/s, so that from eqn. (2.826) Td0 NN = 1/200 or 5 ms.
In general k is not known a priori, even when the frequency-response in Fig. 222 is available. In
2
principle it is possible to determine (1 k ) from the asymptotic value of the magnitude |Ld(jT)| at very
high frequency, but extremely high frequencies are generally needed to reach this condition and the
calculation of Bessel functions with very large arguments (inversely proportional to the skin depth)
then becomes problematic. A simpler approach is to begin with an estimate of the coupling coefficient
k for the fundamental space-harmonic, which can be calculated using Hughes and Miller [13]. In terms
of the dimensions in Fig. 187 on p. 178, for a winding at radius r0,

r0 p
2 1 ( rH / r0 ) 2p
k ' . (2.827)
rS 1 ( rH / rS ) 2p 1 ( r0 / rS ) 2p

The “winding” at radius r0 is a fictitious sine-distributed current-sheet with p pole-pairs and zero radial
thickness. In this context it represents the thin layer of eddy-currents induced in the rotor at very high
frequency, so r0 should be taken as very slightly less than the rotor radius. Now it is the fundamental
2
magnetizing inductance Lmd that is modified by (1 k ): the total subtransient inductance is obtained
by adding the leakage inductance LF, which includes the end-turn leakage, the slot leakage, and the
differential leakage: thus

LdNN ' ( 1 k2 ) Lmd LF (2.828)


The leakage inductance LF is obtained from
LF ' Ld Lmd . (2.829)
The objective is to find the overall coupling coefficient kN that makes

LdNN ' ( 1 kN2 ) Ld (2.830)


and this can be obtained by solving the above equations, giving

LF
kN ' k2 (1 k 2) . (2.831)
Ld

It remains to explain how the frequency response in Fig. 222 is calculated from the solution of the
complex diffusion equation. This involves performing a frequency scan and calculating the phase angle
of the vector potential A at the stator bore, with a current sheet of constant amplitude. A is an integral
component of the solution, as can be seen from eqn. (2.640) on p. 188.
For IPM machines the complex diffusion equation is not applicable and a different approach is needed,
as developed on page 235ff.
Brushless permanent-magnet machines Page 2.233

Effect of segmentation on subtransient reactance

Fig. 2.223 Estimating the effect of segmentation on the subtransient reactance

Fig. 223(a) shows the eddy-currents in a continuous rotor can, together with a representation of the
stator ampere-conductor distribution. When the stator and rotor are both infinitely long, the coupling
coefficient between the stator and the rotor “circuits” is given by the 2D formula (2.827) [13], and here
it will be denoted kd.
We have already seen that when the rotor is represented by a single circuit, the short-circuit inductance
of the stator is given by an equation of the form

L NN ' L0 (1 kd2) (2.832)

where L0 has been identified as the magnetizing inductance Lmd and LNN as the subtransient inductance
(ignoring LF for the moment). Here, LNN is viewed as the primary inductance of a set of coupled circuits;
so in Fig. 223(a) there is only one secondary circuit. In Fig. 223(b) there is a stack of N secondary
circuits, each one corresponding to one ring or segment.
The primitive inductance matrix (SEM-1) is of the form

L0 m m m m

m l

m l

m l

m l

where m is the mutual inductance between the primary and each secondary, and l is the self-inductance
of each secondary. The problem is simplified drastically by assuming that l and m are identical for all
the secondaries; moreover, it is assumed that there is no mutual coupling between any pair of
secondaries. If there were, the blank elements would be non-zero. (In practice they are small and may
even be negative).
Now it is easy to show that when the secondaries are all short-circuited, the matrix can be reduced to
a 1 × 1 matrix representing the short-circuit inductance,

L NN ' L (1 Nkm2) (2.833)


— an exceedingly simple result, with
m
km ' . (2.834)
L0 l
Page 2.234 SPEED’s Electric Machines

The calculation of m and l is practically impossible because of the difficulty of identifying definite
circuits in which the eddy-currents flow, and also because the eddy-current circuits are short-circuited.
Indeed it is questionable whether a circuit-based approach is justifiable at all : the problem is really a
distributed field problem. However, what is most needed is an estimate of the relationship between km
and N, and a very crude estimate can be made by the concept of “loss of effectively coupled length”.
First, the inductances l and m are considered to be proportional to an area that is basically defined by
the pole-pitch in the circumferential direction and the length C/N in the axial direction, where C is the
overall length of the unsegmented can. However, because the eddy-currents must turn to flow in the
circumferential direction, there is a loss of “effectively coupled length” with the stator. This loss of
length is denoted 0, and since it applies at both ends, the effectively coupled length of each section is
reduced from C/N to C/N 20. Even when N = 1, the effectively coupled length is C 20 < C.
Both m and l are proportional to the effectively coupled length in each section, that is, C/N 20,
whereas L0 is constant and proportional to C. So when the can is divided into N rings, we can write

C/N 20 1 20
km ' × kd ' kd . (2.835)
C (C / N 2 0) N C

The number of turns in each secondary circuit is not needed, because when the secondaries are short-
circuited the impedance referred to the primary is independent of the number of secondary turns.
If eqn. (2.835) is substituted in eqn. (2.833), the result is

20
L NN ' L0 1 kd2 1 . (2.836)
C /N
Suppose 0 is taken as J/6, where J is the pole-pitch, and let J = 0@6 C, and kd = 0@96. Then if N = 1 we get

2 × J/6
L NN / L0 ' 1 0@962 1 ' 0@2627 . (2.837)
J / 0@6/1
If N = 4 we get a much higher inductance:

2 × J/6
L NN / L0 ' 1 0@962 1 ' 0@8157 . (2.838)
J / 0@6/4

If N = 5 the factor in curly brackets is zero, so the effective coupling coefficient is zero and LNN/L0 = 1.
For any higher value of N, the coupling coefficient is taken to remain zero, giving LNN/L0 = 1. The value
of N that reduces the coupling coefficient to zero clearly depends on 0. A general rule for the value of
0 cannot be given, but of course it can be chosen to match the results of finite-element analysis or
measurement. 0 has an important influence on the peak short-circuit current, so its value should be
chosen judiciously. A value of 0@1 is probably a reasonable starter.
For any given number of rings N, the most pessimistic value of 0 is 0, since this has the effect of
ignoring the segmentation altogether, which minimizes LNN and the subtransient inductance, and so
produces the maximum short-circuit current. The most optimistic value of 0 is C/2N, since this always
leaves LNN ' L0, so the subtransient reactance will be equal to the steady-state synchronous reactance
and the peak short-circuit current will be minimized.
In practice when N is increased, the self-inductance l will not decrease as fast as the mutual m, because
of the additional “leakage inductance” associated with the flow of currents in the circumferential
direction at the ends. This tends to reduce the effective coupling coefficient more rapidly with
increasing N, and although the effect can be calculated it is not included here.
When the machine has no can, eddy-currents may arise in the magnets, but the above method is applied
only when the magnet forms a complete ring with no segmentation whatsoever. In all other cases the
eddy-currents in the magnet are assumed to be uncoupled to the stator.
Brushless permanent-magnet machines Page 2.235

Coupling coefficient and rotor time-constant of the IPM

Fig. 2.224 Calculation of coupling coefficient — d-axis

Fig. 224 shows one pole of an IPM. The magnet in each pole is divided into two sections, each of width
w/2, which are electrically isolated from each other. The airgap g is shown wider than normal for
clarity in the drawing, and the effective airgap length is gN ' k c g, where kc is Carter's coefficient. The
magnet length is h in the direction of magnetization, and the stator bore radius is R. The pole-pitch is
J = B/p mechanical radians, where p is the number of pole-pairs.
During fast transients, induced currents tend to flow at the edges of the magnets as indicated by the
hatching. Let the induced currents be represented by currents flowing in two coils of wire, one wrapped
all the way around each block. Each of these coils has N turns. The flux/pole produced by the rotor
current is practically identical in form to the open-circuit magnet flux, and the resulting airgap flux can
be written
Fm
Mg ' , (2.839)
Rm RgN
where Fm is the number of ampere-turns in one of the rotor coils, equal to N times the current in each
coil. Rm is the reluctance of the two magnet blocks in parallel, and RgN is the reluctance of a section of
airgap of arcuate extent " mechanical radians, modified to account for rotor leakage flux that passes
from pole-to-pole without crossing the airgap. Thus
Rg
RgN ' ' 8 Rg , (2.840)
1 PL Rg

where PL is the pole-to-pole leakage permeance per pole and 8 is a leakage coefficient which is
introduced at this stage to simplify the algebra. The bridges and centre web are assumed to be
completely saturated and are omitted from the calculation at this stage. In terms of the dimensions in
Fig. 224, if L is the axial length,

h
Rm ' , (2.841)
μ0 μrec w L

and

gN8
RgN ' . (2.842)
μ0 R " L

Substituting for Rm and RgN, we get


Page 2.236 SPEED’s Electric Machines

μ0 L Fm
Mg '
h gN8 (2.843)
μrec w R"

The distribution of this flux in the airgap is a rectangular block of arcuate extent ", so the peak flux-
density is given by Mg/(R"L) or

1 μ0 Fm μ0 Fm
Bg ' ' [T] .
R" h gN8 hR" (2.844)
gN8
μrec w R" μrec w

The fundamental component is easily determined by Fourier analysis of the rectangular block, giving

μ0 Fm 4 p"
B1 ' × sin [T] .
hR" B 2 (2.845)
gN8
μrec w

The fundamental flux associated with B1 is

DL DL μ0 Fm 4 p"
M1 ' B1 ' × × sin [T] ,
p p hR" B 2 (2.846)
gN8
μrec w

where D = 2R. Now suppose the rotor current is sinusoidal, with frequency f. The EMF generated in
one phase of the stator winding is
2B
E ' kw1TphM1 f Vrms (2.847)
/2

where kw1 is the fundamental winding factor and Tph is the number of turns in series per phase. The
peak flux-linkage per phase is /2 E/(2B f) or
Q1 ' kw1TphM1 Vsrms . (2.848)

With 1@0 A peak current flowing in the rotor coils, the mutual inductance is equal to Q1/1@0 or just Q1.
If we set N = 1 and retain 1@0 A rotor current, Fm = 1 and we can write the mutual inductance as

3 DL μ0 4 p"
M ' kw1 Tph × × × sin [T] ,
2 p hR" B 2 (2.849)
gN8
μrec w
The 3/2 factor anticipates a switch to dq axes which is necessary so that the effects of saliency can be
taken into account. So M is the apparent mutual inductance between the rotor winding and one phase,
under conditions of symmetrical operation with currents in all three phases.
Now to obtain the coupling coefficient we need the self-inductance of both windings. From SEM-2 we
have the d- and q-axis magnetizing inductances
Lmd ' 'd Lm0 ;
(2.850)
Lmq ' 'q Lm0 ,

where Lm0 is the airgap component of the synchronous inductance of a nonsalient pole machine having
an airgap length equal to gN, and is given by eqn. (2.168).
Brushless permanent-magnet machines Page 2.237

Lm0, Lmd and Lmq are all associated with the fundamental flux produced by the stator winding; they do
not include the space-harmonic fluxes. For the moment we will content ourselves with Lmd and Lmq,
and add the leakage inductances (including the harmonic leakage inductance) later.
The self-inductance of the rotor coils is given simply by the self flux-linkage divided by the current. The
total airgap flux/pole produced by the rotor current has already been calculated as Mg, so if all the poles
are in series the total flux-linkage of the complete rotor “winding” is N × Mg × 2p. With a current of 1@0
A the self-inductance is equal to 2pNMg. Again if N = 1 and the rotor current is 1@0 A, Fm = 1 and we can
write the rotor self-inductance as
2 p μ0 L
LR ' .
h gN8 (2.851)
μrec w R"

The coupling coefficient follows as

M
kd ' (2.852)
Ld LR

in the d-axis. A similar process can be developed in the q-axis using Lq.

Effect of segmentation of magnets — The concept of loss of effectively coupled area (used previously
in deriving eqn. (2.836)) can be applied to segmented magnets in the IPM. Consider a magnet block of
pole area a × b, as in Fig. 225. Assume that when the block is divided into na × nb isolated segments the
loss of effectively coupled area is represented in each block by a strip of width ga along the a sides and
width gb along the b sides; then the remaining coupled area for each block is (a/na ! 2ga) × (b/nb ! 2gb)
2
= ab (1/na ! 2g)(1/nb !2g). For an unsegmented block the coupled area is ab (1 ! 2g) .
Using the same logic as in eqn. (2.835) we get the following reduction factor for kd :
1
( 1 ! 2 g na ) ( 1 ! 2 g nb ) . (2.853)
1 ! 2g

Rotor time-constant — The essential time-constant of


the conductive components of the rotor is the open-
circuit time-constant Td0NN, which can be identified as
the diffusion time-constant of eddy-currents in these
components.
A related problem analyzed by Lammeraner and Štafl
[93] can be brought to bear on this calculation. Fig. 225
shows a rectangular block of magnet with sides a in the
x-direction and b in the y-direction, in which eddy-
currents are flowing in planes parallel to the xy-plane, Fig. 2.225 Eddy-currents in a magnet block
and the flux-density vector in the magnet obeys the
diffusion equation

M2 B M2 B MB
' μF . (2.854)
Mx 2
My 2 Mt
The flux-density is postulated to have only a z-component and the direction of magnetization is also in
the orthogonal z-direction, that is, vertical. Lammeraner and Štafl state that since for symmetry
reasons the dependence of flux density B in the direction of the x and y axes must be an even function,
we may assume the solution to be in the form
t/J
B ' B0 cos " x cos $ y e (2.855)
where J denotes the time-constant.
Page 2.238 SPEED’s Electric Machines

Substituting in the differential diffusion equation we obtain a condition relating the so far unknown
quantities ", $ and J :

μF
"2 $2 ' . (2.856)
J

They then apply the boundary condition at t = 0, x = ±a/2, y = ±b/2 to eqn. (2.855), to obtain
B0 cos (" a/2) cos ($ b/2) ' 0 . (2.857)
This equation is satisfied if
" ' (2 n 1) B / a ; $ ' (2 m 1) B / b , (2.858)

where n and m are integers. From eqn. (2.856) the time-constant is then obtained as
μF
Jm,n ' . (2.859)
2
[(2 n 1) B/a] [(2 m 1) B/b]2
Although the complete solution involves an infinite series of terms for all non-negative integer values
of n and m, Lammeraner and Štafl point out that the duration of the transient only requires finding the
greatest time-constant which is obtained with n = 0 and m = 0, so we get, finally,

μF ab
Td0NN ' J0,0 ' 2
. (2.860)
B a/b b/a
Lammeraner and Štafl also consider the effect of an airgap in series with the magnet, but in the typical
geometry of an IPM this effect will be small.
The short-circuit subtransient time-constant TdNN required for the symmetrical short-circuit analysis
is obtained from eqn. (2.822).
The time-constant Td0NN is recognizable as a diffusion time-constant and it is interesting to note that the
characteristic dimension ab/(a/b…+ b/a), which has the dimensions of [length], is determined entirely
by dimensions parallel to the xy plane. The “diffusion” is in fact lateral, in the sense that the current
starts in a concentrated layer at the edges and progressively fills the block by diffusing in the x and y
directions. There is no variation or diffusion in the z-direction.
The dimensions a and b have so far been tacitly assumed to be those of a monolithic magnet block, Fig.
225. Suppose the block is divided into M segments in the x-direction such that a = Mu, and N segments
in the y-direction such that b = Nv. Then it can be argued that a should be replaced by u = a/M and that
b should be replaced by v = b/N in eqn. (2.860). Segmentation of the magnets has no effect on the
inductive effect of the eddy-currents (cf. Fig. 208 on p. 213), but it has a marked effect on the resistance
impeding the eddy-currents. As a further example of this, in Fig. 224 the positive and negative “wired”
eddy-currents cancel in the centre, but yet the eddy-currents must pass through those paths and in
doing so they meet considerable additional resistance. Again in Fig. 224, the dimensions to be used in
eqn. (2.860) are w/2 and L (assuming the magnets are not segmented in the axial direction).

Armature time-constant — The final parameter required for the symmetrical short-circuit analysis
[eqn. (814) on p. 229] is the armature time-constant Ta. For salient-pole machines this is given by
2 LdNN LqNN
Ta ' , (2.861)
Rph( LdNN LqNN )
where Rph is the resistance per phase. For nonsalient-pole machines LdNN = LqNN and
LdNN
Ta ' . (2.862)
Rph
Note : Ta is not really a fixed property of the machine but is one of several “decrement” factors that
depend on the type of fault. (See Concordia, [99]).
Brushless permanent-magnet machines Page 2.239

2.36 TRANSIENT MAGNETIC FIELD IN THE MAGNET : THE FOURIER TRANSFORM METHOD

Fig. 2.226 Transient calculated by Fourier transform

The short-circuit fault on p. 229 was calculated assuming constant speed, but this is not necessarily the
worst-case transient for the magnet. Fig. 226 shows a fault condition in which the speed does not remain
constant, but suffers a perturbation resulting in oscillations about synchronous speed at the so-called
“swing frequency”, which is typically of the order of 1 2 Hz. The swing-frequency oscillation is
apparent in the d- and q-axis currents. If the speed was constant, the swing-frequency would be absent
from Fig. 226, and id and iq would become constant once the line-frequency component decayed to zero.
In the rotating dq reference frame, this component is due to the DC offset in the stator current.
Fig. 226 is calculated for a PM machine with a conductive shield which attenuates the line-frequency
component very effectively, but has almost no shielding effect at the swing frequency. To calculate this
type of transient by the finite-element method, it would be necessary to include mechanical equations
permitting the speed to vary. A more efficient method is to calculate the d- and q-axis currents by a
circuit-based simulation process in which the speed is allowed to vary, and then to apply the Fourier
transform as described below. This method was originally developed for analyzing the transient fields
in superconducting alternators, in which the transient field in the rotor is also of concern — a situation
very similar to the one facing the designer of PM machines; (Miller, [67,68]).
Let B(t) represent the transient flux-density at a certain point in the magnet; then considering the effect
of d-axis stator current we can obtain B(t) from the inverse Fourier transform,
1
B(t) ' F {Sd(jT) Id(jT)} . (2.863)
where Id(jT) is the Fourier transform of the transient d-axis current and Sd(jT) is a screening function,
that is, the complex ratio of the flux-density at the point of interest and the d-axis current, both being
expressed as phasors over a sufficiently wide frequency range according to the Nyquist sampling
frequency. An equivalent relationship exists for the q-axis, and the d- and q-axis components are added.
The current transform Id(jT) is the Fourier transform of the transient d-axis current id(t) which is
computed from the circuit model. Starting from the phase currents in Fig. 220 on p. 229, id(t) and iq(t)
can be determined by Park's transformation; or they may arise directly from a simulation in dq axes.
It is noted that any DC offset in the phase current is automatically included in Id(jT) and Iq(jT). At the
same time it is noted that id(t) and iq(t) are “one-shot transients,” not periodic functions.54
The screening function S(jT) is obtained from the solution of the complex diffusion equation, for any
component of flux-density Br or B2 at any point in the magnet. A frequency scan is necessary, which
in principle must extend from T = 0 to T = 4. The response B(t) in eqn. (863) is then evaluated by the
Fast Fourier Transform (F.F.T.); see Cooley et al [86].

54
It is for this reason that the Fourier transform is required, rather than the Fourier series.
Page 2.240 SPEED’s Electric Machines

2.37 FINITE-ELEMENT CALCULATION OF LOSSES


The finite-element method is held to produce
results of assured accuracy, but it is also the
slowest and most expensive method in terms of
engineering time. 3D calculations may take days,
yet what is often needed is adequate accuracy in
seconds or minutes. 2D finite-elements may be
used in eddy-current problems, in which the zero
net current condition

J dS ' 0 (2.864) Fig. 2.227 Incorrect eddy-current calculation

is imposed (usually without proof) individually


across each conducting region by means of a
fictional external circuit connection with a high
resistance shorting the two ends.
This technique requires a voltage-driven solution
and a special formulation in which the circuit
equations are incorporated in the field-solution
matrix. Although there is excellent mathematical
literature on the finite-element method, a review
of several books on finite-elements in electrical
engineering fails to provide any practical
reassurance of the validity of this method. Fig. 2.228 Approximate eddy-current solution: 1 block

Fig. 2.227 shows the calculation of eddy-currents


with a conventional 2D solver using current
excitation (and no external circuit connection).
The problem is to calculate the no-load eddy-
current losses in the magnets at high speed. The
example in Fig. 2.227 uses the time-stepping
algorithm of Crank-Nicholson, and predicts a total
loss of 2@06 W, but this result is incorrect because
the “infinite length” assumption permits induced
current in magnet A to return in magnet B, and
likewise current in magnet C to return in magnet
D. Since there is no electrical connection between
Fig. 2.229 Approximate eddy-current solution: 2 blocks
the magnets, this is a false result.

The simple finite-element formulation satisfies eqn. (2.864) over the whole solution domain, but not
individually in each magnet region. An improved result can be obtained by suppressing the
conductivity of, say, magnets B, C, and D, and simply calculating the loss in magnet A. In a situation
where the losses must be equal in all magnet blocks, the final result is obtained by multiplying the
result for one block by 8. When this is done in Fig. 2.228, the total calculated loss is 0@97 W when scaled
up to include all magnets. This is slightly less than half the incorrect first estimate. The method can be
described as a crude form of residual current suppression without using external circuits.
This method of single-region eddy-current calculation obviously relies on the assumption that the eddy-
currents in any magnet do not affect the eddy-currents in any other magnet. One could assume that this
is characteristic of “resistance-limited” eddy-currents, but it is more correct to describe it as the neglect
of proximity effect, which in other situations is known to be dangerous. Nevertheless, engineers have
long relied on simplified calculations combined with other means of verification than very expensive
calculating tools; and if this approach is still permitted, the method is surely worth trying.
A “refinement”, so to speak, is to suppress the conductivity judiciously in pairs or patterns of magnets
which are not expected to form a circuit. Thus Fig. 2.229 shows an example in which magnets A and C
are conductive, but magnets B and D are not.
Brushless permanent-magnet machines Page 2.241

The total loss calculated in this case is 1@02 W, again scaled up to include all magnets. This is reasonably
close to the 0@97 W calculated using the single-block approximation, and the difference may give some
idea of the accuracy of the method. The only way to be sure is to measure it, although some confidence
might be obtained through more sophisticated calculation tools.
An interesting observation in Figs. 227, 228 and 229 is the “lateral” diffusion of the eddy-currents
towards the edges of the magnets, with little variation in current density in a direction parallel to the
flux. This property is used in [4] in the formulation of simplified loss calculations for the IPM, and the
related calculation of the frequency-dependent synchronous inductance mentioned earlier. With
reference to the skin-depth criterion discussed on p. 173, in this example the “key dimension” to be
compared with the skin depth is the width of the magnet and not its length in the direction of
magnetization.
Page 2.242 SPEED’s Electric Machines

REFERENCES
1. Hendershot J.R. and Miller TJE : Design of brushless permanent-magnet machines, published by Motor Design Books LLC,
Venice, Florida 34292, USA, 2010. ISBN No. 978-0-9840687-0-8. www.motordesignbooks.com.
2. Hendershot J.R. and Miller TJE : Design of brushless permanent-magnet motors, Oxford University Press, Monographs in
Electrical and Electronic Engineering No. 37, 1994. ISBN No. 0-19-859389-9 (or in USA, 1 881855 03 1)
3. Mohan N, Undeland T.M. and Robbins W.P.: Power electronics: converters, applications, and design, John Wiley & Sons, 1989,
1995 [2nd edition]. ISBN 0-471-58408 8.
4. Murphy J.M.D. and Turnbull F.G.: Power electronic control of AC motors, Pergamon Press, 1988. ISBN 0 08 22683 3.
5. Hague B : The principles of electromagnetism applied to electrical machines, Dover Publications Inc., N.Y., 1962.
6. Boules N : Prediction of no-load flux-density distribution in permanent magnet machines, IEEE Transactions on Industry
Applications, Vol. IA-21, No. 4, May/June 1985, pp. 633-643.
7. Jahns, T.M.: Torque production in PM synchronous motor drives with rectangular current excitation, IEEE Transactions,
IA-20, pp 803-813, 1984
8. Demerdash, N.A. with Arkadan, A., Nehl, T., Vaidya, J. and others: papers on brushless DC and AC motors and drives,
published in IEEE Transactions, including EC-3, Sept 88, 722-732; EC-2, March 87, 86-92; PAS-104, Aug 85, 2206-2213; 2214-2222;
2223-2231; PAS-103, July 84, 1829-1836; PAS-102, Jan 83, 104-112; PAS-101, Dec 82, 4502-4506; PAS-100, Sept 81, 4125-4135; EC-3, Sept
88, 714-721; EC-3, Dec 88, 880-889; 890-898.
9. Fitzgerald AE and Kingsley C : Electric Machinery, McGraw Hill (second edition) 1961
10. Kostenko and Piotrovsky, Electric Machines, MIR Publishers
11. Rasmussen KF, Miller TJE, Davies JI, McGilp M and Olaru M : Analytical and numerical computation of airgap magnetic fields
in brushless permanent magnet motors, to be presented at IEEE Industry Applications Society Annual Meeting, Phoenix, Az,
October 1999.
12. Reliance Motion Control Inc.: DC Motors, SPEED Controls, Servo Systems: The Electro craft Engineering Handbook, 6th edn.
13. Hughes A. and Miller TJE : Analysis of fields and inductances in air cored and iron cored synchronous machines, Proceedings
IEE, Vol. 124, No. 2, February 1977, pp. 121 131.
14. Rasmussen KF, Analytical prediction of magnetic field from surface mounted permanent magnet motor, IEEE IEMDC
Conference, Seattle, pp. 34 36, May 9 12, 1999.
15. Clayton AE and Hancock NN, The performance and design of direct current machines, 3rd edn., Pitman, London, 1959 66, p. 36.
16. Kenjo T and Nagamori S, Permanent magnet and brushless DC motors, Sogo Electronics Publishing Company, Tokyo, 1984.
17. Rosa EB and Grover FW, Formulas and tables for the calculation of mutual and self inductance, Department of Commerce and
Labor, Bureau of Standards, Bulletin Vol. 8, No. 1, January 1, 1911.
18. Jones CV, The unified theory of electrical machines, Butterworths, London 1967.
19. Kimbark, EW., Power system stability, Wiley, New York, 1948. (Reprinted by IEEE Press 1995, ISBN 0 7803 1135 3).
20. Harris MR, Stephenson JM and Lawrenson PJ, Per unit systems, Cambridge University Press, 1970.
21. Soong WL and Miller TJE, Field weakening performance of brushless synchronous AC motor drives, IEE Proc. Electr. Power
Appl., Vol. 141, No. 6, November 1994, pp. 331 340.
22. Strauss F, Synchronous machines with rotating permanent-magnet fields, AIEE Transactions, Vol. 71, Pt. II, pp. 887-893, October
1952.
23. Merrill FW, Permanent-magnet excited synchronous motors, Transactions AIEE, Vol. 74, pp. 1754 1760, 1955.
24. DD Hershberger, Design considerations of fractional horsepower size permanent-magnet motors and generators, AIEE
Transactions, pp. 581!584, June 1953.
25. Lajoie-Mazenc M, Foch H and Villanueva C, Feeding permanent-magnet machines by a transistorized inverter,
PCI/MOTORCON, pp. 558!570, September 1983.
26. Jahns TM, Kliman GB and Neumann TA, Interior permanent magnet motors for adjustable speed drives, IEEE Transactions,
Vol. IA-22, No. 4, pp. 738!747, July/August 1986.
27. Popescu M, Ionel DM, TJE Miller, S.J. Dellinger and M.I. McGilp [2005] Improved finite element computations of torque in
brushless permanent magnet motors, IEE Proc-Electr. Power Appl., Vol. 152, No 2, March/April 2005, pp. 271-276.
28. Ionel D M, Popescu M, McGilp MI, Miller TJE and Dellinger SJ [2005] Assessment of torque components in brushless permanent-
magnet machines through numerical analysis of the electromagnetic field, IEEE Transactions on Industry Applications, Vol. 41,
No. 5, September/October 2005, pp. 1149-1158.
29. Cahill DPM and Adkins B, The permanent-magnet synchronous motor, Proc. IEE, Vol. 109, Part A, No. 48, December 1962, pp.
483-491.
30. Volkrodt W, Elektrotech. Z., 82, p. 524, 1961; and ibid, 83, p. 522, 1962.
31. Honsinger VB, The steady-state performance of reluctance machines, IEEE Transactions, Vol. PAS-90, No. 1, 1971, pp. 305-317.
See also Honsinger VB, Synchronous reluctance motor (Allis-Chalmers Manufacturing Co., Milwaukee), United States Patent No.
References Page 2.243

3,652,885, Mar. 28; 1972 and Jorgensen MV, Albertson BE, Michaels E and Turner GE, Synchronous induction machine (Louis Allis
Co., Milwaukee), United States Patent No. 3,210,584, Oct. 5, 1965; and Honsinger VB, Permanent magnet motor (Allis-Chalmers
Manufacturing Co,. Milwaukee), United States Patent No. 3,126,493, Mar. 24, 1964.
32. Kostko JK, Polyphase reaction synchronous motor, Journal AIEE, Vol. 42, pp. 1162-1169, 1923
33. Binns KJ and Jabbar MA, High-field self-starting permanent-magnet synchronous motor, IEE Proc., Vol. 128, Pt. B, No. 3, May
1984, pp. 157-160.
34. Laronze J, Eine neue Generation von bürstenlosen Synchronmotoren, Aktuelle Technik, Feb. 1978, pp. 9-12.
35. Miyashita K et al, Development of a high speed 2-pole permanent-magnet synchronous motor, IEEE Trans., Vol. PAS-99, No. 6,
Nov/Dec 1980, pp. 2175-2183. See also Miyashita K, Yamashita S, Watanabe H and Tanabe S, Permanent magnet rotor (Hitachi Ltd),
U.S. Patent No. 4,403, 161, 6 September 1983.
36. Steen CR, Direct axis aiding permanent magnets for a laminated synchronous motor rotor, United States Patent No. 4,139,790,
February 13, 1979.
37. Binns KJ, Barnard WR and Jabbar MA, Hybrid permanent-magnet synchronous motors, IEE Proc., Vol. 125, No. 3, March 1978,
pp. 203-208
38. Miller TJE, Popescu M, Cossar C, McGilp MI, Strappazzon G, Trivillin N, Santarossa R [2004] Line-Start Permanent-Magnet
Motor Single-Phase Steady State Performance Analysis, IEEE Transactions on Industry Applications, Vol.40, No.2, March/April
2004, pp.516-525.
39. Staton DA, Miller TJE and Wood SE [1993] Maximising the saliency ratio of the synchronous reluctance motor, IEE Proceedings-
B, Vol.140, No.4, July 1993, pp. 249-259.
40. Miller TJE, Single-phase permanent-magnet motor analysis, IEEE Trans. Ind. Appl., IA-21 (4), May/June 1985, pp. 651-658.
41. Adkins B, The general theory of electrical machines, Chapman & Hall, 1957.
42. Miller TJE Synchronization of line-start permanent-magnet AC motors. Transactions IEEE, Vol. PAS-103, 1822-1828, 1984.
43. Honsinger VB, Permanent-magnet machines: asynchronous operation, Transactions IEEE, Vol. PAS-99, No. 4, July/August
1980, pp. 1503-1509.
44. Douglas JFH and Rautimo P, Synchronous induction motor (Louis Allis Company, Milwaukee), U.S. Patent No. 2,913,607, 17
November 1959.
45. Binns KJ, U.K. Patent No. 1,359,548, 9 December 1971; Binns KJ and Barnard WR, U.K. Patent No. 1,324,147, 7 May 1971.
46. Finch JW and Lawrenson PJ, Synchronous performance of single-phase reluctance motors, Proceedings IEE, Vol. 125, No. 12,
pp. 1350-1356, 1978
47. Cros J and Viarouge P, IEEE Transactions on Energy Conversion, Vol. 17, No. 2, June 2002, pp. 248-253.
48. Magnussen F and Lendenmann H, Parasitic effects in PM machines With Concentrated Windings, Transactions IEEE, Vol. 43,
No. 5, September/October 2007, pp. 1123-1232.
49. Magnussen F and Sadarangani C, Winding Factors and Joule Losses of Permanent Magnet Machines with Concentrated
Windings, Proceedings IEMDC Conference , Madison, WI, June 1-4, 2003, pp. 333-339.
50. Miller T J E and Hughes A, Comparative design and performance analysis of air-cored and iron-cored synchronous machines,
Proc. IEE, Vol. 124, No. 2, February 1977, pp. 127-132
51. Carter FW, Pole-face losses, J. IEE, 1916, Vol. 54, p. 168
52. Freeman EM, The calculation of harmonics, due to slotting, in the flux-density waveform of a dynamo-electric machine, Proc.
IEE, Vol. 109C, p. 581, 1962
53. Lawrenson PJ, Reece P and Ralph MC, Tooth-ripple losses in solid poles, Proc. IEE., Vol. 113, No. 4, April 1966, pp. 657-662
54. Oberretl K, Eddy Current Losses in Solid Poles of Synchronous Machines at No-Load and On Load, IEEE Transactions, Vol.
PAS-91, 1972, pp. 152-160
55. Stoll RL and Hammond P, Calculation of the magnetic field of rotating machines, Part 4. Approximate determination of the field
and the losses associated with eddy currents in conducting surfaces, Proc. IEE, Vol. 112, No. 11, November 1965, pp. 2083-2094
56. Lawrenson PJ and Ralph MC, The general 3-dimensional solution of eddy-current and Laplacian fields in cylindrical structures,
Proc. IEE, Vol. 117, pp. 469-472 (1970)
57. Preston T and Reece ABJ, Transverse edge effects in linear induction motors, Proc. IEE, Vol. 116, pp. 973-979, 1969
58. Angst G, Polyphase induction motor with solid rotor : effects of saturation and finite length, Trans. AIEE, Vol. 80, pp. 902-909
(1962)
59. Wood AJ and Concordia C, Analysis of solid rotor induction machines: Pt. III — finite length effects, Trans. AIEE, Vol. 79, pp.
21-26 (1960)
60. Kirtley JL et al, Rotor loss models for high-speed PM motor-generators, ICEM, Istanbul, 1998, pp. 1832-1837
61. Ishak D, Zhu ZQ and Howe D, Eddy-Current Loss in the Rotor Magnets of Permanent-Magnet Brushless Machines Having a
Fractional Number of Slots Per Pole, IEEE Transactions on Magnetics, Vol. 41, No. 9, September 2005, pp. 2462-2469
Page 2.244 SPEED’s Electric Machines
62. Zhu Z Q and Howe D, Instantaneous Magnetic Field Distribution in Brushless Permanent Magnet DC Motors, Part III : Effect
of Stator Slotting, IEEE Transactions on Magnetics, Vol. 29, No. 1, January 1993, pp. 143-151
63. Zhu ZQ, Ng K, Schofield N and Howe D, Improved analytical modelling of rotor eddy current loss in brushless machines equipped
with surface-mounted permanent magnets, IEE Proc.-Electr. Power Appl., Vol. 151, No. 6, November 2004, pp. 641-650
64. Shah MR and Lee SB, Rapid Analytical Optimization of Eddy Current Shield Thickness for Associated Loss Minimization in
Electrical Machines, IEEE Trans. Industry Applications, Vol. 42, No. 3, May/June 2006, pp. 642-649
65. Shah MR and Lee SB, Optimization of Shield Thickness of Finite Length Rotors for Eddy Current Loss Minimization, 41st
Industry Applications Conference, Tampa, Florida, 8-12 October 2006, pp. 2368-2373.
66. Miller TJE and Lawrenson PJ, Penetration of transient magnetic fields through conducting cylindrical structures, with
particular reference to superconducting A.C. machines, Proc. IEE, Vol. 123, No. 5, May 1976, pp. 437-443
67. Miller TJE, Transient magnetic fields in the superconducting alternator. Archiv für Elektrotechnik 62, pp. 131-140, 1980
68. Miller TJE Transient magnetic fields in the superconducting alternator, PhD Thesis, University of Leeds, 1977.
69. Russell RL and Norsworthy KH, Eddy currents and wall losses in screened-rotor induction motors, Proc. IEE, Vol. 105A, pp. 163-
175, 1958
70. Robinson RC, Rowe I and Donelan LE : The calculation of can losses in canned motors, Trans. AIEE, June 1957, pp. 312-315.
71. Takahashi I et al, A Super High-Speed PM Motor Drive System by a Quasi-Current Source Inverter, IEEE Transactions on
Industry Applications, Vol. 30, No. 3, May/June 1994, pp. 683-690.
72. Obaraki H, Tawara K, Endo T, Takahashi I, Toyoshima H and Ishii Y : Characteristics evaluation of a high-speed brushless
motor, in Rec. JIEE Conf. Rotary Machine, June 1991, RM-91-21, pp. 59-66
73. Deng F, Commutation-Caused Eddy-Current Losses in Permanent-Magnet Brushless DC Motors, IEEE Trans. on Magnetics, Vol.
33, No. 5, September 1997, pp. 4310-4318
74. Deng F, Analytical Modeling of Commutation-Caused Eddy-Current Losses in Permanent-Magnet Brushless Direct-Current
Motors, IEEE Trans. on Magnetics, Vol. 33, September 1997, pp. 4310-4318
75. Deng F and Nehl TW, Analytical Modeling of Eddy-Current Losses Caused by Pulse-Width Modulation Switching in Permanent-
Magnet Brushless Direct-Current Motors, IEEE Trans. on Magnetics, Vol. 34, No. 5, September 1998, pp. 3728-3736
76. Nagarkatti A K, Mohammed O A and Demerdash N A, Special Losses in Rotors of Electronically Commutated Brushless DC
Motors Induced by Non-Uniformly Rotating Armature MMFs, IEEE Transactions on Power Apparatus and Systems, Vol. PAS-101,
No. 12, December 1982, pp. 4502-4506
77. Abu Sharkh SM, Harris MR and Irenji NT, Calculation of rotor eddy-current loss in high-speed PM alternators, Electric
Machines and Drives Conference, Cambridge, UK, September 1997, pp. 170-174
78. Irenji NT, Abu-Sharkh SM, and Harris MR, Effect of rotor sleeve conductivity on rotor eddy-current loss in high-speed PM
machines, ICEM 2000, Espoo, Finland, August 2000, pp. 645-648
79. Fukuma A, Kanazawa S, Miyagi D, and Takahashi N, Investigation of AC Loss of Permanent Magnet of SPM Motor Considering
Hysteresis and Eddy-Current Losses, IEEE Trans. on Magnetics 41 No. 5, 2005, pp. 1964-1967
80. Nakano M, Kometani H, Kawamura M and Ikeda Y, Permanent magnet type dynamo-electric machine and permanent magnet
synchronous generator for wind power generation, United States patent application US 2004/0155537 A1, August 12, 2004
81. Saban DM et al, Experimental evaluation of a high-speed permanent-magnet machine, 55th IEEE Petroleum and Chemical
Industry Technical Conference (PCIC), Cincinnati, Ohio, 22-24 September 2008, pp. 1-9
82. Amos D E (Sandia National Laboratories), Algorithm 644 : A Portable Package for Bessel Functions of a Complex Argument
and Nonnegative Order, ACM Transactions on Mathematical Software, Vol. 12, No. 3, September 1986, pp. 265-273
83. Maver A, CERN, Computer program C303, 600 Series Library, June 1969
84. Abramowitz M and Stegun I A, Handbook of Mathematical Functions, Dover Publications Inc., New York, 1965
85. McLachlan N W, Bessel Functions for Engineers, Oxford University Press, 1934
86. Cooley J W, Lewis P A W and Welch P D, Application of the Fast Fourier Transform to Computation of Fourier Integrals,
Fourier Series, and Convolution Integrals, IEEE Trans., 1967, Vol. AU-15, pp. 79-84
87. Carter F W, AirGap Induction, Electrical World and Engineer, Vol. 38, 1901, pp. 884-888
88. Carter F W, The Magnetic Field of the Dynamo-Electric Machine, J.I.E.E., Vol. 64, 1926, p. 1115
89. Gibbs W J, Tooth-Ripple Losses in Unwound Pole Shoes, J.I.E.E., Vol. 94, 1947, Pt. II, p. 2
90. Binns KJ, Lawrenson PJ and Trowbridge CW, The analytical and numerical solution of electric and magnetic fields, John Wiley
& Sons., Chichester, 1992.
91. Stoll R, The analysis of eddy currents, Monographs in Electrical and Electronic Engineering, Clarendon Press, Oxford, 1974,
ISBN 0 19 859311 2
92. Krawczyk A and Tegopoulos J A, Numerical modelling of eddy currents, Monographs in Electrical and Electronic Engineering,
Clarendon Press, Oxford, 1993, ISBN 0 19 859382 1
93. Lammeraner J and Štafl M, Eddy currents, Iliffe Books Ltd., London, 1966
References Page 2.245

94. Štafl M, Electrodynamics of electrical machines, Iliffe Books Ltd., London, 1967
95. Davies E J, Conduction and induction heating, Peter Peregrinus Ltd., Stevenage, UK, ISBN 0 86341 174 6, 1990
96. Perry M, Low frequency electromagnetic design, Marcel Dekker, N.Y., 1985
97. Alger PL: Induction machines, Gordon and Breach Science Publishers, New York, 2nd edn., 1970. (See p. 183).
98. Heller B and Hamata V, Harmonic effects in induction machines, Elsevier Scientific Publishing Company, Amsterdam, 1977
99. Concordia C, Synchronous Machines, Theory and Performance, John Wiley and Sons, New York, 1951
100. Adkins B, The General Theory of Electrical Machines, Chapman and Hall, London, 1964
101. IEEE Standard 115 : Test Procedures for Synchronous Machines, 1994
102. Ralph MC, Eddy-current effects in the cylindrical members of rotating electrical machinery, PhD thesis, University of Leeds,
1968
Page 2.246 SPEED’s Electric Machines

Index
2Q 94 Eddy currents
Airgap windings 24 causes 172
Alignment torque 80, 126 current-forced 174
Amortisseur 77 inductance-limited 174
Back-emf resistance-limited 174
trapezoidal 3 screening effect 173
Back-emf sensing 34 voltage-forced 174
Bessel functions Eddy-current loss 173
large arguments 191 in magnets 213
small arguments 184 in PC-BDC 176
Boules 15 Electromagnetic torque 74
Bracing Ellipse diagram 81
equivalent magnet 20 EMF constant 4
Bridges 20 Equivalent magnet 18
see Bracing 20 with bracing bridges 20
Chopping frequency Field-oriented control 91, 162
and inductance 41 Finite-element analysis 21
Circle diagram 81, 82 Flux-dip-sweeping 176, 177, 220, 224
Clearance gap 18 Flux-spreading
Coenergy 126 in rotor yoke 22
Cogging torque 126 Flux-weakening 80, 81
Commutation 28 Fourier transform 176, 239
Complex diffusion equation 172 Fringing 14
Concentrated windings 39 Gorges diagram 204
Concentric windings 39 Hague 15
Current regulation 26 Harmonic leakage 43, 59
squarewave 26 Harmonic leakage inductance 168
Current ripple 41 Harmonics
Current-limit circle 81 causing eddy-current loss 172
Damper windings 77 effect on average torque 131
Demagnetization 10, 15 Hunting 77
Demagnetizing Id 80 Hysteresis loss
Differential leakage 43, 59 in magnets 175
of multiplex windings 60 Ideal energy converter 74
Differential leakage inductance 168 Imbalance 204
Diffusion equation 172 Inductance 41
Direct axis 78 effect on maximum speed 84
dq-axis theory 75, 78 from finite-element calculations 72
definition of d and q axes 106 practical importance 41
dq-axis transformation 57, 58 synchronous 54
Duplex 137 Interior-magnet motor 78, 80
Index Page 2.247

Interpolar axis 78 Plex 137


IPM 78, 80 PM alignment torque 80
and squarewave drive 91 PM-assisted reluctance machines 78, 106
magnetically nonlinear 158, 160 Polar axis 78
Irregular slotting 168, 170, 171 Power factor 77
Line-start motors 77 Quadrature axis 78
history 105 Reactance
Losses synchronous 55
causes 172 Regeneration
in rotor 172 using step-up chopper principle 32
Magnet Reluctance torque 80, 126
analysis of magnetic circuit 11 Reynolds number
effect of fringing 3, 14 magnetic 191
effect of pole-arc 5 Russell and Norsworthy
in both d- and q-axes 107 finite-length effect 217
surface-magnet 1 Saliency 1
Magnetic circuit analysis 11 Saliency ratio 87, 106
nonlinear 14 Salient-pole
Magnetic Reynolds number 191 definition 78
Multiplex windings 137 Screening
differential leakage 60 by eddy-currents 173
Negative sequence 204 Segmentation
NeoMax 175 of magnets 213
Nonsalient-pole Sensorless control 34
definition 78 Series inductance 84
Normalized equations 87 Simplex 137
Nuisance gap 18 Sinewave drive
Oscillatory power 74 compared with squarewave drive 149
Park transformation 57, 99, 123, 128 Skin depth 174
Park's transformation 57 formula 173
Parking device 38 Skin effect 220
Per-unit equations 87 Slotless motors 24
Period A 94 Space vector 26, 79, 162
Permasyn motor 105 Spread
Permeance coefficient 13 of phase-belt 40
Permeance harmonics 176, 220 Step-up chopper 32
Phase-belt 40 Strong PM machines 106
Phasor diagram Subtransient reactance 176, 229, 230
interpretation of single-phase 120 Swing frequency 239
on open-circuit 76 Symmetrical components 111, 204, 206
Pitch factor 167 natural 119
Page 2.248 SPEED’s Electric Machines

Synchronous inductance 54, 58 Unipolar drive 35


Synchronous reactance 55 Up converter 32
Synchronous reluctance motors Voltage-limit circle 81
line-start 104 Weak PM machines
Tapered gap 38 PM-assisted reluctance machines 106
Torque 126 Winding
alignment and reluctance 80 concentrated 39
Torque angle 77 concentric 39
Torque constant 4, 80 distributed 39
and EMF constant 165 double-layer 40
effect of phase advance 84 full-pitch 40
Torque per ampere 91, 149, 164, 165 inductance 41
Torque ripple 91, 127 lap-wound 39
Torque/speed characteristic 10 phase-belt 40
normalized form 87 Winding factor 167
Trapezoidal back-emf 3 Windings
Triplex 137 differences between squarewave and sinewave
39
Two phases on 4
Zero sequence 207
Two-axis theory 75, 78
Unbalanced operation 204
3. Induction machines

3. 1 Basic theory . . . . . . . . . . . . . . . . . . . . . . . . . . . . . . . . . . . . . . . . . . . . . . . . . . . . . . . . . . . . . . . 1
3.2 Equivalent circuit . . . . . . . . . . . . . . . . . . . . . . . . . . . . . . . . . . . . . . . . . . . . . . . . . . . . . . . . . . 3
3.3 Phasor diagram . . . . . . . . . . . . . . . . . . . . . . . . . . . . . . . . . . . . . . . . . . . . . . . . . . . . . . . . . . . 10
3.4 Winding factors and other winding-related matters . . . . . . . . . . . . . . . . . . . . . . . . . . . 18
3.5 Rotor and stator slot numbers . . . . . . . . . . . . . . . . . . . . . . . . . . . . . . . . . . . . . . . . . . . . . . . 26
3.6 Calculation of the resistances and reactances of the equivalent circuit . . . . . . . . . . . 28
3.7 Location of ampere-conductors . . . . . . . . . . . . . . . . . . . . . . . . . . . . . . . . . . . . . . . . . . . . . . 41
3.8 Transients and eigenvalue analysis . . . . . . . . . . . . . . . . . . . . . . . . . . . . . . . . . . . . . . . . . . 42
3.9 Split-phase motors . . . . . . . . . . . . . . . . . . . . . . . . . . . . . . . . . . . . . . . . . . . . . . . . . . . . . . . . . 50
3.10 Cross-field theory of tapped-winding capacitor motor . . . . . . . . . . . . . . . . . . . . . . . . . . 62
3.11 Interbar currents . . . . . . . . . . . . . . . . . . . . . . . . . . . . . . . . . . . . . . . . . . . . . . . . . . . . . . . . . . 67
3.12 T-connection . . . . . . . . . . . . . . . . . . . . . . . . . . . . . . . . . . . . . . . . . . . . . . . . . . . . . . . . . . . . . . 68
3.13 Approximate calculation of surface losses . . . . . . . . . . . . . . . . . . . . . . . . . . . . . . . . . . . . 69
3.14 Direct analysis of induction machine using primitive impedance matrix . . . . . . . . . 77
3.15 Power balance in wound-rotor induction machines . . . . . . . . . . . . . . . . . . . . . . . . . . . . 85
3.16 Field-oriented control and space vectors . . . . . . . . . . . . . . . . . . . . . . . . . . . . . . . . . . . . . 101
References . . . . . . . . . . . . . . . . . . . . . . . . . . . . . . . . . . . . . . . . . . . . . . . . . . . . . . . . . . . . . . . . . . . . . . 111
Index . . . . . . . . . . . . . . . . . . . . . . . . . . . . . . . . . . . . . . . . . . . . . . . . . . . . . . . . . . . . . . . . . . . . . . . . . . 113
SEM 3 — ii
3. INDUCTION MACHINES

3. 1 BASIC THEORY
Sine-distributed windings
A sine-distributed winding has a sinusoidal distribution of ampere-conductors around the stator bore,
Fig. 3.1, and produces a sine-distributed flux in the airgap, Fig. 3.2.

Fig. 3.1 Sine-distribution of ampere-conductors Fig. 3.2 2-pole sine-distributed field

Rotating magnetic field


The flux distribution of a single sine-distributed winding is fixed in space: in particular, its orientation
is fixed along the winding axis. In Figs. 3.1 and 3.2 the winding axis is the x-axis, 2 = 0. The flux-density
at any point varies in proportion to the current (neglecting magnetic nonlinearity for the moment).
The effect of the current in a sine-distributed winding is closely approximated by a current-sheet
around the stator bore, K1 = K sin p2 [A/m], such that K d2 is the number of ampere-conductors in an
arc d2 at the stator bore. p is the number of pole-pairs. In Figs. 3.1 and 3.2, p = 1. K1 can be interpreted
as the product of the phase current i1 and the conductor distribution C sin p2. If the current varies
sinusoidally in time, i1 = Imax cos Tt, then the current sheet remains fixed in space but its amplitude at
any point varies sinusoidally: thus K1(2, Tt) = K sin p2 cos Tt.
2-phase winding: If a second "phase" is added, with its axis rotated 90E from that of the first phase, and
if it has the same number of turns as the first phase, it will produce an ampere-conductor distribution
orthogonal to the first one: thus K2 = K cos p2. The axis of phase 2 is the angle at which cos p2 = 0, and
since cos p2 = sin (p2 + 90E), this is at !90E/p. If phase 2 carries a sinewave current which leads i1 by
90E in time, we can write i2 = Imax cos (Tt + 90E) = !Imax sin (Tt), and therefore K2 = !K cos p2 sin(Tt).
The total ampere-conductor distribution is the sum of K1 and K2:
K (2, T t) ' K (sin p 2 cos T t ! cos p 2 sin T t) ' K sin (p 2 ! T t) (1)
which is interpreted as a rotating ampere-conductor distribution. The rotation speed is T/p rad/s: or

60 T 120 × f
Ns ' × ' [rpm] (2)
2B p poles

since T = 2B f, where f is the supply frequency in [Hz]. This is the synchronous speed. If the polarity
of one winding was reversed, the ampere-conductor distribution would rotate in the opposite direction.
Similarly, if the current I2 was lagging I1 by 90E, it would reverse the direction of rotation.
Page 3.2 SPEED’s Electric Machines

3-phase windings: Three-phase windings are much more


common in practice than 2-phase windings. Fig. 3.3 shows
the arrangement of three sine-distributed phase windings
arranged symmetrically so that their axes are displaced by
120E from each other. Balanced three-phase currents supply
these windings, so that the total ampere-conductor
distribution is given by
K(2, T t) ' K sin p 2 cos Tt
K sin (p 2 ! 2B /3) cos (Tt & 2B /3)
K sin (p 2 2B /3) cos (Tt 2B /3) (3)
3
' K sin (p 2 & Tt) .
2
Fig. 3.3 3-phase winding

This represents a rotating ampere-conductor distribution whose peak value is 3/2 times the peak value
produced by any one phase. It rotates at T/p rad/s.
Multiple poles: The ampere-conductor distributions shown in Figs. 3.1!3. 3 are two-pole distributions
because there is only one North and one South. Windings can have any number of pole-pairs. For
example, a four-pole winding (p = 2) has NSNS and a six-pole winding (p = 3) has NSNSNS.
Electrical and mechanical radians and degrees: In a P-pole winding the pole-pitch is 2B/P radians,
i.e. B /p since p = P/2. These radians are actual or mechanical radians. However, it is logical to think
of a pole-pitch as spanning B electrical radians, where one electrical radian is equal to 2/P mechanical
radians or 1/p mechanical radians. The synchronous speed is T electrical rad/s, or T/p mechanical
rad/sec. In rpm it is given by eqn. (3.2). Common examples are tabulated below:

Poles Frequency Speed [rpm] Frequency Speed [rpm]


2 3,000 3,600
4 1,500 1,800
50 60
6 1,000 1,200
8 750 900
Table 3.1
SYNCHRONOUS SPEED VS. FREQUENCY AND NUMBER OF POLES

In order to achieve a synchronous speed of 24,000 rpm in a 2-pole motor, the frequency would need to
be f = 400 Hz. To rotate an 8-pole motor at 15 rpm, the frequency would need to be 1 Hz.
Rotor speed and frequency
In the induction motor, the rotor winding is usually short-circuited: that is, a closed winding that links
the flux produced by the stator windings, but is not connected to the supply. The currents flowing in
the rotor are induced, and in order to obtain a driving EMF the rotor must rotate at some speed different
from synchronous speed. For if the rotor were rotating at synchronous speed, the flux linkage of the
rotor winding would not vary, and there would be no induced EMF.
The relative speed between the rotor and the rotating flux is called the slip speed. If the rotor speed is
Tr elec rad/sec, the slip is defined as
T ! Tr
s ' [p.u.] . (4)
T

In general, the induction motor rotates slightly slower than synchronous speed and s > 0. The induction
generator rotates slightly faster than synchronous speed, and s < 0. In variable-frequency motor drives,
negative slip is used for braking purposes because it reverses the torque. The rotor speed is also
expressed as Tr = (1 ! s) T or Nr = 120 f /P × (1 ! s) rpm. The relative speed between the rotating flux
and the rotor is T ! Tr = s T elec rad/s. Therefore the frequency of EMF's and currents induced in the
rotor is equal to s f ; i.e., the slip is the ratio of the rotor frequency to the stator frequency.
Induction machines Page 3.3

In polyphase induction motors designed for high efficiency, the full-load slip is generally small (< 5%),
but in small single-phase machines it can be higher than this. For example, a 4-pole motor running at
2.5% slip from a 60-Hz supply is rotating at (1 ! 0.025) × 1,800 = 1,755 rpm. The slip speed is 45 rpm and
the rotor frequency is 0.025 × 60 = 1.5 Hz.
The slip-frequency EMF’s induced in the rotor drive slip-frequency currents in the rotor conductors,
producing a secondary ampere-conductor distribution which rotates forwards in synchronism with the
stator ampere-conductor distribution, i.e. at synchronous speed. The rotor ampere-conductor distribution
therefore rotates forwards relative to the rotor, at the relative speed s T electrical rad/s. Because both
ampere-conductor distributions rotate at synchronous speed relative to the stator, any voltages induced
in the stator windings by will be at the line frequency f. There is only one resultant magnetic flux,
produced by the combined effect of the rotor and stator. Provided that the windings are sine-
distributed, it rotates at synchronous speed.

Fig. 3.4 Equivalent circuit of one phase of polyphase induction motor, with different frequencies in the primary and secondary.
The ideal transformer has the effective primary:secondary turns ratio n1 /n2 × 1/s for voltage and n2/n1 for current. Here
the circuit is drawn as though n1 /n2 = 1. Rc accounts for the core loss, and Xm for the magnetizing current.

3.2 EQUIVALENT CIRCUIT


The induction motor rotor is similar to the secondary of a transformer, with two major differences:
(a) the rotor is rotating relative to the primary, so the frequency in the secondary is s f ; and
(b) the airgap decreases the coupling between the primary and secondary, so that the leakage
inductances are higher than in a typical power transformer. Consequently the magnetizing
reactance is lower and the magnetizing current is higher.
In all other respects the equivalent circuit is the same as for the transformer, shown in Fig. 3.4 for one
phase. The rotor frequency s f is usually in the range 0.5!5 Hz in normal operation.
To simplify analysis we refer all parameters to the primary (stator) winding. To do this we need the
transformation ratios for the voltage, current, and impedance. By Faraday's Law, the sinusoidal
induced voltage in each winding is proportional to the turns and the frequency. In a transformer the
voltage ratio is n1 f/n2 f = n1 /n2, the turns ratio, where n1 and n2 are the effective numbers of turns in
the stator and rotor windings, respectively: the frequency cancels because it is the same in both
windings. In the induction motor the voltage ratio is n1 f/n2 sf = n1/sn2.
The current ratio is n2/n1, exactly as in the transformer : current balance is a magnetostatic effect
controlled by Ampere's Law, and is unaffected by frequency.
For impedance the referral ratio is the quotient of the voltage and current ratios, i.e., n12/sn22.
Page 3.4 SPEED’s Electric Machines

Fig. 3.5 Equivalent circuit of one phase of polyphase induction motor. The frequency is the line frequency in both primary and
secondary, and the turns ratio is eliminated. The EMF Erb is used to represent leakage flux in saturated rotor slot-
bridges, and does not contribute to torque production. Boxed variables appear in the phasor diagram, Fig. 3.11.

It is now possible to refer all voltages, currents, and impedances in Fig. 3.4 to the stator winding, as in
Fig. 3.5. All voltages and currents in Fig. 3.5 are at line frequency f, and all reactances have their line-
frequency values. While the secondary EMF and reactance lose their dependence on slip, the rotor
resistance becomes R2 /s, and this is further divided into two components in Fig. 3.5, using
R2 1
' R2 R2 ! 1. (5)
s s
2
The first term represents I R loss, while the second term represents electromechanical power
conversion. The referred equivalent circuit (Fig. 3.5 ) shows that any increase in rotor current appears
as an increase in the stator current I1 drawn from the supply. It can be used to calculate all the
important currents and voltages in the induction motor, as well as the torque, power, and efficiency.
Power, torque and efficiency
The current I2 flows in the referred rotor resistance R2/s and causes an apparent power dissipation of

I22 R2
Pgap ' 3 (6)
s
where the 3 accounts for all three phases. This is the electromechanical power or airgap power, i.e. the
power transferred across the airgap from the stator to the rotor. The actual copper loss in the rotor,
however, is only 3I22R2, and since s < 1 (in motoring operation), the airgap power exceeds the rotor
2 2
copper loss by 3I2 R2/s ! 3I2 R2 = Pgap (1 ! s). The electromechanical power is therefore
Pmech ' Pgap ( 1 & s ) . (7)
Since the actual rotor speed is (1 ! s) T/p rad/s, electromagnetic torque must be
Pmech p
Tgap ' ' Pgap [Nm] . (8)
(1 ! s) T T
The airgap power Pgap divides into two. A fraction s goes as rotor copper loss (heating the rotor). The
remaining fraction (1 ! s) is converted into mechanical power at a speed that is equal to (1 ! s) times the
synchronous speed, producing the electromagnetic torque Tgap. The shaft torque Tshaft is less than the
electromagnetic torque because of friction torque Tfric :
Tshaft ' T & Tfric . (9)
The efficiency is now calculated as
Induction machines Page 3.5

Tm × Tshaft
0 ' × 100% , (10)
3 V1 I1 cos N1
where cos N1 is the power factor and Tm = Tr/p is the speed in mechanical radians/sec. Note that V1 is
the phase voltage, since the equivalent circuit is on a per-phase basis. Likewise, I1 is the phase current.
If the motor is connected in wye and the phases are balanced, V1 = VL//3 and I1 = IL, where VL is the
line-line voltage and IL is the line current. If it is connected in delta, V1 = VL and I1 = IL//3.

Effect of slip on torque, efficiency and power factor


Induction motors are usually designed to operate with low values of slip, typically 0.01!0.05. This
increases the fraction of airgap power that is converted to mechanical power, and decreases the fraction
that is dissipated as rotor I 2R loss, helping to maximise the efficiency. A small value of s increases the
ratio (R2/s)/X2, which also improves the power factor.
The speed during normal motoring operation is just below synchronous speed, and it changes only
slightly as the load torque changes. For this reason the induction motor is often described as a "constant
speed" machine, provided that the supply frequency is fixed. From the equivalent circuit (Fig. 3.5), if
we neglect the no-load current Inl and lump the leakage reactances together as XL = X1 + X2,
V1
I2 ' (11)
( R1 R2 / s ) j X L
If this is substituted into eqn. (3.6) for Pgap, then from eqn. (3.8) the airgap torque is given by

3p R2 V12
Tgap ' × × [Nm] . (12)
T s ( R 1 R 2 / s ) 2 X L2

This equation relates the electromagnetic torque to the slip, when the supply voltage and frequency are
fixed. It can therefore be used to plot a graph of torque vs. slip. Since the speed in rev/min is given by
(1 ! s) Ns, the graph also shows the variation of speed with torque. Fig. 3.6 shows a typical example.
At speeds near synchronous speed, s 6 0 and

V12
T 6 3p
T
×
R2
× s, (13)

i.e., the torque is proportional to slip. Since the slip is generally small for torques which are less than
or equal to the rated load torque, the speed remains near synchronous speed as the torque varies. The
actual torque is determined by the load, and the operating point is at the intersection of the motor's
torque/speed characteristic with the torque/speed characteristic of the load, Fig. 3.6.
If the speed rises above synchronous speed, the slip becomes negative and the torque changes sign. This
is called regeneration or braking. If the induction machine is driven (for example, by a wind turbine)
at speeds above synchronous speed, it generates and feeds power into the AC supply system.
At low speed, s 6 1 and XL tends to exceed (R1 + R2/s), so that

V12 R2
T 6 3p
T
×
XL 2
1
× ,
s
(14)

i.e., the torque is inversely proportional to slip at low speed. The locked-rotor torque is the value at
standstill, with s =1. This is the torque when the motor is first switched on.
Page 3.6 SPEED’s Electric Machines

Fig. 3.6 Torque/speed characteristic

Breakdown torque
Fig. 3.6 shows that there is a maximum torque Tmax, occurring at a value of slip slightly beyond the
linear section of the torque/slip curve. This value of slip can be estimated by differentiating the torque
equation with respect to slip, and setting the derivative to zero. The result is

R2
sTmax ' (15)
R12 XL2

The corresponding maximum torque is called the breakdown torque:

1 3p V12
Tmax ' × × [Nm] . (16)
2 T R1 R12 XL2

If the load is gradually increased above the rated load, the slip increases and the motor produces more
torque. The operating point moves up the torque/slip curve until the slip reaches sTmax. Any increase
of load torque then causes the motor to slow down still more, increasing the slip. Beyond sTmax the motor
torque decreases, and the motor rapidly decelerates and stalls; this is called breakdown.
Induction motors are normally rated such that at rated voltage and rated frequency the rated torque is
roughly half the breakdown torque. This provides a margin of safety against stalling due to transient
2
changes of load torque, or undervoltage or underfrequency conditions. Since Tmax is proportional to V1 ,
a 10% reduction in voltage produces a 20% reduction in breakdown torque.
Induction machines Page 3.7

Locked-rotor current and torque, and their dependence on temperature


Neglecting magnetizing current, the current at locked rotor is
V1 V1
ILR ' ' (17)
(R1  R2 )2  (X1  X2 )2 (R1  R2 )2  XL2
where XL ' X1  X2 is the total leakage reactance. If the temperature increases, R1 and R2 both increase,
while XL is unaffected. Therefore the current must decrease as the temperature increases. When the
current decreases, XL increases because it becomes less saturated. Therefore the total decrease in
current must be slightly greater than the decrease due to temperature alone.
Locked-rotor torque is proportional to rotor loss, which is mainly copper loss. Therefore we can write

TLR ' k ILR2 R2 , (18)

where k is a constant. Substituting ILR from eqn. (3.17),

kV 2
TLR ' R2 . (19)
(R1  R2 )2  XL2
When the temperature increases, the denominator increases and so does the numerator. Whether the
torque increases or decreases depends on the value of dTLR/dJ, where J is the temperature.
Assume that R1 and R2 both have the same temperature coefficient, ". If XL << R1  R2, the denominator
in eqn. (3.19) increases faster than the numerator as the temperature increases, and the torque will
decrease. On the other hand, if XL >> R1  R2, the numerator will increase with temperature while the
denominator remains nearly constant, so the torque will increase with temperature. To summarize,

Ratio XL/(R1  R2) Effect of temperature increase

High TLR increases


ILR decreases
Low TLR decreases

There must be a critical value of XL for which there is no rate of change of torque with temperature,
i.e. dTLR/dJ ' 0. To simplify the calculation write R1  R2 ' aR2. Then

k V12
TLR ' R2 (20)
(a R2 )2  XL2
and

d TLR d TLR d R2 (aR2)2  XL2  2 (aR2)2 XL2  (aR2)2


' ' k V12 " ' k V12 " (21)
dJ d R2 d J (aR2)2  XL2
2
(aR2)2  XL2
2

To get dTLR/dJ ' 0 we must have


XL ' aR2 ' R1  R2. (22)

Since R1  R2 is a function of temperature, this condition cannot be sustained over a range of


temperature : strictly speaking, it occurs at only one temperature. However, the rate of change of
resistance with temperature being relatively small, the locked-rotor torque can remain approximately
constant over a limited temperature range, if the condition expressed by eqn. (3.22) is satisfied in this
range. Ultimately if the temperature rises high enough, R1  R2 will exceed XL to such an extent that the
torque begins to decrease, even if it began by increasing, as noted earlier.
Eqn. (3.22) suggests an interesting method for measuring XL and (R1  R2): both are equal to 1//2 times
the locked-rotor impedance per phase at the temperature at which dTLR/dJ ' 0 and TLR(J) is maximum.
Page 3.8 SPEED’s Electric Machines

Minimum current
It might be expected that the minimum line current would occur at
synchronous speed with zero slip, when the resistance R2/s is infinite.
However, if the phase resistance is relatively large, the minimum
current may occur at a finite value of slip. This can be explained with
reference to the simplified equivalent circuit in Fig. 3.7, in which the
rotor leakage reactance is neglected to simplify the analysis slightly.
Consider just the magnetizing current Im, i.e., the current through Xm. Fig. 3.7 Circuit
It is represented in the phasor diagram Fig. 3.8 by OA, and the voltage across Xm is represented by OE.
If the impedance Z1 is resistive, the voltage drop across it is represented by EG, so the terminal voltage
V is the vector sum of OE and EG, that is, OG. Now add the current I2 through R2, AB, so the total
current I becomes OB. This additional current causes an additional voltage drop GH across Z1, so the
terminal voltage becomes OH. While the current I increases from OA to OB, the terminal voltage V
increases from OG to OH. It is possible for the ratio OH/OG to exceed the ratio OB/OA, i.e. OH/OB >
OG/OA, meaning that the impedance increases when current is flowing through R2, even though the
impedance Zm ' R22Xm is less than Xm.
If Z1 is inductive, with only I2 ' OA flowing the voltage drop across it is ES, which leads the current by
90E and is equal in magnitude to EG. When I2 is added, the additional voltage drop is ST, leading AB by
90E. The effect of adding the current through R2 is to increase the terminal voltage from OS to OT, with
hardly any change in magnitude.

Fig. 3.8 Phasor diagram with Z1 ' 30 ohm

In Fig. 3.8 Im ' OA ' j1 A and I2 ' AB ' 0@2 A. The voltage OE ' 100 V. With resistive Z1, EG ' j30 V, and
j16@699E j73@301E
V ' OG ' 104@403e . The apparent impedance at the terminals is OG/OA ' 104@403e ohm.
j78@690E
When I2 is added, the current I increases to OB ' 1@020e A while the voltage V increases to OH '
j15@803E j62@887E
110@164e . The apparent impedance becomes OH/OB ' 108@025e ohm, an increase of 3@5%.
If Z1 is inductive, then with only Im flowing the voltage drop across Z1 is ES ' 30V, and the terminal
voltage is OS ' 130 V. The apparent impedance at the terminals is OS/OA ' j130 ohm. When I2 is added,
the additional voltage drop ST ' j6V appears across it, increasing the terminal voltage to OT '
j2@643E j81@333E
130@138e V. The apparent impedance becomes OT/OB ' 127@611e ohm, a decrease of 1@9%.
This shows that with a sufficiently resistive impedance Z1, the maximum driving-point impedance at
the terminals does not occur at zero slip, but at a finite value of slip. In practice it is likely that only very
small induction motors will have sufficient resistance to bring about this condition, and even then the
minimum current will be close to the current at synchronous speed.
Induction machines Page 3.9

Speed control
Fixed supply: If the supply voltage and frequency are fixed, the speed can be controlled by varying the
rotor resistance by means of an external resistor connected into the rotor circuit by slip-rings and
brushes. This technique is used in large wound-rotor induction motors, especially for controlling the
rate at which they start up. The effect is to change the torque/speed characteristic as shown in Fig. 3.9.
A high resistance R2 maximizes the starting torque. As the rotor accelerates the external resistance is
shorted out and the characteristic changes to a "low-slip, high-efficiency" characteristic. Wound-rotor
machines are expensive and are relatively less common than cage-rotor machines.

Fig. 3.9 Varying R2 to achieve high starting torque Fig. 3.10 Varying the supply frequency at constant V/Hz

Pole amplitude modulation: With a fixed supply, the speed of a cage-rotor machine can be changed
by reconnecting the stator windings in such a way as to change the pole number. For example,
reconnecting the stator from 6 poles to 8 poles reduces the synchronous speed by 25%. Otherwise there
is no practical way to control the synchronous speed of cage-rotor machines on a fixed supply.
Variable voltage: Changing the voltage causes the torque to be scaled in proportion to V12, so the
operating point moves to a higher or lower speed as the slope of the torque/slip curve changes. The
range of speed variation is small, unless the motor is designed with a high rotor resistance, but this
makes the motor inefficient. This technique is used with single-phase and inexpensive triac controllers.
Variable frequency: The ideal way to control the speed of an induction motor is by varying the supply
frequency. This causes the torque/slip curve to be translated along the speed axis, Fig. 3.10. If the
voltage/frequency ratio is kept constant ("constant volts/Hz"), the breakdown torque remains constant
over most of the speed range; at lower speeds it tends to fall as the stator resistance begins to become
significant compared with the leakage reactance. With this type of drive the slip for a given torque can
be held constant while the speed is varied (almost proportional to frequency).
Modern field-oriented drives are capable of extremely rapid torque response. In principle they operate
by orienting the stator MMF distribution at an optimal angle relative to the flux trapped by the rotor
currents, and under transient conditions they are not limited to sinusoidal current. However, the
equivalent circuit model is still the basis of analysis and design of these drives.
Double-cage and deep-bar rotors: Some induction motors are designed with a double cage. The inner
cage has a high leakage reactance and a low resistance, and the outer cage has a low reactance and a
high resistance. The resulting torque/speed characteristic is similar to the sum of the high-resistance
and low-resistance curves in Fig. 3.9, providing high starting torque and low operating slip (therefore
high efficiency) in the one motor. As an alternative to the double cage, skin-effect is used in the deep-bar
rotor to increase the rotor R/X ratio as the slip (and therefore the rotor frequency) increases.
Page 3.10 SPEED’s Electric Machines

3.3 PHASOR DIAGRAM


The phasor diagram for one phase of a balanced polyphase induction machine corresponding to Fig. 3.5
is shown in Fig. 3.11. The phasor diagram represents the steady-state operation of the equivalent circuit
under balanced conditions. Iterative solution of the phasor diagram makes it possible to allow for
nonlinearities such as saturation of the magnetizing reactance and leakage reactance, the effect of slip
on the rotor bar resistance, and the variation of core loss and stray loss with the flux level. However, it
should be remembered that the phasor analysis is based on the fundamental space-harmonic MMF and
is limited to sinusoidal waveforms of voltage and current. Therefore, “parasitic” effects such as stray
loss, or harmonic effects, must somehow be buried in the phasor diagram through modifications to the
equivalent-circuit parameters.

V1

E1 VZ1 VX1
Erb
 VR1
VZ2
VX2
ER2
VR2
I2
Irc

Inl

Imag
I1

Fig. 3.11 Phasor diagram for one phase of a balanced polyphase induction motor. The
phasor Erb represents rotor slot bridge leakage. Several of the voltages and
currents are marked on Fig. 3.5.

Closed rotor slots


Closed rotor slots have bridges that cannot be modelled by a fixed component of the rotor slot
permeance, because they tend to saturate. A simple method of modelling these bridges is to use a fixed
induced voltage Erb that is proportional to the flux through the bridges that close the rotor slots. It is
assumed that there will be a sine-distributed component of the airgap flux that enters the rotor radially
and then travels circumferentially around the rotor, via the bridges. At a position 90Eelec from the point
of peak airgap flux-density, the bridge flux density will be a maximum, and this density is arbitrarily
assumed to be 2.1T. When this is multiplied by the cross-section area of the bridge, the resulting flux
is a component of the fundamental flux in parallel with (and in phase with) the leakage flux through X2.
Erb is shown in both Fig. 3.5 and Fig. 3.11.

Alger’s equivalent circuit


The classical equivalent circuit model is based on the fundamental space-harmonic of the airgap MMF.
To include the effect of phase belt and slot-permeance harmonics, Alger [1970] proposed an extended
equivalent circuit such as the one in Fig. 3.12, which includes two sets of phase-belt harmonics (5th and
7th); the forward and backward slot permeance harmonics of order S1/p±1, and the forward and
backward slot-MMF harmonics of order S1/p±1, where S1 = stator slots and p = pole-pairs.
Induction machines Page 3.11

I1 R1 jX 1 I2
Second cage

V1
jX m Rc

Im

jX 2
b
jX 2m
jX pm R2
R2m
Slot permeance 1 + m(1!s)
harmonics
f jX 2n

jX pn
R2n
1 ! n(1!s)

b jX 25
jX m5
R25
6 ! 5s
Phasebelt
harmonics

f jX 27
jX m7
R27
7s ! 6

b jX 2m
jX mm
R2m
Slot MMF 1 + m(1!s)
(zigzag)
harmonics
f jX 2n
jX mn
R2n
1 ! n(1!s)

Fig. 3.12 Harmonic equivalent circuit for polyphase machines given by Alger [1970]

In a 3-phase motor the MMF wave produced by the 5th harmonic of the winding distribution rotates in
the backward direction at 1/5 synchronous speed, and the MMF wave produced by the 7th harmonic
rotates in the forward direction at 1/7 synchronous speed. Likewise the MMF waves of the (6k ! 1)th
winding harmonic rotate backwards, and those of the (6k + 1)th winding harmonic rotate forwards.
In a two-phase motor the MMF wave produced by the 5th harmonic of the winding distribution rotates
in the forward direction at 1/5 synchronous speed, and the MMF wave produced by the 7th harmonic
rotates in the backward direction at 1/7 synchronous speed. These directions are opposite to those in
the three-phase motor.
Page 3.12 SPEED’s Electric Machines

Alternative forms of the equivalent circuit


The equivalent circuit simplifies and explains
the basic operation of an immensely complicated
device. The operation of the induction motor
could hardly be explained or calculated so
clearly by any other method. The equivalent
circuit is extremely convenient for calculation.
It has been used by designers for over 100 years,
and is not about to be replaced.
The apparent simplicity of the equivalent circuit
hides many complex phenomena, such as
magnetic saturation, temperature-induced
changes, skin effect, and others. These effects
cause differences between the ideal model and
test data. They have the effect of altering the
values of the equivalent-circuit impedances,
while its form and operation remain the same, at
least for balanced steady-state operation.
However, it is useful to study certain important
variants of the equivalent circuit, especially the
ones shown in Figs. 3.13 and 3.14.
In the conventional T circuit of Fig. 3.13(a), the
magnetizing reactance Xm is connected between
the two leakage reactances X1 and X2. The
induced voltage E1 across Xm is associated with
the fundamental space-harmonic component of
Fig. 3.13 Alternative forms of the equivalent circuit
the rotating airgap flux.
The three variants in Fig. 3.13 differ only in the location of the core-loss resistor Rc. This fictitious
resistance is variable because the core loss varies (mainly as a function of voltage and frequency). Also
the value of E1 varies as a function of the supply voltage and the load. Except at a single load-point, the
core loss cannot be accurately represented by a fixed value of Rc : instead, it should be calculated
independently from the flux-density waveforms in different parts of the magnetic circuit, and then Rc
should be assigned a value consistent with the core loss at each load point.
The value of Rc also depends on where it is connected in the circuit, and Fig. 3.13 shows three
alternatives. In the conventional circuit of Fig. 3.13(a), Rc is connected across Xm. It therefore
represents iron loss associated with the fundamental space-harmonic component of the rotating airgap
2
flux. This loss is calculated as E1 /Rc watts per phase.
In Fig. 3.13(b) Rc can be said to be connected across all the flux linking the stator winding, and not just
the fundamental airgap component. In this location it represents additional components of loss in the
iron associated with leakage components of the flux, such as slot-leakage flux.
Fig. 3.13(b) has an important advantage over Fig. 3.13(a). As will be seen later, the “segregation” of
leakage reactance between X1 and X2 is fundamentally arbitrary. As is well known, this presents a
theoretical difficulty when comparing calculated values of X1 and X2 with values determined from the
no-load and locked-rotor tests (p. 16). Relocating Rc to the position in Fig. 3.13(b) makes it possible to
transform the T circuit into the L circuit, which has only a single leakage reactance XL that can be
unambiguously compared with a test value. This issue is treated rigorously in the following pages.
Partly because of the difficulties of the arbitrary segregation of leakage reactance, and partly for
convenience in calculation, Rc is sometimes located at the stator terminals as shown in Fig. 3.13(c). The
voltage across Rc now includes the voltage drop across the stator resistance, which is hard to justify in
physical terms. Really the only value of this circuit is convenience in manual calculations, since it
minimizes the number of inversions of complex impedances and admittances needed to compute the
currents etc. For computer calculations, there is no great need for this simplification.
Induction machines Page 3.13

T, L and Γ forms of the equivalent circuit


I1 R1 X1 X2
Distinguishing between X1 and X2
I2
The conventional equivalent circuit is V1 U1
represented by circuit T in Fig. 3.14, except that R2
Rc Xm
the core-loss resistor has been moved from its T s
usual position in parallel with Xm to a position
“upstream” of the stator leakage reactance X1.1
The relocation of Rc leaves all the inductive
elements X1, X2 and Xm together in a T- I1 R1  X1 2 X 2
connection in the centre of the circuit. This T-
connection can be replaced by the Γ circuit or I 2
V1
the L circuit in Fig. 3.14, or even by a Π XL
 Xm R
connection (not shown), all of which are L
Rc 2 2
s
electrically indistinguishable from one another
when “viewed” from the terminals. The proof of
this is implicit in the equations which follow.
There is a degree of freedom in apportioning the I1 R1 X1  X2
leakage reactance between X1 and X2 : that is,
between the stator and the rotor. This degree of I 2
V1
freedom can be associated with a parameter " XL
that may be freely chosen within certain limits.  Xm R
Γ
Rc 2 2
It is illustrated in Fig. 3.15, which will be derived s
rigorously a little later. By varying " it is
possible to produce an infinite set of circuits
between the extremes represented by the Γ
circuit (for which X2(") ' 0) or the L circuit (for
Fig. 3.14 T, L and Γ forms of the equivalent circuit
which X1(") ' 0).
As " varies, it varies not only the location of the magnetizing reactance and the division of leakage
reactance between stator and rotor, but also the values of all the reactances as well as the effective value
of rotor resistance.
On the next page, the equations of Fig. 3.15 will be established, and it will be shown that the Γ circuit and
the L circuit are special cases with particular values of ". After that, the circuits will be used to assist
in the interpretation of impedances measured in the locked-rotor and no-load tests.

Fig. 3.15 Generalized form of the equivalent circuit

1
The conventional position of Rc in parallel with Xm reflects the assumption that the core loss is proportional to the square of the
fundamental component of the mutual flux between the stator and rotor, which is only an approximation. There is no circuit
element in Fig. 3.14 to represent the “stray-load loss” caused by space-harmonic fluxes and slot-leakage fluxes. It can be argued
that relocating Rc to the new position in Fig. 3.14 is just as effective as having it in the conventional position, especially if the core
loss and the stray-load loss are calculated outside the equivalent circuit, for in that case Rc can be assigned the value 3U12/W,
where W is the sum of the core loss and the stray-load loss at any load point.
Page 3.14 SPEED’s Electric Machines

The conventional equivalent circuit is represented by eqn. (3.23), in which the reactances X1, X2 and Xm
are represented by the corresponding inductances L1 ' X1/T, L2 ' X2/T and Lm ' Xm/T, and the total
induced voltages U1 andU2 are replaced by the corresponding flux-linkages Q1 ' U1/T and Q2 ' U2/T. The
inductances L1, L2 and Lm are assumed known. Also T ' 2Bf, where f is the stator supply frequency.
Q1 ' (L1  Lm) I1  Lm I2 ;
(23)
Q2 ' Lm I1  (L2  Lm) I2 .

Let
Q2" ' " Q2 ;
I2 (24)
I2" ' .
Substituting in eqn. (3.23), we get "

Q1 ' (L1  Lm) I1  " Lm I2" ;


(25)
Q2 " ' " Lm I1  "2 (L2  Lm) I2" .

which can be written in matrix form as

Q1 L1  Lm " Lm I1
' @ . (26)
Q2" " Lm 2
" (L2  Lm) I2"

Eqn. (3.24) is equivalent to a turns-ratio transformation, and if the rotor circuit is permanently short-
circuited while measurements are made only at the stator terminals, " can be arbitrarily chosen. If "
' 1 we get the conventional equivalent circuit, T in Fig. 3.14. Note that the transformation in eqn. (3.24)
2
requires the rotor resistance R2 to be multiplied by " . To maintain the correct value of rotor copper loss
2
and the same torque, we must use " R2 instead of R2.
Consider what happens if L1
" ' 1  . (27)
Lm
Then eqn. (3.26) becomes

Q1 " Lm " Lm I1
' @ (28)
Q2" " Lm " Lm  LL I2"
where

LL ' "L1  "2 L2 . (29)


Eqns. (3.28) and (3.29) represent circuit L in Fig. 3.14, in which the leakage inductances L1 and L2 are
combined in a single leakage inductance LL, with corresponding reactance XL ' TLL. This leakage
inductance is wholly “in the rotor circuit”.
Now consider what happens if
1
" ' .
L2 (30)
1 
Lm
Then eqn. (3.26) becomes

Q1 " Lm  LL " Lm I1
' @ (31)
Q2" " Lm " Lm I2"
where
LL ' L1  " L2 . (32)
Eqns. (3.31) and (3.32) represent circuit Γ in Fig. 3.14, in which the leakage inductances L1 and L2 are
again combined in a single leakage inductance LL, but this time it is wholly “in the stator circuit”.
Induction machines Page 3.15

We can see that by varying ", the leakage inductance can be “distributed” or “apportioned” between the
stator or rotor circuit at will, provided that the values of L1, L2 and Lm (X1, X2 and Xm ) are adjusted
correctly as indicated in Fig. 3.14. This freedom is equivalent to the freedom to place Xm anywhere
between the extreme positions indicated in Fig. 3.14, provided that its value is multiplied by ". The
slider in Fig. 3.15 expresses the freedom associated with the parameter ". When " ' 1, L1 and L2 have the
original values postulated in circuit T in Fig. 3.14.
The most general case is obtained by rearranging eqn. (3.26) as eqn. (3.34), in which Qm is the mutual
flux-linkage given by
Qm ' " Lm ( I1  I2 " ) . (33)
Then we can write

Q1  Qm Q1  " Lm (I1  I2 ") L1( " ) 0 I1


' ' @ . (34)
Q2 "  Qm Q2 "  " Lm (I1  I2 ") 0 L2( " ) I2"

where L1( " ) ' L1  ( 1  " ) Lm (35)


and L2( " ) ' "2 L2  " ( 1  " ) Lm . (36)
This rearrangement isolates the stator and rotor leakage inductances L1(") and L2(") and the
magnetizing inductance Lm(") ' "Lm. When " ' 1, L1(") ' L1, L2(") ' L2, and Lm(") ' Lm, and we have the
T circuit. When " is given by eqn. (3.27), we have the L circuit, and L1(") ' 0 while L2(") ' LL as given by
eqn.(3.29). When " is given by eqn. (3.30), we have the Γ circuit, and L1(") ' LL as given by eqn. (3.32),
while L2(") ' 0.
Note that in the generalized circuit of Fig. 3.15 it is possible to choose " in order to obtain any arbitrary
ratio P between L1(") and L2("). To achieve this we must solve the following equation for " :
L1(") L1  (1  ") Lm
' ' P. (37)
L2(") "2 L2  " (1  ") Lm
Obviously this is possible only if L1, L2, and Lm are known a priori.
In conventional design calculations L1, L2, and Lm are often calculated independently. The theory used
for such calculations tacitly assumes that the three values are indeed independent. However, it is
obvious that individual values of X1 and X2 (or L1 and L2) cannot be determined by measurement at the
terminals of circuit L or circuit Γ, since these circuits both lump X1 and X2 together in a single reactance
XL ' (X1  X2). They must therefore always appear in the form of this sum in any expression for the
terminal impedance. It follows that the T circuit also cannot yield unique individual values of X1 and
X2 from measurements at its terminals, since it is electrically identical to the L and Γ circuits.
The non-uniqueness of X1 and X2 also extends to Xm and R2. Although it is quite well known, it is a
nuisance for design engineers who need to be able to deal with definite unique values for all these
impedances. The problem was analyzed by Veinott [5], who also discussed the practical consequences.
One of these is that “the apparent rotor resistance, as measured by the locked-rotor test, is always less than
the actual rotor resistance”. In discussing the measurement of the impedances Veinott observed that
there is “no way of separating X1 from X2, so they are often assumed to be equal to one another”. He
showed how to obtain precise values of “R2” and “X1  X2”, but did not attempt to distinguish X1 from X2
beyond assuming that they are equal.
The assumption that X1 ' X2 is quite restrictive, and in the next-but-one section on p. 17 it is shown that
measured impedances can be used to generate a T circuit in which the ratio of X1 and X2 can have any
arbitrary value, while the resulting values of Xm and R2 are rigorously consistent with the
measurements.
Page 3.16 SPEED’s Electric Machines

No-load and locked-rotor tests


These tests are normally set up with reference to
circuit T in Fig. 3.14, but here they will be
described in terms of circuit L, Fig. 3.16. This
permits the arbitrary division between X1 and X2
to be dealt with by the theory described above.
The unknown impedances to be determined are
L L L L
Rc , XL , Xm , and R2 , where the superscript L
denotes “measured” in terms of the L-circuit.
The no-load test is conducted with normal voltage
and no load, so that the speed is very near
synchronous and s
0. The complex input
impedance per phase is therefore

ZNL ' R1  Rc L j Xm L (R2 L / s  j XL L) . (38)


L
If s ' 0, the branch containing R2 /s has infinite
impedance and I2" ' 0, as in Fig. 3.16. Then
1 1 1
 ' . (39) Fig. 3.16 No-load and locked-rotor tests (L circuit)
Rc L
j Xm L ZNL  R1

The no-load impedance per phase is obtained from the complex power per phase :

V12
P0  j Q0 ' V1 I1( ' (40)
(
ZNL
so that
V12
ZNL ' . (41)
P0  j Q 0
L L
When this is substituted in eqn. (3.39), Rc and Xm are obtained directly.
L
In the locked-rotor test the current in the shunt branch Rc 2 jXm is often assumed to be negligible, in
which case the locked-rotor impedance per phase is given by

ZLR ' R1  R2 L  j XL L. (42)


If PLR  j QLR is the complex power per phase at locked-rotor, we have
V12
ZLR ' (43)
PLR  j QLR
L L
which can be substituted in eqn. (3.42) to give R2 and XL directly. If the current in the shunt branch
L L
Rc 2 jXm is to be taken into account, the equations become more complicated, with

ZLR ' R1  Rc j Xm L (R2 L  j XL L) , (44)

which is to be equated to eqn. (3.43). Veinott solves this algebraically for the T circuit under the
assumption X1 ' X2. In PC-IMD the solution is obtained for any arbitrary ratio X1/X2, using an iterative
procedure (see p. 17).
A locked-rotor test at full voltage gives saturated values of the leakage reactances, which can be expected
to produce more accurate values for the starting current and torque. For normal saturation levels the
locked-rotor test is conducted at reduced voltage corresponding to rated current. It is sometimes
recommended to conduct the locked-rotor test at reduced frequency to minimize the deep-bar effect
(which is negligible in the no-load test because the slip, and therefore the rotor frequency, is small).
Induction machines Page 3.17

Correlation between measured and calculated impedances


L L L L
Measured values of the L-circuit impedances XL , Xm , R2 and Rc from the no-load and locked-rotor
tests can be used to deduce the impedances of an equivalent T-circuit. For Xm this is just

Xm L
Xm ' , (45)
"
while for R2 it is R2 L
R2 ' . (46)
"2
and for Rc
Rc ' Rc L . (47)

To obtain individual values of X1 and X2 we must choose a value P such that2


X1
' P. (48)
X2
We must now solve the simultaneous equations
X1 ' ("  1) Xm (49)
and L
(XL  " X1)
X2 ' (50)
"2
together with eqns. (3.45) and (3.48). Note that eqn. (3.49) comes from eqn. (3.27), which is the defining
equation for " for the L circuit, while eqn. (3.50) comes from eqn. (3.29). The solution of the four
simultaneous equations (3.45), (3.48), (3.49) and (3.50) for the four unknowns Xm, X1, X2 and " is arguably
more complex than Veinott’s procedure, but it is the price paid for avoiding the assumption X1 ' X2.3
The value of P can evidently be chosen to divide the leakage reactance between X1 and X2 in the same
ratio as the values obtained from design calculations such as those available from PC-IMD.
An alternative procedure for comparing calculated and measured impedances is to compute the
L L L L
impedances of the L-circuit XL , Xm , R2 and Rc from the “design” values X1, X2, Xm, R2 and Rc using
eqns.(3.29), (3.45), (3.46) and (3.47) [or (3.51); see below]. The comparison can then be made in terms of
L L L L
XL , Xm , R2 and Rc instead of X1, X2, Xm, R2 and Rc.
Finally it may be of interest to return the core-loss resistor Rc to its conventional position in parallel
with Xm in the T-circuit. If Rc is sufficiently large, this can be done with minimal error by adjusting its
L 2
value to Rc (E1/U1) , where E1 is the voltage across Xm. This ensures that the correct core-loss is
approximately maintained. The voltage ratio is very nearly equal to 1/", giving

Rc L
Rc ' . (51)
"2

2
In Veinott’s case, as we have seen, P ' 1. However, Veinott uses the T circuit directly, not the L circuit.
3
A recursive solution is necessary, as the equations are nonlinear.
Page 3.18 SPEED’s Electric Machines

3.4 WINDING FACTORS AND OTHER WINDING-RELATED MATTERS


For standard concentric and lap windings these are defined as in the classical literature.

Pitch factor
nB"
kpn ' sin (52)
2
where " is the per-unit coil pitch (i.e., the span in electrical radians divided by B), and n is the order of
the harmonic. Sometimes this is expressed in terms of the “chording angle”, g = 1 ! ", in which case kpn
= cos [gnB/2] for odd non-triplen harmonics.

Distribution factor
nq(
sin
2
kdn ' (53)
n(
q sin
2

where ( is the slot pitch in electrical radians and q is the number of slots per pole per phase. This is used
only with lap windings.

Skew factor
nF
sin
2
ksn ' (54)
nF
2

where F is the skew angle in electrical radians, and p = pole-pairs.

General method for computing harmonic winding factors for any winding
For fractional-slot windings the winding factor is obtained by Fourier analysis of the MMF distribution
of the winding. The harmonic coefficients an and bn of the MMF are calculated for each individual coil.
Then the an and bn are added together for all the coils, assuming that 1A flows in all the windings in
series. The resultant magnitude of the n'th harmonic MMF coefficient is cn = %(an2 + bn2). The winding
factor, is the ratio of cn and the n'th harmonic winding factor of a "base" winding with the same number
of series turns distributed equally in full-pitch coils among the 2p poles. The "base" winding is assumed
to start in slot 0, so that it has only sine coefficients Bn. Thus kwn = cn/Bn. Note that the phase
information in an and bn is lost in this process, so kwn is always positive, even with a winding for which
negative values of kwn are possible. For example, in a 24-slot 2-pole motor we could wind two coils each
with a span of 8 slots (i.e., 2/3 pitch) diametrically opposite to each other. (Coil 1 in slots 1!9; coil 2 in
slots 21!13). This is a concentric winding for which " = 2/3 and so k5 = sin (5 × 2/3 × B/2) = !0.866. The
above procedure gives a5/B5 = !0.836 and b5/B5 = !0.224, so that c5/B5 = +0.866. When the winding is
skewed, the total winding factor for the n'th harmonic is obtained by multiplying kwn by ksn.
The basic analysis is as follows.
Induction machines Page 3.19

Fig. 3.17 Distribution of MMF of a single coil around the airgap

In Fig. 17 a single coil of Nq turns is represented by its “go” coilside G at (q and its return coilside R at
Dq. The span is "q = Dq ! (q and the axis is located at Nq = ((q + Dq)/2 = (q + "q/2. With current i flowing
in the coil, the MMF distribution is as shown, with

1 "q
' / Fiq ' Nq i 1 £
00
Fq , (q ˜ 2 ˜ Dq
00
2 B
00
(55)
00 £ F 1 "q
00 oq
' £ N q
i
2 B
, elsewhere

The Fourier analysis is

4
Fq ' anq cos n 2 bnq sin n 2 , (56)
n'1
where

2 2B 2 2B
anq ' Fq cos n 2 d2 ; bnq ' Fq sin n 2 d2 . (57)
2B 0 2B 0

Thus

2 (q Dq 2B
anq ' £ Foq cos n 2 d2 Fiq cos n 2 d2 £ Foq cos n 2 d2
2B 0 (q Dq
(58)
2 n "q
' Nq i cos n Nq sin .
nB 2

Similarly

2 n "q
bnq ' Nq i sin n Nq sin . (59)
nB 2

For the whole phase winding with Nc coils we can write


Nc Nc

an ' anq ; bn ' bnq . (60)


q'1 q'1
Page 3.20 SPEED’s Electric Machines

Fig. 3.18 MMF of a 4-pole winding with 4 coils symmetrically disposed over 360E.

The harmonic coefficients anq and bnq of a single coil are quite general, and it is interesting to note that
the number of poles does not appear in eqns. (3.58) and (3.59). Indeed a single coil is unalterably a 2-pole
entity, as is obvious from Fig. 3.17: there is only one north and one south pole.4
It follows that a winding with more than two poles must have more than one coil. The number of poles
(or pole-pairs p) is a property of winding conferred by the spatial distribution of a multiplicity of coils.
For example Fig. 18 shows a 4-pole winding obtained with 4 coils. Every other coil has its polarity
reversed, so that the “go” and “return” conductors are conducting in the correct directions. As is well
known, the same MMF distribution could be produced with only two coils 1 and 3 (or 2 and 4): this would
constitute a consequent-pole winding. The important feature is the location of the ampere-conductors
with the correct amplitudes and locations, and from this point of view the only essential difference
between a regular winding and a consequent-pole winding is the routing of the end-connections.
The MMF spectrum of a single coil has an infinite number of harmonics (eqn. (3.56)). But the MMF
spectrum of a winding such as that of Fig. 3.18 evidently has no harmonics with fewer than 4 poles. The
lowest-order harmonic is the second harmonic, n = 2. The “fundamental” n = 1 is absent: this winding
does not produce a 2-pole field harmonic of order n = 1. Since the harmonic n = 1 exists in the spectrum
of every individual coil, its disappearance must be a result of cancellation. Mathematically this
cancellation must be occurring in eqns. (3.60).
If we omit coils 2 and 4 in Fig. 18, the n = 1 harmonic produced by coil 1 is centered at 2 = Nq = B/4, while
the n = 1 harmonic produced by coil 3 is centered at 2 = Nq + B. Since these are out of phase by B radians,
they cancel, provided of course that their amplitudes are equal. This is sufficient to establish the
elimination of the n = 1 harmonic in the MMF distribution of the winding of Fig. 18. Evidently the
cancellation relies on having perfect balance between the coils.
The association of the number of pole-pairs p with the “fundamental” space-harmonic of the MMF
distribution is deeply rooted in electric machine theory, and this association is cemented by the use of
electrical degrees and radians. It is thus easy to forget that harmonics of order n < p are eliminated only
by cancellation. In symmetric windings with integer slots/pole, the cancellation is generally exact and
definite, but in fractional-slot windings it is possible for these harmonics not to be cancelled. Relative
to the “fundamental” p, the harmonics with n < p are classed as “subharmonics”, although in
mechanical angular measure they are quite normal and can be calculated using eqns. (3.56)-(3.60)).
Mathematically their only distinguishing feature is that they are not cancelled in the summations in
eqns. (3.60).
We can now return to the definition of the harmonic winding factors as the ratio between the amplitude
2 2
of a particular MMF harmonic cn = /(a n +b n ) and the amplitude of the same MMF harmonic produced
by a full-pitch winding, with n = p.

4
Although the field strengths of the N and S poles are unequal, the flux/pole is the same. This is the basis of eqn. (3.55).
Induction machines Page 3.21

The full-pitch winding is defined as having p pole-pairs with 2p coils uniformly distributed with the axes
of adjacent coils displaced by B/p mechanical radians, and N/2p turns per coil, where N is equal to the
total number of turns in the actual winding: thus N = N1 + N2 + . . . + Nq + . . . + NNc. The actual span of
every coil is "q = B/p, and if (1 = 0 the “go” coilside of the first coil is in slot 0, so that N1 = "1/2 = B/2p.
Further, N2 = N1 + B/p = 3B/2p and so on, with Nq = (2q ! 1)B/2p. This winding will have no harmonics
of order n < p. Also, because of the symmetry between its positive and negative coils, it will have no
even harmonics, so n is always odd: n = 1,3,5,...
From eqn. (3.58) we can see that anq = 0, because cos nNq = cos n(2q ! 1)B/2p = 0, as n(2q ! 1) is odd. On
the other hand the sine coefficient is

2 nB nB
bnq ' Nq i sin (2 q £ 1) sin
nB 2p 2p
(61)
1 nB nB
' £ N i cos 2 q £ cos 2 (q £ 1) .
nB q 2p 2p

We are interested only in the electrical fundamental, n = p. When q is odd, bnq = +2, and when q is even
again bnq = +2 if the coil polarities alternate. Thus every coil in the full-pitch winding contributes
equally to the fundamental, so that its total fundamental MMF is

2N
Bn ' p ' i. (62)
pB

The n'th harmonic winding factor is then

( an2 bn2 )
kwn ' (63)
Bn'p
Page 3.22 SPEED’s Electric Machines

MMF distribution
The MMF distribution is obtained by adding the slot ampere-conductors in staircase fashion around the
entire stator periphery, and subtracting the mean value to produce a waveform with zero average value;
that is, no homopolar component. The normalized or per-unit MMF distribution is obtained by dividing
the ordinates by the peak value.
Classical winding factor

n )th harmonic of MMF wave for actual coil Fn


kwn ' ' (64)
n )th harmonic of a full&pitch coil of same total conductors Fn0
... applies to only one phase at a time.

MMF Harmonics
With a certain current in one phase, or a combination of currents in all three phases, let
Fn ' Amplitude of nNth harmonic component of MMF wave [A] . (65)
Fn can be determined by Fourier analysis of the actual MMF distribution. An important case which
arises in practice is the set of polyphase currents given by
i1 ' Ipk sin2
i2 ' Ipk sin( 2  120E) (66)
i3 ' Ipk sin( 2  120E)
for any specified value of 2. In a motor, 2 ' Tt. Per-unit base is with all 3 phases on and 2 ' 90E.
The MMF harmonic can be normalized to the fundamental component of the MMF wave: thus

nNth harmonic component of MMF wave Fn


fn ' ' [p.u.] (67)
Fundamental component of MMF wave F1
The per-unit or normalized MMF harmonics remain the same, whether the actual MMF wave is
normalized or not, since the units of Fn and F1 cancel in eqn. (3.67).
Let Fn0 be the nth harmonic component of MMF produced by a full-pitch coil having the same number of
turns, and F10 be the fundamental component produced by this coil. Then fn can be expressed as
Fn Fn Fn0 Fn Fn0 F10 kwn
fn ' ' @ ' @ @ ' , (68)
F1 Fn0 F1 Fn0 F10 F1 n kw1
since F1/F10 ' kw1 and Fn0/F10 ' 1/n. 5
Alternatively the per-unit MMF can be defined as

nNth harmonic of MMF wave Fn


un ' ' ; (69)
Fundamental component of MMF wave of a full&pitch coil F10
that is, Fn Fn Fn0 kwn
un ' ' @ ' ' kw1 fn . (70)
F10 Fn0 F10 n
Since kw1 is usually not much less than 1, un is slightly smaller than fn. The factor 1/n in both fn and un
explains why the space harmonics have much less effect than might be suggested by the classical
winding factors. As an example of this, the slot harmonics which repeat with winding factors equal to
kw1 have per-unit MMFs much less than kw1. Thus in a 36-slot 4-pole machine the first slot-harmonics
are those for which n ' 17 and 19, with u17 ' kw1/17 and u19 ' kw1/19.

5
fn is what is displayed in CGV Table 14-1.
Induction machines Page 3.23

Fourier analysis of MMF waveforms

Fig. 3.19 Fourier analysis of a waveform with a finite number of steps per cycle

The ideal MMF waveform of one phase or of all phases acting together is a stepped waveform which is
periodic in 360Emech. In regular machines with integral slots/pole it is also generally periodic in
360Eelec, i.e., 360/p Emech. A section of such a waveform is shown in Fig. 3.19. The steps are vertical
if it is assumed that the slot ampere-conductors ak are concentrated in filaments located at the centre-
lines of the slot-openings, which are assumed to be infinitely narrow. In this case the steps are equal
in magnitude to the ampere-conductors in each slot. The steps are combined into one waveform which
has an average value of zero : that is, there is no "DC" or homopolar component.
The steps coincide with the slot-openings; between them the waveform is flat. It can be represented as
th
a series of rectangular "pedestals" with vertical sides. Considering the k pedestal in isolation, it has
an amplitude yk and extends from 21 to 22, where (s = 22  21 is the slot-pitch angle. This is also the width
of every segment or pedestal.
th
The Fourier cosine coefficients of the k pedestal are given by Euler's formula

22
2
an ' yk cos n 2 d 2
2B 21

yk sin n2 22 yk
' ' sin n22  sin n 21
B n Bn (71)
21

2 yk n (21 22) n (22  21)


' cos sin
Bn 2 2
2 yk n (s
' cos n 2k sin .
Bn 2

A similar equation can be written for the sine coefficients bn. If we write (s = 2B/N where N is the
number of slots, and add the Fourier coefficients for all the N pedestals together, we get
N
sin (n (s / 2) 2
an ' yk cos n 2k ;
n (s / 2 N k'1
N
(72)
sin (n (s / 2) 2
bn ' yk sin n 2k .
n (s / 2 N k'1

The sums with coefficient 2/N could almost be written by inspection as a "discrete" form of Euler's
integral formulas for the Fourier coefficients. The additional sin (n(s/2)/(n(s/2) coefficient arises
because the original function is piecewise-constant between steps. Also note that a0 = 0.
Page 3.24 SPEED’s Electric Machines

Effect of slot-openings

Fig. 3.20 Pedestals modified with sloping sides due to slot-openings

The effect of the slot-openings is to spread the filament of slot ampere-conductors, forming a strip whose
angular extent is the slot opening-angle F, Fig. 3.20. The pedestals now assume a trapezoidal shape which
can be considered as the sum of an infinite number of rectangles in the same way that skew is
represented by an infinite series of slices; according to this model the Fourier coefficients are all
modified by the factor kFn which can be calculated as
1/2
an cos n (2 z F) dz
1/2 sin( n F/2 )
kF n ' ' . (73)
1/2 ( n F/2 )
an cos n 2 dz
1/2

Here z is a parameter which associates the series of rectangles, each having a "weight" dz. z extends over
a range of 1 from 0@5 to +0@5, so as not to introduce a phase shift. The integral or infinite sum of all the
rectangles is evaluated relative to the result with F = 0, so the amplitude of the rectangles cancels and
does not appear in the formula.
It is interesting to compare the sin (n(s/2)/(n(s/2) factor arising from the stepped nature of the MMF
waveform with the sin (nF/2)/(nF/2) factor arising from the slot-openings. In general both factors are
required. It is possible (but slightly more complicated) to arrive at the same result by direct Fourier
analysis of the series of trapezoidal "pedestals" in Fig. 3.20. The form sin(nx)/(nx) is of course the same
as that which appears in the skew factor and the distribution factor or spread factor, and it appears
again in the analysis of tapered magnets. It appears in Hague (Ch.2, Ref. [5]).

Bifilar winding
Sometimes the auxiliary (start) winding of a split-phase motor is wound with a percentage b of bifilar-
wound “back-turns”, as a way of increasing the resistance without changing the inductance. This
provides a means of increasing the R/X ratio for starting, without the need for an external resistor.
A normal nonbifilar coil has T turns. Its inductive effect is determined by T, and its inductance is
2
proportional to T .
“Back-turns” are in effect wound with a wire extension that is doubled back on itself, so that the total
strand length in one back-turn is twice the strand length in one normal (nonbifilar) turn.
Equivalently, one back-turn is first wound in the same direction as the nonbifilar turns and then wound
back in the opposite direction. Its length, weight, and resistance are all double those of a normal turn,
while its inductive effect (including its inductance) is zero.
A complete coil has T normal turns and a bifilar section of B back-turns. Its inductive effect is entirely
2
determined by T, so that its inductance is 8T , where 8 is a constant. Its strand length, weight and
resistance are all proportional to (T + 2B). In Fig. 3.21 the resistance is shown as R = D(T + 2B), where
D is a constant that depends on the strand cross-section and the resistivity.
Induction machines Page 3.25

Fig. 3.21 Bifilar winding

Each back-turn adds two strand cross-sections to the copper cross-section in the slot, so the slot-fill factor
depends on the sum of the individual (T + 2B) values for all coilsides in a slot. The back-turns have no
effect on the winding factors or the MMF harmonics. The number of turns in series per phase is
determined by T and is unaffected by B.

Specification
Let b be specified as the “percentage bifilar” parameter of a coil, such that the resistance is increased
by b% over the resistance of the normal turns T. Then

b T 2B
1 ' . (74)
100 T

This means we must choose the back-turns B to be the nearest integer to

b T
B ' × . (75)
100 2

If b is specified as a single parameter for an entire winding, the back-turns B can be calculated for each
coil using eqn. (3.75). On the other hand, if B is specified individually for each coil, the percentage bifilar
parameter for the entire winding can be defined in terms of the weighted sum

2 (B1 B2 . . . BN)
b ' × 100 % (76)
T1 T2 . . . TN
Example
Consider a concentric winding with 3 turns per pole with 90, 80, and 70 turns, and b = 50%. Then we will
have the following table from eqn. (3.74):

TURNS [BACK-TURNS]

T1 = 90 B1 = 23 90[23]

T2 = 80 B2 = 20 80[20]

T3 = 70 B2 = 18 70[18]

Note that the B values must be integers, while b is a real number.


On the other hand, if the coils are defined individually as in the table with their separate back-turns,
the overall ”weighted average” bifilar parameter from eqn. (3.76) is
2 (23 20 18)
b ' × 100 % ' 50@83 % . (77)
90 80 70
Page 3.26 SPEED’s Electric Machines

3.5 ROTOR AND STATOR SLOT NUMBERS


For a given stack length, rotor diameter and electromagnetic loading (air-gap flux density and stator
current density) the number of stator slots has little influence on the stator resistance or the copper
weight. The number of stator slots can then be used to control the value of the leakage reactance: it
controls the differential component directly, and the slot component indirectly, through the ratio of slot
depth to slot width. Similar considerations are valid for the number of rotor slots.
To minimise the harmonic effects (including the differential leakage reactance) a "golden" rule is to use
an integral number of stator slots per pole per phase (with a minimum value of 2), the choice being
made, if necessary, to allow the use of special types of windings (such as the 5/6 chorded lap winding).
The number of rotor slots must be correlated with the number of stator slots to reduce harmonic effects
such as parasitic synchronous and asynchronous torques, stray-load losses, vibrations and noise.
Alger [1970] states that the number of rotor slots should be 0.75!0.85 or 1.2!1.35 times the number of
stator slots in order to maximise the secondary zig-zag reactance for the stator slot MMF harmonics and
to minimize other harmonic loss components. According to Alger, synchronous crawling torques are
"present in 3-phase motors at ±Ns × Poles/Rotor slots, if the difference between stator and rotor slots is
1,2, or 4 per pole; while standstill locking will occur if the difference is 3 per pole". (Ns = synchronous
speed). The choice of stator and rotor slot numbers is discussed in Heller and Hamata [1977] and Kopilov
et al [1980].
Two tables from these references with recommended numbers for stator and rotor slots are provided
below. These values were derived from theory, mainly developed for line start machines, and practical
experience. It should be noted that on this subject there are divergences between various authors as can
be seen for example by inspecting the two tables. The tabulated slot combinations should not be
necessarily regarded as “extremely safe” and also it should be kept in mind that there are known cases
of successful motor designs that do not fulfill these recommended combinations.

Poles Stator Slots Rotor Slots (Bars)

24 (16), [20], ([22]), (28), [30]


2
30 (16), [20], (22), [26], [34], [36]

36 [24], 26, [28], 30, ([32]), 42, (44), [46]

48 (32), 34, [36], 38, [40], ([44]), (56), 58, [60]


4

36 24, [26], [46]


6
54 38, 40, [44], [64], 66, [68]

48 34, [62]
8
72 50, 52, 54, [56], 58, 86, 88, [90]
TABLE 3.2
SUITABLE COMBINATIONS FOR THE NUMBERS OF STATOR AND ROTOR SLOTS FOR SMALL AND MEDIUM-SIZE SQUIRREL-CAGE MACHINES
WITH 2-8 POLES AND AN OUTER DIAMETER UP TO 300MM

The number of rotor slots in round brackets are not suitable for reversible drives (because of large
synchronous parasitic torques in the braking region). The number of rotor slots in square brackets
maybe used only if the rotor slots are skewed by one slot pitch. It should be noted that according to [3]:
"no general rules exist for the choice of the number of slots which would be universally valid for small
as well as large machines".
Induction machines Page 3.27

Poles Stator Slots Rotor Slots (Bars)


Unskewed Skewed
1 1
2 12 9 , 15 —
1 1 1 1 1
18 11 , 12 , 15 , 21 , 22 14 , 18 , 19 , 22 , 26, 281, 302, 31, 33, 34,
1 2 1 1

35
24 151, 1612, 171, 19, 32 18, 20, 26, 31, 33, 34, 35
30 22, 38 182, 20, 21, 23, 24, 37, 39, 40
36 26, 28, 44, 46 25, 27, 29, 43, 45, 47
42 32, 33, 34, 50, 52 —
48 38, 40, 56, 58 37, 39, 41, 55, 57, 59
1
4 12 9 151
18 101, 141 181, 221
24 151, 161, 17, 322 16, 18, 202, 30, 33, 34, 35, 36
36 26, 44, 46 242, 27, 28, 30, 322, 34, 45, 48
42 342, 502, 52, 54 332, 34, 382, 512, 53
48 34, 38, 56, 58, 62, 64 362, 382, 392, 40, 442, 57, 59
60 50, 52, 68, 70, 74 48, 49, 51, 56, 64, 69, 71
72 62, 64, 80, 82, 86 61, 63, 68, 76, 81, 83
2
6 36 26, 46, 48 281, 33, 47, 49, 50
54 44, 64, 66, 68 42, 43, 51, 65, 67
72 56, 58, 62, 82, 84, 86, 88 57, 59, 60, 61, 83, 85, 87, 90
90 74, 76, 78, 80, 100, 102, 104 75, 77, 79, 101, 103, 105
2
8 48 34 , 36, 44, 62, 64 35, 44, 61, 63, 65
72 56, 58, 86, 88, 90 56, 57, 59, 85, 87, 89
2
84 66, 68 , 70, 98, 100, 102, 104 682, 692, 712, 972, 992, 1012
96 78, 82, 110, 112, 114 79, 80, 81, 83, 109, 111, 113
10 60 44, 46, 74, 76 57, 69, 77, 78, 79
90 68, 72, 74, 76, 104, 106, 108, 110, 112, 70, 71, 73, 87, 93, 107, 109
114
120 86, 88, 92, 94, 96, 98, 102, 104, 106, 134, 99, 101, 103, 117, 123, 137, 139
136, 138, 140, 142, 144, 146
12 72 56, 64, 80, 88 69, 75, 80, 89, 91, 92
90 68, 70, 74, 88, 98, 106, 108, 110 712, 732, 86, 87, 93, 94, 1072, 1092
108 86, 88, 92, 100, 116, 124, 128, 130, 132 84, 89, 91, 104, 105, 111, 112, 125, 127
144 124, 128, 136, 152, 160, 164, 166, 168, 125, 127, 141, 147, 161, 163
170, 172
14 84 74, 94, 102, 104, 106 75, 77, 79, 89, 91, 93, 103
126 106, 108, 116, 136, 144, 146, 148, 150, 107, 117, 119, 121, 131, 133, 135, 145
152, 154, 158
16 96 84, 86, 106, 108, 116, 118 90, 102
144 120, 122, 124, 132, 134, 154, 156, 164, 138, 150
166, 168, 170, 172
TABLE 3.3
RECOMMENDED COMBINATIONS OF STATOR AND ROTOR SLOT NUMBERS
1 2
used especially for fractional horse power machines. might cause increased motor vibrations.
Page 3.28 SPEED’s Electric Machines

3.6 CALCULATION OF THE RESISTANCES AND REACTANCES OF THE EQUIVALENT CIRCUIT


All the equivalent-circuit impedances must be computed from the motor geometry and the winding
distribution. Classic textbooks describe the basic methods, but it is important to understand the
variation of the impedances with load, speed, supply voltage and temperature. The main variations of
impedance values are as follows.

Stator winding resistance R1


R1 is a function of temperature, primarily because the resistivity of copper is a function of temperature:

D ' D20C [ 1  " (T  20) ] ' 1@724 × 108 × [ 1  0@00393 × (T  20) ] ohm m (78)

where T is in EC. A further correction might be made to account for the linear expansion, but this is
unusual. Then
DL
R1 ' (79)
a 2A
where L is the length of all strands in the winding laid end-to-end, A is the cross-sectional area of one
strand, and a is the number of parallel paths when the wire is finally connected in the phase winding.
The length of wire is generally impossible to calculate accurately from the geometric dimensions of the
motor, mainly because the end-windings do not follow a geometric shape is mathematically known. For
this reason the length L is often obtained empirically from the winding process, or from calculations on
the external coil-forms used to wind the coils.
2
The wire diameter d can vary over the length, and since A depends on d this can lead to small but
annoying uncertainties. For this reason the wire resistance is often specified (for a given wire gauge)
as ohms per km or ohms per 1000ft length, effectively grouping D/A into a single parameter.
If * is the mass density of the wire, the ratio of resistance to copper weight is given by
R 16 D
' . (80)
WCu *B2d 4
4
Since this depends on d , it is better to use the engineer’s data for “ohms per 1000ft” and “lb per 1000ft”
than the scientific formula (3.79), because it is not practical to measure d with sufficient accuracy over
the entire length of wire. Small variations from the nominal gauge are possible, and any error in d
makes it impossible to calculate the resistance and the copper weight correctly at the same time.
The AC value is generally higher than the DC value because of proximity effect. In larger motors the
conductors must be laid up in such a way as to minimize this effect. It is also possible for the apparent
value of R1 to increase as a result of stray-load loss “reflected” into the stator winding circuit, since there
is no other circuit element in Fig. 3.5 or Fig. 3.13 that can account for these additional losses.

Rotor resistance R2
Rotor winding resistance R2 is also a function of temperature. It is also a nonlinear function of slip,
because of the deep-bar effect or double-cage effect. Imperfections in the rotor, such as imperfect
insulation between the rotor cage and the iron core, or holes or inclusions in the rotor casting, can
increase the value of R2. Moreover, as with R1 in the stator circuit, certain stray loss components can
be reflected into the rotor circuit, increasing the value of R2. Since R2 is a referred impedance, its
apparent value depends on the effective turns ratio or coupling coefficient between the stator and the
rotor, and this can vary with the saturation level which depends on both voltage and current.
R2 is also subject to uncertainty in cage rotors because of the fact that the current-density is not
uniformly distributed in the end-rings.
Conductivity of rotor bars — 99.75% pure Al has conductivity 6061% of that of pure electrical grade
copper. After casting, the Al absorbs iron and other impurities that reduce the conductivity to 58!59%.
A figure as low as 50% is often used. For copper cast rotors the conductivity might be typically 80!85%.
Induction machines Page 3.29

Magnetizing reactance Xm

Fig. 3.22 Analysis of airgap flux distribution for calculating Xm

The magnetizing reactance Xm is associated with the fundamental component of airgap flux and the EMF
E1 generated by that flux. Fig. 3.22 shows the actual flux-distribution modulated by the slot-openings.
The “dips” caused by the slot-openings do not rotate, and therefore do not directly affect the generated
EMF. Collectively, however, they modulate the overall permeance of the magnetic circuit and this
produces a ripple in the generated EMF at the slot-passing frequency. The resulting EMF harmonics are
called “permeance harmonics”. For the purposes of the equivalent circuit, it is the fundamental
component of the airgap flux that is important.
Fig. 3.22 shows the flattening of the flux-distribution in the middle, which is due to saturation of the
stator and rotor teeth. The smooth flattened curve is the one calculated by a nonlinear magnetic-circuit
solver such as the one in PC-IMD, in which the slot-openings are not modelled in detail, but through the
use of Carter’s coefficient. Also shown is the fundamental component that would be obtained if there
were no saturation.
Xm is subject to saturation and tends to decrease as the voltage increases. It is common to measure the
effect of this saturation in terms of the no-load characteristic which plots stator current against terminal
voltage at no-load (usually with the voltage on the y-axis). At any voltage, Xm is the ratio of volts and
amps on this curve. Since Xm determines the magnetizing current it is one of the main influences on
the power factor, and it can be seen that if the voltage increases above normal, the magnetizing current
will increase rapidly.

Fig. 3.23 No-load saturation curve


Page 3.30 SPEED’s Electric Machines

A formula for Xm is
μ0 Ts DL 1 Xm0
Xm ' m (kw1 Tph)2 × ' (81)
B gN p 2 ksat ksat
where Ts ' 2Bf, m is the number of phases, kw1 is the fundamental winding factor, Tph is the number of
turns in series per phase, D is the stator bore diameter, L is the stack length, gN is the effective airgap
length, p is the number of pole-pairs, ksat is the saturation factor, and Xm0 is the unsaturated magnetizing
reactance, equal to the gradient V/OA in Fig. 3.23. In terms of Fig. 3.23,
OB
ksat ' > 1. (82)
OA
The no-load current is
E1 V1
INL '
(83)
Xm X1  Xm

where E1 is the generated EMF appearing in Fig. 3.5. The no-load current is an important parameter
because it is so easily measured in the no-load test, so the accuracy of eqn. (3.81) is an important
consideration. This depends critically on the values of ksat and the effective airgap gN. The accuracy
of the calculated value of ksat depends on the quality of the magnetic equivalent-circuit solver, while the
accuracy of gN is traditionally a matter for empirical judgement. The Carter factor for the effect of stator
slot-openings can be approximated as
s
5 
gN g
kCs ' ' (84)
g s s s
5    q
g g 8

where s is the stator slot-opening; g is the physical airgap; and 8 is the slot-pitch. The parameter q
expresses an adjustment introduced by C.G. Veinott. For open slots q should be 0, which makes kCs very
close to Carter’s original formulation. For semi-closed slots, Veinott uses the formula obtained with q '
1, which makes the effective magnetic airgap gN a few percent larger and tends to increase the no-load
current slightly. The rotor may also have open or semi-closed slots, in which case there is a Carter factor
kCr for the rotor. It is general practice to take the overall Carter factor as the product kCs × kCr, and the
validity of this has been established in learned papers by Oberretl and others.
In principle the no-load saturation curve can be checked by finite-element analysis, but finite-element
analysis cannot tell us how much of the departure from the unsaturated linear value Xm0 is dues to
saturation and how much is due to “error” in the Carter coefficient(s). The no-load current is also
sensitive to saturation in the stator and rotor yokes, and an accurate geometrical model is essential in
setting up the magnetic equivalent circuit computation.

Leakage reactances X1 and X2


The leakage reactances X1 and X2 are generally made up of components such as slot-leakage, end-turn
leakage, and “differential” leakage (which accounts for all airgap flux components other than the
fundamental space-harmonic component). Skew also affects the values of X1 and/or X2. There are
numerous different theories and accounts which purport to explain the segregation of these leakage
effects between stator and rotor, but all of them amount to the same thing—the attempt to represent the
imperfect coupling between the stator circuit and the rotor circuit. End-turn inductances are treated
in Chapter 2, so the following pages are mainly concerned with slot-leakage and differential leakage.
Both X1 and X2 are subject to variation due to magnetic saturation of the various flux-paths (including
the main flux path), and this can be severe under high-current, high-slip conditions (See page 38). Also
X2 which is generally decreased by the deep-bar effect.
Induction machines Page 3.31

Slot leakage inductance

The slot-leakage inductance of a single coil is given by an equation of the form

Lc_slot ' μ0 Lstk N 2 ( P1 P2 ) (85)

where N is the number of turns, Lstk is the stack length, and P1 and P2 are the permeance coefficients for
the two slots in which lie the two coilsides. The permeance coefficient for each slot permits the
inductance to be calculated as though all N conductors linked the same flux. In practice they do not:
conductors towards the bottom of the slot link more flux than those towards the top. Therefore in
calculating the permeance coefficient the distribution of flux within the slot must be taken into account.

Fig. 3.24 Slot sections

When the slot can be considered to be made up of sections, that are segments of circles or trapezoids, Fig.
3.24, analytical expressions for P can be derived if it is assumed that the flux crosses the slot in the x-
direction (that is, B = (B x,0,0). The contribution )P of any section depends on the variation B x(y)
through the depth of the slot, and on the MMF of all sections lying below that section.

Consider an isolated section as in Fig. 3.24 (b) or (c) with an elementary flux tube dN. The MMF driving
this flux element is JA(y), where J is the average current-density in the wound part of the slot: i.e., J =
NI/Aw, where Aw is the total wound area. Then dN = μ0JA(y)Lstkdy/x = [μ0LstkNI/Aw] A(y)dy/x. The flux
dN is linked by the fraction A(y)/Aw of the total turns N, so the contribution dR to the total flux-linkage
2 2
is equal to dR = dN.N A(y)/Aw = [μ0LstkN I/Aw ] A2(y)dy/x. We can write this as dP = (1/Aw2) A2(y)dy/x,
where dP is the contribution of the flux element to P.

The contribution )P of any whole section is obtained by integrating dP over the height h of the section.
If there is another current-carrying section of area U below the current section, then we must integrate
dP = (1/Aw2) [U + A(y)]2dy/x over h.

The feasibility of building up P in this way depends on the integrability of the expression for dP. Simple
slots can be treated algebraically, using a few sections, but slots with more complicated shapes may need
to be divided into a large number of layers, each of which is calculated with eqn. (3.88).

As a simple example, )P is calculated for a slot bottom that is a circular segment spanning an angle 2$,
Fig. 3.24(c). Since the section is at the bottom, U = 0 and dP = (1/Aw2) [A(y)]2dy/x. It is convenient to
integrate with respect to 2 rather than y, so we write y = r(1 – cos 2); dy = r sin 2 d2; and x = 2r sin 2. A(y)
is the sector area r2(22 – sin 22)/2, and Aw is also given by this formula with 2 = $. Making all these
substitutions and performing the integration with respect to 2 from 0 to $, we get

$ [ 4$2 / 3 1/2 2 cos 2$ ] ! sin 2$ ! sin 4$ / 4


)P ' . (86)
2 [ 2 $ ! sin 2 $ ]2
Page 3.32 SPEED’s Electric Machines

When $ = B/2, the slot-bottom is semicircular and )P = 0.1424. With $ = B we get the "classical" value for
the slot permeance coefficient of a round slot, 0.6231.

Now consider the trapezoidal section, Fig. 3.24 (b). The area A(y) is written in terms of x as k(x2–w02)
where k = h/2(w1 – w0) and x = w0 + (w1 – w0)y/h, so that when dP is integrated with respect to x from x
= w0 to x = w1, we get the following expression (with B = U – kw02):

2k w1 h k 2 2
)P ' B 2 ln ( w1 w0 ) { B ( w1 w0 ) } (87)
Aw 2 w0 2 4

When the trapezium has parallel sides w1 = w0 = w, so k 6 4 and if a = hw, )P simplifies to

1 a2 h
)P ' U(U a) . (88)
Aw 2 3 w

For a rectangular section at the bottom of the slot, U = 0; and if this is the only section there are no
conductors above it, so Aw = a and
1 h
)P ' (89)
3 w

which is the well known formula for a rectangular slot. Another special case arises at the bottom of a
slot if w0 = 0; then the section is triangular and
3
1 h w1
)P ' (90)
Aw2 16

Empty sections: For a section that is empty of conductor we must integrate dP = (1/Aw2) U 2dy/x over
h. For a trapezoidal section this gives
h w1
)P ' ln (91)
w1 ! w0 w0
and if w1 and w0 are nearly equal this becomes
2h
)P ' (92)
w0 w1

which is commonly quoted in textbooks. Veinott in his VICA-31 program for slot constants uses a
modified form
4h
)P ' ; w1 < w0 (93)
3 w1 w0

in which w1 is equal to the slot opening and w0 is the width at the bottom of the slot wedge. By giving
three times more weight to w1 than to w0, he increases the value of )P and makes an allowance for
fringing in a section of the slot where it is generally most significant. (See table 3.4).

Finally, the contribution of the slot opening region is given by eqn. (3.92) with w1 = w0 = w equal to the
slot opening, and h equal to the depth of the tooth-tip: i.e., )P = h/w if there is no conductor in the slot
opening. If there is conductor in the slot-opening, eqn. (3.88) is used; it gives a slightly lower result.

Closed slots: For slots closed at the top there is no formula for )P that gives a finite result, because this
theory assumes infinitely permeable iron. Closed slots, and even slots with significant saturation of the
tooth tips, require a different treatment and their effective permeance depends on the slot current. See
page 38.
Induction machines Page 3.33

Comparison with finite-element calculations

The analysis assumes that the flux crosses the slot in the x-direction with no fringing. In practice
fringing increases the permeance, and finite-element studies of all the standard example slots in PC-IMD
indicate that the analytical P is typically 10% low. Fig. 3.25 shows a typical flux-plot from this study,
and the table summarizes the results. The permeance coefficient is calculated from the expression
2
E/μ0I , where E is the energy in J/m of axial length.

Bar type FE PC-IMD


1 1.98 1.8
2 2.01 1.88
3 2.36 2.2
4 2.95 2.73
5 2.41 2.3
6 2.72 2.57
7 4.44 3.63
8 3.78 3.41
9 2.03 1.85
10 2.57 2.3
Open 2.42 2.16
custom
Rectangular 2.166 2.167
slot
Fig. 3.25 Typical finite-element flux plots.
The permeance coefficients are a few percent
Table 3.4 higher than those calculated on the assumption
that the flux crosses the slot in parallel tubes.

Comparison with VICA-31


(Type 13 ER=0.5, DR=1.0, CR=0.455, BR=4.0, A1R=3.0)

VICA-31 FE PC-IMD PC-IMD Classical formula


Slot opening region empty Slot opening full Kostenko & Piotrovsky

2.85 2.87 2.71 2.58 2.2


TABLE 3.5

Stator slots
Rectangular Round-bottomed
FE PC-IMD FE PC-IMD
2.03 2.03 1.97 2.02
TABLE 3.6

Deep-bar effect
For the deep-bar effect (skin effect in rotor conductors), PC-IMD has two alternative methods: one is the
classical method for a rectangular slot, and the other is an integration of the complex diffusion equation
throughout the slot, using a layered model developed for SPEED by Prof. I. Boldea. This is similar to
the analysis above, except that the integration of contributions from the layers in the slots is complex,
to account for the change in phase of the current density throughout the conductor.
Page 3.34 SPEED’s Electric Machines

Saturation of leakage reactance

Fig. 3.26 Locked-rotor flux-plot

Saturation of the leakage reactance is an important characteristic of induction motors. Referring to the
equivalent circuit, when the slip is high the current in the leakage reactances X1 and X2 is high, causing
them to saturate. As a result, the current is increased still further. The most important concern is at
zero and low speed, because the inrush current during starting is approximately equal to Vs/(X1 + X2),6
and this must be limited to protect the electrical supply (as well as the motor itself).
The concept of leakage saturation appears simple enough when defined in terms of current-dependent
reduction factors to be applied to X1 and X2. However, the actual physical phenomenon is very complex,
as the finite-element flux-plot in Fig. 3.26 shows. The challenge is to capture the essential behaviour in
terms of the simple electrical circuit. Starting currents of 6 p.u. are quite normal, so the range of current
levels is clearly very wide. Leakage saturation also influences breakdown current and torque, as well
as normal operation; consequently, the required reduction factors are needed as continuous functions
of current, and not simply as single values at locked-rotor.
H. M. Norman's 1934 paper uses a hand-drawn flux-plot remarkably similar to Fig. 3.26 to focus attention
on the zig-zag reactance, followed by the stator and rotor slot-leakage reactances. He considered a sector
in which the zig-zag flux crosses the airgap twice, and wrote:
“The problem resolves itself into finding the proportion of the ampere turns required to drive the
flux across the airgap twice, to the total ampere turns; for this is the same ratio as the zigzag
reactance with saturation to that without saturation.”
He used this to define percent zigzag, which we will express as per-unit zig-zag .. He then produced an
empirical curve relating . to a fictitious flux-density BL that would be obtained in the airgap if the iron
were infinitely permeable. This curve is reproduced in Fig. 3.27. BL can be calculated knowing the
airgap length and the ampere-turns enclosed within the loop; then Fig. 3.27 immediately gives the per-
unit zig-zag ., which is the saturation factor for the zig-zag reactance at the pertinent current.
Norman assumed that the effect of saturation was confined to the iron in the tooth-head region in the
immediate vicinity of the airgap; this explains why only part of the closed flux-loop is considered. He
also went to some length to explain how to calculate the magnetomotive force, using the “average MMF
per slot of the primary and secondary over a phase group”. This is perhaps clarified in Fig. 3.26 which
shows the “solenoidal” nature of the magnetic circuit, with the stator and rotor ampere-conductors more
or less in phase opposition, a condition that is characteristic of locked-rotor. The peripheral extent of
Norman's model was the average of the stator and rotor slot-pitches.

6
This expression refers to the symmetrical AC current; there is also a DC offset which is discussed later.
Induction machines Page 3.35

Fig. 3.27 Per-unit zig-zag . (after H.M. Norman)

2
Norman's original graph used axes of percent and lines/in , but Fig. 3.27 uses per-unit and T, and an
acceptable approximation is

7@75
. '  0@31 (94)
BL 4@65

where BL is in T and is calculated by

μ0 × AT
BL ' [T] (95)
2g$
where g is the airgap length in metres, and AT is the average ampere-conductors per slot-pitch of the
stator and rotor. $ is an empirical correction factor for different airgap geometries, given by

$ ' 2@5 " 0@64 (96)


with
single mechanical airgap
" ' . (97)
rotor slot&pitch stator slot&pitch

For example, suppose AT = 2500 and the airgap is g = 0@5 mm or 0@0005 m. The stator slot-pitch is 11@3 mm
and the rotor slot-pitch 12@7 mm, so that " = 0@020905 and $ = 1@00147. Then

μ0 × 2500
BL ' ' 3@14 [T] . (98)
2 × 0@0005 × 1@00147
From Fig. 3.27 or the approximating formula (94), . = 0@685, and this is the saturation factor for the zig-
zag leakage reactance.
For the slot leakage (which he called “tooth-tip leakage”) Norman postulated that
“The tooth-tip leakage is reduced when the zig-zag leakage is sufficiently heavy to cause saturation
of the tooth-tips. The effect is as though the tooth tips were partially removed, or in other words as
though the slot openings were enlarged. By working with a large number of motors...[it was found
that] the tooth face was approximately reduced in the same proportion as the reduction in zigzag
leakage. Using this larger slot opening, the slot [permeance] constants are then recalculated and
the new slot leakage [reactance] found from these smaller constants. [Author's italics]”

He used this principle to define the “apparent change in slot opening due to saturation”, C,

C ' (8  t)(1  .), (99)


Page 3.36 SPEED’s Electric Machines

in which 8 is the slot-pitch (stator or rotor), t is the physical slot-


opening, and ., as before, is the per-unit zig-zag. He then produced
equations for the change in the slot permeance coefficient that would
result from the widening of the slot-opening. For example, for case N6
in Fig. 3.28,

a1 C
*k ' × . (100)
t C t

This equation is not difficult to derive, but for cases N7, N8 and N9 in
Fig. 3.28, Norman quoted formulas without derivation, and since we
have followed his wild but inspired empiricism thus far, we might as
well quote them:

a1 0@58 a2 C
N7 : *k ' × . (101)
t C 1@5 t

a1 C 3@3 a2 C
N8 : *k ' × × . (102)
0@02 C 0@02 t1 C 0@4 t1

a1 C 2 a2 C  0@15 t1
N9 : *k ' × × . (103) Fig. 3.28 Norman's teeth
0@02 C 0@02 t1 C 0@6 t1

The 0@02 used in the last two formulas appears to be a fictitious “effective unsaturated slot-opening” that
Norman used with closed slots. It should be replaced by 0@508 if the other dimensions are in mm.
The derivation of these formulas is probably lost forever, and even Norman disarmingly made the
following comment in the section of his paper entitled COMPARISON OF CALCULATED AND TESTED VALUES:
“If ever there was a mathematical analysis that needed proving it is surely this one.”
Norman's method is a good example of the dichotomy between practical estimation and scientific
sophistication, or, to put it more bluntly, between formula-plugging and scientific rigour. Nowadays
it would be tacitly assumed that the scientific finite-element method should be used for such difficult
calculations. But Norman wrote this in 1934, and he has given three whole generations of machine
designers a method that takes milliseconds to calculate (on a computer), and which illuminates the
subject brilliantly in terms of a physical explanation of leakage saturation. If it gives acceptable results
for a class or range of similar induction motors, it is tempting to use it. The acceptability of the results
can of course be verified only by testing (mainly through the locked-rotor test at different voltages), as
Norman himself asserted. The same is true of finite-element calculations, though they involve far fewer
simplifying assumptions and require no inspiration, approximation, or divination. Some comfort can
be taken from the fact that all the great electric machines design texts contain many examples of
empiricism, among which Norman's method seems very much at home.
Norman's vision of leakage saturation is so instructive that it is worth asking whether some of the
arbitrary “black art” could be substituted with more scientific analysis. For example the formulas
(particularly N8 and N9) for the change *k in slot permeance coefficient could probably be improved and
made more rigorous (or at least more verifiable) by methods such as the one in the following section.
Even finite-element analysis could conceivably be used to develop coefficients to fit the Norman model.
The alternative way to solve the leakage saturation problem would be to simulate the whole motor
during a period of rotation with a voltage-driven time-stepping finite-element procedure — an
exceptionally slow process that is generally held to be too slow for design work. Even so, if test data is
not available, this method is likely to have some appeal.
Induction machines Page 3.37

Representation of rotor bridge flux by a fixed EMF

Fig. 3.29 Representation of rotor bridge flux by Erb

The calculation of the rotor leakage reactance X2 for closed rotor slots has long been problematic because
of the nonlinear behaviour of the slot bridge. Veinott uses a set of curves for the slot permeance
coefficient, referring to earlier work by Trickey and Richter. Under normal load conditions the main
flux passes through the rotor, and the zig-zag and slot-leakage fluxes are much smaller than they are at
locked-rotor. The rotor slot bridges require very little MMF to saturate them, and it can be assumed that
they are saturated to a density of, say, 2@1 T. If this is multiplied by the bridge area (a1Lstk in Fig. 3.28,
with stack-length Lstk), we have a fixed bridge leakage flux Mrb which can be taken as the peak value of
a sinusoidal distribution of flux that generates voltage or EMF in the equivalent circuit in just the same
way as all the other fluxes. This EMF is called Erb and it always leads the rotor current I2 by 90E.
There remains the slot-leakage flux passing across the slot underneath the rotor bridge. The voltage
generated by this flux is j X I2. Thus the total voltage drop due to rotor leakage flux is
j X2 I2 ' E rb j X I2 . (104)

The apparent rotor leakage reactance is X2 and this is deduced as

Erb
X2 ' X, (105)
I2
where the phasor (boldface) notation has been dropped because all the voltages and currents are in phase
with one another. Clearly X2 is a function of current, becoming infinite when I2 = 0 and decreasing as
I2 increases. The infinite value is not significant, because the voltage X2I2 remains finite. Fig. 3.30
shows an example with Erb = 20 V and X = 0@5 ohm. The graph is drawn with a fixed value of X, but in
practice X2 saturates, as shown by the dotted lines. When Norman's method is used to estimate the effect
of saturation, it produces a slot-permeance correction *k that is most naturally applied to X rather than
to Erb, even though Erb is supposed to represent the saturated bridge region. Norman's method is
unclear about the unsaturated slot-permeance coefficient and it would seem worthwhile to improve it
to include Erb, especially if finite-element analysis is used to guide and check the analysis.

Fig. 3.30 Example of the effect of Erb


Page 3.38 SPEED’s Electric Machines

Saturation of rotor slot-bridges — further analysis

Fig. 3.31 Analysis of saturation of slot leakage inductance in closed slot

Fig. 3.31 shows a simple method for analyzing the saturation of the slot-bridges of closed rotor slots. The
slot-leakage flux has a component M that flows across the bridge. As it funnels into the tapered section
the flux-density increases to a maximum at the slot centre-line. For a given value of M, the MMF drop
across the whole bridge can be computed by integrating the contributions H dx, taking into account the
varying flux-density as a function of x, the distance from the centre-line along the “median” of the
tapered section. For each element dx, H is determined from the BH curve of the lamination steel. There
is also a parallel leakage flux inside the slot, every line of which shares the same MMF drop across its
length, as the corresponding lines inside the steel.
The result of the integration is a pair of values M, F whose quotient is, in effect, the permeance
coefficient for the saturated bridge. This permeance coefficient is a function of the slot current, and its
form is shown in Fig. 3.32. The nonlinear permeance coefficient is incorporated into the overall
magnetic equivalent circuit solution. Note that the variable-permeance model described here is used
instead of the simpler Erb model on page 10.

Fig. 3.32 Saturated slot — bridge permeance coefficient


Induction machines Page 3.39

Differential Leakage
Differential leakage is caused by space-harmonics in the airgap flux distribution, which increase the
self-inductances of the stator and rotor circuits relative to the mutual inductance Xm, which is
associated with the fundamental airgap flux alone. Differential leakage is also called “harmonic
leakage”. The space-harmonic flux waves arise from harmonics in the ampere-conductor distributions
of both the stator and the rotor.
In the following, reactances are normalized to Xm and shown in lower-case type.
The analysis of differential leakage appears to be divided into two schools. Typical of the first school
are Alger [3] and Veinott [5] who both divide the differential leakage into two components:
zigzag leakage and belt leakage, i.e.,
xdiff ' xzz  xbelt . (106)

The zigzag component is associated with slot-harmonics, that is, with the modulation of the airgap flux
due to the slotting on both sides of the airgap. The harmonic orders are 2ks ± 1, where s is the number
of slots/pole and k is any integer. For example in a 3-phase motor with 36 stator slots and 4 poles, s ' 9
and the slot-harmonic orders are 17,19, 35,37,... Alger states that for good motor performance the
secondary induced voltages due to the slot-harmonics should be nearly open-circuited; in other words,
they do not induce any significant currents in the rotor. Further, the slot-harmonics are independent
of the winding pitch but vary with the numbers of slots.
The belt component is associated with “phasebelt” harmonics, of orders 2kq ± 1 where q is the number
of slots/pole/phase, or q ' s/m. In the example 3-phase motor with 36 stator slots and 4 poles, q ' 3 and
the phasebelt harmonic orders are 5,7, 11,13, ... Alger states that the voltages induced in the secondary
by these harmonics are substantially short-circuited. They are nearly independent of the number of
slots, but vary with the pitch.
Alger declares xbelt to be negligible in squirrel-cage motors, in common with many other authors. The
reason given is that the phase-belt harmonics induce currents in the cage which suppress the phase-belt
flux harmonics. For wound-rotor machines, Alger includes a full derivation of xbelt. In contrast, Veinott
quotes a formula for xbelt without proof, stating that it is partly empirical.
Alger’s analysis begins with a rigorous mathematical derivation of closed formulas for the differential
reactance of 90E and 60E phase-belt windings with nearly closed slots. The formulas are complex and
take no account of the slot-openings, so he develops a simpler theory for xzz based on an ideal machine
with q ' 1 on both sides of the airgap, including an attempt to account for the effect of fringing around
the slot-openings. Two extreme cases are (1) with closed slots (infinitesimally small slot-openings) and
(2) open slots with no fringing. Both cases are covered by the formula
2 2
B2 a1 a2
xzz '  . (107)
12 2
s1
2
s2

For infinitesimally small slot-openings a1 and/or a2 are set equal to zero. For finite slot-openings with
no fringing, a1 and a2 are set equal to the ratio t/8, where t is the width of the tooth at the airgap, and 8
is the slot-pitch, respectively on each side of the airgap.
In the general case, Alger attempts to account for the fringing around the slot-openings by reference to
the theory of Carter’s coefficient, and ends up with a suggested approximate formula

B2 (6 a1  1) Fsc (6 a2  1) F2
xzz '   (108)
12 5s 2 5 s22 2
s1
1

where Fsc is a “damping term”. This equation also includes the effect of skew, where F is the rotor skew
measured in stator slot-pitches. Finally Alger gives an alternative derivation of xzz by the so-called
“overlap method”; the result is the same as the no-fringing version of eqn. (3.107).
Page 3.40 SPEED’s Electric Machines

Like Alger, Veinott [5] also uses an “overlap” method to compute Xzz, but his formula leads to an
2
expression for the normalized value xzz ' Xzz /Xm that is proportional to p /D, where p is the number of
pole-pairs and D is the stator bore. This appears to be quite at variance with the other formulations.
The second school is a more formal mathematical one, in which the differential leakage is computed
from a Fourier analysis of the ampere-conductor distribution. Typical of this school is Richter’s
formulation

1 kw < 2
xdiff ' (109)
kw12 < >1 <2

where kw< is the winding factor for the <’th harmonic. This formulation indiscriminately includes all
harmonics including the zigzag and belt harmonics. By itself it makes no attempt to allow for the effect
of slot-openings or the fringing around them. Heller and Hamata [4] refer (without any equations) to the
work of Jordan in modifying it to allow for slot-openings (but not for the effect of fringing). In PC-IMD,
SPEED’s formulation of xdiff is a generalization of Richter’s method which allows for fractional-slotting,
sub-harmonics, and forward- and backward-rotating components; it also allows for the slot openings (but
not for the fringing around them).
Heller and Hamata [4] discuss the “harmonic attenuation” of the differential leakage by induced
currents in the rotor, and present a graph of harmonic attenuation factors according to Richter,
determined as functions of q and the number of rotor bars per pole-pair. In effect, this analysis is a
generalization of Alger’s assertion that xbelt ' 0 for squirrel-cage rotors.
Richter and SPEED both include skew in the formulation of Xdiff, and Alger includes it in Xzz. Therefore
they do not display a separate skew-leakage reactance Xskew. Veinott, however, does compute and
display a separate skew-leakage term Xskew, while skew is not include in Xzz or Xbelt.
Alger, Richter, SPEED all compute Xdiff as a fraction of Xm via the differential leakage coefficient; but
CGV computes it independently of Xm. Therefore if skew decreases Xm, Alger, Richter and SPEED all
see an additional increase in Xdiff, whereas Veinott does not.
Induction machines Page 3.41

3.7 LOCATION OF AMPERE-CONDUCTORS


It is sometimes useful to be able to visualize how the ampere-
conductors in the rotor and stator are distributed, and how they are
oriented with respect to each other, especially in finite-element
calculations.
Fig. 3.33 shows the basic model of a sine-distributed amp-conductor
distribution. When a 3-phase winding is fed with balanced 3-phase
currents, the fundamental ampere-conductor distribution rotates at
synchronous speed. The axis of the airgap MMF lags 90E behind the axis
of the amp-conductor distribution.7
The current Im in the stator sets up a rotating ampere-conductor
Fig. 3.33 Sine-distributed ampere-
distribution ACm whose MMF axis is along the x-axis at the instant conductor distribution
shown in Fig. 3.33. This distribution rotates in the positive direction
(i.e., anticlockwise). The subscript ‘m’ means ‘magnetizing’.
A rotor conductor experiences a magnetic flux-density B (produced by Im) and rotates clockwise relative
to B at slip speed, so its velocity in the magnetic field is the vector v. The EMF v × B in this conductor
is in the positive (dot) direction. It causes current to flow in the same direction. Because of the sine-
distribution of B, the resulting induced current in the rotor also follows a sine-distribution, and it is
labelled AC2. AC2 lags behind the rotor EMF distribution by a small angle because of the rotor leakage
reactance in series with the rotor resistance. Note that the force F = i × B on the rotor conductor is in
the right direction to produce positive torque (counterclockwise).
The principle of ampere-turn balance (i.e., Ampère's law) requires that an equal and opposite amp-
conductor distribution appear in the stator windings, to cancel the MMF produced by the rotor MMF I2.
This "referred" MMF is I2N = !I2, and the corresponding ampere-conductor distribution is AC2N.8
The total stator current is the sum of the originally postulated current Im plus the referred rotor current
I2N : thus I1 = Im + I2N. The resulting ampere-conductor distribution in the stator is AC1 = ACm + AC2N
as shown in Fig. 3.34.
Only two of the ampere-conductor distributions
physically exist : AC1 and AC2. The other two,
ACm and AC2N, are abstract components of the
stator current distribution which are used to
explain the "mechanism" of induction. The
ACm or Im component is the "magnetizing"
component since it alone is responsible for the
mutual flux.
Fig. 3.34 shows that the rotor ampere-
conductor distribution is largely in opposition
to the stator ampere-conductor distribution, as
though the rotor tries to cancel the flux
produced by the stator. This must be so if the
magnetizing current is to be small compared
with the load current. Since the positive
ampere-conductors of AC1 and the negative
ampere-conductors of AC2 are nearly opposite
each other across the airgap, they tend to force
flux in the peripheral direction around the
airgap, i.e. into the leakage paths.

Fig. 3.34 Distribution of ampere-conductors.

7
If the amp-conductor distribution is C cos 2 then the MMF distribution is the space-integral !C sin 2. The axis of the MMF
distribution is also the axis of the corresponding space vector.
8
PC-IMD's phasor diagram shows only the current I2N (labelled as I2).
Page 3.42 SPEED’s Electric Machines

3.8 TRANSIENTS AND EIGENVALUE ANALYSIS


PC-IMD can calculate starting transients. To model the capability of soft-starters with point-on-wave
switching, the AC supply voltage can be switched in two stages, so that the DC offset can be suppressed.
This practically eliminates the oscillatory torque which occurs when the three lines close
simultaneously, and considerably lightens the duty on the shaft coupling, [Wood, 1965].
PC-IMD can also calculate the eigenvalues of the motor operating from a sinusoidal voltage source at
zero slip over a range of frequencies from the normal line frequency down to zero. The resulting root-
locus diagram is indicative of the relative stability of the motor when fed from a variable-frequency
inverter without shaft position or velocity feedback.
The theory of these dynamic calculations is presented here.

Reference Frame Transformations


PC-IMD’s dynamic calculations use two sets of d,q axes:
1. synchronously rotating (Kron), for eigenvalue analysis; and
2. d,q axes fixed on the rotor (Park) for transient analysis (starting calculations).
The conventions are the same as in Fitzgerald and Kingsley [1961], with only one or two minor changes
of notation: see Fig. 3.35.9 Although this approach is rather old-fashioned it is practical and clear. PC-
IMD does not include zero-sequence voltages or currents, so the theory works only for three-wire
connections and the transformation matrices are not square.

Fig. 3.35 Orientation of axes

The transformation from (a,b,c) variables (i.e., direct phase variables) to dq axes is eqn. (3.110), where
2 is the angle between the d-axis and the axis of phase a in elec. rad. The same transformation (and its
inverse, eqn. (3.111)) is used for voltages, currents, and flux-linkages.
2
vd ' [ va cos 2 vb cos (2 ! 2 B/3) vc cos (2 2 B/3) ] ;
3
(110)
2
vq ' ! [ va sin 2 vb sin (2 ! 2 B/3) vc sin (2 2 B/3) ] .
3

9
The main ones are: 2r instead of 22 (rotor angle); subscripts d,q instead of 1d,1q; and subscripts D,Q instead of 2d,2q. The phases
are labelled a,b,c here, but in the SPEED software they are generally labelled 1,2,3 (lines being labelled a,b,c).
Induction machines Page 3.43

Inverse: va ' vd cos 2 ! vq sin 2 ;


vb ' vd cos (2 ! 2 B/3) ! vq sin (2 ! 2 B/3) ; (111)
vc ' vd cos (2 2 B/3) ! vq sin (2 2 B/3) .

At this stage it is not specified whether the dq axes are fixed to the rotor or to the stator or to the flux,
or what. All that is necessary to specialize the transformation to one of the classic frames of reference
is to constrain 2 appropriately.
The total electrical instantaneous power at the terminals is
3
va ia vb ib vc ic ' [ vd id vq iq ] (112)
2

Flux-linkages and inductances


There are three phases a, b, c on the stator and three on the rotor, A,B,C. For the stator,
Ra ' Laa ia Lab ib Lac ic LaA iA LaB iB LaC iC
Rb ' Lba ia Lbb ib Lbc ic LbA iA LbB iB LbC iC (113)
Rc ' Lca ic Lcb ib Lcc ic LcA iA LcB iB LcC iC

The self-inductances are constant and symmetrical: Laa = Lbb = Lcc; LAA = LBB = LCC. The stator-stator
mutual inductances are constant: Lab = Lbc = Lca = Lba = Lcb = Lac. The rotor-rotor mutual inductances
are constant: LAB = LBC = LCA = LBA = LCB = LAC.

The stator-rotor mutual inductances vary sinusoidally with rotor position 2r: for example,

LaA ( 2 ) ' LaA cos 2r


LaB ( 2 ) ' LaA cos ( 2r 2 B/3 ) (114)
LaC ( 2 ) ' LaA cos ( 2r ! 2 B/3 )

A similar set of equations applies for LbA(2), LbB(2), LbC(2),LcA(2), LcB(2), and LcC(2), with appropriate
angles taken from Fig. 3.35. Also an equation similar to eqn. (3.113) is written for the rotor flux-linkages
RA,RB, and RC.

The three stator phases will be replaced by two fictitious, orthogonal coils d and q fixed to (and rotating
with) the d- and q- axes respectively. The flux-linkage of the d-coil is obtained by applying the voltage
transformation to Ra, Rb and Rc :

2
Rd ' [ Ra cos 2 Rb cos ( 2 ! 2 B / 3 ) Rc cos ( 2 2 B / 3 ) ] ' L11 id L12 iD (115)
3

where L11 = Laa ! Lab and L12 = 3/2 LaA and much algebra has been omitted. Similarly in the q-axis,
Rq ' L11 iq L12 iQ . (116)
For the rotor D and Q coils, the transformation is the same but with 2r instead of 2; thus

RD ' L22 iD L12 id ;


(117)
RQ ' L22 iQ L12 iq .

where L22 = LAA ! LAB, and again L12 = 3/2 LaA.


Page 3.44 SPEED’s Electric Machines

Physical attributes of the fictitious d and q coils


Consider the instant when the d-axis is aligned with the axis
of phase a, and ia = I, ib = ic =!½I in the 3-phase winding,
Fig. 3.36. According to the reference frame transformation
id = I. If the flux-linkage in phase a is Q, the flux-linkages of
phases b and c must be !½Q each, so that by the reference eframe
transformation Rd = 2/3 × Q × [1 !½(!½)!½(!½)] = Q. The
flux-linkage per ampere in the d-coil, is Rd/id = Q/I. The d-
coil evidently has the same number of turns as the a-coil. If
we consider I to be a DC current then the voltage drop in
Fig. 3.36 d-axis aligned with phase a
phase a is va = RaI and the voltage-drops in phases b and c
are vb = vc = !RaI/2. According to the reference frame transformation, vd = 2/3 × RaI × [1
!½(!½)!½(!½)] = RaI. The resistance of the d-coil is evidently vd/id = Ra. This is consistent with
having the same number of turns, provided that the total cross-section of copper in the conductors of the
d-coil is the same as that of the a-coil.
Consider the stator flux-linkage produced by the stator currents alone at the position shown in Fig. 3.36,
2r = 0. Ra is made up of LaaI due to self-flux-linkage, plus (!½I) × Lab mutually coupled from each of
phases b and c, for a total of (Laa ! Lab)I = L11I. The flux-linkage of phase b is Laa × (!½I) due to self
flux-linkage, with Lab × (!½I) mutually coupled from phase c, and LabI mutually coupled from phase a,
for a total of !½L11I. The flux-linkage of phase c is the same. So Rd = 2/3 × L11I ×
[1+(!½)×(!½)+(!½)×(!½)] = L11I. According to the transformation equation the d-coil current is I, so
the apparent inductance is L11. The d-coil represents all three stator phase coils along the d-axis,
including their mutual coupling.
Now consider the stator flux-linkage produced by the rotor currents alone, with current I flowing in coil
A and !½I in coils B and C. The flux-linkage of stator coil a is Ra = LaAI from coil A and !½LaA × (!½I)
from each of coils B and C, for a total of LaAI × [1 + (!½) × (!½) + (!½) × (!½)] = 3/2 × LaA I. The flux-
linkage of stator coil b is Rb comprising !½LaA × I from coil A, LaA × (!½I) from coil B, and !½LaA ×
(!½I) from coil C for a total of !¾LaA I. The flux-linkage of coil c is the same, so that according to the
reference frame transformation Rd = 2/3 × [3/2 + (!½) × (!¾) + (!½) × (!¾)] × LaAI = 3/2 × LaAI.
According to the transformation the current iD is I × 2/3 × [1 + (!½) × (!½)+(!½) × (!½)] = I, so that
the apparent mutual inductance is 3/2 × LaA.

Voltage equations
For the stator
va ' Ra ia p Ra ; vb ' Ra ib p Rb ; vc ' Ra ic p Rc (118)
where p is the operator d/dt, and we have assumed that Ra = Rb = Rc. For the stator d- and q-coils the
voltage equations are derived by incorporating eqns. (3.118) into the reference-frame transformation
equations for vd and vq. When p operates on the trigonometric functions of 2, we get the speed voltages.
Missing out a lot of algebra,
vd ' Rd id p Rd ! p 2 . Rq ;
(119)
vq ' Rq iq p Rq p 2 . Rd .

In synchronously rotating axes p2 = T = 2Bf. In d,q axes fixed to the rotor, p2 = Tr, i.e., the rotor angular
velocity in elec rad/s. Note that Rd = Rq = Ra.

A similar procedure applies to the D- and Q-coils on the rotor, except that they are themselves rotating
at angular velocity Tr, the physical angular velocity of the rotor in elec rad/s. The result is
Induction machines Page 3.45

vD ' 0 ' RD iD p RD ! p 2s . RQ
(120)
vQ ' 0 ' RQ iQ p RQ p 2s . RD .

where p2s is the slip velocity in elec rad/s, i.e. the angular velocity of the d,q axes relative to a point
fixed on the rotor. Thus in synchronously rotating axes p2s = T!Tr, with T = 2B f, but in d,q axes fixed
to the rotor, p2s = 0. Note that RD = RQ = RA.
The electromagnetic torque is determined from the power associated with the stator current and the
speed voltages, divided by the angular velocity in mechanical rad/s. If P is the number of poles,
3 P
Te ' [ Rd iq ! Rq id ] [ Nm ] . (121)
2 2

The dynamical equation of motion is


d Tr 2
Te ' J × D Tm (122)
dt P

where Tr is in elec rad/s and Tm = Tr × 2/P is the rotor speed in rad/s. D is a viscous damping term.

Transient simulation
Transients are calculated by integrating the differential eqns. (3.119),(3.120) and (3.122) in the form
p Rd ' vd ! Rd id T Rq
p Rq ' vq ! Rq iq ! T Rd
p RD ' ! RD iD ( T ! Tr ) RQ
(123)
p RQ ' ! RQ iQ ! ( T ! Tr ) RD
P
p Tr ' [ Te ! D Tm ]
2J
where T is the velocity of the rotating reference frame in elec rad/s. At the end of each timestep, new
values of Rd, Rq, RD, RQ, and Tr become available and the currents must be updated by solving eqns.
(3.115!3.117). Likewise the torque must be updated by means of eqn. (3.121). A typical timestep would
be 0.002 s, but it depends on the time-constants of the particular motor. The system of equations is non-
linear because of products like Rd iq in the torque equation and T Rd in the voltage equation.

If the d,q axes are fixed to the rotor, the speed voltages in eqns. (3.123) vanish, since T = Tr. The
integration is "driven" by the applied voltages vd and vq, which are defined in terms of va, vb, and vc.
while the flux-linkages and currents are "outputs". For example, suppose the supply is defined by
va ' vpk cos ( T t * ) ;
vb ' vpk cos ( T t * ! 2 B / 3 ) ; (124)
vc ' vpk cos ( T t * 2 B / 3 ) .

where T = 2B f and * is an arbitrary phase angle. According to the transformation equations


vd ' vpk cos ( T t ! 2 * ) ;
vq ' vpk sin ( T t ! 2 * ) .

If 2 = 2r the d,q axes are fixed to the rotor. If 2 = Tt they rotate at synchronous speed, with T = 2B f.
vd ' vpk cos * ;
(126)
vq ' vpk sin * .

Then vd and vq are constant. In particular, if * = 0, vd = vpk and vq = 0.


Page 3.46 SPEED’s Electric Machines

Non-simultaneous switching of the phases


Fig. 3.37 shows the connection of the motor to the supply. The contacts in line a are assumed to be
already closed, with no current flowing. Those in line b close at the "point on wave" Tt = 21, and line
c closes at the angle Tt = 21 + 22 ; that is, after a delay 22/T measured from the closure of line b.

Fig. 3.37 Non-simultaneous switching of the supply phases

With line-neutral voltages given by eqns. (3.124), then with * = 0 the line-line voltages are

vab ' vpk 3 cos ( T t B / 6 ) ;


vbc ' vpk 3 cos ( T t ! B / 2) ; (127)
vca ' vpk 3 cos ( T t 5 B / 6 ) .
During the interval 21 # Tt # 22 vc is undefined, but ic = 0 and ia = !ib. Current flows in the loop ab. No
torque is produced, and the rotor remains stationary. A particular case of interest is to close line b at
a peak of vab, that is, when Tt = !B/6, and then close line c after a further delay of 90E (21 = 0; 22 = 90E).
This strategy eliminates most of the oscillatory component in the transient torque, [Wood, 1965].
During the delay, stator current flows only in the loop through phases a and b. The line-line resistance
is RLL = 2Ra and the line-line inductance LLL is

LLL ' Laa Lbb ! 2 Lab ' 2 L11 . (128)

Obviously current is induced in the rotor. The rotor circuit can be reduced to a single circuit by fixing
the d-axis to the rotor and aligning it with the axis of the effective stator coil ab that results from the
loop current flowing in lines a and b. Then the whole circuit is reduced to that of a single-phase
transformer. With ic = 0 we have
2
id ' i [ cos 2 ! cos ( 2 ! 2 B / 3 ) ] ' 2 ia / 3 cos ( 2 B / 6 ) ;
3 a
2 (129)
iq ' ! ia [ sin 2 ! sin ( 2 ! 2 B / 3 ) ] ' ! 2 ia / 3 sin ( 2 B / 6 )
3

and the required alignment is achieved by setting 2 = 2r = !B/6, with id =2 ia/%3 and iq = 0: that is, all
the stator current is in the d-axis and none in the q-axis.

On the rotor, the Q coil is orthogonal to the axis of the effective stator coil ab and so RQ = iQ = 0, while
eqn. (3.123) holds for RD with T = Tr = 0:

p RD ' ! RD iD . (130)
The mutual inductance between the rotor D-coil and the stator d-coil is L12 = 3/2 × LaA. Since the d- and
a- coils have the same number of turns, the mutual inductance between the D-coil and phase a is also
3/2 × LaA = L12 when their axes are aligned. With 2 = 2r = !B/6 this mutual inductance is decreased to
Induction machines Page 3.47

Fig. 3.38 Alignment of dq axes with the conducting loop in phases a and b.

L12 cos (!B/6) = %3/2 × LaA. The axis of phase b is at an angle 5B/6 relative to the d-axis, so that with
current in the stator loop ab and ib = !ia phase b makes an equal contribution to Rd. Therefore the total
mutual inductance MD-LL is 2 × L12 × %3/2 = %3 L12, and

RLL ' LLL ia 3 L12 iD


(131)
RD ' L12 id LD iD
where RLL is the flux-linkage of the series connection of phases ab. Using the inverse reference-frame
transformation it can be shown that

RLL ' Rab ' Ra ! Rb ' 3 Rd (132)

and since ia = /3/2 id and LLL = 2 L11, we can write eqns. (3.131) as
Rd ' L11 id L12 iD ;
(133)
RD ' L12 id LD iD .

This agrees with eqns. (3.115) and (3.117), since LD = L22 . The voltage equation for the stator loop is
3
p RLL ' vLL ! RLL id . (134)
2

The solution during the delay interval can therefore proceed by integrating eqns. (3.130) and (3.134),
updating the currents at each timestep using eqns. (3.131); there is no torque and no rotation. After line
c closes, all three motor terminal voltages are known: eqn. (3.126) can be used for vd and vq, and the
solution can proceed in d,q axes as before, provided that the final currents at the end of the delay are
transformed into initial values for id and iq.
Page 3.48 SPEED’s Electric Machines

Steady-state operation
The theory so far assumes that we know the inductances L11, L12 etc., but we need to relate these to the
familiar "equivalent circuit" parameters of the induction motor: R1, R2, X1, X2, and Xm. In the steady
state the terms in eqns. (3.123) containing the p operator are zero so that
vd ' Rd id ! T Rq ' Rd id ! T ( L11 iq L12 iQ ) ;
(135)
vq ' Rq iq T Rd ' Rq iq T ( L11 id L12 iD ) .

We can combine these equations by writing V = Vd + jVq where Vd = vd/%2 and Vq = vq/%2 and similarly
with the currents. This can be seen as follows: if 2 = Tt then
ia ' [ id cos 2 ! iq sin 2 ] ' id cos T t iq cos ( T t B / 2 ) . (136)
From eqn. (3.135) it follows that
Vd j Vq ' [ R1 j T L11 ] ( Id j Iq ) j T L12 ( ID j IQ ) (137)
where R1 = Ra and L12 = 3/2 × LaA. We can write I2 = ID + j IQ directly for the "rotor current", while
for the "stator current" I1 = Id + jIq. The construction of the equivalent circuit requires also the
equivalent of eqn. (3.137) for the rotor, i.e.
0 ' [ R2 j s T L22 ] ( ID j IQ ) j s T L12 ( Id j Iq ) . (138)
where R2 = RA = RD. Eqns. (3.137) and (3.138) represent the conventional equivalent circuit and so we
have done enough to show that Rd = Rq = Ra; L11 = [X1 + Xm]/T, L22 = [X2 + Xm]/T, and L12 = 3/2 × LaA
= Xm/T. Note the appearance of slip s in eqn. (3.138): by dividing this equation throughout by s, the
rotor circuit is "referred" to the stator with the same frequency T and the familiar R2/s appears as the
only manifestation of rotation.

Small-signal analysis
For this we assume small perturbations (denoted by )) about a steady operating point (denoted by
subscript 0): vd = vd0 + )vd ; vq = vq0 + )vq ; id = id0 + )id ; iq = iq0 + )iq ; iD = iD0 + )iD; iQ = iQ0 + )iQ ; Rd =
Rd0 + )Rd ; Rq = Rq0 + )Rq ; RD = RD0 + )RD ; RQ = RQ0 + )RQ ; Tr = Tr0 + )Tr ; and Te = Te0 + )Te. T is missing
because it is considered fixed and equal to 2Bf, but Tr can vary. If we substitute these into the voltage
eqn. (3.123) and the torque eqn. (3.121) and simplify by subtracting the "steady-state" components, while
ignoring products of the form )x)y, then after some grinding we get

p)Rd !1/JsN T k r/JsN 0 0 )Rd )vd


p)Rq !Tr0 !1/JsN 0 k r/JsN 0 )Rq )vq
p)RD = k s/ J r N 0 !1/JrN T ! Tr0 !Rq0 )RD + 0 (3.139
)
p)RQ 0 k s/JrN 0 !1/JrN Rd0 )RQ 0
p)Tr a51 a52 a53 a54 !D/J )Tr 0

where

3 ( P / 2 )2 Rq0
a51 ' [ iq0 ! ] (140)
2 J L sN

3 ( P / 2 )2 Rd0
a52 ' ! [ id0 ! ] (141)
2 J L sN
Induction machines Page 3.49

3 ( P / 2 )2 k r
a53 ' Rq0 (142)
2 J L sN

3 ( P / 2 )2 k r
a54 ' ! Rd0 (143)
2 J L sN

and JsN = LsN/R1, JrN = LrN/R2, LsN = (LsLr ! M2)/Lr, LrN = (LsLr ! M2)/Ls, ks = M/Ls, kr = M/Lr, and P is
the number of poles. The notation has been changed using Ls = L11; Lr = L22, and M = L12 so that the
matrix eqn. (3.87) can be more easily compared with Vas' eqn. 2.10-36 [9] which is derived using space-
vector theory.
In the calculation of the coefficients a51..a54, the determination of Rd0 and Rq0 generally requires the
inversion of a 4 × 4 matrix but this can be avoided if the eigenvalues are calculated for the zero-slip
condition. This makes iD0 = iQ0 = 0 and T!Tr0 = 0. From eqn. (3.135) the steady-state conditions are

vd0 ' Rd id0 ! T Rq0 ; Rq0 ' L11 iq0


(144)
vq0 ' Rq iq0 T Rd0 ; Rd0 ' L11 id0

for which the solution is


T L11 vq0 Ra vd0 T L11 vd0 ! Ra vq0
id0 ' ; iq0 ' ! (145)
2 2
Ra T L112 Ra2 T2 L112
Page 3.50 SPEED’s Electric Machines

3.9 SPLIT-PHASE MOTORS


Fig. 3.39 shows the connection circuit of a split-phase induction motor. In SPEED the term “split-phase”
means that the single-phase supply is split between two motor phases, which generally have different
numbers of turns. One of these phases is called the main winding, and the other is called the auxiliary
winding (sometimes the start winding). The winding axes of the two motor phases are usually
orthogonal (. ' 90Eelec), and they are oriented so that forward-rotating flux generates an EMF in the
auxiliary winding that leads the EMF in the main winding, giving rise to the common expression that
“the auxiliary leads the main”.
Fig. 3.39 shows an auxiliary impedance Zc in series with the auxiliary phase winding. This impedance
is generally a capacitor, but it may be switched between two or more values. The motor can run with
no auxiliary impedance — that is, with the auxiliary winding open-circuited — but it cannot start from
zero speed with only one phase excited. Some common configurations are listed in the table:

Two-value capacitor motor Two capacitors, with a switch to change from


high capacitance at starting to low capacitance
during normal running.

Permanent-split capacitor motor A motor with one capacitor permanently


connected, and no switch.

Capacitor-start, induction-run A motor with a starting capacitor, which is


switched out when as motor nears its
operating speed.

Many variants are used in practice, including various complex tapped-winding arrangements, but the
analysis in this chapter follows the simple basic form of Fig. 3.39.

Fig. 3.39 Equivalent circuit of split-phase motor. During start-up, the cut-out switch switches from "start"
to "run" as the motor nears its operating speed. The OC (open-circuit) position of the cut-out
switch simply represents the electrical connections of a pure single-phase motor.

Split-phase motors are calculated by the method of forward and backward rotating fields (Veinott
[1959], Morrill [1929]); or by the method of symmetrical components (Fitzgerald and Kingsley [1961]);
or by the cross-field method (Puchstein and Lloyd [1941]) . The harmonics of the MMF distribution are
ignored to begin with, but will be treated later.
Induction machines Page 3.51

Fig. 3.40 Phasor diagram for split-phase induction motor in balanced operation. The axes of the main and auxiliary windings are
displaced by 90E (elec).

Balancing theory
Fig. 3.40 shows a simplified form of the phasor diagram for balanced operation. The voltage Vc across
the capacitor is just sufficient to force a 90E phase difference between Va and Vm, while Ia and Im are also
orthogonal. The current Ia is also the current in the capacitor, and leads Vc by 90E. The power factor
angle N is the same for both windings. In this example the main winding has more turns than the
auxiliary, so Vm > Va and Im < Ia.
Total MMF distribution; conditions for balance: The airgap MMF distribution is given by the following
equation in which "m" and "a" refer to the main and auxiliary windings, respectively; . is the angle
between the winding axes, and " is the phase difference between the currents ia and im :

F ' im0Nmcos 2 cos T t


ia0Nacos (2 .) cos (T t ")

(146)
1
' im0Nm[cos (2 T t) cos (2  T t)]
2
1
ia0Na[cos (2 T t . ") cos (2  T t .  ")]
2

Let im0Nm ' ia0Na ' Fm : then


1
F ' Fm[cos (2 T t) cos (2 T t . ")]...[b]
2
1 (147)
Fm[cos (2 ! T t) cos (2  T t .  ")]...[f]
2

For the backward component b to be zero,


" . ' B (148)
Then if . ' B/2, " ' B/2, i.e., the auxiliary current must lead the main current by 90E. In this case the MMF
reduces to F ' Fm cos (2!Tt) .

Let Na Im Va
a ' ' ' (149)
Nm Ia Vm
Page 3.52 SPEED’s Electric Machines

then Im
Ia ' e j" ; V a ' a V m e j" (150)
a
and if . = B/2 then
Im
Ia ' j ; V a ' ja V m (151)
a
These relationships are shown in the phasor diagram, Fig. 3.40.

Capacitor reactance and turns ratio: The circuit connection (Fig. 3.39) makes

Vm ' Vs ' Va Vc (152)

If we write V m ' Zm I m ' Zm Im e j N and V c '  j Xc I a , eqn. (3.152) reduces to

Xc
e  j "  a ' j e j N . (153)
a Zm
Taking real and imaginary parts,
Xc Xc
cos " ' a '  sin N ; sin " ' cos N (154)
a Zm a Zm
Taking the squares of these equations. and adding them, and substituting " = B ! ., we get
aZm sin .
Xc ' (155)
cos N
and if the first of eqns. (3.154) is divided by the second we get

a ' sin . @ tan N ! cos . . (156)

If . = 90E then
a Zm
Xc ' and a ' tan N . (157)
cos N

The turns ratio can be 1 only if N ' 45E, i.e. if the motor power factor is 1/%2.

Capacitor voltage

V c ' V m  V a ' V m [1  a e j"] . (158)

If . = B/2 then
V c ' V m [1  j a] (159)

and

Vc ' Vm 1  a 2 . (160)
Induction machines Page 3.53

Capacitor reactive power (with . ' B/2)

Im 1 a2
Q c ' Vc Ia ' Vm 1 a 2 × ' I m2 Z m . (161)
a a

In the balanced condition the total input power to the motor is

Pin ' 2 Im2 Zm cos N . (162)

But a ' tan N so cos N ' 1/ 1 a 2 and

1 a2 1 a2
Qc ' Pin . ' Pin . (163)
2 a cos N 2a

This has a minimum value Qc ' Pin, if a ' 1. The value of Qc is not unduly sensitive to the turns ratio.
Another expression for Qc is

I m2 Z m
Qc ' . (164)
sin N

Supply current (with . ' B/2)

1 a2 Im
Is ' Im ' . (165)
a sin N

The supply current does not have a minimum value. For all practical cases (N < B/2), the supply current
exceeds the main phase current.

Supply power factor (with . ' B/2) :The apparent impedance at the primary terminals is

j Ns Vs Vm
Z s ' Zs e ' '
Is I m(1 j/a)

Z me j N
' (166)
(1 j/a)

a Zm 1
' e j[N  tan (1/a)]
.
2
a 1

In general a ' tan N to achieve balanced operation, so in the special case when a ' 1, N ' 45E, then Ns '
0 and the supply power-factor is 1. To minimize the capacitor reactive power and maximize the supply
power-factor, we require a ' 1, i.e., a motor with a power-factor of 1/%2.
Page 3.54 SPEED’s Electric Machines

Comparison of three balanced conditions with different turns-ratios

Fig. 3.41 Balancing theory : (a) a ' /2; (b) a ' 1; (c) a ' 1//2

Let the turns ratio (aux/main) be a.


In all cases the main winding is connected directly across the supply and

V m ' 115 e j 0 V (167)


In case (b), a ' 1; the aux current is equal to the main current and leads it by 90E, while the aux voltage
is equal to the main voltage and leads it by 90E. Also, the supply current Is is in phase with the supply
voltage and the power-factor is 1. These conditions are expressed by

I m ' 1@0 e  j 45E A ; I a ' 1@0 e  j 45E A ; Is ' 2 e j0 A (168)

The capacitor voltage is given by

V C ' 115  j 115 ' 162@6 e  j45E V (169)


and the implied capacitor reactance is VC/Ia ' 162@6/1@0 ' 162@6 ohm, so that at 60 Hz the required
capacitance is

106
C ' ' 16@3 μF . (170)
377 × 162@6
The total copper loss is equal to

WCu ' Ra Ia2  Rm Im2 . (171)

If we assume the same copper volume in both windings, then

Ra ' a 2 Rm (172)

and both windings will run at the same current-density. The copper loss is then

WCu ' Rm (a 2 Ia2  Im2) . (173)


2 2 2
Suppose Rm ' 1@0 ohm. Then in case (b), WCu ' 1@0 × (1 × 1  1 ) ' 2@0 watts. Note that both phases are
working at a power-factor of cos 45E ' 0@707 lagging.
Induction machines Page 3.55

In case (a) we have a ' /2, so the aux winding has more turns than the main. The aux current is 1//2
times the main current and its wire diameter is /(1//2) ' 0@8409 times the diameter of the wire in the
main winding. The aux voltage leads the main voltage by 90E, but it is /2 times greater. Consequently
the capacitor voltage is

V C ' 115  j 115 /2 ' 199@2 e  j54@7E V (174)

Since the aux current leads the capacitor voltage by 90E, the phase of Ia is 90  54@7 ' 35@3E (from the
triangles in Fig. 3.41). We now impose the constraint that the electrical input power is the same as in
case (b). This means that Is cos N ' /2 A. From the triangles, N ' 54@7  35@3 ' 19@4E, so Is ' /2/cos 19@4E
' 1@5 A. It follows that

I a ' 0@8660 e j35@3E ; I m ' 1@225 e  j54@7E A . (175)


The implied capacitor reactance is VC/Ia ' 199@2/0@8660 ' 230 ohm, so that at 60 Hz the required
capacitance is

106
C ' ' 11@5 μF . (176)
377 × 230
The total copper loss is equal to

WCu ' Rm (a 2 Ia2  Im2) ' 1@0 × [ ( 2)2 × 0@86602  1@2252 ] ' 3@0 watts , (177)

which is 50% higher than in case (a). Note that both phases are working at a power-factor of cos 19@4E
' 0@94 lagging.
In case (c) we have a ' 1//2, so the aux winding has fewer turns than the main. The aux current is /2
times the main current and its wire diameter is /(/2) ' 1@1892 times the diameter of the wire in the main
winding. The aux voltage leads the main voltage by 90E, but it is /2 times smaller. Consequently the
capacitor voltage is

V C ' 115  j 115 / /2 ' 140@8 e  j35@3E V (178)

Since the aux current leads the capacitor voltage by 90E, the phase of Ia is 90  35@3 ' 54@7E (from the
triangles in Fig. 3.41). Again we impose the constraint that the electrical input power is the same as in
case (b). This means that Is cos N ' /2 A. From the triangles, N ' 54@7  35@3 ' 19@4E, so Is ' /2/cos 19@4E
' 1@5 A. This is the same current as in case (a), but the current Is is leading the supply voltage instead
of lagging. It follows that

I a ' 1@225 e j54@7E ; I m ' 0@8660 e  j35@3E A . (179)


The implied capacitor reactance is VC/Ia ' 140@8/1@225 ' 114@9 ohm, so that at 60 Hz the required
capacitance is

106
C ' ' 23 μF . (180)
377 × 114@9
The total copper loss is equal to

WCu ' Rm (a 2 Ia2  Im2) ' 1@0 × [ (1/ 2)2 × 1@2252  0@86602 ] ' 1@5 watts , (181)

which is only half the loss in case (b), and 75% of the loss in case (b). Note that both phases are working
at a power-factor of cos 35@3E ' 0@82 lagging.
Page 3.56 SPEED’s Electric Machines

Summary

Case Turns ratio a WCu VC Is Im Ia C PF


μF

(a) 1//2 1@5 140@8 1@50 0@9 1@2 23@1 0@94 lead

(b) 1@0 2@0 162@6 1@41 1@0 1@0 16@3 1@00

(c) /2 3@0 199@2 1@50 1@2 0@9 11@5 0@94 lag

Case (b) has the lowest supply current : it is, in fact, the lowest possible supply current.
The table shows that if we decrease the turns ratio, a < 1, as in case (a), the aux current increases and
so does the capacitance, while the copper losses decrease and the supply power-factor tends to become
leading.
These trends continue indefinitely; there is no minimum-loss condition. Ultimately as a decreases, the
copper loss is asymptotic to 1@0 W; this condition is achieved with infinite capacitance C, with VC '
115V, Im ' 0, zero turns in the aux winding and an infinite current in the aux winding. The supply
current is also infinite and the supply power-factor is zero. Clearly a hypothetical case.
Fig. 3.42 shows the variation of C, VC, Ia, Im, Is, and WCu as a function of a over a range from 0@1 to 2. To
the left of a ' 1, with a < 1, the supply power-factor is leading, and a lower copper loss is achieved at the
expense of a higher capacitance. To the right of a ' 1, with a > 1, the capacitance is lower but the copper
losses and the capacitor voltage are both higher, while the supply power-factor is lagging.

CAPACITANCE CURRENTS

Ia Is

Im

CAPACITOR VOLTAGE TOTAL COPPER LOSS

Fig. 3.42 Graphs of capacitance, capacitor voltage, currents, and copper loss vs. turns ratio.
Operation is perfectly balanced in all cases, with the same electrical input power, while the volume of copper
in the main and aux windings is identical, and both windings run at the same current-density.
Induction machines Page 3.57

Forward and backward revolving-field theory

Fig. 3.43 Single-phase equivalent circuit, modified with core-loss resistance

The forward- and backward-revolving field method is generally attributed to Morrill [1929]. The
approach described here is essentially a summary of the lucid account given by Veinott [1959], with the
addition of the iron loss WFe which is represented in the equivalent circuit by Rc.
Fig. 3.43 shows the circuit representation of the pure single-phase motor using forward and backward
revolving fields. Note that the “pure” single-phase motor has no auxiliary winding. The induced voltage
is attributed to two counter-revolving fluxes, which give rise to Ef and Eb respectively, where Ef ' ZfI2
and Eb ' ZbI2 in Fig. 3.43. The forward and backward impedances are

Z f ' j 0@5 Xm 45 Z b ' j 0@5 Xm 45


0@5 R2 0@5 R2
55 55 2  s
 j 0@5 X2 ;  j 0@5 X2 (182)
s

As Veinott puts it, half the mutual reactance is charged to the forward field and half to the backward
field. At zero speed or locked-rotor, s ' 1, Zf ' Zb and Ef ' Eb. But at normal loads, where s is small, Zf
exceeds Zb and most of the induced voltage is attributable to the forward-revolving flux. The backward-
revolving flux induces a backward-revolving pattern of currents in the rotor that actually suppresses
the backward-revolving flux.
Fig. 3.44 shows the revolving-field model applied to the capacitor motor. The currents Im and Ia are the
actual currents in the main and auxiliary windings. Voltages expressed in the form ZI are essentially
due to self-induction terms, while those expressed as voltage sources are due to mutual induction.
Secondary effects such as the deep-bar effect, saturation of the magnetizing reactance, core losses and
stray-load losses are not directly modelled. The voltage equations are
V m ' z1mI m  Z f ( I m  j a I a )  Z b ( I m  j a I a ) ;
(183)
V a ' z1a I a  Z c I a  j a Z f ( I a  j a I m )  j a ( Z b I a  j a I m ) .

The solution for the currents Im and Ia is


V mZ Ta  j a ( Z f  Z b ) V a V aZ Ta  j a ( Z f  Z b ) V m
Im ' ; Ia ' , (184)
2 2
Z TZ Ta  a ( Z f  Z b ) Z TZ Ta  a 2 ( Z f  Z b )2
where

Z T ' z1m  Z f  Z b ; Z Ta ' z1a  Z c  a 2 ( Z f  Z b ) . (185)


Page 3.58 SPEED’s Electric Machines

Fig. 3.44 Rotating-field model of capacitor motor, including core-loss resistance


In eqns. (3.183) the bracketed terms indicate fictitious currents If and Ib defined by
If ' (Im  jaIa);
(186)
Ib ' (Im  jaIa).

These are the so-called forward and backward currents. The voltage induced in the main winding by
the forward-revolving field is Vf ' ZfIf, and the voltage induced in the main winding by the backward-
revolving field is Vb ' ZbIb. Veinott deduces that the separate torque components contributed by the
2 2
forward and backward-revolving fields are given by If Re[Zf] and Ib Re[Zb] respectively, and the sum
of these is the electromagnetic torque Te in synchronous watts10 : thus if p ' pole-pairs and Ts ' 2Bf,

Te ' p (If2 Re [ Z f ]  Ib2 Re [ Z b ])/Ts (187)

When the operation is not perfectly balanced there is a double-frequency pulsating torque given by

Tp ' p ( V fI b  V bI f )/Ts . (188)

The forward and backward currents defined by eqn. (3.186) are closely related to the positive- and
negative-sequence currents defined by eqn (3.194) in the next section. They even suggest the idea of
transformation from variables Im and Ia to If and Ib, since Im and Ia can be recovered from If and Ib by
1
Im ' (If  Ib);
2
(189)
j
Ia ' (Ip  In).
2a

If is a solitary phasor, in the sense that it does not represent a balanced 2-phase set of phasors like Ip in
the next section. Flowing in the main winding alone, it produces the same forward-rotating ampere-
conductor distribution as the actual main and auxiliary currents, and If ' 2Ip. Likewise Ib ' 2In.

10
Note that when torque is quoted in “synchronous watts” TSW, the value in Nm is obtained by dividing by the synchronous speed
in rad/s, i.e. Te = TSW × p/T. In balanced polyphase machines TSW = Pgap.
Induction machines Page 3.59

Iron loss — To include the iron loss WFe in Fig. 3.44 we assume that it is known from an independent
calculation. (In PC-IMD, it is computed from the flux-density waveforms in different parts of the cross-
section). Then if E2m is known, we can write
WFe
I c ' E2m (190)
E2m2

Also by analogy with eqns. (3.184), if Vm is replaced by E2m and ZT is replaced by Zfb ' Zf  Zb ' ZT  z1m,

E2mZ Ta  j a ( Z f  Z b ) V a V aZ fb  j a ( Z f  Z b ) E2m
I2 ' ; Ia ' , (191)
ZfbZ Ta  a 2 ( Z f  Z b )2 ZfbZ Ta  a 2 ( Z f  Z b )2

where
I m ' I c  I2 (192)
and
E2m ' V m  z1m I m . (193)

The system of equations (3.190193) is solved by recursion, a direct algebraic solution being impossible
because of the nonlinearity of eqn. (3.190). Note that the model in Fig. 3.44 associates the iron loss with
the induced voltage E2m which includes both forward- and backward-field components. In other words,
this model does not attempt to distinguish between iron losses caused by the forward-revolving field and
those caused by the backward-revolving field.
In Ref. [19] the forward- and backward-revolving field model is extended to cover tapped-winding motors
of certain configurations, but without the inclusion of the iron-loss current Ic.
Page 3.60 SPEED’s Electric Machines

The symmetrical-component model

Fig. 3.45 Symmetrical-component model of capacitor motor

Fig. 3.45 shows the symmetrical-component model, which is also described in slightly different forms
by Veinott [5], Fitzgerald and Kingsley [7], and Suhr [26]. It can be derived from the forward- and
backward-revolving field model by making the reference-frame transformation
Ip Im Vp Vm
1 1 ja 1 1  j /a
' @ ; ' @ , (194)
In 2 1 ja Ia Vn 2 1 j /a Va

with inverses

Im 1 1 Ip Vm 1 1 Vp
' @ ; ' @ , (195)
Ia j /a  j /a In Va ja ja Vn

where a is the effective auxiliary/main turns ratio. If eqns. (3.195) are substituted in eqns. (3.183), we
get
V p ' (Z d  Z p) I p  Z d I n ;
(196)
V n '  Z d I p  (Z d  Z n) I n

where

Z p ' z1m  ( 2 Z f ) Rcf ; Z n ' z1m  ( 2 Z b) Rcb ; (197)

z1a
and Z d ' 0@5  z1m  Z c . (198)
a2
Note the inclusion of the two core-loss resistors Rcf and Rcb, one for the positive-sequence circuit and
the other for the negative-sequence circuit. Zf and Zb are given by eqns. (3.182).
Induction machines Page 3.61

The solution for the currents is


Vp(Zd  Zn)  VnZd Vn(Zd  Zp)  VpZd
Ip ' ; In ' . (199)
(Z d  Z p) @ (Z d  Z n)  Z d2 (Z d  Z p) @ (Z d  Z n)  Z d2
Eqns. (3.196) are expressed by the equivalent circuit in Fig. 3.45. The method of solution is similar to
that of the forward- and backward-revolving field method, but the incorporation of the iron loss WFe is
more complex because it is represented by the two separate resistors Rcf and Rcb, which are initially
unknown. Again assuming that the iron loss WFe is independently known, we can write

WFe Ep2 En2


'  . (200)
2 Rcf Rcb
The factor 1/2 arises because the transformations in eqns. (3.195) are not power-invariant. Let w be the
fraction of WFe/2 that is associated with the negative-sequence circuit. Then

En2 WFe Ep2 WFe


' w and ' (1  w) . (201)
Rcb 2 Rcf 2
or En2 Ep2
Rcb ' and Rcf ' (202)
w WFe / 2 ( 1  w ) WFe / 2
The circuit equations are solved recursively with a value of w that is updated by the constraint

En2 / Rcb
w ' . (203)
En2 / Rcb  Ep2 / Rcf
until it converges.
When the original revolving-field and symmetrical-component theories were first developed, practical
computation of the results was a laborious manual process. Possibly for this reason the original authors
do not seem to have attempted to incorporate iron-loss resistors in the circuit equations. With fixed
values for the iron-loss resistors, the currents can be calculated explicitly, and in this case the
computational burden is not greatly increased by including them; but there remains the problem of
knowing what values to use. The approach described here relies on two elements not available to the
original authors: one is the extremely fast solution by computer, and the other is the ability to estimate
the iron loss independently from the flux-density waveforms (which themselves can now be computed
numerically). Indeed it is possible to re-evaluate the iron loss recursively from the flux-density
waveforms as the solution proceeds, causing Rcf and Rcb to vary.
This level of computation is far beyond the scope of manual calculations. But we should not let ourselves
be carried away by the power of modern computing methods. It is true that the resistors Rcf and Rcb
represent the iron loss in the electrical equivalent circuit and improve the calculation of input power
and power factor. However, in the split-phase induction motor there are so many other departures from
the ideal model, that this enhancement may make little difference to the overall accuracy. For example,
stray loss, interbar currents, winding harmonics, and various manufacturing imperfections may have
a combined effect that is greater than the iron loss, which is often relatively small in these motors.
Many of the “parasitic” effects are regarded as second-order, and even if they could be included in the
equivalent-circuit model they would destroy its simple mathematical elegance. Yet their importance
is immense. The design engineer therefore must retain or develop a sound experimental understanding,
and ideally he needs almost a “sixth-sense” appreciation of the limitations of the analytical model.
A sound experimental understanding is liable to be largely phenomenological, and not simply a
verification of theory. For example, the values of the resistors Rcf and Rcb cannot be measured directly,
but only roughly correlated with a series of calculations over a range of operating conditions. For this
reason it is an advantage to have more than one analytical model, and in the next section a third method
is described — the cross-field model — which not only includes the iron-loss resistors but also deals with
the important case of the tapped-winding motor in its various configurations.
Page 3.62 SPEED’s Electric Machines

3.10 CROSS-FIELD THEORY OF TAPPED-WINDING CAPACITOR MOTOR

Fig. 3.46 Tapped-winding capacitor motor circuits. (a) Base circuit analyzed; (b) G-tap circuit, [1].

Circuits
Fig. 3.46(a) shows the generic circuit of a tapped-winding capacitor motor, in which the auxiliary
winding is fed from a tapping on the main winding. Positive rotation is in the counter-clockwise (CCW)
direction, and the axis of the main winding is retarded 90E relative to the axis of the aux winding, so that
the aux winding current and voltage normally lead the main winding voltage and current in phase, and
the rotor rotates from the aux towards the main. The directions of positive current (closed arrows) and
positive MMF and flux (open arrows) are shown in Fig. 3.46(a), and the polarities of the winding sections
are denoted by the customary dots. The capacitor is normally connected in series with the aux winding,
but provision is made in Fig. 3.46(a) for a capacitor also in series with the main winding. The tapping
is represented by the tap ratio t. When t = 0 the tap is at full voltage, but when 0 < t < 1 the tap is at a
lower voltage.
Fig. 3.47(a) shows a more flexible circuit that can represent several different configurations of tapped-
winding capacitor motor connections. The intention is to have a basic “core” analysis for the circuit of
Fig. 3.46(a), and to transform all the other circuits into this circuit for analysis.
As an example, the Grundfos or G-tap connection in Fig. 3.47(b) is reproduced in Fig. 3.46(b). This circuit
can be transformed into the base circuit by exchanging the “aux” and “main” labels and reflecting it
about a vertical line. Mathematically this reflection is the same as reversing the direction of rotation.11
The exchange of labels means that the impedances must be exchanged before the analysis starts, and the
appropriate currents must be exchanged after it finishes. Note that the “main” capacitor will be used
in the analysis, and the “aux” capacitor will be short-circuited.
Other proprietary configurations can be modelled by setting the tapping parameters x, y and t in Fig.
3.47(a), assigning the capacitor to the "main" or "aux" winding, and (if necessary) switching the per-unit
speed from positive to negative, so that the voltages and currents of the "aux" winding always lead those
of the main winding.

11
This makes the per-unit speed S equal to (s ! 1) instead of (1 ! s), where s is the slip.
Induction machines Page 3.63

x
y
t
(1 ! x) (1 ! y)
(1 ! t)

Main Aux

(a)

(1 ! t)

Aux Main
(b)

Fig. 3.47 Tapped-winding capacitor motor connections.

(a) General case


(b) G-tap configuration

In all cases, the aux winding axis is assumed to be 90Eelec ahead of the main winding axis.
Page 3.64 SPEED’s Electric Machines

Construction of voltage equations


The tapped winding is shown in Fig. 3.49. The voltage equations are constructed by inspection from Fig.
3.49 and solved by a direct method, except that iteration may be necessary to account for the variation
of magnetizing reactance and/or core loss with the flux level.
2
In the main axis the magnetizing reactance is separated into two uncoupled self-inductances t Xm and
2
(1 ! t) Xm, with two equal mutual reactances t(1  t)Xm, one on each side of the tap. The upper (tapped)
section is coupled to the rotor in the main axis by the mutual reactance tXm, while the lower (untapped)
section is coupled to the rotor by the mutual reactance (1  t)Xm. The rotor has the reactance Xm in the
2
main axis associated with fundamental flux crossing the airgap, and similarly XA ' a Xm in the auxiliary
axis, where a is the effective turns ratio (auxiliary/main).
The rotor is represented by a closed circuit in each axis. On the right-hand leg of each rotor circuit
appear the resistance and leakage reactance, and the self- and mutual reactances corresponding to
fundamental flux crossing the airgap.
The rotor circuits also contain speed voltages induced by rotation. Fig. 3.48 is drawn to assist in
checking the signs of the speed voltages.12 The main winding 1m has a self-impedance volt-drop and a
mutual volt-drop due to current in the “rotor main” winding 2m. Likewise the auxiliary winding 1a has
a self-impedance volt-drop and a mutual volt-drop due to current in the “rotor aux” winding 2a. There
are no speed voltages in the stator windings.
The rotor main circuit 2m has a self-impedance volt-drop and a mutual impedance volt-drop due to
current in the “stator main” 1a. In addition it has speed voltages of the form SxI induced by rotation at
the per-unit speed S through flux-linkages xI established by the orthogonal windings 1a and 2a. For
winding 1a, reactance x excludes the stator leakage; but the rotor leakage is included in x for rotor
winding 2a: in other words all the flux produced by current in rotor aux 2a generates a speed voltage in
rotor main 2m. The direction of the speed voltage in 2a induced by Im1 is shown in Fig. 3.48(c). The speed
voltage in 2m induced by Ia1 is shown in Fig. 3.48(d). Fig. 3.48(e) summarizes the directions of the speed
voltages the rotor circuits in each axis.

Fig. 3.48 Directions of speed voltages. In (a) is shown the direction of positive MMF and flux produced by current in a coil with
the polarity indicated by the cross and the dot. The arrow representing the flux defines the axis of the coil. The axes
of all the four windings are shown in (b). Note that the auxiliary axis is ahead of the main axis, since positive rotation
is counter-clockwise. Also, the axes of the stator and rotor coils 1m and 2m are in the same direction, as are those of 1a
and 2a. This means that positive current in coil 1m produces flux in the same direction as positive current in coil 2m.
In (c), positive current in coil 1m induces a speed voltage in coil 2a in the negative direction. Likewise, positive current
in coil 2m will also induce a negative speed voltage in coil 2a. In (d), positive current in coil 1a induces a speed voltage
in coil 2m in the positive direction. Likewise. positive current in coil 2a will induce a positive speed voltage in coil 2m.
No speed voltage is induced in coils 1m or 1a.

12
The sign convention for the rotor currents is opposite to that used in [3]. All coils on the same axis produce MMF and flux in the
same direction, if their currents are in the same direction.
Induction machines Page 3.65

I1m

V1m
t ( r 1 m + j x 1 m )I1m
main

Icm j t 2x M[I1m ! Icm]


E1

j t ( 1 ! t ) x M [I1m ! I1a ! Icm]

j t x MI2m
aux
Em I1a

V1a Ica
E2 j (1 ! t) x MI2m
j x AI2a
Ea

j t ( 1 ! t ) x M [I1m ! Icm]

j x A I1a
2
j ( 1 ! t ) x M [I1m ! I1a ! Icm]

stator
(1 ! t) ( r 1 m + j x 1 m )[I1m ! I1a] [ r1a + j x 1 a ]I1a

Zcm [I1m ! I1a] Zca I1a

V1a
I2m I2a

(r 2 m + j x 2m)I2m
S a t x M [I 1m ! Icm] ( r 2 a + j x 2 a) I2a

S a x M [I1a ! Ica]
j x MI2m
Sa(1 ! t)xM[I1m ! I1a ! Icm] j x AI2a

j t x M[I1m ! Icm]
Sa(x2m+xM)I2a
S a (xM + x2m)I2m j x A[I1a ! Ica]

j (1 ! t) x M(I 1 m !I 1 a ! Icm]
rotor
xA = a 2xM

Fig. 3.49 Cross-field equivalent circuit for general tapped-winding capacitor motor
Page 3.66 SPEED’s Electric Machines

CORE LOSSES
Core losses are incorporated in Fig. 3.49 by means of conductances connected in parallel with the flux-
generating elements in each branch of the circuit.13 Consider a coil wound on a core in which the losses
2
are W. If the EMF in the winding is E, the expression W = GE reflects the assumption that the core loss
is proportional to the square of the flux, which itself is proportional to the EMF. G is a fictitious
conductance, which draws current from the circuit in phase with the EMF E. In a polyphase motor, G
is the reciprocal of the conventional core-loss resistance Rc, and the total core-loss is given by

WFe ' m G E 2 (204)

where m is the number of phases. The conductance G can be determined by calculating WFe(bal) and Ebal
2
with balanced two-phase operation Then m = 2 and G = WFe(bal)/2Ebal .
In the split-phase motor WFe is represented by separate components for the main and auxiliary axes:

G E a2
WFe ' G Em 2
Ea 2 2
' G Em . (205)
a2 a2

The auxiliary winding has a times as many effective turns, so the core-loss conductance associated with
2 2 2
flux linking the auxiliary winding is G/a , and the associated loss is (G/a )Ea . Note that in the main
axis Em includes both sections of the tapped winding. The value of G is assumed to be the same for both
axes, and can be obtained from a calculation with balanced two-phase operation at the same flux level,
or at least at a comparable flux level to that of the actual split-phase operation.

Airgap power
The airgap power Pgap is given by the interaction of the stator currents and the induced voltages (EMFs)
in each branch: thus in terms of Fig. 3.49,

Pgap ' Re [ E1 ( I1m  I cm )( E2 ( I1m  I1a  Icm )( E a ( I1a  Ica )( ] . (206)

It is convenient to simplify this by writing I1 = I1m  Icm, I2 = I1m  I1a  Icm; I3 = I1a  Ica; and E3 = Ea:
then we have

Pgap ' Re [ E1 I1( E2 I2( E3 I3( ] . (207)

The induced voltages are

E1 ' j t 2xM I1 j t (1  t) xMI2 j t xM I2m ;

E2 ' j ( 1  t ) xM I2m j t ( 1  t ) xM I1 j ( 1  t )2 xM I2 ; (208)

E3 ' j xA I3 j xA I2a .

The EMF E1 can be interpreted as jTQm1 where Qm1 is a flux-linkage equal to tkw1mNmMm//2, and Mm is
the peak phasor component of flux in the main axis, expressed in terms of its peak phasor value. Nm is
the number of turns in the main winding, and kw1m is its fundamental winding factor, and T = 2B f.
Similarly we have E2 = (1  t)kw1mNmMm//2 and E3 = kw1aNaMa//2, where kw1a is the fundamental
winding factor and Na is the number of turns in the auxiliary winding, and Ma is the peak flux phasor
component in the auxiliary axis.
We can also define MMFs Fm1 = tkw1mNmI1/2; Fm2 = (1  t)kw1mNmI2/2; and Fm1 = kw1aNaI3/2, all
expressed in terms of their peak phasor values. If we substitute these in eqn. (3.207), we get

Pgap ' Re { j T [ M m ( F m1( F m2( ) M a F a( ] } ; (209)

13
The flux-generating elements are those associated with the fundamental airgap flux or mutual flux, which means that the core-
loss conductances are connected across the EMF induced by that flux in each axis. The leakage fluxes are not included in this
representation of core loss.
Induction machines Page 3.67

that is,

Pgap ' Re { j T [ M m F m( M a F a( ] } , (210)

where Fm = Fm1 + Fm2 is the total main-axis MMF.


We now introduce forward and backward components
Mf ' Mm  jMa; Mb ' Mm jMa
(211)
Ff ' Fm  jFa; Fb ' Fm jFa.
with inverses
1 1
Mm ' (M f M b ); Ma ' j (Mf  Mb)
2 2
(212)
1 1
Fm ' (F f F b ); Fa ' j (Ff  Fb).
2 2

Substituting from eqns. (3.212) in eqn. (3.210), and simplifying, we get

T ( (
Pgap ' Re ( j M f Ff j M b Fb ) . (213)
2

Torque
In a balanced polyphase motor the electromagnetic torque is equal to Pgap in synchronous watts, but in
unbalanced motors this is not so. The airgap power Pgap minus the rotor copper loss WCu R divides
between two components of electromechanical power conversion, one of which is associated with a
forward torque Tf and the other with a backward torque Tb. These torques are immediately identifiable
in eqn. (3.213) as Tf is produced by Mf and Ff, while Tb is produced by Mb and Fb.14 The average
electromagnetic torque is then given in synchronous watts by 15
T
Te '
2
(
Re ( j M f Ff ! j M b F(b ) . (214)

The same electromagnetic torque reacts on the rotor, but since the mechanical speed is (1  s) times the
synchronous speed, the power converted from electrical to mechanical form is (1  s)Te and therefore
the rotor copper loss is

WCu R ' Pgap ! (1 ! s ) Te . (215)

The amplitude of the double-frequency pulsating torque is given in Nm by


p
Tpls '
2 /
j M f Fb ! j M b Ff / , (216)

3.11 INTERBAR CURRENTS


See [31].

14
An alternative is to use dq-axis theory. Puchstein & Lloyd [1941] quote a torque formula that is effectively based on dq-axis
theory, and Veinott [1959] provides a physical explanation of this method; but in many early works the formal mathematical
development of the dq theory is not given in detail, greater reliance being placed on the physical arguments.
15
Eqns. (3.211)(3.214) apply to all AC induction machines provided that Mm, Ma, Fm and Fa are defined as the actual fluxes and
MMF’s. Equivalent equations are obtained with symmetrical component analysis except that the factor ½ is replaced by a factor
that depends on the form of the symmetrical components transformation; for capacitor motors the factor is usually 2.
Page 3.68 SPEED’s Electric Machines

3.12 T-CONNECTION

Fig. 3.50 (a) T-connection; (b) Equivalent single-phase connection of tapped-winding motor

The T-connection is a 3-phase connection of a single-phase induction motor. The motor has a main
winding MN and an auxiliary winding TA. For capacitor operation it is connected as shown in Fig.
3.50(b) to a 50% tapping on the main winding. For 3-phase operation it is connected as shown in Fig.
3.50(a) to a balanced three-phase supply. The phasor diagram of the three line voltages V1, V2, V3 is
shown in Fig. 3.51. In the small diagram at upper left, these phasors are shown in a star to make the
phase sequence clear. The larger diagram shows them in a closed mesh, convenient for analysis.
The T-connection can be analysed by the same cross-field theory that is used for the tapped-winding
capacitor motor. Comparing Figs. 3.50(a) and 3.50(b), we can see that the T-connection will operate the
same as the capacitor motor if the potential of terminal A is maintained the same in both cases.
Physically the capacitor in branch AN is replaced by two line-line source voltages V2 connected between
A and M, and V3 connected between N and A, such that VAM ' V2 and VNA ' V3.
The capacitor works slightly differently in that for balanced operation it sustains a voltage VC ' VAN
whose amplitude and phase are such as to render Va exactly 90E ahead of Vm. The phasors are shown
dotted in Fig. 3.51. The equivalence between the T-connection and the capacitor operation is established
if it is recognized that VAN '  VNA, whereupon

V AN '  V NA '  V1 e j 2 B / 3 ' V1e  j B / 3 (217)

Note that Va/Vm ' cos 30E ' /3/2 ' 0@866, so that the turns ratio must be Na/Nm ' 0@866.

Fig. 3.51 Phasor diagrams for the T-connection and the tapped capacitor motor.
Induction machines Page 3.69

3.13 APPROXIMATE CALCULATION OF SURFACE LOSSES


The following derivations are for estimating additional iron loss (mainly eddy-current loss) in the rotor
and stator tooth-heads, as a result of slot-openings and steps in the ampere-conductor distributions.
These losses contribute to the stray loss.
Additional iron loss in rotor tooth-heads, caused by stator slot-openings
(Wrt_so)

Fig. 3.52 Dips in airgap flux distribution caused by stator slot-openings.


In PC-IMD, B0 ' Bg1L.

Fig. 3.52 shows dips in the airgap flux-density distribution attributed to the stator slot-openings.
Considering the dip at the peak of the B-wave, its amplitude is given by
Bd ' kd B0 , (218)
where kd is well known from the conformal transformation originally due to Carter (see, for example,
[3] and [4]). The variation of kd with the slot-opening/gap ratio is shown in Fig. 3.53.

Fig. 3.53 Variation of kd with the ratio of slot-opening/gap

The flux-density variation in the tooth-head opposite the slot-opening is postulated as


2
B '  Bd cos B (219)
2d

where 2d is the angular extent of the dip, estimated as


Page 3.70 SPEED’s Electric Machines

w
2d ' (220)
r1g

where w is the stator slot-opening and r1g is the radius to the stator bore. (This is r1  g, where r1 is the
rotor radius and g is the physical airgap).
The rotation gives rise to a rate of change
dB dB B 2
' Tm ' Tm Bd sin B , (221)
dt d2 2d 2d

where Tm is the angular velocity in mech. rad/sec. If )t ' 2d/Tm is the time taken to traverse the dip,
the loss occurring in each rotor tooth-head as it passes one stator slot-opening is

)t/2 dB
2 Ce1 Tm Bd 2B2
U ' W Ce1 dt ' W joules (222)
)t/2 dt 2d
where W is the weight of one rotor tooth-head and Ce1 is the Steinmetz eddy-current coefficient given
2
in the WinSPEED manual as Ce/(2B ).16 Each rotor tooth-head passes a stator slot-opening f2 ' (rpm/60
× S1 ' Tm/2B × S1) times per second, and there are S2 rotor teeth, so the total rotor tooth-head loss due
to stator slot-openings is

1 1 Bd2 B2 Tm2
Wrt so ' U f2 S2 ' S1 S2 Ce1 W × W. (223)
2 2 2d 2B

The factor ½ in this equation accounts for the fact that successive Bd dips are sinusoidally modulated
by the main flux distribution (Fig. 3.52), so the mean value of its square is ½.
The effect of rotor slot-openings is not computed, on the grounds that rotor slot-openings of cast-cage
rotors are generally small. However, eqn. (3.223) could be adapted for this purpose if required, by
appropriate exchange of parameters between stator and rotor.
Example17 — suppose we have a 4-pole 7@5 kW motor with S1 ' 36 slots and S2 ' 32 rotor bars, running
at 1447@5 rpm. Then Tm ' 1447@5 × B/30 ' 151@6 rad/sec and f2 ' 1447@5/60 × 36 ' 868@5 Hz. The example
motor (p. 70) has a stator bore of 2r1g ' 130 mm, airgap g ' 0@5 mm, stator slot opening w ' 2@6 mm, stack
length = 154 mm, stacking factor 0@97, and rotor tooth head dimensions 0@7 mm × 11@3 mm, giving W ' 0@7
3
× 11@3 × 154 ×0@97/1E9 × 7800 ' 9@2 × 10 kg. Also Ce1 is 1@0626E5 in W/kg units (M19 24-gauge). 2d is
obtained from eqn. (3.220) as 2@6/65 ' 0@04 rad. The ratio of slot-opening/gap is 2@6/0@5 ' 5@2 so from Fig.
3.53 we get kd ' 0@65, and if the peak airgap flux-density is B0 ' 0@88 T18 we get Bd ' 0@65 × 0@88 ' 0@572 T.
From eqn. (3.223) we get

1 0@5722 × B2 151@62
Wrt so ' × 36 × 32 × 1@0626 × 105 × 9@2 × 103 × × ' 16@7 W. (224)
2 0@04 2B

16
Ce1 appears in the SPEED programs and SteelDBM as the parameter CfCe.
17
The example motor can be obtained in PC-IMD by File|New|Sizing, OK. Use M24 19g and run at 1447@5 rpm.
18
Taken from Bg1L in PC-IMD.
Induction machines Page 3.71

Additional iron loss in rotor tooth-heads, caused by steps in the stator ampere-conductor distribution
(Wrth)

Fig. 3.54 Step in flux-density distribution caused by slot ampere-conductors

Fig. 3.54 shows a step in the airgap flux-density distribution caused by the “MMF step” associated with
the ampere-conductors A in one stator slot:
μ0
)B ' A. (225)
g
The ampere-conductors are assumed to be sinusoidally distributed around the airgap. For a 3-phase
motor the maximum number of ampere-conductors in any slot is given by

3 4 kw1 Tph
A1 slot [pk] ' × × 2 Iph rms (226)
2 S1
which is derived as eqn. (3.253) on pp. 74ff. Note that A1 slot [pk] is a peak value in space, located at one
point on the stator bore at a particular instant of time. Around the stator bore, the slot ampere-
conductors are assumed to be sinusoidally distributed.
A point on the rotor surface passes the stator slot-opening w in a time )t given by
w
)t ' (227)
r1g Tm

where r1g is the stator bore radius, and Tm is the angular velocity in mech. rad/sec. If )t is sufficiently
short, we can assume that the slot ampere-conductors remain constant during )t. For example, in a 4-
pole machine at 1800 rpm, if w ' 1@5 mm and r1g ' 40 mm, )t ' 1@5/40 /(B/30 × 1800) ' 0@199 ms, which is
only 4@3Eelec of rotation.
Using the Steinmetz eddy-current loss term only, the energy loss per step19 is
2
dB )B 2
U ' Ce1 × )t × W . Ce1 W joules , (228)
dt )t
where W is the weight of the affected region, taken as the weight of the rotor tooth-head illustrated
(shaded) in Fig. 3.52(c). For one step, )B is calculated from eqn. (3.225) with A1 slot [pk] substituted for A1.
This same rotor tooth-head passes S1 stator slot-openings in one revolution, but the slot ampere-
conductors are sinusoidally distributed around the airgap. Therefore when multiplying by S1 to get the
total value of U for one revolution, we should use the root mean squared value of slot ampere-conductors
which is given simply by

19
“per step” means that we are considering one rotor tooth head passing one stator slot-opening, the one with the peak value of
ampere-conductors/slot. The others will be added shortly.
Page 3.72 SPEED’s Electric Machines

1 3 4 kw1 Tph
A1 slot [rms] ' × A1 slot [pk] ' × × 2 Iph rms (229)
2 2 2 S1
Note that the denominator /2 produces the root-mean-square value in space, while the nominator /2
produces the required peak phase current in time; although these instances of /2 cancel, they are both
retained so as not to lose sight of this important point.
Using eqn. (3.225) with A ' A1 slot[rms] from eqn. (3.229) to get )B, the total loss in one rotor tooth-head in
one revolution is

)B 2
S1 U ' S1 Ce1 W joules , (230)
)t
The time to rotate one revolution is J ' 2B/Tm, and there are S2 rotor teeth, so the total mean power loss
in all the rotor tooth-heads combined is

)B 2 Tm
Wrth ' S1 S2 Ce1 W × W, (231)
)t 2B
where )B is obtained from eqn. (3.225) using A ' A1 slot[rms] from eqn. (3.229).
Using eqn. (3.227) we can write this as

r1g T m2
Wrth ' S1 S2 Ce1 W )B 2 × W. (232)
w 2B

Example — The example motor (p. 70) has a stator bore of 2r1g ' 130 mm, stator slot opening w ' 2@6 mm,
stack length = 154 mm, and rotor tooth head dimensions 0@7 mm × 11@3 mm. Then W ' 0@7 × 11@3 × 154 ×
3
0@97 /1E9 × 7800 ' 9@2 × 10 kg. We have Tph ' 204 and kw1 ' 0@9598. The airgap length is g ' 0@5 mm. The
phase current is 8@82 A rms. From eqn. (3.229) we get A1 slot [rms] ' 287@8 ampere-conductors [space-RMS],
and then from eqn. (3.225) we get )B ' μ0 × 287@8/(0@5/1E3) ' 0@723 T.
There are 36 stator slots (S1) and 32 rotor slots (S2). The Steinmetz coefficient Ce1 is 1@0626E5 in W/kg
units (M19 24-gauge), and the speed is 1447@4 rpm so Tm ' 1447@5 × B/30 ' 151@6 rad/sec. Hence from eqn.
(3.232) we get

65 151@62
Wrth ' 36 × 32 × 1@0626 × 105 × 9@2 × 103 × × (0@723)2 × ' 5@4 W . (233)
2@6 2B
Induction machines Page 3.73

Additional iron loss in stator tooth-heads, caused by steps in the rotor ampere-conductor distribution
(Wsth)
By making the appropriate exchanges of rotor and stator variables, the stator tooth-head loss Wsth due
to steps in the rotor MMF distribution can be estimated in the same way. Thus w becomes the rotor slot-
opening and W the weight of one stator tooth-head. Also in eqn. (3.232) we should use r1 (the rotor
surface radius) instead of r1g (the stator bore radius).
The value of A2 slot[rms] for the rotor slot is computed slightly differently, as follows. By analogy with
eqn. (3.253) we can write

n 4 kw2 Tph2
A2 slot [pk] ' × × I2 2 ampere&conductors/slot , (234)
2 S2
where n is the number of rotor phases, kw2 is the rotor winding factor, Tph2 is the number of series
turns/phase in the rotor “winding”, S2 is the number of rotor slots, and I2 is the rotor current. By
ampere-turn balance,
I2 n kw2 Tph2 ' Iph m kw1 Tph . (235)
Let n ' S2 and Tph2 ' ½; since the rotor is short-circuited, this freedom of choice is permitted and kw2 '
1. Then with these values if we substitute for I2 from eqn. (3.235) in eqn. (3.234) we get
2 m kw1 Tph
A2 slot [pk] ' × Iph 2 ampere&conductors/slot . (236)
S2
This is the total slot current including the contributions induced from all m stator phases. However,
as in the case of the stator slot-ampere-conductors, we must use the space-RMS value in eqn. (3.225) to
calculate )B. This is given by

1 m 4 kw1 Tph
A2 slot [rms] ' × × × Iph 2 ampere&conductors/slot . (237)
2 2 S2

Example — The example motor (p. 70) has a rotor slot-opening w ' 1@36 mm and stator tooth head
3
dimensions 1@0 mm × 8@73 mm. Then W ' 1@0 × 8@73 × 154 × 0@97 /1E9 × 7800 ' 10@2 × 10 kg. We have Tph
' 204 and kw1 ' 0@9598. The airgap length is g ' 0@5 mm. The phase current is 8@82 A rms. From eqn.
(3.237) we get A2 slot [rms] ' 323@8 ampere-conductors [space-RMS], and then from eqn. (3.225) we get )B '
μ0 × 323@8/(0@5/1E3) ' 0@814 T. As before, the Steinmetz coefficient Ce1 is 1@0626E5 in W/kg units (M19
24-gauge), and Tm ' 188@5 rad/sec. Hence from eqn. (3.232) we get

65 151@62
Wsth ' 36 × 32 × 1@0626 × 105 × 10@2 × 103 × × (0@814)2 × ' 14@5 W . (238)
1@36 2B
Page 3.74 SPEED’s Electric Machines

Derivation of eqn. (3.226) on page 71 : peak ampere-conductors per slot (sinusoidally distributed)

Fig. 3.55 Distribution of slot ampere-conductors

The objective is to derive an equation for the peak ampere-conductors per slot, in a form that is general
enough to be used with any AC winding. The ampere-conductors in a slot containing Tc conductors (one
coil-side) is
i
Ac ' Tc × . (239)
a

where i is the instantaneous phase current and a is the number of parallel paths in the winding. This
formula is not very convenient because the number of coilsides varies from slot to slot, and the
disposition of the parallel paths is not necessarily known.
The approach is to work with an equivalent sinusoidal distribution of slot ampere-conductors, which
is derived as follows, starting from the case of a practical winding.
Consider the winding shown in Fig. 3.55. The number of stator slots is S1 ' 36; the number of pole-pairs
is p ' 2; the number of coils per pole is 3; the number of phases is m ' 3; and the number of turns per coil
is Tc ' 10. The winding is a lap winding with a coil-span of 8 slots, i.e. 8/9 pitch.
Within the encircled region there are 4 slots containing 1,2,2 and 1 coil-sides. Therefore the total number
of conductors in the encircled region is C ' (1  2  2  1) × 10 ' 60. In general this is equal to the number
of coil-sides per pole per phase multiplied by Tc :
2 S1 S1 Tc
C ' × Tc ' conductors/pole , (240)
(2 p) × m pm
since the total number of coils is S1 and there are 2 coil-sides per coil. In the example of Fig. 3.55
36 × 10
C ' ' 60 . (241)
2 × 3

Let i be the instantaneous phase current. Then the current in each conductor is i/a. The total number
of ampere-conductors in the encircled region is therefore
i
A ' C . (242)
a

This is the total number of ampere-conductors/pole. The number of turns in series per phase is
S1 1
Tph ' Tc × × . (243)
m a
Induction machines Page 3.75

Combining eqns. (3.242), (3.240) and (3.243) we get


Tph
A ' × i (244)
p
for the total number of ampere-conductors per pole. This equation is more convenient than eqns. (3.239)
or (242), because it does not contain the number of parallel paths, and Tph is a common parameter
because it determines the induced voltage.
Now if all the C conductors per pole were concentrated in one slot, there would be a step in the ampere-
conductor distribution equal to A, recurring with alternating signs every B/p radians, and the resulting
MMF distribution would be a square wave with a peak value equal to A/2 and a fundamental whose
amplitude is equal to 4/B × A/2 ' 2A/B ampere-conductors/radian. In the real winding the total step
in ampere-conductors is spread over 4 slots as shown in Figs. 3.55 and 3.56, and the fundamental MMF
distribution has an amplitude of
4 A
F1 ' kw1 × × ampere&conductors/radian . (245)
B 2

where kw1 is the fundamental winding factor.

Fig. 3.56 MMF distribution of the winding of Fig. 3.55

If the conductors were sinusoidally distributed around the stator bore, the MMF distribution would be
a pure sinewave with an amplitude of 20
A1
F1 [sine] ' ampere&conductors/radian , (246)
2
where A1 is the total number of ampere-conductors/pole, sinusoidally distributed. By equating F1 and
F1 [sine] from eqns. (3.245) and (3.246), we get
4
A1 ' kw1 × × A. (247)
B

This is the number of sine-distributed ampere-conductors per pole that produces the same fundamental
MMF as the real winding. Combining eqns. (3.244) and (3.247) we get

4 kw1 Tph
A1 ' × i. (248)
B p

20
See Miller TJE, Brushless permanent-magnet and reluctance motors, Oxford University Press, 1989
Page 3.76 SPEED’s Electric Machines

Now if A1 slot [avg] is the average number of ampere-conductors per slot it must be the case that
S1
A1 slot [avg] × ' A1 , (249)
2p
since there are S1/2p slots/pole. Also if the number of ampere-conductors per slot is sinusoidally
distributed it must be the case that the peak ampere-conductors per slot is equal to
B
A1 slot [pk] ' × A1 slot [avg] . (250)
2

Combining eqns. (3.248), (3.249) and (3.250) we get


4 kw1 Tph
A1 slot [pk] ' × i ampere&conductors/slot . (251)
S1
Note that the term “peak” here means the peak of a sinusoidal space distribution of ampere-conductors.
So far we have said nothing about the current waveform : all the ampere-conductors have been
calculated for the instantaneous phase current i. If this current is sinusoidal with a time-RMS value Iph,
then A1 slot[pk] is also a sinusoidal function of time and its peak (time) value is simply
4 kw1 Tph
A1 slot [pk] ' × Iph 2 ampere&conductors/slot . (252)
S1
In an m-phase motor the ampere-conductors of the m phases are superimposed in such a way that the
peak MMF distribution is multiplied by m/2. Also, instead of remaining fixed in space (but alternating
sinusoidally in time), the fundamental MMF distribution remains fixed in space but rotates at
synchronous speed. The peak ampere-conductors/slot can be considered to be rotating in space at
synchronous speed, and its value is

m 4 kw1 Tph
A1 slot [pk] ' × × Iph 2 ampere&conductors/slot . (253)
2 S1
Induction machines Page 3.77

3.14 DIRECT ANALYSIS OF INDUCTION MACHINE USING PRIMITIVE IMPEDANCE MATRIX


Eqn. (3.254) is a general form of the voltage equation for an electrical machine, in which [v] is a vector
of terminal voltages, [R] is the matrix of resistances, [L] is the matrix of inductances, and [i] is the vector
of currents:
[ v ] ' [ R ] [ i ] p{[ L ] [ i ]}
' [ R ] [ i ] p{[ L ] } [ i ] [ L ] {p[ i ]}
(254)
' [ R ] [ i ] Tm[ G ] [ i ] [ L ] {p[ i ]}

' {[ R ] Tm[ G ]} [ i ] [ L ] {p[ i ]}

where p is the operator d/dt and Tm is the angular velocity in [mech rad/s], and

M[ L ]
[G] ' (255)
M2m

is the matrix of the rate of change of inductance with rotor position 2m. We can write

d[ i ] d[ i ] p Tm
p[ i ] ' ' Te ' Te q[ i ] ' q[ i ] , (256)
dt d2 e 1s
where 2e is the position of a vector rotating at synchronous speed in [elec rad], and s is the slip. Thus

Te
Tm ' (1  s) (257)
p
with
Te ' 2 B f , (258)

f being the supply frequency. Eqn. (3.256) makes it possible to integrate the voltage equation with
respect to the electrical angle 2e rather than time.21 The voltage equation (3.254) is rearranged in the
Euler form for integration by the Runge-Kutta algorithm:

1
q[ i ] ' [ L ]1 [ v ]  ([ R ] Tm[ G ])[ i ] . (259)
p Tm

The electromagnetic torque is computed from22

1
T ' [ i ] [ G ] [i] . (260)
2 t

The inductance matrix [L] and the resistance matrix [R] have to be computed from the physical
dimensions of the machine and the winding layout. Most of the theory required for this is already
described elsewhere in SEM, but some special aspects are needed where the rotor is treated as a
collection of individual loops or circuits made up from pairs of adjacent bars, as shown in Fig. 3.57.
The procedure involves, first, the calculation of the primitive inductance and resistance matrices at
every integration-step. These matrices contain the self- and mutual inductances and the resistances of
every identifiable circuit in the machine before these circuits are connected into complete phase windings
and before any connections are made to the supply.
At every integration step, the primitive inductance and resistance matrices are reduced to the forms
actually used in the integration of eqn. (3.259), by means of connection matrices that represent the
interconnection of some of the circuits or coils to form complete phases.

21
This is convenient for controlling or managing the integration step-length. It is also convenient for plotting graphs, because
the phase relationships between different waveforms are more easily interpreted with respect to angle than with respect to time.
22
Note the coefficient ½. This is correct when [G] is [ML/M2] in ordinary phase variables. If the equations are transformed into
dq axes, this coefficient may disappear. See, for example, Kron G, Tensors for Circuits, General Electric Company, 1942; or
Hancock N N, Matrix Analysis of Electric Machinery, Pergamon Press, 1964.
Page 3.78 SPEED’s Electric Machines

Fig. 3.57 Rotor circuits formed from pairs of adjacent bars

An example is shown in Fig. 3.58, with 12 stator slots and 19 rotor bars. The stator has 3 phases, and each
phase has 4 coils in series, so the total number of “primitive” circuits or coils in the stator is 3 × 4 = 12.23
With reference to Fig. 3.57, it can be seen that the rotor has 19 “primitive” circuits. Thus the total
number of primitive circuits is 12 + 19 = 31. This is the order or dimension of the primitive [L] and [R]
matrices, and also of the primitive [v] and [i] vectors.

Fig. 3.58 Example with 12 stator slots and 19 rotor bars

However, it will be intuitively clear that each phase winding can be treated as a single circuit, even
though it comprises 4 coils in series. Therefore we expect to be able to reduce the number of stator
circuits from 12 to 3. This is indeed possible by means of a connection matrix, as explained in SEM-1:

[ L ]N ' [ C ]t [ L ] [C] and [ R ]N ' [ C ]t [ R ] [C] , (261)

where [C] is the connection matrix and [C]t is its transpose. Note that [C] is not square. In the example
of Fig. 3.58, it has 31 rows and 22 columns, and is denoted [C]31 × 22. Thus

[ L ]N22 × 22 ' [ C ]t 22 × 31 [ L ]31 × 31 [C]31 × 22 (262)

and the reduced matrices [L]N and [R]N are square.

23
It is of no technical significance, but curious to note in passing, that Kron referred to the “primitive circuits” as “coils”.
Induction machines Page 3.79

a b c A B C D
a1 1
a2 1
b1 1
b2 1
c1 1
c2 1
A 1
B 1
C 1
D 1

An example of a connection matrix [C] is shown above. This is for a simpler example than the machine
in Fig. 3.58. Instead of 4 coils per phase it has only 2: thus a1 and a2 in phase a, etc. Instead of 19 rotor
bars it has only 4 : A, B, C and D. Thus the primitive impedance matrices are of order (3 × 2 + 4) = 10, and
with both coils in each stator phase connected in series, the reduced matrices are of order (3 + 4) = 7.
The example shows the partition of the connection matrix into sub-matrices associated with the three
phases and the rotor. Thus there are 4 sub-matrices. The rotor sub-matrix is not reduced by the
connection, so it remains square. The stator sub-matrices are of dimension 2 × 1, and are not square.
After reduction of the stator circuits, as described above, the [L]N matrix for this simple example has the
square form on the left, and the [R]N matrix has the square form on the right:

a b c A B C D a b c A B C D

a ! • • • • • • a !

b • ! • • • • • b !

c • • ! • • • • c !

A • • • ! • • • A ! •

B • • • • ! • • B • ! •

C • • • • • ! • C • ! •

D • • • • • • ! D • !

REDUCED INDUCTANCE MATRIX REDUCED RESISTANCE MATRIX

These forms are quite general for a 3-phase motor. For a 2-phase motor, the c-row and c-column would
be absent. Note that the [L]N matrix is generally full. It is shown with ! symbols for the diagonal
elements, because these are enlarged by the leakage inductances. Smaller • symbols show mutual
inductances. Similarly the off-diagonal (mutual) resistance elements are smaller than the diagonal ones;
they appear only in the rotor partition. (See Fig. 3.61 below).
It is clear from eqn. (3.259) that only the [L]N matrix needs to be inverted. PC-IMD uses the Gauss-Jordan
elimination method. As to how frequently it needs to be inverted, that depends on the rate at which the
inductances are changing relative to the integration step. From preliminary tests, in which the Runge-
th
Kutta integration of eqn. (3.259) uses an integration step of 1/8 of an electrical degree, it appears that
the inductance matrix needs to be inverted only every 4 integration steps, that is, every 1/2 electrical
degree. However, such rules cannot be made too general because they obviously depend on the number
of slots on both the rotor and the stator, and the coil-pitches.
Page 3.80 SPEED’s Electric Machines

The voltage equation, eqn. (3.259), has a vector of voltages which for the simplified example is

a va

b vb

c vc

A 0

B 0

C 0

D 0

where va, vb and vc are the voltages applied at the terminals of the phase windings. For a cage rotor, the
individual circuits are shorted, and therefore the terminal voltage for each rotor circuit is zero.
If the stator is wye-connected, the phase voltages va, vb and vc are not actually known; what is known
is the set of applied line-line voltages
vab ' va  vb ; vbc ' vb  vc ; vca ' vc  va . (263)

These equations can be solved for va, vb and vc if we assume that


va vb vc ' 0 . (264)
With a wye connection certainly ia + ib + ic = 0, so we would expect that with sufficient symmetry eqn.
(3.264) would be satisfied, although this is only a conjecture and not a proof.
Fig. 3.59 shows an alternative approach in which the phase
currents ia, ib ic are replaced by two independent line
currents iu, iv by the connection matrix

ia 1 iu

ib = 1 1 iv (265)

ic 1

and then the voltages vu, vv are obtained as

vu 1 1 va
=
vv 1 1 vb (266)
Fig. 3.59 Wye (star) connection
vc

If we write
[ iabc ] ' [ W ] [ iuv ] and [ vuv ] ' [ W ] t [ vabc ] , (267)

the matrices [L]N and [R]N can be further reduced by one row and one column by

[ L ]NN ' [ W ] t [ L ]N [ W ] and [ R ]NN ' [ W ] t [ R ]N [ W ] , (268)


so that the applied line-line-voltages vu = vab and vv = vbc can be used directly. The solution will initially
be in terms of iu and iv, but all three phase currents can be recovered easily from eqn. (3.267).
Induction machines Page 3.81

Fig. 3.60 Calculation of end-ring leakage inductance Fig. 3.61 Mutual resistance between bars

Special features of inductance and resistance calculations


As already stated, most of the inductance and resistance theory is explained elsewhere in SEM, but
generally in relation to the impedances of complete windings. For the primitive coil impedances, one
or two special calculations are needed.
The first and most obvious is a modification to the airgap component of inductance Fig. 2.58, because
the coil-span of a rotor coil is generally not the same as that of a stator coil. This leads to a modification
in the logic of the algorithm, which is simple in principle (but delicate to program). It should be pointed
out that the algorithm in Fig. 2.58 assumes that airgap flux crosses the airgap in the radial direction.
A better method is to use Hague’s spatial-harmonic analysis of winding filaments, because this gives a
smoother variation of inductance with rotor position and better representation of harmonic effects, as
well as recognizing the dependence on the airgap length.24 Both methods rely on the Carter coefficient
for the effective airgap length.
The second modification is the calculation of the inductance of a segment of end-ring, and this uses the
following formula derived from Fig. 3.60, together with an estimate of the internal inductance of the end-
ring segment that uses the internal inductance of a rectangular bar.25

μ0 B r1 g 2 CERseg 1 " $
LERseg ' CERseg ln 2 ln 0@2235 × 1010 [H] (269)
B B r0 g " $ 2 CERseg

Here CERseg is the circumferential length of the end-ring segment, r0 and r1 are shown in Fig. 3.62, and
" and $ are approximated as r0 and 2r0 respectively. All the dimensions are in [mm].
Finally there is a mutual resistance between adjacent rotor coils, Fig. 3.61. This is easy to calculate from
th th
the resistance of the straight section of the bar in the slot, and it appears in every ( j, j +1) and ( j1, j)
element of the resistance matrix; that is, adjacent to the elements on the leading diagonal.
Note that although the leakage inductances are small, they are of critical importance (as is already
known from the conventional theory of the induction-motor based on the equivalent circuit). If they are
not in the correct proportion to the mutual inductances, the current and power-factor will be incorrect,
and so will the torque. However, the effects of winding harmonics will still appear at the same
frequencies (speeds).

24
See Design of Brushless Permanent-Magnet Machines, J.R. Hendershot and T.J.E. Miller, 2010. Also Design of Brushless
Permanent-Magnet Motors, J.R. Hendershot and T.J.E. Miller, Oxford University Press, 1994.
25
See Rosa and Grover, Formulas and Tables for the Calculation of Mutual and Self-Inductance, U.S. Bureau of Standards, 1911.
Page 3.82 SPEED’s Electric Machines

Fig. 3.62 Example of locked-rotor transient

Examples
Fig. 3.62 shows an example for a 3-phase motor with 4 coils/phase at locked-rotor, and Fig. 3.63 shows
a case with 16@7% slip. The upper traces show the phase currents together with the current in one rotor
circuit, while the lower trace shows the torque. The torque waveform is somewhat noisy, and this is
put down to the method used to calculate the airgap inductances, which assumes the flux crosses the gap
radially (see p. 81).

Fig. 3.63 Example of transient at 16@7% slip

The simulations in Figs. 3.62 and 3.63 have 7200 integration steps.
Induction machines Page 3.83

Wound rotor
In a wound rotor the coils are not independently shorted, but iA
are connected together in phases. Suppose for simplicity that
we have a rotor with 6 coils, connected in pairs (in series) to vA
form three phases, as in Fig. 3.64. 1
iB
If the coils are numbered 1,2,...6 we have
i1 ' i4 ' i A ; 2
v
i2 ' i5 ' i B ; B iC
(270)
i3 ' i6 ' i C .
3
In the stator phases, which have already been reduced to vC
single circuits a, b, c, we have the identities
ia ' ia ; 4
ib ' ib ; (271)
ic ' ic .

These equations are written in matrix form as 5

ia 1 ia
6
ib 1 ib N

ic 1 ic
Fig. 3.64 3-phase rotor circuit
i1 = 1 @ iA (272
)

i2 1 iB

i3 1 iC

i4 1

i5 1

i6 1

that is
[ iuvw ] ' [ C ] @ [ iabc ]
PRIMITIVE CONNECTED (273)
9 ×1 9 ×6 6 ×1

[iuvw] can be regarded as a vector of currents in a set of primitive circuits, including the stator circuits
already connected into phases. [iabc] denotes the currents in the final connected circuits, and [C] is the
connection matrix. Note that [C] is not square, and it cannot be inverted.
We can write a similar constraint for the voltages. By inspection of Fig. 3.64, we have
v A ' v1 v4 ;
v B ' v2 v5 ; (274)
v C ' v3 v6 .
and for the stator again the identities va ' va ;
vb ' vb ; (275)
vc ' vc .
Page 3.84 SPEED’s Electric Machines

This can be written in matrix form

va 1 va

vb 1 vb

vc = 1 @ vc (276)

vA 1 1 v1

vB 1 1 v2

vC 1 1 v3

v4

v5

v6

that is [ vabc ] ' [ C ] t @ [ vuvw ]


CONNECTED PRIMITIVE (277)
6 ×1 6 ×9 9 ×1

The relationship between the voltages and currents in the primitive circuits is

[ vuvw ] ' [ Z ] uvw @ [ iuvw ] . (278)

If we substitute this equation with eqn. (3.273) in eqn. (3.277), we get

[ vabc ] ' {[ C ] t [ Zuvw] [C] } @ [ iabc ] (279)

which means that the effective impedance matrix relating [vabc] and [iabc] in the connected circuits is

[ Zabc ] ' [ C ] t [ Zuvw] [C] . (280)


In the example, [Zuvw] has 9 rows and 9 columns, corresponding to the 6 primitive rotor circuits together
with the three phase circuits of the stator. But [Zabc] has only 6 rows and 6 columns: three each for the
rotor and the stator. Note that both [Z] matrices are square, even though [C] and [C]t are not.
Eqn. (3.276) can be written in terms of its partitions,

[vS] ZSS ZSR [iS]


= @ (281)
[vR] ZRS ZRR [iR]

If [vR] = [0], as is common in the rotor circuits of induction machines, this equation can be reduced to

[ vs ] ' {[ ZSS ]  [ ZRR ] 1 [ ZSR ] [ ZRS ] }[ is ] ' [ ZSS ] N [ is ] . (282)

Here, [ZSS]N is a 3 × 3 matrix that represents a much simplified form of the equations, but it is not as
useful as might appear because the reduction only works for complex (phasor) values and this restricts
it to sinusoidal operation without any harmonics. Even so, it can serve as the basis for many useful
calculations of unbalanced operation, for example, or split-phase machines.
For our purposes we apply eqn. (3.280) separately to the resistance and inductance matrices, and then
calculate the [G] matrix from eqn. (3.255).
Induction machines Page 3.85

3.15 POWER BALANCE IN WOUND-ROTOR INDUCTION MACHINES

Fig. 3.65 Per-phase equivalent circuit for wound-rotor machine

In general the wound-rotor machine has an external circuit connected via slip-rings. In Fig. 3.65 this
circuit is represented by the voltage V2 and the current I2. The external circuit could be a simple
resistor R, such that V2 = R I2 ; the analysis in this case can be incorporated in the equations of the cage-
rotor machine if R is added to the rotor resistance R2 to make a composite resistance R + R2.
More generally, the voltage V2 is applied by another machine or an electronic converter, as in Fig. 3.66.
The most advanced examples of the rotor external circuit are found in doubly-fed induction generators
used, for example, in wind turbine-generators; or in large variable-speed drives where the slip energy
is controlled rather than the main power through the stator. Historically, various other types of
electrical machine have been used in the rotor circuit, particularly to control the phase of the rotor
current.26 In the most advanced examples the electronic converter is a current-regulated PWM inverter
capable of bi-directional power flow, with a sophisticated control system based on space-vector
representation of V2 and I2.
For the purposes of analysing and designing the machine, however, it is sufficient to work with phasor
values of V2 and I2. The theory is then only slightly extended from that of the cage-rotor machine
considered earlier.
We will assume balanced conditions, so that only one phase need be analysed, as in Figs. 3.65 and 3.66.
The voltage-drop VCR across the collector-rings and brushes will initially be considered to be a
component of V2, so that it does not clutter the equations. (VCR is in phase with I2).
The basic power balance of the wound-rotor machine is exactly the same as that of the cage-rotor
machine expressed again here as
Pmech ' Pgap ( 1  s ) , (283)
where Pmech is the mechanical power (the sum of the shaft power and all the friction and windage
power), s is the slip, and Pgap is the electromagnetic power crossing the airgap. Of the total Pgap, the
fraction (1  s) is converted to mechanical power. In the cage-rotor machine, the remainder sPgap is
2
entirely dissipated in the rotor resistance R2 as I2 R2 (per phase). In the wound-rotor machine, the power
2
sPgap is the sum of the dissipation I2 R2 plus the real power in the external circuit, m Re{V2I2*}, where
m is the number of phases. We can write this as
s Pgap ' WCuR PR , (284)
where WCuR is the “rotor copper loss” and PR is the slip power communicated to the external rotor
circuit. In large machines WCuR is typically small, and it is intended that most of the slip power passes
out to the external rotor circuit, so that PR = sPgap approximately. In general,

26
Examples include the Leblanc exciter, the Krämer system, the Scherbius phase advancer, and several other slip-energy-control
schemes. In some cases all the elements are incorporated in a single machine, as in the Schrage motor.
Page 3.86 SPEED’s Electric Machines

P I2
grid

P
R1 jX 1 jsX2 R2 mech

V1 P E1 E2 V2 Slip rings
S
fS fR = s fS Shaft

Rf Lf
fS fR
P Vd
R
DC DC
ROTOR
FREQUENCY C dc
CONVERTER

Fig. 3.66 Doubly-fed induction machine

s
PR ' P  WCuR . (285)
1  s mech

The electrical power at the stator terminals is PS. If WCuS is the “stator copper loss” and WFe is the iron
loss, we have
1
PS ' Pgap WCuS WFe ' Pmech WCuS WFe . (286)
1  s

The power transfer to or from the grid is


Pgrid ' PS  PR . (287)
If we neglect losses completely, eqns. (3.285) and (3.287) take the simple and useful form
1 s
PS ' Pmech ; PR ' Pmech , (288)
1  s 1  s

with PR = sPS. Also


Pgrid ' PS  PR ' Pmech . (289)

Eqn. (3.289) states that the mechanical power is the difference between the stator input power PS and the
power PR passing from the rotor to the brushes via the slip-rings. It is written with a sign convention
suitable for subsynchronous motoring, in which PS, PR and Pmech are all positive; (see Fig. 3.67).
Other sign conventions might be more convenient for generating. For example if the signs of Pmech and
PS are reversed, all three quantities will be positive in supersynchronous generating, and
1 s
PS ' Pmech ; PR ' Pmech , GENERATING (290)
1  s 1  s

with PR = sPS. With s < 0 at supersynchronous speeds, PR will be positive. Also

PS PR ' Pmech GENERATING . (291)

Eqn. (3.288) shows the important essential feature of systems in which the slip power or rotor power PR
is controlled. In the doubly-fed induction generator, the mechanical power Pmech is regulated by the
control of the prime mover. If the slip power PR is also controlled, the slip is thereby determined, and
once the slip is determined, the stator power PS is also determined. If the range of slip is relatively
small, the rating of the frequency converter in the rotor circuit is much smaller than the total power
Pmech. Thus precise speed control is obtained over a limited range with a relatively small inverter.
Induction machines Page 3.87

+4/3 +1 !4/3 !1
Pgrid PS Pgrid PS
Pmech Pmech

PR +1/3 PR !1/3

SUBSYNCHRONOUS — MOTORING SUBSYNCHRONOUS — GENERATING

+4/3 +1 !4/5 !1
Pgrid PS Pgrid PS
Pmech Pmech

PR !1/3 PR +1/5

SUPERSYNCHRONOUS — MOTORING SUPERSYNCHRONOUS — GENERATING

+4/3 +1
Pgrid PS Sign convention :
Pmech PS PR Pmech
Resistor
only PR All positive for
+1/3 subsynchronous motoring

SUBSYNCHRONOUS — MOTORING
Fig. 3.67 Subsynchronous and supersynchronous motoring and generating

Subsynchronous motoring : slip-energy recovery


Suppose the slip is s = +0@25. The machine is running subsynchronously and the per-unit speed is 1  0@25
= 0@75. Then PR/Pmech = 0@25/(1  0@25) = 1/3, while PS/Pmech = 1/(1  0@25) = 4/3. The stator power PS
exceeds the mechanical power by 1/3, that is, by PR. A positive value of PR means that electrical power
is passing from the slip-rings to the external rotor circuit, which is effectively generating. If this power
PR is converted from slip-frequency to mains frequency by the rotor electronic controller, it can be fed
back into the mains so that the total power taken from the AC bus is PS  PR which is just 1@0 Pmech. The
bottom diagram shows the case where the rotor circuit is a plain resistor, and PR is simply dissipated.
Although the simplified calculations neglect losses, it emerges that returning the slip power to the AC
bus renders the machine capable of operating at a higher slip than would be the case if PR was merely
dissipated in a resistance.

Subsynchronous generating
Now suppose PR is reversed so the rotor electronic converter takes 1/3 p.u. power from the AC bus (with
Pmech as base power) and feeds it to the rotor. This means PR = 1/3. If the speed remains unchanged
at 0@75 p.u. of synchronous speed, the mechanical power Pmech must reverse; from eqn. (3.288), Pmech =
1, so PS = 4/3 Pmech, or 4/3 p.u. The machine is now generating at a subsynchronous speed of 0@75 p.u.
The power input at the stator terminals is 4/3 p.u.: that is, a power output of 4/3 p.u. But the rotor
requires 1/3 p.u. from the mains via the frequency converter, so the total available output is just 1@0 p.u.

Supersynchronous motoring
Suppose the machine is running supersynchronously at 1@25 p.u. of synchronous speed, so s = 0@25. If
Pmech > 0 the machine is motoring, and from eqn. (3.288) we have PS = 1/[1  (0@25)] Pmech = 4Pmech/5 or
0@8 Pmech, while PR = 0@25/1@25 Pmech = Pmech/5 or 0@2 Pmech. The negative PR again means that the
frequency converter is taking (from the mains) 0@2 p.u. of the total power Pmech to make up the deficit
required to run at the supersynchronous speed, since PS is only 0@8 Pmech.
Page 3.88 SPEED’s Electric Machines

Supersynchronous generating
Again suppose the speed is 1@25 p.u. with mechanical power fed to the shaft, Pmech = 1. Eqn. (3.288) gives
PS = Pmech × 1/[1  (0@25)] = 4/5 p.u. or 0@8 p.u., while the rotor power PR is 0@25/1@25 × Pmech =
+Pmech/5 or +0@2 Pmech. Positive PR means that the frequency converter is absorbing Pmech/5 from the
slip-rings and delivering it to the AC bus, while the negative PS means that the stator is also delivering
power, 4 Pmech/5, to the AC bus : in other words, both the rotor and the stator are generating.

Electrical analysis
The following treatment of the electrical circuit starts from first principles and follows the same path
as the conventional analysis for the cage-rotor machine, but it singles out the effects of the applied rotor
voltage. It also develops specific equations for the components of the power balance, as is necessary for
design calculations.
The induced EMF in the primary (stator) is given by the familiar equation

2B
E1 ' kw1T1 M fS , (292)
2
where kw1 is the fundamental winding factor and T1 is the number of turns in series per phase in the
stator winding. M is the fundamental airgap flux/pole and f S is the frequency in the stator circuit, that
is, the supply frequency f.
The rotor winding has kw2T2 effective turns in series per phase. The frequency in the rotor circuit is
f R = s × f S or just sf. So the induced EMF in the secondary (rotor) is

2B
E2 ' kw2T2 M fR , (293)
2
Taking the ratio E2/E1, we get

1
E2 ' s E1 × , (294)
n

where n is the effective turns ratio between an equivalent winding with kw1T1 effective turns in series
per phase and the actual winding with kw2T2 effective turns in series per phase: thus
kw1T1
n ' . (295)
kw2T2
Now referring to the equivalent circuit of Fig. 3.65, evidently
E2  V2
I2 ' , (296)
R2 j s X2
where X2 is the rotor leakage reactance at the frequency f. Following the conventional procedure, let
kw2T2 1
I2N ' I2 ' I2 . (297)
kw1T1 n
This current flows in the equivalent winding with kw1T1 effective turns and produces the same MMF as
the actual rotor current I2 in the actual rotor winding with kw2T2 effective turns in series per phase.
Now we use eqn. (3.294) to substitute for E2 and eqn. (3.297) to substitute for I2 in eqn. (3.296), giving

1 s E1 / n  V2
I2N ' . (298)
n R2 j s X2

If we multiply the numerator and denominator of the right-hand side expression by n/s, we can express
this equation as
Induction machines Page 3.89

Fig. 3.68 Simplification of the equivalent circuit in stages, to solve for I2N.

E1  V2N
I2N ' . (299)
R2N/s j X2N

This equation represents the right-hand branch of the circuit in Fig. 3.68(a). Also

R2N ' n 2 R2 and X2N ' n 2 X2 , (300)


and
n
V2N ' V . (301)
s 2
Eqns. (3.300) give the rotor resistance and reactance “referred from the rotor to the stator winding”, and
are familiar in the literature. Eqn. (3.301) gives the applied rotor voltage V2N referred from the rotor
to the stator winding. This equation is not common in the literature, as most texts are dealing only with
cage-rotor machines in which V2 = 0. Note that when s is small, a small voltage injected at the slip-rings
appears as a much larger voltage in the stator circuit.
The solution of the equivalent circuit revolves around the solution of eqn. (3.299), and the method is
illustrated by the sequence of circuit diagrams in Fig. 3.68. First, Fig. 3.65 is reduced to Fig. 3.68(a),
using eqn. (3.299) with the impedances R2N/s and jX2N and the voltage V2N referred to the equivalent
winding. Since this winding has the same effective turns as the stator winding, and is driven by the EMF
E1, the ideal transformer disappears and all the circuits in Fig. 3.68 work entirely at the frequency f.
Next, the impedance Z1 = R1 + j X1 and the parallel combination of Rc and jXm are combined into a single
admittance Ym, so that the left-hand part of the circuit can be reduced to its Thévenin equivalent,
V1 Z1
V1N ' and Z1N ' . (302)
1 Z1Y m 1 Z1Y m
Page 3.90 SPEED’s Electric Machines

The solution is iterative. I2 is first evaluated using Fig. 3.68(b):


V1N  V2N
I2N ' . (303)
Z1N R2N/s j X2N
Then we get E1 = (R2N/s + j X2N)I2 + V2N. From this we get Im as E1Ym, and then I1 = Im + I2N. At each
iteration the saturation factor for Xm and the core losses are updated from the current iterate of E1,
while the saturation factors for X1 and X2 are updated from the current iterates of I1 and I2.

Power balance
The power balance can now be evaluated, and this is where the physical interpretation is most
interesting. Before looking at the individual components, note that the voltage “referral” ratio from
the rotor to the stator is n/s, which we used in eqn. (3.301) and other equations; the current referral ratio
2
is 1/n, as in eqn. (3.297); and the impedance referral ratio is the quotient of these, i.e. (n/s)/(1/n) = n /s,
which we used in eqns. (3.300). We would expect the power ratio — or more generally the volt-ampere
ratio — to be the product of the voltage and current ratios : thus (n/s) × (1/n) = 1/s. To use this, we
would say that 1 W in the actual rotor circuit would appear as 1/s apparent watts in the stator circuit.27
It is useful to bear this 1/s ratio in mind as we work through the individual power components.
2
Starting with the rotor copper loss, the actual value is I2 R2 watts per phase, and this appears directly
in Fig. 3.65. In the referred circuit of Fig. 3.68(a), however, the power associated with R2 is apparently
2 2 2 2
equal to I2N × R2N/s = (I2/n) × (n R2)/s = I2 R2/s watts per phase. Immediately we can see the power
2 2
referral ratio 1/s, as anticipated above. Since I2N × R2N = I2 × R2, there appears in the stator circuit an
2 2
additional power component equal to I2N × R2N(1/s  1) or I2 × R2(1/s  1), or WCuR × (1/s  1) which is
not apparent in the rotor circuit. This additional power somehow vanishes in crossing the airgap, and
we know it to be attributable to the electromechanical power conversion Pmech.
In a cage-rotor motor, WCuR × (1/s  1) is the only component of Pmech. But in the wound-rotor machine,
there is also virtual power associated with V2N, which appears as Re{V2NI2N*} watts per phase in the
stator circuit. This can be written Re{(n/s)V2 × (1/n)I2} = Re{V2I2*}/s, which is 1/s times the actual real
power associated with V2, that is, Re{V2I2*}. We recognize this term as PR when multiplied by the
number of phases, m. Again there is a lost component of power (1/s  1) × PR which appears in the stator
circuit but not in the rotor circuit, and like the previous component WCuR × (1/s  1) it can only be
interpreted as an additional component of Pmech. Thus exactly as we had it in eqn. (3.285),

1
Pmech '  1 × (WCuR PR) . (304)
s
But now we have precise and rigorous expressions for PR and WCuR :

PR ' m s Re V2N I2N( ' m Re V2 I2( (305)


and

WCuR ' m Re I2N2 R2N ' m Re I22 R2 . (306)

The electrical analysis does nothing to alter the power balance equations (1.2831.287) or their lossless
variants eqns. (3.2883.291). However it provides formulas for the evaluation of the component powers,
and in particular the slip-ring power PR which does not appear in the theory of the cage-rotor motor.
Eqn. (3.304) can be written as

(WCuR PR) R2
Pmech ' ( 1  s ) × ' m ( 1  s ) × I2N2 Re V2N I2N( . (307)
s s

This shows that Pmech is (1  s) times the virtual rotor power appearing in the stator circuit, which is
often called the airgap power Pgap, as in eqn. (3.283). The airgap power is thus given by
27
It might be better to speak of “virtual watts”, to avoid confusion with the standard term “apparent power”, which is not the same
thing as this “referred power”.
Induction machines Page 3.91

Fig. 3.69 Phasor diagram (subsynchronous, motoring)

(WCuR PR) R2
Pgap ' ' m × I2N2 Re V2N I2N( . (308)
s s

It is actually simpler to evaluate Pgap using

Pgap ' m Re E1I2N( . (309)

This equation is consistent with eqn. (3.308), and it is also obvious from Fig. 3.68.
It only remains to note that the power at the stator terminals PS is equal to Pgap plus the losses arising
2
in the stator circuit proper, that is, WCuS = m I1 R1 plus the iron loss WFe, which appears in the
2
equivalent circuit as m E1 /Rc. This part of the power balance is expressed by eqn. (3.286) .

Phasor diagram
The phasor diagram is drawn for Fig. 3.68(b), with E1 = EuN + VZ2N. VZ2N is simply the combined voltage
drop in the rotor resistance and leakage reactance, referred to the stator. EuN has two components ERN
= (1/s  1)R2N I2N and V2N. ERN is in phase with I2N and is associated with that part of the mechanical power
attributable to the rotor resistance; the remaining part of the mechanical power is associated with the
component of V2N that is in phase with I2N. All the mechanical power is attributable to the single voltage
EuN.
The full phasor diagram is shown in Fig. 3.69 for a subsynchronous motoring condition, but it is helpful
to simplify it before trying to use it or interpret it. The main simplification is to set Ym = 0 and neglect
the magnetizing current. The leakage reactances X1 and X2N can then be added to give
Xs ' X1 X2N . (310)
Thévenin voltage V1N becomes equal to V1, and eqn. (3.303) becomes
V1  E uN V1  E uN
I2N ' I1 '
. (311)
R1 R2N j Xs j Xs

This is represented by the electrical equivalent circuit in Fig. 3.70, which is that of a nonsalient-pole
synchronous machine with an EMF EuN and synchronous reactance Xs, connected to an infinite AC bus
of voltage V1. Xs is a property of the machine itself, and does not depend on the frequency converter.
Fig. 3.70 also shows the power flows discussed earlier. (The sign convention remains the original one,
in which PS, PR and Pmech are all positive for subsynchronous motoring).
Page 3.92 SPEED’s Electric Machines

Fig. 3.70 Simplified equivalent circuit of lossless wound-rotor machine

The EMF EuN is analogous to the open-circuit EMF of the synchronous machine. It is given by
E uN ' E RN V2N . (312)
Its amplitude is partially controlled by the frequency converter, which also provides the “field” current
I2. The analogy with the synchronous machine must not be taken too far because, unlike the exciter of
a synchronous machine, the frequency converter handles a significant fraction of the total power.
Moreover, in the synchronous machine the EMF has only one component, whereas here EuN has two
components that are not necessarily equal or even in phase with one another.
Even so, when taken together with the power-balance relations, Fig. 3.70 suggests that we can control
V2 and I2 to control EuN and hence control the power and the power-factor at the grid terminals. It
further suggests that we can use the classical theory of the synchronous machine connected to an
infinite AC bus as the basis for the theory of operation.28
Equivalent synchronous machine
The general form of the simplified phasor diagram is shown in Fig. 3.71. We have
V2N E RN Z I1 ' V1 , (313)
where Z = R1 + R2N + jXs. The reference phasor is V1 : thus
V1 ' V1 j 0 ; (314)
The referred rotor voltage is defined by its RMS value V2N [eqn. (3.301)] and its phase angle *2 relative to
V1 : thus
V2N ' V2N cos *2 j V2N sin *2 ; (315)

The current is I1 has a phase angle N1 relative to V1, and an RMS value I1, with real and imaginary
components IR and IX :

j N1
I1 ' ' I1 e ' I1 cos N1 j I1 sin N1 ' IR j IX ; (316)

The voltage ERNdefined in Fig. 3.68(b) is expressed as


j N1
E RN ' ER e ' R E I1 ; (317)

where
1
R E ' R2 1 . (318)
s

28
See, for example, Miller T.J.E., Reactive Power Control in Electric Systems, John Wiley & Sons, New York, 1982; or Acha A. et
al, Power Electronic Control in Electrical Systems, Newnes Power Engineering Series, 2002; or Hendershot J.R. and Miller T.J.E.,
Design of Brushless Permanent-Magnet Machines, Chapter 9, Generating, Motor Design Books LLC, 2010 (hendershot@ieee.org).
All these references develop the theory of the operation of synchronous machines in exactly the form used here.
Induction machines Page 3.93

Fig. 3.71 Simplified phasor diagram

In terms of real and imaginary components, eqn. (3.313) becomes


V2N cos *2 R IR  Xs IX ' V1 ;
(319)
V2N sin *2 Xs IR R IX ' 0 ,
where
R2N
R ' R1 R2 R E ' R1 . (320)
s
These equations can be solved for the real and imaginary components of I1 : thus

 Xs V2N sin *2 R (V1  V2N cos *2 )


IR ' I cos N1 ' ;
R 2 X s2
(321)
 R V2N sin *2  Xs (V1  V2N cos *2 )
IX ' I sin N1 ' .
R 2 X s2

Also from Fig. 3.71 we have eqn. (3.312) that defines EuN : hence in real and imaginary components

EuN cos * ' V1  (R1 R2N) IR Xs IX


(322)
EuN sin * '  (R1 R2N) IX  Xs IR .

The complex power at the stator terminals is defined by


j N1
PS j QS ' V1 I1( ' V1 I1 e
(323)
' V1 I1 cos N1  j V1 I1 sin N1 ,

where N1 is the phase angle of I1 with V1 as reference phasor. In Fig. 3.71, N1 < 0 so that both PS and QS
are positive. Positive QS means that reactive power is being absorbed at the stator terminals from the
PCC, the point of common coupling between the grid, the stator, and the rotor frequency-converter.

The same analysis is now performed to find the complex power at the referred rotor terminals:
j *2 j N1
PRN j QRN ' V2N I1( ' V2N e I1 e
(324)
' V2N I1 cos N2 j V2N I1 sin N2 ,

where N2 = *2 N1. In Fig. 3.71, *2 < 0, N1 < 0, and N2 < 0 but *N2* < 90E: thus PRN > 0 while QRN < 0. With
the sink convention at the rotor terminals in Fig. 3.71, positive PRN means that power is flowing from the
slip-rings to the rotor frequency converter. Negative QRN here means that reactive power is being
generated by the rotor frequency converter and fed through the slip-rings to the rotor. The frequency
converter is therefore providing a proportion of the excitation.
Page 3.94 SPEED’s Electric Machines

From eqn. (3.322) we can solve for IR and IX and then write the real and reactive powers PS and QS as

 Xs EuN sin * (R1 R2N) (V1  EuN cos * )


PS ' V1 IR ' V1 ;
(R1 R2N) 2 Xs2
(325)
 (R1 R2N) EuN sin * Xs (V1  EuN cos * )
QS ' V1 IX ' V1 .
(R1 R2N) 2 Xs2
When the combined resistance R1 + R2N is much smaller than Xs, as is commonly the case, these
equations simplify considerably to give
V1 EuN
PS '  sin * (326)
Xs
and
V1 ( V1  EuN cos * )
QS ' . (327)
Xs
The P* relationship in eqn. (3.326) is similar to that of the synchronous machine. The angle * is called
the load angle, as in synchronous-machine parlance. In Fig. 3.71 * < 0, so PS > 0. This is consistent with
the motoring sign in which the power is positive if V1 leads EuN, meaning that power is passing from the
stator terminals to the virtual rotor terminals inside the machine.
Fig. 3.71 is drawn with EuN cos * < V1, so the reactive power per phase QS at the stator terminals given
by eqn. (3.327) is positive, as before. Again, positive QS calculated by eqn. (3.327) is interpreted as the
reactive power supplied to the stator from the PCC.
For the virtual complex power at the referred rotor terminals, the simple model of a synchronous
machine connected to an AC bus through a reactance does not apply. The voltages V2N and EuN in Fig.
3.71 are separated by ERN which is a virtual resistive voltage REI1 in phase with the current. This is not
quite as simple as in the case of V1 and EuN, which are separated by an essentially reactive voltage that
has little or no power associated with it. Even so, it is possible to develop interesting relations for PRN
and QRN as follows:

j*2
( E uN  V2N)( V2N E uN(  V2N 2 V2Ne E uNe j*  V2N 2
PRN j QRN ' V2N I1N(
' V2N ' ' , (328)
RE RE RE

from which with *R = *2  * we get

V2N [ EuN cos *R  V2N ] V2N EuN sin *R


PRN ' and QRN ' . (329)
RE RE
In Fig. 3.71 *2 < * < 0, so *R < 0 and QRN < 0. As before, negative QRN here means that reactive power is
being generated by the rotor frequency converter and fed through the slip-rings to the rotor. Also Fig.
3.71 is drawn with EuN cos *R > V2N, so PR > 0. As before, power is flowing from the slip-rings to the rotor
frequency converter.
The difference between QS and QRN is

QS  QRN '  V1 I1 sin N1  V2N I1N sin N2 , (330)

where N1 + N2 = *2 (Fig. 3.71). If R = 0 (eqn. 3.320), then from eqns. (3.321) we have

V2N sin *2 V1  V2N cos *2


I1 cos N1 '  ; I1 sin N1 '  . (331)
Xs Xs
Substituting eqns. (3.331) in eqn. (3.330) and simplifying,

QS  QRN ' I12 Xs . (332)


Induction machines Page 3.95

This accounts for the reactive power absorbed in the series reactance Xs. As to how much is supplied
from the stator and how much from the rotor terminals, that depends on the particular signs and
magnitudes of QS and QRN. Eqns. (3.331) should not be used normally, as R is generally not negligible.
The actual reactive power at the rotor terminals would be expected to be QR = sQRN, according to the
power referral ratio deduced on p. 90. This can also be deduced by substituting V2N and I2N in eqn. (3.324),
using eqns. (3.297) and (3.301). The behaviour of QR and QRN is analogous to that of PR and PRN, but the
difference (1  s)QRN is simply lost: there is no electromechanical conversion of reactive power, and it
is not conserved through the frequency-changing transformer that is the induction machine.
The power at the grid terminals is determined by summation at the PCC, as in eqn. (3.289). The reactive
power at the grid terminals cannot be determined by such a summation, because the reactive power is
not conserved through the frequency converter. A frequency-converter that comprises two back-to-back
4-quadrant PWM inverters has independent control of the reactive power at both its input and output
terminals. Such a frequency-converter permits complete flexibility in the control of power in motoring
and generating at both subsynchronous and supersynchronous speeds.
It is possible to consider restricted operation using a frequency converter that has a diode rectifier
connected to the brushes. In this case the reactive power QR will be practically zero. Eqn. (3.329) then
shows that *R = 0, or *2 = *, a constraint on V2.

Examples
As an example consider a machine connected to an AC bus such that the phase voltage is V1 = 100 V, with
R1 = 0, R2 = 0@10, and Xs = 1@0. The units of R and X could be ohms or per-unit (in which case V1 would
be 1), but ohms will be used, and the discussion will be confined to per-phase values. The maximum
power available from this machine as a motor is obtained when s = R2/Xs = 0@1, and the phasor diagram
for this condition is shown in Fig. 3.72(a), with the rotor short-circuited at the slip-rings. The powers
and reactive powers are tabulated in kW:

PS PR Pmech Pgrid WCuR QS QR

5.00 0 4@50 5@00 0@50 5@00 0

j90E
In Fig. 3.72(b) the referred rotor voltage is V2N = 100 e , and the powers and reactive powers are
tabulated below.

PS PR Pmech Pgrid WCuR QS QR

10@00 0 9@00 10@00 1@00 0 1@00

The power output Pmech is doubled, while the current increases from 70@71 A to 100 A and the power
factor improves from 0@707 to unity. While the power has doubled, the rotor copper loss WCuR has also
doubled, so the efficiency remains roughly the same. No power is supplied by the frequency converter,
but only reactive power QR = 1@0 kVAr; as before, the negative sign means that reactive power is being
generated by the frequency converter and fed through the slip-rings to the rotor. The actual reactive
power of the frequency converter is QR = sQR = 0@1 × (10@0) = 1@0 kVAr.
j135E
In Fig. 3.72(c) the referred rotor voltage is V2N = 100 e , and the powers and reactive powers are
tabulated below. The rotor frequency converter is causing the machine to operate in what is, in effect,
an overload condition, with higher power but worse efficiency and power-factor.

PS PR Pmech Pgrid WCuR QS QR

12.07 0@50 10@86 12@57 1@71 5@00 1@207


Page 3.96 SPEED’s Electric Machines

V1

(a) N1
Z I1

E uN = E RN
I1

N1 = 0 V1 , I 1

N =*
2 2 *
(b) Z I1

E uN
V2N

E RN

V1
*2
(c) N1 Z I1

N *
2

*R
V2N I1

E uN

E RN

Fig. 3.72 Example phasor diagrams


(a) Short-circuited rotor
j90E
(b) V2N = 100 e
j135E
(c) V2N = 100 e

The power output Pmech is increased to 10@86 kW, while the current increases to 130@66 A. The power
factor falls to 0@924 but is still high, with reactive power absorbed at the stator terminals. The rotor
copper loss WCuR has increased to 1@71 kW, by a greater factor than the increase in Pmech, so the efficiency
will be lower. With PR = 0@50 kW, the frequency converter supplies 0@05 kW to the slip-rings. With QR
= 1@207 kVAr reactive power is again being generated by the frequency converter and fed through the
slip-rings to the rotor. Also note from eqn. (3.307) that the mechanical power is
Induction machines Page 3.97

1  s 1  0@1
Pmech ' [ WCuR PR ] ' × [ 1@707  0@500 ] ' 10@863 kW . (333)
s 0@1
We can also test the “synchronous machine” equations (1.326) and (1.327) for this condition. With
accurately calculated values * = 71@85E and EuN = 121@77 V we get

V1 EuN 100 × 121@77


PS '  sin * '  sin (71@85E) ' 11@57 kW (334)
Xs 1@0
and

V1 ( V1  EuN cos * ) 100 × ( 100  121@77 cos (71@85E) )


QS ' ' ' 6@21 kVAr . (335)
Xs 1@0

By comparison with the table at the bottom of p. 95, it can be seen that these formulas are quite
approximate, and it should be concluded that they only serve to show the analogy with the synchronous
machine. For accurate calculations eqns. (3.325) should always be used.
For the rotor, eqns. (3.329) give

V2N [ EuN cos *R  V2N ] 100 × (121@77 cos (63@15E)  100)


PRN ' ' '  5@00 kW and
RE 0@1 × (1/0@1  1)
(336)
V2N EuN sin *R 100 × 121@77 sin (63@15E)
QRN ' ' '  12@07 kVAr .
RE 0@1 × (1/0@1  1)

Unlike the stator equations (1.326) and (1.327), these equations are exact; they do not involve any
approximations in neglecting resistances.

Generating
Figs. 3. shows three generating conditions for the same machine, beginning (a) with s = 0@1 and a short-
circuited rotor.

V2N *2 PS PR Pmech Pgrid WCuR QS QR

(a) 0 — 5@00 0 5@50 5@00 0@50 5@0 0

(b) 50 180E 7@50 0@375 8@25 7@125 1@125 7@50 0@375

(c) 100 180E 10@00 1@00 11@00 9@00 2@00 10@0 1@0

Increasing V2N has the effect of increasing ERN and hence the power. Taking case (b) as typical, the
negative value of PR indicates power fed back from the PCC by the frequency converter to the slip-rings,
as depicted in Fig. 3.76. (Fig. 3.76 shows the magnitudes and directions, not the signs).
In all cases QS > 0, meaning that the generator is underexcited: this is consistent with Figs. 3.(b) and (c)
which are typical for an underexcited generator. The current I1 is also shown; this current has a
“source” convention at the stator terminals and is therefore the normal generator current; in all cases
it is leading V1. Note that QR > 0, which means that the frequency converter is also absorbing reactive
power from the slip-rings.
A final and most interesting condition is shown in Fig. 3.74 with unity power-factor, together with the
tabulated powers and reactive powers.
Page 3.98 SPEED’s Electric Machines

!II 1
!

(a) E uN = E RN

Z I1
N1

V1
I1

!I 1
(b) E RN
E uN

* * Z I1
2

V2N
N V1
2 *R

N1
I1

!I 1
E RN

(c) E uN N1

Z I1

V2N
V1

I1

Fig. 3.73 Example phasor diagrams — generating


(a) Short-circuited rotor
j180E
(b) V2N = 50 e = 50 V
j180E
(c) V2N = 100 e = 100 V
Induction machines Page 3.99

Fig. 3.74 Phasor diagram : generating, unity power-factor

V2N *2 PS PR Pmech Pgrid WCuR QS QR

100 90E 10@00 0 11@00 10@00 1@00 0 1@00


Generating; unity power-factor

PS = Pgrid, and there is no real power in the frequency-converter. Consequently Pmech is just the sum of
the stator output power PS and the rotor copper loss WCuR, 11@0 kW. The reactive power at the stator
terminals is QS = 0 (unity power-factor). The frequency converter supplies
Standby
The applied rotor voltage V2N in this example is in phase with the supply voltage V1, which is also the
reference phasor. It has the effect of forcing a reduction in Eu, causing a reduction in the torque
attributable to the rotor resistance. At the same time it introduces a positive value of PR which tends
to offset the depletion in the “resistance torque”. This graphically shows how the torque becomes less
attributable to the rotor copper loss and more attributable to the slip-ring power and the frequency
converter. Since the frequency converter returns the slip-ring power PR to the supply, the operation
is more efficient than it would be with only resistance in the rotor circuit. Of course there are additional
losses in the frequency converter itself, but these are typically small.

Strategy for design and simulation calculations


For a given machine and given supply voltage and frequency, n, V1 and Xs are fixed. For a given
operating speed, s is fixed. For a given power, * is determined by eqn. (3.329), and then V2 and the
currents and reactive powers are all determined according to the above equations. With all the
equations programmed, it is therefore possible to set up operating charts showing the currents and
reactive powers as functions of power and speed. At the same time the efficiency and temperature-rise
can be calculated, operating-point by operating-point. Such charts will show the capability of the
machine, as to whether it can work satisfactorily at a given operating point.
If this analysis is correct, it appears that the P-* relationship is fundamental to the operating strategy,
and that * is an important design and simulation variable.
The theory presented here is focussed on the machine, and it does not immediately relate to the design
of the controllers of the frequency-converter or of the prime mover. However, in a simulation of the
complete system V2 and * could very well be the independent control variables that set the operating
conditions at a given operating point. Alternatively the variables V2 and * could be exchanged for
another pair of variables (for example, V2 and I2, or I2 and its phase angle with V1). Such a mapping
between the variables V2 and * would not be difficult, since all the necessary equations are given here
or could be developed from the equations given here.
Page 3.100 SPEED’s Electric Machines

MOTOR MOTOR
UNDEREXCITED OVEREXCITED

V N V
N * * jX I
jX I

E
E
I

GENERATOR E GENERATOR
UNDEREXCITED OVEREXCITED
E
I

*
!I jX I *
jX I
V V
I N
N !I

Fig. 3.75 Phasor diagram showing motoring/generating in all four quadrants

Fig. 3.76 Power and reactive power in generating condition


3.16 Field-oriented control and space vectors Page 3.101

3.16 FIELD-ORIENTED CONTROL AND SPACE VECTORS


3.16.1 Introduction
Field-oriented control is the principle by which AC motor drives are designed to control the flux and the
torque-producing current independently, just as in a classical DC motor drive with separate field and
armature currents.
In induction machines the flux is generally associated with the magnetizing current which flows in the
magnetizing inductance. The torque-producing current is the current in the rotor branch of the
equivalent circuit, and this can be said to flow through the total leakage inductance. The leakage
inductance is usually much smaller than the magnetizing inductance, and this gives rise to the idea that
the rotor current can be changed much more rapidly than the magnetizing current. Obviously that is
a desirable objective when rapid torque response is needed, as is the case in servo drives and many other
high-performance variable speed drives.
In the classical separately excited DC machine the armature and field currents flow in separate circuits,
but in the induction machine there is only one current at the terminals. Regarded as a vector quantity,
it has two components, which are normally seen as the magnitude and phase (or real and imaginary
components) when the phasor diagram is used, or in steady-state AC calculations. But AC phasors are
by definition steady-state quantities, so they are not suitable for dealing with rapid changes that might
occur in a fraction of a cycle.
Enter the space vector, which is an expression of the instantaneous current in all three phases combined.
The space vector is the basic measure of current in the theory of field-oriented control. As it is a
complex number, it can be resolved into components along particular axes, for example, along the axis
of the flux (the direct axis) and a quadrature-axis orthogonal to the flux. This resolution is the key to
the separation of flux-producing and torque-producing components of the current. Once these separate
components are identified as control variables, it becomes possible to apply the classical principles of
DC motor control in the outer control loops for torque, velocity, and position. Of course, the current
components are control variables in the digital controller, so they are in a sense fictitious: the total
current is of course the combination of these components.
With this background, the section begins with a description of the classical principles of DC motor
control. It then proceeds to define the space vector. The equations of the induction motor are written
in terms of space-vectors of current and voltage. This sets the scene for the application of the field-
oriented control principle, which is described for a particular embodiment due to Leonhard [33].
Innumerable other embodiments are used in the industry, but the one described here is both
fundamental and historic.

3.16.2 Classical DC motor control


We have seen that the speed of an induction motor can be controlled by varying the frequency, and that
the voltage must also be controlled to maintain constant Volts/Hz. Adjustable-speed drives based on
these principles are adequate for simple speed adjustment, but not adequate for motion control systems
that require (1) rapid dynamic response; (2) operation over a wide speed range particularly including
very low speed and even zero speed; and (3) the ability to operate in all four quadrants of the
torque/speed diagram with rapid transition between quadrants (i.e. from motoring to braking or from
forward to reverse).
Until the 1980's these “highly dynamic” applications were served by DC machines with armature current
control. In the DC motor the electromagnetic torque is given by

Te ' K M Ia (337)
where M is the flux. If the flux is constant, the torque is proportional to the current. This gives rise to
the classical feedback control structure shown in Fig. 77. The response to various operating scenarios
can be analysed using techniques based on the Laplace Transform—for example, the root-locus method
for determining the stability with different values of gain.
Page 3.102 SPEED’s Electric Machines

Fig. 3.77 Block diagram of classical DC motor control

The proportionality between electromagnetic torque and current is based on the decoupling (i.e.,
independent control) of the flux and current. The flux and current are often said to be “at right angles”
or orthogonal to one another: more precisely, the axis of the armature MMF or ampere-conductor
distribution is orthogonal to the axis of the field. The field axis normally defines the direct axis or d-
axis, so the armature current is “in” the quadrature axis or q-axis. In the DC motor, this orthogonality
results from the physical action of the commutator and brushes.
Also note that the armature inductance is generally much lower than the field inductance, so any change
in torque can be made more rapidly by changing the armature current, not the field current. This echoes
the earlier observation about the magnetizing and leakage inductance of the induction motor.
The DC motor also has a back-EMF given by

E ' K M Tm (338)
where Tm is the speed in rad/s. In practice this means that the supply voltage must vary in proportion
to speed (apart from a slight compensation required for the resistance volt-drop RI a ). With a limited
supply voltage, higher speeds can be achieved by field weakening, i.e., by reducing the flux M at high
speed so that E remains less than the maximum available supply voltage. Of course this also reduces the
torque/ampere, so for a given maximum current the torque decreases with increasing speed. But the
product of torque and speed can be maintained constant over a wide speed range, giving “constant power
operation”.
The term “field weakening” indicates that the flux is reduced by reducing the field current. But in
induction motors, and also in permanent-magnet DC or brushless DC motors, the field current does not
exist as such, so the term “flux-weakening” has come to be preferred in these cases. Evidently what must
be weakened in the induction motor is the flux-producing component of the current, but in permanent-
magnet motors the flux is weakened by introducing a demagnetizing component of armature current.
(In commutator motors, that can be done by brush-shifting, while in brushless motors it can be done by
adjusting the phase of the armature current, which is an AC current).
For many years the DC motor was the preferred technology for servo-drives and other high-performance
variable-speed drives. Because of the high cost of the DC motor, and other drawbacks associated with
the brushes and commutator, it was recognized for a long time that the AC induction motor would be
less expensive and perhaps smaller and more reliable, but it was not until about 1970 that a practical
control system was devised for the AC motor that could compete with the DC drive in dynamics and
servo performance. Since that time, the AC drive has developed so rapidly that it is now the dominant
technology in adjustable-speed drives.
Why did it take so long for the AC induction motor to replace the DC commutator motor in adjustable-
speed drives? There are three basic reasons:
3.16 Field-oriented control and space vectors Page 3.103

1. Only in the 1970's did power semiconductor devices become available with sufficient voltage and
current capability, switching speed, and low enough cost to make the variable-frequency
inverter commercially viable.
2. Only around 1970 was the control theory of the AC induction motor drive developed in a form
in which the flux and current were decoupled and oriented orthogonally, permitting the AC
motor to be used in classical servo control applications. The key development was the idea of
field-oriented control.
3. Only from the mid-1970's was it possible to implement the required control algorithms on digital
microelectronic integrated circuits. Subsequently, microprocessors, digital signal processors
and microcontrollers have been used not only to implement the basic control algorithms but to
enhance the overall dynamic performance to an extraordinary degree.
To understand how the AC machine model can be "transformed" into an equivalent DC machine, with
decoupled control of flux and current, we need to know about space vectors.

3.16.3 Definition of space vectors


Space vectors are complex numbers, like phasors, but they have two major differences :
1. Space vectors represent the spatial orientation of ampere-conductor and flux distributions within
the machine;
2. Unlike phasors, space vectors are defined in terms of instantaneous quantities, so they can vary
rapidly in time and are not restricted to steady-state operation.
Let winding "w" be sine-distributed with its axis at an angle 2 measured from the x-axis. Then i w is the
complex number whose magnitude is equal to the instantaneous current i w and whose argument is the
angle 2.
j0
In a 3-phase motor the axis of phase a is the x-axis: i.e., 2 = 0, so i a = i a e . If the axis of phase b is at 120E,
j2 B /3 j2 B /3 ! j2 B /3
the space vector i b = i b e = a i b , where a = e . Similarly i c = i c e = a2 i c if the axis of phase
c is at !120E.
The stator current space vector is defined as

2
is ¤ (i ¢ ib ¢ ic) (339)
3 a
j2B/3 !j2B/3
Since e = !1/2 + j /3/2 and e = !1/2 ! j /3/2, this can be written

ib £ ic
i s ¤ ia ¢ j . (340)
3

The b factor ensures that the magnitude of i s is equal to the peak value of the phase current when the
motor is operating with balanced 3-phase currents.
Fig. 78 shows a geometric representation of eqn. (339), at a particular instant when i a = 0@327, i b = 0@655,
and i c = !0@982. These values occur when T t = 70@893E if the currents are in a steady state given by

ia ¤ ipk cos T t
ib ¤ ipk cos ( T t £ 120E ) (341)
ic ¤ ipk cos ( T t 120E )

with i pk = 1. The actual phase currents can be recovered from the space vector i s by the equations

ia ¤ Re ( i s ) ; ib ¤ Re ( a 2 i s ) ; ic ¤ Re ( a i s ) . (342)

This is shown by the projections in Fig. 79.


Page 3.104 SPEED’s Electric Machines

f
120E c
d
is
b ic

ib

0
0 a
ia

!120E
SPV_Construction.wpg

Fig. 3.78 Space vector construction

Fig. 3.79 Space vector resolution into a, b, c components


3.16 Field-oriented control and space vectors Page 3.105

Sometimes eqn. (339) is called a 362 transformation, from “stator coordinates” (i a ,i b ,i c ) to “2-axis”
coordinates, the two axes being the real and imaginary axes fixed to the stator such that the real axis
is the axis of phase a. These two axes are also called d and q axes, or sometimes ds and qs axes, the
additional s being added to associate the reference frame with the stator.
Eqn. (342) is the reverse or 263 transformation. These transformations are easy to implement in
software if digital signals representing the currents are available. They can equally be implemented
with analogue circuits, since the only operations required are multiplication and addition.
Note in Fig. 1 that the orientations of the a, b, and c-axes are opposite in sequence to those which we
normally see in phasor diagrams. This is because the a, b, and c-axes are effectively coordinate axes
fixed in space, whereas a-, b-, and c-phasors are rotating complex numbers.
The transformations (339) and (342) can be applied to sinusoidal MMF distributions, to instantaneous
phase flux-linkages and phase voltages in just the same way as they are applied to the currents. We will
see many examples of this.

3.16.4 Projecting a space vector on another axis or frame of reference


Suppose the space vector i s is defined in the stator reference frame. This frame is stationary, and its
reference axis is the x-axis, 2 = 0. The rotor reference frame is fixed to the rotor, and rotates at the rotor
speed T m elec rad/s. Suppose that at one instant the reference axis of the rotor reference frame is at an
angle $ relative to the reference axis of the stator, as in Fig. Consider a space vector i r N defined in the
rotor reference frame. The prime (N) means “defined in the rotor reference frame”. Viewed from the
stator reference frame, this space vector has the same magnitudes but its angle appears to be offset by
j$
$. This is written i r = e i r N . The space vector i r is a space vector defined in the stator reference frame,
that produces the same effect as the original space vector i r N defined in the rotating reference frame of
the rotor. Physically, i r and i r N are one and the same entity.
If the stator phases are fed with balanced sinewave currents at angular frequency T s , the corresponding
Tt
stator space vector rotates at synchronous speed T s elec rad/s, and is represented as i s e j , which
follows a circular locus. This is a compact form of expression for the rotating ampere-conductor
distribution in the sinusoidal steady-state. In general, however, the stator currents may not have
sinusoidal waveforms and the space vector i s follows a non-circular locus as the currents change.
Regardless of the waveforms of the individual phase currents, however, the resulting ampere-conductor
distribution is always implicitly sine-distributed in space. It can rotate at a non-uniform speed, go
forwards or backwards, change rapidly in amplitude—but it is always sine-distributed. This is
tantamount to the assumption that the windings are sine-distributed; in other words, space-harmonics
of the winding distribution are ignored. The only restriction in the definition of the space vectors used
here is that the stator has a 3-wire connection so that i a + i b + i c = 0.
Page 3.106 SPEED’s Electric Machines

Fig. 3.81 Induction motor equivalent circuit in terms of space vectors

3.16.5 The flux-linkage equations (coupling equations) between rotor and stator
In terms of the corresponding space vectors, the stator and rotor flux-linkages are given by

8s ¤ (LF s LM) i s LM i r
(343)
8 r ¤ LM i s ( LF r LM ) i r

where L M is the magnetizing (mutual) inductance, and L F s and L F r are the stator and rotor leakage
inductances respectively. Note that i r is written without the prime N — eqn. (343) is written in the stator
reference frame. The flux-linkages and currents are shown in the equivalent circuit, Fig. 3.

3.16.6 Stator and rotor voltage equations


In terms of the corresponding space vectors, the stator voltage equation is

d8 s
v s ' Rs i s ¢ (344)
dt

The rotor voltage equation is written first of all in the rotating reference frame:

d8 rN
v rN ' Rr i rN ¢ (345)
dt

We need to transform this equation into the stationary reference frame so that we can incorporate it in
the (stationary) equivalent circuit of Fig. 81:

v r ¤ v rN e j $
¤ Rr i rN e j $ d8 rN / dt × e j $
(346)
¤ Rr i rN e j $ d (8 r e £ j $ ) / dt × e j $
¤ Rr i r ¢ d8 r / dt £ j T 8 r

The term !jT8 r is a speed EMF. It is given the symbol e 0 , and it appears in the equivalent circuit as
shown in Fig. 81. T comes from d$/dt, and it is the instantaneous angular velocity of the rotor measured
in elec. rad/sec. Under steady-state conditions at constant speed, T = 2B f — but we should avoid the
temptation to think of steady-state sinusoidal conditions, because vector control is all about dynamics,
sudden changes, and rapid response.
The power converted from electrical to mechanical form is given by

3 3
Pm ¤ Re e0 i r( ¤ Re £ j T 8 r i r( . (347)
2 2
3.16 Field-oriented control and space vectors Page 3.107

The 3/2 factor in eqn. (347) derives from the fact that there are 3 phases, and the space vectors have
magnitudes corresponding to the peaks of their respective space distributions. When the (spatial)
average is taken of the product of two sine distributions, the result is multiplied by 1/2. (This is
analogous to the use of 1//2 for the value of phasors in conventional AC theory).
Eqn. (347) suggests the idea of field-oriented control. Suppose that the two space vectors 8 r and i r can
be oriented at such angles that the term ! j T8 r i r * is always real. This requires that 8 r and i r * are at
right angles to each other at all times. Then if the rotor angular velocity in actual rad/s is T/p, where
p is the number of pole-pairs, the electromagnetic torque is

3
Te ¤ p 8r,pk ir,pk (348)
2

where 8 r,pk is the peak value of the rotor flux-linkage per phase, and i r,pk is the peak value of the rotor
current per phase (both referred to the stator winding). This equivalent to eqn. (337) for the DC motor.

3.16.7 Rotor-flux-oriented vector control


While eqns. (347) and (348) suggest the possibility of “field orientation” principle, they give no clue as
to how it might be implemented. At first sight it appears impossible to implement, because the space
vectors 8 r and i r are rotor quantities that are inaccessible for measurement at the stator terminals. In
control engineering parlance, they appear to be “unobservable” and maybe also “uncontrollable”. While
eqns. (347) and (348) could have been derived long before the field orientation principle was invented,
the next steps leading to its invention most definitely were not. In Prof. Leonhard's words, the next steps
were developed by a mixture of “intuition and mathematical reasoning”.
The rotor flux-linkage space vector 8 r is given by eqn. (343) on p. 106:

8 r ' LMi s ¢ ( LF r ¢ LM ) i r
(349)
¤ LM [ i s ¢ ( 1 ¢ Fr ) i r ]

where F r = L F r /L M . The term in square brackets [] is written as a single current,

i mr ¤ i s ¢ ( 1 ¢ Fr ) i r (350)
so that equation (1) becomes

8 r ¤ LM i mr . (351)

i mr looks very much like a magnetizing current that controls the rotor flux-linkage 8 r , which we
previously thought “unobservable”. It is a key idea that i mr controls the rotor flux-linkage and not the
stator or the mutual flux-linkage. It is also time to start thinking of i mr as a component of the stator
current space vector i s , in much the same way as we think of the real and imaginary components of the
phasor current I s in conventional steady-state theory. Remember, though, that i mr is an instantaneous
value: unlike a phasor, it does not depend on the assumption of a sinusoidal current waveform.
We now return to the rotor voltage. If we write the differential operator d/dt as p, then together with
eqn. (351), eqn. (346) becomes

v r ¤ Rr i r ¢ ( p ! j T ) 8 r ¤ Rr i r ¢ ( p ! j T ) LM i mr . (352)

We can use equations (2-3) to eliminate the “unobservable” terms 8 r and i r . If we back-substitute for
i r from eqn. (343), and at the same time take advantage of the fact that the rotor circuit of an induction
motor is shorted so that vr = 0, we get
LM (1 ¢ Fr )
i mr ¤ i s ¢ ( p ! j T ) i mr . (353)
Rr

If we now recognize the rotor time constant


Page 3.108 SPEED’s Electric Machines

LLR ¢ LM ( 1 Fr ) LM
Tr ' ¤ (354)
Rr Rr
then eqn. (353) becomes

[ 1 Tr p ! j T Tr ) ] i mr ' i s . (355)

Eqn. (355) shows the possibility of controlling the rotor flux-linkage space vector 8 r , because the
“magnetizing current” i mr appears related to the stator current i s by the operator in square brackets.
This operator includes three terms which are all capable of implementation either in hardware or
software. The “1” term is just a gain. The “p” term is a differentiator, and if the control block diagram
is suitably arranged, its effect can be implemented by an integrator, as we will see. The “!jT” term
represents an angular rotation of !90 elec. deg.
The next step in the mathematical development is the one that achieves the “decoupling” of the flux-
linkage and the current referred to earlier. This step, together with the subsequent construction of the
control block diagram, is the “stroke of genius” which, again in the words of Prof. Leonhard, is so simple
that it is surprising that it took so long to find it.
What is done is to take real and imaginary parts of eqn. (355), but first the magnetizing current space
vector i mr is written

i mr ¤ i mr e j D , (356)

where D is the angle of the rotor flux-linkage 8 r in the stator reference frame. It is clear from eqn. (351)
that i mr is aligned with 8 r , since L M is real; so D is the “angle of orientation to the rotor flux”.
To take real and imaginary parts we first substitute eqn. (356) in eqn. (355), then expand it (remembering
that p is the operator d/dt and D is a function of time):

[ 1 Tr p ! jTTr ] imr e j D ' i s . (357)

( 1 ! j T Tr ) imr e j D ¢ Tr { p imr e j D ¢ j p D imr e j D } ' i s . (358)

( 1 ! j T Tr ) imr ¢ Tr { p imr ¢ j p D imr } ' i s e !j D . (359)


Then

( 1 Tr p ) imr ' Re ( i s e !j D ) ' isd (360)


and
dD Im ( i s e !j D ) isq
pD ¤ ¤ T ¤ T . (361)
dt imr Tr imr Tr

Eqns. (360) and (361) form the basis of the control block diagram, Fig. 82. The currents isd and isq are the
instantaneous d- and q- axis components of the stator current in a reference frame that is synchronized
with the rotor flux-linkage space vector.29 See Fig. 2. They correspond to the field and armature currents
of the DC machine.

3.16.8 Decoupled torque equation


What is needed finally is the torque equation in terms of the independently controlled currents i sd and
i sq . This can be derived by straightforward manipulation of eqn. (347), using eqns. (350,351) and (360,
361): thus
3p
Te ¤ Re !j T ( LM i mr ) i r( . (362)
2T
29
They are distinct from the Park components which are synchronized with the physical d-axis of the rotor. In an induction motor
there is no unique physical d-axis; the flux is not synchronized with the rotor; and the rotor flux is due to the sum of stator and
rotor MMF's. All these factors are incorporated in the field-oriented control.
3.16 Field-oriented control and space vectors Page 3.109

Now

1
Re [ !j T { LM i mr } i r( ] ¤ ! Im [ LMi mr ( i mr ! i s )( / ( 1 ¢ Fr ) ]
T
(363)
¤ Im [ LMi mr i s( / ( 1 ¢ Fr ) ]

¤ Im [ LMi mr ( isd ¢ j isq )( / ( 1 ¢ Fr ) ] .

Therefore

3 p LM
Te ¤ ! imr isq . (364)
2 1 Fr

The negative sign in this equation arises because of the direction of rotor current in Fig. 81, which is
chosen as though a separate rotor voltage were impressed at the short-circuited rotor terminals: this
choice is made to make all the terms in eqn. (343) positive. With this sign convention, either i mr or i sq
will be negative when the torque is positive.

3.16.9 Measurement requirements


The only measurements (feedback) needed from the motor are the rotor speed T and the phase currents.
In fact only two phase currents need be measured if i a + i b + i c = 0, as in a 3-wire connection. The
rotor-flux-oriented control scheme extracts separate values for the rotor flux-linkage and the rotor
current, without directly measuring them. To do this, the control model must be programmed with the
values of T r , p (pole-pairs), and L M /(1+F r ). The value of stator resistance is not required. The rotor time
constant Tr is subject to variation because of the temperature effect on the rotor resistance. Although
variations of 50% are quite common, the variation is slow and can be compensated by adaptive
modifications performed automatically by the controller. The inductance L M /(1+F r ) is also subject to
change through saturation, but only by a few percent. It is possible to compensate for this by additional
software in the controller, provided that the variation is known as a function of, say, total stator current
and slip.
The speed is normally measured by a 1000-line encoder or resolver, permitting very low speed operation
with excellent torque control virtually down to zero speed. The rotor-flux-oriented scheme does not
require any measurement of the terminal voltage of the motor, which is distorted with PWM harmonics.
There are many variants of field-oriented controller, some of which use orientation to reference frames
other than that of the rotor flux. What is described here is the basic rotor flux-oriented scheme. It is
known as an “indirect” vector control because there is no direct measurement of the rotor flux. In the
early days of vector control, attempts were made to measure the rotor flux by fitting Hall sensors or
search coils in the motor airgap. However, the signals from these devices were subject to severe
distortion and interference and required expensive special modifications to the motors.

3.16.10 Performance example


Fig. 83 shows oscillograms typical of the response of a field-oriented controlled induction motor. The
example is taken from the Control Techniques Drives and Servos Yearbook, 1990, for a 7@5 kW motor with
a Control Techniques VECTOR drive. Although the control system is not the one described in Fig. 82,
it is a good example of vector control or field-oriented control, in which the instantaneous current and
torque are controlled to achieve a very fast dynamic response.
Page 3.110 SPEED’s Electric Machines

Fig. 3.82 Block diagram of rotor flux control

20 ms

200 rpm/div

CURRENT

SPEED 500 rpm

Fig. 3.83 Speed reversal at 500 rpm


Courtesy Control Techniques, 1990
References Page 3.111

REFERENCES
1. SPEED’s Electric Motors, the theory text that is used with the SPEED training courses.
2. Say MG, The performance and design of alternating current machines, Pitman, London, Second
Edition, 1948
3. Alger PL, Induction machines, their behavior and uses, Gordon & Breach Science Publishers, New
York, London, Paris, Second Edition, 1970. [Original edition, copyrighted 1965 under the title The
nature of induction machines]. Library of Congress Catalog Card No. 64-18799
4. Heller B and Hamata V, Harmonic field effects in induction machines, Elsevier, Amsterdam,
Oxford, New York, 1977 ISBN0-444-99856-X
5. Veinott CG, Theory and design of small induction motors, McGraw-Hill, New York, 1959
6. Kostenko M and Piotrovsky L, Electrical machines, (two volumes), MIR Publishers, Moscow, 3rd
edition, 1974
7. Fitzgerald AE and Kingsley C Jr., Electric machinery, McGraw-Hill, Second Edition, 1961
8. Richter R., Elektrische Maschinen, Springer, 1954
9. Schuisky W, Berechnung Elektrischer Maschinen, Springer, 1960
10. Vas P, Electrical machines and drives: a space-vector theory approach, Clarendon Press, Oxford,
1992 ISBN 0-19-859378-3
11. Wood WS, Flynn F and Shanmugasundaram A, Transient torques in induction motors, due to
switching of the supply, Proc. IEE, Vol. 112, No. 7, July 1965, pp. 1348-1354
12. Engelmann RH and Middendorf WH [Eds], Handbook of electric motors, Marcel Dekker, New York,
Basel, Hong Kong, 1995, ISBN 0-8247-8915-6
13. Morrill, WJ, The revolving-field theory of the capacitor motor, Trans AIEE, April 1929, pp. 614!632.
14. Levi E, Polyphase motors: a direct approach to their design, John Wiley & Sons Inc., New York,
1984 ISBN 0-471-89866-X
15. Hendershot JR and Miller TJE, Design of brushless permanent-magnet motors, Magna Physics
Publications/Oxford University Press, 1994 ISBN0-19-859389-9
16. Veinott CG and Martin JE, Fractional and subfractional horsepower electric motors, Fourth
edition, McGraw-Hill Book Company 1992, ISBN 0-07-067393-4
17. Boldea I, Deep bar effect for slots of any shape, hand-written notes, SPEED Laboratory, 1995.
18. Ionel DM, Cistelecan MV, Miller TJE and McGilp MI, A new analytical method for the computation
of airgap reactances in 3-phase induction motors, IEEE Industry Applications Society, Annual
Meeting, St. Louis 12-15 October 1998, pp. 65!72.
19. Miller TJE, Gliemann JH, Rasmussen CB and Ionel DM, Analysis of a tapped-winding capacitor
motor, ICEM ‘98, Istanbul, 2-4 September 1998, Vol. I, pp. 581!585.
20. Kopilov IP, Goriainov FA, Klokov BK, Design of Electrical Machines (in Russian: Proektirovanie
elektriceskih masin), Moscow, Energhia, 1980.
21. Puchstein AF and Lloyd TC, The cross-field theory of the capacitor motor, Trans. AIEE, Vol. 60,
February 1941, pp. 58!63.
22. Puchstein AF and Lloyd TC, Capacitor motors with windings not in quadrature, Trans. AIEE,
November 1935, pp. 1235!1239.
23. Kingsley C and Lyon WV, Analysis of unsymmetrical machines, Trans. AIEE, May 1936, pp.
471!476.
24. Trickey PH, Performance calculations on capacitor motors; the revolving field theory, Trans. AIEE,
Vol. 60, February 1941, pp. 73!76.
25. Trickey PH, Capacitor motor performance calculations by the cross-field theory, Trans.AIEE, Vol.
76, February 1957, pp. 1547!1553.
Page 3.112 SPEED’s Electric Machines

26. Suhr FW, Symmetrical components as applied to the single-phase induction motor, Trans.AIEE, Vol.
64, September 1945, pp. 651!655.
27. McFarland TC, Current loci for the capacitor motor, Trans. AIEE, Vol. 61, March 1942, pp. 152!155.
28. Bewley LV, Alternating current machinery, Macmillan, N.Y. 1949.
29. Norman HM, Induction motor locked saturation curves, Trans. AIEE, Vol. 53, 1934, pp. 536!541.
30. Kylander, G, Thermal modelling of small cage induction motors, Technical report No. 265, 1995,
PhD dissertation, Chalmers University of Technology.
31. Dorrell DG, Miller TJE, Rasmussen CB [2001] Interbar Currents in Induction Machines, IEEE
Industry Applications Society, IAS 2001, Chicago, USA, 30 Sep – 5 Oct. 2001.
32. Miller TJE, Boldea I, Dorrell DG, Rasmussen CB [2000] Leakage Reactance Saturation in
Induction Motors, ICEM 2000, Helsinki, Finland, 28-30 August 2000 pp. 203-207.
33. Leonhard W, Control of Electrical Drives, Springer-Verlag, Berlin, 1985
Index Page 3.113

Index
2-phase winding, 1 Cross-field method, 50, 62
3-phase windings, 2 used with T-connection, 68
AC drives Current density, 26, 33
development of, 102, 103 Current-vs-speed curve
Airgap flux distribution, 29 minimum current, 8
Airgap power, 4, 5, 66, 67, 90 DC motor control, 101
Airgap torque, 5 block diagram, 102
Alger, 10, 11, 26, 39, 40, 111 Decoupling
Ampere-conductors per slot, 74 of flux and current, 102, 108, 109
Auxiliary winding, 50 Deep-bar effect, 33
Back turns, 24 Deep-bar, 9, 16, 28, 30, 33, 57
Balancing theory, 51, 54 Differential, 26, 30, 39, 40, 45, 107
of split-phase motor, 51 Differential leakage, 39
Belt leakage Distribution factor, 18
see Differential leakage, 39 Double-cage, 9, 28
Bifilar winding, 24 Efficiency, 3-5, 9, 95, 96, 99
Breakdown torque, 6, 9 Eigenvalue analysis, 42
C, 56 Electromagnetic torque, 4, 5, 45, 58, 67, 77, 101,
102, 107
Cage-rotor, 9, 85, 88-90
Equivalent circuit, 3-5, 8-14, 28-30, 34, 37, 38, 48,
Capacitor motor, 50, 57, 58, 60, 62, 63, 65, 68, 111,
50, 57, 61, 65, 81, 85, 88, 89, 91, 92,
112
101, 106
balance theory, 51
in terms of space vectors, 106
Capacitor reactance, 52, 54, 55
single-phase, 50, 57, 58, 60, 65
Capacitor voltage, 52, 54-56
Erb, 10, 37
Carter factor, 30
Field weakening, 102
Chording angle, 18
Field-oriented control, 101
Closed rotor slots, 10, 37, 38
Finite-element, 30, 33, 34, 36, 37, 41
Closed slot, 10
Flux weakening, 102
rotor bridges, 37
Forward and backward components, 50, 51
saturation of leakage reactance, 38
Forward and backward currents, 58
Closed slots, 30, 32, 36, 39
Forward and backward revolving-field theory,
Conductivity, 28 57
Connection matrix, 77 Fourier analysis
Constant Volts/Hz, 9, 101 of MMF distribution, 18, 23
Control Techniques Yearbook, 109 Fractional-slot, 18, 20
Core loss, 3, 10, 12, 13, 64, 66 Harmonic leakage
Core losses, 57, 66, 90 see Differential leakage, 39
Page 3.114 SPEED’s Electric Machines

Harmonic winding factors, 18 Saturation, 10, 12, 16, 28-30, 32, 34-38, 57, 90, 109,
112
Harmonics, 10, 18, 20-22, 25, 26, 29, 39, 40, 50, 61,
81, 84, 105, 109 effect on no-load current, 29
Inrush current, 34 of slot leakage reactance, 38
Interbar currents, 61, 67, 112 Saturation factor
Laplace transform, 101 for zig-zag reactance, 34
Leakage reactance, 7-10, 12, 13, 17, 26, 34, 35, 37, Shaft torque, 4
40, 41, 64, 88, 91, 112
Sine-distributed windings, 1
Location of ampere-conductors, 41
Skew, 18, 24, 30, 39, 40
Locked-rotor current, 7
Skew factor, 18
Locked-rotor torque, 7
Slip, 2-6, 8-10, 16, 28, 30, 34, 41, 42, 45, 48, 49, 62,
Magnetizing current, 29 77, 82, 85-90, 93-97, 99, 109
Magnetizing reactance, 3, 10, 12, 13, 29, 30, 57, 64 Slot numbers, 26, 27
formula for, 30 ratio, 26
Main winding, 50 Slot permeance, 10, 32, 36, 37
Microcontroller, 103 Slot-openings
Minimum current, 8 effect on MMF harmonics, 24
MMF harmonics, 22 effect on surface loss, 69
No-load current, 30 Slot-MMF harmonics, 10
No-load V/I curve, 29 Space vector, 101, 103-105
Non-simultaneous switching, 46 Speed control, 9, 86
Norman H M, 34 Split-phase motor, 50
Park, 42, 108 Split-phase, 24, 50, 51, 61, 66, 84
Permeance harmonics, 29 Split-phase motors, 50
Phase-belt harmonics, 10, 39 Start winding, 50
Phasor diagram, 4, 8, 10, 41, 51, 52, 68, 91-93, 95, Starting current, 34
99-101
Starting torque, 9
of split-phase motor, 51
Steps
Pitch factor, 18
in ampere-conductor distribution, 69
Pole amplitude modulation, 9
Stray loss, 10, 28, 61, 69
Power, 3-5, 16, 27, 29, 43, 45, 51-56, 61, 66, 67, 72,
Surface loss, 69
81, 85-88, 90-97, 99, 100, 102, 103,
106 Symmetrical components, 50, 60
Power factor, 5, 29, 51-53, 61, 95, 96 Synchronous speed, 1-5, 8, 9, 11, 26, 41, 45, 58, 67,
76, 77, 87, 105
Primitive impedance matrix, 77
Synchronous watts, 58, 67
Proximity effect, 28
T-connection, 68
Root locus method, 101
Tapped windings, 59
Rotating magnetic field, 1
The, 72, 73
Rotor flux-oriented vector control, 107
Tooth-tip leakage, 35
Index Page 3.115

Torque, 2, 4-7, 9, 14, 16, 34, 41, 42, 45-48, 58, 67, 77, Winding factors, 18-21
81, 82, 99, 101, 102, 107-109
Wound-rotor machine, 85
Torque/slip curve, 6, 9
Wound-rotor, 9, 39, 85, 90, 92
Torque/speed characteristic, 5, 9
Xm, 29
Transformation
Zigzag leakage
from 3-phase to space-vector, 105
see Differential leakage, 39
Transients, 42, 45
Zig-zag, 26, 34-37
Triplen harmonics, 18
Vector control
see Field-oriented control, 101
4. Switched reluctance machines

4.1 Definitions . . . . . . . . . . . . . . . . . . . . . . . . . . . . . . . . . . . . . . . . . . . . . . . . . . . . . . . . . . . . . . . . . 3
4.2 Linear analysis of the voltage equation and torque production . . . . . . . . . . . . . . . . . . . 5
4.3 Nonlinear analysis of torque production . . . . . . . . . . . . . . . . . . . . . . . . . . . . . . . . . . . . . . . 9
4.4 Continuous torque production . . . . . . . . . . . . . . . . . . . . . . . . . . . . . . . . . . . . . . . . . . . . . . 12
4.5 Energy conversion analysis of the saturated machine . . . . . . . . . . . . . . . . . . . . . . . . . . 15
4.6 Obtaining the magnetization curves . . . . . . . . . . . . . . . . . . . . . . . . . . . . . . . . . . . . . . . . . 18
4.7 Solution of the machine equations . . . . . . . . . . . . . . . . . . . . . . . . . . . . . . . . . . . . . . . . . . . 20
4.8 Control principles . . . . . . . . . . . . . . . . . . . . . . . . . . . . . . . . . . . . . . . . . . . . . . . . . . . . . . . . . 21
4.9 Variation of current waveform with torque and speed . . . . . . . . . . . . . . . . . . . . . . . . . 23
4.10 Current regulation . . . . . . . . . . . . . . . . . . . . . . . . . . . . . . . . . . . . . . . . . . . . . . . . . . . . . . . . . 28
4.11 Mathematical description of chopping . . . . . . . . . . . . . . . . . . . . . . . . . . . . . . . . . . . . . . . 34
4.12 Regulation algorithms . . . . . . . . . . . . . . . . . . . . . . . . . . . . . . . . . . . . . . . . . . . . . . . . . . . . . 35
4.13 Optimisation of the control variables . . . . . . . . . . . . . . . . . . . . . . . . . . . . . . . . . . . . . . . . 37
4.14 Magnetic gear ratio . . . . . . . . . . . . . . . . . . . . . . . . . . . . . . . . . . . . . . . . . . . . . . . . . . . . . . . . 38
4.15 References . . . . . . . . . . . . . . . . . . . . . . . . . . . . . . . . . . . . . . . . . . . . . . . . . . . . . . . . . . . . . . . . 41
SEM 4 — ii
4. SWITCHED RELUCTANCE MACHINES

4.1 DEFINITIONS
A reluctance machine is an electric machine in which torque is
produced by the tendency of its moveable part to move to a position
where the inductance of the excited winding is maximized. As we
have seen in chapter 1, this definition covers both switched and
synchronous reluctance machines. The switched reluctance motor
has salient poles on both the rotor and the stator, and operates like
a controlled stepper motor.
A primitive example is shown in Fig. 4.1. This machine is denoted
"2/2" because it has two stator poles and two rotor poles. The two
coils wound on opposite stator poles are excited simultaneously, and
generate magnetic flux as shown. There is only one phase. In the
position shown, the resulting torque tends to rotate the rotor in the
counterclockwise direction towards the aligned position, Fig. 4.2.
This machine can produce torque only over a limited arc of
Fig. 4.1 Simple reluctance machine
rotation, roughly corresponding to the stator pole arc $s. However, with one phase and two poles
it is the basic model on which the theory of torque production is on both the stator and rotor.
based, so we will analyze it first, and then consider methods of
starting and the extension to multiple poles and phases.

Fig. 4.2 Aligned position Fig. 4.3 Unaligned position

Aligned and unaligned positions


For the primitive reluctance machine in Figs. 4.1!4.3, the aligned and unaligned positions are
characterized by the properties summarized in Table 4.1.

Aligned Unaligned
2 = 0, 180E 2 = ±90E
Maximum inductance Minimum inductance
Magnetic circuit liable to saturate Magnetic circuit unlikely to saturate
Zero torque : stable equilibrium Zero torque : unstable equilibrium
TABLE 4.1
PROPERTIES OF THE ALIGNED AND UNALIGNED POSITIONS

Variation of inductance with rotor position


In the simple machine shown in Figs. 4.1!4.3 the coil inductance L varies with rotor position 2 as shown
in Fig. 4.4. Positive rotation is in the counterclockwise direction. Assume that the coil carries a
constant current. Positive motoring torque is produced only while the inductance is increasing as the
rotor approaches the aligned position; that is, between positions J and A. At J, the leading edge of the
rotor pole is aligned with the first edge of a stator pole; at A, the rotor and stator poles are fully aligned.
Thus J defines the start of overlap, A the maximum overlap, and K the end of overlap.
Page 4.4 SPEED’s Electric Motors

Fig. 4.4 Variation of inductance and torque with rotor position; coil current is constant. The
small icons show the relative positions of the rotor and stator poles, with the rotor
moving to the right. A = aligned position; U = unaligned position; J = start of overlap;
K = end of overlap.

The torque changes direction at the aligned position. If the rotor continues past A, the attractive force
between the poles produces a retarding (braking) torque. If the machine rotates with constant current
in the coil, the negative and positive torque impulses cancel, and therefore the average torque over a
complete cycle is zero. To eliminate the negative torque impulses, the current must be switched off
while the poles are separating, i.e., during the intervals AK, as in Fig. 4.5.

Fig. 4.5 Variation of inductance, current, flux-linkage, torque, and EMF with rotor position, with ideal pulsed unidirectional
current
Switched reluctance machines Page 4.5

The ideal current waveform is therefore a series of pulses synchronized with the rising inductance
intervals. The ideal torque waveform has the same waveform as the current. The cycle of torque
production associated with one current pulse is called a stroke. Evidently the production of continuous
unidirectional torque requires more than one phase, such that the gaps in the torque waveform are
filled in by currents flowing in the other phases. The numbers of phases and poles are discussed in §4.4.
Normally there is one stroke per rotor pole-pitch in each phase, and the current in any phase is
generally flowing for only a fraction of the rotor pole-pitch. Note that the current and inductance
waveforms imply a sawtooth waveform of flux-linkage R = Li. Such a waveform is not practical because
the sudden extinction of the flux and current would require an infinite negative voltage dR/dt = !4.
Similarly the current cannot be established in step fashion unless the inductance at the beginning of
the stroke (J) is zero. In practice the inductance along UJ is very small so the leading-edge di/dt can
be very large, presenting a possible problem for the power semiconductors. The rectangular current
waveform in Fig. 4.5 can be approximated at low speed by chopping the current along JA, which has the
effect of reducing the average forward applied voltage along JA to a value Va much lower than the
supply voltage Vs. If there is no chopping after commutation at the end of the stroke, the reverse
voltage !Vs makes the current fall to zero over a very small angle of rotation.

4.2 LINEAR ANALYSIS OF THE VOLTAGE EQUATION AND TORQUE PRODUCTION


Linear analysis assumes that the inductance is unaffected by the current: that is, there is no magnetic
saturation. For simplicity we also ignore the effect of fringing flux around the pole corners, and assume
that all the flux crosses the airgap in the radial direction. Mutual coupling between phases is normally
zero or very small, and is ignored. The voltage equation for one phase is
dR dR
v ' Ri ' R i Tm
dt d2
(4.1)
d (L i) di dL
' R i Tm ' Ri L Tm i
d2 dt d2
where v is the terminal voltage, i is the current, R is the flux-linkage in volt-seconds, R is the phase
resistance, L is the phase inductance, 2 is the rotor position, and Tm is the angular velocity in rad/s.
The last term is sometimes interpreted as a “back-EMF” e:
dL
e ' Tm i . (4.2)
d2

It is helpful to visualize the supply voltage as being dropped across the three terms in (4.1): namely, the
resistance voltage drop, the L di/dt term, and the back-EMF e. The instantaneous electrical power vi is
di dL
vi ' R i 2 L i Tm i 2 . (4.3)
dt d2

The rate of change of magnetic stored energy at any instant is given by

d 1 2 1 2 dL di 1 2 dL di
Li ' i Li ' i Tm Li (4.4)
dt 2 2 dt dt 2 d2 dt
According to the law of conservation of energy, the mechanical power conversion p = TmTe is what is
2
left after the resistive loss Ri and the rate of change of magnetic stored energy are subtracted from the
power input vi , Te being the instantaneous electromagnetic torque. Thus from eqns. (4.2) and (4.3),
2 2
writing Te = p/Tm = vi ! Ri ! d(½Li )/dt, we get
1 2 dL
Te ' i . (4.5)
2 d2

Note that dL/d2 is the slope of the inductance graph in Fig. 4.5.
Page 4.6 SPEED’s Electric Motors

Drive circuit — unidirectional current, bidirectional voltage


Eqn. (4.5) says that the torque does not depend on the direction of
2
the current, since i is always positive. However, the voltage
must be reversed at the end of each stroke, to return the flux-
linkage to zero. By Faraday’s law, this requires a negative voltage
applied to the coil, to ensure that dR/dt < 0. Fig. 4.6 shows the
half-bridge phaseleg circuit that accomplishes this. When Q1 and
Q2 are both on, the voltage across the motor windings is v = Vs,
the supply voltage. When Q1 and Q2 are both off, v = !Vs while
the current freewheels through D1 and D2.
To reduce R and i to zero as quickly as possible, as in Fig. 4.5, the
reverse voltage must be much larger than the forward voltage,
otherwise the flux-linkage will persist beyond the aligned
position, producing an unwanted negative pulse of torque. At low
speeds this is achievable by chopping the forward voltage,
reducing its effective value compared to the reverse voltage.
Fig. 4.6 Single phaseleg circuit
The circuit of Fig. 4.6 can operate the machine as a motor or as a
generator, since the electrical power vi can be positive or
negative. If the average power is negative (i.e., generating), the energy supplied during transistor
conduction in one stroke must be less than the energy recovered during freewheeling. The transistor
conduction period (with positive applied voltage) is still necessary to establish the flux, which is built
up from zero and returned to zero each stroke. The voltage-time integrals during transistor conduction
and freewheeling must be approximately equal (apart from resistive volt-drop), regardless of whether
the machine is motoring or generating. As regards control, the main difference between motoring and
generating is the phasing of the conduction pulse relative to the rotor position. From Fig. 4.4 it appears
that generating current pulses must coincide with AK, just as motoring pulses coincide with JA.

Fig. 4.7 Alternative single-phaseleg circuits

Other circuits use only one transistor per phase, and employ various means to produce the "suppression
voltage" (i.e., reverse voltage) needed to de-flux the windings at the end of each stroke. The circuit in
Fig. 4.7a uses separate voltage sources for “fluxing” and “de-fluxing”. The circuit in Fig. 4.7b goes one
stage further by having two isolated windings with a common magnetic circuit. The two parts of the
winding could be bifilar-wound or they could have completely different numbers of turns and wire sizes.
Unfortunately the leakage inductances of the two parts of the winding are usually quite large, even with
a bifilar winding, and this leads to problems with transistor overvoltage. Consequently the circuit of
Fig. 4.7b is rarely used, although it is common in stepper-motor drives.
Switched reluctance machines Page 4.7

Additional phases simply use additional drive circuits of the same form as the first phase, usually with
a common voltage source. With four-phase motors it is possible to use a common chopping transistor
between two phases, so that only six power transistors are required; but the control range of conduction
angles must be limited limited to allow de-fluxing of the “complementary” phase in each pair.
Limitations of the ideal linear model
At the aligned position A in Fig. 4.5, the current
must be switched off quickly to avoid the production
of negative torque after the poles have passed the
2
aligned position. The magnetic stored energy ½Lai
must be returned to the supply. In a nonsaturating
reluctance machine of this type, the magnetic
energy stored at A is large, because both L and i are
at their maximum values. We can get further
insight from an energy audit taken over one stroke
as the rotor moves from J to A. The process is
shown in the energy conversion diagram, Fig. 4.8,
which plots flux-linkage against current. The slope
of OU is the inductance at the unaligned position,
Lu, and the slope of OA is the inductance at the
aligned position, La. At intermediate positions the
Fig. 4.8 Linear energy conversion diagram
inductance is represented by a line of intermediate
slope between Lu and La. At the J position (start of overlap), if fringing is neglected (as in the idealized
inductance variation in Fig. 4.4), LJ = Lu. The complete stroke is represented by the locus OJAO. In
motoring operation it is traversed in the counterclockwise direction, and in generating operation in
the clockwise direction.
Although the current in Fig. 4.5 has a step from 0 to a maximum value im at the position J, in Fig. 4.8 this
2
step is along OJ and energy OJC = ½Luim must be supplied to the magnetic circuit as the current
increases from 0 to im. The step cannot be accomplished in zero time, since that would require infinite
voltage, but if the angular velocity is low, the angle of rotation along OJ is small. Along JA the
electromechanical energy conversion is W or OJA, given by
1 2 dL 1 2
W ' OJA ' Te )2 ' i )2 ' i )L . (4.6)
2 m d2 2 m

Along JA there is a back-EMF e which absorbs energy equal to the area ABCJ:
dL )2
e i )t ' ABCJ ' im2 Tm × ' im2 )L (4.7)
d2 Tm

where )t = )2/Tm is the time taken to rotate through the interval )2 = JA, and )L = La ! Lu is the
change in L. The total energy supplied is the sum of areas OCJ and ABCJ, i.e., area S = W + R = OJAB,
and
1 2
S ' im2 )L im Lu . (4.8)
2

We can now define the energy ratio as


energy converted W W
Q ' ' (4.9)
energy supplied S W R

and if we write 8 = La/Lu we can substitute for W and S from (4.6) and (4.8) to give
8 ! 1
Q ' . (4.10)
28 ! 1

In this type of nonsaturating motor, less than half the energy S supplied by the drive is converted into
Page 4.8 SPEED’s Electric Motors

mechanical work in each stroke (even neglecting losses). During the “working” part of the stroke JA,
the energy is partitioned equally between mechanical work and stored field energy; this is evident from
the ratio of eqns. (4.6) and (4.7), or OJA/ABCJ. The energy ratio would have a value of 0.5 but for the
“overhead” of stored field energy OJC which must be built up before the torque zone JA. This makes
Q < 0.5. The inverse of the energy ratio is the converter volt-ampere-seconds per joule of energy
conversion C = S/W = 1 + R/W, an important quantity in understanding the basic requirement for
“silicon” in the converter.
The stored field energy reaches a maximum value R at A, and must be returned to the supply at the end
of the stroke by commutating the current into the diodes, so that the voltage reverses and forces the
flux-linkage to fall to zero. In the ideal locus this fall is along AO. However this is not possible with
finite supply voltage. If the current is chopped with a duty-cycle d along JA, the average forward
applied voltage along JA is d × Vs. In a circuit of the form of Fig. 4.6, the reverse voltage after
commutation is !Vs. By integrating Faraday’s law the rise and fall periods of flux-linkage can be shown
to be in the ratio
tf
' d. (4.11)
tr

This shows that “instant” suppression of the flux is effectively achieved at very low speed when d is
small. But at higher speed there is no chopping: the forward and reverse voltages are equal in
magnitude, and d =, so tf = tr and the flux-linkage waveform is triangular. The time taken along OA is
the same as the time taken along OJA, The waveforms of flux-linkage and current corresponding to Fig.
4.8 are shown in Fig. 4.9, and show a tail in the current waveform extending past the aligned position
so that some negative torque must be produced.
Figs. 4.8 and 4.9 represent an important operating condition where the back-EMF is just sufficient to
maintain a flat-topped current waveform. With full voltage applied, the speed at which this occurs is
called the base speed. The current im could be called the base current. (It is not necessarily the rated
current, because that depends on the cooling arrangements).

In practical terms the nonsaturating switched reluctance motor makes poor use of the power
Fig. 4.9 Current and flux-linkage waveforms corresponding to Fig. 4.8
Switched reluctance machines Page 4.9

semiconductors, because they have to supply more than 2J of energy in order to get 1 J of mechanical
work, and they must also provide a means to recover the unconverted energy at the end of the stroke.
The DC link filter capacitance is directly related to the value of R, which is a large fraction of the energy
conversion W: in fact
1 ! Q
R ' W × . (4.12)
Q

For example, if 8 = 8, Q = 0.467, C = 2.14, and R = 1.14W. This implies that for zero ripple voltage at the
DC link, the filter capacitance must be large enough to absorb R joules with negligible change of voltage.
This requirement may be reduced by overlapping charge/discharge requirements of adjacent phases,
but still it is a serious consideration.
Eqns. (4.1) and (4.2) imply the existence of an equivalent
circuit of the form shown in Fig. 4.10, in which there is a
back-EMF e = Tm idL/d2. Unfortunately e is not an
independent parameter, but depends on the current. In a
normal equivalent circuit we interpret the product e i as
the electromechanical power conversion TmTe, implying
2
that Te = i dL/d2. However, eqn. (4.5) states that the
2
torque is only ½i dL/d2. Of the power ei, only half is
converted into mechanical power during the “working”
part of the stroke JA. The other half is being stored as
Fig. 4.10 Equivalent circuit
magnetic field energy in the increasing inductance. With
L also varying, the equivalent circuit is misleading and cannot be interpreted in the same way as it can,
for example, with permanent-magnet motors. This means that the simulation of switched reluctance
machines and their drives requires the direct solution of eqns. (4.1) and (4.5), even when saturation is
ignored. An elegant and thorough solution for the nonsaturating motor was presented by Ray and Davis
[1979]. Usually saturation cannot be ignored and the full nonlinear equations must be solved.

4.3 NONLINEAR ANALYSIS OF TORQUE PRODUCTION


We have already seen that the nonsaturating (i.e.,
magnetically linear) switched reluctance machine has
a low energy ratio and makes poor utilization of the
drive. Practical switched reluctance machines are
more effective but they are far from being
magnetically linear. To underst and the
electromechanical energy conversion properly, we
need a nonlinear analysis that takes account of the
saturation of the magnetic circuit. One such analysis
is based on the magnetization curves. A magnetization
curve is a curve of flux-linkage R versus current i at a
particular rotor position, Fig. 4.11.
We also need to define the stored magnetic energy Wf Fig. 4.11 A magnetization curve at one rotor position
and the coenergy Wc graphically, as in Fig. 4.12.

Mathematically,

m
Wf ' i dR ;
(4.13)
m
Wc ' R di .

In a magnetically linear device with no saturation, the magnetization curve is straight, and Wf = Wc.
Page 4.10 SPEED’s Electric Motors

The effect of saturation is to make Wf < Wc. In


machines with magnetic circuits similar to the one in
Fig. 4.1, saturation of a typical magnetization curve
occurs in two stages. When the overlap between rotor
and stator pole corners is quite small, the
concentration of flux saturates the pole corners, even
at low current. When the overlapping poles are closer
to the aligned position, the yokes saturate at high
current, tending to limit the maximum flux-linkage.
Magnetization curves near the aligned position may
appear “double-jointed” as in Fig. 4.15, if the airgap is
small and the curve is plotted to high flux levels.

In a displacement )2 or AB at constant current (Fig.


4.13), the energy exchanged with the supply is

dR
m m m
Fig. 4.12 Definition of energy Wf and coenergy Wc
)We ' ei dt ' i dt ' i dR ' ABCD (4.14)
dt

and the change in magnetic stored energy is

)Wf ' OBC ! OAD . (4.15)

The mechanical work done must be

)Wm ' ) We ! ) Wf
' ABCD ! ( OBC ! OAD )
(4.16)
' OABCD ! OBC
' OAB

and this is equated to Te )2, so that when )2 6 0,

MWc
Te ' (4.17)
M2 i'const
Fig. 4.13 Determination of electromagnetic torque

Ideal cases: In a motor with no saturation the magnetization


curves would be straight lines, Fig. 4.14a. At any position, Wf =
2 2
Wc = ½L(2)i , and in this case (4.17) reduces to Te = ½i dL/d2,
which we saw earlier.

In a motor with a very small airgap, and especially if the steel


has a "square" B/H curve, the magnetization curves
approximate to parallel straight lines with a shallow slope, Fig.
4.14b. In this case the stored field energy is small and Wc - Ri,
so Te = i dR/d2 = e i. This condition approximates to the
permanent-magnet motor which has an EMF that is independent
of the current. Because of the small airgap, only a small fraction
of the current is needed to raise the flux-density to the
saturation level in the overlap region, and the flux-linkage
varies linearly with the overlap angle. The energy ratio is 1, and
the utilization of the power semiconductors in the drive is high.
Practical reluctance motors, especially highly-rated ones, are
often designed to try to approximate this ideal condition.

Fig. 4.14 Ideal magnetization curves


Switched reluctance machines Page 4.11

Average torque: So far we have seen that the torque is


produced in impulses as the rotor rotates, and we have
determined the instantaneous torque Te in terms of the
rate of change of coenergy at constant current. The
average torque could be calculated by integrating Te
over one cycle, (i.e., one rotor pole-pitch J = 2B/Nr,
where Nr is the number of rotor poles), and dividing by
J. However, it is difficult in practice to calculate Te
accurately, and it is better to calculate the average
torque from the enclosed area W in the energy-
conversion diagram, Fig. 4.15.

In one cycle of operation the maximum possible energy


conversion at a current I is the area W enclosed
between the unaligned magnetization curve U, the
aligned magnetization curve A, and the vertical line UA
at the current I. One cycle of operation, i.e., one
execution of this loop, is called a stroke. If S is the
number of strokes per revolution, the average Fig. 4.15 Calculation of average torque
electromagnetic torque in Nm is

SW
Te[avg] ' . (4.18)
2B

The drive must switch the current on and off at the correct rotor angles to cause the operating point to
follow this loop as closely as possible. Along UA, the current can be regulated by chopping, but at high
speed this may not be achievable and the loop may be smaller than the maximum loop, Fig. 4.30.

Fig. 4.16 Current, torque, and flux-linkage waveforms with a naturally-determined flat-topped
current waveform. i = phase current; T = phase electromagnetic torque; o = per-unit
overlap between active stator and rotor poles; R = phase flux-linkage; W = energy
conversion loop area; U = unaligned; A = aligned.

Fig. 4.16 shows an example of a motor operating with a naturally flat-topped current waveform, which
is obtained when the back-EMF is approximately equal to the DC supply voltage. This motor is analyzed
in more detail later. The speed in this example is 1015 rpm.
Page 4.12 SPEED’s Electric Motors

4.4 CONTINUOUS TORQUE PRODUCTION

The motor in Fig. 4.1 is useful for analyzing torque


production, and although it can maintain a nonzero average
torque when rotating in either direction, this torque is
discontinuous, which means that continuous rotation
depends on the momentum or flywheel effect. Moreover it
cannot self-start from every rotor position. For example, at
the unaligned and aligned positions the torque is zero.
Unidirectional torque can be produced only over a limited
angle where the overlap angle 8 between the rotor and
stator poles is varying. To provide continuous
unidirectional torque, with starting capability from any
position, the motor needs more phases, and this requires a
“multiplicity” of stator and rotor poles, as in Fig. 4.17.

The number of strokes per revolution is related to the Fig. 4.17 3-phase 6/4 switched reluctance motor
number of rotor poles Nr and the number of phases m, and
in general
S ' m Nr . (4.19)

This can be substituted in eqn. (4.18) to give the average torque including all m phases, provided that
S is the same for all of them. The motor in Fig. 4.17 has m = 3 and Nr = 4, so S = 12. The stroke angle is
g = 360/12 = 30E. The three phases are labelled AA', BB ' and CC ,' and the ideal current/torque pulses
are shown in Fig. 4.18. The resultant torque is ideally constant and covers 360E of rotation. In practice,
of course, the waveforms are more complex than the ideal ones in Fig. 4.18, and their computation
requires a numerical simulation of the transient electromagnetic behaviour throughout one stroke.

Fig. 4.18 Waveforms in 3-phase 6/4 switched reluctance machine

Magnetic frequency: The fundamental frequency f1 of the current in each phase is evidently equal
to the rotor pole passing frequency, i.e.,
rpm
f1 ' × Nr Hz . (4.20)
60

The number of strokes per second is given by


f ' m f1 Hz . (4.21)
This frequency and its harmonics appear in the flux waveforms in various parts of the magnetic circuit.
Switched reluctance machines Page 4.13

Stator/rotor pole numbers

To provide a structure for ordering the numbers of stator and rotor poles, we can start by defining a
regular switched reluctance motor as one in which the rotor and stator poles are symmetrical about
their centre-lines and equally spaced around the rotor and stator respectively. An irregular motor is
one which is not regular. Here we are mainly concerned with regular machines, since they usually
have the most sophisticated power electronic control requirements; and therefore the emphasis is on
machines with m = 3 or 4 phases. Machines with m = 1 or 2 are usually irregular and they are discussed
elsewhere together with various other irregular and special machines.

The absolute torque zone Ja is defined as the angle through which one phase can produce non-zero
torque in one direction. In a regular motor with Nr rotor poles, the maximum torque zone is Ja(max) =
B/Nr. The effective torque zone Je is the angle through which one phase can produce useful torque
comparable to the rated torque. The effective torque zone is comparable to the lesser pole-arc of two
overlapping poles. For example, in Fig. 4.17 the effective torque zone is equal to the stator pole-arc: Je
= $s = 30E.

The stroke angle g is given by 2B/(strokes/rev) or


2B 2B
g ' ' , (4.22)
S m Nr

The absolute overlap ratio Da is defined as the ratio of the absolute torque zone to the stroke angle:
evidently this is equal to m/2. A value of at least 1 is necessary if the regular motor is to be capable of
producing torque at all rotor positions. In practice a value of 1 is not sufficient, because one phase can
never provide rated torque throughout the absolute torque zone in both directions. The effective
overlap ratio De is defined as the ratio of the effective torque zone to the stroke angle, De = Je/g . For
regular motors with $s < $r this is approximately equal to $s/g. For example, in Fig. 4.17 the effective
overlap ratio is 30E/30E = 1. Note that De < Da. A value of De of at least 1 is necessary to achieve good
starting torque from all rotor positions with only one phase conducting, and it is also a necessary (but
not sufficient) condition for avoiding torque dips.

Three-phase regular motors: With m = 3, Da = 1.5 and De can have values of 1 or more, so regular 3-
phase motors can be made for 4-quadrant operation. In the 6/4 motor in Fig. 4.17 , forward rotation
corresponds to negative phase sequence. This is characteristic of vernier motors, in which the rotor
pole-pitch is less than B/m. The 3-phase 6/4 motor has S = mNr = 12 strokes/rev, with a stroke angle g
= 30E, giving De = $s/g = 30/30 = 1.

With regular vernier motors there is always the


choice of having either Nr = Ns ! 2, as in the 6/4; or
Nr = Ns + 2, which gives the 6/8 motor shown in
Fig. 4.19; it has S = 24 strokes/rev and g = 15E. The
advantage of the larger Nr is a smaller stroke angle,
leading possibly to a lower torque ripple; but
inevitably the price paid is a lower inductance
ratio which may increase the controller volt-
amperes and decrease the specific output. The
stator pole arc has to be reduced below that of the
6/4 motor and this decreases the aligned
inductance, the inductance ratio, and the
maximum flux-linkage (although it increases the
slot area). The consequent reduction in available
conversion energy tends to offset the increase in
the number of strokes/rev, and the core losses may Fig. 4.19 3-phase 6/8 motor. Each phase has two coils on
opposite poles.
be higher than those of the 6/4 motor because of the
higher switching frequency.
Page 4.14 SPEED’s Electric Motors

The 12/8 three-phase motor is effectively a 6/4 with


a "multiplicity" of two. It has S = 24 strokes/rev,
with a stroke angle g = 15E and Da = 1.5. In Fig. 4.20,
De = 15/15 = 1, the same as for the 6/4 motor
discussed earlier. A high inductance ratio can be
maintained and the end-windings are short: this
minimizes the copper losses, shortens the frame,
and decreases the unaligned inductance.
Moreover, the magnetic field in this machine has
short flux-paths because of its four-pole magnetic
field configuration, unlike the two-pole
configuration in the 6/4 (or the 8/6; see below), and
the four-pole magnetic circuit helps to minimize
acoustic noise (see chapter 4). Although the MMF
per pole is reduced along with the slot area, the
effects of long flux-paths through the stator yoke
are alleviated. The 12/8 is possibly the most Fig. 4.20 3-phase 12/8 motor. Each phase has four coils, and
popular configuration for three-phase machines. the magnetic flux-pattern is 4-pole.

Four-phase regular motors: The 4-phase regular


8/6 motor shown in Fig. 4.21 has 24 strokes/rev and
a stroke angle of 15E, giving Da = 2. With $s = 21E, De
= 1.33, which is sufficient to ensure starting torque
from any rotor position, and it implies that there
will be no problem with torque dips. However, it is
generally impossible to achieve the same flux-
density waveform in every section of the stator
yoke, because of the polarities of the stator poles
( NNNNSSSS , NNSSNNSS , or NSNSNSN ). This
configuration was one of the first to be produced
commercially.1

With Ns = Nr + 2 = 10, S = 32 strokes/rev and g =


11.25E. The inductance ratio is inevitably lower
than in the 8/6, and the poles are narrower, while
the clearance between pole-corners in the
unaligned position is smaller, increasing the
unaligned inductance. This motor is probably on
the borderline where these effects cancel each Fig. 4.21 4-phase 8/6 motor
other out; with higher pole-numbers, the loss of
inductance ratio and energy-conversion area tends to dominate the gain in strokes/rev. For this reason,
higher pole-numbers are not considered here.

Table 4.2 gives some examples of stator/rotor pole-number combinations for motors with up to m = 7
phases. The parameter NwkPP is the number of working pole-pairs: that is, the number of pole-pairs in
the basic magnetic circuit. For example, the 4-phase 8/6 has NwkPP = 1 (a 2-pole flux pattern), while the
3-phase 12/8 has NwkPP = 2 (a 4-pole flux pattern). The unshaded boxes in Table 4.2 are probable the best
choices, the others having too many poles to achieve a satisfactory inductance ratio or too high a
magnetic frequency.

1
i.e., the well-known OULTON motor introduced in 1983 by Tasc Drives Ltd., Lowestoft, England.
Switched reluctance machines Page 4.15

m Ns Nr NwkPP gE S m Ns Nr NwkPP gE S
2 4 2 1 90.00 4 4 8 6 1 15.00 24
2 8 4 2 45.00 8 4 16 12 2 7.50 48
2 4 6 1 30.00 12 4 24 18 3 5.00 72
2 8 12 2 15.00 24 4 32 24 4 3.75 96
2 12 18 3 10.00 36 4 8 10 1 9.00 40
2 16 24 4 7.50 48
m Ns Nr NwkPP gE S
m Ns Nr NwkPP gE S 5 10 4 1 18.00 20
3 6 2 1 60.00 6 5 10 6 1 12.00 30
3 6 4 1 30.00 12 5 10 8 1 9.00 40
3 6 8 1 15.00 24 5 10 12 1 6.00 60
3 12 8 2 15.00 24
3 18 12 3 10.00 36 m Ns Nr NwkPP gE S
3 24 16 4 7.50 48 6 12 10 1 6.00 60
6 24 20 2 3.00 120
6 12 14 1 4.29 84

m Ns Nr NwkPP gE S
7 14 10 1 5.14 70
7 14 12 1 4.29 84

TABLE 4.2
EXAMPLES OF VALID STATOR/ROTOR POLE NUMBER COMBINATIONS

4.5 ENERGY CONVERSION ANALYSIS OF THE SATURATED MACHINE

Energy ratio and converter volt-ampere requirement

Fig. 4.22 shows a model of the energy


conversion process in which the unaligned
magnetization curve is assumed to be
straight, while the aligned curve is
composed of two sections, a straight line OS
and a parabola SA. For a given peak
current im the energy conversion
capability W is completely defined by the
three points U,S,A. It is shown in Miller
and McGilp [1990] that the parabola section
SA is represented by

( R ! Rs0 )2 ' 4 a ( i ! is0) (4.23)


2
with is0 = is ! a/Lau , Rs0 = Rs ! 2a/Lau, a =
2
Rms /4(ims ! Rms/Lau), Rms = Rm ! Rs, and
ims = im ! is. These relationships ensure
that the first derivatives of the segments OS Fig. 4.22 Nonlinear energy conversion analysis, with aligned
and SA are equal at S. The area R can be magnetization curve represented by a straight line OS and
a parabola SA.
calculated by direct integration:

Rm 2 Rs ( Rm Rs ) Rs0 ! (Rm Rs ) 1
R ' Rms Rs0 is0 Lau is2 (4.24)
12 a 4a 2
Page 4.16 SPEED’s Electric Motors

and then the energy-conversion area W can be obtained as


1
W ' im Rm ! R ! Lu im2 . (4.25)
2

The energy ratio Q is defined as W/(W + R), with inverse C = 1 + R/W. For example, in Fig. 4.22 the
unsaturated inductance ratio 8 = Lau/Lu = 8, and is drawn with Rm = 0.7 V-s, Rs = 0.5 V-s, Ru = 0.25 V-s,
im = 80A, and is = 20A. The resulting values are approximately W = 27.7J, R = 13.3J , Q = 0.67, and C =
1.5. The energy ratio is about 45% greater than in the linear nonsaturating machine (Q = 0.467), and the
converter volt-ampere-second requirement of the nonsaturating machine is about 45% higher.

Estimation of the commutation angle

Fig. 4.23 shows a model of the energy conversion


process in which the unaligned magnetization
curve is again straight, while the aligned curve
is fitted by straight lines OS and SA. For a given
peak current the energy conversion capability W
is again defined by the three points U,S,A. This
model is like the one which J.V. Byrne [1970]
used to describe controlled saturation. The
saturation effect is characterized by the ratio
Rm / im
F ' (4.26)
Rs / is
which is effectively the ratio of the saturated to
the unsaturated inductance in the aligned
position. At the base speed the current waveform
is naturally flat-topped with peak value im,
because the back-EMF e of the motor equals the Fig. 4.23 Nonlinear energy-conversion analysis, with aligned
supply voltage (resistance is neglected). If the magnetization curve represented by two straight
lines.
angular rotation JA is assumed equal to the
stator pole-arc $s, then it can be shown that e =
Vs when
V s $s
' im Lu ( 8 F ! 1 ) ' ) Rm (4.27)
Tm
Commutation is at point C such that the change of flux-linkage between J and C is c)Rm, where c # 1.
After commutation the current continues to flow throughout the angle (1 ! c)$s and for an undetermined
interval k$s thereafter. Since the peak flux-linkage Rc is c)Rm + Lui, we can write
V s $s
Rc ' ( 1 ! c k ) ' c ) Rm Lu im (4.28)
Tm
which describes the de-fluxing interval with !Vs applied via the diodes. Eqns. (4.27) and (4.28) can be
used to solve for c:
(1 k)(8F ! 1) ! 1
c ' . (4.29)
2(8F ! 1)

When the rotor reaches the aligned position the flux-linkage is


k
RE ' Rc (4.30)
1 ! c k

and it follows that the current at the aligned position is


RE
iE ' . (4.31)
Lau
The example in Fig. 4.22 has F = 0.350 and 8 = 8. We can consider two extreme cases:
Switched reluctance machines Page 4.17

(i) Commutation at the aligned position — In this case c = 1 and from eqn. (4.29) k = 1.556, so the
current continues to flow for 1.556 times the angle $s after the aligned position. The flux-linkage
at the aligned position is Rm and the current is iE = Rm/Lau = 40Rm = 28.0A.

(ii) Commutation such that the current extinguishes at the aligned position — In this case
k = 0 and from eqn. (4.29), c = 0.222, and RE = iE = 0. Although all negative torque is completely
eliminated, the energy conversion with c = 0.222 is far below the capability of the machine.
In practice the commutation angle is selected to maximize the torque per ampere, and falls between
these extremes, with k > 0 and some negative torque. In the example shown in Fig. 4.16, the negative
torque is very small and c = 2/3, giving k = 0.89 and RE = 0.727Rc. Although resistance accelerates the
flux suppression, it can be seen that the current tail in the negative torque zone has little impact on the
overall energy conversion.

Basic torque/speed characteristic

An interesting simplified analysis of the speed range at constant power was given by Byrne and
McMullin [1982], in which they derived a formula for the speed range at constant power. If Tp is the
maximum speed at which the power can be developed equal to the maximum power at base speed Tb,
then in terms of the parameters used in this chapter,
2
Tp 1 2p (8 F)2
n ' ' (4.32)
Tb 2 2b 8F ! 1

where 2p and 2b are the dwell angles (transistor conduction angles) at the speeds Tp and Tb respectively.
For the example motor 8F = 8 × 0.35 = 2.8 and if we assume 2p = 1.52b, we get n = 4.9. This is probably
optimistic but it shows the importance of phase advance and the saturated inductance ratio 8F. A
typical speed range at constant power is probably nearer 3.

The parameter c in Fig. 4.23 effectively controls the energy conversion loop area and the average torque.
At low speeds it is possible to work with c = 1, so that the entire available energy conversion area
between the aligned and unaligned curves, bounded on the right by the peak current, can be used.
Miller [1985b] gives this average electromagnetic torque the equation
m Nr
T a ' i m2 L u ( 8 F ! 1 ) c (2 ! c/s) (4.33)
4B
where s = (8 ! 1)/(8F ! 1). At the base speed c has a value cb < 1 and from (4.33) the ratio of the torque
at zero speed to the torque at base speed is derived as
T0 2s ! 1
' . (4.34)
Tb cb ( 2 s ! cb )

In the example motor cb = 2/3 and s = (8 ! 1)/(8 × 0.35 ! 1) = 3.89, and T0/Tb = 1.43. The motor can
evidently produce 43% more torque at standstill than at the base speed, for the same peak current. (The
r.m.s. current must be increased because the dwell is greater). With the same peak current, the peak
torque is the same in both cases, implying that the torque waveform must be peakier at the base speed
than at standstill. Miller [1985b] goes on to compare the volt-ampere requirements of the switched
reluctance motor with those of a comparable high-efficiency induction motor, and concludes that the
switched reluctance motor requires 14% more volt-amperes based on peak current, or 20% more based
on r.m.s. current. Such comparisons are, however, extremely difficult to generalize.
Page 4.18 SPEED’s Electric Motors

Analysis of the energy-conversion loop

During a typical motoring stroke the locus of the


operating point (i,R) follows a curve similar to the one
in Fig. 4.24, which is drawn with the aligned and
unaligned magnetization curves and another
magnetization curve at the commutation angle C. At
this point the supply voltage is reversed and the current
freewheels through the diode. At C the accumulated
energy from the supply is equal to the total area U = Wmt
+ WfC. The stored magnetic energy is equal to WfC.
Therefore the mechanical work done between turn-on
and commutation is Wmt, during the period of transistor
conduction. In Fig. 4.24 this is roughly comparable to
WfC, meaning that only half of the energy supplied has
been converted to mechanical work. The other half is
stored in the field.
Fig. 4.24 Analysis of energy-conversion loop:
transistor conduction
After commutation, Fig. 4.25, the supply voltage is
reversed and the energy Wd is returned to the supply via
the diodes. The mechanical work done during the
freewheel interval is Wmd = WfC ! Wd. In Fig. 4.25 this is
less than half of WfC. The energy balance can be
deduced from the areas in Figs. 4.24 and 4.25. Suppose
that the energy supplied from the controller during the
"fluxing" interval (transistor conduction) is U = Wmt +
WfC = 10 Joules. At C, 5J has been converted to
mechanical work and 5J are stored in the field. During
the "de-fluxing" period (diode freewheeling), Wd = 3.5J is
returned to the supply and Wmd = 1.5J is converted to
work. The total work is therefore W = Wmt + Wmd = 5 +
1.5 = 6.5J or 65% of the energy supplied by the
controller. The energy returned to the supply is Wd =
3.5J or 35% on each "stroke".

The entire stroke is shown in Fig. 4.26, which combines


the two previous diagrams. The energy conversion is Fig. 4.25 Analysis of energy-conversion loop: diode
freewheeling
now shown as the area W, while the energy returned to
the supply is R = Wd. The original energy supplied by
the controller is U = W+R, and the energy ratio is 0.65.

4.6 OBTAINING THE MAGNETIZATION CURVES

Calculation

The aligned magnetization curve can be calculated by


lumped-parameter magnetic circuit analysis, with an
allowance for the stator slot-leakage which increases at
high flux levels. The unaligned curve is more difficult
to calculate because of the complexity of the magnetic
flux paths in this position, but practical results are
reported by Miller and McGilp [1990] using a dual-
energy method based on a quadrilateral discretization
of the slotted region. Earlier work by Corda and
Stephenson [1979] also produces adequate results for
many practical or preliminary design calculations. Fig. 4.26 Analysis of energy-conversion loop: the
combined loop
Switched reluctance machines Page 4.19

The computation should include the “partial linkage” effect, meaning that at certain rotor positions
the turns of each coil do not all link the same flux. In a finite-element calculation this implies that the
flux-linkage should be calculated as the integral of A@dl along the actual conductors, with the coilsides
in the finite-element mesh in their correct positions in the slot.

End effects are important in switched reluctance motors. (See Michaelides and Pollock [1994]; Reece
and Preston, [2000]). When the rotor is at or near the unaligned position, the magnetic flux tends to
"bulge out" in the axial direction. The associated increase in the magnetic permeance can raise the
unaligned inductance Lu by 20!30%. Since this inductance is critical in the performance calculations,
it is important to have a reasonable estimate of it. Unfortunately 2-dimensional finite-element analysis
cannot help with this problem, and 3-dimensional finite-element calculations tend to be expensive and
slow. When the rotor is at or near the aligned position, the flux is generally higher and the "bulging"
of flux outside the core depends on the flux level in the laminations near the ends of the stack. At or
near the aligned position at high flux levels, the stator and rotor poles can be highly saturated and the
external flux-paths at the ends of the machine can increase the overall flux-linkage by a few percent.

In spite of the complexity of the field problem, good results have been obtained with relatively simple
end-effect factors for Lu. For example, the PC-SRD computer program [Miller, 1999] splits the end-effect
calculation into two parts. For Lu,
Lu ' Lu0 × (eu fu) (4.35)

where eu = Lend/Lu0 represents the self-inductance Lend of the end-windings (including any extension),
expressed as a fraction of the uncorrected 2-dimensional unaligned inductance Lu0. Lend is the
inductance of a circular coil whose circumference is equal to the total end-turn length of one pole-coil,
including both ends. It is multiplied by the appropriate function of turns/pole and parallel paths before
being normalized to Lu0. fu is a factor that accounts for the axial fringing in the end-region. It is
calculated by the approximation
R1 ! R0 Lstk
fu ' (4.36)
Lstk
which is derived by analogy with the fringing formula for two opposite teeth or poles. For the aligned
position the procedure is similar, with a factor fa = (g + Lstk)/Lstk.

At positions intermediate between the


aligned and unaligned positions, the
calculation of individual magnetization
curves is not practical by analytical methods
and the finite-element method should be
used. Fig. 4.1 shows a simple example of a
finite-element flux-plot at a position of
partial overlap, and Fig. 4.27 a more complex
example. Considerable success has been
achieved with interpolating procedures
especially in computer programs for rapid
design, e.g., [Miller and McGilp, 1990],
[Miller et al 1998].

Measurement of the magnetization curves is


described in [Cossar 1992].
Fig. 4.27 Finite-element flux-plot in a partial-overlap position
Page 4.20 SPEED’s Electric Motors

4.7 SOLUTION OF THE MACHINE EQUATIONS

The PC-SRD computer program has been widely used for design and analysis of switched reluctance
machines for many years. Each phase is treated as a variable inductance in which the flux-linkage R
is a nonlinear function of both the current i and the rotor position 2: thus R = R(i, 2). PC-SRD computes
the function R(i, 2) from the geometry, the winding details, and the B/H curve of the steel, and it
represents the result graphically as a set of static magnetization curves in which R is plotted vs. i at
several rotor positions between the unaligned and the aligned positions. For a given machine, the
curves are a fixed property. It is not necessary to recalculate them unless changes are made to the
geometry, the windings, or the steel. For precise work PC-SRD can import external magnetization
curves which have been obtained either by measurement or by 3D finite-element calculation.

PC-SRD solves the electrical circuit by stepwise integration of eqn. (4.1) for one phase by Euler’s
method. Each integration step produces a new value of flux-linkage R, and PC-SRD computes a new
value of current i from the function R(i,2), i.e., from the magnetization curves. The method of solving
for each new current value i depends on whether the curves are internal or external. With internal
magnetization curves PC-SRD uses a fast algebraic interpolation method based on so-called gauge
curves [Miller and McGilp, 1990]. With external curves, PC-SRD fits the curves with a set of cubic
splines, and interpolates them. This is slower than the gauge curve method, but more accurate.

The instantaneous torque is calculated from the rate of change of coenergy MWc (i,2)/M2. With internal
magnetization curves, the derivative is evaluated using approximate algebraic expressions derived
from the gauge curve model. With external mag curves it is evaluated from a precalculated set of spline
functions that represent the coenergy Wc as a function of current and rotor position, Wc (i,2). The
average electromagnetic torque is computed from the loop area W in Fig. 4.15, and a typical example
is given in Fig. 4.16. Several computed examples are given later. The average electromagnetic torque
is given by (4.18). Since W is evidently an integral quantity, errors in the distribution of the
magnetization curves tend to cancel out, provided that the aligned and unaligned curves are accurate.
On the other hand, the calculation of the instantaneous torque is sensitive to the precision in the
intermediate magnetization curves and the method of representing them mathematically is critical.

PC-SRD’s model is a single-phase model. The currents in phases 2,3,... are determined by phase-shifting
the current waveform of phase 1, which is calculated as though it were the only current flowing. The
fluxes and torques of the other phases are added to those of phase 1 without taking account of
interactions in shared magnetic circuits. This is one of the main limitations of PC-SRD, but it also
explains the extraordinary speed of computation. A full magnetic model of a polyphase switched
reluctance motor, including all magnetic interactions between phases, requires a multi-dimensional
set of magnetization curves and is a formidably complex thing to contemplate. PC-SRD’s simple model
is successful when the yokes are sufficient to avoid mutual interaction between phases. Under
conditions of extreme loading or (e.g., faults), the PC-SRD model cannot be expected to give accurate
results, even with external magnetization curves, since these are valid only for one phase conducting.

A method that uses coenergy and avoids integration

The somewhat convoluted process described in the previous section could in principle be replaced by
a more direct method based on a coenergy map. The voltage equation for one phase is

v ' e Ri (4.37)

where the back-EMF is given by


MR MR
e ' ' Tm (4.38)
Mt M2

and this can be obtained from the R(i,2) curves by differentiating with respect to 2 at constant current.
Switched reluctance machines Page 4.21

The solution proceeds at each timestep by calculating


v ! e
i ' (4.39)
R
using the current values of v and e. Once the “new” current is calculated from (4.39), e is re-evaluated
using (4.38) for the next integration step. The torque is calculated using
MWc
T ' (4.40)
M2
which is also evaluated at constant current. It is interesting to note that the flux-linkage can also be
evaluated from the coenergy using
MWc
R ' (4.41)
Mi
evaluated at constant 2. This suggests that the back-EMF can be evaluated using
M2Wc
e ' Tm (4.42)
M2 Mi
This suggests that the machine can be represented by a surface Wc (i,2) whose derivatives can be used
at any position 2 and any current i to determine the back-EMF e (4. 38) the flux-linkage R (4.41) and the
torque T (4.40). The finite-element solution of the magnetization curves can therefore be expressed in
terms of the surface Wc (i,2) without even computing the flux-linkage R, given that the finite-element
method can compute Wc directly by means of a global integration. A difficulty with this approach, as
with the previous one, is the representation of the coenergy function by a sufficiently smooth
interpolating function that is differentiable both 2 and i. Also, it does not naturally provide data which
can be compared with measurements.

4.8 CONTROL PRINCIPLES

Torque in the switched reluctance machine is produced by pulses of phase current synchronized with
rotor position. The timing and regulation of these current pulses are controlled by the drive circuit and
the torque control scheme. Usually there are also outer feedback loops for controlling speed or shaft
position, as shown in Fig. 4.28. The outer loops are generally similar to those used in other types of
motor drive, but the inner torque loop is specific to the switched reluctance machine.

T*

Fig. 4.28 Nested control loops. T * = torque demand; Tm* = speed demand, Tm = speed; 2* = position demand; 2 = shaft position.
Tacho = tachometer or speed transducer; Enc = encoder or position transducer.

The torque demand signal generated by the outer control loops is translated into individual current
reference signals for each phase, [Bose, 1987]. The torque is controlled by regulating these currents.
Usually there is no torque sensor and therefore the torque control loop is not a closed loop.
Consequently, if smooth torque is required, any variation in the torque/current or torque/position
relationships must be compensated in the feed-forward torque control algorithm. This implies that the
torque control algorithm must incorporate some kind of “motor model”.
Page 4.22 SPEED’s Electric Motors

Unlike the DC or brushless DC motor drive, the switched reluctance motor drive cannot be
characterized by a simple torque contstant kT (torque/ampere). The drive must be specifically
programmed for a particular motor, and even for particular applications. One cannot take a switched
reluctance motor from one source and connect it to a drive from another source, even when the voltage
and current ratings are matched. On the contrary, the motor and drive control must be designed
together, and usually they must be optimized or tuned for a particular application.

The power electronic drive circuit is usually built from phaselegs of the form shown in Fig. 4.6. These
circuits can supply current in only one direction, but they can supply positive, negative, or zero voltage
at the phase terminals. Each phase in the machine may be connected to a phaseleg of this type, and the
phases together with their phaseleg drive circuits are essentially independent. The circuits in Fig. 4.7
can be adapted to operate the phases with separate DC supplies of different voltages, although the most
usual case is to connect them all to a common DC supply. Figs. 4.7a and 4.7b also show the possibility
of “fluxing” at one voltage V1 and “de-fluxing” at another voltage !V2.

At lower speeds the torque is limited only by the current, which is regulated either by voltage PWM
or current regulation. As the speed increases the back-EMF increases to a level at which there is
insufficient voltage available to regulate the current; the torque can then be controlled only by the
timing of the current pulses. This control mode is called “single-pulse mode” or “firing angle control”,
since the firing angles alone are controlled to produce the desired torque. Many applications require
a combination of the high-speed and low-speed control modes. Even at lower speeds with voltage PWM
or current regulation, the firing angles must be varied with speed to optimise performance.

This chapter is concerned with control of average torque, i.e., the torque averaged over one stroke ( g
= 2B/mNr). The amplitude and phase of the current reference signal (relative to the rotor position) are
assumed to remain constant during each stroke. This corresponds to the operation of a “variable-speed
drive”, as distinct from a servo drive which would be expected to control the instantaneous torque.
Average torque control requires a lower control-loop bandwidth than instantaneous torque control.

Differences between switched reluctance machines and classical machines: Much of the classical
theory of torque control in electric drives is based on the DC machine, in which torque is proportional
to flux × current. The flux and current are controlled independently, and the “orientation” of the flux
and the ampere-conductor distribution, both in space and in time, is fixed by the commutator. In AC
field-oriented control, mathematical transformations are used, in effect, to achieve independent control
of flux and current, and the commutator is replaced by a shaft-position sensor which is used by the
control processor to adjust the magnitude and phase of the currents to the correct relationship with
respect to the flux. The current can be varied rapidly so that a rapid torque response can be achieved.
Generally speaking, in classical DC and AC machines the flux is maintained constant while the current
is varied in response to the torque demand. In both cases the torque control theory is characterized by
the concept of “orthogonality”, which loosely means that the flux and current are “at right angles”. In
the architecture of the machine and the drive, this concept has a precise mathematical meaning which
depends on the particular form or model of the system.

In switched reluctance machines, unfortunately there is no equivalent of field-oriented control. Torque


is produced in impulses and the flux in each phase must usually be built up from zero and returned to
zero each stroke. The “orthogonality” of the flux and current is difficult to contemplate, because the
machine is “singly excited” and therefore the “armature current” and “field current” are
indistinguishable from the actual phase current. Although this appears to be the case also with
induction machines, the induction machine has sine-distributed windings and a smooth airgap, so that
the theory of space vectors can be used to resolve the instantaneous phase currents into an MMF
distribution which has both direction and magnitude, and the components of this MMF distribution can
be aligned with the flux or orthogonal to it. The switched reluctance machine does not have sine-
distributed windings or a smooth airgap, and there is virtually no hope of “field-oriented” control. To
achieve continuous control of the instantaneous torque, the current waveform must be modulated
according to a complex mathematical model of the machine.
Switched reluctance machines Page 4.23

4.9 VARIATION OF CURRENT WAVEFORM WITH TORQUE AND SPEED

The average electromagnetic torque is given by eqn. (4.14), and the energy-conversion loop area W is
shown in Figs. 4.12 and 4.13. The objective of “average torque control” is a simple current pulse
waveform which produces the required value of W corresponding to the torque demand. Even in simple
cases, this is more complex than simply determining the required “value of current”, since the
torque/ampere varies with both position and current. The following sections describe the general
properties of the current waveform at different points in the torque/speed diagram, Fig. 4.35.

Fig. 4.29 Low-speed motoring waveforms. i = phase current, R = phase flux-linkage, T = phase torque, and o = overlap between
stator and rotor poles. Horizontal axis is rotor angle (degrees). Unaligned position U = 45E; aligned position A = 90E.
The position J is the start of overlap between the active rotor poles and the stator poles of this phase.

Low-speed motoring — At low speed the motor EMF e is low compared to the available supply voltage
Vs, and the current can be regulated by chopping. If voltage-drops in the semiconductor devices are
neglected, the drive can apply three voltage levels +Vs, !Vs or 0 to the winding terminals to raise or
lower the flux and current. A simple strategy is to supply constant current throughout the torque zone,
i.e., over the angle through which the phase inductance is substantially rising. Fig. 4.29 shows a typical
low-speed motoring current waveform of this type in a 3-phase 6/4 motor at 500 rev/min.

The current i is chopped at about 8 A, starting 5E after the unaligned position (at 45E) and finishing 10E
before the aligned position (at 90E). At first no torque is produced because the inductance is low and
unchanging, but when the corners of the stator and rotor poles are within a few degrees of conjunction
J, torque suddenly appears. It is controlled by the regulating the current. When the transistors are
switched off, 10E before the aligned position, the current commutates into the diodes and falls to zero,
reaching the “extinction” point a few degrees later, so that virtually no negative torque is produced.

The flux-linkage R grows from zero and falls back to zero every stroke. When the driving transistors
are first switched on, R grows linearly at first because the full supply voltage is applied across the
winding terminals. When the current regulator starts to operate, R is also regulated to a constant value
at first because the constant current is being forced into an inductance that is still almost constant at
the low value around the unaligned position, before the poles begin to overlap. As soon as the pole
corners approach conjunction J, the inductance starts to increase, so the flux-linkage R also increases
as constant current is now being forced into a rising inductance. The flux-linkage continues to increase
until the commutation point. After that, the diodes connect a negative “de-fluxing” voltage !Vs across
the winding terminals and therefore R falls to zero very rapidly. In this example the resistive voltage-
drop is small, and therefore the rate of fall of flux-linkage is almost linear. At low speed the dwell is
made approximately equal to $s, since this is the “width” of the “torque zone”, and this angle might
typically be a little less than 30E in a typical 6/4 motor. De-fluxing is completed over only a small angle
of rotation since the speed is low, so the entire conduction stroke occupies only about 30E.
Page 4.24 SPEED’s Electric Motors

The process is summarized in the energy-conversion loop, which fits neatly between the aligned and
unaligned magnetization curves as a result of the selection of the firing angles. It appears that the
energy conversion W could be increased slightly, by retarding the commutation angle to extend the
loop up to the aligned magnetization curve. This would not require any increase in peak current, but
it would increase the average and r.m.s. values. It is also possible that delayed commutation could
incur a period of negative torque just after the aligned position, which would appear as a re-entrant
distortion of the energy-conversion loop, limiting the available gain in torque. Operation is at point M1
in the torque/speed characteristic, Fig. 4.35. It is possible to maintain torque constant with essentially
the same current waveform as the speed increases up to a much higher value, since the motor EMF is
still much lower than the supply voltage.

High-speed motoring — At high speed the motor EMF is increased and the available voltage may be
insufficient for chopping, so that the torque can be controlled only by varying the firing angles of a
single pulse of current. Fig. 4.32 shows a typical example, in which the speed is 1300 rev/min.

Fig. 4.30 High-speed motoring waveforms.

The driving transistors are switched on at 50E and off at 80E, the same as in Fig. 4.29. At first the overlap
between poles is small, and the supply voltage forces an almost linear rise of current di/dt = Vs/Lu into
the winding. Just before the start of overlap the inductance begins to increase and the back-EMF
suddenly appears, with a value that quickly exceeds the supply voltage and forces di/dt to become
negative, making the current fall. The higher the speed, the faster the current falls in this region.
Moreover, for a given motor there is nothing that can be done to increase it, other than increasing the
supply voltage. The torque also falls. Operation is at point M2 in Fig. 4.35.

Operation at much higher speed — At a certain “base speed” the back-EMF rises to a level at which
the transistors must be kept on throughout the stroke in order to sustain the rated current. Any
chopping would reduce the average applied voltage and this would reduce the current and torque. The
“base” speed is marked B in Fig. 4.35. If resistance is ignored, the peak flux-linkage achieved during
the stroke is given by Vs )2/T, where )2 is the “dwell” or conduction angle of the transistors. If the peak
flux is to be maintained at higher speeds, the “dwell” must be increased linearly with speed above the
base speed. At high speed the turn-on angle can be advanced at least to the point where the sum of the
fluxing and de-fluxing intervals is equal to the rotor pole-pitch, at which point conduction becomes
continuous (i.e. the current never falls to zero). This corresponds to a dwell of 45E and a total
conduction stroke of 90E, neglecting the effect of resistance (which tends to shorten the de-fluxing
interval).
Switched reluctance machines Page 4.25

Fig. 4.31 Very high speed motoring

Thus it appears that the dwell or “flux-


building angle” can increase from 30E at low
speed to 45E at high speed, an increase of
50% or 1.5:1. Over a speed range of 3:1, the
peak flux-linkage might therefore fall to
1.5/3 = 0.5, or one-half its low-speed value.
This is illustrated in Fig. 4.31 for a speed of
3900 rev/min. The peak current is
approximately unchanged but the loop area
W is only about one-third of its low-speed
value. The comparison between the loop
areas at 1300 and 3900 rev/min is shown
more clearly in Fig. 4.32. The average
torque is therefore only about one-third of
its low-speed value, but the power remains
almost unchanged. Operation is at point M3
in Fig. 4.35.
Fig. 4.32 Energy-conversion loops at low and high speed, 1300 and
3900 rev/min.

Low-speed generating — Low-speed generating is similar to low-speed motoring except that the firing
angles are retarded so that the current pulse coincides with a period of falling inductance. Fig. 4.33
shows a typical example. The average torque is negative and the energy-conversion loop is traversed
in the clockwise direction. At the start of the stroke, there is a slight positive torque because the
current is switched on shortly before the aligned position, while the inductance is still rising. In this
example the torque falls to zero before the current is commutated, indicating that the commutation
angle could be advanced slightly without reducing the average torque. The reduction in copper loss
would increase the efficiency. During that “tail” period when there is current but no torque, the
current is maintained by the drive which is simply exchanging reactive energy with the DC link filter
capacitor. Operation is at G1 in Fig. 4.35.
Page 4.26 SPEED’s Electric Motors

Fig. 4.33 Low-speed generating waveforms

High-speed generating — High-speed generating is similar to high-speed motoring, except that the
firing angles are retarded so that the current pulse coincides with a period of falling inductance. Fig.
4.34 shows a typical example. The torque is negative and the energy-conversion loop is again traversed
in the clockwise direction. At the start of the stroke, there is a slight positive torque because the
current is switched on a few degrees before the aligned position, while the inductance is still rising.
Operation is at G2 in Fig. 4.35.

Fig. 4.34 Energy-conversion loop: high-speed generating

Operating regions — torque/speed characteristic

For control purposes the torque/speed envelope can be divided into regions as shown in Fig. 4.35.

Constant torque region—The base speed is the maximum speed at which maximum current and
rated torque can be achieved at rated voltage. In this region the torque is controlled by regulating the
current, with relatively minor adjustments in the firing angles as necessary to alleviate noise or
improve the current or torque waveform, or to improve efficiency.
Switched reluctance machines Page 4.27

Fig. 4.35 Torque-speed characteristics

Constant power region—As the speed and back-EMF increase, the dwell is increased to maintain the
peak flux-linkage at the highest possible level. If the dwell is equal to half the rotor pole-pitch and the
de-fluxing angle is negligible at the base speed, then in principle the dwell can be doubled before the
onset of continuous conduction. Therefore if the dwell is increased in proportion to speed, the peak flux-
linkage can be maintained up to about twice the base speed. However, constant power can be
maintained to a higher speed than this, because the loss of loop area dW/dT is compensated by the
increase in speed. If power is taken as TT and T % W, then P % TW and for constant power we require
that )P = T)W + W)T = 0, which says that constant power can be maintained up to the point where
)W/W = !T/)T. In other words, the maximum speed at constant power is the speed at which the rate
of loss of loop area is balanced by the rate of increase of speed. The rate of increase in back-EMF is less
than proportional to the speed, because the current decreases with speed and MR/M2 is reduced. (In the
linear analysis e = iT dL/d2, and i is decreasing while T is increasing and dL/d2 remains constant.

Falling power region—Eventually as the speed increases, the turn-on angle can be advanced no more,
and the torque falls off more rapidly so that constant power cannot be maintained, even though very
high speeds can be attained against a light load. The maximum phase advance depends on the drive
controller. If the turn-on angle is advanced beyond the point where the dwell becomes equal to about
half the rotor pole-pitch, continuous conduction will begin: the phase current never falls to zero and the
energy-conversion loop “floats” away from the origin. As it does so, it moves to a region where the
separation between the aligned and unaligned curves is increased, and the torque per ampere actually
increases. For this reason, operation with continuous conduction is a possible means of increasing the
power density, not only at high speeds but even at low speeds. The increase in copper loss is acceptable
if there is a greater gain in converted power and the machine can withstand the temperature rise. A
similar effect can be achieved with a DC bias winding in 3-phase motors, [Horst, 1995].

Reversibility—Fig. 4.35 shows only two quadrants of the torque/speed characteristic, corresponding
to motoring and generating (or braking). The direction of rotation is the same in both quadrants.
Operation in the opposite direction is symmetrical, provided that the rotor position transducer can
provide the correct reference position and direction sense. The firing angles for motoring in one
direction become generating angles in the reverse direction, at least at low speed. The machine is thus
reversible and regenerative, and able to operate in all four quadrants of the torque/speed diagram.

Multiple-phase operation — To produce torque at all rotor positions the entire 360E of rotation must
be ‘covered’ by segments of rising inductance from different phases, as shown in Fig. 4.16, and the phase
currents must be sequenced to coincide with the appropriate segments. The total torque averaged over
one revolution is usually assumed to be the sum of the torque contributions from each phase. Although
the calculation and control of torque are both referred to one phase, some degree of overlap is required
in practice to minimise notches in the instantaneous torque waveform when the phases are
commutated, and to produce adequate starting torque at all rotor positions.
Page 4.28 SPEED’s Electric Motors

4.10 CURRENT REGULATION

Soft chopping, hard chopping, and conduction modes

At high speed the current is controlled solely by the on/off timing of the power transistor switching,
but at low and medium speeds it is regulated by chopping. This means that the power transistors are
switched on/off, usually at a high frequency compared with the fundamental frequency of the phase
current waveform. The voltage applied to the winding terminals is +Vs if both transistors are on, 0 if
one is on and the other is off, and !Vs if both transistors are off and the phase current is freewheeling
through both diodes. In the zero-volt state the phase current freewheels through one transistor and one
diode. These three conduction modes are shown in Fig. 4.36, and Table 4.3 shows the states of the power
transistors and diodes in the three conduction modes.

Fig. 4.36 Conduction modes

Soft chopping is when only one transistor is chopping. The other transistor remains on, and it is
called the "commutating" transistor because its only function is to steer or commutate the current into
its associated phase winding at the beginning and end of the conduction period. The voltage applied
to the winding switches between +Vs and 0. During the zero-volt period the rate of change of flux-linkage
is very small (in fact it is equal to !Ri), and therefore the current falls slowly. This means that the
chopping frequency and DC link capacitor current can both be greatly reduced for a given current
ripple or hysteresis band (see below).

State Q1 Q2 D1 D2 V
A 1 1 0 0 Vs
B 1 0 0 1 0
C 0 1 1 0 0
D 0 0 1 1 -Vs
TABLE 4.3
TRUTH TABLE FOR THE STATES OF THE TRANSISTORS AND DIODES

Hard chopping is when both transistors are switched on/off together. It generally produces more
acoustic and electrical noise, and increases the current ripple and DC link capacitor current for a given
current ripple or hysteresis band. It is necessary in certain conditions particularly during
regeneration, to prevent loss of control of the current waveform, and of course the final “chop” at 2c at
the end of the conduction period is a hard chop.
Switched reluctance machines Page 4.29

Single-pulse control at high speed

The flux must be established from zero every stroke. Its build-up is controlled by switching both power
transistors on at the turn-on angle 20 and switching them off at the commutation angle 2c. In motoring
operation the dwell )2 = 2c ! 20 is timed to coincide with a period of rising inductance, and in
generating operation with a period of falling inductance. At a sufficiently high speed, the waveforms
of voltage, flux-linkage, current, and idealised inductance are as shown in Fig. 4.30 and 4.31 (motoring)
and Fig. 4.34 (generating). The "idealised" inductance that would be obtained with no fringing and with
infinitely permeable iron has a waveform similar to that of the pole-overlap waveform, and provides
a convenient means for relating the waveforms to the rotor position.

At constant angular velocity T the build-up of flux-linkage proceeds according to Faraday's Law:
2c
1
Rc ' ( Vs ! R i) d2 R0 (4.43)
T 20
where R0 is the flux-linkage pre-existing at 20 (ordinarily zero). Vs is the supply voltage, R is the phase
resistance, and i is the instantaneous current. All impedances and volt-drops in the controller and the
supply are ignored at this stage. Eqn. (4.43) can be written as

TRc ' Vs (1 & u1 ) . 2D (4.44)

where 2D = (2c ! 20)is the dwell and v1 = u1Vs is the mean volt-drop in the resistance and transistors
during 2D. If u1 << 1 the flux-linkage rises linearly. In motoring operation the flux should ideally be
reduced to zero before the poles are separating, otherwise the torque changes sign and becomes a
braking torque. To accomplish this the terminal voltage must be reversed at 2c, and this is usually done
by the action of the freewheeling diodes when the transistors turn off. The angle taken for the negative
voltage to drive the flux back to zero at the "extinction angle" 2q is again governed by Faraday's Law:
2q
1
0 ' Rc ( ! Vs & R i) d 2 (4.45)
T 2c
and this can be written as
T Rc ' Vs (1 u2 ) ( 2q & 2c ). (4.46)

where v2 = u2Vs is the mean volt-drop in the resistance and diodes in the de-fluxing period (2q - 2c). If u2
<< 1 the flux-linkage falls linearly, and at constant speed the angle traversed is nearly equal to the dwell
angle, both being equal to Rc/Vs. The peak flux-linkage Rc occurs at the commutation angle 2c. The total
angle of phase current conduction covers the fluxing and de-fluxing intervals and is equal to
TRc 2 ! u1 u2
2q ! 20 • . . (4.47)
Vs ( 1 u2 )( 1 ! u1 )
If u1 = u2 = 0 this reduces to 2TRc/Vs. The entire conduction period must be completed within one rotor
pole-pitch "r = 2B/Nr, otherwise there will be a ratcheting or pumping effect in which R0 has a series of
non-zero values increasing from stroke to stroke.2 This condition is also called "continuous
conduction". That is, 2q ! 20 # "r. Eqns. (4.44) and (4.47) combine to give the maximum permissible dwell
angle,
1 u2
2D max ' "r . . (4.48)
2 ! u1 u2
If the mean volt-drops u1 and u2 are both approximately the same fraction of Vs, so that v1/Vs = v2/Vs =
u, then eqn. (4.48) reduces to
(1 u)
2D max ' "r . . (4.49)
2

For example, in a symmetrical 6/4 motor the pole-pitch is "r = 90E (360 elecE) and if u = 0 the maximum

2
Note that "r is the angle of rotation between two successive aligned positions.
Page 4.30 SPEED’s Electric Motors

dwell angle is 2D = 45E, giving a total angle of conduction in the phase winding of 90E. But if u = 0.2 the
maximum dwell angle is 54E. In a regular switched reluctance motor the angle of rising inductance is
only "r/2. Ideally the flux should be zero throughout the period of falling inductance, because current
flowing in that period produces a negative or braking torque. To avoid this completely, the conduction
angle must be restricted to "r/2 and the maximum dwell angle is then
"r 1 u
2D # . . (4.50)
2 2
In the 6/4 motor, with u = 0.2 this indicates a maximum dwell angle of 27E (108 elecE) and a conduction
angle of 54E. In practice, larger dwell angles than this are used because the gain in torque-impulse
during the rising-inductance period exceeds the small braking-torque impulse, which generally occurs
in a region when the torque/ampere is low (i.e., near the aligned and/or unaligned positions). This
condition is shown in Figs. 4.30 and 4.31, where the current has a "tail" extending beyond the aligned
position. The torque is negative during this tail period, but it is small.

The turn-on angle in Fig. 4.30 is just after the unaligned position, and the current rises linearly until
the poles begin to overlap. The rising inductance generates a back-EMF which consumes an increasing
proportion of the supply voltage, until at the peak of the current waveform the back-EMF equals Vs.
Subsequently the back-EMF grows greater than Vs because the flux-linkage is still increasing, while the
speed is constant. What was an excess of applied forward voltage now becomes a deficit, and the current
begins to decrease. At the point of commutation the applied terminal voltage reverses, and there is a
sharp increase in the rate of change of current. At the aligned position the back-EMF reverses, so that
instead of augmenting the negative applied terminal voltage, it diminishes it, and the rate of fall of
current decreases. In this period there is a danger that the back-EMF may exceed the supply voltage and
cause the current to start increasing again. It is for this reason that in single-pulse operation,
commutation must precede the aligned position by several degrees. The commutation angle must be
advanced as the speed increases.

Figs. 4.30 and 4.31 also show the importance of switching the supply voltage on before the poles begin
to overlap. This permits the current to grow to an adequate level while the inductance is still low. For
as long as the inductance remains nearly constant, there is no back-EMF and the full supply voltage is
available to force the increase in current. The turn-on angle may be advanced well ahead of the
unaligned position at high speed, even into the previous zone of falling inductance.

Current regulation and voltage-PWM at low and medium speeds

The method of current regulation is a question of the timing of the voltage pulses. Broadly speaking
there are two main methods: current-hysteresis control and voltage-PWM control, but many
variations exist on these basic schemes. The drive circuit is assumed to be the same for both methods,
although several variants of drive circuit have been devised to effect various improvements in the
current waveform control or to reduce the cost of the controller. In both cases there is a “flux-building”
interval from initial turn-on 20 to commutation at 2c, when the flux is built up from zero to its peak
value. This interval is called the “dwell” or “transistor conduction angle”. At 2c both transistors are
switched off, and the freewheeling diodes connect the reverse of the supply voltage to the phase
terminals, causing the flux to decay to zero. The “de-fluxing interval” lasts from 2c to 2q, and in general
is shorter then the fluxing interval.

Voltage PWM (Fig. 4.37): In voltage-PWM, at least one of the two transistors in a phaseleg is switched
on and off at a predetermined frequency fchop, with a duty-cycle D which is interpreted as ton × fchop =
ton/Tchop, ton being the on-time and Tchop = 1/fchop being the period at the switching frequency. In
voltage-PWM there is no closed-loop control of the instantaneous current. The current waveform has
its "natural" shape at all speeds, as though the supply voltage was "chopped down" to the value D × Vs.
However, for safety and protection a current-limiting function is provided such that if the current
reaches a predetermined level iHi, the current will be limited by switching off at least one of the
phaseleg transistors.
Switched reluctance machines Page 4.31

Fig. 4.37 Voltage-PWM waveforms with soft chopping

With soft chopping, Q2 remains on throughout the dwell angle, Fig. 4.36. When Q1 is on, voltage Vs is
connected to the phase winding. When it is off, the winding is short-circuited through Q2 and D2. Q1
is called the "chopping transistor" and D2 the "chopping diode". Q2 is called the "commutating
transistor" and D1 the "commutating diode", because they change state only at the commutation angles
20 and 2c. During the dwell angle the average voltage applied to the phase winding is D × Vs. Again
using u to represent the averaged per-unit effect of volt-drops in the resistance and the semiconductors,
the flux-linkage rise in the dwell period can be equated to the flux-linkage fall in the de-fluxing period:
TRc ' 2D ( D & u ) Vs ' ( 2q & 2c ) ( 1 u ) Vs . (4.51)

This can be rearranged to show that the total conduction angle is


1 D
2q & 20 ' 2D . (4.52)
1 u

To prevent continuous conduction, 2D must be restricted to


1 u
2D < "r . . (4.53)
1 D

For example, in the 6/4 motor, if u = 0.2 and D = 0.5, the maximum dwell is 1.2/1.5 × 90 = 72E. To prevent
any braking torque, 2D must be restricted to
"r 1 u
2D < . , (4.54)
2 1 D
i.e. one-half of the absolute maximum, or 36E in the example. The dwell can be increased as the duty-
cycle is decreased, up to the maximum given by eqn. (4.48) or (4.50).

A similar analysis can be carried out for hard chopping, in which both transistors are switched
together at high frequency. In both soft and hard chopping, the flux-linkage waveform increases in
regular steps with a more-or-less constant average slope. Before the start of overlap, the average slope
of the current waveform is also nearly constant as the linearly-increasing flux is forced into a constant
inductance. Thereafter, the inductance increases more or less linearly while the flux-linkage continues
to rise linearly. Consequently the current tends to become constant or flat-topped. Voltage-PWM tends
to produce quieter operation than current hysteresis control.

The waveforms in Figs. 4.29 and 4.33 show that at low speed, when chopping is the preferred control
strategy, the whole of the absolute torque zone can be used. As is evident from eqn.(4.51), the ratio of
the slopes of the rising and falling parts of the flux-linkage waveform is approximately equal to D, so
that with a low duty-cycle (needed to "throttle" the voltage at low speed), the de-fluxing is accomplished
in a very few degrees, permitting late commutation.

Although the pole arcs do not appear in any of the equations constraining the limiting values of the
firing angles, they are important in determining their optimum values.
Page 4.32 SPEED’s Electric Motors

Fig. 4.38 Architecture of voltage-PWM controller

The duty-cycle is typically set by the outer speed and position control loops, while the firing angles can
be scheduled with speed to optimise efficiency. Fig. 4.38 shows the concept of average torque control
with voltage PWM, with typical voltage and current waveforms as shown in Fig. 4.37. The torque
demand is represented by the duty-cycle command signal D*, which may vary as the torque demand
varies, with consequent variation in the current. Because no attempt is made to control the currents
instantaneously, there is no need for current sensors in individual phases. Voltage-PWM schemes may
therefore be designed with only one current sensor at the DC link for over-current protection.

Current hysteresis (Fig. 4.39): In current hysteresis control, at least one of the two transistors in a
phaseleg is switched off when the current exceeds a specified set-point value iHi. It is switched on again
when the current falls below a second level iLo = iHi ! )i, where )i is called the "hysteresis-band".
Current hysteresis control maintains a generally flat current waveform, as shown in Fig. 4.29 or 4.33,
with ripple determined by )i and the bandwidth of the current-regulator. At high speed, the back-EMF
may prevent the current from ever reaching iHi, and then the current waveform is naturally determined
by the changing inductance and back-EMF as the rotor rotates (this is sometimes called single-pulse
mode). Fig. 4.39 shows the waveforms obtained with a hysteresis-type current regulator and soft
chopping, in which one power transistor is switched off when i > iHi and on again when i < iLo. The
instantaneous phase current i is measured using a wide-bandwidth current transducer, and fed back
to a summing junction. The error is used directly to control the states of the power transistors. Both
soft and hard chopping schemes are possible, but only the soft-chopping waveforms are shown in Fig.
4.39. The waveforms for hard chopping are similar. As in the case of voltage-PWM, soft chopping
decreases the current ripple and the filter requirements, but it may be necessary in braking or
generating modes of operation.

Fig. 4.39 Device current waveforms


Switched reluctance machines Page 4.33

Fig. 4.40 Architecture of current-hysteresis controller

Delta modulation: A variant of the current hysteresis controller is delta modulation in which the
current is sampled at a fixed frequency. If the phase current has risen above the reference current i*
the phase voltage is switched off, and if it has fallen below i* it is switched on. The switching frequency
is not fixed but is limited by the sampling rate.

Soft braking: Soft braking is used for regenerative braking and uses the same zero-voltage loop
principle as soft chopping. Initially both transistors are on and full supply voltage is applied until the
phase current in the winding exceeds a predetermined limit, i > iHi. Both transistors are then switched
off and !Vs is applied until the current falls below iLo. Thereafter one transistor is switched on and off
to regulate the current until the end of the conduction period. The state of the switches alternates
between B and D, or between C and D, in Table 4.4. During the freewheeling state B or C, the flux-
linkage remains approximately constant and at low speed the rate of rise of current is low, so that this
strategy can be used to limit the switching frequency and the current ripple. At high speed, however,
the current rise may be too rapid during the zero-volt periods B and C, and hard chopping may become
necessary, in which the supply voltage Vs is used to suppress the rate of rise of current and the switch
states alternate between A and D. Generated energy is returned to the DC link during state D.

Zero-volt loop: Just as current-hysteresis control can be used to maintain constant current, zero
voltage can be used to maintain constant flux-linkage during part of the stroke. [Miller et al, 1985],
[Gunnar, 1990]. An example is shown in Fig. 4.41. By this means it is possible to reduce the torque
without current chopping and its associated losses. The peak flux is limited to a lower value. The
energy conversion loop still makes good utilization of the available energy. This technique has been
used also for noise reduction. [Horst, 1995].

Fig. 4.41 Zero-volt loop used to achieve an interval of approximately constant flux-linkage.
Page 4.34 SPEED’s Electric Motors

4.11 MATHEMATICAL DESCRIPTION OF CHOPPING

Fig. 4.42 Definition of firing angles

For analysis or computer simulation the firing angles must be defined with respect to a reference value
of rotor position, Fig. 4.42. The rotor position reference is the graph of “per-unit overlap” between the
stator poles of a reference phase and any pair of rotor poles. This graph is periodic with a period of "r,
the rotor pole-pitch, which defines a range of “principal values” of the rotor position 2. It is convenient
to start at the “previous aligned position” and end at the “current aligned position”, A. When 2 is
outside the principal range it can be “reduced” to the principal range by adding or subtracting integral
multiples of "r. In a symmetrical machine the magnetization curves are generally defined only between
the unaligned and aligned positions, and it is convenient to divide the principal range into two sections
AU and UA. If 2 is in AU, it is replaced by A!2 so that the reduced rotor angle is always in the range
UA. A flag ST is set to !1 to represent the sign of the torque when 2 is in the AU range, and +1 when it
is in the UA range. Otherwise all calculations can proceed as if the rotor was in the range UA. If the
machine is not symmetrical, the principal range cannot be divided and must extend over a complete
rotor pole-pitch; also the magnetization curves must be available over this entire range.

Fig. 4.42 also shows the definition of “electrical degrees”. The origin for electrical degrees is the
unaligned position and one electrical cycle is equal to the rotor pole-pitch. Therefore the conversion
from electrical degrees to mechanical degrees is

2elec ' ( 2mech ! U ) × Nr . (4.55)

The interval between unaligned and aligned positions is always 180E(elec). In a 3-phase 6/4 motor with
a pole-arc of 30E, the J position (start of overlap) is at 60E(mech) or (60!45) × 4 = 60E(elec).

Referring to the power circuit in Fig. 4.36, transistors Q1 and Q2 are turned on at 20 and off at 2c. In
current hysteresis control, the applied voltage during the conduction interval 2c ! 20 is Vs. In a
computer simulation, at the end of each integration step the phase current i is compared with iHi. If i
> iHi, Q1 is switched off; otherwise it is switched on. In hard chopping, Q2 is switched off as well as Q1.
When i falls below iLo, the chopping transistor is switched on again. In voltage-PWM, the chopping
transistor is switched on and off at the frequency fchop, with a duty-cycle D. At 2c, Q1 and Q2 are
switched off, and the reverse ("de-fluxing") voltage is !Vs. The voltage equation for one phase is:
Switched reluctance machines Page 4.35

dR
T ' d1[Vs ! Rph i ! 2Rq i ! 2Vq]
d2
d2 [ !Rph i ! Rq i ! Vq ! Vd]
(4.56)
d3 [ !Vs ! Rph i ! 2 Vd]

' (d1 ! d3)Vs ! Rph i ! 2 (d1 d2)(Rq i Vq) ! (d2 2 d3)Vd

where d1 is the duty-cycle with Q1 and Q2 on, d2 with Q1 off, and d3 with Q1 and Q2 off. This equation
caters for all combinations of the states of Q1 and Q2. At each integration step d1, d2 and d3 are assigned
the correct values, and d1+d2+d3 = 1 within each integration step. The compact voltage equation
expressed in this form with d1, d2 and d3 embodies all the switching states and logic required for both
current hysteresis control and voltage- PWM control. In current hysteresis control, d1, d2 and d3 are
scalar values that multiply the voltage terms in the equation. This is the principle of "state space
averaging", and is based on the notion of an upstream chopper controlling the DC source voltage, with
infinite chopping frequency. d1 is either equal to D or 0; d2 is either 1 ! D or 0; and d3 is either 0 or 1. In
voltage-PWM, d1, d2 and d3 are binary states having the value 0 or 1. The states are determined by the
combined states of the transistors and diodes. Only one of the three states d1,d2 and d3 can be non-zero
during one integration step.

The transistor and diode currents and their squares are accumulated in each integration step using iQ1
= d1 × i; iQ2 = (d1+d2) × i; iD1 = d3 × i; iD2 = (d2+d3) × i; and iDC = (d1!d3) × i {= iQ1 ! iD1 = iQ2 ! iD2}. When
the integration is finished, the mean and mean-squared values are calculated from the accumulations
by dividing by the number of steps. This process is the same for both current hysteresis control and
voltage-PWM. An exception to this calculation is the DC link ripple current. This has to be constructed
from the phase-shifted sums of the phaseleg currents, which flow in both directions in the DC link. It
can be constructed from the array of samples of phase current.

4.12 REGULATION ALGORITHMS

Motor control

As already noted at the beginning of this chapter, the regulation algorithm in DC motor drives is based
on the simple linear relationship between torque and current, and the torque demand produced by the
velocity loop is translated directly into a current command by a simple constant of proportionality, the
torque constant kT. A similar principle is implemented in field-oriented AC motor drives. The
translation of the torque demand signal T* into a current command signal i* is the function of the
“feedforward” part of the torque controller, since there is usually no torque transducer and therefore
no torque feedback. In the switched reluctance drive the relationship between torque and current is
not linear. An example is shown in Fig. 4.43.

Fig. 4.43 Typical variation of torque with current


Page 4.36 SPEED’s Electric Motors

The torque also depends on the firing angles 20 and 2c. To complicate matters, 20 and 2c may be required
to vary for reasons other than torque control — for example, to minimize noise or to compensate for
back-EMF at high speed. The result is that the feedforward torque control must usually be implemented
as a mapping from T* to i* with, with additional links to the firing angles and possibly also the supply
voltage. The mapping can be implemented in a digital memory, with interpolation in the processor, or
possibly by equations. In either case, the mapping must be computed or determined by experiment; this
can be a laborious and time-consuming process. Unfortunately the mapping will be specific to a
particular motor and drive, and usually also specific to a particular application.

Fig. 4.44 Graphical representation of look-up table for turn-on angle

Fig. 4.44 shows a graphical representation of a look-up table for the turn-on angle 20 as a function of
speed and torque demand. With single-pulse control the variation of torque with 20 and 2c is equally
complex. The architecture of a single-pulse controller is shown in Fig. 4.45.

For closed-loop control of the phase currents both linear and non linear current regulators may be used
[Kjaer, Gribble and Miller, 1996]. Linear regulators normally use proportional-integral (PI) control to
eliminate the error between the reference and actual currents, and to give a smooth variation of phase
voltage with reduced torque ripple and electromagnetic noise. The main disadvantage stems from the
variation of inductance with rotor position, which can cause the electrical time constant to vary by as
much as 10:1, making it difficult to tune for satisfactory transient performance.

Fig. 4.45 Architecture of single-pulse controller


Switched reluctance machines Page 4.37

Generator control

The switched reluctance machine will regenerate power to the DC supply if the current pulse is timed
to coincide with an interval of falling inductance, and typical generator waveforms are shown in Figs.
4.33 and 4.34. Excitation power is supplied by the DC source when the transistors are both on, and
generated power is returned to the DC source when they are both switched off. The power circuits of
Figs. 4.7 and 4.8 may be used, but several variants have been published, e.g. [Radun, 1994].

In the steady state the switched reluctance machine can sustain itself in the generating mode with the
DC source disconnected, but the DC link capacitor must be retained to provide excitation power during
the “fluxing interval” during the first part of each stroke. Generally the load will be connected in
parallel with the DC link capacitor, and in general its impedance will be variable and not under the
control of the switched reluctance generator controller. Inevitably the DC voltage will decrease during
the fluxing interval, and increase during the de-fluxing interval when power is being returned through
the diodes. The variation or ripple in the DC link (capacitor) voltage depends on the energy conversion
per stroke, the energy ratio, and the capacitance. The controller must maintain the average DC voltage
constant in much the same way as it must maintain constant average torque in motoring mode. This
is equivalent to the maintenance of constant torque, averaged over at least one stroke. Therefore the
architecture of a generator controller can be similar to that of a motor controller. However, the DC link
capacitance has an integrating effect such that it requires lower bandwidth to control the DC voltage
than to control the DC current (or torque). At low speed, therefore, the DC link voltage is controlled by
varying the set-point current i* or the duty-cycle D*, and at high speed by varying the control angles
20 and 2c. As in motoring operation, a control map is required to determine how the control variables
must vary in response to the voltage error )Vd. Various linearising schemes have been presented to
simplify the control, [Radun, 1993], [Kjaer, Cossar, Gribble, Li and Miller, 1994].

Control mode Control variables


Current hysteresis control iHi, iLo
Delta-modulation i*
Voltage-PWM control D
Zero-volt-loop mode iHi, iLo, 2z
Single-pulse control 20, 2c
TABLE 4.4
CONTROL MODES WITH THEIR CONTROL VARIABLES

4.13 OPTIMISATION OF THE CONTROL VARIABLES

Table 4.4 summarizes the different control schemes for torque (in motoring operation) or DC voltage
(in generating operation), and their associated control variables. Average torque can be controlled by
varying any one or indeed all of the control variables in a given mode, but the configuration depends
on the performance requirements, the acceptable level of complexity, and the cost. Since there are
many possible combinations of control variables which produce the same torque, a secondary control
objective is needed to select and define the variation of control variables for optimum performance.
Examples of such secondary control objectives are efficiency, acoustic noise, and torque ripple.
Page 4.38 SPEED’s Electric Motors

4.14 MAGNETIC GEAR RATIO

For a “vernier” machine such as a switched-reluctance motor, normally


strokes / rev ' m Nr (4.57)
where m ' No. of phases and Nr ' No. of rotor poles.

The average electromagnetic torque of the “vernier” motor is


m Nr
Tv ' W (4.58)
2B

where W ' area of i-R loop (energy conversion loop).

A conventional synchronous machine has 2m energy-conversion loops per electrical cycle, and
p electrical cycles per revolution, where p ' No. of pole-pairs. Thus there are 2mp energy-conversion
loops per revolution. Hence the average electromagnetic torque of the conventional machine is
2mp
Tc ' W. (4.59)
2B
The magnetic gear ratio G can be defined as the ratio of the electromagnetic torque of the vernier
machine to that of the conventional machine:

Tv Nr
G ' ' . (4.60)
Tc 2p

By definition, a conventional synchronous machine has


Nr ' 2 p . (4.61)
Therefore for a conventional synchronous machine,
G ' 1. (4.62)
As explained in Ref. [1], the switched reluctance motor can have G > 1. For example, the 3-phase 6/4
motor has Nr ' 4 and m ' 3, giving 4 × 3 ' 12 strokes/rev. The equivalent synchronous machine can be
defined in two ways: with the same number of actual rotor poles or the same number of active magnetic
poles in the flux pattern . In the first case the equivalent synchronous machine would have p ' 2, giving
G ' 4/(2 × 2) ' 1@0. In the second case the number of active magnetic poles in the flux pattern is 2, giving
p ' 1 and G ' 2@0.

The 4-phase 8/6 motor has m ' 4 , Nr ' 6, and 24 strokes/rev. The equivalent synchronous machine can
have either Nr ' 6 poles and p ' 3, giving G ' 1@0; or Nr ' 2 active magnetic poles, giving p ' 1 and G ' 3.

The 3-phase 12/8 motor has m ' 3, Nr ' 8, and 24 strokes/rev. The equivalent synchronous machine can
have either Nr ' 8 poles and p ' 4, giving G ' 1@0; or Nr ' 4 active magnetic poles, giving p ' 2 and G ' 2@0.

The magnetic gear ratio is thus ambiguous unless the definition of the “equivalent conventional
synchronous machine” is stated: in particular, it must be specified whether this machine has the same
number of actual rotor poles or the same number of active magnetic poles in the flux-pattern.
Nevertheless, the average torque is not affected by this ambiguity, being always evaluated by eqn. (58).
It is probably best to choose the number of active magnetic poles in the flux-pattern of the switched-
reluctance motor as the basis for the choice of p in the equivalent conventional synchronous machine,
because the magnetic loading is based on this number, rather than the actual number of rotor poles.
In general the number of active magnetic pole-pairs per phase in the switched-reluctance motor is equal
to Ns/2m, where Ns is the number of stator poles. We can thus write
Switched reluctance machines Page 4.39

D B

A' A

B' D'

C'

Fig. 4.46 3-phase 6/4 SRM Fig. 4.47 3-phase 12/8 SRM Fig. 4.48 4-phase 8/6 SRM

Ns
p ' . (4.63)
2m
Substituting eqn. (63) in eqn. (60),
m Nr strokes/ rev
G ' ' . (4.64)
Ns stator poles

There is another, third, criterion for choosing the number of pole-pairs p of the equivalent conventional
synchronous machine: that is, to make the magnetic frequency the same in both machines. In the
switched-reluctance motor the magnetic frequency is simply Nr, but in the conventional synchronous
machine it is Nr/2 ' p. If we make p equal to the Nr of the switched-reluctance machine, then G is always
1@0. The corollary of this is that any advantage derived from a magnetic gear ratio greater than 1 must
be at the expense of an increase in magnetic frequency. If p is defined on the basis of the number of
active magnetic poles, the magnetic frequency increases by the factor G.

So far the magnetic gear ratio has been considered in terms of torque. In a mechanical gearbox, the
speed ratio is the inverse of the torque ratio. It is therefore of interest to see if the same applies in the
case of the magnetic gear ratio. In the switched reluctance motor the stator excitation pattern
advances ±2B/Ns during one stroke, while the rotor advances 2B/Nr  2B/Ns. (See Figs. 4648). The
speed ratio is thus

Speed of rotor 2 B / Nr  2 B / Ns Ns  Nr
G) ' ' ' . (4.65)
Speed of stator excitation pattern 2 B / Ns Nr

Normally switched-reluctance motors are arranged with


Ns  Nr ' ± 2 p , (4.66)
where p is the number of active magnetic pole-pairs. Substituting eqn. (66) in eqn. (65) , and ignoring
the negative sign, we get
2p
G) ' . (4.67)
Nr
From eqns. (60) and (67), it appears that GNG ' 1, which appears to justify the analogy with a mechanical
gearbox. However, the analogy is artificial because the electric motor does not actually convert or
transform mechanical power from one speed and one torque to another, but voltage and current into
speed and torque. Although the output speed and torque are real, the input speed and torque are
artificial. In fact the “input torque” does not exist. The only identifiable torque other than the
electromagnetic torque acting on the rotor is the reaction on the stator, which by Newton’s third law
is equal and opposite to the torque acting on the rotor (at constant speed). What this means is that the
“magnetic gear ratio” should be interpreted cautiously!
Page 4.40 SPEED’s Electric Motors

It is not correct to say that “switched-reluctance motors produce higher torque than AC motors because
of the magnetic gear ratio”. A more supportable statement is that “switched reluctance motors produce
comparable torque to that of AC motors having the same number of active magnetic poles, by means
of a larger number of energy conversion cycles per revolution”. The energy-conversion loop areas in
the switched reluctance motor are likely to be smaller than in an AC motor, particularly a PM brushless
motor with high-energy magnets, so it can be said that if it were not for the “magnetic gear ratio”, the
switched-reluctance motor would be fairly useless.

In switched-reluctance motors the magnetic gear ratio can be increased by increasing the number of
rotor poles Nr and minimizing the number of active magnetic pole-pairs p. The smallest possible value
of p is 1, and from eqn. (66) this means that Nr ' Ns ± 2. Usually Nr is made equal to Ns  2, because this
gives a lower unaligned inductance and permits the use of wider pole-arcs, both of which increase the
energy-conversion capability W. However, when Ns and Nr are increased, the stator and rotor become
more finely divided, and since inductance is proportional to linear dimension, the difference between
the aligned and unaligned inductances decreases, so that W decreases. At the same time, the magnetic
frequency increases, so the iron losses increase. There may be some merit in having a large number
of poles if the number of phases is also large, because then the torque capability can be more fully
exploited by having multiple phases conducting at the same time (phase overlap). But this is inherently
expensive and would only be useful in special cases.

Permanent-magnet brushless motors

Note that eqn. (61) is completely independent of the number of slots, teeth, and coils. For example, a
12/10 permanent-magnet brushless motor with 12 slots and 10 rotor poles has Nr ' 10 and p ' 5. Likewise
a 30/10 PM brushless motor with 30 slots and 10 rotor poles has Nr ' 10 and p ' 5.

To investigate further, Figs. 4951 show three PM brushless motors, all with Nr ' 10 and p ' 5, and G '
1. The first two have 12 slots and the third has 30 slots. Although all these machines have the same
magnetic gear ratio and the same number of energy-conversion loops per revolution, there are huge
differences between them, in the way in which they generate the energy conversion W.

Motor 1 in Fig. 49 has only 2 coils per phase. Although the lowest-order harmonic in its winding
distribution has 2 poles, the rotor actually locks on to the 5th harmonic of the rotating MMF of the stator
winding. With only 2 working poles on the stator, they must carry a large peak ampere-turns to produce
a satisfactory torque, leading to increased risk of demagnetization in this configuration. Even with
current flowing simultaneously in more than one phase, the utilization of the periphery is poor, in
much the same way as it is in switched -reluctance motors.

Motor 2 has a double-layer winding with twice the number of coils per phase compared to Motor 1.
There is now a phase displacement between the EMF’s in the coils of one phase, so this winding shares
that attribute with a conventional “distributed” winding. In fact a distribution factor can be defined
for it by the conventional theory of winding factors. The coil-pitch is close to the pole-pitch because the
slots/pole ' 12/10, which is close to 1. Consequently the rotor locks on to the 5th harmonic of the rotating
MMF of the stator winding, as it does in Motor 1. Because of this fact, both Motor 1 and Motor 2 have
a high inductance, because the low-order harmonics in the winding distribution contribute massively
to the harmonic leakage or differential leakage inductance. For this reason, these machines are
sometimes favoured for wide speed-range at constant power.

Motor 3 in Fig. 51 has an integral-slot full-pitch winding. The lowest harmonic in its MMF distribution
is the 5th mechanical harmonic: in other words, its fundamental has the same number of poles as the
rotor. Consequently the harmonic leakage is very low. This winding is a “concentrated” winding in
the old-fashioned sense, meaning that all the coil-sides of one pole-group are concentrated in the same
slots. The same is true of Motor 1, which can also be classified as a “concentrated” winding. But the
winding in Motor 2 is electrically more like a distributed winding, even though it has single-tooth non-
overlapping coils (which are often erroneously called “concentrated” without regard to the distribution
of the other phase coils).
Switched reluctance machines Page 4.41

Fig. 4.49 12/10 motor with single-layer Fig. 4.50 12/10 motor with double-layer Fig. 4.51 30/10 motor with double-layer
winding winding winding

Magnetic Gear Ratio and Electric and Magnetic Loading

In Ref. [1] the magnetic gear ratio of eqn. (60) is derived by a more involved process involving the
electric and magnetic loadings, which are defined for a switched-reluctance motor in relation to the
energy conversion loop area W and its typical shape. The definition of magnetic and electric loadings
for different machines is quite involved, especially when precise definitions are required for making
comparisons between different types of machine, and even for comparing machines of the same basic
type (as in Figs. 4951). The magnetic loadings of Motors 1,2 and 3 are clearly identical, because they
all have the same rotor. But the electric loading depends on the amount of copper and the permissible
current-density, and on thermal factors and performance factors (such as inductance) which depend
on the shapes and sizes of the slots. Moreover, as has already been mentioned, the three windings are
not equivalent in terms of the risk of demagnetization, which adds to the complexity of the comparison.
Yet again, the motors are not equivalent in terms of their cogging torque, which is much higher in
Motor 3.

4.15 REFERENCES

See Miller TJE (Ed.), Electronic control of switched reluctance machines, Newnes, 2001, ISBN 0 7506 50737
and Miller TJE, Switched reluctance motors and their control, Oxford University Press/Magna Physics
Publishing, 1993, ISBN 1-881855-02-3 (available from Motorsoft). All references in brackets are listed
in this book, together with many others.
Page 4.42 SPEED’s Electric Motors

Index

Acoustic noise 14, 37 Linear analysis 5, 27


Aligned 3, 4, 6-8, 10-13, 15-20, 22-27, 29, 30, 34, 40 Look-up table 36
Average torque 4, 11, 12, 17, 22, 25, 32, 37, 38 Low-speed generating 25, 26
Back-EMF 5, 7-9, 11, 16, 20-22, 24, 27, 30, 36 Low-speed motoring 23, 25
Braking 4, 27, 29-33 Magnetization curves 9, 10, 18-21, 24, 34
Brushless DC motor 22 Motor 3, 6-14, 16, 17, 20-24, 29-31, 34-41
Byrne 16, 17 Multiple-phase operation 27
Capacitor 25, 28, 37 Multiplicity 14
Chopping 5-8, 11, 23, 24, 28, 30-35 Nonlinear analysis 9
Classical machines 22 Number of strokes per revolution 11, 12
Coenergy 9-11, 20, 21 Overlap ratio 13
Commutation angle 16-18, 24, 25, 29, 30 PC-SRD 19, 20
Constant power 17, 27, 40 Pole arc 3, 13
Constant torque 26, 37 Pole numbers 13
Control 6, 7, 13, 21, 22, 26-37, 41 Ripple 9, 13, 28, 32, 33, 35-37
Converter volt-ampere requirement 15 Saturation 5, 9, 10, 16
Current hysteresis 31-35, 37 Single-pulse 29, 30, 36, 37
Current regulation 22, 28, 30 Soft chopping 28, 30-33
Definition 3, 10, 34, 38, 41 Stator/rotor pole number 15
Drive circuit 6, 21, 22, 30 Stator/rotor pole numbers 13
Electromagnetic torque 5, 10, 11, 17, 20, 23, 38, 39 Stepper motor 3
Energy conversion 7-9, 11, 15-18, 24, 33, 37, 38, 40, Stepper-motor 6
41 Stored field energy 8, 10
Energy ratio 7-10, 15, 16, 18, 37 Stroke 5-9, 11-14, 18, 22-26, 29, 33, 37, 39
Energy-conversion diagram 11 Stroke angle 12-14
Energy-conversion loop 18, 23-27, 40 Strokes/rev 13, 14, 38
Equivalent circuit 9 Suppression voltage 6
Euler’s method 20 Switching frequency 13, 30, 33
Finite-element 19-21 Synchronous reluctance 3
Firing angles 22, 24-27, 31, 32, 34, 36 Torque 3-14, 17, 20-27, 29-41
Flat-topped current 8, 11 Torque zone 8, 13, 17, 23, 31
Gear ratio 38-41 Torque/speed characteristic 17, 24, 26, 27
Half-bridge phaseleg 6 Turn-on angle 24, 27, 29, 30, 36
Hard chopping 28, 31-34 Unaligned 3, 4, 7, 11, 12, 14-20, 23, 24, 27, 30, 34, 40
High-speed generating 26 Vernier 13, 38
High-speed motoring 24, 26 Voltage equation 5, 20, 34, 35
Inductance 3-5, 7, 9, 13, 14, 16, 17, 19, 20, 23-27, Voltage PWM 22, 30, 32
29-32, 36, 37, 40, 41 Zero-volt 28, 33, 37
Instantaneous torque 11, 20, 22, 27
5. Commutator machines

5.1 Permanent-magnet DC commutator machines . . . . . . . . . . . . . . . . . . . . . . . . . . . . . . . . . . . . . . . . . . 1


5.1.1 Electric circuit model . . . . . . . . . . . . . . . . . . . . . . . . . . . . . . . . . . . . . . . . . . . . . . . . . . . . . . . . 1
5.1.2 Torque . . . . . . . . . . . . . . . . . . . . . . . . . . . . . . . . . . . . . . . . . . . . . . . . . . . . . . . . . . . . . . . . . . . . . 2
5.1.3 Efficiency . . . . . . . . . . . . . . . . . . . . . . . . . . . . . . . . . . . . . . . . . . . . . . . . . . . . . . . . . . . . . . . . . . . 2
5.1.4 Torque/speed characteristic . . . . . . . . . . . . . . . . . . . . . . . . . . . . . . . . . . . . . . . . . . . . . . . . . . 2
5.1.5 kT and kE . . . . . . . . . . . . . . . . . . . . . . . . . . . . . . . . . . . . . . . . . . . . . . . . . . . . . . . . . . . . . . . . . . . 3
5.1.6 Operating characteristics . . . . . . . . . . . . . . . . . . . . . . . . . . . . . . . . . . . . . . . . . . . . . . . . . . . . . 8
5.1.7 Time constants of DC motors . . . . . . . . . . . . . . . . . . . . . . . . . . . . . . . . . . . . . . . . . . . . . . . . . 8
5.1.8 Resistance and inductance . . . . . . . . . . . . . . . . . . . . . . . . . . . . . . . . . . . . . . . . . . . . . . . . . . . 10
5.1.9 Magnetic circuit analysis . . . . . . . . . . . . . . . . . . . . . . . . . . . . . . . . . . . . . . . . . . . . . . . . . . . . 11
5.1.10 Demagnetizing effect of armature reaction . . . . . . . . . . . . . . . . . . . . . . . . . . . . . . . . . . . . 13
5.1.11 Torque per rotor volume . . . . . . . . . . . . . . . . . . . . . . . . . . . . . . . . . . . . . . . . . . . . . . . . . . . . 13
5.1.12 Armature windings . . . . . . . . . . . . . . . . . . . . . . . . . . . . . . . . . . . . . . . . . . . . . . . . . . . . . . . . . 14
5.1.13 Winding examples . . . . . . . . . . . . . . . . . . . . . . . . . . . . . . . . . . . . . . . . . . . . . . . . . . . . . . . . . . 17
5.1.14 Distribution of ampere-conductors . . . . . . . . . . . . . . . . . . . . . . . . . . . . . . . . . . . . . . . . . . . 18
5.1.15 Third brush . . . . . . . . . . . . . . . . . . . . . . . . . . . . . . . . . . . . . . . . . . . . . . . . . . . . . . . . . . . . . . . . 20

5.2 Wound-field commutator machines . . . . . . . . . . . . . . . . . . . . . . . . . . . . . . . . . . . . . . . . . . . . . . . . . . . 22


5.2.1 Universal motors . . . . . . . . . . . . . . . . . . . . . . . . . . . . . . . . . . . . . . . . . . . . . . . . . . . . . . . . . . . 22
5.2.2 Equivalent circuit for DC operation . . . . . . . . . . . . . . . . . . . . . . . . . . . . . . . . . . . . . . . . . . . 24
5.2.3 Rotational emf — waveform method . . . . . . . . . . . . . . . . . . . . . . . . . . . . . . . . . . . . . . . . . . 27
5.2.4 The airgap flux-density distribution . . . . . . . . . . . . . . . . . . . . . . . . . . . . . . . . . . . . . . . . . . 30
5.2.5 Generating the E/I curves . . . . . . . . . . . . . . . . . . . . . . . . . . . . . . . . . . . . . . . . . . . . . . . . . . . 31
5.2.6 Equivalent circuit for AC operation . . . . . . . . . . . . . . . . . . . . . . . . . . . . . . . . . . . . . . . . . . . 33
5.2.7 Phasor diagram . . . . . . . . . . . . . . . . . . . . . . . . . . . . . . . . . . . . . . . . . . . . . . . . . . . . . . . . . . . . 35
5.2.8 Iron losses . . . . . . . . . . . . . . . . . . . . . . . . . . . . . . . . . . . . . . . . . . . . . . . . . . . . . . . . . . . . . . . . . 36
5.2.9 Commutation . . . . . . . . . . . . . . . . . . . . . . . . . . . . . . . . . . . . . . . . . . . . . . . . . . . . . . . . . . . . . . 38
5.2.10 Effect of short-circuited coils undergoing commutation . . . . . . . . . . . . . . . . . . . . . . . . . 39
References . . . . . . . . . . . . . . . . . . . . . . . . . . . . . . . . . . . . . . . . . . . . . . . . . . . . . . . . . . . . . . . . . . . . . . . . . 40
Index . . . . . . . . . . . . . . . . . . . . . . . . . . . . . . . . . . . . . . . . . . . . . . . . . . . . . . . . . . . . . . . . . . . . . . . . . . . . . 41
SEM 5 — ii
5.1 Permanent-magnet DC commutator motors Page 5.1

5. COMMUTATOR MACHINES
5.1 Permanent-magnet DC commutator machines
Permanent-magnet DC commutator machines are widely used in automotive auxiliaries (for example,
windscreen wipers, heater blowers, cooling fans), where the supply is low-voltage DC at 12 or 24V.
Because of their excellent control characteristics they are also used in low-power motion-control
applications, where they can be controlled with low-cost electronic drives such as choppers and phase-
controlled converters.1 The theory of the DC machine is also fundamental in the understanding of many
other types of motor and drive system.
5.1.1 ELECTRIC CIRCUIT MODEL
The DC motor model comprises an electrical circuit model and a magnetic field model. The electric
circuit model is shown in Fig. 5.1, in which V is the terminal voltage. It is determined by the supply,
which can be a DC voltage source such as a battery, or a chopper, or a phase-controlled rectifier.

Fig. 5.1 Equivalent circuit.

The motor model is represented by a single differential equation,


dia
V ' ea ( Ra Rleads ) ia La 2 Vb (5.1)
dt
in which ia is the instantaneous armature current, ea is the instantaneous back-EMF, Vb is the volt-drop
in each brush, Ra is the armature resistance, Rleads is the resistance of the leads, and La is the armature
inductance. With pure DC, dia/dt = 0 and the instantaneous quantities become constant DC quantities,
ia = Ia and ea = Ea. The circuit equation then reduces to Ia = (V ! Ea ! 2Vb)/(Ra + Rleads). With a chopper
or phase-controlled rectifier drive, the differential equation (1) must be solved as described in Ref. [6].
The parameters Ea, Ra and La can be calculated from the motor geometry, the winding details, and the
properties of the steel and magnet materials specified via the materials database. Ea is obtained from
the magnetic circuit model in the classical form

E a ' k E Tm (5.2)

where the back-EMF constant kE is given by


p Z Mg
kE ' . (5.3)
aB
The flux/pole Mg is calculated in the magnetic circuit model. Z is the total number of armature
conductors, p is the number of pole-pairs, and a is the number of parallel paths. Tm is the angular
velocity in rad/sec, equal to 2B × RPM/60.

1
Large DC machines generally have field windings. They have traditionally been used in rail traction and high-power motion
control applications such as steel rolling mills, elevators, excavators, etc. Many of these now use AC motor drives.
Page 5.2 SPEED's Electric Motors
5.1.2 TORQUE
In S.I. units the back-EMF constant kE is equal to the torque constant k T, which is used to calculate the
electromagnetic torque Te = kT ia. The torque constant is one of the most important parameters of a DC
motor and is usually quoted in catalogue data. It can be measured by running the machine on a
dynamometer and plotting the torque vs. current, then measuring the slope of the resulting graph. The
EMF constant kE can be measured by driving the machine with an auxiliary motor and measuring the
generated voltage. Generally this has a linear variation with speed. The measured k T is often less than
kE because of friction, and at high current the armature reaction distorts the magnetic flux and reduces
k T still further. In spite of the "saturation" of k T at high current, the linearity of Te vs. ia is an important
characteristic of the PM DC motor, because it ensures good behaviour in control systems.
The shaft torque T is equal to the electromagnetic torque minus the rotational loss torque Trot, which
includes the windage and friction loss. The iron loss is also often lumped in Trot.
T ' kT ia ! Trot Nm . (5.4)

5.1.3 EFFICIENCY
The mechanical power output is TmT and the electrical input power is VIa, so the efficiency is

output power Tm T
0 ' ' × 100% . (5.5)
input power V Ia

2
The copper loss is calculated from I Ra, using the r.m.s. value of Ia. The iron loss WFe can be calculated
using the modified Steinmetz equation, taking into account the variation of the flux-density in the teeth
and the rotor core as the rotor rotates.

5.1.4 TORQUE/SPEED CHARACTERISTIC


We can now derive an equation relating the speed Tm to the shaft torque T :
Ea V ! 2 Vb ! Ra Ia V ! 2 Vb Ra
Tm ' ' ' ! ( T Trot ) (5.6)
kE kE kE kE2
or Ra
Tm ' T0 ! T (5.7)
kE2
where T0 is the no-load speed:

V ! 2 Vb Ra
T0 ' ! Trot . (5.8)
kE kE2

The torque/speed characteristic given by eqn. (8) has the form shown in Fig. 5.3 in normalized form.
With a fixed supply voltage, normal operation at rated load would be at a speed slightly below the no-load
speed, preferably in the neighbourhood of maximum efficiency.
The speed can be varied by introducing series resistance, which steepens the negative slope of the
torque/speed characteristic. This is inherently wasteful. If, for example, the series resistance is
sufficient to reduce the voltage at the motor terminals to 50% of the supply voltage, then the power
dissipated in the series resistance must be equal to the entire input power to the motor, so the overall
efficiency must necessarily be less than 50%.
A better way to control the speed is by varying the supply voltage V, which varies the no-load speed T0
and therefore raises or lowers the whole speed curve. DC choppers can be used for this when the supply
is DC; or phase-controlled rectifiers when the supply is AC.
5.1 Permanent-magnet DC commutator motors Page 5.3

Fig. 5.2 Open-circuit measurement of EMF constant k EG

5.1.5 KT AND KE

The torque constant k T and the EMF constant k E are widely used in connection with permanent-magnet
DC motors, and they are frequently included in published catalogue data.
In the equivalent circuit of the DC commutator machine, the armature is
represented by the symbol shown in Fig. 3. The conversion from
electromagnetic to mechanical power is entirely characterized by this
symbol. The electromagnetic torque T e is expected to be proportional to
the armature current I a , so

Te ¤ kT I a . (5.9)
The EMF E generated in the armature is expected to be proportional to the
speed: thus
E ¤ kE T , (5.10)

where T = 2BN/60 is the angular velocity in rad/s, and N is the rotor speed Fig. 5.3 Armature
in rpm.
The energy conversion process in this model is time-invariant and lossless, and if the values of k E and
k T are assumed constant, then
E I a ¤ Te T . (5.11)

The left-hand side EI a is the instantaneous electrical power entering the idealized armature in Fig. 3,
while T e T is the instantaneous mechanical power. If we substitute eqns. (9) and (10) in eqn. (11), we get

kE ¤ kT . (5.12)
This equality is fundamental to the idealized armature model in Fig. 3. If it is not satisfied, there will be
an imbalance between the electrical and mechanical power, contravening the law of conservation of
energy.
Page 5.4 SPEED's Electric Motors
The equality of k E and k T is apparent only when they are expressed in consistent units, usually S.I. units
of [V-s/rad] and [Nm/A]. However, other units are commonly used. For example the EMF constant is
often expressed in [V/krpm] (volts per thousand rpm), and we can write this as
1000
KE ' kE ¤ 104@72 kE V/krpm . (5.13)
30 / B
The torque constant is often expressed in oz-in/A, and we can write this as
KT ¤ 141@6 kT oz-in A . (5.14)
Then

104@72 KT
KE ¤ KT ¤ 0@7396 KT ¤ . (5.15)
141@6 1@352

Engineering units may mask the essential equality of k E and k T . In practice, this is often the case, for
reasons that will be explored below. In what follows, only the S.I. values k E and k T will be used.

Measurement of kE — The obvious way to measure k E is to drive the motor with another machine at
a known speed, and measure the EMF directly with a voltmeter that measures the true mean value. If the
ripple is sufficiently small, an RMS-reading instrument will give a satisfactory result, but it is the mean
value that interacts with the true mean value of armature current I a in the time-averaged energy-
conversion process. An example of this measurement is shown in Fig. 2.
As shown in Fig. 2, the open-circuit EMF is nearly exactly proportional to the speed, so it is a good
approximation to take a constant value for k E . To identify this value with the open-circuit generating
test, it will be assigned the symbol k EG . It is also sometimes labelled k b , the “back-EMF constant”.
In the open-circuit generating test, the flux is entirely determined by the permanent magnet. There is
no current, so there is no “armature reaction” to distort the flux, and it is also reasonable to assume zero
voltage-drop in the brushes. Because the magnet flux is temperature-dependent, the magnet temperature
should always be recorded in this test.

Measurement of kT — It is possible to measure k T in a special rig with the rotor stationary, but the most
usual practical method is based on the normal dynamometer load test in which measurements are taken
of terminal voltage, current, speed, and shaft torque.
Typical current/torque and speed/torque graphs are shown in Figs. 4 and 5 respectively. It is
straightforward to extract k T directly from Fig. 4 as the inverse of the slope of a straight-line fit.

Reconciliation of kT with kE — With measurements of k T and k EG , it should be possible to reconstruct


the speed/torque graph in Fig. 5. To do this it is necessary to set out the full equivalent circuit and its
associated equations, from which an equation for the speed/torque graph can be derived.
The full equivalent circuit is shown in Fig. 6 on p. 6. The DC source is represented by its open-circuit
voltage V s and internal resistance R s . The armature resistance is R a and the combined voltage-drop in
the two brushes is denoted V b . The shaft torque T sh is less than the electromagnetic torque T e by the
friction torque T Fric :
Tsh ¤ Te £ TFric . (5.16)

The friction torque is assumed to have a static component T f0 and a speed-dependent component, so
TFric ¤ Tf 0 Tf 1 T . (5.17)
The speed-dependent term can represent the combination of windage and armature core-loss, while the
constant term more closely represents bearing and brush/commutator friction.
5.1 Permanent-magnet DC commutator motors Page 5.5

Fig. 5.4 Typical current-vs-torque characteristic

Fig. 5.5 Typical speed-vs-torque characteristic


Page 5.6 SPEED's Electric Motors

Fig. 5.6 DC motor equivalent circuit

The shaft torque is thus given by

Tsh ¤ kT Ia £ Tf 0 £ Tf 1 T . (5.18)

The current is given by


Vs £ Vb £ kE T
Ia ¤ . (5.19)
Ra Rext
Let
Ra Rext
K ¤ rad s Nm . (5.20)
kT kE
and let
Vs £ Vb
T0 ¤ rad s . (5.21)
kE
The value of k E is necessarily taken equal to k T , as explained earlier. Then eqn. (19) becomes
T0 £ T
Ia ¤ A . (5.22)
kT K

If we substitute this expression for Ia into eqn (18) and rearrange, we get this expression for T :

T0 £ K ( Tsh Tf 0 )
T ¤ . (5.23)
1 ¢ K Tf 1
This equation describes the speed/torque characteristic. Note that the slope of this characteristic is
dT £K
¤ rad s Nm . (5.24)
d Tsh 1 ¢ K Tf 1

If the friction torque is constant, T f1 = 0 and the slope becomes


dT
¤ £ K rad s Nm . (5.25)
d Tsh

Eqn. (21) is often quoted as the no-load speed, usually without Vb. However, a more precise value can be
obtained from eqn. (23) by setting T sh = 0 :
T0 £ K Tf 0
T ¤ rad s . (5.26)
1 ¢ K Tf 1
5.1 Permanent-magnet DC commutator motors Page 5.7
Again if T f1 = 0 (constant friction torque), we get a simplified equation

T ¤ T0 £ K (Tsh ¢ Tf 0 ) rad s . (5.27)


It is helpful to write this in the commonly-used form
Vs £ Vb
T ¤ £ K Tf 0 £ K Tsh rad s . (5.28)
kE
If V b and T f0 are both negligible, this becomes
Vs
T ¤ £ K Tsh rad s . (5.29)
kE
The expressions in square brackets represent the no-load speed, when T sh = 0.
If R ext is negligible and k E = k T , then
Ra
K ¤ rad s Nm . (5.30)
kT2
These equations permit the reconstruction of the speed/torque characteristic with different levels of
approximation. The most detailed version is eqn. (23) together with eqn. (21). Note carefully that eqn.
(21) only gives an apparent no-load speed: the true no-load speed is given by eqn. (26).
The process of reconstructing the speed/torque characteristic will naturally begin with the estimation
of k T from Fig. ? on p. ?. If this characteristic is not perfectly straight, it is recommended to take the slope
at low values of current so that the resulting value of k E (set equal to k T ) is more likely to agree with k EG .
At the same time this will produce a better estimation of the no-load speed, which by definition is a low-
current condition.
It will often be the case in practice that the current/torque characteristic is not perfectly straight. At
higher currents there is the well-known phenomenon of “k T -roll-off” where the torque per ampere
decreases as a result of saturation (mainly in the armature teeth). This is not the only possible cause of
nonlinearity of the current/torque characteristic, and it may be decided to publish a value of k T that
more faithfully reproduces the average behaviour over the whole range of currents.

Summary
It may be decided to publish the value of k E as k EG from the open-circuit generating test, with good
reason, because this may be the most likely test that a customer will use to check the motor. If this is
done, eqn. (21) or the square-braketed term in eqn. (28) would be expected to give a satisfactory value for
the no-load speed.
It may be decided to publish the value of k T as an average over the whole current/torque range, as a
matter of expediency in stating how much torque will be available with a given current.
If k T and k E are unequal (in S.I. units), the idealized armature in the DC motor model in Figs. 3 and 6 will
contravene the law of conservation of energy, and it cannot be expected that the speed/torque
characteristic will be correctly calculated unless compensating adjustments are made to other
parameters such as R a , V b , T f0 , and T f1 . These compensating adjustments cannot be guaranteed to correct
the power-conversion imbalance associated with unequal k E and k T . It should also be pointed out that
these parameters may need to be adjusted even when k E = k T .
Strictly speaking, it must further be admitted that variable saturation will lead to a variable value of k T ,
and with it, a variable value of the equivalent k E that is made equal to it in order to satisfy the law of
conservation of energy. However, it is doubtful if such complexity can be justified in practical cases, and
even the enforcement of equality between k T and k E would seem to be rare in practice.
Page 5.8 SPEED's Electric Motors

Fig. 5.7 Torque/speed characteristics of PM DC motor

5.1.6 OPERATING CHARACTERISTICS


Normalized (i.e., per-unit) operating characteristics at constant voltage are shown in Fig. 5.2. There is a
peak efficiency and a peak output power, occurring at different loads. The load torque at which the peak
efficiency occurs depends on the way in which the losses vary with torque and speed.
Maximum efficiency — Suppose the total power loss comprises a fixed loss W1 and a variable loss W2
2
= kT , i.e., the variable loss varies with the square of the torque (effectively, with the square of the
current since torque is proportional to current). The efficiency 0 is given by
Tm T
0 ' (5.31)
Tm T W1 k T 2

and if this is differentiated and d0/dT set to zero, the condition for maximum efficiency is found as W1
= W2 : i.e., the maximum efficiency occurs when the variable loss is equal to the fixed loss.

5.1.7 TIME CONSTANTS OF DC MOTORS


In the steady state, assuming no load and neglecting rotational losses, the speed is given by

V
Tm ' . (5.32)
kE
This steady-state equation gives no indication as to how the speed varies dynamically if the supply voltage
V is changed, as it might be by the action of the electronic controller. The dynamic response can be
determined from the transfer function between speed and voltage, which is derived as follows from the
equations that describe the system. These equations are:
V ' Ra Ia kETm (5.33)

where J = Jm + JL, is the total inertia of Te ' kT Ia (5.34)


motor + load, and " is the angular
acceleration, dTm/dt. dTm
Te ' J " ' J . (5.35)
dt
5.1 Permanent-magnet DC commutator motors Page 5.9
Assuming that the speed is zero at time t = 0, in terms of Laplace transforms,

" ' s Tm (5.36)


so that

Te kT Ia kT (V & kETm)
Tm ' ' ' . (5.37)
Js Js Ra J s

This can be rearranged to give the transfer function between speed and voltage as

Tm (s) 1/kE 1/kE


' ' .
V (s) Ra J 1 s Jm (5.38)
1 s
kT kE
The denominator has only one root, so the transfer function has only one pole at s = !1/Jm, and the system
is said to be a first-order system. Jm is called the mechanical time constant:
Ra J
Jm ' . (5.39)
kT kE
After a disturbance such as a step-change in voltage, the speed will settle exponentially to a final steady-
state value, because the transfer function is of the form 1/(s + a), which has an inverse transform that is
a decaying exponential function of time. After an infinite time, in the steady-state all derivatives are zero
and the corresponding condition in the frequency domain is s = 0. If this is substituted in eqn. (18) the s-
term in the denominator disappears and the transfer function degenerates to the steady-state expression
1/k E, as in eqn. (12). This value is called the gain of the system.

Effect of inductance — the electrical time-constant


The simple dynamic motor model can be extended to include the effects of inductance as follows. The
electrical equation of the motor is

dIa
V ' La Ra Ia kETm . (5.40)
dt

where V is the terminal voltage and brush volt-drops are neglected. By Laplace transformation,

V ( s ) ' ( La s Ra ) Ia ( s ) kETm( s ). (5.41)

The mechanical equation of the motor is

dTm
Te ' J D Tm Trot TL . (5.42)
dt
where DTm is an additional "viscous" torque sometimes described as a damping torque that is
proportional to speed. Of most interest is the transfer function between speed and voltage, and the
simplest case is at no load, i.e. with TL = 0. If we also neglect Trot, then by Laplace Transformation,
From equations (5.14),(5.21) and Te( s ) ' ( J s D ) Tm( s ) (5.43)
(5.21),

( V ! kETm( s ) ) kT
Te( s ) ' ' ( J s D ) Tm ( s ) . (5.44)
La s Ra
Page 5.10 SPEED's Electric Motors
From this the transfer function between speed and voltage can be extracted as

Tm(s) kE
' . (5.45)
V ( La s Ra ) J s D kE kT

The poles of the system are the roots of the denominator of eqn. (25), which is quadratic. There are
therefore two roots, and it is common to write eqn. (25) in a form which separates these two roots: thus

Tm(s) 1/kE
G(s) ' ' (5.46)
V(s) ( s Je 1 ) ( sJm 1 )

where Je is the electrical time constant and Jm is the mechanical time constant. As before, 1/kE is the system
gain. Assuming no motor damping (D = 0) and low inductances, the time constants are

La Ra J
Je . , Jm . . (5.47)
Ra kE kT

5.1.8 RESISTANCE AND INDUCTANCE


The armature resistance Ra is calculated from the winding details at the operating temperature:

Z × LT / 2 a 2
Ra ' DCu × (5.48)
Nsh × B dCu2 / 4

where Nsh is the number of strands-in-hand in each conductor, dCu is the strand diameter, DCu is the
resistivity of copper, and LT is the mean length of turn.
The armature inductance La is also calculated from the winding details and the motor geometry. This
calculation is considerably more complex than the resistance calculation, and the theory is given in full
in Ref. [8]. Basically there are three components of armature inductance,

La ' Lslot Lgap Lend , (5.49)

where Lslot is the slot-leakage component, Lgap is the airgap component, and Lend is the end-winding
component. The airgap component Lgap is liable to be affected by frame saturation especially in 2-pole
motors, or where the frame is very thin. Therefore, for work requiring very accurate values of La it might
be advisable to use finite-element calculations to check the value.
The inductance is important for two particular reasons. One is in the formulation of the electrical time-
constant Je = La /Ra , which is sometimes important in determining the dynamic response, especially for
servo-type applications. The other is in the calculation of the current waveform when the motor is driven
from a chopper or phase-controlled rectifier. In these cases the current waveform may be sensitive to the
value of La, and if an accurate value is not used, the current and the torque will be calculated incorrectly.
5.1 Permanent-magnet DC commutator motors Page 5.11
5.1.9 MAGNETIC CIRCUIT ANALYSIS
The flux produced by the magnet at no-load has the general form shown in Fig. 5.3. The magnet flux Mm
can be separated into two components: the larger of these is the airgap flux Mg = f LKG × Mm, which crosses
the airgap and links the armature winding. The fraction f LKG is called the leakage factor, and a typical
value is 0.9. The leakage flux component (1 ! f LKG)Mm does not link the armature winding.

Fig. 5.8 No-load flux distribution in PM DC motor


Fig. 5.4 shows a graph of the airgap flux-density under one pole at no-load. The peak value is Bg and the
airgap flux per pole Mg is taken to be the integral of B over 180Eelec, over the whole armature length. If
$M is the magnet arc in Eelec, we can get a very rough estimate of the average airgap flux-density B = Bg
× $M/180. Then Mg = BAg, where Ag = Larm × BDarm/2 p is the airgap area per pole, Larm being the armature
length and 2 p being the number of poles. Darm is the rotor diameter.

Fig. 5.9 Flux-density waveform under one pole at no-load

A more detailed analysis must include the leakage flux and the iron teeth, the rotor core, and the frame.
For this purpose a lumped-parameter equivalent-circuit is used, Fig. 5.5. The permeances in this model
correspond roughly to the main flux-paths in Fig. 3. They are calculated from the geometry and from the
material characteristics. The iron parts are non-linear, so the solution method is iterative.
Under load conditions the armature current produces a MMF distribution ("armature reaction") that
distorts the airgap flux-density waveform. This effect is less severe in PM motors than in wound-field
machines. For accurate analysis of the magnetic field, it is desirable to use a finite-element analysis.
Page 5.12 SPEED's Electric Motors

Fig. 5.10 Magnetic equivalent circuit

Magnet operating point


The magnet operates at a point P on the recoil line shown in Fig. 5.7, where Bm = Mm/Am. The recoil line
is described by the equation
Bm ' Br μrec μ0 Hm , (5.50)

where μrec is the relative recoil permeability and Br is the remanence. The magnetic circuit must be
designed so that the operating point (Hm,Bm) lies on the straight part of this line, to avoid irreversible loss
of magnetization. Fig. 5.7 also shows the non-linear effect of saturation on the load line OP.
The slope of the load line is known as the permeance coefficient PC, and
PC
Bm ' Br × . (5.51)
μrec PC

Fig. 5.11 Magnet operating point P at the intersection of the load line and the recoil line, which is obtained from the B-H curve.
The H-axis is scaled by μ0 so that both axes are in [T] and a recoil line with μrec = 1 has a slope of 1.
5.1 Permanent-magnet DC commutator motors Page 5.13
5.1.10 DEMAGNETIZING EFFECT OF ARMATURE REACTION
The armature ampere-conductors are distributed uniformly around the rotor in blocks of alternating
polarity, such that the ampere-conductors under one magnet pole are all in the same direction, Fig. 5.8.
Under loaded conditions the trailing pole tips of the magnets are subject to a demagnetizing MMF Fd :
Z Ia / a $M
Fd ' × . (5.52)
2p × 2 180

where p is the number of pole-pairs and $M is the magnet arc in Eelec. The effect of Fd on the magnet
operating point can be calculated by combining eqn. (33) with Ampère’s Law,
Hm LM Hg g ' F d (5.53)
where g is the effective airgap length. Assuming that Bm . Bg = μ0 Hg,
g
±Fd ! Br
μ0 (5.54)
Hm ' .
LM μrec g

The positive value of Fd applies at the leading pole-tip and the negative value at the trailing pole-tip. The
demagnetization current Idemag can be extracted from eqns. (5.30) and (5.32) after setting Hm to the
coercivity, which is usually taken to be the intrinsic coercivity HcJ.

Fig. 5.12 Effect of armature reaction on magnet operating point

5.1.11 TORQUE PER ROTOR VOLUME


For DC machines the torque per rotor volume TRV is given by the output equation in the form

TRV ' 2BQ Nm/m 3 (5.55)

where B is the average flux-density in the airgap in [T] and Q is the electric loading in [A/m], given by
Z × Ia / a
Q ' . (5.56)
B Darm
If J is the RMS current density in the armature conductors, and each conductor has a cross-section Acond,
then JAcond = kf Ia/a, where kf = Irms/Ia is the current form factor, i.e., the ratio of RMS to average current.
Note that the average magnetic flux-density in the airgap at no-load is called the magnetic loading.
Page 5.14 SPEED's Electric Motors
5.1.12 ARMATURE WINDINGS
The conductors are laid in slots in the rotor. The ampere-conductors under each magnet pole must all
be in the same direction, so that all the ampere-conductors produce torque in the same direction. Since
the magnet polarities alternate NSNS..., the polarities of the ampere-conductors must also alternate with
the same "pitch" or "wavelength", Fig. 5.8.
There are two main forms of winding: the lap winding, Fig. 5.9, and the wave winding, Fig. 5.10. In the
lap winding the coil ends are terminated on adjacent commutator segments, but in wave windings they
are approximately 360Eelec apart; see Figs. 5.11, 5.13 and 5.14. In small machines the winding is usually
wound in one operation on a winding fixture, but large machines use separate form-wound coils.

Fig. 5.13 Armature amp-conductor distribution

Fig. 5.14 Single coil in lap-wound DC armature

Fig. 5.15 Single coil in a wave-wound DC armature


5.1 Permanent-magnet DC commutator motors Page 5.15
In DC armatures all coils have the same throw, that is, the number of slot-pitches between the "go" and
"return" coilsides. In Figs. 5.9 and 5.10 the throw is 4 slot-pitches. The throw is also known as the span or
back pitch of the coil, yB. The throw must be chosen so that the coil links all the flux of one magnet when
its axis is aligned with the magnet axis. If there are Ns slots and 2 p poles, the pole-pitch is Ns/2p slots or
2 B/Ns radians or 360/Ns × p Eelec. The magnet arc $M is usually rather less than the pole-pitch, perhaps
120Eelec. The coil throw yB should be chosen such that Ns/2 p > yB > $M (in consistent units).

Fig. 5.16 Front pitch yC and back pitch yB. (a) Progressive lap; (b) retrogressive lap; (c) progressive wave

The ends of the coils are connected to commutator segments. The front pitch of a coil is the number of
commutator segments between the two ends of the coil, yC. Coils can be laid up in two ways,
progressive and retrogressive, depending on the value of yC compared to the number of commutator
segments per pole. Table 1 summarizes the permissible values of yC. C is the number of commutator
segments, and x is known as the plex or multiplicity of the windings. If x = 1 the winding is simplex; if x
= 2 it is duplex, x = 3 triplex, and so on. The overwhelming majority of windings are simplex (x = 1).
Multiplex windings are interleaved and may be independent of each other.

Lap windings Wave windings


yC = +x progressive yC = (C + x)/p progressive
yC = !x retrogressive yC = (C ! x)/p retrogressive
TABLE 1

Multiple coilsides per layer


It is common to use multiple coilsides per layer (CSL) in each slot and to connect the ends of the coilsides
in one layer to CSL adjacent commutator segments, Fig. 5.12. This reduces the leakage inductance of the
coil which is being shorted by the brush as it undergoes commutation, that is, the reversal of the direction
of its current. It is usually at the position where its flux-linkage is maximum and its induced voltage
dN/dt is ideally zero, so that the brush can reverse the polarity of the current under zero-voltage
conditions, which helps to avoid sparking. However, the coil flux-linkage includes leakage flux
components in the end-windings and in the slots. Unlike the main flux, these are directly proportional
to the current in the coil or coils undergoing commutation, and the reversal of current therefore induces
a voltage (sometimes called the reactance voltage) between the commutator segments that are shorted by
the brush. If the reactance voltage exceeds about 2!3 V, sparking will occur. Clearly this is liable to
increase when the armature current increases. Therefore it is always desirable to design for minimum
leakage inductance by using multiple coilsides per slot, and by other means such as wide slot-openings
and shallow slots. In general
Page 5.16 SPEED's Electric Motors
C
CSL ' . (5.57)
Ns

Fig. 5.17 Multiple coilsides/layer; CSL= 3

CSL is also known as the "number of bars per slot". It should not be confused with the multiplicity or plex
of the winding.

Numbers of parallel paths and brushes


In wave windings there are always two parallel paths through the winding, a = 2, but in lap windings the
number of parallel paths is equal to the number of poles: a = 2p. The number of brushes b in lap
windings is also equal to the number of poles: thus a = b = 2p. In wave windings only two brushes are
needed to make the electrical circuit, but in practice it is common to use b = 2p, i.e. p sets in parallel, so
that the commutator can be made shorter and the current density per brush can be decreased. These
relationships are summarized in Table 2.

Lap windings Wave windings


a = 2px a = 2x
b = 2p b = 2 or b = 2p
TABLE 2
Multiplex windings
Multiplex windings provide an additional means of increasing the number of parallel paths in the
winding, without affecting the number of brushes. A duplex winding has two sets of armature coils. In
a progressive duplex lap winding, for example, the "return" coilside of coil n is connected not to bar n+1
but to bar n+2, and there is an inactive bar between any two active bars, that will be used by the second
set. The two sets may be completely independent and not connected except by the brushes; such a
winding is termed "doubly closed". One half of the slots contains all the odd-numbered coils, and the
other contains all the even-numbered coils. On the other hand it may be "singly closed", in which case
a first tour around the periphery will trace all the odd-numbered coils, and a second tour will trace all
the even-numbered coils.
The multiplicity of a winding is sometimes called the "plex". A simplex winding has plex = 1; duplex
windings have plex = 2, and triplex windings have plex = 3. Some texts erroneously use the term "duplex"
to refer to a coil with two coilsides per layer and two commutator segments per slot , and "triplex" to refer
to a coil with three commutator segments per slot. The distinction is important. Windings with multiple
commutator segments per slot are common, but duplex and triplex windings are rare in small motors.
For more information, see the PC-DCM Reference Manual and the winding editor in PC-DCM or PC-WFC.
5.1 Permanent-magnet DC commutator motors Page 5.17
5.1.13 WINDING EXAMPLES
Fig. 5.13 shows a progressive lap winding in a 15-slot, 4-pole armature with 2 coilsides/slot. The throw
or back pitch is yB = 3 slot-pitches, and since the slot-pitch is 360/15 × 4/2 = 48Eelec, yB = 3 × 144Eelec. In
this machine the magnet arc is $M = 120Eelec and yC = 1. Fig. 5.14 shows an example of a retrogressive
wave winding with yC = 7. In both examples CSL = 1.

Symmetric windings
A symmetric winding is one in which the conductors in every parallel path are distributed identically
with respect to a north or a south magnet pole, so that the generated EMF is exactly the same in every
path. Many windings do not satisfy this condition exactly. A winding can be classified as symmetric if
the number of commutator segments is divisible by the number of poles and the number of coils per pole
is equal to the number of parallel paths times the multiplicity or "plex".

Fig. 5.18 Progressive lap winding with 2 coilsides/slot in a 15-slot, 4-pole armature. yB = 3, yC = 1

Fig. 5.19 Retrogressive wave winding with 1 coilside/slot in a 15-slot, 4-pole armature. yB = 3, yC = 7
Page 5.18 SPEED's Electric Motors
5.1.14 DISTRIBUTION OF AMPERE-CONDUCTORS

The electrical layout of a simple winding with one commutator segment per slot is shown in Fig. 5.20.
There are two brushes and two parallel paths through the winding. The electrical equivalent circuit is
shown in Fig. 5.21.
The supply current does not necessarily divide equally between the two parallel paths. The respective
currents in the parallel paths depend on the back-EMFs e1 and e2 and on the impedances of the two circuits.
The EMFs e1 and e2 are formed from the series connection of coils between the brushes.

Fig. 5.20 Electrical layout of commutator winding

Because of the action of the commutator, both the EMFs and the impedances are liable to vary. Moreover,
since the armature winding forms a closed loop, any inequality between e1 and e2 is liable to drive current
round the upper short-circuit loop in Fig. 5.21. This circulating current does not pass through the
brushes and therefore is not observable from the terminals. While it plays no part in the elementary
theory, it may be a matter of concern in large machines, and "equalizing rings" are sometimes fitted to
suppress it.

Fig. 5.21 Equivalent circuit

In finite-element analysis it is necessary to assign the correct number of ampere-conductors in every slot.
Since the number of slots per pole-pair is often odd, the slot-ampere-conductors cannot generally be
assigned by symmetry, and in fact they depend on the rotor position. It is therefore desirable to have a
procedure for determining the slot ampere-conductors mathematically, and such a process is now
described.
Note first that even when the brush currents are prescribed, it is not possible to solve the circuit of Fig.
5.21 uniquely because of the short-circuit loop. But if the loop current is assumed zero, the currents in
the two parallel paths become equal. Then in those coils whose commutator segments are not in contact
with the brushes, the current can be assigned straightaway as ib/a, where a is the number of parallel
paths.
5.1 Permanent-magnet DC commutator motors Page 5.19
In the coils whose commutator segments are in contact with the brushes, the current is changing between
a positive and a negative value, or vice-versa. We can write
where ik is the current in the k-th coil and cq is the
iN current
 c1 in the q-th commutator segment, with k ' 1,2
i1 '
. . .N and q ' 1,2, . . .N, the numbers of coils and commutator segments both being N. Assuming linear
i1  c2
i2 ' segments
commutation, the currents in the commutator in contact with the brushes can be calculated
(5.58)
from the fractional widths w1, w2 etc. as shown in @ @ Fig.
@ 5.22 for commutator segments 1 and 2. Thus
iN ' iN  1  c N ,
w1 w2
c1 ' ib ; c2 ' i (5.59)
w2  w2 w2  w2 b

and of course c1  c2 ' ib. If this assignment is done correctly it is also the case that

c1  c2  @ @ @  cN ' 0 . (5.60)
The procedure is: (i) find all the non-zero commutator-segment currents c;
(ii) find the first coil k whose commutator segments are not connected to a
brush; (iii) assign ik = ib/a ; (iv) assign the remaining coil currents in order,
ik+1, ik+2, ... using eqn. (5.36). Finally the slot ampere-conductors must be
determined from the coil currents, taking into account the location of the
coil-sides in the slots. In general the number of coil-sides per layer (or Fig. 5.22 Division of current
commutator segments per slot) is greater than 1, so that the ampere-conductors in each slot are the
resultant of several coil currents. The coil pitch also affects the result. Some idea of the complexity of
this process can be gained from Fig. 5.23. In the PC-DCM and PC-WFC programs, the winding editors
accomplish the process automatically for PC-FEA and also produce text files containing the necessary
data. A similar process is used to recover the total flux-linkage observed at the brushes, from the
combination of slot vector-potentials obtained by finite-element analysis. The result can be used for
computing the armature inductance and the mutual inductance between the armature and the field,
which is important in the AC series motor, (see below).

Fig. 5.23 Highlighted current path in a simple lap winding with 2 commutator segments per slot.
Page 5.20 SPEED's Electric Motors

A
E F

0 0

8
$
D C

G H
B 3rd
brush

Fig. 5.24 Third brush configuration

5.1.15 THIRD BRUSH


A third brush is sometimes used as a means of operating the motor at a different speed. In Fig. 24 normal
operation uses the brushes at positions AB, while the alternative connection uses the brushes at
positions AC. It is assumed that there is no brush-shift in the normal configuration.
In normal operation the armature EMF is collected in two parallel paths BHFA on the right and BGEA on
the left. However, in the neutral zone 8 the coil EMFs are zero. Consequently, in effect, the EMF is
collected in the two parallel paths GE on the left and HF on the right in Fig. 24. The EMFs lying within
these arcs represent the EMFs of coils whose conductors are passing under the field poles or magnets.
We can say that the non-zero EMFs are collected from an arc (1 ! 8)B electrical radians on each side of Fig.
24.
In the alternative 3rd-brush connection, the EMFs are collected from an arc CFA on the right and
CHBGDEA on the left. In the right-hand arc, only CF is active. This arc is denoted 0. In the left-hand
arc, DE is active while CH and GD cancel one another, and HBG and EA are inactive. The EMF is thus
collected from equal arcs 0 on either side, CF on the right and DE on the left.
If $B is the displacement angle of the 3rd brush in electrical radians, we have $ + 0 + 8/2 = 1. Also if "B
is the pole-arc in electrical radians, we have "B = (1 ! 8)B or 8 = 1 ! ". Thus

1 "
0 ' ! $. (5.61)
2

For example if the pole-arc is 140E elec, " = 140/180 = 7/9 = 0@7778, and if the 3rd brush is displaced 25E, $
= 25/180 = 5/36 = 0@1389. Then 0 = (32 ! 5)/36 = 27/36 = 3/4 or 0@75, that is, an angle of 135E elec.
The ratio of EMF in the 3rd brush configuration to the EMF in the normal configuration is

(1 ") / 2 ! $ 2$ 1
e ' ' 1 ! ' . (5.62)
(1 ") / 2 1 " 1 $/0

Using the example already quoted, we get e = 1 ! 2 × 0@1389/(1+0@7778) = 0@8438, or 1/(1+0@1389/0@75) =


0@8438. In other words, a 25E shift of one brush reduces the EMF by about 15% at constant speed.
Note that there will be no reduction of EMF unless $ > 8/2, that is, $ > (1 ! ")/2. In the example, 8/2 = 20E.
Third brush in PM DC motor Page 21
rd
The armature resistance is also affected by the displacement of the 3 brush. In the normal
configuration the resistance is that of the two parallel paths paths BHFA on the right and BGEA on the
left. The resistance of one path can be assumed proportional to the arc angle, in this case B electrical
radians. The resistance of the parallel combination is thus proportional to B/2.
In the 3rd brush configuration the right-hand arc is shortened to (1 ! $) B, while the left-hand arc is
lengthened to (1 + $)B. Using the formula for the resistance of two parallel resistors, the ratio of
resistance between the 3rd brush configuration and the normal configuration is

(1 ! $) B × (1 $) B 2
r ' × ' 1 ! $2 . (5.63)
(1 ! $) B (1 $) B B

In the example with $ = 25/180 ' 0@1389, r = 0@9807. The reduction in resistance is much less noticeable
than the reduction in EMF.
While the total EMFs remain equal in both parallel paths, the armature current does not divide equally.
The ratio of the current in the short path to the current in the long path is (1 + $)/(1 ! $). In the example
this is (1 + 5/36)/(1 ! 5/36) = 1@3226. Although this will introduce a circulating current in the closed
circuit of the armature, it has no effect on the time-waveform of current-density in any single armature
conductor, since every conductor passes through both circuits sequentially at the rate of rotation.
However, the current-density on the “short” side of the rotor will be greater than on the “long” side, and
although this will lead to a considerable difference in the loss density, it will not result in any distortion
of the rotor because the rotation evens out the heat load. The uneven split between the path currents will
adversely affect the commutation at at least one brush.
It is better to interpret the e ratio in terms of the EMF constant kE, than in terms of the EMF itself. kE is the
ratio of the EMF to the speed, measured in volts per rad/s or volts per thousand rpm. For the present
purposes we can assume that the torque constant kT (= T/I ) is equal to kE, provided that kT is measured
in Nm/A and kE in Vs/rad.
Suppose the example motor is operating at 1800 rpm in the normal configuration with an EMF that is 70%
of the supply voltage Vs = 24 V, with a total brush volt-drop of 2 V. Then E = 0@7 × 24 = 16@8 V and kE =
16@8/1@8 = 9@33 V/krpm or 16@8/(1800 × B/30) = 0@0891 Vs/rad. Suppose the current is 5 A. Then the torque
is T ' 0@0891 × 5 ' 0@4456 Nm. Friction is neglected. The armature resistance is (24 ! 2 ! 16@8)/5 = 1@04 ohm,
2
and the volt-drop in the armature resistance is 1@04 × 5 = 5@2 V. The armature copper loss is 5 × 1@04 = 26
W and the input power is 24 × 5 = 120 W, while the mechanical power output (neglecting friction and
windage) is 16@8 × 5 = 84@0 W. The efficiency is thus 84@0/120 ' 70%.
In the 3rd brush configuration with a 25E shift, kT decreases to 0@8438 × 0@0891 = 0@0752, so the current
increases to 0@4456/0@0752 = 5@926 A. The resistance decreases to 0@9807 × 1@04 = 1@02 ohm, so the EMF must
become 24 ! 2 ! 5@926 × 1@02 = 15@956 V. The speed therefore changes to 15@956/0@0752 = 212@17 rad/s or 2026
rpm, an increase of 12@56%. The mechanical power increases by the same percentage, while the current
2
increases by 18@5% and the armature copper loss increases in the ratio 1@185 × 0@9807 ' 1@377. The
mechanical power increases to 15@956 × 5@926 = 94@55 W, an increase of 12@56% (as expected at constant
torque). The input power increases to 24 × 5@926 = 142@2 W and the efficiency decreases to 94@55/142@2 or
66@5%.
Suppose we increase the third-brush shift to 40E. Then $ ' 40/180 ' 2/9 = 0@2222 and 0 ' (1 + ")/2 ! $ ' (1
+ 7/9)/2 ! 2/9 ' 0@6667. Then e = 1/(1 + 0@2222/0@6667) = 0@750 and kE decreases to 0@750 × 0.0891 = 0@0668
2
Vs/rad, and kT likewise to 0@0668 Nm/A. The resistance ratio is r = 1 ! 0@2222 = 0@9506 giving a resistance
of 0@9506 × 1@04 = 0@9887 ohm. Again assuming a constant-torque load, the current increases to
0@4456/0@0668 = 6@6667 A. The resistance volt-drop increases to 6@6667 × 0@9887 = 6@591 V, so the EMF must
become 24 ! 2 ! 6@591 ' 15@409 V. The speed must therefore change to 15@409/0@0668 = 230@5 rad/s or 2201
rpm, an increase of 22@3% over the original 1800 rpm. The mechanical power increases to 15@409 × 6@6667
= 102@7 W while the input power increases to 24 × 6@6667 = 160@0 W, giving an efficiency of 64@2%.
Page 5.22 SPEED's Electric Motors

5.2 Wound-field commutator machines

5.2.1 UNIVERSAL MOTORS


The "universal" motor is a commutator motor with a stationary wound field connected in series with the
armature, as shown diagrammatically in Fig. 5.25. It is often called a "series" motor.

Fig. 5.25 Diagrammatic representation of series motor.

There are only two connecting leads, and one current I = Ia = If flows through both the field and the
armature circuits. The current If flowing through the field winding produces flux Md which passes
through the field poles in a direction parallel to their axis, the direct axis or d-axis. It helps to visualise
this flux if the lamination outline is superimposed on Fig. 5.25, as in Fig. 5.26. The flux pattern for Md
produced by Id is shown in Fig. 5.27. The permeance of this flux-path is high, and so is the inductance
associated with it, because the flux is generally confined within the iron and only has to cross two narrow
airgaps in series.

Fig. 5.26 Series motor with the diagrammatic representation overlaid by the lamination outline

Ideally the commutator and brushes constrain the armature current to flow in the directions indicated
by the dots and crosses in Figs. 5.25 and 5.26. The production of torque is often explained by saying that
the axially-directed conductors under the poles are located in a radial flux (Md) which gives rise to a
tangential force, according to Fleming's left-hand rule. In Figs. 5.25 and 5.26 these forces produce a torque
in the counter-clockwise (i.e., positive) direction.
5.2 Wound-field (universal) commutator motors Page 5.23

Fig. 5.27 Finite-element flux-plot with current in the field Fig. 5.28 Finite-element flux-plot with current in the
winding only armature winding only

The armature ampere-conductors also produce a magnetic field, called armature reaction. This is
generally said to be oriented in the direction of the q-axis, with flux-paths as shown in Figs. 5.28 and 5.30.
The permeance of this flux-path is generally low, except under the poles. In motoring, the armature-
reaction flux tends to strengthen the flux-density at the leading edges of the poles, and to weaken it at the
trailing edges. It produces unequal saturation of the leading and trailing pole-tips, one effect of which
is to decrease the effective main flux Md in the d-axis. The armature-reaction flux does not directly link
the field winding. Likewise the armature circuit depicted by the dots and crosses in Figs. 5.29 and 5.30
does not link the d-axis flux Md, as long as the brushes are electrically aligned with the q-axis as shown.

Fig. 5.29 d-axis flux produced by field winding Fig. 5.30 q-axis flux produced by armature current

The torque is proportional to the product of the flux Md and the armature current Ia , and since Md is
2
approximately proportional to Ia , the torque is proportional to Ia and thus does not depend on the
direction of the current. It is this that enables the motor to operate with either DC or AC supply.
Nevertheless, the direction of rotation can still be reversed by reversing the connections to either the field
or the armature.
Page 5.24 SPEED's Electric Motors
5.2.2 EQUIVALENT CIRCUIT FOR DC OPERATION

Fig. 5.31 Equivalent circuit of DC series machine

The equivalent circuit for DC operation is shown in Fig. 5.31. The resistance volt-drop RI and brush volt-
drop Vb are straightforward to calculate. The resistance R is the combined resistance of the field and
armature windings, together with the resistance of the leads:
R ' Ra  Rf  Rleads . (5.64)

The rotational EMF Ea is more complicated. In §5.1 it was calculated by the simple classical formula, eqn.
(5.2), without regard to the effects of short-pitching the windings, skew, brush shift, or the nonlinear
effects of armature reaction. In this section, eqn. (5.2) is expanded analytically to show the effects of
brush-shift and armature reaction, and it will quickly appear that a more sophisticated method is
required, especially for dealing with the winding distribution and the effects of skew, saturation and
armature reaction. Such a method — the waveform method — is introduced in §§5.2.3!5. In §§5.2.6!7 the
circuit is extended for AC operation, and its solution is described in §5.2.6.

Rotational EMF in the armature — analytical treatment


The rotational EMF in the armature winding can be obtained directly from eqns. (5.2) and (5.3) as
2 p Na
Ea ' Tm M V (5.65)
aB

where M is the airgap flux per pole, Na ' Z / 2 is the total number of armature turns, a is the number of
parallel paths in the armature, and p is the number of pole-pairs.
The flux M is normally considered as d-axis flux with symbol Md. Since the EMF Ea appears between
brushes located on the q-axis (or interpolar axis), it is normally considered as a q-axis EMF Eq. These
conventions are represented in Fig. 5.32. Current If in the field coils F produces d-axis flux Md in the
direction of the positive d-axis, and with rotation in the counter-clockwise direction the EMF-conductors
are shown by the ring marked Eq. This ring is incomplete because only those conductors passing under
the poles have EMF in them. The ampere-conductors are shown by the ring marked I, and in a motor they
are in the opposite direction to the EMF-conductors. Conductors with opposing EI products in them
contribute positive motoring torque. The armature-reaction flux Mq produced by the ampere-conductors
is in the direction of the positive q-axis (downwards in Fig. 5.32), as in Fig. 5.30.
Fig. 5.33 shows the effect of shifting the brushes through an angle $, against the direction of rotation.
The ampere-conductors are shifted along with the brushes, but the EMF-conductors do not move. For
small brush-shift angles the displacement of the ampere-conductors relative to the EMF-conductors does
not by itself alter the torque, because the EMF's in the shift zone 2$ are zero. Any shift beyond the angle
shown in Fig. 5.33 would cause a reduction in torque, as some of the positive ampere-conductors would
coincide with positive EMF-conductors and some of the negative ampere-conductors with negative EMF-
conductors. (Like signs indicate generating or braking torque with the chosen conventions).
5.2 Wound-field (universal) commutator motors Page 5.25

2
Neutral
L I T
zone
I
F F

Field Field
pole pole

d Ed d
Eq Eq
d d
q
q

T L
m Neutral
m
zone

Fig. 5.32 Location of EMF and ampere-conductors


Fig. 5.33 Location of EMF and ampere-conductors when the
brushes are shifted by angle $, against the direction
of motor rotation.

It is evident in Fig. 5.33 that the brush-shift divides the ampere-conductors into two belts, each of extent
2$; and two others each of extent (B ! 2$). The belts of extent 2$ form a "coil" whose MMF opposes that of
the field current when $ is against the direction of rotation. In Fig. 5.33 they are enclosed within dotted
lines. They represent a component of armature reaction that is directly demagnetizing in the d-axis. The
proportion of armature ampere-conductors active in the d-axis is
2$
G($) ' (5.66)
B

and this proportion of armature ampere-conductors is so placed that it can be added directly to the field
ampere-conductors (with the appropriate sign), and the effect on the d-axis flux Md can be calculated.
The other two belts of ampere-conductors account for the proportion
2 *$*
F($) ' 1  (5.67)
B

of the total armature ampere-conductors. These are active in producing a q-axis component of armature-
reaction flux, the distribution of which is more complicated than that of the d-axis flux. Nevertheless,
the q-axis component of armature-reaction flux Mq generates a d-axis component of EMF which is labelled
Ed in Fig. 5.33. Of all the armature conductors that have EMF generated by q-axis flux, only the fraction
G($) contributes to the EMF at the brushes. Although these EMF's are small (because they are generated
in a region where the q-axis field is relatively weak), they tend to increase the EMF between brushes
provided that the brush shift is against the direction of rotation. Graphically this is indicated in Fig. 5.33
by the fact that the EMF-conductors labelled Ed replace the missing EMF-conductors in the same sector,
supplementing the Eq ampere-conductors and tending to produce a continuous ring of conductors in
which the EMF and current are opposite, as required to maximize the motoring torque.
Returning to the calculation of E, the two components Ed and Eq can be calculated separately and added
together. For Eq eqn. (5.65) can be used with M ' Md, the d-axis flux crossing the airgap. Now Md can be
written
Md ' Fmd Pd (5.68)

where Pd is the non-linear permeance for d-axis flux, and Fmd is the peak value of the total d-axis MMF
effective in producing d-axis flux. Thus for a series-connected machine
Page 5.26 SPEED's Electric Motors
I Nd Nf Na
Fmd ' with Nd '  G($). (5.69)
2p q a
Nd is the effective turns in series in the d-axis relating the MMF/pole to the current I at the terminals. Nf
is the total number of turns and q the number of parallel paths in the field circuit.
Likewise the q-axis flux Mq can be written
Mq ' Fmq Pq (5.70)

where Pq is the non-linear permeance for q-axis flux. The calculation of Pq will be discussed later in
connection with AC operation. Fmq is the total q-axis MMF effective in producing q-axis flux: thus
I Nq Na
Fmq ' with Nq ' F($). (5.71)
2p a
Nq is the effective number of armature turns in series in the q-axis. Because of the somewhat complicated
distribution of q-axis flux, the permeance Pq is not as easy to visualize as Pd.
Md generates the EMF Eq and Mq generates the EMF Ed. Both can be calculated using eqn. (5.65) with
appropriate number of turns Nd and Nq for the two axes, and the overall result for the total EMF is

E a ' Eq  Ed ' Xa I (5.72)

in which
Na Nf Na Na
Xa ' Tm  G ($) Pd F($)  F ($) Pq G ($) ' Xad  Xaq (5.73)
aB q a a
with
Tm Tm G ($)
Xad ' Nd Nq Pd and Xaq ' Nq2 Pq . (5.74)
B B F ($)
Xa has the dimensions of reactance, but behaves as a speed-dependent resistance. Note that Xaq is
positive if G($) is positive when the brushes are rotated against the direction of rotation. This being so,
the effective number of d-axis turns Nd is less than the effective field turns Nf/q: in effect, this is a flux-
weakening condition that tends to diminish Ea and make the motor run faster, provided that Xaq does not
increase more than the decrease in Xad. (This is unlikely, given that Pq << Pd).
Eqn. (5.73) is an important part of the theory for both DC and AC operation. It shows the effect of the
demagnetizing ampere-turns of armature reaction and the additional EMF component produced by the
cutting of q-axis armature-reaction flux. Although eqn. (5.73) together with eqn. (5.72) is more advanced
than what is found in most textbooks, it still has a number of serious limitations:
1. It takes no account of saturation, and it is especially unclear how Pq will be affected by
saturation.
2. Even without saturation, the calculation of Pq is difficult because of the complicated
distribution of q-axis armature reaction flux.
3. Because eqns. (5.73) and eqn. (5.72) are derived from the formula (5.65), they do not take
account of short-pitching or skew.
In principle the individual terms in eqn. (5.73), such as Pd, Pq, G($) and F($) can be measured using special
tests, but they cannot be as easily compared with finite-element calculations as can the waveform data
to be discussed in the next section. This is important in view of the fact that physical testing on prototype
machines is becoming less common, while the use of finite-element analysis is increasing even in the
early stages of design. For this reason the analytical treatment is here left off, and the calculation of the
rotational EMF by the waveform method is started from first principles, the aim being to calculate the
rotational EMF as in eqn. (5.73), complete with its dependence on saturation, winding distribution, skew,
and brush shift.
5.2 Wound-field (universal) commutator motors Page 5.27
5.2.3 ROTATIONAL EMF — WAVEFORM METHOD
Strictly speaking, Faraday's law applies to the EMF generated by the rate of change of flux-linkage dR/dt
in a complete circuit comprising a coil or group of coils. The flux-cutting EMF BLv in a conductor of
length L moving through a fixed magnetic field B with velocity v provides the same result as the more
rigorous dR/dt formulation, provided that B is independent of the motion of the rotor. The accumulation
of the BLv products from all the "go" and "return" conductors in the entire winding is, in effect, an
integration of the dR/dt contributions of all the coils in the winding.
The B value experienced by any conductor depends on its position and, as is well known, the BLv product
for a single conductor as a function of time, e(t), is a waveform that is a scaled replica of the flux-density
distribution B(2) around the surface of the armature. The EMF waveform for one coil of T turns is
eC ( t ) ' eG ( t )  eR ( t ) ' [ B (2G)  B (2R) ] × L r T T , (5.75)

where eG(t) is the EMF in the "go" coilside and eR(t) is the EMF in the "return" coilside, T is the angular
velocity and r is the rotor radius. The B(2) distribution can be measured from the waveform e(t) of a full-
pitch search-coil obtained by means of an oscilloscope. For a full-pitch coil, the pitch is B radians, 2R =
2G + B, B(2R) = ! B(2G), and eC(t) = 2B(2) × LrTT. The classical eqn. (5.65) is consistent with this
expression and implicitly assumes full-pitch coils, or at least, coils with sufficient pitch to link all of the
field flux when they are aligned with the d-axis. In other words, as has already been stated, eqn. (5.65)
does not account for coil pitch.
The series connection of individual coil EMF's in a simple armature is shown in Fig. 5.34. The eC values
in this diagram remain fixed in space while the actual coils rotate. The EMF between two brushes is the
sum of all the instantaneous eC(t) values lying between them. If the brushes are spaced by 180Eelec the
total EMFs in the two parallel paths through the winding are practically equal, so that no net current
circulates around the winding as a whole. Fig. 5.34 also shows the ideal arrangement in which the
rotational EMF in a coil shorted by a brush is zero.
The EMF of the entire winding can be determined as the path EMF
iN
eP ( t ) ' j eCi ( t ) (5.76)
i

where the sum is accumulated over all the N coils in series between the brushes. In this equation the coils
are counted from i to i + N. By starting the accumulation at a different coil i, the effect of brush shift can
be calculated assuming that B(2) is known. With a sufficient number of coils and commutator segments,
the substitution of one coil for another as the commutator rotates past the brushes renders the path EMF
nearly constant. In practice it contains a ripple component at the bar-passing frequency and there may
be voltage spikes, but for most purposes it can be taken as a constant EP equal to the time-average of eP(t)
over the time taken to rotate through one bar-pitch.

Fig. 5.34 Connection of coil EMF's in an armature, showing the connection of the brushes.
Page 5.28 SPEED's Electric Motors

Fig. 5.35 Path EMF versus brush-shift angle, with point-contact brushes.

Fig. 5.35 shows the path EMF “picked off” from the commutator at two diametrically opposite points, as
though we had “point-contact brushes”. The point-contact brushes have zero brush width and are
represented in Fig. 5.34(a). The steps in this waveform occur at the points where the point-contact
brushes pass from one commutator segment to the next. In this example, with an even number of
commutator segments, both brushes pass from one commutator segment to the next at the same instant.
The two adjacent commutator segments are never “shorted” together, because the brush-width is zero.
Consequently the path EMF waveform in Fig. 5.35 shows the maximum possible ripple voltage.
With point-contact brushes the maximum value of path EMF occurs close to the position of zero brush-
shift. If the brushes are shifted forwards or backwards, the path EMF varies abruptly in steps whose
spacing is equal to the bar-pitch. In Fig. 5.34 (b) it is clear that real brushes short-circuit the adjacent
commutator segments and may even short-circuit more than two segments together. Ipso facto they
eliminate the ripple from the waveform of the terminal voltage. The steps in the EMF waveform
represent a difference in voltage “picked up” by the leading and trailing sections of the brush, and this
voltage difference drives a circulating current through the brush, and indeed around the entire
winding. The circulating current is limited by the brush contact resistance, which of course varies as
the rotor rotates. It is also limited by the inductance in the closed circuit.
In the absence of a detailed circuit-model of the circulating current, a simple expedient is to use the
fundamental component, which is also shown in Fig. 5.35.
5.2 Wound-field (universal) commutator motors Page 5.29
Effect of brush-shift
Fig. 5.33 shows the conductor-currents and the conductor-EMFs, when the brushes are rotated against the
direction of rotation. In a DC machine all EMFs are “motional” EMF, and there are three effects:
(i) The path EMF is unchanged for small values of brush-shift angle $, because the conductors in the
neutral zone have very little EMF in them (white circles). However, if $ is large enough, the path
EMF starts to “lose” EMF-carrying conductors, and the path EMF decreases.

(ii) In Fig. 33 on page 25, the four “dot” ampere-conductors at the bottom and the four “cross” ampere-
conductors at the top constitute a “coil” which sets up a demagnetizing flux in the d-axis. This
tends to decrease the path EMF still further.
(iii) There is a q-axis flux produced by armature reaction. This generates a “d-axis EMF” labelled Ed in
Fig. 33, arising in the conductors situated in the neutral zone. This EMF tends to offset the
reductions in EMF from causes (i) and (ii).
In an AC machine there are also “transformer” EMFs, associated with the mutual inductance between the
field and armature, and with the self-inductances of the field and armature windings. We therefore have
three more effects to consider:
(iv) The field self-inductance is theoretically independent of the brush position, so brush-shift has no
effect via the field self-inductance.
(v) The armature self-inductance between brushes theoretically will have a minimum value when the
brushes are in the geometric neutral axis (zero brush shift), and will increase if the brushes are
shifted either way. In effect this increases the series impedance, so at constant torque the speed
must decrease in order to restore the current and the torque.
(vi) The mutual inductance M between the field and the armature is theoretically zero when the
brushes are in the neutral axis. If they are shifted against the direction of rotation, M acquires a
negative value, tending to reduce the total terminal inductance. So at constant torque the speed
will tend to increase. On the other hand, if the brushes are rotated in the direction of rotation, M
will be positive and the speed will tend to decrease.
In summary, effects (i) and (ii) tend to decrease the motional EMF, leading to an increase in speed, while
(iii) has the opposite effect. (iv) has no effect. (v) tends to decrease the speed regardless of the direction
of brush-shift, while (vi) tends to increase the speed if the brush-shift is against the direction of rotation.
The “transformer” EMFs are also characterized as “reactive voltages”, and they are discussed in more
detail on p. 33ff.
Page 5.30 SPEED's Electric Motors
5.2.4 THE AIRGAP FLUX-DENSITY DISTRIBUTION

Fig. 5.36 Airgap flux-density distribution. F = idealized distribution with field ampere-turns only, and no saturation or fringing;
F + A = actual distribution with armature reaction, saturation, fringing, and skew. The arrow shows the direction of
motion of the armature. The diagram is drawn for the left-hand pole in Fig. 5.32. L is the leading pole-tip and T is the
trailing pole-tip.

Fig. 5.36 shows the general form of the airgap flux-density distribution. The basic field is set up by the
ampere-turns of the field winding. If there were no fringing and no slot-openings, and if the airgap length
was uniform under the pole, the form of this basic field would be the dotted rectangle F.
The actual flux distribution is marked F + A in Fig. 5.36. It includes the effects of armature reaction,
brush-shift, saturation, fringing, and skew. A finite-element calculation of this distribution would also
show the effects of slotting, in the form of dips centred on the rotor slot-openings , but these dips are
stationary with respect to the armature and therefore they do not appear in the EMF waveforms eG(t) and
eR(t) of §5.2.3. For this reason the slot modulation is omitted from the B(2) distribution in Fig. 5.36.
The B(2) distribution is markedly distorted by the armature reaction MMF which strengthens the field
under the leading pole-tip and weakens it under the trailing pole-tip, where it may even be reversed.
Under the leading pole-tip, saturation (particularly of the rotor teeth) limits the flux-density to a level that
depends on the saturation flux-density of the steel and the ratio of rotor tooth width to rotor slot-pitch.
Because of the truncation of the flux distribution under the leading pole-tip, the increase of flux under
the leading half of the pole is less than the decrease under the trailing pole-tip, so that there is a net
reduction of flux even when the brushes are centred on the q-axis.
In PC-WFC the calculation of the effect of saturation under the pole uses the method described by
Kostenko and Piotrovsky [5, vol. I, p. 181]; (see also [4], p. 99). The saturation curve for airgap MMF vs. B(2)
is calculated for the teeth + airgap only, and the total mmf to the left and right of the d-axis is used
together with this characteristic to determine the actual flux-density under the pole. Fig. 5.37 shows the
typical form of the relevant saturation curve, together with the total saturation curve which includes the
MMF drops in the rotor yoke, the stator poles, and the stator yokes.

Fig. 5.37 Saturation curves for calculating the effect of armature reaction .
5.2 Wound-field (universal) commutator motors Page 5.31
5.2.5 GENERATING THE E/I CURVES

Field current i F
1
Mag. curve Fd

d-axis flux 
d
B()

Brush shift Skew

Armature MMF B()

Mag. curve F TG

Fig. 5.38 Method of calculating the saturation curves of Fig. 5.37.


B()
Fig. 5.38 shows the process by which the basic
saturation curves of Fig. 5.37 are calculated. For each 4
of N values of d-axis airgap flux Md, the required MMF
Fringing by
Fd is calculated from the lumped-parameter window convolution
magnetic equivalent circuit. The same process is
repeated for the (tooth + gap) saturation curve. At
each point on the curve, the flux is the independent
(input) variable and the MMF is the dependent
(output ) variable. B()

The next step is to calculate the E/I curve of EMF vs. 5


current, as shown in Fig. 5.39. This process uses
current as the independent variable. The first thing Armature EMF
it requires is the inversion of the magnetization
curve of Fig. 5.38, so that flux can be obtained as a
function of current instead of the other way round.
This is done by fitting the curve with a set of N cubic
splines and using the Newton-Raphson method to e(t)
solve for the flux at each of a series of levels of
current or MMF up to Fm. The results are fitted with
a new set of cubic splines so that they can be rapidly Fig. 5.39 Calculation of E/I curves
re-used and interpolated in the top box 1 of Fig. 5.39
to provide the flux Md for any value of d-axis MMF.
The entire process of Fig. 5.39 is repeated for a series of currents covering the range of MMFs up to Fm.
For each value of current IF, the flux Md is assumed to have a rectangular distribution B(2).
At this point 2 the effect of any skew is introduced, by phase-shifting and adding m copies of the B(2)
distribution, the phase-shift between adjacent copies being 1/mth of the skew angle. Thus the B(2)
distribution is used as a vehicle for the effect of skew, since it directly forms the EMF waveforms in the
individual coilsides which are skewed. A suitable choice of m is 18.
Page 5.32 SPEED's Electric Motors
Next the effect of armature reaction is introduced, 3. The MMF is determined by the same current iF and
its physical distribution by the brush positions. The airgap flux distribution is modulated by the method
described earlier, using the FTG curve of Fig. 5.38. At this stage also the q-axis flux of armature reaction
is added in the interpolar space.
At stage 4 the fringing of flux at the pole corners is modelled by convolving the B(2) distribution with a
windowing function
1  cos k 2
w (2) ' , (5.77)
2

where the parameter k controls the "sharpness" of the edges. Although this convolution has no rigorous
physical basis, it is scaled so that the net flux is unaffected, and the process of calibrating the resulting
B(2) distribution with a finite-element calculation elevates it from the level of informed guesswork to that
of a reliable curve fit.
The final stage 5 in Fig. 5.39 is to calculate the EMF waveform of individual coilsides from eqn. (5.75), and
then the path EMF between brushes is obtained from eqn. (5.76). When this is plotted as a function of the
original exciting current (iF in Fig. 5.39), the result is as shown in Fig. 5.40. For DC operation there is only
one E/I curve, that is, the one labelled "Actual" in Fig. 5.40.

Fig. 5.40 E/I curves

For AC operation, the curve labelled "Actual" is appropriate for the instantaneous values of current and
EMF. However, when AC operation is to be analysed using the phasor diagram, what is required is the
relationship between fundamental current I1 and fundamental EMF E1. Such a relationship can be derived
from the "Actual" curve if it is assumed that either the EMF or the current is sinusoidal. Supposing the
EMF to be sinusoidal, for every point on the curve a sinusoidal waveform of E is formed with peak value
equal to the point on the curve. Then the "Actual" curve in Fig. 5.40 is used to develop the corresponding
current waveform, from which the fundamental is extracted by Fourier analysis. This manipulation is
facilitated by first fitting the "Actual" curve with a series of N cubic splines, which can be used with the
Newton-Raphson method for inversion of the function, and finally the "Fundamental" curve itself can be
fitted with a series of cubic splines, to facilitate the nonlinear solution of the electric circuit equation.
Of course this equation is complex for AC operation, but the phasors of E and I can basically be assumed
to be in phase.
The extensive manipulation of the flux distribution to get the required E/I curves is performed efficiently
in PC-WFC at the beginning of a calculation, so that it does not have to be repeated in the solution of the
equivalent circuit.
Before addressing the solution of the equivalent circuit (in §5.2.8 below), we will first extend the DC
circuit of Fig. 5.31 to cover AC operation.
5.2 Wound-field (universal) commutator motors Page 5.33
5.2.6 EQUIVALENT CIRCUIT FOR AC OPERATION

Fig. 5.41 Equivalent circuit for AC operation

In AC operation the supply voltage Vs is alternating. It is assumed to be sinusoidal and is represented


by the phasor Vs. The EMF Ea is also assumed to be sinusoidal, represented by the phasor Ea, and the
circuit is calculated for the fundamental component of current I1. The relationship between the RMS
values of Ea and I1 is the "fundamental" curve in Fig. 5.40, and Ea is in phase with I1. The subscript 1 is
dropped from this point onwards.
While the frequency of the EMF Ea is the supply frequency f, its amplitude is proportional to the angular
velocity and the number of pole-pairs. We can define a "per-unit speed" S as the ratio
S ' p Tm / T with T ' 2Bf. (5.78)

If Xa0 is defined to be the value of Xa when Tm ' T/p (i.e., when S ' 1), then from eqns. (5.72) and (5.78),
E a ' S Xa0 I . (5.79)

This equation is instructive, but if the waveform method is used to calculate Ea, it is not needed.
In AC operation there are three reactive voltage drops Vd, Vq and jXLI that do not appear in the DC
equivalent circuit. Vd and Vq are sometimes called "transformer" voltages because they are induced at
fundamental frequency by the airgap components of the d- and q-axis fluxes Md and Mq. jXLI is the
reactive voltage drop across the leakage reactance XL. These are now examined in more detail.

Reactive voltages
The field winding links the airgap or "mutual" component of d-axis flux/pole Mmd , which is alternating
at the supply frequency f and induces the voltage
Nf
V fd ' j B 2 f M md V rms . (5.80)
q
where Mmd is a peak value. If the brushes are shifted from the q-axis, the main armature winding links
the fraction G($) of Mmd, inducing the voltage
Na
V ad ' j B 2 f G ( $ ) M md V rms . (5.81)
a
Vfd and Vad are in phase with each other and they can be added directly:

Nf Na
V d ' V fd  V ad ' j B 2 f  G ( $ ) M md ' j B 2 f Nd M md ' j Xmd I , (5.82)
q a

where Vd is the total reactive voltage drop in the d-axis. Xmd is the d-axis magnetizing reactance
Page 5.34 SPEED's Electric Motors
Pd
Xmd ' T Nd2 . (5.83)
2p
The permeance Pd = Mmd /Fmd in the unsaturated condition is dominated by the airgap permeance per
2
pole Pg. Since Pg is approximately inversely proportional to p, the expected dependence of Xmd on 1/p
is obtained. It may be influenced by the q-axis flux because of cross-saturation of the stator pole-pieces,
but still the flux is assumed to remain in phase with the current.
When the brushes are shifted against the direction of rotation, Nd and Xd are reduced. This tends to
increase the current and the torque, and therefore the speed, boosting the “flux-weakening” effect.
Similarly the q-axis flux of armature reaction induces the voltage-drop
V q ' V aq ' j Xmq I (5.84)

where Xmq is the q-axis magnetizing reactance,


Pq
Xmq ' T Nq2 . (5.85)
2p
Because of saturation, both Xmd and Xmq decrease with increasing current. Normally Xmq < Xmd.
The voltage-drop jXLI is due to the combined leakage reactance of the armature and field windings,
XL ' Xa slot  Xa end  Xf slot  Xf end . (5.86)

Here Xa slot is the slot-leakage reactance of the armature, and Xa end the end-turn leakage reactance. The
fluxes associated with Xa slot and Xa end do not cross the airgap and do not link the field winding.
Likewise Xf slot is the slot-leakage reactance of the field, and Xf end its end-turn leakage reactance. The
fluxes associated with Xf slot and Xf end do not link the armature winding.
The reactance and transformer voltages jXL I, Vd and Vaq are all in phase with each other and can be
combined into a single voltage drop
V ' jXI (5.87)
in which the total reactance is
X ' Xmd  Xmq  XL . (5.88)

The sum of the voltages RI, Vb, jXI, and Ea is the terminal voltage Vs : thus, as in Figs. 5.41 and 5.42,

V s ' V b  [ ( R  S Xa0 )  j X ] I (5.89)


This equation is nonlinear because X and Xa0 depend on the current, but it can be solved by recursion,
making use of the saturation curve in Fig. 5.40 to adjust X and Xa0, or X and Ea. The nonlinear brush-
voltage drop can be included in the recursion.
The electromagnetic torque is given by

1 Ea I S
Te ' Re E a(I ' ' Xa0 Ia2 Nm . (5.90)
Tm Tm Tm
Both real forms of this expression are valid for DC operation. All of them include (at least approximately)
the reluctance torque exerted by the field poles on the armature when the brush-shift is non-zero. This
is because the reactance Xa0 includes a "reluctance" term of the form (Xd ! Xq), whose form depends on
the functions F($) and G($). 2

2
Few texts mention this reluctance torque. It is similar to the torque of the repulsion motor, which relies entirely upon it. Of
course in the repulsion motor Ia and If are unequal. The reluctance torque also appears in the "generalised theory of electrical
machines" [14], but that theory assumes sinusoidal distributions of windings and fluxes, which is not well suited to the universal
motor. The equations developed in this section become consistent with the generalised theory if the windings and fluxes are
sinusoidally distributed.
5.2 Wound-field (universal) commutator motors Page 5.35
5.2.7 PHASOR DIAGRAM

Fig. 5.42 Phasor diagram

The phasor diagram expresses the way in which the various voltage-drops and EMFs add up around the
entire circuit, according to eqn. (5.89). This diagram also applies to DC operation when all the inductive
voltages are zero.

Fig. 5.43 Phasor diagram with iron loss and brush circulating current.

In Fig. 5.43 the phasor diagram is modified for iron loss and for the circulating current in the coils short-
circuited by the brushes.
The iron loss can be deemed to have an "electrical" component caused by the alternating flux, as in a
transformer, and a "mechanical" component that is caused by the rotation of the rotor through the flux,
producing a drag torque that should be subtracted from the electromagnetic torque. The apportionment
between electrical and mechanical components is not exact, but the electrical component should include
all the stator iron loss and most of the rotor iron loss at low speed, whereas the mechanical component
should include most of the rotor loss at high speed. The electrical component requires a component of
current Ic that produces no flux and is therefore in the q-axis of the phasor diagram, Fig. 5.43.
The current also requires a component Ib to neutralize the MMF of the current circulating in the coils
short-circuited by the brushes. As explained in [11], Ib leads the magnetizing component Im by a
considerable angle and, together with Ic, tends to improve the power factor slightly.
Page 5.36 SPEED's Electric Motors
5.2.8 IRON LOSSES
In the stator the frequency is everywhere equal to the supply frequency f, so the Steinmetz equation

WFe ' Ch Bpk n f  Ce Bpk2 f 2 W/kg (5.91)

can be used in its sinewave form, provided that the peak flux-density is calculated correctly for each
section of the lamination.
In the rotor the flux-density waveforms in the teeth and yoke vary with speed. Fig. 5.44 shows the
waveform NT(Tt) in a rotor tooth rotating in a constant field excited by the field winding. The transition
from zero to maximum takes place in an angle of rotation roughly equal to one slot-pitch. Fig. 5.44 also
shows the waveform of the EMF eT(Tt) that would be induced in a search-coil wound around the tooth. For
a single-turn search-coil,
d NT (T t) d NT (2)
eT ( T t ) ' ' p Tm , (5.92)
dt d2
where 2 is the angle of rotation. If the eT(Tt) waveform can be calculated, it gives dB/dt in the tooth, from
which the losses can be estimated using the variant of the Steinmetz equation given in eqn. (1.71):
2
a  b B pk dB
WFe ' Ch f Bpk  Ce1 . (5.93)
dt

The yoke can be treated in the same way if the waveform of yoke flux NY(Tt) is known.

  ( t)

e  ( t)

phi(T)_1000 rpm.wpg

Fig. 5.44 Waveforms of tooth flux-density and tooth EMF on the rotor, when the magnetic flux is constant

When the flux is alternating the waveforms become more complicated, but the same equations apply.
Figs. 5.45!50 show calculated tooth flux and EMF waveforms at speeds of 83.3, 250, 3000, 6000, 18,000 and
36,000 rev/min in a 2-pole motor excited at 50 Hz. At very low speed the supply frequency is much larger
than the rotation frequency and the flux waveform is like that of a 50 Hz carrier modulated by the
waveform of Fig. 5.44.
At 36,000 rev/min the rotation frequency is much higher than the supply frequency and the converse
applies: i.e., the waveform of Fig. 5.44 becomes the carrier and the 50 Hz supply modulates it, with 12 revs
per cycle of the supply frequency. Similarly at 18,000 rev/min there are 6 revs per cycle.
At intermediate frequencies the waveform changes markedly. For example at 3000 rev/min there is one
revolution per cycle of supply frequency, and the waveshape depends on the phase angle between the flux
and the rotational position of the tooth. At 6000 rev/min the phase effect is less evident as the "modulated
carrier" characteristic begins to re-appear.
5.2 Wound-field (universal) commutator motors Page 5.37
  ( t )   ( t )

e  ( t ) e  ( t )

phi(T)_1000 rpm.wpg phi(T)_3 krpm.wpg

Fig. 5.45 Tooth waveforms at 83.3 rev/min (1 rev) Fig. 5.46 Tooth waveforms at 250 rev/min (1 rev)
  ( t )

e  ( t )

phi(T)_6 krpm.wpg

Fig. 5.47 Tooth waveforms at 3000 rev/min (1 rev) Fig. 5.48 Tooth waveforms at 6000 rev/min (12 revs)

  ( t )   ( t )

e  ( t ) e  ( t )

phi(T)_18 krpm.wpg phi(T)_36 krpm.wpg

Fig. 5.49 Tooth waveforms at 18,000 rev/min (12 revs) Fig. 5.50 Tooth waveforms at 36,000 rev/min (12 revs)
Page 5.38 SPEED's Electric Motors
5.2.9 COMMUTATION

Fig. 5.51 Commutation

Current reversal in a coil takes place while the coil is short-circuited by a brush: see Figs. 5.20!22 and Fig.
5.51. A linear transition of current is called "linear commutation". Because of the inductance Lcs in the
coil, linear commutation requires a constant voltage Er = Lcs di/dt to change the current, [8, 11!13].
Distortion of the magnetic field due to armature reaction causes Er to acquire a rotational voltage
component which in DC machines is sometimes countered by rotating the brushes.
During commutation the short-circuited coil normally links the entire pole flux which in the AC-fed
universal machine is alternating at the supply frequency, giving rise to an additional "transformer" EMF
component of Er. Moreover, the current itself is varying with an approximately sinusoidal waveform,
so that the reactance voltage Er varies through the cycle as shown in Fig. 5.52.

Peak current envelope

Reactance voltage E r1

Fig. 5.52 Commutation in the universal motor — reactance voltage


5.2 Wound-field (universal) commutator motors Page 5.39
5.2.10 EFFECT OF SHORT-CIRCUITED COILS UNDERGOING COMMUTATION

Fig. 5.53 Short-circuited coils undergoing commutation

Fig. 53 shows two parallel armature coils A1, A2 undergoing commutation at the two separate brushes in
a 2-pole AC commutator motor. The field coils F1 and F2 each have Nf turns, while the armature coils
each have TcR turns. The relevant self- and mutual inductances are

Lff ' 2 Nf2 kSat Pmd0 μ0 Larm  Lf Slot  Lf End ;


Lcc ' (2 TcR)2 kSat Pmd0 μ0 Larm  Lc Slot/4  La End/Nslot/4 ; (5.94)
Lfc ' 2 Nf (2 TcR) kSat Pmd0 μ0 Larm .

where Lff is the self-inductance of the entire field winding; Lcc the self-inductance of the combined pair
of short-circuited armature coils; and Lfc the mutual inductance between these two circuits. Pmd0 is the
unsaturated permeance per pole of the d-axis magnetic circuit, and ksat is the saturation factor for this
magnetic circuit. Lf slot is the slot-leakage component of the field winding and Lc slot is the slot-leakage
inductance of one armature coil. Lf end is the end-winding leakage inductance of the field winding and
La end is the end-winding leakage inductance of the armature winding. Nslot is the number of slots.
If If is the AC field current and Icc is the current in the parallel combination of the two short-circuited
armature coils, the voltage equations for the two circuits can be written
V f ' j T Lff I f  j T Lfc I cc ;
(5.95)
0 ' ( Rcc  Rcb  j T Lcc ) I cc  j T Lfc I f .

Using the second equation to eliminate Icc from the first, and simplifying the result,

j T kfc2Tcc
V f ' j T Lf 1  I f ' j T Lf C (5.96)
1  j T Tcc
where Lfc
kfcc ' (5.97)
(Lff Lcc)
is the coupling coefficient between the two circuits and
Lcc
Tcc ' (5.98)
Rcc  Rcb
is the open-circuit time-constant of the circuit of short-circuited coils; Rcc is the parallel resistance of the
two coils, and Rcb is the resistance through the brush.
The complex coefficient C represents the effect of the short-circuited coils not only on the imaginary
component of the of the field circuit impedance, but on the entire reactive voltage in the d-axis.
Moreover, because the flux is proportional to If, the same coefficient is used to modify the complex AC
EMF in the calculation of the equivalent AC circuit.
Page 5.40 SPEED's Electric Motors
REFERENCES

[1] Hamdi, ES [1994] Design of small electrical machines, John Wiley & Sons, Chichester, ISBN 0 471
95202 8
[2] Perrine, R [anticipated 1999] DC motor design : contact Motorsoft Inc., P.O. Box 442, 30 E. Mulberry
St., Suite 1, Lebanon, OH 45036, USA [e-mail: motorsoft@msn.com]
[3] Gieras JF and Wing M [1997] Permanent magnet motor technology, Marcel Dekker, New York, ISBN
0-8247-9794-9
[4] Clayton AE and Hancock NN [1966] The performance and design of direct current machines, Pitman
& Sons Ltd., London
[5] Kostenko M and Piotrovsky L [1974] Electrical machines, MIR Publishers, Moscow
[6] Sen PC [1981] Thyristor DC drives, John Wiley & Sons Inc, New York, ISBN 0-471-06070-4
[7] Staton DA, McGilp MI, and Miller TJE [1994] Interactive computer-aided design of permanent-
magnet DC motors. IEEE Transactions on Industry Applications. Vol. 31, No.4, July/August 1995,
pp 933-940
[8] Miller TJE, McGilp MI, Staton DA, Bremner JJ [1999] Calculation of inductance in permanent-
magnet DC motors, IEE Proceedings on Electric Power Applications., Vol 146, No. 2, March 1999,
pp. 129!137.
[9] Moczala H et al : [1998] Small electric motors, IEE Power and Energy Series No. 26, London, ISBN
0 85296 921 X
[10] The Electro-craft engineering handbook, Reliance Motion Control, Inc.
[11] Openshaw Taylor E: [1958] The performance and design of A.C. commutator motors, Pitman
[12] Adkins B and Gibbs WJ [1951] Polyphase commutator machines, Cambridge University Press.
[13] Spreadbury FG [1951] Fractional HP electric motors, Pitman, London
[14] Adkins B [1957] The general theory of electrical machines, Chapman & Hall, London
5.2 Wound-field (universal) commutator motors Page 5.41
Index
Armature inductance 1, 10, 19 Multiplex 15, 16
Armature resistance 1, 4, 10, 21 No-load speed 2, 6, 7
Armature windings 14, 24, 29 Operating characteristics 8
Automotive 1 Parallel paths 1, 16-18, 20, 21, 24, 26, 27
Back pitch 15, 17 Permeance coefficient 12
Back-EMF 1, 2 Phase-controlled rectifier 1, 10
Back-EMF constant 1, 2 Pitch 14, 15, 17, 19, 27, 28, 30, 36
Bar pitch 28 Plex 15-17
Battery 1 Power 1-3, 7, 8, 21, 35, 40
Brush 1, 4, 9, 15, 16, 18-21, 24-30, 32, 34, 35, 38, 39 Progressive 15-17
Brush shift Reactance voltage 15, 38
effect of 26, 29, 33, 34 Rectifier 1, 10
Retrogressive 15, 17
Brushes 4, 16, 18-20, 22-30, 32-35, 38, 39 Saturation 2, 7, 10, 12, 23, 24, 26, 30, 31, 34, 39
Chopper 1, 10 Series resistance 2
Commutation 38 Shaft torque 2, 4, 6
effect of short-circuited coils 39 Short-circuited coils 39
Demagnetizing 13, 25, 26, 29 Simplex 15, 16
Duplex 15, 16 Slot-leakage 10, 34, 39
Efficiency 2, 8, 21 Span 15
Electric circuit 1, 32 Throw 15, 17
Electric loading 13 Time constant 9, 10
Flux/pole 1, 33 Torque 2-10, 13, 14, 21-25, 29, 34, 35
Form factor 13 Torque constant 2-4, 21
Front pitch 15 Torque per rotor volume 13
Inertia 8 Torque/speed characteristic 2
Iron loss 2, 35 Triplex 15, 16
kE 1-4, 6, 7, 9, 10, 21 Wave winding 14, 17
kT 2-4, 6, 7, 21 Windings 1, 14-17, 24, 29, 34
Lap winding 14, 16, 17, 19
Losses 8, 36
Magnet operating point 12, 13
Magnetic circuit analysis 11
Magnetic loading 13
Maximum efficiency 2, 8
S P E E D 's E L E C T R O M A G N E T IC P R I M E R

1. INTRODUCTION

Electromagnetics can be a bit confusing?

Not to worry! It took the philosophers about nineteen centuries to figure out even
the rudiments of the subject. So we shouldn’t feel too discouraged.

After the philosophers, the engineers got to work. Working about 100 times faster
than the philosophers they applied the subject so vigorously that by 1900 many of
the fundamental torque-producing mechanisms had been developed, including the
d.c. commutator machine, and a.c. synchronous and induction machines. The
transformer, too, came at about this time. Even though it produces no torque, it's
quite a useful thing.

Thanks to the early engineer s, the form of all these electromagnetic energy
converters is such as to exploit the fundamental elect romagnetic principles in
their simplest form. In other words, electric machines is a good place to begin the
study of electromagnetics.

All electromagnetic energy converters comprise two fundamental elements :


(a) an electric circuit, and
(b) a magnetic circuit.

With the exception of the transformer, they all also have a third element :
(c) a mechanical system of moving parts.

We’ll forget this and concentrate on the electric and magnetic parts.
Page 2 SPEED's Electromagnetic Primer

2. E N G I N E E R I N G EF F E C T S OF MA G N E TI C FI E L D S AN D EL E C TR I C CU R R E N T S

There were three natural philosophers who did more than any others to get the
engineers started in their exploitation of electromagnetic phenomena. They were
Oersted, Ampere and Faraday 1 , and their most important work was done between
1820 and 1831. Our modern view of electromagnetics owes much to Clerk Maxwell,
who formulated the basic laws mathematically as Maxwell's Equations. Others
made fundamental contributions in the application of the electromagnetic laws.
One of these is Steinm etz , who is credited with some of the basic early work on
phasors and the properties of electrical steels.

It is perhaps bes t if we take the three original discoverers in the wrong


chronological order.

Faraday discovered that an electromotive force ( EMF ), and from it a current, could
be generated by means of a changing magnetic field. The change in the magnetic
field can arise i n two ways, corresponding t o t he t rans for m er and t he "dynamo-
electric" machine.
A coil of copper wire which is stationary in a magnetic field of varying intensity
has a voltage ( EMF ) induced in it. This is "transformer action".
A coil which is moving relative to a magnetic field of fixed intensity also has an
EMF induced in it. This is "generator action" (sometimes called "flux-cutting").

Faraday formulated this discovery precisely: the EMF induced is proportional to


the number of turns, N, on the coil; and is also proportional to the rate of change
of what he called “magnetic flux”, N. Thus

(1)

But what is magnetic flux? What is a magnetic field?

Flux is an abstract concept, justified by its usefulness. 2 It is so useful, and we talk


about it s o freely, t hat i t i s easy t o come t o t hink t hat i t r eally ex is ts . In r eality
the only evidence for its existence is the very phenomenon it is suppos ed to
explain — induced voltage!

1
In the United States, Joseph Henry made the di scovery of electromagnetic induction at
about the same time as Faraday. Faraday i s a ccorded priority because he was first to publish his
results. H enry's name is used as the unit of inductance.
2
The word "flux" means "flow", and magnetic flux is mathematically analogous to other
quantities which obey Laplace's equation and are known as "streamlines". Many examples exist in
aerodynamics, fluid flow, electrostatics, and other fields.
SPEED's Electromagnetic Primer Page 3

Fig. 1 Flux linking a coil

We usually think of flux in terms of flux lines, or “lines of force” which l ink t he
coil. This is how Faraday thought of it. 3

So far, the only way we can define the flux N numerically is in terms of Faraday’s
Law itself. (We can measure e, but we have no independent way of measuring N).
Solving eqn. (1) for N, we get

(2)

The concept of magnetic flux is suggested by another (earlier) experiment of


Faraday’s which is of equal important in applications: that there is a physical
force acting on a conductor which carries current through a magnetic field. This
is the historical basis of motor technology. Faraday formulated this discovery
precisely also: the force is proportional to the current, to the length of the
conductor and to what we now call the magnetic “flux-density”, B:
(3)
The three quantities F, B and i are mutually at r ight angles in space. The flux-
density B can be thought of as the number of flux lines passing through an area of
one square metre. In general the flux passing through any area A is given by the
simple integral,

(4)

If the flux-density is uniform (i.e. if B does not vary across the area A), then this
relationship simplifies to
(5)

3
The lines are defined by another property of the field which is that they are the paths which
would be followed by isolated magnetic poles free to move in the field.
Page 4 SPEED's Electromagnetic Primer

3. T H E ES T A B L I S H M E N T OF MA G N E T I C FI E L D S BY EL E C T R I C CUR RE N T

Fig. 2 Magnetic field produced by a current

Although Faraday’s discoveries give us a precise way of quantifying flux and flux-
density [eqns. (1) and (3)], they cannot be used for engineering design and
analysis unless we can calculate N or B by some other, independent, method. For
example, we cannot predict the EMF induced in a coil unless we already know the
rate of change of the flux linking it.

Up to now we’ve probably been vaguely aware that the flux discussed in Section
2 was caused or established by another current not shown in the diagrams, or
perhaps by a permanent magnet. This is t r ue. The basic law governing the
establishment of magnetic fields by electric current was formulated by Ampere,
following experiments by Oersted which shows that electric currents do indeed
establish magnetic fields.

Ampere’s Law says that there is a magnetizing force, or magnetic field strength
H, encircling any electric current and that the integrated value of H around any
closed contour surrounding the current is equal to the current itself. Thus,

(6)

where ds is an element or the path. The current is sometimes called the total
"magnetomotive force" ( MMF ) in this context. If there are I s t r ands or turns of
conductor, each car r ying the current i, then the MMF is F = Ni. The simplest
example is that of a long, straight wire. If we take our closed cont our to be any
circle of r adius a with the current Ni at the centre, then by symmetry H is the
same all the way around and

(7)

H in this case is in the circumferential direction, as shown in the diagram.


SPEED's Electromagnetic Primer Page 5

4. M A T E R I A L EF F E C T S : PE R M E A B I L I TY

Fig. 3 B/H relationship for iron and air

We now have two apparently independent systems of equations for describing the
magnetic field; one in terms of B (derived from Faraday) and one in t er m s of H
(derived from Oersted and Ampere). We have t o r elate t hese two systems to one
anot her to get a complete set of equations capable of describing the magnetic
field.

It turns out that the relationship between B and H is a property of the medium
(i.e., material) in which they are measured. This property is called the
permeability, :, and is the ratio between B and H:
(8)
2
Since B has the units Wb/m and H has the units A/m, it turns out that : has the
units of inductance per unit length, H/m, and it has sometimes been called the
“specific inductivity” of a material.

In air, : has the value

(9)

In iron, : is not constant and we often represent the relationship between B and
H graphically:

For low flux-densities, : in iron can be hundreds and even thousands of times
greater than : 0 . We say that iron is "highly permeable" and observe that in iron
only a small value of H is required to achieve a large value of B.
Page 6 SPEED's Electromagnetic Primer

As an example, suppose we enclose the current of a previous diagram by an iron


ring. H inside the iron will have the same value as before (by symmetry), i.e. H =
i/2Ba. But B has the value :H, which is much larger than the value outside the
ring, or in the absence of the ring.

The high flux-density exists only within the iron. To get access to it (for example,
to produce a force on a current carrying conductor), we must cut a gap in the ring,
as shown :

Fig. 4 Effect of an airgap in a magnetic circuit


What is the flux-density in the gap?

If we assume now that the iron is infinitely permeable then, since Ampere’s Law
still holds for H, the integral ŠH.ds must be developed entirely across the gap, and
is equal to H × g. This is equal to i (or Ni if there are N turns or strands). In other
words, all the MMF is expended in forcing flux across the gap and none in the iron.
The flux density in the gap is just : 0 H g = : 0 Ni/g.

If, instead of a long straight wire, we have a complete coil wound around the iron,
the topology becomes essentially the same as that of an electric machine or
transformer, except that the moveable parts are still missing.

Another way of looking at the high permeability of iron is in terms of "induced


magnetization". Permanent magnets acquire this induced magnetization when
they are magnetized (by a coil with a high current), and they have the property of
retaining this m agnet ization when the current is switched off, so they tend to
sustain the flux. This shows t hat the permanent magnet has an internal
"excitation" similar to that of the original magnetizing current.
SPEED's Electromagnetic Primer Page 7

It may bother some people as to why we have two parameters (B and H) to describe
magnetic fields. What’s the difference between them anyway?
(a) B obeys Faraday’s Law (through N = BA): H does not.
(b) H obeys Ampere’s Law: B does not.

Therefore we have no choice but to use both. To solve magnetic problems we must
relate Ampere’s Law with Faraday’s Law and this we do through t he
"constitutive" relationship B = :H.

5. G A U S S ’ LA W
Gauss' Law states that magnetic flux always flows in closed loops.
Mathematically it is stated as

(10)

where dS is an element of the area over which B is being integrated. This is a


mathematical way of stating that there are no isolated magnetic poles or
"charges". If we draw any closed surface, the flux entering must equal the flux
leaving. A simple example is the polepiece of a laboratory magnet that is tapered
to concentrate the flux, i.e. to increase the flux-density :

(11)

An alternative to the "integral" formulation of Gauss’ Law is t he "differential"


formation: div B = 0, which is one of Maxwell's equations.

Un i t s — In the SI system of units, A is measured in square metres and N in


Webers. One Weber equals one volt-second, which is clearly the correct unit for
eqn. ( 2), if e i s m easured i n v olts and t i n s econds. Consequent ly, B i s m easured
2
in Webers per square metre (Wb/m ) and this unit is called the Tesla, after the
inventor of the induct ion motor. In the USA, B is often measured in lines per
square inch, the flux being measured in lines (also called "maxwells"). One Weber
8
equals 10 lines.
Page 8 SPEED's Electromagnetic Primer

6. SUMMARY

The basic laws of magnetics are :

(a) Faraday’s Law e = N dN/dt

(b) Ampere’s Law H.ds = Ni


m

(c) Material property B = : H between B and H in any material

(d) Gauss’ Law B.dS = 0


m

7. EXTRAS

Pe rm a n e n t m a g n e t s behave like sources of flux, but with very low permeability


called the recoil permeability. They are described by the equation

(12)
where B r is the remanence and : rec is the relative recoil permeability (i.e., relative
to : 0 ). This equation describes the so-called r ecoil line, which is an
approximation to a very narrow minor hys teresis loop along which the magnet
operates.

Fl u x - l i n k a g e is often defined as the product of "flux × turns", NN. In magnetic


devices it is always the case that "not all the flux links all the turns", and this
gives rise to a distinction between "leakage flux" and "main flux" or "mutual flux".
But flux-linkage is always the integral of the induced voltage e in eqn. (1).

V e c t o r p o t e n t ia l is another magnetic field variable like B or H (and like them it


is a vector quantity). It is defined such that B can be derived from it by
differentiation. The two vital things to know about vector potential are
(a) it combines two or more Maxwell's equations int o a single equation
(Laplace's or Poisson's equation) that can be solved by the finite-
element method; and
(b) the flux linkage of a coil can be calculated directly from the difference
in vector potential between the two coil sides.
SPEED's Electric Motors

Problems
1. Foundation

Sizing, gearing, permanent magnets, permanent-magnet equivalent circuits, cooling

Answers

1.2 B
1.3 B
1.4 A
1.5 A — stator; B — rotor
1.6
1.7 C
1.8 *BmHm*
1.9 398 J
1.10 C
1.11 Y

1.12 36.4 mm; 51 mm


3 2
1.13 14.4 kNm/m ; 7.7 kN/m or 1.04 psi
2
1.14 3.6 A/mm
2 2
1.15 4.49 × 10!3 kg-m ; 3.35; 5,225 rad/s
3 2 3
1.16 (a) 17@1 A/mm; (b) 18@7 kNm/m , 9.35 kN/m ; (c) 823 kW/m .
3 2
1.17 (a) 145 kW/m ; (b) 1@92 kW/m ; (c) 215EC; (d) 49@5EC.
1.18 (a) 1.37EC/W; (b) 32 min; (c) 765 W; (d) 51.5 min.
3
1.19 PC = 2.618; Bm = 0.2894 T; Bg = 1.1056 T; Energy product = 25.5 kJ/m (3.2 MGOe)
3
1.20 PC = 1.8933; Bm = 0.6544 T; Bg = 0.6567 T; Energy product = 180 kJ/m (22.6 MGOe)
3
1.21 PC = 3.9184; Bm = 0.7967 T; Bg = 0.2033 T; Energy product = 130 kJ/m (16.2 MGOe)
1.22 (a) 0.55; (b) Bm = 0.4 T, Hm = 111.4 kA/m; (c) 23%; (d) 0.27.

Full solutions are given at the end


1.1 Sketch the complete B-H curve for a typical ‘hard’ permanent magnet. Indicate the remanent
flux-density, the coercive force, and the recoil permeability.
1.2 In which of the following electrical machines would you expect to find permanent magnets?
A Induction motor
B Brushless DC servomotor
C Variable-reluctance stepper motor

1.3 Which of the following motors never has permanent magnets?


A DC commutator motor
B induction motor

1.4 Which of the following materials is a 'hard' permanent magnet?


A Samarium Cobalt
B Stainless steel
C Gallium Arsenide

1.5 In the following motors, are the permanent magnets on the rotor or the stator?
A DC commutator motor
B Brushless DC motor

1.6 State Ampere's Law.

1.7 Which of the following units are identical to the henry?


A Wb/At (Webers per ampere-turn)
B At/Wb (Ampere-turns per Weber)
C V-s/A (Volt-seconds per ampere)

1.8 Define the energy product of a permanent magnet.

1.9 If magnetic flux could be bottled, how much energy would be stored in a one-litre bottle
containing flux at a uniform density of 1.0T ?

1.10 Large magnets are sometimes assembled from smaller magnets in much the same way as a wall
is built from bricks. It is necessary to test the polarity of each small magnet before adding it to
the assembly. If the material is high-energy Neodymium-Iron-Boron, which of the following
polarity tests would you recommend? Give reasons for your choice.
A floating the small magnet on a cork in water and noting which way it points in the earth's
field;
B offering the small magnet up to a compass and noting which way the compass needle
swings;
C offering the small magnet up to another small magnet whose polarity is known, and
noting whether the poles attract or repel.

1.11 An electric motor contains coils and magnets and the flux is fixed in magnitude. Can the
flux-linkage of any coil vary? Y — yes, N — no.
1.12 Calculate the rotor diameter and length of a motor that develops 100 W shaft power at 1500 rpm
3
with a torque per unit rotor volume of 12 kN-m/m and a rotor length/diameter ratio of 1.4.

1.13 An AC motor has an electric loading of 12 A/mm and an average airgap flux-density of 0.6 T. What
is the torque per unit rotor volume and the electromagnetic shear stress in the airgap? Assume
a fundamental winding factor of 0.9, and assume that the rotor flux is orthogonal to the stator
ampere-conductor distribution.

1.14 In the motor of problem 1.13, the slot fill factor is 0.42, the slot depth is 18 mm, and the tooth-width
to tooth-pitch ratio is 0.56. Estimate the current density in the winding.

1.15 A brushless servo motor has a peak rated torque of 14 Nm and a torque/inertia ratio of 35,000
2 2
rad/s . It is found to be capable of accelerating an inertial load at 4,600 rad/s through a 2:1
2
speed-reduction gearbox. Estimate the inertia of the load in kg-m . What value of gear ratio would
result in the maximum acceleration of this motor and load combination, and what would be the
value of the maximum acceleration?

2
1.16 (a) An AC induction motor has 24 slots, each with a slot area of 62.2 mm . The stator bore
diameter is 52.2 mm. The factory can manufacture windings with a slot-fill factor of 47%.
2
If the maximum current density allowable in the copper is 4 A/mm , calculate the
maximum electric loading.
(b) The fundamental winding factor of the induction motor in part (a) is 0.911. The motor can
be operated with a peak airgap flux-density of 0.85 T, sine-distributed. Calculate the
3 2
torque per unit rotor volume in kNm/m and the airgap shear stress in kN/m , using the
electric loading from part (a).
(c) If the stator OD is 106 mm and the stack length is 57 mm, calculate the power per unit
stator volume when the motor delivers its rated torque at 1745 rev/min.

1.17 (a) If the efficiency of the motor in Q1.16 is 85%, calculate the average power loss per unit of
stator volume.
(b) Assume that 50% of the power loss is removed by conduction through the motor
mountings, and 50% at the curved cylindrical surface area of the stator. Calculate the
heat transfer rate required at the stator surface in kW/m2.
(c) Estimate the temperature rise if the heat removal at the curved cylindrical surface of the
stator is by natural convection. Comment on the result.
(d) Estimate the temperature rise if the heat removal at the stator surface is by forced
convection with an air velocity of 5.5 m/s.

1.18 (a) At rated power, the motor of Q1.16 has a winding temperature rise of 100EC when the
ambient temperature is 35EC. Calculate its effective thermal resistance with forced
convection as in Q1.17(d).
(b) If the thermal capacity of the motor is 1.4 kJ/EC, calculate the thermal time-constant
corresponding to the conditions in (a).
(c) Assuming constant efficiency, what is the maximum power output that can be sustained
for 25 min without exceeding the rated temperature rise?
(d) What is the maximum time for which the motor can operate at 125% of rated power
without exceeding the rated temperature rise? (Assume cold start.)
Fig. 1.8

1.19 Fig. 1.8 shows a magnet with pole-pieces arranged to focus flux into a circular airgap of length
10mm and diameter 100mm. The magnet dimensions are 100 × 150 × 100 mm. Determine the
permeance coefficient, the magnetic flux-density in the magnet and in the airgap, and the magnet
energy-product. Neglect fringing and assume that the pole-pieces are infinitely permeable. The
magnet remanent flux-density is 0.4 T and its relative recoil permeability is 1.0.

Fig. 1.9

1.20 Fig. 1.9 shows a rotary device excited by a permanent magnet whose demagnetization curve is
straight throughout the second quadrant of the hysteresis
loop. The remanent flux density is 1.0 T and the relative recoil
permeability is 1.0. Calculate the airgap flux-density when
the rotor is in the 'aligned' position as shown. Also determine
the permeance coefficient, the magnet flux-density and the
magnet energy-product.

1.21 Fig. 1.10 shows a loudspeaker magnet assembly. If the magnet


has a straight demagnetization characteristic with a
remanent flux-density of 1.0 T and a relative recoil
permeability of 1.0, calculate the magnetic flux-density in the
airgap. Also determine the permeance coefficient and the
magnet flux-density. Neglect all fringing and leakage effects,
and assume that the steel parts are infinitely permeable.
What is the magnet’s energy product when it is installed in
the assembly?

Fig. 1.10
Fig. 1.11

1.22 Fig. 1.11 shows the demagnetization characteristics of a permanent magnet at two temperatures,
T1 = 20EC and T2 = 100EC. The magnet operates initially at point X. The horizontal axis is scaled
by :0, i.e., :0Hm is plotted in [T ]instead of Hm in [A/m].
(a) What is the permeance coefficient at point X ?
(b) The magnet operates initially at point X. Then the temperature increases to 100EC, after
which there is a change in the configuration of the magnetic circuit which causes the
permeance coefficient to increase to 3@0. Determine graphically the values of Bm and Hm
at the new operating point.
(c) If the magnet temperature is maintained at 100EC while the magnet is keepered (i.e., Hm
is reduced to zero), what is the loss of remanence, expressed as a percentage of the
original remanence at 20EC?
(d) What fraction of the loss of remanence in (c) is irreversible?
SPEED's Electric Motors

Solutions to Problems

1. Foundation
S1.9
S1.12
S1.13

Use eqn. (1.4):


S1.14

Use eqn. (1.8):


S1.15

Use eqn. (1.14):

Use eqn. (1.16):


S1.16
(a) Copper cross-section area per slot = 62@2 x 0@47 = 29@23 mm2.
Max. amp-conductors per slot = 29@23 x 4 = 116@9 A.
Total amp-conductors around stator periphery = 24 x 116@9 = 2806@5 A.
Stator periphery = p x 52@2 = 164 mm.
Electric loading = 2806@5 / 164 = 17@1 A/mm.

(b) Magnetic loading = Bavg = 2/p x Bpeak = 2/p x 0@85 = 0@541 T.


TRV = (p/%2) kw1 BA = p/%2 x 0@911 x 0@541 x (17@1 x 103) = 18@7 kNm/m3.
Airgap shear stress F = TRV/2 = 9@35 kN/m2.
[Note: in Imperial units F = 9360 /4@45 /1550 = 1@36 lbf/in2 since 1 lbf = 4@45 N and 1 m2 = 1550 in2.]

(c) The rated torque is the torque produced when the TRV = 18@7 kNm/m3, since this corresponds
to the maximum permitted electric and magnetic loadings.
Rotor volume Vr = p/4 x Dr2 x Lstk = p/4 x (52 x 10!3)2 x 57 x 10!3 = 1@211 x 10!4 m3.
(note: we’ve assumed the airgap is small so that Dr . 52 mm)

Torque = Vr x TRV = 1@211 x 10!4 x 18,700 = 2@27 Nm.


Output power = 2@27 x (1745 x 2p/60) = 414 W
Stator volume = p/4 x OD2 x Lstk = p/4 x (106 x 10-3 )2 x 57 x 10!3 = 0@503 x 10!3 m3.
Power/stator volume = 414 / 0@503 x 10!3 = 823@2 kW/m3. (= 13@5 W/in3).
S1.17

(a) Power loss = Output ! Input = Output x (1/0 ! 1) = 414 x (1/0@85 ! 1) = 73 W.


Power loss/stator volume = 73 / 0@503 x 10!3 = 145@2 kW/m3 (2.38 W/in3)

(b) Cylindrical surface area = p x OD x Lstk = p x 106 x 57 x 10!6 = 19@0 x 10!3 m2.
Required convection rate = (0@5 x 73) / 19@0 x 10!3 = 1@92 kW/m2 (= 1@24 W/in2).

(c) Use eqn. (1.28). The actual heat transfer rate is h)T, and this has to be equal to 1@92 kW/m2.
From equation (1.28),
h)T = 7@5 x ()T)1/4 x )T / (OD)1/4 = 7@5 x ()T)5/4 / (OD)1/4
so that )T = [ 1@92 x 103 / 7@5 x 1061/4 ]4/5 = 215 EC.
This is clearly much too hot, indicating that natural convection is not adequate for cooling.

(d) Use equation (1.29). The actual heat transfer rate is h)T, and this has to be equal to 1@92
kW/m2. From equation (1.29),
h = 125 x % (5@5 / 57 ) = 38@8 W/m2 /EC
so that with h)T = 1,920 W/m2, )T = 1,920/38@8 = 49@5 EC — much better.
When radiation is taken into account, the temperature rise will be even lower.
S1.18

(a) Effective thermal resistance R = )T/Q where )T is the winding temperature rise (100EC) and
Q is the total heat transfer rate (73 W).
ˆ R = 100 / 73 = 1@37 EC/W.
R represents the combined effect of conduction and convection.

(b) T = RC = 1@37 x 1400 = 1,918 s = 32 min.

(c) Use equation (1.49):


k2 = 1 / (1 ! e!ton/J) = 1 / (1 ! e!25/32) = 1@844.
The power loss (dissipation) can increase by the factor k2 to 73 x 1@844 = 134@6 W for 25 min.
If the efficiency is constant at 85%, this means that
134@6 = Output x (1/0@85 ! 1) = Output x 0@176
so the Output power = 134@6 /0@176 = 765 W.

(d) Use equation (1.53):


With k2 = 1@25 and J = 32 min,
ton = 32 ln [1@25 / (1@25 ! 1)] = 51@5 min.
S1.19

Using eqn. (1.63)...


the area ratio is

the length ratio is

the permeance coefficient is

ˆ Bm = Br x 2.618/(1.0+2.618)
= 0.7236 Br
= 0.2894 T

Bg = (Am/Ag) x Bm = "Bm
= 3.8197 x 0.2894
= 1.1056 T
S1.20

Using eqn. (1.63)...


the area ratio is

the length ratio is

the permeance coefficient is

ˆ Bm = Br x 1.8933/(1.0+1.8933)
= 0.6544 Br
= 0.6544 T

Bg = (Am/Ag) x Bm = "Bm
= 1.0036 x 0.6544
= 0.6567 T
S1.21

Using eqn. (1.63)...


the area ratio is

the length ratio is

the permeance coefficient is

ˆ Bm = Br x 3.9184/(1.0+3.9184)
= 0.7967 Br
= 0.7967 T

Bg = (Am/Ag) x Bm = "Bm
= 0.2552 x 0.7967
= 0.2033 T
S1.22

(a) PC at X = Bm/:0Hm = 0@27/0@49 = 0@55.

(b) Draw YC parallel to the recoil line through B. Draw load-line with a slope of 3. From the
intersection at Z, read off the scales: Bm = 0@4 T, :0Hm = 0@14 T, so Hm = 111@4 kA/m.

(c) The magnet ends up keepered at C, with Bm = Br = 0@5 T. Original remanence at A was 0@65 T.
The loss is 0@65 ! 0@5 = 0@15 T, i.e., 0@15/0@65 x 100% = 23% of the original remanence at A.

(d) The fraction that is irreversible is BC/AC = (0@54 ! 0@5)/0@15 = 0@27.


SPEED's Electric Motors

Problems
2. Brushless Permanent-Magnet Machines

Answers

2.2 (i) 0.291 T; (ii) 0.07257 Nm/A; (iii) 3,045 rpm; (iv) 231 A; (v) 16.75 Nm
2.3 (a) 2,050 rpm; (b) 0.604; (c) 2.585 A, 3.34 W
2.4 (a) 2.0 ms, 36.3 ms; (b) 5.55 rad/s/V; (c) 40%; (d) 4.4 Hz
2.5 (c) 1.06 mWb; (d) 0.50 T; (e) 3,064 A, !383 A/mm; (f) 31.8 mV-s; (h) 12.7 V; (i) 0.19 mH; (j) 159 mJ
2.6 (a) 3,810 rpm; (b) 153.3 A, 18.4 Nm; (c) (i) 1.029 Nm; (ii) 8.575 A; (iii) 22.06 W; (iv) 85%;
(v) 17.15 W; (vi) 81.4%
2.7 191 Nm
2.8 173 V
2.9 (a) 10.8 ohm; (b) 6.612 Nm; (c) 182.33 V, 3.45E (lagging)
2.10 8,260 rpm
2.11 0.921 Nm
2.12 20.29E, 0.313 Nm
2.13 3,668 rpm
2.14 Xd = 8.8347 ohm, Xq = 27.405 ohm

Full solutions are given at the end


2.1 (a) Draw the drive circuit for a delta-connected brushless DC squarewave motor.
(b) The attached blank circuit diagrams show a wye-connected brushless DC motor and
drive. Highlight the main conduction loops for the following conditions:
(i) normal conduction in lines A and C, with two transistors conducting;
(ii) normal conduction in lines A and C, with only one transistor conducting;
(iii) during the commutation of current from line C to line B; and
(iv) when the machine is generating at high speed, with current in two lines.
2.2 A 3-phase, 4-pole, wye-connected brushless DC motor has a stator bore diameter D = 52 mm, and
stack length Lstk = 50mm. The number of turns in series per phase is 48, and the resistance
measured between terminals is 0.052 ohm. When it is driven at 1,000 rev/min by an auxiliary
motor, the line-line EMF has a peak value of 7.6 V. Calculate

(i) the peak value of the airgap flux-density due to the magnets.
(ii) the torque constant k T
(iii) the no-load speed if the supply voltage is Vs = 24 V
(iv) the locked-rotor current
(v) the locked-rotor torque

2.3 A wye-connected brushless DC squarewave motor has a torque constant of 0.177 Nm/A and its
phase resistance is 0.5 ohm/phase. It operates with a DC supply voltage of 48 V, and the on-state
resistance of each MOSFET in the controller is rDS(on) = 0.5 ohm. Calculate
(a) the base speed in rev/min if the set-point current is 5 A;
(b) the switching duty-cycle of the controller transistors at half the base speed;
(c) the RMS current in each transistor, assuming that all transistors perform the same duty
in sequence during each cycle of 360E elec. Also calculate the conduction loss in each
MOSFET transistor at half speed.

2.4 A wye-connected brushless DC motor has a back-EMF constant of 18.85 V/1000 rpm. Its line-line
inductance is 4.7 mH and its phase resistance is 1.175 ohm. The motor inertia is JM = 0.5 × 10!3
kg-m2 and it is coupled to a load whose inertia is 3JM, referred to the motor shaft.
(a) Calculate the electrical and mechanical time-constants.
(b) The motor is required to move a polishing head on a machine for polishing special
aluminium mirrors. At the beginning of the process the polishing head cycles back and
forth at a frequency of 0.1 Hz. As the polishing process continues, the cycle time
decreases until the frequency is 10 Hz. Calculate the value of the gain G(jT) =
Tm(jT)/V(jT) at the low initial frequency.
(c) Calculate the amplitude of the polishing stroke at 10 Hz, as a percentage of its value at
0.1 Hz.
(d) Estimate the frequency at which the amplitude of the polishing stroke is reduced to 70%
of its value at the start of the process.
2.5 A brushless DC motor has the cross-section shown in Fig. 2.5 with D = 60 mm (stator bore
diameter), Lm = 8 mm (magnet length), g = 0.4 mm (airgap), and Lstk = 35 mm (axial length).
Slotting is neglected and a single full-pitch stator coil is shown with 30 turns. The ceramic
magnet has Br = 1.25 T and µrec = 1.05. The current in the stator coil is zero.

Fig. 2.5

(a) Sketch the magnetic field set up by the magnet, by drawing 10 flux lines.
(b) Sketch the variation of the radial component of airgap flux-density B around the inside
of the stator, from 0 to 360E.
(c) Draw an equivalent magnetic circuit and use it to calculate the flux crossing the airgap
under each magnet pole, assuming that 10% of the magnet flux fails to cross the airgap.
Assume that the permeability of the steel in the rotor and stator is infinite.
(d) Deduce the value of Bg in the airgap at the centre of the magnet arc, i.e. on the direct
axis.
(e) Determine the MMF across the magnet and the internal magnetizing force Hm.
(f) Calculate the flux-linkage of the stator coil in the position shown.
(g) Sketch the waveform of the coil flux-linkage as the rotor rotates through 360E.
(h) Determine the waveform and the peak value of the EMF induced in the stator coil if the
rotor rotates at 4,000 rev/min.
(i) Estimate the inductance of the stator coil.
(j) If the current in the stator coil is maintained constant at 5 A, determine the mechanical
work that is done in rotating the rotor 180E from the position shown. Is this work
positive or negative? Explain what you mean by “positive” or “negative”.
2.6 A wye-connected PM brushless DC motor operated in squarewave mode has a torque constant
of 0.12N-m/A referred to the DC supply. The supply voltage is 48 V DC.
(a) Estimate its no-load speed in rev/min.
(b) If the armature resistance is 0.15 ohm/phase and the total voltage drop in the controller
transistors is 2V, determine the stall current and the stall torque.
(c) The motor is delivering 330W of mechanical power to a load at 3,400rev/min. The voltage
drop across each transistor is a constant 1 V. The friction torque has been separately
measured as 0.046N-m at this speed, and the iron loss can be taken as a constant
mechanical loss of 20W. Calculate
(i) the electromagnetic torque Te
(ii) the current I
(iii) the power loss in the winding resistance PCu
(iv) the motor efficiency
(v) the total power loss in the transistors, and
(vi) the overall efficiency of the drive and motor.

Fig. 2.7

2.7 Fig. 2.7 shows the top view of a permanent-magnet actuator used for guiding cables on to a
vertical-axis cable-laying drum. The length of the arc-shaped magnets in the direction of
magnetization is 60 mm and that of the rectangular magnets is 55 mm. The vertical height of the
electromagnetic assembly is 250 mm. The airgap length is 10 mm. If the magnet is Ferrite with
a remanent flux-density of 0.4 T and relative recoil permeability of 1.0, calculate the flux-density
in the airgap, assuming infinitely permeable iron components. Neglect fringing and leakage.
The magnetic flux path is completed by soft iron rings. Part of the inner ring is encircled by a
coil of 200 turns mounted on the rotating 'armature'. If the current in the coil is 20 A, calculate
the torque.
2.8 A 3-phase, 4-pole brushless PM motor has 36 stator slots. Each phase winding is made up of 3
coils per pole with 20 turns per coil, and all the coils are in series. The coil span is 7 slots. If the
fundamental component of magnet flux is 1.8 mWb, calculate the open-circuit phase EMF E at
3,000 rpm.

2.9 The stator bore diameter of the motor of Problem 2.8 is 102 mm and its axial length is 120 mm.
The airgap is 1mm, and Carter’s coefficient for slotting is 1.055. The magnet is mounted on the
rotor surface and has a radial thickness of 9.5 mm and a relative recoil permeability of 1.06.
(a) Calculate the airgap component of synchronous reactance at 100 Hz.
(b) What is the electromagnetic torque at 3,000 rpm if the phase current is 4.0 A and ( = 0?
(i.e. the current is in the q-axis).
(c) The phase resistance is 3.7 ohm and the leakage reactance is 5 mH. With the phase
current of 4.0 A at ( = 15E at 3,000 rpm, calculate the terminal voltage and the
power-factor angle.

2.10 A brushless permanent-magnet sinewave motor has an open-circuit voltage of 173 V at its
corner-point speed of 3,000 rpm. It is supplied from a PWM converter whose maximum voltage
is 200 V RMS. Neglecting resistance and all other losses, estimate the maximum speed at which
maximum current can be supplied to the motor.

2.11 Show that with rated phase current I, the angle ( which maximises the torque of the hybrid
PM/reluctance motor satisfies the relationship sin (/cos 2( = )X.I /Eq, where )X = Xq ! Xd.
Hence show that for a two-phase motor with Eq = 35.8 V, Xd = 1.18 ohm, and Xq = 2.47 ohm at 3,000
rpm, and rated current of 4.0A, the torque is maximised with ( = 7.97E. What is this maximum
torque? Neglect losses.

2.12 If the high-energy magnets in the motor of Problem 2.11 are replaced by ceramic magnets, the
ohmic values of the reactances remain the same but the open-circuit phase EMF is decreased
from 35.8 V to 11.3V. Determine the optimum value of ( and the maximum torque, assuming
that the rated current remains 4.0A. What percentage of this maximum torque is reluctance
torque?

2.13 The absolute maximum speed of a permanent-magnet brushless motor can be estimated
approximately by assuming that the current is at its rated value and is entirely in the negative
d-axis, i.e. all the current capacity of the drive is used up in flux-weakening. If the maximum
controller voltage is Vc then, neglecting losses, V = 0 + jVq = jVc = jEq + jXdId where Id is
negative. If Vc = 38 V and Eq = 35.8 V at 3,000 rpm, and if Xd = 1.18 ohm at 3,000 rpm, estimate the
absolute maximum speed with I = 4.0 A, remembering that both Eq and Xd are proportional to
frequency or speed.
2.14 Calculate the synchronous reactances Xd and Xq of the interior permanent-magnet motor shown
in Fig. 2.14 at 1000 rpm, given the following parameters: Stator bore diameter D = 50.8 mm;
airgap length g = 0.4 mm; stack length Lstk = 50 mm; pole arc $M = 120E elec. Carter coefficient
= 1.177. Winding factor kw1 = 0.837; No. of turns in series per phase = 480. Magnet length in
direction of magnetization Lm = 5.5 mm; magnet width = 22 mm; per-unit rotor leakage
permeance prl = 0.091; magnet relative recoil permeability µrec = 1.1. Leakage reactance XF =
3.264 ohm. The width of the web in the q-axis is 1.0 mm.1

Fig. 2.14

1
This problem can be solved with PC-BDC using the [Alt+7] standard example and changing Embed = Type1, Tw = 3.5,
web = 1.0, Slots = 24, gap = 0.4, Bridge = 0.5, RNSQ = Round, TC = 60, wire = 0.5, Coils/P = 2, Throw = 5, Vs = 150, ISP = 1.0,
fChop = 24.0; with a NeIGT 30H magnet.
SPEED's Electric Motors

Solutions to Problems
2. Brushless Permanent-Magnet Machines
S2.1

(a)

(b)
S2.2

(i) kE = kT = 4pTph Mg /B = 2 Tph Bg D Lstk since Mg = Bg x BDLstk / 2p.


ˆ Bg = 0@07257 /(2 x 48 x 52 x 50 x 10!6) = 0@291 T

(ii) kT = kE = ELL/Tm = 7@6/(2B / 60 x 1,000) = 0@07257 Vs/rad or Nm/A

(iii) T0 = Vs / k E = 24/0@07257 = 318@8 rad/s = 3,045 rev/min

(iv) I0 = Vs /RLL = 24/(2 x 0@052) = 231 A

(v) T0 = k T I0 = 0@07257 x 231 = 16@75 Nm


S2.3

2 transistors and 2 motor phases


in series at any time. RDS(on) + Rph
(a)

Vs & 2 R ISP 48 ! 2 x (0.5 % 0.5) x 5


Tb ' ' ' 214.7 rad/s ' 2,050 rpm
kE 0.177

(b) At half- speed,


214.7
0.177 x % 2 x (0.5 % 0.5) x 5
2
d ' ' 0.604
48

(c) Transistor conduction waveform is

60 60
IQ1(rms) ' 5 x 0.604 x % 1 x ' 2.585 A
360 360

2
Conduction loss ' IQ1(rms) x rDS(on) ' 2.5852 x 0.5 ' 3.34 W in each transistor
S2.4
L 4.7
(a) Te ' ' ' 2.0 ms Jmotor % Jload ' Jmotor ( 1 % 3 )
R 2 x 1.175
' 0.125 x 4 ' 0.5 m kg&m 2
RJ 2 x 1.175 x 0.5
Tm ' ' ' 36.3 ms
kE2 18.85 30
2
x
1000 B

(b) 1/0.18
G (j T ) ' .
1 % j T x 0.002) (1 % j T x 0.0363 )
When fpolish ' 0.1 Hz, T ' 2 B x 0.1 and

1/0.18
G ' • 1/0.18 ' 5.55 rad/sec/V
(1 % j 0.00126 ) (1 % j 0.0228 )

Virtually “DC”: imaginary parts negligible.

(c)
When fpolish ' 10 Hz, T ' 2 B x 10 and

1/0.18
G ' • 1/0.18 x 0.4 e !73.5E ' 2.22 e !73.5E rad/sec/V
(1 % j 0.126 ) (1 % j 2.28 )

40% amplitude

(d)
1
' 0.7
* (1 % j 0.002 T) (1 % j 0.0363 T) *

this term is dominant


neglect

1 1
0.7 . ' (Recognizing that 70% is close
2 *1 % j1* to the “!3 dB” point)

The required frequency is approximately the frequency at which

1 1
0.0363 T ' 1, i.e. x ' 4.4 Hz .
2B 0.0363
S2.5
(a)

(b)

(c)

Since 10% of Mm fails to cross the gap, Mg/Mm = 0.9: i.e. f LKG = 0.9.
To calculate the magnet pole area Am, take a cylindrical surface 1/3 of the way through the magnet,
from the inner surface: the radius is 60/2 ! 0.4 ! 8 = 21.6 mm; the arc is 120E (by scaling from the
diagram – it is not given in the problem specification); and the length is 35 mm. Thus

B
Am ' x 120 x 21.6 x 35 ' 1583 mm 2
180
The permeance of one magnet is

µr µ0 Am 4 B x 10!7 x 1.05 x 1583 x 10!6


Pm0 ' ' ' 2.61 x 10!7 Wb/At
Lm !3
8 x 10

To calculate the airgap area Ag (one pole), take a cylindrical surface at the stator bore, radius 30 mm.
The length is Lstk = 35 mm and the arc is approximately 120E, so

B
Ag ' x 120 x 30 x 35 ' 2119 mm 2
180
The airgap reluctance (neglecting fringing) is

g 8 x 10!3
Rg ' ' ' 2.895 x 106 At/Wb
!7 !6
µ0 A g 4 B x 10 x 2199 x 10

From eqn. (2.18),

Mr 1.25 x 1583 x 10!6


Mg ' ' ' 1.06 x 10!3 Wb
1 1
% Pm0 Rg % 2.61 x 10!7 x 2.895 x 10!6
fLKG 0.9

(d) Bg = Mg/Ag = 1.06 x 10!3/(2119 x 10!6) = 0.500 T.


(e) The flux through the magnet is Mm = Mg/fLKG = 1.06 x 10!3/0.9 = 1.18 x 10!3 Wb, so Bm = Mm/Am =
118 x 10!3/(1583 x 10!6) = 0.744 T.
B r ! Bm 1.25 ! 0.744
Hm ' ' ' ! 383 A/mm
µ0 µrec 4 B x 10!7 x 1.05
MMF across magnet is Fm = HmLm = 383 x 8 = 3,064 A.
(f) The flux-linkage of the stator coil is 30 x Mg = 30 x 1.06 x 10!3 = 31.8 mV-s.

(g)

m
This waveform is obtained by integrating N Bg r Lstk d 2 where N is the number of turns in the coil.
(h) The EMF waveform has the same shape as the Bg waveform because the coil has a pitch of 180E.
The stator coil flux-linkage changes from !31.8 mVs to +31.8 mVs in about 120E, so the peak value of
EMF is

dR dR 2B 2 x 31.8 x 10!3
epk ' ' Tm ' 4000 x x ' 12.7 V
dt d2 60 B
120 x
180

(i) The permeance P “seen” by the coil is approximately µ0A/h, where A is a cylindrical surface of about
180E arc and 35 mm length at radius 30 ! (0.4 + 8)/2 = 25.8 mm, and h = 2 x (8.4) = 16.8 mm, so

µ0 A 25.8 x B x 35 x 10!6
P ' ' µ0 ' 0.169 µ0 Wb/At
h 16.8 x 10!3

Then the inductance is L = N2P = 302 x 0.169 µ0 = 0.19 mH.

(j) Work done = 1 x )Q = 5 x 31.8 x 10!3 = 159 mJ. Since the rotor is in stable equilibrium, this work
must be done by exerting a torque on the rotor. If we regard the work as mechanical input, as though
the machine was a generator, then it is positive. If we regard the work as mechanical output, as though
the machine was a motor, then it is negative.
S2.6
V 48
(a) T0 ' ' ' 400 rad / sec ' 3810 rpm
kT 0.12

48 ! 2
(b) I0 ' ' 153.3 A
0.15 x 2

T0 ' 153.3 x 0.12 ' 18.4 N m

(c) Electromagnetic (airgap) power Pg = Pmech + Pfriction + Piron loss. Since Tfriction = 0.046 W,

2B
Pfriction ' 0.046 x 3400 x ' 16.38 W
60
and Pg = 330 + 16.38 + 20 = 366.38 W. So the electromagnetic torque is

366.38
(i) Te ' ' 1.029 Nm
2B
3400 x
and therefore the current is 60

Te 1.029
(ii) I ' ' ' 8.575 A
kT 0.12

(iii) The power loss in the winding resistance is PCu = 8.5752 x 0.15 x 2 = 22.06 W.
Total power input to motor = 366.38 + 22.06 = 388.44 W;
(iv) ˆ Motor efficiency = 330/388.44 = 0.85 or 85%

(v) Power loss in transistors = 2 x 1 x 8.575 = 17.15 W


Power delivered by supply = 388.44 + 17.15 = 405.59 W
(vi) ˆ Overall efficiency of drive + motor = 330/405.59 = 81.4%
S2.7
The magnets are magnetized N-S-N-S. Analyze one half pole, which has one-half of an arc magnet in
series with one-half of a rectangular magnet in series with an airgap of arc 95/2 = 47.5E.
The airgap reluctance is

g 10 x 10!3
Rg ' ' ' 1.3713 x 105 At/Wb
µ0 A g B
4 B x 10!7 x 47.5 x x 280 x 250 x 10!6
180

The magnets are in series, so we need their “Thevenin” equivalent circuits with parameters Fm1 = Mr1/Pm1
and Rm1 = 1/Pm1 and Fm2 = Mr2/Pm2 and Rm2 = 1/Pm2:
We have Am1 = Ag = 47.5 x (B/180) x 280 x 250 x 10!6 = 0.0580 m2, and Lm1 = 60 mm, so

1 L m1 60 x 10!3
R m1 ' ' ' ' 8.232 x 105 At/Wb
!7
Pm1 µ0 µrec Am1 4 B x 10 x 1.0 x 0.0580

and Mr1 = 0.4 x 0.0580 = 0.0232 Wb so Fm1 = 0.0232 x 8.232 x 105 = 19,099 At.
Also Am2 = 340/2 x 250 x 10!6 = 0.0425 m2, and Lm2 = 55 mm, so

1 Lm2 55 x 10!3
Rm2 ' ' ' ' 1.0298 x 106 At/Wb
!7
Pm2 µ0 µrec Am2 4 B x 10 x 1.0 x 0.0425

and Mr2 = 0.4 x 0.0425 = 0.0170 Wb so Fm2 = 0.0170 x 1.0298 x 106 = 17,507 At.

The airgap flux (whole pole) is therefore

Fm1 % Fm2
Mg ' 2 x
R g % Rm1 % R m2

17507 % 19099
' 2 x
(0.8232 % 1.0298) x 106

' 0.0395 Wb

and the airgap flux-density is Bg = Mg/2Ag = 0.0395/2 x 0.0580 = 0.341 T.

The tangential force on one coilside is F = Bg x L x NI = 0.341 x 250 x 10!3 x 200 x 20 = 341 N, so the
torque is 2 F x r = 2 x 341 x 280 x 10!3 = 191 Nm.
S2.8
Total turns = 4 x 3 x 20 = 240 turns.
q = 3 slots/pole/phase
q(
sin ( = (360/36) x 2 = 20E
2 sin 30E (slot-pitch angle. Eelec)
kd1 ' ' ' 0.9598
( 3 sin 10E
q sin
2

g 1 2
kp1 ' cos ' cos B ' 0.9397
2 2 9

kw1 ' kd1 kp1 ' 0.9397 x 0.9598 ' 0.9019

kw1 Nph ' 0.9019 x 240 ' 216.5 turns

Frequency f = 3000/60 x 2 = 100 Hz, so

2B
E ' x 216.5 x 1.8 x 10!3 x 100 ' 173.1 V rms
2
S2.9
(a) Effective airgap is
Lm 9.5
gO ' gN % ' 1.0 x 1.055 % ' 10.02 mm
µrec 1.06

Then the airgap component of synchronous reactance is

6 µ0 D L stk f 6 x 4 B x 10!7 x 102 x 120 x 10!6 x 100


Xsg ' (kw1 Nph)2 ' x 216.5 2 ' 10.8 ohm
2 2 !3
p gO 2 x 10.02 x 10

(b) The electromagnetic torque is

Eq 3 x 173.1
Te ' 3 x Iq ' x 4 x sin ( 0 ) ' 6.612 Nm
Tm 2B
x 3000
60

(c) The total synchronous reactance (including leakage reactance) is

Xs ' Xsg % XF ' 10.8 % 2 B x 5 x 10!3 ' 13.94 ohm

From the phasor diagram,

V ' E % ( R ph % j Xs )II

' j 173.1 % ( 3.7 % j 13.94 ) x 4.0 e j (90 % 15E)

' 182.33 e j 108.45E V

Power-factor angle N = 108.45 ! (90 + 15) = 3.45E. PF = cos N = 0.998 lag.

Also note the solution in dq components:


V = (0 + jEq ) + (Rph + j Xs)(Id + j Iq) = (RphId ! XsIq) + j (Eq +RphIq + XsId)
In the general case (salient-pole machine) we use Xd on the d-axis with Id, and Xq on the q-axis with Iq;
but in this example we have a surface-magnet motor and so Xd = Xq = Xs.
Also Id = ! I sin ( and Iq = I cos (, so
Vd = RphId ! XqIq = 3.7 x (!4 sin 15E) ! 13.94 x 4 cos 15E = !57.6906 V
Vq = Eq + RphIq + XdId = 173.1 + 3.7 x 4 cos 15E+ 13.94 x (!4 sin 15E) = 172.9636 V
ˆ V = Vd + j Vq = !57.6906 + j 172.9636 = 182.33 ej108.45E V
S2.10

The per-unit EMF is u = 173/200 = 0.865


The ratio between base speed and maximum speed is

NQ
' u ! 1 ! u2 ' 0.865 ! 1 ! 0.8652 ' 0.3632
ND

ˆ Max speed = 3,000/0.3632 = 2.753 x 3,000 = 8,260 rpm


S2.11

T ' m p [ Qd I q ! Qq I d ] ( m ' No of phases )


X d Id Xq I q
' m p [ (Q1Md % ) Iq ! I ] ( Q1Md,Qd and Qq are all RMS )
T T d
mp
' [ E q Iq % ( X d ! Xq ) I d Iq ]
T
mp
' [Eq I cos ( % (Xq !Xd).I 2 sin ( cos ( ]
T
mp
' [Eq I cos ( % ) X.I 2 sin ( cos ( ] where ) X ' Xq ! Xd > 0
T

mp ) X.I
' [Eq cos ( % sin 2 (] I
T 2

dT mp
' [ !Eq sin ( % ) X. I cos 2 ( ] I ' 0
d( T

Eq cos 2 ( sin ( ) X.I Eq


ˆ ' or ' ' ) x (pu) , if the base impedance is
) X.I sin ( cos 2 ( Eq I

The value of ( that gives maximum torque depends on the current I. To solve for this value of (, let
s = sin (; then cos 2( = 1 ! 2 s2 and s/(1 ! 2s2) = )x, which is a quadratic equation in s, with solution

!1 ± 1 % 8 ) x 2
s '
4 )x

(2.47 ! 1.18) x 4
)x ' ' 0.1441 pu
35.8

!1 ± 1 % 8 x 0.14412
ˆ s ' ' 0.1386 or !3.608
–––
4 x 0.1441

so ( ' 7.97E .

The frequency is 100 Hz and with p = 2, m = 2 the torque is

2 x 2 2.47 ! 1.18
Tmax ' [ 35.8 cos 7.97E % x 4.0 sin ( 2 x 7.97E ) ] x 4.0
2 B x 100 2
' 0.0255 x [ 35.4542 % 0.7085 ]
' 0.921 N&m
In this example, with an Xq/Xd ratio of 2.1 and )x = 0.1441, only 2% of the maximum torque is contributed
by reluctance torque.
S2.12

( 2.47 ! 1.18 ) x 4.0


)x ' ' 0.4566
11.3

!1 ± 1 % 8 x 0.45662
s ' ' 0.3468 Y ( ' 20.29E
4 x 0.4566

2 x 2 ( 2.47 ! 1.18 ) x 4.0


T ' 11.3 cos 20.29E % sin ( 2 x 20.29E ) x 4.0
2 B x 100 2
' 0.0255 x [ 10.5987 % 1.6784 ]
' 0.313 N&m

In this example with Xq/Xd = 2.1 and )x = 0.4566 pu, the reluctance torque contributes 14% to the
maximum torque.
S2.13

We’re assuming that all the current is in the d-axis, to make the solution tractable, but this means the
torque will be zero, and therefore the solution is only approximate (to the extent that losses can be
neglected). What we’re really estimating is an upper bound for the speed, with the given motor and drive
parameters.
Iq = 0, T = 0, R = 0

Vd ' 0

Vq ' E q % Xd I d

Id ' !4.0 A ; Vq ' 38 V

N N
38 ' 35.8 x % x 1.18 x ( !4.0 )
3000 3000

N
' ( 35.8 ! 1.18 x 4 )
3000
ˆ N ' 1.2227 x 3,000 ' 3,668 rpm

The phasor diagram shows the flux-weakening effect of Id. Evidently the flux-weakening is more
pronounced when XdId is an appreciable fraction of Eq, suggesting that a high reactance extends the
speed range. Though this is generally a valid inference, it says nothing about the variation of torque
with speed. In this motor the speed range extends only 22% above 3,000 rpm, and the torque will fall
rapidly between base speed (3,000 rpm) and maximum speed (3,668 rpm).
S2.14
Use the formulas in chapter 6 of Miller [1989]2 or chapter 6 of Hendershot and Miller [1994]3 We need
to work through quite a few of them... 4
Am ' 22 x 50 ' 1100 mm 2

µrec µ0 Am 4 B x 10!7 x 1.1 x 1100 x 10!6


Pm0 ' ' ' 2.7646 10!7 Wb/At
Lm !3
5.5 x 10

Pm ' ( 1 % prl ) Pm0 ' 1.091 x Pm0 ' 3.0162 10!7 Wb/At

2B 1
Ag ' x x 25.4 x 50 ' 1330 mm 2
3 2

gN 1.177 x 0.4 10!3


Rg ' ' ' 2.8169 x 105 At/Wb
!7 !6
µ0 Ag 4 B x 10 x 1330 x 10

1 % Pm Rg ' 1 % 3.0162 x 10!7 x 2.8169 x 105 ' 1.085

4 "B 120 2
k1 ' sin ' 1.1027 (" ' ' )
B 2 180 3

sin ( " B / 2 ) sin 60E 3


k" d ' ' ' x sin 60E ' 0.827
"B/2 2 B B
x
3 2

2
sin B
sin " B 2 3
k1ad ' " % ' % ' 0.9423
B 3 B

gN gN
g dO ' ' ' 9.822 gN
k1 k" d 1.1027 x 0.827
k1ad ! 0.9423 !
1 % Pm R g 1.085
gN 1
'd ' ' ' 0.102 .
g dO 9.822

6 µ0 D Lstk f
Xd ' ( k w1 Nph )2 % XF
p g dO
2

6 x 4 B x 10!7 x 50.8 x 50 x 10!6 x 33.33


' x ( 0.837 x 480 )2 % 3.264
2 !3
2 x 9.822 x 1.177 x 0.4 x 10
' 5.5707 % 3.264 ' 8.8347 ohm

2
Brushless permanent-magnet and reluctance motor drives, Oxford University Press, 1989
3
Design of brushless permanent-magnet motors, Magna Physics Publications, 1994
4
This problem can be solved with PC-BDC using the [Alt+7] standard example and changing Embed = Type1, Tw = 3.5,
web = 1.0, Slots = 24, gap = 0.4, Bridge = 0.5, S-Slot = Round, TC = 60, wire = 0.5, Coils/P = 2, Throw = 5, Vs = 150, ISP = 1.0,
fChop = 24.0; with a NeIGT 30H magnet.
2
sin 0.0251 B ! sin B
sin S B ! sin" B 2 3
k1aq ' " % S % ' % 0.0251 % ' 0.4412
B 3 B

gN gN
g qO ' ' ' 2.2665 gN
k1aq 0.4412 gN 1
'q ' ' ' 0.441 .
g qO 2.2665
6 µ0 D Lstk f
Xq ' ( k w1 Nph )2 % XF
p gqO
2

6 x 4 B x 10!7 x 50.8 x 50 x 10!6 x 33.33


' x ( 0.837 x 480 )2 % 3.264
2 !3
2 x 2.2665 x 1.177 x 0.4 x 10
' 24.141 % 3.264 ' 27.405 ohm
SPEED's Electric Motors

Problems
3. Induction Machines

Answers

3.1 C.
3.2 7920 rpm, 12 Hz
3.3 1200 rpm, 1140 rpm, 3 Hz
3.4 (a) A,B,E; (b) C,D; (c) F.
3.6 1666 W, 0.85 (lagging)
3.7 11.76 kW, 502.6 W, 13.57 kW, 86.7%.
3.8 (a) 1485 rpm; (b) 1469 rpm; (c) 1461 rpm.
3.9 9.0, 0.886 (lagging)
3.11 25.64 kW, 87.7%; 55.8%.
3.12 (a) 0.033; (b) 31.8 kW; (c) 1.06 kW; (d) 336.5 Nm; (e) 30.66 kW; (f) 92.9%.
3.13 (a) 27.0 A; (b) 0.896 lag.; (c) 89.8%; (d) 141.9 Nm; (e) 349 Nm; (f) 159.3 Nm; (g) 156 A.
3.14 1350 rpm; 2250 rpm; freqency and amplitude are the same; phase sequence is reversed.

Full solutions are given at the end


3.1 A three-phase induction motor is fed from a three-phase supply of fixed voltage and frequency.
At what speed is its torque zero?
A Standstill
B when s < 0
C synchronous speed.
An induction motor has 2 poles and operates from a 120 Hz supply. If the slip s is 0.1, what is the
3.2
speed in rev/min? What is the frequency of the rotor currents?
3.3 What is the speed of the rotating field of a 6-pole, three-phase AC induction motor connected to
a 60 Hz supply? Give the answer in rev/min. Calculate the rotor speed if the slip is 5%. What
is the frequency of the rotor currents?

3.4 (a) In which of the following machines would you expect to find a commutator?
A A permanent-magnet d.c. servomotor;
B a series-wound d.c. generator;
C A wound-rotor induction motor;
D An a.c. steam-turbine generator;
E A “universal” motor;
F A single-phase induction motor.

(b) In which of the machines in (A) to (F) above would you expect to find slip-rings?
(c) In which of the machines in (A) to (F) above would you expect to find no brushes?
Discuss two methods for controlling the speed of a cage induction motor. Draw a sketch of the
3.5
speed/torque characteristic of an induction motor with
A low rotor resistance;
B high rotor resistance;
C a double-cage rotor.
Show a typical load characteristic on each sketch.

A 230-V, wye-connected, 3-phase AC induction motor delivers 8 Nm at 1750 rpm. If the efficiency
3.6
is 88% and the line current is 4.92 A, find the input power and power-factor.

3.7 A 4-pole, 50 Hz, 3-phase induction motor develops an electromagnetic airgap torque of 80 Nm
when running at full load. The frequency of the rotor currents is 2 Hz. Calculate the shaft power.
If the torque absorbed by windage and friction is 2 Nm, and if the stator losses total 1 kW,
calculate the rotor copper loss, the input power, and the efficiency.

3.8 A 4-pole, 50Hz, high-efficiency induction motor develops full-load torque at 1470 rev/min.
(a) What will be its speed at half rated torque?
(b) What will be its speed at half rated torque and 70% voltage?
(c) What will be its speed at rated torque and voltage, if the rotor resistance increases by 30%
as a result of temperature rise?

3.9 A 7.5-kW, 3-phase, 60 Hz, 460-V, star-connected 4-pole induction motor has a full-load speed of 1764
rev/min. In the per-phase equivalent circuit, the stator resistance is 0.25 ohm, the referred rotor
resistance is 0.5 Ohm, the total leakage reactance is 2.5 ohm, and the magnetizing reactance is 60
ohm. Calculate the ratio between the standstill current and the full-load current. What is the
full-load power factor? Ignore friction and core losses.

3.10 Explain how a rotating magnetic field is set up in the airgap of a 3-phase AC machine.
3.11 A 3-phase, 6-pole, 50 Hz wound-rotor induction motor delivers 22.5 kW at a speed of 950 rev/min
with its slip-rings shorted. Assuming constant friction torque of 1.5 Nm and constant stator
losses of 1.8 kW, find the input power and the efficiency. When the speed is reduced to 600
rev/min by increasing the rotor circuit resistance, the load torque remains constant at the
full-load value. Find the efficiency at the reduced speed. State any assumptions used.

3.12 An 8-pole, 3-phase, 60-Hz induction motor is operating at a speed of 870 rev/min. The input power
is 33 kW and the stator copper loss is 1200 W. Friction and windage loss is 80W. Core loss is
negligible. Find
(a) the slip;
(b) the airgap power Pgap;
(c) the rotor copper loss;
(d) the shaft torque in Nm;
(e) the shaft power in kW;
(f) the efficiency.

3.13 A 6-pole, 3-phase, Y-connected, 460-V, 60 Hz induction motor has the following equivalent-circuit
parameters (all in ohms):

Magnetizing reactance = 30.0 Stator resistance 0.80

Total leakage reactance = 1.40 Rotor resistance referred to stator = 0.30

If the rotor speed is 1164 rev/min, calculate


(a) the line current;
(b) the power factor;
(c) the efficiency;
(d) the shaft torque;
(e) the breakdown torque;
(f) the locked-rotor torque;
(g) the locked-rotor current.
Assume rated voltage and frequency, and ignore friction and core losses.

3.14 A 4-pole wound-rotor induction motor is to be used as a frequency-converter. The stator is


connected to a 60 Hz 3-phase supply. The load is connected to the rotor slip-rings via brushes. At
what two speeds could the rotor be driven to supply 15 Hz to the load? In what way would the
3-phase voltages at the load terminals differ at these two speeds?
SPEED's Electric Motors

Solutions to Problems

3. Induction Machines
S3.2
S3.3

S3.4 : See Answers on front page


S3.5 : See theory manual, chapter 3
S3.6
S3.7
S3.8

It is a high-efficiency motor and the slip is small, so we can assume .

At full load the slip is where

(a) If T is ½, S is ½ x 0.020, so N = 1500 (1 ! 0.01) = 1485 rpm

(b) If T is ½ (i.e. 50% of rated) and Vs is 0.7,

(c) If T is 1 (i.e. 100% of rated) and Vs is 1 and RR = 1.3, s = 0.02 x 1.3 = 0.026
so N = 1500 (1 ! 0.026) = 1461 rpm
S3.9

At full load

At standstill (s = 1)

S3.10 : See theory manual, chapter 3.


S3.11

Total mech. Shaft power Friction


power delivered to power
developed load

At 600 rpm,

Assume friction torque is constant at 1.5 Nm


S3.12
S3.13

3 phases

Ignoring friction, Pshaft = Pmech


At standstill, s = 1 and

s=1
S3.14

If fr = 15 Hz then

But s can be + or ! depending on whether the rotor is rotating slower than, or faster than, synchronous
speed respectively.

The frequency and amplitude of the voltages generated at the slip rings would be the same in both
cases; but the phase sequence would be reversed, because at 1350 rpm the rotating field is overtaking
the rotor, while at 2250 rpm the rotor is overtaking the field.
SPEED's Electric Motors

Problems
4. Switched Reluctance Machines

Answers

4.1 8.33 Nm
4.2 15E, 800 Hz
4.3 18E, 400 Hz
4.4 (a) 0.430 Nm. (b) 0.225 J; (c) 1.608 J, 3.07 Nm
4.6 (b) 1.48 T; (c) 1.09 Nm; (d) 30E
4.7 (a) 8.36 mH; (b) 8.70 A; (c) 2.62 Nm

Full solutions are given at the end


4.1 A limited-rotation actuator has a rotor winding and a stator winding. The geometry is such that
the self-inductances are constant but the mutual inductance varies linearly from zero to 109 mH
in a rotation of 75E. Calculate the torque when the windings are connected in series carrying a
current of 10A.

4.2 What is the stroke angle of a 3-phase switched reluctance motor having 12 stator poles and 8
rotor poles? What is the commutation frequency in each phase at a speed of 6,000 rpm?

4.3 What is the step angle of a 5-phase switched reluctance motor having 10 stator poles and 4 rotor
poles? What is the commutation frequency in each phase at a speed of 6,000 rpm?

4.4 A switched reluctance motor with 6 stator poles and 4 rotor poles has a stator pole arc $ s = 30E
and a rotor pole arc $r = 32E. The unsaturated aligned inductance is Lau = 10.7 mH and the
unaligned inductance is Lu = 1.5 mH, and saturation can be neglected.
(a) Calculate the instantaneous electromagnetic torque when the rotor is 15E before the
aligned position and the phase current is 7 A. Neglect fringing.
(b) What is the maximum energy conversion in one stroke if the current is limited to 7.0A?
Determine the average torque corresponding to this energy conversion.
(c) What is the flux-linkage in the aligned position when phase current is 7.0A? If this
flux-linkage can be maintained constant while the rotor rotates from the unaligned
position to the aligned position at low speed, determine the energy conversion per stroke
and the average torque.

4.5 Show that for an unsaturated switched reluctance motor operating with a fixed conduction angle
2 2
and flat-topped current waveform, the average torque is proportional to V /T where V is the
supply voltage and T is the angular velocity. Hence show that to maintain constant torque per
ampere it is necessary to maintain the 'volts per Hertz' constant. Deduce that with fixed supply
voltage, a constant-power characteristic can be obtained by making the conduction angle
proportional to the speed.

4.6 Fig. 4.6 shows the cross-section of a switched reluctance


motor with the two coils of one phase on opposite stator
poles. The rotor is in such a position that the 'overlap
angle' between these stator poles and a pair of rotor poles
is 15E. The airgap is g = 0.5 mm, stator bore diameter D =
50 mm, and axial length Lstk = 50 mm. There are 98 turns
on each stator pole. The stator and rotor pole arcs are
both 30E. Neglect fringing and leakage, and assume that
the steel parts are infinitely permeable.
(a) A current of 6 A flows through the two coils in
series. Sketch the flux paths on Fig. 4.6 for this
condition, showing six flux-lines.
(b) Calculate the flux-density in the airgap between
the active poles. Fig. 4.6
(c) Estimate the torque.
(d) Through what angle of rotation is the torque essentially constant, if the current is
constant and there is no fringing?
4.7 The switched reluctance motor in Fig. 4.7A has an airgap g = 0.2 mm, a stator outside diameter
of 79.2 mm, a pole arc of 13 mm, and a stack length Lstk = 50 mm. All other dimensions can be
scaled from the diagram. Also shown are the two coils of phase 1. Each coil has 32 turns.
(a) Calculate the unsaturated aligned inductance.
(b) Estimate the current is required to bring the airgap flux-density to 1.75 T when the rotor
is in the aligned position as shown in Fig. 4.7A.
(c) The maximum permissible flux-density in the stator poles is 2.15 T, and in the aligned
position the current required for this flux-density is 3is. The unaligned inductance is 1/8
of the unsaturated aligned inductance. Estimate the maximum average electromagnetic
torque that this motor can produce at low speed.

Fig. 4.7
4.8 The switched reluctance motor in Fig. 4.7A is shown again in Figs. 4.7 B...E. Diagram B is of the
normal “unaligned” position, but diagrams C, D and E all show faults in the winding or its
connections. In each case, draw a flux-plot with 5 ! 10 lines of B, and comment on the value of
the inductance of phase 1 (compared with to case A) when the rotor is in each of the positions
shown. The winding conditions are:
A Normal aligned position; coils in series.
B Normal unaligned position: coils in series.
C Aligned position; left-hand coil open-circuited; normal current in coil 1.
D Aligned position; coils in series; left-hand coil connected with wrong polarity.
E Aligned position; series connection; left-hand coil short-circuited; normal current.
SPEED's Electric Motors

Solutions to Problems

4. Switched Reluctance Machines


S4.1
S4.2

12/8 motor
S4.3

10/4 motor
S4.4
(a)

(b)

(c)
S4.5

Neglecting losses, a flat-topped current waveform can be obtained if

As T increases, i decreases in inverse proportion

Let 2D be the dwell angle (i.e., conduction angle of transistors). The energy converted per stroke is

Since the electromagnetic torque is proportional to W, it is proportional to V2/T2.


S4.6

(a)

(b)

(c) Assume that the inductance changes from Lmax at maximum overlap to zero when there is no
overlap. Then

(d) 30E.
S4.7

(a) Lau = 2 Np2 Pg where Np is the number of turns on each pole-coil and P g is the permeance of one
airgap, :0Ap/g. The pole area is Ap = Pole width x Stack length = 13 x 50 = 650 mm2. So

(b)

(c)

First draw the aligned and unaligned magnetization curves. Then calculate the enclosed area to the left
of the line i = 3 x 8.70 = 26/1 A.
Then W = 89.36 x 26.1 ! ½ (27.27 x 26.1) ! ½ (72.73 x 8.70) ! (89.36 ! 72.73) x (8.70 + 26.1)/2
= 1371 mJ
Maximum available torque (averaged over one revolution) is
SPEED's Electric Motors

Problems
5. DC Machines

Answers

5.1 Yes.
5.2 (d) 0@354 T; (e) 225@4 A-t; (f) 23@67 V-s; (h) 9@47 V; (i) 143@2 :H; (j) 12,800 A-t; (k) 0@237
J.
5.4 (a) 1,722 rev/min; (b) 1,589 rev/min; (c) 16.0 A.
5.5 (a) 0@0306 Nm/A; (b) 3,121 rev/min; (c) 37@5 A; (d) 3@20 V/1000 rpm; (e) 53@4 mS

5.6 (a) 1,146 rev/min (b) 1,194 rev/min

Full solutions are given at the end


5.1 An electric motor contains coils and magnets and the flux is fixed in magnitude.
Can the flux-linkage of any coil vary? Y — yes N — no

5.2 A permanent-magnet DC motor has the cross-section shown in


Fig. 1 with an armature diameter D = 2r1 = 60 mm; magnet
length Lm = 8 mm; airgap length g = 0.8 mm, and stack length
Lstk = 35 mm. The magnet arc is $m = 120E. A single full-pitch
rotor coil is shown with 30 turns. The ceramic magnet has Br =
0.35 T, :rec = 1, and Hc= 278 kA/m. The current in the rotor coil
is zero.
(a) Sketch the magnetic field set up by the magnet, by
drawing 10 flux lines.
(b) Sketch the variation of the radial component of airgap
flux-density Bg(2) around the airgap from 0 to 360E.
Fig. 1
(c) Draw an equivalent magnetic circuit and use it to
calculate the flux crossing the airgap under each magnet pole. Assume that the leakage
flux can be represented by a permeance equal to 0.15 times the magnet internal permance,
and assume that the permeability of the steel in the rotor and stator is infinite.
(d) Calculate Bg in the airgap at the centre of the magnet arc, i.e. on the direct axis (d-axis).
(e) Determine the MMF across the magnet and the magnetic field strength Hm inside the
magnet.
(f) Calculate the flux-linkage R of the rotor coil in the position shown.
(g) Sketch the waveform of the coil flux-linkage R as the rotor rotates through 360E.
(h) Determine the waveform and the peak value of the EMF induced in the stator coil if the
rotor rotates at 4,000 rev/min.
(i) Estimate the inductance of the stator coil.
(j) If the magnet material requires a magnetizing force of 1600 kA/m to magnetize it fully,
estimate the ampere-turns required to magnetize one of the magnets.
(k) If the current in the coil is maintained constant at 5 A, determine the mechanical work
that is done by the rotor in rotating 180E from the position shown.

5.3 Draw the layout diagrams for the following windings in a 4-pole, 15-slot armature:
(a) Progressive lap winding with 1 coilside/layer and span = 3;
(b) Retrogressive lap winding with 1 coilside/layer and span = 3;
(c) Progressive wave winding with 1 coilside/layer and span = 3;
(d) Retrogressive wave winding with 1 coilside/layer and span = 3;
(e) Progressive wave winding with 3 coilsides/layer and span = 3.
5.4 A permanent-magnet DC motor operates from a supply of 240 V. Its armature resistance is 1.2 ohm
and the torque constant is k T = 1.31 Nm/A. Friction torque is constant at 1 Nm, and the brush
voltage-drop is 1.4 V per brush. Calculate
(a) the no-load speed;
(b) the speed for a steady load of 20 Nm; and
(c) the armature current for this load.

5.5 Show that if friction and core losses are neglected, the maximum efficiency of a permanent-
magnet DC commutator motor is equal to the ratio of the open-circuit voltage E to the supply
voltage Vs.
A DC commutator motor is to be designed to deliver 300 W at 2,500 rev/min when supplied at 12V.
The efficiency must not be less than 2/3 (i.e., 66.67 %) when measured on a dynamometer that
eliminates friction torque. The brush material is such that the voltage drop across each brush
will be 1@0 V, regardless of the current. Calculate
(a) the torque constant k T in Nm/A
(b) the no-load speed
(c) the current
(d) the EMF constant k E expressed in "Volts per 1000 rpm"
(e) the maximum permissible armature winding resistance Ra.

5.6 A 10-hp 230-V DC shunt-wound motor has the equivalent circuit shown in Fig. 2.

Fig. 2

The armature resistance is Ra = 0.3 ohm and the field resistance is Rf = 170 ohm. At no-load and
rated voltage, the speed is 1200 rev/min and the armature current is Ia = 2.7 A. At full load and
rated voltage, the line current is IL = Ia + If = 38.4 A, where If is the field current. Calculate the
speed at full load,
(a) assuming that the flux is constant
(b) assuming that the flux at full-load is 4% less than the no-load value.
SPEED's Electric Motors

Solutions to Problems

5. DC Machines
S5.2
(a)

(b)

(c) M r = B rA m
Pm = :rec:0 Am/Rm
PL = > P m
Rg = Rg/:0Ag
Bg = Mg/Ag
5.2 (c) cont’d/...
Am = 2ð /3 x (60/2 + 0@8 + 8/2) x 35 = 2,551 mm2
Ag = 2ð /3 x (60/2 + 0@8/2) x 35 = 2,228 mm2

Pm = 1 x 4ð x 10!7 x 2,551 x 10!6 / 8 x 10!3 = 4@007 x 10!7 Wb/A


Rg = 0@8 x 10!3 / (4ð x 10!7 x 2,228 x 10!6) = 2@857 x 105 A/Wb

Mg = Mr / (1 + (1 + >)PmRg) where > is the leakage fraction (0@15)


= 0@35 x 2,551 x 10!6 /(1 + 1@15 x 4@007 x 10!7 x 2@857 x 105)
= 0@789 mWb

(d) Bg = Mg /Ag = 0@789 x 10!3 / 2,228 x 10!6 = 0@354 T

(e) Fm = Fg = Mg Rg = 0@789 x 10!3 x 2@857 x 105 = 225@4 A-t


*Hm* = Fm/Rm = 225@4 / 8 = 28@2 A/mm, i.e. 28@2 kA/m

(f) Flux-linkage R = N Mg = 30 x 0@789 = 23@67 V-s, where N = 30 turns and it is assumed


that all the flux Mg links the coil. This is the peak flux-linkage, Rpk.

(g)

(h) Peak EMF is epk = d R/dt = MR/M2 x d2/dt = T x MR/M2 where T = 4,000 x 2B/60 = 418@9
rad/s
MR/M2 = 23@67 x 10!3/ (60E x B/180) = 22@60 mVs/rad
ˆ epk = 418@9 x 22@60 x 10!3 = 9@47 V
5.2 cont’d/...

(i) L = 8/i where 8 is the flux-linkage produced by i A


8 = N M = N B Ag = N :0 H Ag = N :0 x Ni/2(Rg + Rm) x Ag,
so L = :0N2Ag/2(Rg + Rm) = 4B x 10!7 x 302 x 2,228 x 10!6/2(8 + 0@8) x 10!3 = 143@2 :H

(j) To get Hm = 1600 kA/m we need Fm = HmRm = 1600 x 103 x 8 x 10!3 = 12,800 At

(k) Work done = 5@0 x (2 x 23@6


S5.3
The windings are shown on the following three pages. Note the last one (which is not part of the
question, but may be of interest).

5.3(a) Progressive lap winding in 4-pole, 15-slot armature with 1 coilside/layer; span = 3

5.3(b) Retrogressive lap winding in 4-pole, 15-slot armature with 1 coilside/layer; span = 3
5.3(c) Progressive wave winding in 4-pole, 15-slot armature with 1 coilside/layer; span = 3

5.3(d) Retrogressive wave winding in 4-pole, 15-slot armature with 1 coilside/layer; span = 3
5.3(e) Progressive wave winding in 4-pole, 15-slot armature with 3 coilsides/layer; span = 3

A wave winding is not possible if the number of slots is even.


S5.4
Vs = 240 V Vb = 1.4 V per brush kE = kT = 1@31 V-s/rad or Nm/A

(a) T0 = (Vs ! 2 Vb)/kE ! Ra/kE2 x Tf = (240 ! 2 x 1@4)/1@31 ! 1@2/1@312 x 1@0


= 180@4 rad/s = 1,722 rev/min
(b) T = (Vs ! 2 Vb)/kE ! Ra/kE2 x (T + Tf) = (240 ! 2 x 1@4)/1@31 ! 1@2/1@312 x (20 + 1@0)
= 166@4 rad/s = 1,589 rev/min
(c) Ia = Te/kT = (T + Tf) /kT = (20 + 1@0)/1@31 = 16@0 A

Check : RaIa = 1@2 x 16@0 = 19@2 V


Ea = Vs ! RaIa ! 2Vb = 240 ! 19@2 ! 2@8 = 218 V
T = Ea/kE = 218/1@31 = 166@4 rad/s .....OK
S5.5
Input power = VsIa
Output power = EaIa = TeT if friction and rotor core loss are neglected
Efficiency = Output/Input = Ea/Vs

(a) At 2,500 rev/min and 300 W we need Effcy $ 0@667, so Ea $ 0@667 x 12 = 8@0 V
kE = Ea/T = 8@0/(2,500 x 2B/60) = 0@0306 V-s/rad = kT
(b) No-load speed T0 = (12 ! 2 x 1@0)/0@0306 = 326@8 rad/s = 3,121 rev/min
(c) Tshaft = 300 W / (2,500 x 2B/60) = 1@146 Nm
Ia = 1@146/0@0306 = 37@45 A

(d) kE = 0@0306 V-s/rad


...= 0@0306 x 2 B/60
...= 0@0032 V/(rev/min)
...= 3@204 V/(1000 rev/min)

(e) The volt-drop RaIa must be limited to 2 V if Ea is to be 8@0 V (with 1@0 V across each brush).
ˆ the winding must be designed so that Ra is no greater than 2/37@45 = 53@4 mohm.
S5.6
At no-load, Ea = Vs ! RaIa = 230 ! 0.3 x 2@7 = 229@2 V.
ˆ kE = kT = Ea/T0 = 229@2/(1,200 x 2B/60) = 1@824 V-s/rad
The field current is If = 230/170 = 1@35 A

(a) At full load, Ia = 38@4 ! 1@35 = 37@05 A


ˆ Ea = 230 ! 37@05 x 0@3 = 218@9 V
and T = Ea/kE = 218@9/1@824 = 120 rad/s = 1,146 rev/min

(c) Ia and If are unchanged but the flux is reduced by 4% and therefore kE and kT are reduced by 4%
also; kE = kT = 0@96 x 1@824 = 1@751 V-s/rad. So T is 4% higher at 218@9/1@751 = 125@0 rad/s
= 1,194 rev/min.

S-ar putea să vă placă și